cost estimation cvp analysis and decision making luke corporation produces a variety 691001

Cost Estimation, CVP Analysis, and Decision Making

Luke Corporation produces a variety of products, each within their own division. Last year, the managers at Luke developed and began marketing a new chewing gum, Bubbs, to sell in vending machines. The product, which sells for $5.25 per case, has not had the market success that managers expected and the company is considering dropping Bubbs. The product line income statement for the past twelve months follows:

Revenue

$14,682,150

Costs

   

Manufacturing costs

$14,440,395

Allocated corporate costs (@5%)

734,108

15,174,503

Product line margin

$ (492,353)

Allowance for tax (@20%)

98,470

Product line profit t

$ (393,883)

All products at Luke receive an allocation of corporate overhead costs, which is computed as 5 percent of product revenue. The 5 percent rate is computed based on the most recent year’s corporate cost as a percentage of revenue. Data on corporate costs and revenues for the past two years follow:

 

Corporate Revenue

Corporate Overhead Costs

Most recent year

$106,750,000

$5,337,500

Previous year

$ 76,200,000

4,221,000

Roy O. Andre, the product manager for Bubbs, is concerned about whether the product will be dropped by the company and has employed you as a financial consultant to help with some analysis.  In addition to the information given above, Mr. Andre provides you with the following data on product costs for Bubbs:

Month

Cases

Production Costs

1

207,000

$1,139,828

2

217,200

1,161,328

3

214,800

1,169,981

4

228,000

1,185,523

5

224,400

1,187,827

6

237,000

1,208,673

7

220,200

1,183,699

8

247,200

1,226,774

9

238,800

1,225,226

10

252,600

1,237,325

11

250,200

1,241,760

12

259,200

1,272,451

Required

a. Bunk Stores has requested a quote for a special order of Bubbs. This order would not be subject to any corporate allocation (and would not affect corporate costs). What is the minimum price Mr. Andre can offer Bunk without reducing profit t any further?

b. How many cases of Bubbs does Luke have to sell in order to break even on the product?

c. Suppose Luke has a requirement that all products have to earn 5 percent of sales (after tax and corporate allocations) or they will be dropped. How many cases of Bubbs does Mr. Andre need to sell to avoid seeing Bubbs dropped?

d. Assume all costs and prices will be the same in the next year. If Luke drops Bubbs, how much will Luke’s profits increase or decrease? Assume that fixed production costs can be avoided if Bubbs is dropped.

pear computers assembles computer parts into finished systems for sale through mass 691004

Pear Computers assembles computer parts into finished systems for sale through mass market retailers. It sells two models differentiated by the speed of the processor and other peripherals, the X3G and the X2G. Data on the two models for April follow. The time required for either unit is the same. Pear Computers uses operations costing and assigns direct labor and overhead based on the number of units assembled. Compute the cost of the X3G and X2G models for April.

 

X3G

X2G

Total

Number of units . .

6,000

14,000

20,000

Parts cost

 

 

 

per unit

$ 250

$ 175

 

Parts cost

$1,500,000

$2,450,000

$3,950,000

Other costs:

 

 

 

Direct labor

 

 

$ 414,000

Overhead

 

 

806,000

Total

 

 

$1,220,000

basic cost flow model assume that the following events occurred at a division of gen 691007

Basic Cost Flow Model

Assume that the following events occurred at a division of Generic Electric for March of the current year.

1. Purchased $45 million in direct materials.

2. Incurred direct labor costs of $24 million.

3. Determined that manufacturing overhead was $40.5 million.

4. Transferred 80 percent of the materials purchased to work in process.

5. Completed work on 72 percent of the work in process. Costs are assigned equally across all work in process.

6. The inventory accounts have no beginning balances. All costs incurred were debited to the appropriate account and credited to Accounts Payable.

Required

Give the amounts for the following items in the Work in Process account:

a. Transfers in ( TI ).

b. Transfers out ( TO ).

c. Ending balance ( EB ).

basic product costing enviro corporation manufactures a special liquid cleaner at it 691011

Basic Product Costing

Enviro Corporation manufactures a special liquid cleaner at its Green plant. Operating data for June follow:

Materials

$714,000

Labor

61,200

Manufacturing overhead

244,800

The Green plant produced 850,000 gallons in June. The plant never has any beginning or ending inventories.

Required

Compute the cost per gallon of liquid cleaner produced in June.

basic product costing kim amp smith refiners starts business on march 1 the followin 691013

Basic Product Costing

Kim & Smith Refiners starts business on March 1. The following operations data are available for March for the one lubricant it produces:

 

Gallons

Beginning inventory

–0–

Started in March

900,000

Ending work in process inventory (80% complete)

100,000

Costs incurred in March follow:

Materials

$188,000

Labor

48,400

Manufacturing overhead

98,000

     

All production at Kim & Smith is sold as it is produced (there are no finished goods inventories).

Required

a. Compute cost of goods sold for March.

b. What is the value of work in process inventory on March 31?

basic product costing ethical issues old tyme soda produces one flavor of a popular 691014

Basic Product Costing: Ethical Issues

Old Tyme Soda produces one flavor of a popular local soft drink. It had no work in process on October 31 in its only inventory account. During November, Old Tyme started 10,000 barrels. Work in process on November 30 is 1,200 barrels. The production supervisor estimates that the ending work in process inventory is 30 percent complete. An examination of Old Tyme’s accounting records shows direct material costs of $18,072 and conversion costs of $20,400 for November. All production is sold as it is produced.

Required

a. Compute cost of goods sold for November.

b. What is the value of work in process inventory on November 30?

c. The president tells the controller that stock analysts expect higher income for the month and asks the controller to change the production manager’s estimate about the ending work inprocess inventory.

(1) If the controller wanted to comply with the president’s request would he raise or lower the estimated percentage complete from the 30 percent estimate of the production supervisor? Explain.

(2) What should the controller do?

process costing van goe produces paints on january 1 it had no work in process inven 691015

Process Costing

Van Goe produces paints. On January 1, it had no work in process inventory. It starts production of 300,000 gallons of paint in January and completes 240,000 gallons. The costs of the resources used by Van Goe in January consist of the following:

Materials

$328,800

Conversion costs (labor and overhead)

420,000

Required

The production supervisor estimates that the ending work in process is 80 percent complete. Compute the cost of paint transferred to finished goods and the amount in work in process ending inventory as of January 31.

process costing opech inc produces oil and ships it in a pipeline on may 1 it had no 691016

Process Costing

Opech, Inc., produces oil and ships it in a pipeline. On May 1, it had no work in process inventory. It started production of 300 million barrels of oil in May and shipped 270 million barrels in the pipeline. The costs of the resources used by Opech in May consist of the following:

Materials

$5,000 million

Conversion costs (labor and overhead)

$6,640 million

Required

The production supervisor estimates that the ending work in process is 70 percent complete on May 31. Compute the cost of oil shipped in the pipeline and the amount in work in process ending inventory as of May 31.

process costing oholics ltd produces chocolate that it sells to candy makers on apri 691017

Process Costing

Oholics, Ltd., produces chocolate that it sells to candy makers. On April 1, it had no work in process inventory. It started production of 20,000 pounds of chocolate in April and completed production of 19,000 pounds. The costs of the resources used by Oholics in April consist of the following:

Materials

$33,000

Conversion costs (labor and overhead)

40,500

Required

The production supervisor estimates that the ending work in process is 60 percent complete on April 30. Compute the cost of chocolate completed and the cost of the chocolate in work in process ending inventory as of April 30.

predetermined overhead rates tiger furnishings produces two models of cabinets for h 691018

Predetermined Overhead Rates

Tiger Furnishings produces two models of cabinets for home theater components, the Basic and the Dominator. Data on operations and costs for March follow:

 

Basic

Dominator

Total

Units produced

1,000

250

1,250

Machine hours

4,000

2,000

6,000

Direct labor hours

3,000

2,000

5,000

Direct materials costs

$10,000

$ 3,750

$ 13,750

Direct labor costs

64,500

35,500

100,000

Manufacturing overhead costs

 

 

174,100

Total costs

 

 

$287,850

Required

Compute the predetermined overhead rate assuming that Tiger Furnishings uses direct labor hours to allocate overhead costs.

predetermined overhead rates tiger furnishings produces two models of cabinets for h 691019

Predetermined Overhead Rates

Tiger Furnishings produces two models of cabinets for home theater components, the Basic and the Dominator. Data on operations and costs for March follow:

 

Basic

Dominator

Total

Units produced

1,000

250

1,250

Machine hours

4,000

2,000

6,000

Direct labor hours

3,000

2,000

5,000

Direct materials costs

$10,000

$ 3,750

$ 13,750

Direct labor costs

64,500

35,500

100,000

Manufacturing overhead costs

 

 

174,100

Total costs

 

 

$287,850

Required

Compute the predetermined overhead rate assuming that Tiger Furnishings uses direct labor costs to allocate overhead costs.

tiger furnishings produces two models of cabinets for home theater components the ba 691020

Tiger Furnishings produces two models of cabinets for home theater components, the Basic and the Dominator. Data on operations and costs for March follow:

 

Basic

Dominator

Total

Units produced

1,000

250

1,250

Machine hours

4,000

2,000

6,000

Direct labor hours

3,000

2,000

5,000

Direct materials costs

$10,000

$ 3,750

$ 13,750

Direct labor costs

64,500

35,500

100,000

Manufacturing overhead costs

 

 

174,100

Total costs

 

 

$287,850

Required

Compute the predetermined overhead rate assuming that Tiger Furnishings uses machine hours to allocate overhead costs.

learning curves appendix b refer to the example in appendix b the numbers in exhibit 690988

Learning Curves (Appendix B)

Refer to the example in Appendix B. The numbers in Exhibit 5.21 for the fifth, sixth, and seventh units were given.

 

 

Labor Time

 

 

 

 

Required to

 

 

 

 

Produce the X th

 

 

 

 

Unit (i.e., the last

Cumulative

 

 

Unit

single unit

Total Time

 

 

Produced

produced)1

in Labor

Total

Average Cost

(X )

(Y )

Hours2

Cost3

per Unit4

1

100

100

$ 5,000.00

$5,000.00

2

80

180

9,000.00

4,500.00

3

70.21

250.21

12,510.50

4,170.17

4

64

314.21

15,710.50

3,927.63

5

59.56

373.77

18,688.50

3,737.70

6

56.17

429.94

21,497.00

3,582.83

7

53.45

483.39

24,169.50

3,452.78

8

51.2

534.59

26,729.50

3,341.19

Exhibit 5.21

Learning Curve Time and Costs

1 Going down the column, the labor time for each unit comes from the formula, Y = aXb. For example, the labor time to produce the third unit is found as follows: Y _ 100 hours to produce the first unit times 3, because this is the third unit produced, to the exponent 0.3219, which is the learning rate coefficient for an 80% learning rate.

So,

Y = 100 × 3 0.3219 = 70.21 hours

2 This is the sum of the hours worked on the units. For example, three units requires

100.00 + 80.00 + 70.21 = 250.21 hours

3 This is the total cost of the labor time worked, which is the cumulative total time in labor hours times the $50 per hour labor cost given in the text on page 169. For example, the total cost of producing three units = 250.21 hours × $50 = $12,510.50.

4 This is the average cost per unit, which is the total cost of the units produced divided by the number of units produced. For example, the average cost per unit of producing three units = $12,510.50 ÷ 3 = $4,170.17.

Required

Using the formula Y = aXb and the data given in the problem, verify the labor time required and the cost amounts for the fi fth, sixth, and seventh units. (“Verify” means that you should check the accuracy of the amounts given in Exhibit 5.21.)

account analysis the accounting department of a large limousine company is analyzing 690989

Account Analysis

The accounting department of a large limousine company is analyzing the costs of its services. The cost data and level of activity for the past 16 months follow:

 

 

Customer

Paychecks

Accounting

Month

Special Analyses

Accounts

Processed

Service Costs

1

2

325

1,029

$ 63,800

2

4

310

993

68,900

3

2

302

1,268

64,000

4

1

213

1,028

61,300

5

2

222

984

61,600

6

0

214

712

50,800

7

1

131

762

51,020

8

1

123

739

54,300

9

0

115

708

50,500

10

2

296

1,232

64,800

11

2

213

978

58,000

12

1

222

929

57,500

13

2

217

1,059

62,200

14

2

132

942

54,900

15

4

300

1,299

71,530

16

4

315

1,283

64,800

Totals

30

3,650

15,945

$959,950

In addition to the above information, you learn that the accounting department had the following total costs for the past 16 months for each of the following:

Total cost of paychecks processed

$180,100

Total cost of maintaining customer accounts

109,600

Total cost of performing special analyses

120,000

Total fixed costs (total for 16 months)

550,250

Total costs

$959,950

Required

a. What is the cost per unit for (1) paychecks processed, (2) customer accounts maintained, and (3) special analyses performed?

b. Assuming the following level of cost driver volumes for a month, what are the accounting department’s estimated costs of doing business using the account analysis approach?

• 1,000 paychecks processed

• 200 customer accounts maintained

• 3 special analyses

regressions from published data obtain 13 years of data from the published financial 690990

Regressions from Published Data

Obtain 13 years of data from the published financial statements of a company. You will be able to find the data on the Internet. Also, Moody’s, Standard & Poor’s, and Value Line are good sources of financial data. Using the first 12 years of data, perform a regression analysis in which the dependent variable is cost of goods sold and the independent variable is revenue (some companies call it sales ).

Required

a. Use the results from the regression on the first 12 years of data to estimate the cost of goods sold for year 13. How far off was your estimate of cost of goods sold for year 13?

b. Prepare a report that describes your work and discusses reasons why your estimate of cost of goods sold is different than the actual cost of goods sold for year 13.

make versus buy the liquid chemical company manufactures and sells a range of high g 690967

Make versus Buy

The Liquid Chemical Company manufactures and sells a range of high grade products. Many of these products require careful packaging. The company has a special patented lining made that it uses in specially designed packing containers. The lining uses a special material known as GHL. The firm operates a department that maintains and repairs its packing containers to keep them in good condition and that builds new ones to replace units that are damaged beyond repair.

Mr. Walsh, the general manager, has for some time suspected that the firm might save money  and get equally good service by buying its containers from an outside source. After careful inquiries, he has approached a firm specializing in container production, Packages, Inc., and asked for a quotation. At the same time, he asked Mr. Dyer, his chief accountant, to let him have an up to date statement of the costs of operating the container department. Within a few days, the quotation from Packages, Inc., arrived. The firm proposed to supply all the new containers required—at that time, running at the rate of 3,000 per year—for $1,250,000 a year, the contract to run for a guaranteed term of five years and thereafter renewable from year to year. If the number of containers required increased, the contract price would increase proportionally. Packages, Inc., also proposed to perform all maintenance and repair work on existing packaging containers for a sum of $375,000 a year, on the same contract terms.

Mr. Walsh compared these figures with Mr. Dyer’s cost figures, which covered a year’s operations of the container department of the Liquid Chemical Company and appear in Exhibit 4.13.

Walsh concluded that he should immediately close the packing container department and sign the contracts offered by Packages, Inc. He felt an obligation, however, to give the manager of the department, Mr. Duffy, an opportunity to question his decision before acting. Walsh told Duffy that Duffy’s own position was not in jeopardy. Even if Walsh closed his department, another managerial position was becoming vacant to which Duffy could move without any loss of pay or prospects. The manager Duffy would replace also earned $80,000 per year. Moreover, Walsh knew that he was paying $85,000 per year in rent for a warehouse a couple of miles away that was used for other corporate purposes. If he closed Duffy’s department, he’d have all the warehouse space he needed without renting additional space.

Duffy gave Walsh a number of considerations to think about before he closed the department: “For instance,” he said, “what will you do with the machinery? It cost $1,200,000 four years ago, but you’d be lucky if you’d get $200,000 for it now, even though it’s good for another five years. And then there’s the stock of GHL (a special chemical) we bought a year ago. That cost us $1,000,000, and at the rate we’re using it now, it’ll last another four years. We used up only about one fifth of it last year. Dyer’s figure of $700,000 for materials includes $200,000 for GHL. But it’ll be tricky stuff to handle if we don’t use it up. We bought it for $5,000 a ton, and you couldn’t buy it today for less than $6,000. But you’d get only $4,000 a ton if you sold it, after you’d covered all the handling expenses.”

Materials

 

$ 700,000

Labor

 

 

Supervisor

 

50,000

Workers

 

450,000

Department overheads

 

 

Manager’s salary

$ 80,000

 

Rent on Container Department

45,000

 

Depreciation on machinery

150,000

 

Maintenance of machinery

36,000

 

Other expenses

157,500

 

 

 

468,500

 

 

$1,668,500

Proportion of general administrative overheads

 

225,000

Total cost of department for the year

 

$1,893,500

Walsh also worried about the workers if he closed the department. “I don’t think we can find room for any of them elsewhere in the firm. However, I believe Packages would take all but Hines and Walters. Hines and Walters have been with us since they left school 40 years ago. I’d feel bound to give them a supplemental pension—$15,000 a year each for five years, say. Also, I’d figure a total severance pay of $20,000 for the other employees, paid in a lump sum at the time we sign the contract with Packages.”

Duffy showed some relief at this. “But I still don’t like Dyer’s figures,” he said. “What about this $225,000 for general administrative overheads? You surely don’t expect to sack anyone in the general office if I’m closed, do you?” Walsh agreed. “Well, I think we’ve thrashed this out pretty well,” said Walsh, “but I’ve been turning over in my mind the possibility of perhaps keeping on the maintenance work ourselves. What are your views on that, Duffy?”

“I don’t know,” said Duffy, “but it’s worth looking into. We wouldn’t need any machinery for that, and I could hand the supervision over to the current supervisor who earns $50,000 per year. You’d need only about one fifth of the workers, but you could keep on the oldest and save the pension costs.

You’d still have the $20,000 severance pay, I suppose. You wouldn’t save any space, so I suppose the rent would be the same. I don’t think the other expenses would be more than $65,000 a year.”

“What about materials?” asked Walsh.

“We use 10 percent of the total on maintenance,” Duffy replied.

“Well, I’ve told Packages that I’d give them my decision within a week,” said Walsh. “I’ll let you know what I decide to do before I write to them.”

Assume the company has a cost of capital of 10 percent per year and uses an income tax rate of 40 percent for decisions such as these. Liquid Chemical would pay taxes on any gain or loss on the sale of machinery or the GHL at 40 percent. (Depreciation for book and tax purposes is straight line over eight years.) The tax basis of the machinery is $600,000. Also assume the company had a five year time horizon for this project and that any GHL needed for Year 5 would be purchased during Year 5.

Required

a. What are the four alternatives available to Liquid Chemical?

b. What action should Walsh take? Support your conclusion with a net present value analysis of all the mutually exclusive alternatives. Be sure to consider factors not explicitly discussed in the case that you think should have a bearing on Walsh’s decision.

c. What, if any, additional information do you think Walsh needs to make a sound decision? Why?

brown rsquo s baskets makes decorative baskets for sale at local craft shops mary br 690968

Brown’s Baskets makes decorative baskets for sale at local craft shops. Mary Brown, the owner and founder, has collected the following information on costs based on two years of operations and has asked you to help her analyze the behavior of her overhead costs. Mary summarized monthly data as two year totals:

Indirect materials

$ 27,200

Indirect labor

44,300

Lease

56,000

Utilities (heat, light, etc.)

19,200

Power to run machines.

18,500

Insurance

16,400

Maintenance

14,500

Depreciation

9,000

Total overhead

$205,100

Direct labor hours

12,000

Direct labor costs

$180,000

Machine hours

14,400

Units produced

20,000

After visiting the workshop and discussing operations with Mary, you determine that three costs—indirect materials, indirect labor, and the power to run the machines—are variable. All other costs are fixed.

Prepare three analyses of overhead costs that, using the account analysis method, calculate the monthly average fixed costs and the variable rate per (1) direct labor hour,  (2) machine hour, and (3) unit of output.

the following computer output presents the results of two simple regressions for the 690969

The following computer output presents the results of two simple regressions for the Brown’s Baskets overhead costs using (1) direct labor hours and (2) units of output (baskets) as the independent variables. Each regression has 24 data points, one data point per month for two years. Which activity base, units of output or labor hours, do you believe best explains variation in overhead costs?

 

A

B

C

D

E

F

G

1

SUMMARY OUTPUT

 

 

 

 

 

 

2

 

 

 

 

 

 

 

3

Regression Statistics

 

 

 

 

 

 

4

Multiple R

0.73161563

 

 

 

 

 

5

R Square

0.53526143

 

 

 

 

 

6

Adjusted R Square

0.51413695

 

 

 

 

 

7

Standard Error

942.922704

 

 

 

 

 

8

Observations

24

 

 

 

 

 

9

 

 

 

 

 

 

 

10

ANOVA

 

 

 

 

 

 

11

 

df

SS

MS

F

Significance F

 

12

Regression

1

22528490.87

22528490.9

25.33844

4.85539E 05

 

13

Residual

22

19560270.97

889103.225

 

 

 

14

Total

23

42088761.84

 

 

 

 

15

 

 

 

 

 

 

 

16

 

Coefficients

Standard Error

t Stat

P value

Lower 95%

Upper 95%

17

Intercept

4705.95547

826.1235525

5.696430583

9.94E 06

2992.678242

6419.23269

18

Labor Hours

8.08808907

1.606778337

5.033730468

4.86E 05

4.75583117

11.420347

19

 

 

 

 

 

 

 

 

 

 

 

 

 

 

 

Continue

 

A

B

C

D

E

F

G

1

SUMMARY OUTPUT

 

 

 

 

 

 

2

 

 

 

 

 

 

 

3

Regression St

atistics

 

 

 

 

 

4

Multiple R

0.881023603

 

 

 

 

 

5

R Square

0.77620259

 

 

 

 

 

6

Adjusted R Square

0.76602998

 

 

 

 

 

7

Standard Error

590.8985221

 

 

 

 

 

8

Observations

24

 

 

 

 

 

9

 

 

 

 

 

 

 

10

ANOVA

 

 

 

 

 

 

11

 

df

SS

MS

F

Significance F

 

12

Regression

1

26642104

26642104

76.30319

1.33090E 08

 

13

Residual

22

7681543.39

349161.063

 

 

 

14

Total

23

34323647.39

 

 

 

 

15

 

 

 

 

 

 

 

16

 

Coefficients

Standard Error

t Stat

P value

Lower 95%

Upper 95%

17

Intercept

4338.878836

519.189015

8.357031275

2.84E 08

3262.14556

5415.61211

18

Baskets

5.293345397

0.60598082

8.735169864

1.33E 08

4.03661675

6.55007405

19

 

 

 

 

 

 

 

 

 

 

 

 

 

 

 

[[Discussion Questions]]

methods of estimating costs account analysis the accounting records for miller fixtu 690974

Methods of Estimating Costs: Account Analysis

The accounting records for Miller Fixtures report the following production costs for the past year:

Direct materials

$210,000

Direct labor

175,000

Variable overhead

154,000

Production was 210,000 units. Fixed manufacturing overhead was $240,000.

For the coming year, costs are expected to increase as follows: direct materials costs by 20 percent, excluding any effect of volume changes; direct labor by 4 percent; and fixed manufacturing overhead by 10 percent. Variable manufacturing overhead per unit is expected to remain the same.

Required

a. Prepare a cost estimate for a volume level of 220,000 units of product this year.

b. Determine the costs per unit for last year and for this year.

methods of estimating costs account analysis a consulting firm rsquo s accounting re 690975

Methods of Estimating Costs: Account Analysis

A consulting firm’s accounting records show the following costs for year 1:

Direct materials supplies

$ 360,000

Direct labor

2,580,000

Total overhead

1,140,000

Production was 150,000 billable hours. Fixed overhead was $600,000. For year 2, direct materials costs are expected to increase by 10 percent per unit. Direct labor costs are expected to increase by 15 percent. Variable overhead per billable hour is expected to remain the same, but fixed overhead is expected to increase by 5 percent.

Required

a. Year 2 production is expected to be 195,000 billable hours. What are the estimated direct materials, direct labor, variable overhead, and fixed overhead costs for year 2?

b. Determine the total costs per billable hour for year 1 and year 2.

methods of estimating costs high low ethical issues oak island amusements center pro 690976

Methods of Estimating Costs: High Low, Ethical Issues

Oak Island Amusements Center provides the following data on the costs of maintenance and the number of visitors for the last three years:

Number of Visitors

 

per Year:

Maintenance Costs

(thousands)

($000)

1,825

$2,925

2,007

3,225

2,375

3,750

Required

a. Use the high low method to estimate the fixed cost of maintenance annually and the variable cost of maintenance per visitor.

b. The company expects a record 2,600,000 visitors next year. What would be the estimated maintenance costs?

c. Company management is considering eliminating the maintenance department and contracting with an outside firm. Management is especially concerned with the fixed costs of maintenance. The maintenance manager tells you, the cost analyst, that 2,375 visitors is an outlier and should not be used in the analysis. Assume that this will lower estimated fixed costs. Is it ethical to treat this observation as an outlier?

methods of estimating costs high low adriana corporation manufactures football equip 690977

Methods of Estimating Costs: High Low

Adriana Corporation manufactures football equipment. In planning for next year, the managers want to understand the relation between activity and overhead costs. Discussions with the plant supervisor suggest that overhead seems to vary with labor hours, machine hours, or both. The following data were collected from last year’s operations:

Month

Labor Hours

Machine Hours

Overhead Costs

1

3,625

6,775

$513,435

2

3,575

7,035

518,960

3

3,400

7,600

549,575

4

3,700

7,265

541,400

5

3,900

7,955

581,145

6

3,775

7,895

572,320

7

3,700

6,950

535,110

8

3,625

6,530

510,470

9

3,550

7,270

532,195

10

3,975

7,725

565,335

11

3,375

6,490

503,775

12

3,550

8,020

564,210

Required

a. Use the high low method to estimate the fixed and variable portions of overhead costs based on machine hours.

b. Managers expect the plant to operate at a monthly average of 7,500 machine hours next year. What are the estimated monthly overhead costs, assuming no inflation?

methods of estimating costs high low adriana corporation manufactures football equip 690978

Methods of Estimating Costs: High Low

Adriana Corporation manufactures football equipment. In planning for next year, the managers want to understand the relation between activity and overhead costs. Discussions with the plant supervisor suggest that overhead seems to vary with labor hours, machine hours, or both. The following data were collected from last year’s operations:

Month

Labor Hours

Machine Hours

Overhead Costs

1

3,625

6,775

$513,435

2

3,575

7,035

518,960

3

3,400

7,600

549,575

4

3,700

7,265

541,400

5

3,900

7,955

581,145

6

3,775

7,895

572,320

7

3,700

6,950

535,110

8

3,625

6,530

510,470

9

3,550

7,270

532,195

10

3,975

7,725

565,335

11

3,375

6,490

503,775

12

3,550

8,020

564,210

Required

Prepare a scattergraph based on the overhead and machine hour data

methods of estimating costs simple regression simple reression results from the data 690979

Methods of Estimating Costs: Simple Regression

Simple reression results from the data of Adriana Corporation  are as follows:

Equation: Overhead =$206,469 + ($45.83 × Machine hours) Statistical data

 

Correlation coefficient

.962

R 2 .

.925

Required

Estimate overhead if the company expects the plant to operate at a monthly average of 9,000 machine hours next year.

methods of estimating costs simple regression simple regression results from the dat 690980

Methods of Estimating Costs: Simple Regression

Simple regression results from the data of Adriana Corporation are as follows:

Equation: Overhead = $217,610 + ($88.61 × Labor hours) Statistical data

 

Correlation coefficient

.610

R 2

.372

Required

Estimate overhead if the company expects the plant to operate at a monthly average of 3,000 laborhours next year.

methods of estimating costs multiple regression multiple regression results from the 690981

Methods of Estimating Costs: Multiple Regression

Multiple regression results from the data of Adriana Corporation are as follows:

Equation: Overhead = $124,570 + ($31.93 × Labor hours) + ($41.10 × Machine hours) Statistical data

 

Correlation coefficient

.982

R 2

.964

Required

Estimate overhead using the multiple regression results, assuming that the company expects the plant to operate at a monthly average of 9,000 machine hours and 3,000 labor hours next year.

interpretation of regression results multiple choice cortez company is planning to i 690982

Interpretation of Regression Results: Multiple Choice

Cortez Company is planning to introduce a new product that will sell for $96 a unit. The following manufacturing cost estimates have been made on 20,000 units to be produced the first year:

Direct materials

$800,000

Direct labor

640,000 (=$16 per hour × 40,000 hours)

Manufacturing overhead costs have not yet been estimated for the new product, but monthly data on total production and overhead costs for the past 24 months have been analyzed using simple linear regression. The following results were derived from the simple regression and provide the basis for overhead cost estimates for the new product.

Simple Regression Analysis Results

Dependent variable—Factory overhead costs

Independent variable—Direct labor hours

Computed values

Intercept

$120,000

Coefficient on independent variable

$ 5.00

Coefficient of correlation

0.921

R2

0.848

Required

a. What percentage of the variation in overhead costs is explained by the independent

variable?

(1) 84.8%.

(2) 45.0%.

(3) 92.1%.

(4) 8.48%.

(5) Some other amount.b. What is the total overhead cost for an estimated activity level of 50,000 direct labor hours?

(1) $120,000.

(2) $370,000.

(3) $250,000.

(4) $320,000.

(5) Some other amount.c. How much is the variable manufacturing cost per unit, using the variable overhead estimated by the regression (assuming that direct materials and direct labor are variable costs)?

(1) $88.00.

(2) $82.00.

(3) $86.80.

(4) $72.00.

(5) 161Some other amount.

d. What is the expected contribution margin per unit to be earned during the first year on 20,000 units of the new product? (Assume that all marketing and administrative costs are fixed.)

(1) $96.

(2) $24.

(3) $56.

(4) $14.

(5) Some other amount.e. What is the manufacturing cost equation implied by these results?

(1) Total cost = $640,000 + $5.00 × Number of units.

(2) Total cost = $120,000 + $86.80 × Number of units.

(3) Total cost = $120,000 + $72.00 ×Number of units.

(4) Some other equation.

interpretation of regression results brodie company rsquo s advertising manager want 690983

Interpretation of Regression Results

Brodie Company’s advertising manager wants to know whether the company’s advertising program is successful. The manager used a spreadsheet program to estimate the relation between advertising expenditures (the independent variable) and sales dollars. Monthly data for the past two years were entered into the program. The regression results indicated the following relation:

Sales dollars = $169,000 ($200 × Advertising expenditures)

Correlation coefficient = .864

These results seemed to imply that advertising was reducing sales. The manager was about to conclude that statistical methods were so much nonsense when you walked into the room.

Required

Help the manager. What might cause the negative relationship between advertising expenditures and sales?

interpretation of regression results ross enterprises maintains a fleet of agricultu 690984

Interpretation of Regression Results

Ross Enterprises maintains a fleet of agricultural equipment for rental to local farmers. Ross maintains all its equipment in a company owned facility. Data on maintenance costs and operating hours of the equipment have been collected for the past 24 months to help managers plan financial needs. Managers at Ross were initially excited about having the data and the analysis available for planning, but the initial regression results revealed the following equation:

Maintenance costs = $10,564 $67.13 × Operating hours

The coefficient on operating hours was highly significant and the adjusted R 2 was 0.89.

Required

How would you explain a negative coefficient; does it seem likely that the more the equipment is operated, the less the company spends on maintenance?

interpretation of regression results simple regression a local restaurant fred rsquo 690985

Interpretation of Regression Results: Simple Regression

A local restaurant, Fred’s Fish Fry, is estimating nonfood kitchen costs (labor, supervision, utilities,  etc.) based on food cost. Data were gathered for the past 24 months and analyzed using a spreadsheet program. The following output was generated:

Equation Intercept

$14,000

Coefficient on food cost

 225%

Statistical data

 

Correlation coefficient

0.483

R2

0.233

The company is planning to operate at a level of $15,000 of food costs per month for the coming year.

Required

a. Use the regression output to write the nonfood cost equation.

b. Based on the cost equation, compute the estimated nonfood kitchen costs (labor, supervision, utilities, etc.) per month for the coming year.

c. Fred has asked you for advice on whether he should rely on the estimate. What will you say?

learning curves assume that general dynamics which manufactures high technology inst 690986

Learning Curves

Assume that General Dynamics, which manufactures high technology instruments for spacecraft, is considering the sale of a navigational unit to a government agency in India that wishes to launch its own communications satellite. The government agency plans to purchase 8 units, although it would also consider buying 16 units. General Dynamics has started a chart relating labor time required to units produced.

 

Time Required to

Unit Produced (X)

Produce the Xth Unit

1

10,000 hours

2

8,000 hours

4

6,400 hours

8

?

16

?

Required

a. Complete the chart by filling in the labor time required to produce 8 and 16 units.

b. Assume that labor time costs $125 per hour. Compare the cost of producing the first unit to the cost of producing the 16th unit. What is the percentage of the cost of the 16th unit to the cost of the first unit?

learning curves assume that whee cheatham and howe is an auditing firm that has foun 690987

Learning Curves

Assume that Whee, Cheatham, and Howe is an auditing firm that has found that its summer interns are subject to a 90 percent learning curve for one of its important tasks, proofreading financial statements. For one of its interns, Kim Down, the firm has started to analyze the relation between time and financial statement proofreading.

Financial Statements

Time Required to Proofread the

Proofread (X )

Xth Financial Statement

1

2.0 hours

2

1.8 hours

4

?

8

?

16

?

Required

a. Complete the chart by filling in the time required to proofread 4, 8, and 16 financial statements.

b. Assume that Kim’s labor time costs $20 per hour. Compare the cost of proofreading the first financial statement to the cost of proofreading the 16th financial statement. What is the percentage of the cost of proofreading the 16th financial statement to the cost of proofreading the first financial statement?

special order andreasen corporation manufactures thermostats for office buildings th 690942

Special Order

Andreasen Corporation manufactures thermostats for office buildings. The following is the cost of each unit:

Materials

$18.00

Labor

7.00

Variable overhead

2.00

Fixed overhead ($900,000 per year; 100,000 units per year)

9.00

Total

$36.00

Simpson Company has approached Andreasen with an offer to buy 5,000 thermostats at a price of $30 each. The regular price is $50. Andreasen has the capacity to produce the 5,000 additional units without affecting its current production of 100,000 units. Simpson requires that each unit use their branding, which requires a more expensive label, resulting in an additional $1 per unit material cost. The labor cost of affixing the label will be the same as for the current models. The Simpson order will also require a one time rental of packaging equipment for $15,000.

Required

a. Prepare a schedule to show the impact of filling the Simpson order on Andreasen’s profits for the year.

b. Would you recommend that Andreasen accept the order?

c. Considering only profit t, what is the minimum quantity of thermostats in the special order that would make it profit table assuming capacity is available?

special order mission electronics manufactures and sells basic dvd players for sale 690943

Special Order

Mission Electronics manufactures and sells basic DVD players for sale under various generic store brand names. The cost of one of their models follows:

Materials

$18.00

Labor

12.00

Variable overhead

5.00

Fixed overhead ($2,700,000 per year; 450,000 units per year) . .

6.00

Total

$41.00

Pacific Cash & Carry, a chain of low price electronic sales and rental outlets, has asked Mission to supply them with 30,000 players for a special promotion Pacific is planning. Pacific has offered to pay Mission a unit price of $42 per DVD player. The regular selling price is $60. The special order would require some modification to the basic model. These modifications would add $4.00 per unit in material cost, $1.50 per unit in labor cost, and $0.50 in variable overhead cost. Although Mission has the capacity to produce the 30,000 units without affecting its regular production of 450,000 units, a one time rental of special testing equipment to meet Pacific’s requirements would be needed. The equipment rental would be $45,000 and would allow Mission to test up to 50,000 units.

Required

a. Prepare a schedule to show the impact of filling the Pacific order on Mission’s profits for the year.

b. Would you recommend that Mission accept the order?

c. Considering only profit t, what is the minimum quantity of DVD players in the special order that would make it profit table?

make or buy decisions mobility partners makes wheelchairs and other assistive device 690947

Make or Buy Decisions

Mobility Partners makes wheelchairs and other assistive devices. For years it has made the rear wheel assembly for its wheelchairs. A local bicycle manufacturing firm, Trailblazers, Inc., offered to sell these rear wheel assemblies to Mobility. If Mobility makes the assembly, its cost per rear wheel assembly is as follows (based on annual production of 2,000 units):

Direct materials

$ 25

Direct labor

53

Variable overhead

16

Fixed overhead

47

Total

$141

Trailblazers has offered to sell the assembly to Mobility for $110 each. The total order would amount to 2,000 rear wheel assemblies per year, which Mobility’s management will buy instead of make if Mobility can save at least $10,000 per year. Accepting Trailblazers’s offer would eliminate annual fixed overhead of $40,000.

Required

Should Mobility make rear wheel assemblies or buy them from Trailblazers? Prepare a schedule that shows the differential costs per rear wheel assembly.

make or buy decisions mel rsquo s meals 2 go purchases cookies that it includes in t 690948

Make or Buy Decisions

Mel’s Meals 2 Go purchases cookies that it includes in the 10,000 box lunches it prepares and        sells annually. Mel’s kitchen and adjoining meeting room operate at 70 percent of capacity. Mel’s purchases the cookies for $0.60 each but is considering making them instead. Mel’s can bake each cookie for $0.20 for materials, $0.15 for direct labor, and $0.45 for overhead without increasing its capacity. The $0.45 for overhead includes an allocation of $0.30 per cookie for fixed overhead. However, total fixed overhead for the company would not increase if Mel’s makes the cookies.

Mel himself has come to you for advice. “It would cost me $0.80 to make the cookies, but only $0.60 to buy. Should I continue buying them?” Materials and labor are variable costs, but variable overhead would be only $0.15 per cookie. Two cookies are put into every lunch.

Required

How would you advise Mel? Prepare a schedule to show the differential costs.

dropping product lines atlantic soup company is presently operating at 75 percent of 690949

Dropping Product Lines

Atlantic Soup Company is presently operating at 75 percent of capacity. Worried about the company’s performance, the president is considering dropping its clam chowder line. If clam chowder is dropped, the revenue associated with it would be lost and the related variable costs saved. In addition, the company’s total fixed costs would be reduced by 15 percent.

Product

Tomato

Clam Chowder

Chicken Noodle

Sales

$32,600

$42,800

$51,200

Variable costs

22,000

38,600

40,100

Contribution margin

$10,600

$ 4,200

$ 11,100

Fixed costs allocated to each product line

4,700

6,000

7,100

Operating profit (loss)

$ 5,900

$ (1,800)

$ 4,000

Required

Prepare a differential cost schedule like the one in Exhibit 4.8 to indicate whether Atlantic should drop the clam chowder product line.

 

Status Quo:

Alternative:

Difference

 

Keep Prints

Drop Prints

$10,000 decrease

Sales revenue           

$80,000

$70,000

8,000 decrease

Cost of sales (all variable)  

53,000

45,000

$ 2,000 decrease

Contribution margin     

$27,000

$25,000

 

Less fi xed costs:

     

Rent                 

4,000

4,000

–0–

Salaries               

5,000

4,000

1,000 decrease

Marketing and administrative

3,000

2,750

250 decrease

Operating profi t (loss)     

$15,000

$14,250

$ 750 decrease

Exhibit 4.8 Differential Analysis— U Develop

dropping product lines free flight airlines is presently operating at 70 percent of 690950

Dropping Product Lines

Free flight Airlines is presently operating at 70 percent of capacity. Management of the airline is considering dropping Freeflight’s routes between Europe and the United States. If these routes are dropped, the revenue associated with the routes would be lost and the related variable costs saved. In addition, the company’s total fixed costs would be reduced by 20 percent. Segmented income statements for a typical month appear as follows (all amounts in millions of dollars):

Routes

Within U.S.

Within Europe

Between U.S. and Europe

Sales

$3.4

$2.6

$ 2.8

Variable costs

1.4

1.0

1.5

Fixed costs allocated to routes

1.7

1.3

1.4

Operating profit t (loss)

$0.3

$0.3

$(0.1)

 

Required

 

 

Status Quo:

Alternative:

Difference

 

Keep Prints

Drop Prints

$10,000 decrease

Sales revenue           

$80,000

$70,000

8,000 decrease

Cost of sales (all variable)  

53,000

45,000

$ 2,000 decrease

Contribution margin     

$27,000

$25,000

 

Less fi xed costs:

     

Rent                 

4,000

4,000

–0–

Salaries               

5,000

4,000

1,000 decrease

Marketing and administrative

3,000

2,750

250 decrease

Operating profi t (loss)     

$15,000

$14,250

$ 750 decrease

Exhibit 4.8 Differential Analysis— U Develop

 

Prepare a differential cost schedule like the one in Exhibit 4.8 to indicate whether Free flight should drop the routes between the United States and Europe.

theory of constraints playful pens inc makes a single model of a pen the cartridge f 690952

Theory of Constraints

Playful Pens, Inc., makes a single model of a pen. The cartridge for the pen (which contains the ink) is manufactured on one machine. The cartridge holder (which you hold when you hold the pen) is manufactured on another machine. Monthly capacities and production levels are as follows:

 

Machine 1 (Cartridge)

Machine 2 (Holders)

Monthly capacity

1,000,000

800,000

Monthly production

800,000

800,000

The company could sell 1,000,000 pens per month. The units (cartridge inside of holder) sell for $10 each and have a variable cost of $4 each. Fixed costs are $4,000,000 per month.

Required

a. Is there a bottleneck at Playful Pens? If so, where is it?

b. Playful Pens’s production supervisors state they could increase machine 2’s capacity by 200,000 per month by producing holders on the weekend. Producing on the weekend would not affect the sales price. Variable cost per unit would increase by $1 for those produced on the weekend because of the premium paid to labor. Fixed costs would also increase by $800,000 per month. Should Playful Pens produce holders on the weekend?

c. Independent of the situation in requirement ( b ), Playful Pens could expand the capability of machine 2 by adding additional workers to perform ongoing maintenance. This would increase its capacity by 100,000 holders per month. This would not affect sales price or fixed costs, but would increase variable cost to $4.50 per unit for all units produced. Should Playful Pens expand machine 2’s capability by adding these additional workers?

special order hi speed electronics manufactures low cost consumer grade computers it 690953

Special Order

Hi Speed Electronics manufactures low cost, consumer grade computers. It sells these computers to various electronics retailers to market under store brand names. It manufactures two computers, the Lightning 2.0 and the Lightning 2.4, which differ in terms of speed, included memory, and included hard drive capacity. The following information is available:

Costs per Unit

Lightning 2.0

Lightning 2.4

Direct materials

$ 65

$ 75

Direct labor

30

40

Variable overhead

15

20

Fixed overhead

90

120

Total cost per unit

$ 200

$ 255

Price

$ 290

$ 390

Units sold

4,000

2,000

The average wage rate is $20 per hour. Variable overhead varies with the quantity of direct labor hours. The plant has a capacity of 20,000 direct labor hours, but current production uses only 10,000 direct labor hours.

Required

a. A nationwide discount chain has offered to buy 2,500 Lightning 2.0 computers and 2,500 Lightning 2.4 computers if the price is lowered to $200 and $250, respectively, per unit. If Hi Speed accepts the offer, how many direct labor hours will be required to produce the additional computers? How much will the profit t increase (or decrease) if Hi Speed accepts this proposal? Prices on regular sales will remain the same.

b. Suppose that the nationwide discount chain has offered instead to buy 3,500 each of the two models at $200 and $250, respectively. This customer will purchase the 3,500 units of each model only in an all or nothing deal. That is, Hi Speed Electronics must provide all 3,500 units of each model or none. Hi Speed’s management has decided to fill the entire special order for both models. In view of its capacity constraints, Hi Speed will reduce sales to regular customers as needed to fill the special order. How much will the profits change if the order is accepted? Assume that the company cannot increase its production capacity to meet the extra demand.

c. Answer the question in requirement ( b ), assuming instead that the plant can work overtime. Direct labor costs for the overtime production increase to $30 per hour. Variable overhead costs for overtime production are $5 per hour more than for normal production.

special orders sherene nili manages a company that produces wedding gowns she produc 690954

Special Orders

Sherene Nili manages a company that produces wedding gowns. She produces both a custom product that is made to order and a standard product that is sold in bridal salons. Her accountant prepared the following forecasted income statement for March, which is a busy month:

 

Custom Dresses

Standard Dresses

Total

Number of dresses

10

20

30

Sales revenue

$50,000

$25,000

$75,000

Materials

$10,000

$ 8,000

$18,000

Labor

20,000

9,000

29,000

Machine depreciation

600

300

900

Rent

4,200

2,800

7,000

Heat and light

1,000

600

1,600

Other production costs

 

 

2,800

Marketing and administration

 

 

7,700

Total costs

 

 

$67,000

Operating profit

 

 

$ 8,000

         

Ms. Nili already has orders for the 10 custom dresses reflected in the March forecasted income statement. The depreciation charges are for machines used in the respective product lines. Machines depreciate at the rate of $1 per hour based on hours used, so these are variable costs. In March, cutting and sewing machines are expected to operate for 900 hours, of which 600 hours will be used to make custom dresses. The rent is for the building space, which has been leased for several years at $7,000 per month. The rent, heat, and light are allocated to the product lines based on the amount of floor space occupied.

A valued customer, who is a wedding consultant, has asked Ms. Nili for a special favor. This customer has a client who wants to get married in early April. Ms. Nili’s company is working at capacity and would have to give up some other business to make this dress. She can’t renege on custom orders already agreed to, but she can reduce the number of standard dresses produced in March to 10. Ms. Nili would lose permanently the opportunity to make up the lost production of standard dresses because she has no unused capacity for the foreseeable future. The customer is willing to pay $25,000 for the special order. Materials and labor for the order will cost $6,000 and $10,000, respectively. The special order would require 140 hours of machine time. Ms. Nili’s company would save 150 hours of machine time from the standard dress business given up. Rent, heat and light, and other production costs would not be affected by the special order.

Required

a. Should Ms. Nili take the order? Explain your answer.

b. What is the minimum price Ms. Nili should accept to take the special order?

c. What are the other factors, if any, besides price that she should consider?

pricing decisions sam martina aretha roxi and tomas smart operate a management consu 690955

Pricing Decisions

Sam, Martina, Aretha, Roxi, and Tomas (SMART) operate a management consulting firm. The firm has just received an inquiry from a prospective client about its prices for educational seminars to teach its supervisors better communications skills. The prospective client wants bids for three alternative activity levels: (1) one seminar with 20 participants, (2) four seminars with 20 participants each (80 participants total), or (3) eight seminars with 140 participants in total. The consulting firm’s accountants have provided the following differential cost estimates:

Setup costs for the entire job

$ 600

Materials costs per participant (brochures, handouts, coffee, lunch, etc.)  

100

Differential Direct Labor Costs:

 

One seminar

$1,200

Four seminars

5,200

Eight seminars

8,800

In addition to the preceding differential costs, SMART allocates fixed costs to jobs on a directlabor cost basis, at a rate of 75 percent of direct labor costs (excluding setup costs). For example, if direct labor costs are $100, SMART would also charge the job $75 for fixed costs. SMART charges clients for its costs plus 25 percent. For the purpose of charging customers, costs equal the setup costs plus materials costs plus differential labor costs plus allocated fixed costs. SMART has enough excess capacity to handle this job with ease.

Required

a. Assume SMART’s bid equals the total cost, including fixed costs allocated to the job, plus the 25 percent markup on cost. What should SMART bid for each of the three levels of activity?

b. Compute the differential cost (including setup costs) and the contribution to profit t for each of the three levels of activity. Note that fixed costs are not differential costs.

c. Assume the prospective client gives three options. It is willing to accept either of SMART’s bids for the one seminar or four seminar activity levels, but the prospective client will pay only 85 percent of the bid price for the eight seminar package. SMART’s president responds, “We can’t make money in this business by shaving our bids! Let’s take the four seminar option because we make the most profit t on it.” Do you agree? What would be the contribution to profit t for each of the three options?

comprehensive differential costing problem davis kitchen supply produces stoves for 690957

Comprehensive Differential Costing Problem

Davis Kitchen Supply produces stoves for commercial kitchens. The costs to manufacture and market the stoves at the company’s normal volume of 6,000 units per month are shown in the following table.

Unit manufacturing costs Variable materials

$50

 

Variable labor

75

 

Variable overhead

25

 

Fixed overhead

60

 

Total unit manufacturing costs

25

 

Unit marketing costs Variable

 

$210

Fixed

70

 

Total unit marketing costs

 

95

Total unit costs

 

$305

Unless otherwise stated, assume that no connection exists between the situation described in each question; each is independent. Unless otherwise stated, assume a regular selling price of $370 per unit. Ignore income taxes and other costs that are not mentioned in the table or in the question itself.

Required

a. Market research estimates that volume could be increased to 7,000 units, which is well within production capacity limitations if the price were cut from $370 to $325 per unit. Assuming that the cost behavior patterns implied by the data in the table are correct, would you recommend taking this action? What would be the impact on monthly sales, costs, and income?

b. On March 1, the federal government offers Davis a contract to supply 1,000 units to military bases for a March 31 delivery. Because of an unusually large number of rush orders from its regular customers, Davis plans to produce 8,000 units during March, which will use all available capacity. If it accepts the government order, it would lose 1,000 units normally sold to regular customers to a competitor. The government contract would reimburse its “share of March manufacturing costs” plus pay a $50,000 fixed fee (profit t). (No variable marketing costs would be incurred on the government’s units.) What impact would accepting the government contract have on March income? (Part of your problem is to figure out the meaning of “share of March manufacturing costs.”)

c. Davis has an opportunity to enter a highly competitive foreign market. An attraction of the foreign market is that its demand is greatest when the domestic market’s demand is quite low; thus, idle production facilities could be used without affecting domestic business. An order for 2,000 units is being sought at a below normal price to enter this market. For this order, shipping costs will total $40 per unit; total (marketing) costs to obtain the contract will be $4,000.

No other variable marketing costs would be required on this order, and it would not affect domestic business. What is the minimum unit price that Davis should consider for this order of 2,000 units?

d. An inventory of 460 units of an obsolete model of the stove remains in the stockroom. These must be sold through regular channels (thus incurring variable marketing costs) at reduced prices or the inventory will soon be valueless. What is the minimum acceptable selling price for these units?

e. A proposal is received from an outside contractor who will make and ship 2,000 stoves per month directly to Davis’s customers as orders are received from Davis’s sales force. Davis’s fixed marketing costs would be unaffected, but its variable marketing costs would be cut by 20 percent for these 2,000 units produced by the contractor. Davis’s plant would operate at two thirds of its normal level, and total fixed manufacturing costs would be cut by 30 percent. What in house unit cost should be used to compare with the quotation received from the supplier? Should the proposal be accepted for a price (that is, payment to the outside contractor) of $215 per unit?

f. Assume the same facts as in requirement ( e ) except that the idle facilities would be used to produce 1,600 modified stoves per month for use in extreme climates. These modified stoves could be sold for $450 each, while the costs of production would be $275 per unit variable manufacturing expense. Variable marketing costs would be $50 per unit. Fixed marketing and manufacturing costs would be unchanged whether the original 6,000 regular stoves were manufactured or the mix of 4,000 regular stoves plus 1,600 modified stoves were produced. Should the proposal be accepted for a price of $215 per unit to the outside contractor?

make or buy king city specialty bikes kcsb produces high end bicycles the costs to m 690958

Make or Buy

King City Specialty Bikes (KCSB) produces high end bicycles. The costs to manufacture and market the bicycles at the company’s volume of 2,000 units per month are shown in the following table:

Unit manufacturing costs Variable costs

$240

 

Fixed overhead

120

 

Total unit manufacturing costs

60

 

Unit nonmanufacturing costs Variable

 

$360

Fixed

140

 

Total unit nonmanufacturing costs . .

 

200

Total unit costs

 

$560

The company has the capacity to produce 2,000 units per month and always operates at full capacity.

The bicycles sell for $600 per unit.

Required

a. KCSB receives a proposal from an outside contractor who will assemble 800 of the 2,000 bicycles per month and ship them directly to KCSB’s customers as orders are received from KCSB’s sales force. KCSB would provide the materials for each bicycle, but the outside contractor would assemble, box, and ship the bicycles. The variable manufacturing costs would be reduced by 40 percent for the 800 bicycles assembled by the outside contractor. KCSB’s fixed nonmanufacturing costs would be unaffected, but its variable nonmanufacturing costs would be cut by 60 percent for these 800 units produced by the outside contractor. KCSB’s plant would operate at 60 percent of its normal level, and total fixed manufacturing costs would becut by 20 percent. What in house unit cost should be compared with the quotation received from the outside contractor? Should the proposal be accepted for a price (that is, payment to the contractor) of $140 per unit?

b. Assume the same facts as in requirement (a) but assume that the idle facilities would be used to produce 80 specialty racing bicycles per month. These racing bicycles could be sold for  $8,000 each, while the costs of production would be $5,600 per unit variable manufacturing cost. Variable marketing costs would be $200 per unit. Fixed nonmanufacturing and manufacturing costs would be unchanged whether the original 2,000 regular bicycles were manufactured or the mix of 1,200 regular bicycles plus 80 racing bicycles was produced. Considering this opportunity to use the freed up space, what is the maximum purchase price per unit that KCSB should be willing to pay the outside contractor to assemble regular bicycles? Should the contractor’s proposal of $140 per unit be accepted?

decision whether to add or drop carson corporation produces and sells three products 690959

Decision Whether to Add or Drop

Carson Corporation produces and sells three products, Alpha, Beta, and Gamma, in a local market and in a regional market. At the end of the first quarter of the current year, the following income statement (in thousands of dollars) has been prepared:

 

Total

Local

Regional

Sales revenue

$15,600

$12,000

$3,600

Cost of goods sold

12,120

9,300

2,820

Gross margin

$ 3,480

$ 2,700

$ 780

Marketing costs

1,260

720

540

Administrative costs

624

480

144

Total marketing and administrative

$ 1,884

$ 1,200

$ 684

Operating profits

$ 1,596

$ 1,500

$ 96

Management has expressed special concern with the regional market because of the extremely poor return on sales. This market was entered a year ago because of excess capacity. It was originally believed that the return on sales would improve with time, but after a year, no noticeable improvement can be seen from the results as reported in the preceding quarterly statement.

In attempting to decide whether to eliminate the regional market, the following information has been gathered:

 

Products

 

Alpha

Beta

Gamma

Sales revenue$6,000

$6,000

$4,800

$4,800

Variable manufacturing costs as a percentage of sales revenue

60%

70%

60%

Variable marketing costs as a percentage of sales revenue

3

2

2

           

 

Product Sales by Markets

Local

Regional

Alpha

$4,800

$1,200

Beta

3,600

1,200

Gamma

3,600

1,200

All administrative costs and fixed manufacturing costs would not be affected by eliminating the regional market. Marketing costs that are not listed above as variable are fixed for the period and separable by market. Fixed marketing costs assigned to the regional market would be saved if that market was eliminated.

Required

a. Assuming there are no alternative uses for Carson’s present capacity, would you recommend dropping the regional market? Why or why not?

b. Prepare the quarterly income statement showing contribution margins by products. Do not allocate fixed costs to products.

c. It is believed that a new product can be ready for sale next year if Carson decides to go ahead with continued research. The new product would replace Gamma and can be produced by simply converting equipment presently used in producing product Gamma. This conversion will increase fixed costs by $120,000 per quarter. What must be the minimum contribution margin per quarter for the new product to make the changeover financially feasible?

decision whether to close a store power music owns five music stores where it sells 690960

Decision Whether to Close a Store

Power Music owns five music stores, where it sells music, instruments, and supplies. In addition, it rents instruments. At the end of last year, the new accounts showed that although the business as a whole was profit table, the Fifth Avenue store had shown a substantial loss. The income statement for the Fifth Avenue store for last month follows:

POWER MUSIC

Fifth Avenue Store

Partial Income Statement

Sales

 

$650,000

Cost of goods sold

 

560,000

Gross margin

 

$ 90,000

Costs:

 

 

Payroll, direct labor, and supervisiona

$ 51,000

 

Rentb

16,100

 

State taxesc

2,500

 

Insurance on inventory

18,400

 

Depreciationd

7,500

 

Administration and general officee

20,000

 

Interest for inventory carrying costsf

4,500

 

Total costs

 

120,000

Loss

 

$ (30,000)

Additional computations:

aThese costs would be saved if the store was closed.

bThe rent would be saved if the store was closed.

cAssessed annually on the basis of average inventory on hand each month.

d8.5 percent of cost of departmental equipment. The equipment has no salvage value, and

Power Music would incur no costs in scrapping it.

eAllocated on the basis of store sales as a fraction of total company sales. Management estimates that 10% of these costs allocated to the Fifth Avenue store could be saved if the store was closed.

fBased on average inventory quantity multiplied by the company’s borrowing rate for three month loans.

Analysis of these results has led management to consider closing the Fifth Avenue store. Members of the management team agree that keeping the Fifth Avenue store open is not essential to maintaining good customer relations and supporting the rest of the company’s business. In other words, eliminating the Fifth Avenue store is not expected to affect the amount of business done by the other stores.

Required

What action do you recommend to Power Music’s management? Write a short report to management recommending whether or not to close the Fifth Avenue store. Include the reasons for your recommendation.

closing a plant you have been asked to assist the management of ironwood corporation 690961

Closing a Plant

You have been asked to assist the management of Ironwood Corporation in arriving at certain decisions. Ironwood has its home office in Michigan and leases factory buildings in Wisconsin, Minnesota, and North Dakota, all of which produce the same product. Ironwood’s management provided you a projection of operations for next year as follows:

 

Total

Wisconsin

Minnesota

North Dakota

Sales revenue

$880,000

$440,000

$280,000

$160,000

Fixed costs

 

 

 

 

Factory

220,000

112,000

56,000

52,000

Administration

70,000

42,000

22,000

6,000

Variable costs

290,000

 133,000

85,000 

72,000

Allocated home office costs

100,000

45,000

35,000

20,000

Total

$680,000

$332,000

$198,000

$150,000

Operating profit

$200,000

$108,000

$ 82,000

$ 10,000

The sales price per unit is $5.

Due to the marginal results of operations of the factory in North Dakota, Ironwood has decided to cease its operations and sell that factory’s machinery and equipment by the end of this year. Ironwood expects that the proceeds from the sale of these assets would equal all termination costs. Ironwood, however, would like to continue serving most of its customers in that area if it is economically feasible and is considering one of the following three alternatives:

·         Expand the operations of the Minnesota factory by using space presently idle. This move would result in the following changes in that factory’s operations:

Increase over Minnesota factory’s current operations

Sales revenue

50%

Fixed costs

 

Factory

20

Administration

10

Under this proposal, variable costs would be $2 per unit sold.

·         Enter into a long term contract with a competitor who will serve that area’s customers. This competitor would pay Ironwood a royalty of $1 per unit based on an estimate of 30,000 units being sold.

·         Close the North Dakota factory and not expand the operations of the Minnesota factory. Total home office costs of $100,000 will remain the same under each situation.

Required

To assist the management of Ironwood Corporation, prepare a schedule computing Ironwood’s estimated operating profit t from each of the following options:

a. Expansion of the Minnesota factory.

b. Negotiation of long term contract on a royalty basis.

c. Shutdown of North Dakota operations with no expansion at other locations.

optimum product mix austin enterprises makes and sells three types of dress shirts m 690962

Optimum Product Mix

Austin Enterprises makes and sells three types of dress shirts. Management is trying to determine the most profit table mix. Sales prices, demand, and use of manufacturing inputs follow:

 

Basic

Classic

Formal

Sales price

$30

$64

$190

Maximum annual

 

 

 

demand (units)

20,000

10,000

30,000

Input requirement per unit

 

 

 

Direct material

.5 yards

.3 yards

.6 yards

Direct labor

.7 hours

2 hours

7 hours

 

Costs

 

Variable costs

 

Materials

$20 per yard

Direct labor

$16 per hour

Factory overhead

$4 per direct labor hour

Marketing

10% of sales price

Annual fixed costs

 

Manufacturing

$36,000

Marketing

$8,000

Administration

$30,000

The company faces two limits: (1) the volume of each type of shirt that it can sell (see maximum annual demand) and (2) 30,000 direct labor hours per year caused by the plant layout.

Required

Show supporting data in good form.

a. How much operating profit t could the company earn if it were able to satisfy the annual demand?

b. Which of the three product lines makes the most profit table use of the constrained resource, direct labor?

c. Given the information in the problem so far, what product mix do you recommend?

d. How much operating profit t should your recommended product mix generate?

e. Suppose that the company could expand its labor capacity by running an extra shift that could provide up to 10,000 more hours. The direct labor cost would increase from $16 to $19 per hour for all hours of direct labor used. What additional product(s) should Austin manufacture and what additional profit t would be expected with the use of the added shift?

optimum product mix the bubble company produces a variety of bottles from recycled p 690963

Optimum Product Mix

The Bubble Company produces a variety of bottles from recycled plastic. The company has one particular machine on which it can produce either of two types of water bottles, 1 liter bottles or 1/2 liter bottles. Sales demand for both products is such that the machine could operate at full capacity on either of the products, and Bubble can sell all output at current prices. One unit of the 1/2 liter product requires one hour of machine time per unit of output and one unit of the 1 liter bottle requires two hours of machine time. Each “unit” is a box that contains 150 bottles. Following are the costs per unit for the bottles:

 

Per Unit (150 bottles in a unit)

 

1/2 Liter Bottles

1 Liter Bottles

Selling price

$30

$55

Costs

 

 

Materials

$ 8

$15

Labor

1

1

Machine maintenance and depreciationa

8

16

Allocated portion of fixed factory costsb

6

6

Total cost per unit

$23

$38

Gross margin per unit

$ 7

$17

a This item is a variable cost because it is based on machine usage.

b This item is a fixed cost because it is unaffected by the usage of the machine.

All other costs are the same whether Bubble produces 1 liter bottles, 1/2 liter bottles, or both, so you may ignore them.

Required

Should Bubble produce 1 liter bottles, 1/2 liter bottles, or both?

optimum product mix excel solver slavin corporation manufactures two products alpha 690964

Optimum Product Mix Excel Solver

Slavin Corporation manufactures two products, Alpha and Delta. Each product requires time on a single machine. The machine has a monthly capacity of 500 hours. Total market demand for the two products is limited to 150 units (each) monthly. Slavin is currently producing 110 Alphas and 110 Deltas each month. Cost and machine usage data for the two products is shown in the following spreadsheet, which Slavin managers use for planning purposes:

 

A

B

C

D

E

 

F

G

H

 

1

 

 

Alpha

 

 

Delta

 

 

 

 

2

Price

 

$ 120

 

 

$ 150

 

 

 

 

3

Less variable costs per unit

 

 

 

 

 

 

 

 

 

4

Material

 

20

 

 

 

35

 

 

 

5

Labor

 

26

 

 

 

37

 

 

 

6

Overhead

 

14

 

 

 

14

 

 

 

7

Contribution margin per unit

 

$ 60

 

 

 

$ 64

 

 

 

8

 

 

 

 

 

 

 

 

 

 

9

Fixed costs

 

 

 

 

 

 

 

Total

 

10

Manufacturing

 

 

 

 

 

 

 

$8,000

 

11

Marketing and administrative

 

 

 

 

 

 

 

$5,000

 

12

 

 

 

 

 

 

 

 

$ 13,000

 

13

Machine hours per unit

 

2.0

 

 

 

2.5

 

 

 

14

 

 

 

 

 

 

 

 

 

 

15

 

 

 

 

 

 

 

 

 

 

16

Machine hours used

 

 

 

 

 

 

 

495

 

17

Machine hours available

 

 

 

 

 

 

 

500

 

18

 

 

 

 

 

 

 

 

 

 

19

Quantity produced

 

110

 

 

 

110

 

 

 

20

Maximum demand

 

150

 

 

 

150

 

 

 

21

Profit

 

 

 

 

 

 

 

 

 

22

 

 

 

 

 

 

 

 

 

 

Required

a. What is the optimal production schedule for Slavin? In other words, how many Alphas and Deltas should the company produce each month to maximize monthly profit t?

b. If Slavin produces at the level found in (a), how much will monthly profit t increase over the current production schedule?

optimum product mix excel solver layton machining company lmc manufactures two versi 690965

Optimum Product Mix Excel Solver

Layton Machining Company (LMC) manufactures two versions of a basic machine tool. One version is a standard model and one is a custom model, which requires some additional work and slightly higher grade materials. The manufacturing process at LMC requires that each product go through two departments, Grinding and Finishing. The process in each department uses a single type of machine. Total machine capacity in Grinding is 50,000 hours, and in Finishing, total machine capacity is 30,000 hours. (Each department has multiple machines.) Total market demand is limited to 100,000 standard units and 120,000 custom units monthly. LMC is currently producing 90,000 standard units and 50,000 custom units each month. Cost and machine usage data for the two products follow:

 

A

B

C

D

E

F

G

H

 

1

 

 

Standard

 

 

Custom

 

 

 

2

Price

 

$6.00

 

 

$8.00

 

 

 

3

Less variable costs per unit

 

 

 

 

 

 

 

 

4

Material

 

1.50

 

 

2.00

 

 

 

5

Labor

 

1.25

 

 

1.50

 

 

 

6

Overhead

 

1.75

 

 

2.50

 

 

 

7

Contribution margin per unit

 

$1.50

 

 

2.00

 

 

 

8

 

 

 

 

 

 

 

 

 

9

Fixed costs

 

 

 

 

 

 

Total

 

10

Manufacturing

 

 

 

 

 

 

$76,000

 

11

Marketing and administrative

 

 

 

 

 

 

$37,000

 

12

 

 

 

 

 

 

 

$ 113,000

 

13

 

 

 

 

 

 

 

 

 

14

Grinding machine hours per unit

 

0.2

 

 

0.3

 

 

 

15

Finishing machine hours per unit

 

0.1

 

 

0.4

 

 

 

16

Grinding machine hours used

 

 

 

 

 

 

33,000

 

17

Grinding machine hours available

 

 

 

 

 

 

50,000

 

18

Finishing machine hours used

 

 

 

 

 

 

29,000

 

19

Finishing machine hours available

 

 

 

 

 

 

30,000

 

20

 

 

 

 

 

 

 

 

 

21

Quantity produced

 

 

 

 

50,000

 

 

 

22

Maximum demand

 

 

 

 

120,000

 

 

 

23

Profit

 

 

 

 

 

 

 

 

Required

a. What is the optimal production schedule for LMC? In other words, how many standard units and custom units should the company produce each month to maximize monthly profit t?

b. If LMC produces at the level found in requirement (a), how much will monthly profit t increase over the current production schedule?

[[Integrative Cases]]

the effect of cost structure on predatory pricing to win a predatory pricing case la 690966

The Effect of Cost Structure on Predatory Pricing

To win a predatory pricing case, law enforcement offcials traditionally have had to prove that a company has sold products or services for less than their average variable cost. Companies with relatively high fixed costs and low variable costs are less likely to be accused of predatory pricing than are companies with high variable and low fixed costs. A court case in which the U.S. Department of Justice alleged that American Airlines had committed predatory pricing against smaller airlines demonstrates this point.

The airline industry has relatively high fixed costs and low variable costs, as least in the short run. If one defines a “unit” as a passenger flying an already scheduled flight, the additional cost of a passenger is small—charges for credit cards, a small amount of fuel because of extra weight, a beverage or two, and not much else. If one defines a “unit” as a flight, then more costs are variable—flight crew costs, fuel, and the cost of baggage handling, for example. Even if the unit is a flight, a large portion of the total costs is fixed.

American Airlines had dropped its fares when smaller airlines scheduled competing flights from the Dallas–Fort Worth airport to Kansas City, Wichita, and other cities, arguing that this was simply business competition in the marketplace. The judge in the case acknowledged that American had been a tough competitor but ruled that American had priced its tickets above their average variable cost. Therefore, he ruled that the case against American should be dropped.

Required

a. Why is the relation between price and variable cost an issue in predatory pricing?

b. Identify companies or industries in which variable costs are relatively low compared to fixed costs, thus making predatory pricing hard to prove.

c. Identify companies in which variable costs are relatively high compared to fixed costs.

cvp analysis mdash sensitivity analysis spreadsheet recommended alameda tile sells p 690917

CVP Analysis—Sensitivity Analysis (spreadsheet recommended)

Alameda Tile sells products to many people remodeling their homes and thinks that they could profitably offer courses on tile installation, which might also increase the demand for their products. The basic installation course has the following (tentative) price and cost characteristics:

Tuition

$ 400 per student

Variable costs

 

(tiles, supplies, and so on)

240 per student

Fixed costs (advertising,

 

salaries, and so on)

80,000 per year

Required

a. What enrollment will enable Alameda Tile to break even?

b. How many students will enable Alameda Tile to make an operating profit t of $40,000 for the year?

c. Assume that the projected enrollment for the year is 800 students for each of the following (considered independently):

1. What will be the operating profit t (for 800 students)?

2. What would be the operating profit t if the tuition per student (that is, sales price) decreased by 10 percent? Increased by 20 percent?

3. What would be the operating profit t if variable costs per student decreased by 10 percent? Increased by 20 percent?

4. Suppose that fixed costs for the year are 10 percent lower than projected, whereas variable costs per student are 10 percent higher than projected. What would be the operating profit t for the year?

extensions of the cvp model mdash semifixed step costs sam rsquo s sushi serves only 690918

Extensions of the CVP Model—Semifixed (Step) Costs

Sam’s Sushi serves only a fixed price lunch. The price of $10 and the variable cost of $4 per meal remain constant regardless of volume. Sam can increase lunch volume by opening and staffing additional check out lanes. Sam has three choices:

 

Monthly Volume Range

Total

 

(Number of Meals)

Fixed Costs

1 Lane

0–5,000

$33,000

2 Lanes

5,001–8,000

39,000

3 Lanes

8,001–10,000

52,500

Required

a. Calculate the break even point(s).

b. If Sam can sell all the meals he can serve, should he operate at one, two, or three lanes? Support your answer.

extensions of the cvp model mdash semifixed step costs cesar rsquo s bottlers bottle 690919

Extensions of the CVP Model—Semifixed (Step) Costs

Cesar’s Bottlers bottles soft drinks in a factory that can operate either one shift, two shifts, or three shifts per day. Each shift is eight hours long. The factory is closed on weekends. The sales price of $2 per case bottled and the variable cost of $0.90 per case remain constant regardless of volume. Cesar’s Bottlers can increase volume by opening and staffing additional shifts. The company has the following three choices:

 

Daily Volume Range

Total Fixed Costs

 

(Number of Cases Bottled)

per Day

1 Shift

(0–2,000)

$1,980

2 Shifts

(2,001–3,600)

3,740

3 Shifts

(3,601–5,000)

5,170

Required

a. Calculate the break even point(s).

b. If Cesar’s Bottlers can sell all the units it can produce, should it operate at one, two, or three shifts? Support your answer.

extensions of the cvp model mdash taxes odd wallow drinks is considering adding a ne 690920

Extensions of the CVP Model—Taxes

Odd Wallow Drinks is considering adding a new line of fruit juices to its merchandise products. This line of juices has the following prices and costs:

Selling price per case (24 bottles) of juice

$ 50

Variable cost per case (24 bottles) of juice

$ 24

Fixed costs per year associated with

 

this product

$8,112,000

Income tax rate

40%

Required

a. Compute Odd Wallow Drinks’s break even point in units per year.

b. How many cases must Odd Wallow Drinks sell to earn $1,872,000 per year after taxes on the juice?

extensions of the cvp model mdash taxes frightproof commuter airlines is considering 690921

Extensions of the CVP Model—Taxes

Frightproof Commuter Airlines is considering adding a new flight to its current schedule from Metro to Hicksville. This route has the following prices and costs:

Selling price per passenger per flight

$ 80

Variable cost per passenger per flight

$ 20

Fixed cost per flight

$2,400

Income tax rate

30%

Required

a. Compute Frightproof’s break even point in number of passengers per flight.

b. How many passengers per flight must Frightproof have to earn $1,050 per flight after taxes?

c. Each aircraft has the capacity for 70 passengers per flight. In view of this capacity limitation, can Frightproof carry enough passengers to break even? Can the company carry enough passengers to earn $1,050 per flight after taxes?

extensions of the cvp model mdash taxes lomas electronics manufactures a portable te 690922

Extensions of the CVP Model—Taxes

Lomas Electronics manufactures a portable testing device for use in oil exploration. The product’s price and cost characteristics are:

Selling price per unit

$ 130

Variable cost per unit

70

Fixed cost per year

420,000

Required

The company must sell 10,000 units annually in order to earn $117,000 in profits after taxes. What is Lomas Electronics’s tax rate?

extensions of the cvp analysis mdash taxes eagle company makes the musicfinder a sop 690924

Extensions of the CVP Analysis—Taxes

Eagle Company makes the MusicFinder, a sophisticated satellite radio. Eagle has experienced a steady growth in sales for the past five years. However, Ms. Luray, Eagle’s CEO, believes that to maintain the company’s present growth will require an aggressive advertising campaign next year. To prepare for the campaign, the company’s accountant, Mr. Bednarik, has prepared and presented to Ms. Luray the following data for the current year, Year 1:

Variable costs:

Direct labor (per unit)

$ 100.00

Direct materials (per unit)

45.00

Variable overhead (per unit)

20.00

Total variable costs (per unit)

$ 165.00

Fixed costs (annual):

Manufacturing

$ 400,000

Selling

300,000

Administrative

800,000

Total fixed costs (annual)

$ 1,500,000

Selling price (per unit)

$400.00

Expected sales revenues, Year 1 (25,000 units)

$10,000,000

Eagle has an income tax rate of 35 percent.

Ms. Luray has set the sales target for Year 2 at a level of $11,200,000 (or 28,000 radios).

Required

a. What is the projected after tax operating profit t for Year 1?

b. What is the break even point in units for Year 1?

c. Ms. Luray believes that to attain the sales target (28,000 radios) will require additional selling expenses of $300,000 for advertising in Year 2, with all other costs remaining constant. What will be the after tax operating profit t for Year 2 if the firm spends the additional $300,000?

d. What will be the break even point in sales dollars for Year 2 if the firm spends the additional $300,000 for advertising?

e. If the firm spends the additional $300,000 for advertising in Year 2, what is the sales level in dollars required to equal the Year 1 after tax operating profit t?

f. At a sales level of 28,000 units, what is the maximum amount the firm can spend on advertising to earn an after tax operating profit t of $750,000?

extensions of the cvp model mdash multiple products on the go inc produces two model 690925

Extensions of the CVP Model—Multiple Products

On the Go, Inc., produces two models of traveling cases for laptop computers: the Programmer and the Executive. The bags have the following characteristics:

Programmer

Executive

Selling price per bag

$70

$100

Variable cost per bag

$30

$40

Expected sales (bags) per year

8,000

12,000

The total fixed costs per year for the company are $819,000.

Required

a. What is the anticipated level of profits for the expected sales volumes?

b. Assuming that the product mix is the same at the break even point, compute the break even point.

c. If the product sales mix were to change to nine Programmer style bags for each Executive style bag, what would be the new break even volume for On the Go?

extensions of the cvp model mdash multiple products sundial inc produces two models 690926

Extensions of the CVP Model—Multiple Products

Sundial, Inc., produces two models of sunglasses: AU and NZ. The sunglasses have the following characteristics:

 

AU

NZ

Selling price per unit

$80

$80

Variable cost per unit

$30

$40

Expected units sold per year

60,000

40,000

The total fixed costs per year for the company are $1,104,000.

Required

a. What is the anticipated level of profits for the expected sales volumes?

b. Assuming that the product mix is the same at the break even point, compute the break even point.

c. If the product sales mix were to change to four pairs of AU sunglasses for each pair of NZ sunglasses, what would be the new break even volume for Sundial, Inc.?

extensions of the cvp model mdash multiple products sell block prepares three types 690927

Extensions of the CVP Model—Multiple Products

Sell Block prepares three types of simple tax returns: individual, partnerships, and (small) corporations.

The tax returns have the following characteristics:

 

Individuals

Partnerships

Corporations

Price charged per tax return

$200

$1,000

$2,000

Variable cost per tax return (including

 

 

 

wage paid to tax preparer)

$180

$900

$1,800

Expected tax returns prepared per year

60,000

4,000

16,000

The total fixed costs per year for the company are $3,690,000.

Required

a. What is the anticipated level of profits for the expected sales volumes?

b. Assuming that the product mix is the same at the break even point, compute the break even point.

c. Suppose the product sales mix changes so that, for every ten tax returns prepared, six are for individuals, one is for a partnership, and three are for corporations. Now what is the breakeven  volume for Sell Block?

extensions of the cvp basic model mdash multiple products and taxes assume that ocea 690929

Extensions of the CVP Basic Model—Multiple Products and Taxes

Assume that Ocean King Products sells three varieties of canned seafood with the following prices and costs:

 

 

Selling Price

Variable Cost

Fixed Cost

 

per Case

per Case

per Month

Variety 1

$ 3

$2

Variety 2

5

3

Variety 3

10

6

Entire firm

$46,200

extensions of the cvp model mdash multiple products and taxes assume that limitless 690931

Extensions of the CVP Model—Multiple Products and Taxes

Assume that Limitless Labs, Inc., offers three basic drug testing services for professional athletes. Here are its prices and costs:

 

Price

Variable Cost

Units Sold

 

per Unit

per Unit

per Year

Basic

$ 500

$ 120

850

Retest

800

400

100

Vital

4,000

2,800

50

Variable costs include the labor costs of the medical technicians at the lab. Fixed costs of $390,000 per year include building and equipment costs and the costs of administration. A basic “unit” is a routine drug test administered. A retest is given if there is concern about the results of the fi rest test, particularly if the test indicates that the athlete has taken drugs that are on the banned drug list. Retests are not done by the laboratory that performed the basic test. A “vital” test is the laboratory’s code for a high profit le case. This might be a test of a famous athlete and/ or a test that might be challenged in court. The laboratory does extra work and uses expensive expert technicians to ensure the accuracy of vital drug tests. Limitless Labs is subject to a 40 percent tax rate.

Required

a. Given the above information, how much will Limitless Labs earn each year after taxes?

b. Assuming the above sales mix is the same at the break even point, at what sales revenue does Limitless Labs break even?

c. At what sales revenue will the company earn $180,000 per year after taxes assuming the above sales mix?

d. Limitless Labs is considering becoming more specialized in retests and vital cases. What would be the company’s break even revenues per year if the number of retest   increased to 400 per year and the number of vital tests increased to 200 per year, while the number of basic tests dropped to 100 per year? With this change in product mix, the company would increase fixed costs to $420,000 per year. What would be the effect of this change in product mix on Limitless Labs’s earnings after taxes per year? If the laboratory’s managers seek to maximize the company’s after tax earnings, would this change be a good idea?

extensions of the cvp model mdash multiple products and taxes assume that painless d 690932

Extensions of the CVP Model—Multiple Products and Taxes

Assume that Painless Dental Clinics, Inc., offers three basic dental services. Here are its prices and costs:

 

Price

Variable Cost

Units Sold

 

per Unit

per Unit

per Year

Cleaning

$ 120

$ 80

9,000

Filling

400

300

900

Capping

1,200

500

100

Variable costs include the labor costs of the dental hygienists and dentists. Fixed costs of $400,000 per year include building and equipment costs, marketing costs, and the costs of administration. Painless Dental Clinics is subject to a 30 percent tax rate on income.

A cleaning “unit” is a routine teeth cleaning that takes about 45 minutes. A filling “unit” is the work done to fill one or more cavities in one session. A capping “unit” is the work done to put a crown on one tooth. If more than one tooth is crowned in a session, then the clinic counts one unit per tooth (e.g., putting crowns on two teeth counts as two units).

Required

a. Given the above information, how much will Painless Dental Clinics, Inc., earn each year after taxes?

b. Assuming the above sales mix is the same at the break even point, at what sales revenue does Painless Dental Clinics, Inc., break even?

c. Assuming the above sales mix, at what sales revenue will the company earn $140,000 per year after taxes?

d. Painless Dental Clinics, Inc., is considering becoming more specialized in cleanings and fillings. What would be the company’s revenues per year if the number of cleanings increased to 12,000 per year, the number of fillings increased to 1,000 per year, while the number of capping dropped to zero? With this change in product mix, the company would increase its fixed costs to $450,000 per year. What would be the effect of this change in product mix on the clinic’s earnings after taxes per year? If the clinic’s managers seek to maximize the clinic’s after tax earnings, would this change be a good idea?

financial modeling three entrepreneurs were looking to start a new brewpub near sacr 690933

Financial Modeling

Three entrepreneurs were looking to start a new brewpub near Sacramento, California, called Roseville Brewing Company (RBC). Brewpubs provide two products to customers—food from the restaurant segment and freshly brewed beer from the beer production segment. Both segments are typically in the same building, which allows customers to see the beer brewing process. After months of research, the owners created a financial model that showed the following projections for the first year of operations:

Sales

 

Beer sales

$ 781,200

Food sales

1,074,150

Other sales

97,650

Total sales

$1,953,000

Less cost of sales

525,358

Gross margin

$1,427,642

Less marketing and administrative expenses

1,125,430

Operating profit

$ 302,212

In the process of pursuing capital through private investors and financial institutions, RBC was approached with several questions. The following represents a sample of the more common questions asked:

• What is the break even point?

• What sales dollars will be required to make $200,000? To make $500,000?

• Is the product mix reasonable? (Beer tends to have a higher contribution margin ratio than food, and therefore product mix assumptions are critical to profit t projections.)

• What happens to operating profit t if the product mix shifts?

• How will changes in price affect operating profit t?

• How much does a pint of beer cost to produce?

It became clear to the owners of RBC that the initial financial model was not adequate for answering these types of questions. After further research, RBC created another financial model that provided the following information for the first year of operations:

Sales

 

 

Beer sales (40% of total sales)

$ 781,200

 

Food sales (55% of total sales)

1,074,150

 

Other sales (5% of total sales)

97,650

 

Total sales

 

$1,953,000

Variable Costs

 

 

Beer (15% of beer sales)

$ 117,180

 

Food (35% of food sales)

375,953

 

Other (33% of other sales)

32,225

 

Wages of employees (25% of sales)

488,250

 

Supplies (1% of sales)

19,530

 

Utilities (3% of sales)

58,590

 

Other: credit card, misc. (2% of sales)

39,060

 

Total variable costs

 

$1,130,788

Contribution margin

 

$ 822,212

Fixed Costs

 

 

Salaries: manager, chef, brewer

$ 140,000

 

Maintenance

30,000

 

Advertising

20,000

 

Other: cleaning, menus, misc

40,000

 

Insurance and accounting

40,000

 

Property taxes

24,000

 

Depreciation

94,000

 

Debt service (interest on debt)

132,000

 

Total fixed costs

 

$ 520,000

Operating profit

 

$ 302,212

Required

a. What were potential investors and financial institutions concerned with when asking the questions listed in the case?

b. Why was the first financial model prepared by RBC inappropriate for answering most of the questions asked by investors and bankers? Be specific.

c. If you were deciding whether to invest in RBC, how would you quickly check the reasonableness of RBC’s projected operating profit t?

d. Why is the question “How much does a pint of beer cost to produce?” difficult to answer?

e. Perform sensitivity analysis by answering the following questions.

1. What is the break even point in sales dollars for RBC?

2. What is the margin of safety for RBC?

3. Why can’t RBC find the break even point in units?

4. What sales dollars would be required to achieve an operating profit t of $200,000? $500,000? What assumptions are made in this calculation?

live oak products has an annual plant capacity to produce 50 000 units its predicted 690934

Live Oak Products has an annual plant capacity to produce 50,000 units. Its predicted operations for the year follow:

Sales revenue (40,000 units at $20 each)

$800,000

Manufacturing costs

 

Variable

$8 per unit

Fixed

$200,000

Selling and administrative costs

 

Variable (commissions on sales)

$2 per unit

Fixed

$40,000

Should the company accept a special order for 4,000 units at a selling price of $15 each, which is subject to half the usual sales commission rate per unit? Assume no effect on fixed costs or regular sales at regular prices. What is the effectof the decision on the company’s operating profit t?

on the move inc manufactures two types of roof racks for automobiles bikerac and kay 690936

On the Move, Inc., manufactures two types of roof racks for automobiles: BikeRac and KayakRac. Data concerning selling prices and costs for each unit follow:

 

BikeRac

KayakRac

Selling price

$100

$80

Materials (variable)

26

23

Direct labor (variable)

5

4

Overhead (90% fixed)

50

40

Gross margin

$ 19

$13

Marketing costs (variable)

4

4

Administrative costs (fixed)

10

8

Profit

$ 5

$ 1

Management decided that at least 5,000 BikeRacs and at least 2,000 KayakRacs must be manufactured and sold each month.

The company’s production facilities are limited by machine capacity in the Assembly Department. Each BikeRac requires 6 minutes and each KayakRac requires 3 minutes in the Assembly Department. A total of 650 hours (39,000 minutes) is available per month in the Assembly Department; there are no other relevant constraints on production.

a. What is the contribution per unit for BikeRacs? For KayakRacs?

b. At the required monthly levels of production (5,000 BikeRacs and 2,000 KayakRacs), how many minutes are used in the Assembly Department?

c. Suppose there is unlimited demand for BikeRacs and KayakRacs at current prices. What production schedule (number of BikeRacs and number of KayakRacs) should On the Move adopt to maximize profit t while meeting its constraint to produce and sell at least 5,000 BikeRacs and 2,000 KayakRacs.

d. Suppose demand is limited to 2,500 units of KayakRacs.  What production schedule should On the Move adopt to maximize profit t while meeting its constraint on the minimum levels for the two products?

special orders maria rsquo s food service provides meals that nonprofit organization 690938

Special Orders

Maria’s Food Service provides meals that nonprofit organizations distribute to handicapped andelderly people. Here is her forecasted income statement for April, when she expects to produce and sell 3,000 meals:

 

Amount

Per Unit

Sales revenue

$18,000

$6.00

Costs of meals produced

13,500

4.50

Gross profit

$ 4,500

$1.50

Administrative costs

2,100

0.70

Operating profit

$ 2,400

$0.80

Fixed costs included in this income statement are $4,500 for meal production and $600 for administrative costs. Maria has received a special request from an organization sponsoring a picnic to raise funds for the Special Olympics. This organization is willing to pay $3.50 per meal for 300 meals on April 10. Maria has sufficient idle capacity to fill this special order. These meals will incur all of the variable costs of meals produced, but variable administrative costs and total fixed costs will not be affected.

Required

a. What impact would accepting this special order have on operating profit t?

b. Should Maria accept the order?

special orders carlsbad enterprises has a capacity to produce 400 000 computer cases 690939

Special Orders

Carlsbad Enterprises has a capacity to produce 400,000 computer cases per year. The company is currently producing and selling 320,000 cases per year at a selling price of $40 per case. The cost of producing and selling one case follows:

Variable manufacturing costs

$16

Fixed manufacturing costs

4

Variable selling and administrative costs

8

Fixed selling and administrative costs

2

Total costs

$30

The company has received a special order for 20,000 cases at a price of $25 per case. Because it does not have to pay a sales commission on the special order, the variable selling and administrative costs would be only $5 per case. The special order would have no effect on total fixed costs. The company has rejected the offer based on the following computations:

Selling price per case

$25

Variable manufacturing costs

16

Fixed manufacturing costs

4

Variable selling and administrative costs

5

Fixed selling and administrative costs

2

Net loss per case

$(2 )

Required

a. What is the impact on profit t for the year if Carlsbad accepts the special order? Show computations.

b. Do you agree with the decision to reject the special order? Explain.

pricing decisions assume that cold rock sells ice cream for 4 80 per gallon the cost 690940

Pricing Decisions

Assume that Cold Rock sells ice cream for $4.80 per gallon. The cost of each gallon follows:

Materials

$1.80

Labor

.60

Variable overhead

.30

Fixed overhead ($24,000 per month, 20,000 gallons per month)

1.20

Total costs per gallon

$3.90

     

One of Cold Rock’s regular customers asked the company to fill a special order of 400 gallons at a selling price of $3.60 per gallon for a fund raising picnic for a local charity. Cold Rock has capacity to fill it without affecting total fixed costs for the month. Cold Rock’s general manager was concerned about selling the ice cream below the cost of $3.90 per gallon and has asked for your advice.

Required

a. Prepare a schedule to show the impact on Cold Rock’s profits of providing 400 gallons of ice cream in addition to the regular production and sales of 20,000 gallons per month.

b. Based solely on the data given, what is the lowest price per gallon at which the ice cream in the special order could be sold without reducing Cold Rock’s profits?

c. What other factors might the general manager want to consider in setting a price for the special order?

pricing decision mother rsquo s bottlers inc is a small bottling company that bottle 690941

Pricing Decision

Mother’s Bottlers, Inc., is a small bottling company that bottles and sells cold teas for $5 per unit. The cost of each unit follows:

Materials

$1.50

Labor

1.00

Variable overhead

0.50

Fixed overhead ($20,000 per month, 20,000 units per month)

1.00

Total costs per unit

$4.00

One of Mother’s regular customers asked the company to fill a special order of 2,000 units at a selling price of $3.50 per unit. Mother’s can fill the order using existing capacity without affecting total fixed costs for the month. However, Mother’s manager was concerned about selling at a price below the $4.00 cost per unit and has asked for your advice.

Required

a. Prepare a schedule to show the impact of providing the special order of 2,000 units on Mother’s profits in addition to the regular production and sales of 20,000 units per month.

b. Based solely on the data given, what is the lowest price per unit at which the bottled teas could be sold for the special order without reducing Mother’s profits?

c. What other factors might Mother’s managers want to consider in setting a price for the special order?

cost allocation and regulated prices the city of imperial falls contracts with everg 690888

Cost Allocation and Regulated Prices

The City of Imperial Falls contracts with Evergreen Waste Collection to provide solid waste collection to households and businesses. Until recently, Evergreen had an exclusive franchise to provide this service in Imperial Falls, which meant that other waste collection firms could not operate legally in the city. The price per pound of waste collected was regulated at 20 percent above the average total cost of collection.

Cost data for the most recent year of operations for Evergreen are as follows:

Administrative cost

$ 400,000

Operating costs—trucks

1,280,000

Other collection costs

320,000

 

Data on customers for the most recent year are:

 

 

Households

Businesses

Number of customers

12,000

3,000

Waste collected (tons)

4,000

12,000

The City Council of Imperial Falls is considering allowing other private waste haulers to collect waste from businesses, but not from households. Service to businesses from other waste collection firms would not be subject to price regulation. Based on information from neighboring cities, the price that other private waste collection firms will charge is estimated to be $0.04 per pound (= $80 per ton).

Evergreen’s CEO has approached the city council with a proposal to change the way costs are allocated to households and businesses, which will result in different rates for households and businesses. She proposes that administrative costs and truck operating costs be allocated  based on the number of customers and the other collection costs be allocated based on pounds collected. The total costs allocated to households would then be divided by the estimated number of pounds collected from households to determine the cost of collection. The rate would then be 20 percent above the cost. The rate for businesses would be determined using the same calculation.

Required

a. Based on cost data from the most recent year, what is the price per pound charged by Evergreen for waste collection under the current system (the same rate for both types of customers)?

b. Based on cost and waste data from the most recent year, what would be the price per pound charged to households and to businesses by Evergreen for waste collection if the CEO’s proposal were accepted?

c. As a staff member to one of the council members, would you support the proposal to change the way costs are allocated? Explain.

reconstruct financial statements san ysidro company manufactures hiking equipment th 690889

Reconstruct Financial Statements

San Ysidro Company manufactures hiking equipment. The company’s administrative and manufacturing operations share the company’s one building. Eighty percent of the building is used for manufacturing and the remainder is used for administrative activities. Indirect labor is 8 percent of direct labor.

The cost accountant at San Ysidro has compiled the following information for the year ended December 31:

<> 

A

B

C

1

Administrative salaries

$ 192,000

 

2

Attorney fees to settle zoning dispute

22,960

 

3

Building depreciation (manufacturing portion only)

181,440

 

4

Cost of goods manufactured

2,776,760

 

5

Direct materials inventory, December 31

248,000

 

6

Direct materials purchased during the year

1,008,000

 

7

Direct materials used

1,069,880

 

8

Distribution costs

4,480

 

9

Finished goods inventory, January 1

224,000

 

10

Finished goods inventory, December 31

252,000

 

11

Insurance (on plant machinery)

53,200

 

12

Maintenance (on plant machinery)

33,880

 

13

Marketing costs

103,600

 

14

Other plant costs

82,160

 

15

Plant utilities

104,160

 

16

Sales revenue

4,550,000

 

17

Taxes on manufacturing property

38,800

 

18

Total (direct and indirect) labor

1,209,600

 

19

Work in process inventory, January 1

72,520

 

20

Work in process inventory, December 31

68,880

 

21

 

 

 

Required

Prepare a cost of goods manufactured and sold statement and an income statement.

analyze the impact of a decision on income statements you were appointed the manager 690890

Analyze the Impact of a Decision on Income Statements

You were appointed the manager of Drive Systems Division (DSD) at Tunes2Go, a manufacturer of portable music devices using the latest developments in hard drive technology, on December 15 last year. DSD manufactures the drive assembly, M 24, for the company’s most popular product.  Your bonus is determined as a percentage of your division’s operating profits before taxes.  One of your fist major investment decisions was to invest $3 million in automated testing equipment for the M 24. The equipment was installed and in operation on January 1 of this year.

This morning, J. Bradley Finch III, the assistant manager of the division (and, not coincidentally, the grandson of the company founder and son of the current CEO) told you about an offer by Pan Pacific Electronics. Pan Pacific wants to rent to DSD a new testing machine that could be installed on December 31 (only two weeks from now) for an annual rental charge of $690,000. The new equipment would enable you to increase your division’s annual revenue by 7 percent. This new, more efficient machine would also decrease fixed cash expenditures by 6 percent.

Without the new machine, operating revenues and costs for the year are estimated to be as shown below. Revenues and fixed and variable operating costs are all cash.

Sales revenues

$4,800,000

Variable operating costs

600,000

Fixed operating costs

2,250,000

Equipment depreciation

450,000

Other depreciation

375,000

If you rent the new testing equipment, DSD will have to write off the cost of the automated testing equipment this year because it has no salvage value. Equipment depreciation shown in the income statement is for this automated testing equipment. Equipment losses are included in the bonus and operating profit computation.

Because the new machine will be installed on a company holiday, there will be no effect on operations from the changeover. Ignore any possible tax effects. Assume that the data given in your expected income statement are the actual amounts for this year and next year if the current equipment is kept.

Required

a. Assume the new testing equipment is rented and installed on December 31. What will be the impact on this year’s divisional operating profit t?

b. Assume the new testing equipment is rented and installed on December 31. What will be the impact on next year’s divisional operating profit t?

c. Would you rent the new equipment? Why or why not?

finding unknowns bs amp t partners has developed a new hubcap with the model name sp 690892

Finding Unknowns

BS&T Partners has developed a new hubcap with the model name Spinnin’ Wheel. Production and sales started August 3. As of August 2, there were no direct materials in inventory. Data for the month of August include the following:

Direct labor cost per unita

$6.25

Direct labor hours worked, August

________

Direct labor wage rate per direct labor hour

$20.00

Direct materials cost per unita

$5.00

Direct materials cost per pound of direct material

$10.00

Direct materials inventory (cost), August 31

$3,500

Direct materials inventory (units), August 31

________

Finished goods inventory (cost), August 31

$10,800

Finished goods inventory (units), August 31

________

Manufacturing overhead cost per unita.

$15.75

Operating profit, August

$55,200

Production (units), August

________

Revenues, August

$414,000

Sales (units), August

________

Selling price per unit

________

Selling, general, and administrative costs per unitb

$12.00

a Unit cost based on units produced in August.

b Unit cost based on units sold in August.

Required

Complete the table.

the following information for jennifer rsquo s framing supply is given for march sal 690893

The following information for Jennifer’s Framing Supply is given for March:

Sales.

$360,000

Fixed manufacturing costs

35,000

Fixed marketing and administrative costs

25,000

Total fixed costs

60,000

Total variable costs

240,000

Unit price

90

Unit variable manufacturing cost

55

Unit variable marketing cost

5

Compute the following:

a. Monthly operating profit t when sales total $360,000 (as here).

b. Break even number in units.

c. Number of units sold that would produce an operating profit t of $120,000.

d. Sales dollars required to earn an operating profit t of $20,000.

e. Number of units sold in March.

f. Number of units sold that would produce an operating profit t of 20 percent of sales dollars.

suppose hdc rents spaces for both rvs and tent camping the price and cost characteri 690895

Suppose HDC rents spaces for both RVs and tent camping. The price and cost characteristics for each are as follows (one unit is a tent or RV space rented for one day):

Price per

Variable Cost

Units Rented

Unit

per Unit

per Year

Tent space

$ 6

$3

6,000

RV space

15

7

9,000

The fixed costs of HDC are $60,000 annually. Assuming the mix of tent and RV spaces is the same as the current mix, how many tent spaces and how many RV spaces must be rented annually for HDC to break even?

basic decision analysis using cvp ana rsquo s amusement center has collected the fol 690899

Basic Decision Analysis Using CVP

Ana’s Amusement Center has collected the following data for operations for the year:

Total revenues

$800,000

Total fixed costs

$218,750

Total variable costs

$450,000

Total tickets sold

50,000

Required

a. What is the average selling price for a ticket?

b. What is the average variable cost per ticket?

c. What is the average contribution margin per ticket?

d. What is the break even point?

e. An has decided that unless the operation can earn at least $43,750 in operating profits, she will close it down. What number of tickets must be sold for Ana’s Amusements to make a $43,750 operating profit t for the year on ticket sales?

cvp analysis mdash ethical issues mark ting desperately wants his proposed new produ 690901

CVP Analysis—Ethical Issues

Mark Ting desperately wants his proposed new product, DNA diamond, to be accepted by top management. DNA diamond is a piece of jewelry that contains the DNA of a boy or girl friend, spouse, or other loved one. Top management will not approve this product in view of its high break even point.

Mark knows that if he can reduce the fixed costs in his proposal, then the break even point will  be reduced to a level that top management finds acceptable. Working with a friend in the company’s finance department, Mark finds ways to credibly misstate the estimated fixed costs of producing DNA diamonds below those that any objective person would estimate. Mark knows that if the product is successful (and he is certain that it will be), then top management will not find out about the understatement of fixed costs. Mark believes that this product, once it is successful, will benefit the shareholders and employees of the company.

Required

Are Mark’s actions ethical? Explain.

basic decision analysis using cvp cambridge inc is considering the introduction of a 690902

Basic Decision Analysis Using CVP

Cambridge, Inc., is considering the introduction of a new calculator with the following price and cost characteristics:

Sales price

$ 18 each

Variable costs

10 each

Fixed costs

20,000 per month

 

Required

a. What number must Cambridge sell per month to break even?

b. What number must Cambridge sell to make an operating profit t of $16,000 for the month?

basic decision analysis using cvp cambridge inc is considering the introduction of a 690903

Basic Decision Analysis Using CVP

Cambridge, Inc., is considering the introduction of a new calculator with the following price and cost characteristics:

Sales price

$ 18 each

Variable costs

10 each

Fixed costs

20,000 per month

Assume that the projected number of units sold for the year is 7,000. Consider requirements ( b ), ( c ), and ( d ) independently of each other.

Required

a. What will the operating profit be?

b. What is the impact on operating profit t if the sales price decreases by 10 percent? Increases by  20 percent?

c. What is the impact on operating profit t if variable costs per unit decrease by 10 percent? Increase by 20 percent?

d. Suppose that fixed costs for the year are 10 percent lower than projected, and variable costs per unit are 10 percent higher than projected. What impact will these cost changes have on operating profit t for the year? Will profit t go up? Down? By how much?

basic decision analysis using cvp balance inc is considering the introduction of a n 690904

Basic Decision Analysis Using CVP

Balance, Inc., is considering the introduction of a new energy snack with the following price and cost characteristics:

Sales price

$ 1.00 per unit

Variable costs

0.20 per unit

Fixed costs

400,000 per month

Required

a. What number must Balance sell per month to break even?

b. What number must Balance sell per month to make an operating profit of $100,000?

analysis of cost structure foxx company rsquo s cost structure is dominated by varia 690906

Analysis of Cost Structure Foxx Company’s cost structure is dominated by variable costs with a contribution margin ratio of .25 and fixed costs of $100,000. Every dollar of sales contributes 25 cents toward fixed costs and profit t. The cost structure of a competitor, Beyonce, Inc., is dominated by fixed costs with a higher contribution margin ratio of .80 and fixed costs of $400,000. Every dollar of sales contributes 80 cents toward fixed costs and profit t. Both companies have sales of $600,000 per month.

Required

a. Compare the two companies’ cost structures using the format shown in Exhibit 3.5.

b. Suppose that both companies experience a 20 percent increase in sales volume. By how much would each company’s profits increase?

Exhibit 3.5.

 

Lo Lev Company

Hi Lev Company

 

(1,000,000 units)

Percentage

(1,000,000 units)

Percentage

Sales

$1,000,000

100

$1,000,000

100

Variable costs

750,000

75

250,000

25

Contribution margin

$ 250,000

25

$ 750,000

75

Fixed costs

50,000

5

550,000

55

Operating profit t

$200,000

20

$ 200,000

20

Break even point

200,000 units

733,334 units

Contribution margin per

unit $0.25

$0.75

 

using microsoft excel to perform cvp analysis cambridge inc is considering the intro 690908

Using Microsoft Excel to Perform CVP Analysis

Cambridge, Inc., is considering the introduction of a new calculator with the following price and cost characteristics:

Sales price

$ 18 each

Variable costs

10 each

Fixed costs

20,000 per month

Required

Using the Goal Seek function in Microsoft Excel,

a. What number must Cambridge sell to break even?

b. What number must Cambridge sell to make an operating profit t of $6,000 per month?

using microsoft excel to perform cvp analysis balance inc is considering the introdu 690909

Using Microsoft Excel to Perform CVP Analysis

Balance, Inc., is considering the introduction of a new energy snack with the following price and cost characteristics:

Sales price

$ 1.00 per unit

Variable costs

0.20 per unit

Fixed costs

400,000 per month

Required

Using the Goal Seek function in Microsoft Excel,

a. What number must Balance, Inc., sell to break even?

b. What number must Balance, Inc., sell to make an operating profit t of $8,000 per month?

cvp with income taxes crest industries sells a single model of satellite radio recei 690910

CVP with Income Taxes

Crest Industries sells a single model of satellite radio receivers for use in the home. The radios have the following price and cost characteristics:

Sales price

$ 80 per radio

Variable costs

$ 32 per radio

Fixed costs

$360,000 per month

Crest is subject to an income tax rate of 40 percent.

Required

a. How many receivers must Crest sell every month to break even?

b. How many receivers must Crest sell to earn a monthly operating profit t of $90,000 after taxes?

multiproduct cvp analysis rio coffee shoppe sells two coffee drinks a regular coffee 690911

Multiproduct CVP Analysis

Rio Coffee Shoppe sells two coffee drinks, a regular coffee and a latte. The two drinks have the following prices and cost characteristics:

 

Regular Coffee

Latte

Sales price (per cup)

$1.50

$2.50

Variable costs (per cup)

.70

1.30

The monthly fixed costs at Rio are $6,720. Based on experience, the manager at Rio knows that the store sells 60 percent regular coffee and 40 percent lattes.

Required

How many cups of regular coffee and lattes must Rio sell every month to break even?

multiproduct cvp analysis mission foods produces two flvors of tacos chicken and fis 690912

Multiproduct CVP Analysis

Mission Foods produces two flvors of tacos, chicken and fish, with the following characteristics:

 

Chicken

Fish

Selling price per taco

$3.00

$ 4.50

Variable cost per taco

$1.50

$ 2.25

Expected sales (tacos)

200,000

300,000

Required

The total filed costs for the company are $117,000.

a. What is the anticipated level of profits for the expected sales volumes?

b. Assuming that the product mix would be 40 percent chicken and 60 percent fish at the breakeven point, compute the break even volume.

c. If the product sales mix were to change to four chicken tacos for each fish taco, what would be the new break even volume?

cvp analysis and price changes argentina partners is concerned about the possible ef 690913

CVP Analysis and Price Changes

Argentina Partners is concerned about the possible effects of inflation on its operations. Presently, the company sells 60,000 units for $30 per unit. The variable production costs are $15, and fixed costs amount to $700,000. Production engineers have advised management that they expect unit labor costs to rise by 15 percent and unit materials costs to rise by 10 percent in the coming year. Of the $15 variable costs, 50 percent are from labor and 25 percent are from materials. Variable  overhead costs are expected to increase by 20 percent. Sales prices cannot increase more than 10 percent. It is also expected that fixed costs will rise by 5 percent as a result of increased taxes and other miscellaneous fixed charges.

The company wishes to maintain the same level of profit in real dollar terms. It is expected  that to accomplish this objective, profits must increase by 6 percent during the year.

Required

a. Compute the volume in units and the dollar sales level necessary to maintain the present profit t level, assuming that the maximum price increase is implemented.

b. Compute the volume of sales and the dollar sales level necessary to provide the 6 percent increase in profits, assuming that the maximum price increase is implemented.

c. If the volume of sales were to remain at 60,000 units, what price increase would be required to attain the 6 percent increase in profits?

cvp analysis and price changes scholes systems supplies a particular type of office 690914

CVP Analysis and Price Changes

Scholes Systems supplies a particular type of office chair to large retailers such as Target, Costco, and Office Max. Scholes is concerned about the possible effects of inflation on its operations. Presently, the company sells 80,000 units for $60 per unit. The variable production costs are $30, and fixed costs amount to $1,400,000. Production engineers have advised management that they expect unit labor costs to rise by 15 percent and unit materials costs to rise by 10 percent in the coming year. Of the $30 variable costs, 50 percent are from labor and 25 percent are from materials. Variable overhead costs are expected to increase by 20 percent. Sales prices cannot increase more than 10 percent. It is also expected that fixed costs will rise by 5 percent as a result of increased taxes and other miscellaneous fixed charges.

The company wishes to maintain the same level of profit t in real dollar terms. It is expected that to accomplish this objective, profits must increase by 6 percent during the year.

Required

a. Compute the volume in units and the dollar sales level necessary to maintain the present profit t level, assuming that the maximum price increase is implemented.

b. Compute the volume of sales and the dollar sales level necessary to provide the 6 percent  increase in profits, assuming that the maximum price increase is implemented.

c. If the volume of sales were to remain at 80,000 units, what price change would be required to attain the 6 percent increase in profits?

cvp analysis mdash missing data durant manufacturers has performed extensive studies 690915

CVP Analysis—Missing Data

Durant Manufacturers has performed extensive studies on its costs and production and estimates the following annual costs based on 150,000 units (produced and sold):

 

Total Annual

 

Costs

 

(150,000 units)

Direct material

$300,000

Direct labor

270,000

Manufacturing overhead

225,000

Selling, general, and administrative

150,000

Total

$945,000

Required

a. Compute Durant’s unit selling price that will yield a profit t of $300,000, given sales of 150,000 units.

b. Compute Durant’s dollar sales that will yield a projected 20 percent profit t on sales, assuming variable costs per unit are 60 percent of the selling price per unit and fixed costs are $420,000.

c.Of the $30 variable costs, 50 percent are from labor and 25 percent are from materials. Variable overhead costs are expected to increase by 20 percent. Sales prices cannot increase more than 10 percent.

basic concepts the following data apply to the provision of psychological testing se 690858

Basic Concepts

The following data apply to the provision of psychological testing services.

Sales price per unit (1 unit _ 1 test plus feedback to client)

$ 300

Fixed costs (per month):

 

Selling and administration

20,000

Production overhead (e.g., rent of testing facilities)

30,000

Variable costs (per test):

 

Labor for oversight and feedback

120

Outsourced test analysis

20

Materials used in testing

5

Production overhead

10

Selling and administration (e.g., scheduling and billing)

15

Number of tests per month

1,000 tests

Required

Give the amount for each of the following (one unit _ one test):

a. Variable production cost per unit.

b. Variable cost per unit.

c. Full cost per unit.

d. Full absorption cost per unit.

e. Prime cost per unit.

f. Conversion cost per unit.

g. Contribution margin per unit.

h. Gross margin per unit.

i. Suppose the number of units decreases to 800 tests per month, which is within the relevant range. Which parts of (a) through ( h ) will change? For each amount that will change, give the  new amount for a volume of 800 tests.

cost allocation mdash ethical issues in one of its divisions an aircraft components 690860

Cost Allocation—Ethical Issues

In one of its divisions, an aircraft components manufacturer produces experimental navigational  equipment for spacecraft and for private transportation companies. Although the products are essentially identical, they carry different product numbers. The XNS 12 model is sold to a government agency on a cost reimbursed basis. In other words, the price charged to the government is equal to the computed cost plus a fixed fee. The JEF 3 model is sold to the private transportation companies on a competitive basis. The product development cost, common to both models, must be allocated to the two products in order to determine the cost for setting the price of the XNS 12.

Required

a. How would you recommend the product development cost be allocated between the two products?

b. What incentives do managers have to allocate product development costs? Why?

prepare statements for a service company chuck rsquo s brokerage service cbs is a di 690863

Prepare Statements for a Service Company

Chuck’s Brokerage Service (CBS) is a discount financial services firm offering clients investment advice, trading services, and a variety of mutual funds for investment. Chuck has collected the following  information for October:

<> 

A

B

C

1

Advertising and marketing

$180,000

 

2

Brokerage commissions (revenues)

6,000,000

 

3

Building rent and utilities

350,000

 

4

Fees from clients for investment advice

3,000,000

 

5

Fees paid to execute trades

4,000,000

 

6

Labor cost for advice

1,600,000

 

7

Managers’ salaries

600,000

 

8

Sales commissions to brokers

500,000

 

9

Training programs for brokers

850,000

 

10

 

 

 

Required

Prepare an income statement for October for CBS.

prepare statements for a manufacturing company the following balances are from the a 690866

Prepare Statements for a Manufacturing Company

The following balances are from the accounts of Secol Machining Company:

 

January 1 (Beginning)

December 31 (Ending)

Direct materials inventory

$48,000

$59,000

Work in process inventory

58,000

56,000

Finished goods inventory

43,800

45,000

Direct materials purchased during the year amount to $299,000, and the cost of goods sold for the year was $1,086,200.

Required

Reconstruct a cost of goods sold statement and fill in the following missing data:

a. Cost of direct materials used during the year.

b. Cost of goods manufactured during the year.

c. Total manufacturing costs incurred during the year.

prepare statements for a merchandising company the cost accountant for sun amp surf 690867

Prepare Statements for a Merchandising Company

The cost accountant for Sun & Surf Apparel Shop has compiled the following information for last year’s operations:

Administrative costs

$ 146,400

Merchandise inventory, January 1

26,000

Merchandise inventory, December 31

25,000

Merchandise purchases

1,220,000

Sales commissions

85,700

Sales revenue

1,868,000

Store rent

28,800

Store utilities

5,460

Transportation in costs

9,240

Required

Prepare an income statement with a supporting cost of goods sold statement.

prepare statements for a merchandising company powell street electronics has provide 690868

Prepare Statements for a Merchandising Company

Powell Street Electronics has provided the following information for February:

Sales revenue

$ 807,000

Store rent

43,600

Store utilities

27,100

Administrative costs

57,750

Sales commissions

128,300

Merchandise purchases

545,500

Transportation in costs

21,200

Merchandise inventory, February 1

37,100

Merchandise inventory, February 28

41,800

Required

Prepare an income statement for February with a supporting cost of goods sold statement.

cost behavior and forecasting ramirez company manufactured 60 000 units of product l 690869

Cost Behavior and Forecasting

Ramirez Company manufactured 60,000 units of product last year and identified the following costs associated with the manufacturing activity:

Variable costs:

 

Direct materials used

$ 1,020,000

Direct labor

2,240,000

Indirect materials and supplies

240,000

Power to run plant equipment.

280,000

Fixed costs:

 

Supervisory salaries

930,000

Plant utilities (other than power to run plant equipment)

220,000

Depreciation on plant and equipment (straight line, time basis)

135,000

Property taxes on building

195,000

Required

Unit variable costs and total fixed costs are expected to remain unchanged next year. Calculate the unit cost and the total cost of 51,000 units are produced next year.

components of full costs gibson corporation has compiled the following information f 690871

Components of Full Costs

Gibson Corporation has compiled the following information from the accounting system for the one product it sells:

Sales price

$300 per unit

Fixed costs (for the month)

 

Marketing and administrative

$36,000

Manufacturing overhead

$54,000

Variable costs (per unit)

 

Marketing and administrative

$6

Direct materials

$90

Manufacturing overhead.

$20

Direct labor

$55

Units produced and sold (for the month)

1,800

Required

Compute:

a. Product costs per unit.

b. Period costs for the period.

gross margin and contribution margin income statements larker manufacturing rsquo s 690873

Gross Margin and Contribution Margin Income Statements

Larker Manufacturing’s cost accountant has provided you with the following information for January operations:

Direct materials

$105 per unit

Fixed manufacturing overhead costs

$675,000

Sales price

$395 per unit

Variable manufacturing overhead

$60 per unit

Direct labor

$120 per unit

Fixed marketing and administrative costs

$585,000

Units produced and sold

30,000

Variable marketing and administrative costs

$24 per unit

Required

Prepare:

a. A gross margin income statement.

b. A contribution margin income statement.

value income statement gene rsquo s diner has the following information for october 690876

Value Income Statement

Gene’s Diner has the following information for October, when several new employees were added to the wait staff:

Sales revenue

$180,000

Cost of food serveda

60,000

Employee wages and salariesb

45,000

Manager salariesc

18,000

Building costs (rent, utilities, etc.)d

27,000

a5 percent of this cost was for food that was not used by the expiration date and 10 percent  was for food that was incorrectly prepared because of errors in orders taken.

b15 percent of this cost was for time spent by cooks to re prepare orders that were incorrectly prepared because of errors in orders taken.

c20 percent of this cost was time taken to address customer complaints about incorrect orders.

d80 percent of the building was used.

Required

a. Using the traditional income statement format, prepare a value income statement.

b. What value would there be to Gene from preparing the same information in November?

value income statement paul rsquo s limo service has the following information for j 690877

Value Income Statement

Paul’s Limo Service has the following information for June:

1

A

B

C

D

 

 

 

 

2

Sales revenue

$ 50,000

 

 

3

Variable costs of operations, excluding labor costsa

15,000

 

 

4

Employee wages and salariesb

20,000

 

 

5

Manager salariesc

4,000

 

 

6

Fixed cost of automobilesd

5,000

 

 

7

Building costs (rent, utilities, etc.)e

2,500

 

 

8

 

 

 

 

9

a5 percent of this cost was wasted due to poor directions given to limo drivers.

 

 

 

10

b5 percent of this cost was for time spent by limo drivers because of poor directions.

 

 

 

11

c10 percent of this cost was time taken to address customer complaints.

 

 

 

12

dThe limos have 40 percent unused capacity.

 

 

 

13

eThe building has 10 percent unused capacity.

 

 

 

14

 

 

 

 

Required

a. Using the traditional income statement format, prepare a value income statement.

b. What value would there be to Paul from preparing the same information in July?

cost concepts the following information comes from the accounting records for santa 690878

Cost Concepts

The following information comes from the accounting records for Santa Cruz, Inc., for March:

Direct material inventory, March 1

$6,000

Direct material inventory, March 31

5,000

Work in process inventory, March 1

3,000

Work in process inventory, March 31

2,000

Finished goods inventory, March 1

18,000

Finished goods inventory, March 31

24,000

Direct materials purchased during March

80,000

Direct labor costs, March

64,000

Manufacturing overhead, March

84,000

Required

Compute, for the month of March:

a. Total prime costs.

b. Total conversion costs.

c. Total manufacturing costs.

d. Cost of goods manufactured.

e. Cost of goods sold.

cost concepts princeton fabrication inc produced and sold 1 200 units of the company 690880

Cost Concepts

Princeton Fabrication, Inc., produced and sold 1,200 units of the company’s only product in March. You have collected the following information from the accounting records:

Sales price (per unit)

$ 896

Manufacturing costs:

 

Fixed overhead (for the month)

100,800

Direct labor (per unit)

70

Direct materials (per unit)

224

Variable overhead (per unit)

140

Marketing and administrative costs:

 

Fixed costs (for the month)

134,400

Variable costs (per unit)

28

Required

a. Compute:

1. Variable manufacturing cost per unit.

2. Full cost per unit.

3. Variable cost per unit.

4. Full absorption cost per unit.

5. Prime cost per unit.

6. Conversion cost per unit.

7. Profit margin per unit.

8. Contribution margin per unit.

9. Gross margin per unit.

b. If the number of units decreases from 1,200 to 800, which is within the relevant range, will the fixed manufacturing cost per unit increase, decrease, or remain the same? Explain.

prepare statements for a manufacturing company pioneer parts a manufacturer of windo 690881

Prepare Statements for a Manufacturing Company

Pioneer Parts, a manufacturer of windows for commercial buildings, reports the following account information for last year (all costs are in thousands of dollars):

Information on January 1 (Beginning):

 

Direct materials inventory

$ 18

Work in process inventory

24

Finished goods inventory

328

Information for the year:

 

Administrative costs

$ 720

Direct labor

2,120

Direct materials purchases

1,640

Factory and machine depreciation

2,320

Factory supervision

420

Factory utilities

180

Indirect factory labor

560

Indirect materials and supplies

140

Marketing costs

300

Property taxes on factory

56

Sales revenue

9,080

 

Information on January 31 (Ending):

 

Direct materials inventory

$ 16

Work in process inventory

28

Finished goods inventory

294

Required

Prepare an income statement with a supporting cost of goods sold statement.

prepare statements for a manufacturing company the administrative offices and manufa 690882

Prepare Statements for a Manufacturing Company

The administrative offices and manufacturing plant of Oakdale Tool & Die share the same building. The following information (in $000s) appears in the accounting records for last year:

Administrative costs

$ 4,800

Building and machine depreciation

 

(75% of this amount is for factory)

2,700

Building utilities (90% of this amount is for factory)

3,750

Direct labor

2,520

Direct materials inventory, December 31

42

Direct materials inventory, January 1

36

Direct materials purchases

10,950

Factory supervision

1,470

Finished goods inventory, December 31

195

Finished goods inventory, January 1

162

Indirect factory labor

2,736

Indirect materials and supplies

2,055

Marketing costs

2,613

Property taxes on building

 

(80% of this amount is for factory)

2,520

Sales revenue

38,910

Work in process inventory, December 31

87

Work in process inventory, January 1

96

Required

Prepare an income statement with a supporting cost of goods sold statement.

cost allocation with cost flow diagram coastal computer operates two retail outlets 690883

Cost Allocation with Cost Flow Diagram

Coastal Computer operates two retail outlets in Oakview, one on Main Street and the other in Lakeland Mall. The stores share the use of a central accounting department. The cost of the accounting department  for last year was $180,000. Following are the operating results for the two stores for the year.

 

Main Street

Lakeland Mall

Revenues

$1,000,000

$2,000,000

Number of computers sold

2,000

1,600

Required

a. Allocate the cost of the central accounting department to the two stores based on:

(1) Number of computers sold.

(2) Store revenue.

b. Draw a cost flow diagram to illustrate your answer to requirement (a), part (2).

cost allocation with cost flow diagram wayne casting inc produces a product made fro 690884

Cost Allocation with Cost Flow Diagram

Wayne Casting, Inc., produces a product made from a metal alloy. Wayne buys the alloy from two different suppliers, Chillicothe Metals and Ames Supply, in approximately equal amounts because of supply constraints at both vendors. The material from Chillicothe is less expensive to buy, but more difficult to use, resulting in greater waste. The metal alloy is highly toxic and any waste  requires costly handling to avoid environmental accidents. Last year the cost of handling the waste totaled $300,000. Additional data from last year’s operations are shown below.

 

Chillicothe

Ames

 

Metals

Supply

Amount of material purchased (tons)

130

120

Amount of waste (tons)

12.8

2.2

Cost of purchases

$624,000

$876,000

Required

a. Allocate the cost of the waste handling to the two suppliers based on:

1. Amount of material purchased.

2. Amount of waste.

3. Cost of material purchased.

b. Draw a cost flow diagram to illustrate your answer to item (a), part (1).

cost allocation with cost flow diagram the library at pacific business school pbs se 690885

Cost Allocation with Cost Flow Diagram

The library at Pacific Business School (PBS) serves both undergraduate and graduate programs. The dean of PBS is interested in evaluating the profitability of the degree programs and has asked the head of the library, Rex Gilmore, to allocate the annual library cost of $4,035,000 to the two programs.

Rex believes that two cost drivers explain most of the costs: number of students and credit hours. Using information from a previous analysis, he split the annual library budget as follows:

<> 

A

B

C

D

1

Costs driven by number of students

 

 

 

2

Library management

$ 950,000

 

 

3

Acquisitions

1,300,000

 

 

4

 

$ 2,250,000

 

 

5

 

 

 

 

6

Costs driven by number of credit hours

 

 

 

7

Computer support

$ 135,000

 

 

8

Building maintenance

496,000

 

 

9

Library staff

788,000

 

 

10

Utilities and supplies

366,000

 

 

11

 

$ 1,785,000

 

 

12

Total library costs

$ 4,035,000

 

 

13

 

 

 

 

14

 

 

 

 

15

Data on students and credit hours

Undergraduate

Graduate

 

16

Number of students

900

600

 

17

Number of credit hours

13,500

16,500

 

18

 

 

 

 

Required

a. Allocate the cost of the library to the two programs (undergraduate and graduate).

b. Draw a cost flow diagram to illustrate your answer to requirement (a).

find the unknown information after a computer failure you are trying to reconstruct 690886

Find the Unknown Information

After a computer failure, you are trying to reconstruct some financial results for the year just ended. While you know that backups are available, it will take too long to get the information you want. You have been able to collect the following information:

Direct materials inventory, January 1 (Beginning)

$ 16,000

Direct materials inventory, December 31 (Ending)

12,000

Work in process inventory, January 1 (Beginning)

21,200

Work in process inventory, December 31 (Ending)

10,000

Finished goods inventory, December 31 (Ending)

14,080

Purchases of direct materials

38,400

Cost of goods manufactured during this year

88,800

Total manufacturing costs

77,600

Cost of goods sold

87,040

Gross margin

52,480

Direct labor

12,160

Manufacturing overhead

23,040

Average selling price per unit

8

Required

Find the following:

a. Finished goods inventory, January 1.

b. Direct materials used for the year.

c. Gross margin percentage (as a percentage of sales).

find the unknown information just before class starts you realize that you have mist 690887

Find the Unknown Information

Just before class starts, you realize that you have mistakenly recycled the second page of your cost accounting homework assignment. Fortunately, you still have the first page of printout from your spreadsheet (shown below) and you remember that you were able to determine the items on the recycled page from this information.

<> 

A

B

C

1

Direct materials inventory, January 1

$ 2,520

 

2

Direct materials inventory, December 31

2,088

 

3

Work in process inventory, January 1

5,440

 

4

Work in process inventory, December 31

6,110

 

5

Finished goods inventory, January 1

22,320

 

6

Finished goods inventory, December 31

25,520

 

7

Cost of goods manufactured during this year

598,400

 

8

Total manufacturing costs

612,320

 

9

Direct labor

270,400

 

10

Manufacturing overhead

225,000

 

11

Average selling price per unit

18

 

12

Gross margin percentage (as a percentage of sales)

38%

 

13

 

 

 

Required

Find the following:

a. Cost of goods sold.

b. Direct materials used.

c. Purchases of direct materials.

d. Sales revenue

non mcq s and answers are needed as soon as they re solved 10 questions 5 chapters 2 689116

Non MCQ’s and answers are needed as soon as they’re solved. 10 questions 5 chapters, 2 questions from each chapter. the file attached has the exact chapters that will be covered on the test from the book (Intermediate Accounting 14th ed D. Kieso,).

Document Preview:

Exam III Review ACC 3023 Spring, 2011 10 Problems (No Multiple Choice) PLEASE! Show your work! 1. Chapter 10. Exchange of assets: Computation and write journal entries Practice problems: Brief Exercises: 10 8 to 10 12 BRIEF EXERCISE 10 8 Equipment ?3,300???Accumulated Depreciation—Trucks ?18,000??? Trucks ??20,000?? Cash ??500?? Gain on Disposal of Trucks ??800?? BRIEF EXERCISE 10 9 Equipment ($3,300 – $800) ?2,500???Accumulated Depreciation—Trucks ?18,000??? Trucks ??20,000?? Cash ??500?? BRIEF EXERCISE 10 10 Equipment ?5,000???Accumulated Depreciation—Machinery ?3,000???Loss on Disposal of Machinery ?4,000??? Machinery ??9,000?? Cash ??3,000?? BRIEF EXERCISE 10 11 Trucks (new) ?37,000???Accumulated Depreciation—Trucks ?27,000???Loss on Disposal of Trucks ?2,000??? Trucks (used) ??30,000?? Cash ??36,000?? BRIEF EXERCISE 10 12 Trucks (new) ?35,000???Accumulated Depreciation—Trucks ?17,000???Loss on Disposal of Trucks ?1,000??? Trucks (used) ??20,000?? Cash ??33,000?? Exercises: 10 17 to 10 20 EXERCISE 10 17 (10–15 minutes) Alatorre Corporation????Machinery ($320 + $85) ?405???Accumulated Depreciation—Machinery ?140???Loss on Disposal of Machinery ?65*??? Machinery ??290?? Cash ??320?? ?*Computation of loss:???? Book value of old machine ($290 – $140)?$150??? Less: Fair value of old machine? 85??? Loss on exchange?$ 65?? Mills Business Machine Company????Cash ?320???Inventory ?85???Cost of Goods Sold ?270??? Sales Revenue ??405?? Inventory ??270?? EXERCISE 10 18 (20–25 minutes) (a)?Exchange has commercial substance:?????Depreciation Expense ?800???? Accumulated Depreciation—    Equipment ?? 800??? ($12,700 – $700 = $12,000;????? $12,000 ÷ 5 = $2,400;????? $2,400 X 4/12 = $800)??????????Equipment ?15,200**????Accumulated Depreciation—Equipment ?8,000???? Gain on Disposal of Equipment ??500*??This document was truncated here because it was created in the Evaluation Mode.

Attachments:

alex shore practiced accounting with a partnership for five years recently he opened 689121

Alex Shore practiced accounting with a partnership for five years. Recently he opened his own accounting firm, which he operates as a proprietorship. The name of the new entity is Alex Shore, CPA. Shore experienced the following events during the organizing phase of the new business and its first month of operations. Some of the events were personal and did not affect the business.

Feb

4

Shore received $27,000 cash from former accounting partners.*

5

Deposited $50,000 in a new business bank account titled Alex Shore,CPA. The business gave capital to Shore.

6

Paid $100 cash for letterhead stationery for the new office.

7

Purchased office furniture for the office. The business will pay the account payable, $9,700, within three months.

10

Shore sold personal investment in Amazing.com stock, which he had owned for several years, receiving $50,000 cash.*

11

Shore deposited the $50,000 cash from sale of the Amazing.com stock in his personal bank account.*

12

A representative of a large company telephoned Shore and told him of the company”s intention to transfer its accounting business to Shore.

18

Finished tax hearings on behalf of a client and submitted a bill for accounting services, $17,000. Shore expected to collect from this client within two weeks.

25

Paid office rent, $1,500.

28

Shore withdrew cash of $1,000.

Requirements

1. Analyze the effects of the events on the accounting equation of the proprietorship of Alex Shore, CPA. Use a format similar to Exhibit 1 6.

2. As of February 28, compute Alex Shore”s

a. total assets.

b. total liabilities.

c. total owner”s equity.

d. net income or net loss for February.

pea corporation purchased 75 percent of the outstanding voting stock of sen corporat 689207

Pea Corporation purchased 75 percent of the outstanding voting stock of Sen Corporation for $2,400,000 on January 1, 2011. Sen’s stockholders’ equity on this date consisted of the following (in thousands):

Capital stock, $10 par……………………$1,000

Additional paid in capital……………….. 600

Retained earnings December 31, 2010….. 800

Total stockholders’ equity……………….$2,400

The excess fair value of the net assets acquired was assigned 10 percent to undervalued inventory (sold in 2011), 40 percent to undervalued plant assets with a remaining useful life of eight years, and 50 percent to goodwill. Comparative trial balances of Pea Corporation and Sen Corporation at December 31, 2015, are as follows:



REQUIRED: Determine the amounts that would appear in the consolidated financial statements of Pea Corporation and Subsidiary for each of the following items:

1. Goodwill at December 31, 2015

2. Noncontrolling interest share for 2015

3. Consolidated retained earnings at December 31, 2014

4. Consolidated retained earnings at December 31, 2015

5. Consolidated net income for 2015

6. Noncontrolling interest at December 31, 2014

7. Noncontrolling interest at December 31,2015 View less »

refer to the in action discussion of options backdating if stock options and other f 690829

Refer to the In Action discussion of options backdating. If stock options and other forms of performance based compensation result in some managers engaging in unethical or illegal behavior, why do firms still use them?

Options Backdating at Apple

Stock options are a popular compensation tool used to motivate senior executives. Recently, executives at several companies have been accused of setting an earlier date for an option grant than the day the option actually is awarded. (If the stock price had been rising, this would increase the value of the option to the executive.) Such a decision requires the firm to recognize as cost of compensation the difference between the stock price on the date of the grant and the stock price on the earlier date indicated. Failure to do so is potentially fraudulent. One company accused of backdating is Apple, Inc. Two senior executives, the general counsel and the chief financial officer, settled charges with the Securities and Exchange Commission (SEC) over the matter. After an internal investigation, Apple’s Board of Directors, “admitted to frequent backdating but exonerated [CEO Steven P.] Jobs— in part because Jobs ‘did not appreciate the accounting implications’ of backdating.” (Because the backdating took place in 2001, the CEO was not required to attest to the financial statements, indicating his knowledge of the accounting implications of transactions.) Part IV.2 of the Code of Ethics of the Institute of Management Accounting (see Appendix) requires members to: Disclose all relevant information that could reasonably be expected to influence an intended user’s understanding of the reports, analyses, or recommendations. The former CFO claims that he “warned Jobs at the time that Apple would likely need to take an accounting charge if it issued options on any day other than January 2.”

cost data for managerial purposes beige computers operates retail stores in both dow 690833

Cost Data for Managerial Purposes

Beige Computers operates retail stores in both downtown (City) and suburban (Mall) locations. The company has two responsibility centers: the City Division, which contains stores in downtown locations, and the Mall Division, which contains stores in suburban locations. Beige’s CEO is concerned about the profitability of the City Division, which has been operating at a loss for the last several years. The most recent income statement follows. The CEO has asked for your advice on shutting down the City Division’s operations. If the City Division is eliminated, corporate administration is not expected to change, nor are any other changes expected in the operations or costs of the Mall Division.

BEIGE COMPUTERS, CITY DIVISION
Divisional Income Statement
For the Year Ending January 31

Sales revenue                

$ 12,900,000

Costs

 

Advertising—City Division     

525,000

Cost of goods sold          

6,450,000

Divisional administrative salaries

870,000

Selling costs (sales commissions)

1,730,000

Rent                      

2,215,000

Share of corporate administration

1,425,000

Total costs                   

$13,215,000

Net loss before income tax benefit

$ (315,000)

Tax benefit at 40% rate         

126,200

Net loss                     

$ (189,000 )

Required

What revenues and costs are probably differential for the decision to discontinue City Division’s operations? What will be the effect on Beige’s profits if the division is eliminated?

cost data for managerial purposes state university business school subs offers sever 690834

Cost Data for Managerial Purposes

State University Business School (SUBS) offers several degrees, including Bachelor of Business Administration (BBA). The new dean believes in using cost accounting information to make decisions and is reviewing a staff developed income statement broken down by the degree offered. The dean is considering closing down the BBA program because the analysis, which follows, shows a loss. Tuition increases are not possible. The dean has asked for your advice. If the BBA degree program is dropped, school administration costs are not expected to change, but direct costs of the program, such as operating costs, building maintenance, and classroom costs, would be saved. There will be no other changes in the operations or costs of other programs.

STATE UNIVERSITY BUSINESS SCHOOL, BBA DEGREE
Degree Income Statement
For the Academic Year Ending June 30

Revenue                      

$400,000

Costs

 

Advertising—BBA program      

15,000

Faculty salaries               

204,000

Degree operating costs (part time staff)

26,000

Building maintenance          

37,000

Classroom costs (building depreciation)

85,000

Allocated school administration costs

43,000

Total costs                     

$410,000

Net loss                       

$ (10,000 )

Required

What revenues and costs are probably differential for the decision to drop the BBA program? What will be the net effect on the SUBS contribution (profit) if the BBA program is dropped?

cost data for managerial purposes mdash budgeting refer to exhibit 1 5 which shows b 690835

Cost Data for Managerial Purposes—Budgeting

Refer to Exhibit 1.5, which shows budgeted versus actual costs. Assume that Carmen’s Cookies is preparing a budget for the month ending June 30. Management prepares the budget by starting with the actual results for April 30 that appear in Exhibit 1.5. Next, management considers what the differences in costs will be between April and June.

Management expects the number of cookies sold to be 15 percent greater in June than in April, and it expects all food costs (e.g., flour, eggs) to be 15 percent higher in June than in April. Management expects “other” labor costs to be 20 percent higher in June than in April, partly because more labor will be required in June and partly because employees will get a pay raise. The manager will get a pay raise that will increase the salary from $3,000 in April to $3,750 in June. Rent and utilities are not expected to change.

 width=

Exhibit 1.5 Budget versus Actual Data

 

Required

Prepare a budget for Carmen’s Cookies for June.

ethics and channel stuffing continental condiments is a large food products firm in 690836

Ethics and Channel Stuffing

Continental Condiments is a large food products firm in Pennsylvania. Its sales staff has a strong incentive plan tied to meeting quarterly budgets. On June 25, Maria Tuzzi, a divisional controller, learns that some of the sales staff asked customers to take delivery of sizable quantities of products before June 30. The customers were told they could return the products after July 1 if they determined the items were not needed. (This is referred to as “channel Stuffing.”) The sales staff also offered to reimburse the customers for any storage costs incurred.

Required

a. From the viewpoint of the IMA’s “Statement of Ethical Professional Practice,” what are Maria’s responsibilities?

b. What steps should she take to resolve this problem?

cost data for managerial purposes graphic components gc has offered to supply the fe 690838

Cost Data for Managerial Purposes

Graphic Components (GC) has offered to supply the Federal Aviation Agency (FAA) with computer monitors at “cost plus 20 percent.” GC operates a manufacturing plant that can produce 22,000 monitors per year, but it normally produces 20,000. The costs to produce 20,000 monitors follow:

 

Total Cost

Cost per Case

Production costs:

 

 

Materials

$ 1,000,000

$ 50

Labor

2,000,000

100

Supplies and other costs that will vary with production

600,000

30

Indirect cost that will not vary with production

600,000

30

Variable marketing costs

400,000

20

Administrative costs (all fixed)

1,200,000

60

Totals

$ 5,800,000

$290

Based on these data, company management expects to receive $348 (= $290 × 120 percent) per monitor for those sold on this contract. After completing 500 monitors, the company sent a bill (invoice) to the government for $174,000 (= 500 monitors × $348 per monitor). The president of the company received a call from an FAA representative, who stated that the per monitor cost should be

Materials

$ 50

Labor

100

Supplies and other costs that will vary with production

30 $180

Therefore, the price per monitor should be $216 (= $180 × 120 percent). The FAA ignored marketing costs because the contract bypassed the usual selling channels.

Required

What price would you recommend? Why?

cost data for managerial purposes ringer company makes a variety of products it is o 690839

Cost Data for Managerial Purposes

Ringer Company makes a variety of products. It is organized in two divisions, East and West. West Division normally sells to outside customers but, on occasion, also sells to the East Division. When it does, corporate policy states that the price must be cost plus 15 percent to ensure a “fair” return to the selling division. West received an order from East Division for 600 units. West’s planned output for the year had been 2,400 units before East’s order. West’s capacity is 3,000 units per year. The costs for producing those 2,400 units follow.

 

Total

Per Unit

Materials

$ 240,000

$ 100

Direct labor

115,200

48

Other costs varying with output

76,800

32

Fixed costs

1,008,000

420

Total costs

$1,440,000

$ 600

Based on these data, West’s controller calculated that the unit price for East’s order should be $690 (= $600 × 115 percent). After producing and shipping the 600 units, West sent an invoice for $414,000. Shortly thereafter, West received a note from the buyer at East stating that this invoice was not in accordance with company policy. The unit cost should have been

Materials

$100

Direct labor

48

Other costs varying with outpput

32

Total

 $180

The price per unit would be $207 (= $180 × 115 percent).

Required

If the corporation asked you to review this intercompany policy, what policy would you recommend? Why? ( Note: You need not limit yourself to the East or West Division’s calculation.)

cost data for managerial purposes pete rsquo s taxi amp limo provides transportation 690840

Cost Data for Managerial Purposes

Pete’s Taxi & Limo provides transportation services in and around Centerville. Its profits have been declining, and management is planning to add a package delivery service that is expected to increase revenue by $400,000 per year. The total cost to lease the necessary package delivery vehicles from the local dealer is $30,000 per year. The present manager will continue to supervise all services at no increase in salary. Due to expansion, however, the labor costs and utilities would increase by 50 percent. Rent and other costs will increase by 20 percent.

<> 

A

B

C

1

PETE’S TAXI & LIMO

 

 

2

Annual Income Statement before Expansion

 

 

3

 

 

 

4

Sales revenue

$ 1,216,000

 

5

Costs

 

 

6

Vehicle leases

480,000

 

7

Labor

384,000

 

8

Utilities

64,000

 

9

Rent

128,000

 

10

Other costs

64,000

 

11

Manager’s salary

192,000

 

12

Total costs

$ 1,312,000

 

13

Operating profit (loss)

$ (96,000)

 

14

 

 

 

Required

a. Prepare a report of the differential costs and revenues if the delivery service is added.

b. Should management start the delivery service?

cost data for managerial purposes valley lawn amp tree inc provides landscaping serv 690841

Cost Data for Managerial Purposes

Valley Lawn & Tree, Inc., provides landscaping services in Eastmont. Renee Moffo, the owner, is concerned about the recent losses the company has incurred and is considering dropping its lawn services, which she feels are marginal to the company’s business. She estimates that doing so will result in lost revenues of $50,000 per year (including the lost tree business from customers who use the company for both services). The present manager will continue to supervise the tree services with no reduction in salary. Without the lawn business, Renee estimates that the company will save 15 percent of the equipment leases, labor, and other costs. She also expects to save 20 percent on rent and utilities.

<> 

A

B

C

1

VALLEY LAWN & TREE, INC.

 

 

2

Annual Income Statement

 

 

3

(Before Dropping Lawn Services)

 

 

4

 

 

 

5

Sales revenue

$ 304,000

 

6

Costs

 

 

7

Equipment leases

$ 120,000

 

8

Labor

96,000

 

9

Utilities

16,000

 

10

Rent

32,000

 

11

Other costs

16,000

 

12

Manager’s salary

40,000

 

13

Total costs

$ 320,000

 

14

Operating profit (loss)

$(16,000)

 

15

 

 

 

Required

a. Prepare a report of the differential costs and revenues if the lawn service is discontinued.

b. Should Renee discontinue the lawn service?

cost data for managerial purposes b you is a consulting firm that works with manager 690842

Cost Data for Managerial Purposes

B You is a consulting firm that works with managers to improve their interpersonal skills. Recently, a representative of a high tech research firm approached B You’s owner with an offer to contract for one year with B You to improve the interpersonal skills of a newly hired manager. B You reported the following costs and revenues during the past year.

<> 

A

 width=

B

C

1

B YOU

 

 

2

Annual Income Statement

 

 

3

 

 

 

4

Sales revenue

$ 360,000

 

5

Costs

 

 

6

Labor

171,000

 

7

Equipment lease

25,200

 

8

Rent

21,600

 

9

Supplies

16,200

 

10

Officers’ salaries

105,000

 

11

Other costs

11,400

 

12

Total costs

$350,400

 

13

Operating profit (loss)

$9,600

 

14

 

 

 

If B You decides to take the contract to help the manager, it will hire a full time consultant at $85,000. Equipment lease will increase by 5 percent. Supplies will increase by an estimated 10 percent and other costs by 15 percent. The existing building has space for the new consultant. No new offices will be necessary for this work.

Required

a. What are the differential costs that would be incurred as a result of taking the contract?

b. If the contract will pay $90,000, should B You accept it?

c. What considerations, other than costs, are necessary before making this decision?

cost data for managerial purposes mdash budgeting refer to exhibit 1 5 assume that c 690844

Cost Data for Managerial Purposes—Budgeting

Refer to Exhibit 1.5. Assume that Carmen’s Cookies is preparing a budget for the month ending September 30. Management prepares the budget by starting with the actual results for April that appear in Exhibit 1.5. Then, management considers what the differences in costs will be between April and September.

Management expects cookie sales to be 20 percent greater in September than in April, and it expects all food costs (e.g., flour, eggs) to be 20 percent higher in September than in April because of the increase in cookie sales. Management expects “other” labor costs to be 25 percent higher in September than in April, partly because more labor will be required in September and partly  because employees will get a pay raise. The manager will get a pay raise that will increase the salary from $3,000 in April to $3,500 in September. Utilities will be 5 percent higher in September than in April. Rent will be the same in September as in April.

Now, fast forward to early October and assume the following actual results occurred in September:

 

A

B

C

1

CARMEN’S COOKIES

 

 

2

Retail Responsibility Center

 

 

3

Actual Costs For the

 

 

4

Month Ending September 30

 

 

5

 

Actual

 

6

Food

September)

 

7

Flour

   

8

Eggs

$2,700

 

9

Chocolate

6,500

 

10

Nuts

2,100

 

11

Other

2,300

 

12

Total food

2,700

 

13

Labor

$16,300

 

14

Manager

   

15

Other

$3,500

 

16

Total labor

1,850

 

17

Utilities

2,200

 

18

Rent

5,000

 

19

Total cookie costs

$28,850

 

20

Number of cookies sold

38,400

 

21

 

$5,350

 

22

     

Required

a. Prepare a statement like the one in Exhibit 1.5 that compares the budgeted and actual costs for September.

b. Suppose that you have limited time to determine why actual costs are not the same as budgeted costs. Which three cost items would you investigate to see why actual and budgeted costs are different? Why would you choose those three costs?

cost data for managerial purposes mdash budgeting refer to exhibit 1 5 which shows b 690845

Cost Data for Managerial Purposes—Budgeting

Refer to Exhibit 1.5, which shows budgeted versus actual costs. Assume that Carmen’s Cookies is preparing a budget for the month ending November 30. Management prepares the budget for the month ending November 30 by starting with the actual results for April that appear in  Exhibit 1.5. Then, management considers what the differences in costs will be between April and November.

Management expects cookie sales to be 100 percent greater in November than in April because of the holiday season. Management expects that all food costs (e.g., flour, eggs) will be 120 percent higher in November than in April because of the increase in cookie sales and because prices  for ingredients are generally higher in the high demand holiday months. Management expects “other” labor costs to be 120 percent higher in November than in April, partly because more  labor will be required in November and partly because employees will get a pay raise. (120 percent higher means that the amount in November will be 220 percent of the amount in April.) The manager will get a pay raise that will increase the salary from $3,000 in April to $3,500 in November. Utilities will be 5 percent higher in November than in April. Rent will be the same in November as in April.

Now, move ahead to December and assume the following actual results occurred in November:

 

A

B

C

1

Number of cookies sold

64,000

 

2

 

 

 

3

Flour

$ 4,600

 

4

Eggs

11,200

 

5

Chocolate

4,500

 

6

Nuts

4,450

 

7

Other

4,800

 

8

Manager’s salary

3,500

 

9

Other labor

3,220

 

10

Utilities

1,950

 

11

Rent

5,000

 

12

 

 

 

 

 

 

 

Required

a. Prepare a statement like the one in Exhibit 1.5 that compares the budgeted and actual costs.

b. Suppose that you have limited time to determine why actual costs are not the same as budgeted costs. Which three cost items would you investigate to see why actual and budgeted costs are different? Why would you choose those three costs?

cost data for managerial purposes mdash finding unknowns quince products is a small 690846

Cost Data for Managerial Purposes—Finding Unknowns

Quince Products is a small company in southern California that makes jams and preserves. Recently, a sales rep from one of the company’s suppliers suggested that Quince could increase its profitability by 50 percent if it introduced a second line of products, packaged fruit. She offered to do the analysis and show the company her assumptions.

When Quince’s management opened the spreadsheet sent by the sales rep, they noticed that there were several blank cells. In the meantime, the sales rep had taken a job with a competitor and told the managers at Quince that she could no longer advise them. Although they were not sure they should rely on the analysis, they asked you to see if you could reconstruct the sales  rep’s analysis. They had been considering this new business already and wanted to see if their analysis was close to that of an outside observer. The incomplete spreadsheet is shown below.

 

 width=

Required

Fill in the blank cells.

identifying unethical actions appendix the managers of quince products problem 1 31 690847

Identifying Unethical Actions (Appendix)

The managers of Quince Products (Problem 1 31) decide they will hire a management accountant to help them analyze the decision to expand their product line. They solicit bids from various accountants in the city and receive three proposals. In describing their qualifications for the job, the three state:

Accountant A: “I have recently advised the symphony on how to raise money and therefore I know the local area well.”

Accountant B: “I have advised several small firms on expansion plans.”

Accountant C: “I have advised Pear Company [Quince’s main competitor] and can share its experiences

and insights with you.” All of the proposals have the same price.

Required

a. As the accounting manager of Quince Products, prepare a memo recommending which accountant you would prefer to retain. Be sure to include your reasons.

b. Which, if any, of the accountants making a proposal are violating the IMA’s code of ethics? What is (are) the violation(s)?

responsibility for unethical action the following story is true except that all name 690848

Responsibility for Unethical Action

The following story is true except that all names have been changed and the time period has been compressed.

Charles Austin graduated from a prestigious business school and took a job in a public accounting firm in Atlanta. A client hired him after five years of normal progress through the ranks of the accounting firm. This client was a rapidly growing, publicly held company that produced  software for the health care industry. Charles started as assistant controller. The company promoted him to controller after four years. This was a timely promotion. Charles had learned a lot and was prepared to be controller.

Within a few months of his promotion to controller, the company’s chief financial officer abruptly quit. Upon submitting her resignation, she walked into Charles’s office and said, “I have given Holmes (the company president) my letter of resignation. I’ll be out of my office in less than an hour. You will be the new chief financial officer, and you will report directly to Holmes. Here is my card with my  personal cell phone number. Call me if you need any advice or if I can help you in any way.”

Charles was in over his head in his new job. His experience had not prepared him for the range of responsibilities required of the company’s chief financial officer. Holmes, the company president, was no help. He gave Charles only one piece of advice: “You have lots of freedom to run the finance department however you want. There is just one rule: Don’t ever cross me. If you do, you’ll never work again in this city.” Charles believed his boss could follow through on that threat because he was so well connected in the Atlanta business community.

The end of the company’s fiscal year came shortly after Charles’s promotion to chief financial officer. After reviewing some preliminary financial amounts, Holmes stormed into Charles’s office and made it clear that the results were not to his liking. He instructed Charles to “find more sales.” Charles was shocked, but he did as he was told. He identified some ongoing software installation work that should not have been recorded as revenue until the customer signed off on the job. Charles recorded the work done as of year end as revenue, even though the customer had not signed off on the job. He sent an invoice to the customer for the amount of the improper revenue, then called her to say that the invoice was an accounting error and she should ignore it.

Next year, Charles’s work life was better but his personal life was not. He went through a costly divorce that resulted in limited time spent with his two small children. Now he was particularly concerned about not crossing his boss because of the threat that he would never work in Atlanta if he did. He could not bear to look for a new job that would take him away from his children. Further, it would be difficult to find a job anywhere that came close to paying the salary and benefits of his current job. With high alimony and child support payments, Charles would feel a dire financial strain if he had to take a cut in pay.

The company struggled financially during the year. Clearly, the company would not generate the level of revenues and income that Holmes wanted. As expected, he again instructed Charles to  find some way to dress up the income statement. It did not matter to Holmes whether what Charles did was legal or not.

Charles had exhausted all legitimate ways of reducing costs and increasing revenues. He faced an ethical dilemma. He could resign and look for a new job, or he could illegitimately record nonexistent sales. He now understood why the former chief financial officer had resigned so abruptly. He wished that he could talk to her, but she was traveling in Australia and could not be contacted. The board of directors would be no help because they would take the president’s side in a dispute.

After considering his personal circumstances, Charles decided to record the illegitimate sales as the president had instructed. Charles knew that what he did was wrong. He believed that if the fraud was discovered, Holmes, not he, would be in trouble. After all, Charles rationalized, he was just following orders.

Required

a. Can you justify what Charles did?

b. What could Charles have done to avoid the ethical dilemma that he faced? Assume that the company president would have made it impossible for Charles to work in Atlanta in a comparable job.

c. What if the Securities and Exchange Commission discovered this fraud? Would Charles’s boss get in trouble? Would Charles?

basic concepts for each of the following costs incurred in a manufacturing firm indi 690854

Basic Concepts

For each of the following costs incurred in a manufacturing firm, indicate whether the costs are most likely fixed (F) or variable (V) and whether they are most likely period costs (P) or product costs (M) under full absorption costing.

a. Energy to run machines producing units of output in the factory.

b. Depreciation on the building for administrative staff offices.

c. Bonuses of top executives in the company.

d. Overtime pay for assembly workers.

e. Transportation in costs on materials purchased.

f. Assembly line workers’ wages.

g. Sales commissions for sales personnel.

h. Administrative support for sales supervisors.

i. Controller’s office rental.

j. Cafeteria costs for the factory.

basic concepts place the number of the appropriate definition in the blank next to e 690856

Basic Concepts

Place the number of the appropriate definition in the blank next to each concept.

Concept

Definition

________Period cost

1. Cost that varies with the volume of activity.

________Indirect cost

2. Sacrifice of resources.

________Fixed cost

3. Cost charged against revenue in a particular accounting period.

________Opportunity cost

4. Cost that is part of inventory.

________Outlay cost

5. Cost that can more easily be attributed to time intervals.

________Direct cost

6. Past, present, or near future cash flow.

________Expense

7. Lost benefit from the best forgone alternative.

________Cost

8. Cost used to compute inventory value according to GAAP.

________Variable cost

9. Cost that cannot be directly related to a cost object.

________Full absorption cost

10. Cost that can be directly related to a cost object.

________Product cost

11. Cost that does not vary with the volume of activity.

on the first of may linda buys a coffee business from tom for 20000 for this she rec 688788

On the first of May Linda buys a coffee business from Tom for $20000. For this she receives a car originally purchased for $25000 with accumulated depreciation of $10000 with a built in coffee machine purchased for $5000 with accumulated depreciation of $4000 together with $2000 of coffee, sugar and disposable coffee cups.

During May she sells a total of 2500 cups of coffee @ $4 each at various locations around the city. She also purchases $3000 of coffee, $1000 of milk and $500 of sugar. She pays for the milk and sugar with cash, but the coffee supplier allows her 30 day terms before she has to pay for the coffee. Finally she pays herself a $4000 salary for the month.

In June she decides to move into the corporate market, thinking that it might be more lucrative, and runs several corporate functions worth $20000. She collects $8000 of this income during the month and expects to collect 90% of the remainder. She also purchases $4000 of coffee, $1000 of milk, $500 of sugar and $500 of disposable coffee cups. She pays for the milk and sugar with cash, plus she pays the $3000 that she owes the coffee supplier from May. The coffee supplier once again allows she 30 day terms before she has to pay for the new order of coffee. She also pays herself a $4000 salary for month.

The other major issue in June is that her coffee machine breaks and so she sells it for scrap for $500 and purchases a new machine for $6000 (paying with a $3000 cash deposit and terms to pay the other $3000 within 30 days).

At the end of June she calculates that her car has depreciated by $500 for the two months and she performs a stocktake to find that she has $2000 of coffee, sugar and disposable coffee cups remaining.

What do Linda’s financial accounts look like at the end of June? She will have a tax rate of 20%.

Document Preview:

On the first of May Linda buys a coffee business from Tom for $20000. For this she receives a car originally purchased for $25000 with accumulated depreciation of $10000 with a built in coffee machine purchased for $5000 with accumulated depreciation of $4000 together with $2000 of coffee, sugar and disposable coffee cups. During May she sells a total of 2500 cups of coffee @ $4 each at various locations around the city. She also purchases $3000 of coffee, $1000 of milk and $500 of sugar. She pays for the milk and sugar with cash, but the coffee supplier allows her 30 day terms before she has to pay for the coffee. Finally she pays herself a $4000 salary for the month. In June she decides to move into the corporate market, thinking that it might be more lucrative, and runs several corporate functions worth $20000. She collects $8000 of this income during the month and expects to collect 90% of the remainder. She also purchases $4000 of coffee, $1000 of milk, $500 of sugar and $500 of disposable coffee cups. She pays for the milk and sugar with cash, plus she pays the $3000 that she owes the coffee supplier from May. The coffee supplier once again allows she 30 day terms before she has to pay for the new order of coffee. She also pays herself a $4000 salary for month. The other major issue in June is that her coffee machine breaks and so she sells it for scrap for $500 and purchases a new machine for $6000 (paying with a $3000 cash deposit and terms to pay the other $3000 within 30 days). At the end of June she calculates that her car has depreciated by $500 for the two months and she performs a stocktake to find that she has $2000 of coffee, sugar and disposable coffee cups remaining. What do Linda’s financial accounts look like at the end of June? She will have a tax rate of 20%.

Attachments:

july 1 purchased merchandise from boden company for 6 000 under credit terms of 1 15 688791

July 1

Purchased merchandise from Boden Company for $6,000 under credit terms of 1/15, n/30, FOB shipping point, invoice dated July 1.

2

Sold merchandise to Creek Co. for $900 under credit terms of 2/10, n/60, FOB shipping point, invoice dated July 2. The merchandise had cost $500.

3

Paid $125 cash for freight charges on the purchase of July 1.

8 Sold merchandise that had cost $1,300 for $1,700 cash.
9

Purchased merchandise from Leight Co. for $2,200 under credit terms of 2/15, n/60, FOB destination, invoice dated July 9.

11

Received a $200 credit memorandum from Leight Co. for the return of part of the merchandise purchased on July 9.

12

Received the balance due from Creek Co. for the invoice dated July 2, net of the discount.

16

Paid the balance due to Boden Company within the discount period.

19

Sold merchandise that cost $800 to Art Co. for $1,200 under credit terms of 2/15, n/60, FOB shipping point, invoice dated July 19.

21

Issued a $200 credit memorandum to Art Co. for an allowance on goods sold on July 19.

24

Paid Leight Co. the balance due after deducting the discount.

30

Received the balance due from Art Co. for the invoice dated July 19, net of discount.

31

Sold merchandise that cost $4,800 to Creek Co. for $7,000 under credit terms of 2/10, n/60, FOB shipping point, invoice dated July 31.

please unlock the pdf file by using the following codes if you cannot open it 688808

Please unlock the PDF file by using the following codes if you cannot open it.

1059 1388 290E1 000290E0

Document Preview:

joka joka joka joka joka joka joka joka joka joka joka joka joka joka joka joka joka joka joka joka joka joka joka joka joka joka joka joka joka joka joka joka joka joka joka joka joka joka joka joka joka joka joka joka joka joka joka joka joka joka joka joka joka joka joka joka joka joka joka joka joka joka joka joka joka joka joka joka joka joka joka joka joka joka joka joka joka joka joka joka joka joka joka joka joka joka joka joka joka joka joka joka joka joka joka joka joka joka joka joka joka joka joka joka joka joka joka joka joka joka joka joka joka joka joka joka joka joka joka joka joka joka joka joka joka joka joka joka joka joka joka joka joka joka joka joka joka joka joka joka joka joka joka joka joka joka joka joka joka joka joka joka joka joka joka joka joka joka joka joka joka joka joka joka joka joka joka joka joka joka joka joka joka joka joka joka joka joka joka joka joka joka joka joka joka joka joka joka joka joka joka joka joka joka joka joka joka joka joka joka joka joka joka joka joka joka joka joka joka joka joka joka joka joka joka joka joka joka joka joka joka joka joka joka joka joka joka joka joka joka joka joka joka joka joka joka joka joka joka joka joka joka joka joka joka joka joka joka joka joka joka joka joka joka joka joka joka joka joka joka joka joka joka joka joka joka joka joka joka joka joka joka joka joka joka joka joka joka joka joka joka joka joka joka joka joka joka joka joka joka joka joka joka joka joka joka joka joka joka joka joka joka joka joka joka joka joka joka joka joka joka joka joka joka joka joka joka joka joka joka joka joka joka joka joka joka joka joka joka joka joka joka joka joka joka joka joka joka joka joka joka joka joka joka joka joka joka joka joka joka joka joka joka joka joka joka joka joka joka joka joka joka joka joka joka joka joka joka joka joka joka joka joka joka joka joka joka joka joka joka joka joka joka UVA F 1353 joka joka joka joka joka joka joka joka joka joka joka joka joka…

mr dewey a senior partner of the cpa firm dewey cheatem 688850

Holmes Institute HI5026 Memo 01 – Semester 02, 2014

Background:

Mr. Dewey, a Senior Partner of the CPA firm

Dewey, Cheatem, & Howe, after noting that

many positivist accounting theorists and rightwing economists recommend that general

purpose financial statements (GPFS) be

deregulated and let market forces control the

quality of GPFS, has expressed concern that

the role (and revenues) of independent external

auditors will be dramatically reduced.

Required:

Use the events and outcomes surrounding the

collapse of Enron and Arthur Andersen to give

insight on this issue in a ½ to 1½ page Memo

that is addressed to Mr. Tu Dewie and resolves

his concerns.

A Full Service Creative Accounting Firm

Loopholes Located Books Juggled Taxes Evaded

Funds Diverted Money Laundered Shortages Hidden

Pensions Plundered Credit Denied Parking Validated

Bankruptcies our Specialty!

Document Preview:

Holmes Institute HI5026 Memo 01 – Semester 02, 2014 Background: Mr. Dewey, a Senior Partner of the CPA firm Dewey, Cheatem, & Howe, after noting that many positivist accounting theorists and right wing economists recommend that general purpose financial statements (GPFS) be deregulated and let market forces control the quality of GPFS, has expressed concern that the role (and revenues) of independent external auditors will be dramatically reduced. Required: Use the events and outcomes surrounding the A Full Service Creative Accounting Firm collapse of Enron and Arthur Andersen to give Loopholes Located Books Juggled Taxes Evaded insight on this issue in a ½ to 1½ page Memo Funds Diverted Money Laundered Shortages Hidden that is addressed to Mr. Tu Dewie and resolves Pensions Plundered Credit Denied Parking Validated his concerns. Bankruptcies our Specialty!

Attachments:

mr dewey a senior partner of the cpa firm dewey cheatem howe after noting that many 688851

Mr. Dewey, a Senior Partner of the CPA firm

Dewey, Cheatem, & Howe, after noting that

many positivist accounting theorists and rightwing economists recommend that general

purpose financial statements (GPFS) be

deregulated and let market forces control the

quality of GPFS, has expressed concern that

the role (and revenues) of independent external

auditors will be dramatically reduced.

Required:

Use the events and outcomes surrounding the

collapse of Enron and Arthur Andersen to give

insight on this issue in a ½ to 1½ page Memo

that is addressed to Mr. Tu Dewie and resolves

his concerns.

Document Preview:

Holmes Institute HI5026 Memo 01 – Semester 02, 2014 Background: Mr. Dewey, a Senior Partner of the CPA firm Dewey, Cheatem, & Howe, after noting that many positivist accounting theorists and right wing economists recommend that general purpose financial statements (GPFS) be deregulated and let market forces control the quality of GPFS, has expressed concern that the role (and revenues) of independent external auditors will be dramatically reduced. Required: Use the events and outcomes surrounding the A Full Service Creative Accounting Firm collapse of Enron and Arthur Andersen to give Loopholes Located Books Juggled Taxes Evaded insight on this issue in a ½ to 1½ page Memo Funds Diverted Money Laundered Shortages Hidden that is addressed to Mr. Tu Dewie and resolves Pensions Plundered Credit Denied Parking Validated his concerns. Bankruptcies our Specialty!

Attachments:

hi i have uploaded my assigment as i need to submit this assigment by tthursaday 12a 688864

hi i have uploaded my assigment . as i need to submit this assigment by tthursaday 12am. and first question we have to do three numberic

Document Preview:

HI5028 TAXATION T2, 2014 ASSIGNMENT 2 SOLUTIONS Due date: Friday Week 10 Instructions: This assignment is to be submitted by the due date in both soft copy (Safeassign – Bb) and hard copy. The assignment is to be submitted in accordance with assessment policy stated in the Subject Outline and Student Handbook It is the responsibility of the student submitting the work to ensure that the work is in fact his/her own work. Ensure that when incorporating the works of others into your submission that it appropriately acknowledged. Maximum marks: 20 (20%)Assignment 1: You should attempt both parts to this assignment Note: you should incorporate all sections of the various Acts/regulations where appropriate. Part 1: Case study Periwinkle Pty Ltd (Periwinkle) is a bathtub manufacturer which sells bathtubs directly to the public. On 1 May 2013, Periwinkle provided one of its employees, Emma, with a car as Emma does a lot of travelling for work purposes. However, Emma’s usage of the car is not restricted to work only. Periwinkle purchased the car on that date for $33,000 (including GST). For the period 1 May 2013 to 31 March 2014, Emma travelled 10,000 kilometres in the car and incurred expenses of $550 (including GST) on minor repairs that have been reimbursed by Periwinkle. The car was not used for 10 days when Emma was interstate and the car was parked at the airport and for another five days when the car was scheduled for annual repairs. On 1 September 2013, Periwinkle provided Emma with a loan of $500,000 at an interest rate of 4.45%. Emma used $450,000 of the loan to purchase a holiday home and lent the remaining $50,000 to her husband (interest free) to purchase shares in Telstra. Interest on a loan to purchase private assets is not deductible while interest on a loan to purchase income producing assets is deductible. During the year, Emma purchased a bathtub manufactured by Periwinkle for $1300. The…

Attachments:

harris decided that the easiest way to make the fabricator division appear more prof 688875

Harris decided that the easiest way to make the Fabricator Division appear more profitable was through manipulating the inventory, which was the largest asset on the books. Harris found that by increasing inventory by 2 percent, income could be increased by 5 percent. With the weakness in inventory control, he felt it would be easy to overstate inventory. Employees count the goods using count sheets, and Harris was able to add two fictitious sheets during the physical inventory, even though the auditors were present and were observing the inventory. A significant amount of inventory was stored in racks that filled the warehouse. Because of their height and the difficulty of test counting them, Harris was able to cover an overstatement of inventory in the upper racks.

After the count was completed, Harris added four additional count sheets that added $350,000, or 8.6 percent, to the stated inventory. Harris notified the auditors of the “omission” of the sheets and convinced them that they represented overlooked legitimate inventory.

The auditors traced the items on these additional sheets four sheets to purchase invoices to verify their existence and approved the addition of the $350,000 to the inventory. They did not notify management about the added sheets. In addition, Harris altered other counts sheets before sending them to the auditors by changing unit designations (for example, six engine blocks became six “motors”), raising counts, and adding fictitious line items to completed count sheets. These other fictitious changes added an additional $175,000 to the inflated inventory. None of them was detected by the auditors.

Required:

a. What audit procedures did the auditors apparently not follow that should have detected Harris’ fraudulent increase of inventory?

b. What implications would there be to the auditor of failure to detect material fraud as describe here?

c. What responsibility did the auditors have to discuss their concerns with the entity’s audit committee?

section a 14 marks use your kills to analyze compare criticize evaluate and justify 688889

SECTION A: (14 MARKS)

Use your kills to Analyze, compare, criticize, evaluate and justify the answers in a process to solve the assignment.

RAY Limited

Balance Sheet as at 30

thJune

$

$

Current Assets

2013

2012

Cash

40,000

60,000

Account Receivables

650,000

300,000

Allowance for doubtful debts

(50,000)

(50,000)

Inventory

700,000

290,000

1,340,000

600,000

Non Current Assets

Equipment

1,800,000

1,100,000

Accumulated depreciation

(550,000)

(100,000)

Capitalized borrowing cost

200,000

1,450,000

1,000,000

Total Assets

2,790,000

1,600,000

Current Liabilities

Account payable

670,000

556,000

Tax payable

60,000

44,000

730,000

600,000

Non Current Liabilities

Loan

580,000

600,000

Total Liabilities

1,310,000

1,200,000

Net Assets

1,480,000

400,000

Shareholder’s Equity

Share Capital

1,150,000

250,000

Retained Profit

330,000

150,000

1,480,000

400,000

Sales (all on credit)

1000,000

640,000

Net profit after tax

200,000

128,000

EBIT

290,000

197,000

Tax expenses

41,000

32,000

Attachments:

deagan formed a lawn service business as a summer job to start the business on 1 may 688895

Deagan formed a lawn service business as a summer job. To start the business on 1 May, he deposited $1,000 in a new bank account in the name of the proprietorship. The $1,000 consisted of a $600 loan from his father and $400 of his own money. Deagan rented lawn equipment, purchased supplies, and hired fellow students to mow and trim his customer’s lawns.

At the end of each month, Deagan mailed bills to his customers. On August 31, he was ready to dissolve the business and return to University for the next semester. As he was so busy, he kept few records other than his cheque book and a list of amounts owed to him by customers.

At 31 August, Deagan’s cheque book shows a balance of $690, and his customers still owe him $500. During the summer, he collected $4,250 from customers. His cheque book lists payments for supplies totalling $400, and he still has fuel, whipper snapper cord, and other supplies that cost a total of $50. He paid his employees $1,900, and he still owes them $200 for the final week of the summer.

Deagon rented some equipment from Sea Machine Shop. On 1 May, he signed a six month lease on mowers and paid $600 for the full lease period. Sea will refund the unused portion of the prepayment if the equipment is in good shape. In order to get the refund, Deagan has kept the mowers in excellent condition. In fact, he had to pay $300 to repair a mower.

To transport employees and equipment to jobs, Deagan used a trailer that he bought for $300. He figures that the summer’s work used up one third of the trailer’s service potential. The business cheque book lists a payment of $460 for cash withdrawals by Deagan during the summer. Deagan paid his father back during August.

Required

a) Prepare the income statement of Deagan Lawn Service for the four months May through August.( 2 Marks)

b) Prepare the classified balance sheet of Deagan Lawn Service at 31 August.( 2Marks)

c) Was Deagan’s summer work successful? Give the reason for your answer.(1 Mark)

c) Was Deagan’s summer work successful? Give the reason for your answer.(1 Mark)

Scenario 2 (5 Marks)

Suppose your group is opening a Billabong office in your area. You must make some important decisions—where to locate, how to advertise, and so on—and you must also make some accounting decisions. For example, how often will you need financial statements to evaluate operating performance and financial position? Will you use the cash basis or the accrual basis? When will you account for the revenue that the business earns? How will you account for the expenses?

Required

Write a report to address the following considerations:

a) Will you use the cash basis or the accrual basis of accounting? Give a complete description of your reasoning.

b) How often do you want financial statements? Why? Discuss how you will use each financial statement.

c) What kind of revenue will you earn? When will you record it as revenue? How will you decide when to record the revenue?

Scenario 3 (6 Marks)

Brent and Den worked for several years as sales representatives for Xerox Corporation. During this time, they became close friends as they acquired expertise with the firm’s full range of copier equipment. Now they see an opportunity to put their experience to work and fulfil lifelong desires to establish their own business. The local TAFE College is expanding, and there is no copy centre within eight kilometers of the campus. Business in the area is booming—office buildings and flats are springing up, and the population of this section of the city is growing.

Brent and Den want to open a copy centre near the campus. A small shopping centre across the street from the TAFE has a vacancy that would fit their needs. Brent and Den each have $20,000 to invest in the business, but they forecast the need for $30,000 to renovate the store. Xerox will lease two large copiers to them at a total monthly rental of $4,000. With enough cash to see them through the first six months of operation, they are confident they can make the business succeed. The two work very well together, and both have excellent credit ratings. Brent and Den must borrow $80,000 to start the business, advertise its opening, and keep it running for its first six months.

Required

Assume the roles of Brent and Den, the partners who will own the Copy Centre.

1. On behalf of Brent and Den write a loan request that Brent and Den will submit to a bank with the intent of borrowing $80000 to be paid back over three years. The loan will be a personal loan to the partnership of Brent and Den, not to the Copy Centre. The request should specify all the details of Brent’s and Den’s plan that will motivate the bank to grant the loan. Include a budgeted income statement for the first six months of the copy centre’s operation.

huron company produces a commercial cleaning compound known as zoom the direct mater 688920

Huron Company produces a commercial cleaning compound known as Zoom. The direct materials and direct labor standards for one unit of Zoom are given below:



During the most recent month, the following activity was recorded:

(a)Twenty thousand pounds of material were purchased at a cost of $2.35 per pound.

(b)All of the material purchased was used to produce 4,000 units of Zoom.

(c) 750 hours of direct labor time were recorded at a total labor cost of $10,425.

Required:

1. Compute the direct materials price and quantity variances for the month.

2. Compute the direct labor rate and efficiency variances for themonth.

accounting for business decisions hi5001 trimester 2 2014 the assignment allows stud 688923

Accounting for Business Decisions –HI5001 Trimester 2 2014 The assignment allows students to exhibit their knowledge and understanding of the subject matter of Accounting. The students will use the skills of analyzing, comparing, criticizing, evaluating and justifying their responses in answering the assignment in a report format. The students need to be well read to answer the questions. Requirements: The students will be working in a group of four (Maximum) or as specified by the concerned lecturer. Forming the group is a responsibility of the students, lecturer’s role is to facilitate the group if required. Week 5 would be the last date for the students to submit the names of their group members to the concerned lecturer. Assignment work load should be shared equally. Each Group member to state and list in front of the assignment, as to what part they have completed and accomplished (i.e. Duties and Responsibilities). Each part would be thoroughly answered. Maximum word Limit is 2500 words. Students to download assessment criterion available in the blackboard and attach it in front of the assessment. SECTION A: (14 MARKS) SECTION B: (16 MARKS) TOTAL: (30% MARKS) Due Date: Week 10 before 5.00 pm. Don’t forget to upload onto Safe assign, one copy per group (do not upload more than one copy). Also, one hard copy to be handed to the lecturer before the schedule time as mentioned above. 2 HI5001_ Accounting for Business Decisions SECTION A: (14 MARKS) Use your kills to Analyze, compare, criticize, evaluate and justify the answers in a process to solve the assignment. RAY Limited Balance Sheet as at 30th June $ $ Current Assets 2013 2012 Cash 40,000 60,000 Account Receivables 650,000 300,000 Allowance for doubtful debts (50,000) (50,000) Inventory 700,000 290,000 1,340,000 600,000 Non Current Assets Equipment 1,800,000 1,100,000 Accumulated depreciation (550,000) (100,000) Capitalized borrowing cost 200,000 1,450,000 1,000,000 Total Assets 2,790,000 1,600,000 Current Liabilities Account payable 670,000 556,000 Tax payable 60,000 44,000 730,000 600,000 Non Current Liabilities Loan 580,000 600,000 Total Liabilities 1,310,000 1,200,000 Net Assets 1,480,000 400,000 Shareholder’s Equity Share Capital 1,150,000 250,000 Retained Profit 330,000 150,000 1,480,000 400,000 Sales (all on credit) 1000,000 640,000 Net profit after tax 200,000 128,000 EBIT 290,000 197,000 Tax expenses 41,000 32,000 Required: (Each question is 2 marks) a. What is the interest expense for 2013? b. How much equipment was purchased during the year? c. What was the depreciation expense for 2013? d. Were any share issues? If any, calculate the value. e. How much in dividend was paid during the year 2013? f. How much cash was received from customers during the year? g. How much was paid in tax? 3 HI5001_ Accounting for Business Decisions SECTION B: (16 MARKS) (Scenario based) Scenario 1 (5 Marks) Deagan formed a lawn service business as a summer job. To start the business on 1 May, he deposited $1,000 in a new bank account in the name of the proprietorship. The $1,000 consisted of a $600 loan from his father and $400 of his own money. Deagan rented lawn equipment, purchased supplies, and hired fellow students to mow and trim his customer’s lawns. At the end of each month, Deagan mailed bills to his customers. On August 31, he was ready to dissolve the business and return to University for the next semester. As he was so busy, he kept few records other than his cheque book and a list of amounts owed to him by customers. At 31 August, Deagan’s cheque book shows a balance of $690, and his customers still owe him $500. During the summer, he collected $4,250 from customers. His cheque book lists payments for supplies totalling $400, and he still has fuel, whipper snapper cord, and other supplies that cost a total of $50. He paid his employees $1,900, and he still owes them $200 for the final week of the summer. Deagon rented some equipment from Sea Machine Shop. On 1 May, he signed a six month lease on mowers and paid $600 for the full lease period. Sea will refund the unused portion of the prepayment if the equipment is in good shape. In order to get the refund, Deagan has kept the mowers in excellent condition. In fact, he had to pay $300 to repair a mower. To transport employees and equipment to jobs, Deagan used a trailer that he bought for $300. He figures that the summer’s work used up one third of the trailer’s service potential. The business cheque book lists a payment of $460 for cash withdrawals by Deagan during the summer. Deagan paid his father back during August. Required a) Prepare the income statement of Deagan Lawn Service for the four months May through August.( 2 Marks) b) Prepare the classified balance sheet of Deagan Lawn Service at 31 August.( 2Marks) c) Was Deagan’s summer work successful? Give the reason for your answer.(1 Mark) 4 HI5001_ Accounting for Business Decisions Scenario 2 (5 Marks) Suppose your group is opening a Billabong office in your area. You must make some important decisions—where to locate, how to advertise, and so on—and you must also make some accounting decisions. For example, how often will you need financial statements to evaluate operating performance and financial position? Will you use the cash basis or the accrual basis? When will you account for the revenue that the business earns? How will you account for the expenses? Required Write a report to address the following considerations: a) Will you use the cash basis or the accrual basis of accounting? Give a complete description of your reasoning. b) How often do you want financial statements? Why? Discuss how you will use each financial statement. c) What kind of revenue will you earn? When will you record it as revenue? How will you decide when to record the revenue? Scenario 3 (6 Marks) Brent and Den worked for several years as sales representatives for Xerox Corporation. During this time, they became close friends as they acquired expertise with the firm’s full range of copier equipment. Now they see an opportunity to put their experience to work and fulfil lifelong desires to establish their own business. The local TAFE College is expanding, and there is no copy centre within eight kilometers of the campus. Business in the area is booming—office buildings and flats are springing up, and the population of this section of the city is growing. Brent and Den want to open a copy centre near the campus. A small shopping centre across the street from the TAFE has a vacancy that would fit their needs. Brent and Den each have $20,000 to invest in the business, but they forecast the need for $30,000 to renovate the store. Xerox will lease two large copiers to them at a total monthly rental of $4,000. With enough cash to see them through the first six months of operation, they are confident they can make the business succeed. The two work very well together, and both have excellent credit ratings. Brent and Den must borrow $80,000 to start the business, advertise its opening, and keep it running for its first six months. Required Assume the roles of Brent and Den, the partners who will own the Copy Centre. 1. On behalf of Brent and Den write a loan request that Brent and Den will submit to a bank with the intent of borrowing $80000 to be paid back over three years. The loan will be a personal loan to the partnership of Brent and Den, not to the Copy Centre. The request should specify all the details of Brent’s and Den’s plan that will motivate the bank to grant the loan. Include a budgeted income statement for the first six months of the copy centre’s operation. 5 HI5001_ Accounting for Business Decisions Note: All answers must be brief and thoroughly explained and not one line answers. Students not to look at the marks allocated of the questions. Marks are for the concerned lecturer to mark accordingly. Please keep in mind the Academic Policy of Misconduct (ie Plagiarism and Collusion). Late submission of assignment by one day incurs a penalty of 10% each day. For students who enroll late, it’s their responsibility to be on the top of their work. All assignment to be submitted as on time or at the discretion of the concerned lecturer.

Attachments:

on 17 october 2011 sydney ltd created a group entity when it purchased 90 of the 688964

On 17 October 2011 Sydney Ltd created a group entity when it purchased 90% of the issued capital of Tower Ltd for $289,980 cash. On acquisition,TowerLtd’s accounts showed: Share capital $200,000 and Retained earnings $58,000. All assets and liabilities appearing in TowerLtd’s financial statements were fairly valued, except:

  • One of their blocks of land was recorded at $100,000 when its fair value was judged by the group to be $130,000. During the following financial year this land was sold for $140,000 cash.
  • An item of plant was undervalued by $50,000. At that time it had a remaining life of 5 years and accumulated depreciation of $36,000. The plant is still an asset of Tower Ltd at 30 June 2014.
  • A contingent liability relating to an unsettled legal claim with a fair value of $50,000 was recorded in the notes to the financial statements. This amount will be tax deductible when paid. The court case is still in progress at 30 June 2014.

During the year ended 30 June 2013:

  1. On 1 July 2012 Tower Ltd sold an item of plant to Sydney Ltd for $59,000. This was financed by a short term interest free loan from Tower Ltd that was repaid 14 months later. The plant had cost $64,000 when purchased on 13November 2011. It’s expected useful life was originally 5 years and this original estimate is still considered to be valid. The plant is still an asset of Sydney Ltd at 30 June 2014.
  2. During the year Sydney Ltd made sales of inventory to Tower Ltd of $569,600. The inventory balance ofTower Ltd at the end of the year included stock of $84,000 acquired from Sydney Ltd. Sydney Ltd declared and paid dividends of $90,000 for the year. Tower Ltd did not declare or pay any dividends for the year.

During the year ended 30 June 2014:

  1. On 23 December 2013Sydney Ltd sold an item of plant to Tower Ltd for $100,000 when its carrying value in Sydney’s books was $170,000 (original cost $212,500 and original estimated life of 10 years). The plant is still an asset of Tower Ltd at 30 June 2014.
  2. During the year Tower Ltd made sales of inventory to Sydney Ltd of $88,200.The inventory balance of Sydney Ltd at the end of the year included stock of $54,300 acquired from Tower Ltd.
  3. The management of Sydney Ltd believes that the goodwill acquired on acquisition of Tower Ltd was impaired by $5,000 in the current year. This is in addition to a total of $8,000 of impairment in previous years.
  4. Sydney Ltd charged management fees to Tower Ltd.
  5. Dividends were declared/paid by both companies.
  6. Non controlling interests in Tower Ltd to be recognised. This is the only subsidiary in the group.

ADDITIONAL INFORMATION:

  • The company tax rate is currently 30% and it has been this rate for many years.
  • SydneyLtd has the following accounting policies for the group:

(i) Revaluation adjustments on acquisition are to be made on consolidation only, not in the books ofany subsidiary;

(ii) Non controlling interests are measured at fair value;

(iii) Intragroup sales of inventory to be at a selling price of cost plus a mark up of 50%;

(iv) Plant is depreciated using the straight line method with no residual value. For part years, depreciation is to be calculated on the number of days the asset is held in the relevant year, with the day of acquisition counting as one day while the day of disposal does not count; and

(v) All calculated amounts are to be rounded to the nearest whole dollar. Companies in the group do not show cents in any journals, worksheets, or financial statements.

NOTE:

  • You MUST number your journal entries and present them in the order as they relate to the number given for each “event”. Where more than one journal is needed for an “event” to be completely accounted for add the letters a,b,c,…etc to them as necessary. [For example, if three separate journal entries are required to fully record the information detailed in point number 1, then the first journal will be 1a and the second is to be 1b and the third 1c.] Short narrations are expected for each journal entry. Marks will be lost if journals are not presented in a clear and professional manner (i.e. poor or unclear presentation can include showing the debit entry on one page but the credit entry on another, or not clearly distinguishing between debit and credit entries).
  • The required statements for both the group and the parent company are: the statement of comprehensive income, statement of financial position, and statement of changes in equity. Follow the formats shown in Chapter 29 of the textbook. Notes to the statements are not required. Marks will be lost if statements are not presented in a clear and professional manner (i.e. poor or unclear presentation can include splitting the reports over two pages, so start each statement on a new page!).
  • You may “cut and paste” the financial information on the next page into your excel file, but no other information is to be copied into your file from anywhere else.
  • You are expected to use at least the basic formula functions in Excel when preparing worksheets and financial statements (i.e. use Excel formulas to add totals and sub totals etc, rather than calculating values manually and then just typing them in to the spreadsheet!).
  • This is the final unit in the accounting major where you will have to produce complex journal entries and financial reports at a professional level. Therefore, a very high standard is expected. Approach it as if you are preparing it for your employer. The reports cannot be late for the board meeting and the direrctors carefully review all of the information you give them. They pay you well, but they expect quality work. It needs to be technically correct and presented well, otherwise you will need to look for another job!!!

AT 30 JUNE 2014 SYDNEY LTD TOWER LTD
$ $
INCOME STATEMENTS
Sales revenue 1,365,300 992,200
Cost of goods sold 692,000 618,500
Gross profit 673,300 373,700
Other income
Management fee revenue 12,900
Dividend revenue 87,960
Expenses
Depreciation expense (133,300) (59,000)
Management fee expense (12,900)
Loss on sale of asset (70,000)
Other expenses (426,200) (163,400)
Profit before tax 144,660 138,400
Income tax expense (17,010) (41,520)
Profit for the year after tax 127,650 96,880
Retained earnings at start of year 159,220 134,320
Dividend paid/declared (75,000) (96,400)
Retained earnings at end of year 211,870 134,800
BALANCE SHEETS
Equity
Share capital 450,000 200,000
Retained earnings 211,870 134,800
Current Liabilities
Accounts payable 210,280 148,180
Income tax payable 10,910 42,420
Dividends payable 37,500 48,200
Non Current Liabilities
Bank Loans 710,000 650,000
Provision for employee benefits 29,100 14,300
Deferred tax liability 6,100
1,665,760 1,237,900
Current Assets
Accounts receivable 176,800 98,700
Allowance for doubtful debts (22,100) (9,500)
Dividends receivable 43,980
Inventory 98,300 121,200
Non Current Assets
Land and buildings 790,000 910,800
Plant – at cost 449,700 301,200
Accumulated depreciation – plant (169,400) (185,400)
Deferred tax asset 900
Shares in The Rocks Pty Ltd 8,500
Investment in Tower Ltd 289,980
1,665,760 1,237,900

Attachments:

hal smith opened smith s repairs on march 1 of the current year during march the fol 688992

Hal Smith opened Smith’s Repairs on March 1 of the current year. During March, the following transactions occurred and were recorded in the company’s books:

1. Smith invested $38,000 cash in the business.

2. Smith contributed $113,000 of equipment to the business.

3. The company paid $3,300 cash to rent office space for the month.

4. The company received $29,000 cash for repair services provided during March.

5. The company paid $7,500 for salaries for the month.

6. The company provided $4,300 of services to customers on account.

7. The company paid cash of $1,800 for monthly utilities.

8. The company received $4,400 cash in advance of providing repair services to a customer.

9. Smith withdrew $6,300 for his personal use from the company.

Based on this information, the balance in Hal Smith, Capital reported on the Statement of Owner’s Equity at the end of March would be:

accounting principles 688995

Hal Smith opened Smith’s Repairs on March 1 of the current year. During March, the following transactions occurred and were recorded in the company’s books:

1. Smith invested $38,000 cash in the business.

2. Smith contributed $113,000 of equipment to the business.

3. The company paid $3,300 cash to rent office space for the month.

4. The company received $29,000 cash for repair services provided during March.

5. The company paid $7,500 for salaries for the month.

6. The company provided $4,300 of services to customers on account.

7. The company paid cash of $1,800 for monthly utilities.

8. The company received $4,400 cash in advance of providing repair services to a customer.

9. Smith withdrew $6,300 for his personal use from the company.

Based on this information, the balance in Hal Smith, Capital reported on the Statement of Owner’s Equity at the end of March would be:

$169,800.
$165,400.
$157,800.
$7,100.
$26,500.

dear student campus security has been getting reports 689000

Dear Student,

Campus Security has been getting reports that students have been refused taxi rides because the fare from downtown to UB is not worth it to the taxi drivers. Any student who is refused a ride from the downtown area to the UB campus should immediately walk up into the Metro North train station and report this to the Metro North Police Officer on duty. It is against the law for any taxi driver to refuse any person a ride (No matter how short the distance) and the penalty is that they could lose their license. If this happens again, please report this to Campus Security and we will follow up.

Enjoy your summer!

Richard Loehn

Associate Director

Department of Campus Security

221 University Avenue

Bridgeport, CT 06604

Phone: (203) 576 4942 Fax (203) 576 4910 Dear Student,

With fall payment deadlines approaching, we want to encourage you to visit bridgeport.peertransfer.com when making your next payment to UB.

peerTransfer continually strives to provide you with the most convenient way to pay, and is always adding additional payment options for you to choose from. Whether you are looking to make an online payment using your international credit/debit card or send a wire/bank transfer UB, this method of payment is the most secure and efficient way for moving money from your home bank account.

Paying with peerTransfer allows you to:

• Save on bank fees and exchange rates

• Receive peace of mind with dedicated multilingual customer support

• Track your payment from start to finish

Thank you for your cooperation. I hope your semester is going well and you have a safe trip home. Your I 20 is ready for pick up.

I have included the reporting and Health Alert information for you as below:

Reporting to International Student Services upon returning

Please bring your passport to ISS for us to make a copy and lift the hold posted in the University’s system as soon as you reenter the U.S. Before we report your registration in SEVIS, we must know you are physically present in the United States. The reporting deadline for the Fall 2014 semester is August 25 for undergraduate and graduate students, and August 10 for Health Sciences students. Human Nutrition students must follow your class schedule. Failure to do so or returning late could lead to termination of your SEVIS record.

Health Alert

Prior to leaving the U.S., please visit the follow pages for travel advisories, vaccine preventable illnesses and other health related resources:

Center for Disease Control Traveler’s Page: http://wwwnc.cdc.gov/travel/destinations/list

Bridgeport Area Travel Clinics:

Passport Health: www.passporthealthusa.com/

Bridgeport Hospital: www.bridgeporthospital.org/healthytraveler/ More Listings are on: http://www.travelhealthresource.com/clinics/CONNECTICUT.asp Dear Student,

Shuttle services will be canceled from 2:00pm to 12 mid on Sunday, July 13 due to the PR Day Celebration. Dear students,

The dean has volunteered to give you a prep class next week for the ETS exam on

July 16 (room 301). Enjoy!

See (some of) you in two weeks!

Best,

A Hello,

The Office of Information Technology will perform a planned, monthly systems maintenance procedure this Saturday night, June 21 between 8PM and Sunday 6AM. During the maintenance window, there will be significant disruptions in service on the majority of University systems including (but not limited to) Portal, Email, Canvas, and the website. All systems are expected to get back to normal by Sunday 6AM. We appreciate your patience as we work to better our quality of service andapologize in advance for any inconvenience this may cause. Sorry, I do see YOUR return of June 17
th. My apology for making you go through all that extra looking for the ticket.

Your I 20s are all ready for pick up.

Regards,

Marlene Bojarski

Secretary

International Student Services

From: Alshammari, Sowelim [mailto:salshamm@my.bridgeport.edu]
Sent: Monday, June 09, 2014 4:00 PM

To: Bojarski, Marlene

Subject: Re: Alshammari
Need help finding a job, preparing for an interview, writing a resume & cover letter, or figuring out what path is right for you?

Career Services is here to help you ALL summer!

Please call
203 576 4466
to make an appointment

Summer Hours:
Monday Friday 8:30am 4:30pm
Not near Bridgeport for the summer?
We are still offering resume reviews through Optimal Resume. Submit yourresumeto the virtual Review Center andreceivequick feedback from our staff members.

https://bridgeport.optimalresume.com/

No matter where you are, Follow us on Social Media for career tips and daily job and internship listings!
Twitter:

https://twitter.com/UB_CareerCenter

Facebook:
https://www.facebook.com/pages/University of Bridgeport Career Services Center/291620114198174 Dear All,

Today’s class is canceled due to an emergency. I apologize for the inconvenience. We will have a lecture next week as scheduled. Best

Dear All,
Today’s class is canceled due to an emergency. I apologize for the inconvenience. We will have a lecture next week as scheduled. We will have the quiz next week as well.
Best,
Elif Kongar

Dear Student,

If you have not attended the mandatory immigration meeting held before or have an hold on your account caused by late reporting to ISS after traveling, I urge you to attend this meeting:
Not sure if you need to do an internship? Have questions about the process? Need to learn more about the value of having an internship?

Join the Career Services Office and UB Alumni to learn more about the world of internships.

You will have the opportunity to network with successful alumni while having all your internship questions answered.

When: April 21st from 4:30 5:30 pm

Where: Great Room, John J. Cox Student Center

Please register on UBjobnet No matter your major, the information provided at this event will be valuable for you and leave you more prepared for your future career steps.

Nuriye Elif Kongar Bahtiyar just sent you a message in Canvas:


Dear All,
There is a quiz tomorrow in the class. After that, we will be watching a video clip and conclude the class. Best, EK

We will have time for News articles today. If you have not been raising your hand or joining the discussion in class, you participation grade is LOW.

Improve it today by finding a RELEVANT news article, and raising your hand!

Those of you who always carry the load will be my reserve today! Thank you for your on line registration. Please be sure to check

your student account on WebAdvisor for the billing related to this

registration. Also, check “my schedule” on WebAdvisor to

ensure that your registration has been fully processed. If you do

not see your schedule please go to “Drop and Register Sections”

in WebAdvisor and make sure to choose an action for each course

and hit submit at the bottom of the screen. Also, please be aware

that dropping classes may result in changes to your Financial Aid.

Please contact the Student Financial Services Offic

Great job everyone!

I took notes during your presentation; here are a few comments in the hopes

that they might help you in the future:

EZ Campus

Abdulah Almushaigeh

Thamer Aljohney

Sowelim Alshammari

Osamah Alinaizi

Good intro & project scope

Good responsibility assignment matrix

Good Gantt chart & activities & durations

Good budget & costs

Nice Risk charts

I like the lessons learned slide Dear Student,

We are approaching the end of the Spring 2014 semester. Hope you have enjoyed this semester!

We know by the end of a semester you are preoccupied with preparation for your exams, traveling and graduation, etc. This email, being sent to all students who are registered for the Spring 2014 semester, serves as a reminder on how to maintain your F 1 status.

Degree completion with OPT

If you will be completing your degree requirements this semester and have not applied for OPT, you need to apply for it NOW! Please visit our website http://www.bridgeport.edu/life/international student services/immigration/f 1 student status/f 1 optional practical training/ and follow the instructions on OPT application.

Degree Completion Without OPT

If you will be completing your degree requirements, but will not apply for OPT, please make sure to apply for graduation and inform the ISS of your intention through email internationaloffice@bridgeport.edu. For example, will you be changing to a new program at U.B., leaving the U.S., transferring to another U.S. school, or changing your non immigrant status to something else? Failure to inform us might leave a negative record in SEVIS and affect your reentry to the U.S. in the future.

I 20 Extension

If you will not be completing your degree requirements and will require an extension of your I 20 form, please download the I 20 extension application at http://www.bridgeport.edu/life/international student services/immigration/f 1 student status/f 1 i 20 form extension/. Please submit your application for an I 20 form extension as soon as possible, but no later than one week prior to the expiration of your I 20 form. Remember that current original financial documentation is a required part of the application.

Traveling Outside the United States

1. Travel signature

Travel signatures will be authorized on a first come first served basis. You must apply for it at least two weeks in advance of the planned travel date.

Prior to applying for a travel signature, please do the following:

• Visit ISS website on travel at http://www.bridgeport.edu/life/international student services/immigration/travel/

• Pre register for the Fall 2014 semester

• No holds on your account

• No past due balance

• Visit myUB at http://www.bridgeport.edu/myub, select Travel Signature Request Form under Forms Library; Fill out the form, sign and print it

• Bring your original I 20, Travel Request Form and round trip flight reservation to ISS for a travel signature

2. Health Alert

Prior to leaving the U.S., please visit the follow pages for travel advisories, vaccine preventable illnesses and other health related resources:

Center for Disease Control Traveler’s Page: http://wwwnc.cdc.gov/travel/destinations/list

Bridgeport Area Travel Clinics:

Passport Health: www.passporthealthusa.com/

Bridgeport Hospital: www.bridgeporthospital.org/healthytraveler/

More Listings are on: http://www.travelhealthresource.com/clinics/CONNECTICUT.asp

Reporting to International Student Services upon Returning

Please bring your passport to ISS for us to make a copy and lift the hold posted in the University’s system as soon as you reenter the U.S. Before we report your registration in SEVIS, we must know you are physically present in the United States. The reporting deadline for the Fall 2014 semester is August 24 for undergraduate and graduate students, and August 10 for Health Sciences students. Human Nutrition students must follow your class schedule. Failure to do so or returning late could lead to termination of your SEVIS record.

Hope you all have a successful conclusion to this semester.

Take care and travel safe!!!

Yumin Wang

Director/PDSO/RO

International Student Services

University of Bridgeport|126 Park Avenue, G133, Bridgeport, CT 06604|Tel: 203 576 4395

Fax: 203 576 4461|www.bridgeport.edu/iss|Email: fsa@bridgeport.edu

Congratulations Graduate!

Thank you for submitting your request.

Please allow 3 business days for Processing of requests for a commencement packet.

Requests for extra tickets are placed in a lottery. The Extra Tickets Lottery drawing will be April 24, 2014. If your name is selected you will be notified by your my.bridgeport.edu email account. Please do not contact the Special Events Office, this will only delay the notification of the winning recipients.

Please send any additional questions to ubcommencement2014@bridgeport.edu.

The Office of Special Events Dear Student,

If you are interested in becoming a Visiting Assistant Volunteer at the International Student Services Office, there will be one Group Interview Session this week. Please pass by the office with your updated resume and cover letter to participate in the session. The interview will take place in the ISS Conference Room. You must have a minimum of two semesters remaining, be available this summer break and your GPA must be 3.3 GPA or above. Dear Engineering Student,

This year the School of Engineering is planning an extraordinary event to recognize the exceptional achievements of our students. The Engineering Achievement Ceremony will include:

1. Announcement of the students receiving academic achievement awards from each department / program in the School.

2. Announcement of students recognized for their performance through the academic year.

3. Recognition of all the Engineering winners at the UB Faculty Research Day.

4. Recognition of all the UB Engineering winners at the ASEE Zone 1 conference.

5. Graduating Ph.D. students hooding ceremony.

Dr. Sobh and I would like to invite you, your family, and your friends to attend this memorable day.

Looking forward to seeing you at the “Engineering Achievement Ceremony” on Friday May 9, 2014 at 2:00 p.m. in the Social Room at John Cox Student Center.

Best regards Hi students,

I might be a few minutes late to class this evening.

Blaine Dear Student,

If you are still interested in exhibiting your art, photos and/or cultural souvenirs on Monday, April 7, 2014, please pass by the Student Center, Great Room at 3PM for set up.

All other students, faculty and staff members are welcome to pass by from 4PM 6PM to enjoy this unique and cultural event featuring drinks from around the world and international snacks!

Hope to see you there!

Nadine Boudissa

Assistant Director

University of Bridgeport

International Student Services

126 Park Avenue

Bridgeport, CT 06604

Office: 203 576 4395 Fax: 203 576 4461 Hello,

The Office of Information Technology will perform a required systems maintenance procedure this Saturday night, March 29 between 9PM and Sunday 1AM. During the maintenance window, there will be intermittent disruptions in service on the Portal. All systems are expected to get back to normal by Sunday 1AM. We appreciate your patience as we work to better our quality of service andapologize in advance for any inconvenience this may cause.

Sincerely,
OPT Q&A SESSIONS

Are you an international student who will be graduating soon?

Are you still confused about OPT?

Come to one of the Q&A Sessions to learn more about

OPT and HOW TO FIND A JOB!

DATES:

Wednesday, April 2, 2014: 12pm to 1pm

Wednesday, May 7, 2014: 12pm to 1pm

LOCATION:

Discovery Pavilion (Wahlstrom Library, 1st Floor)

Heather Varnet

International Student Advisor

International Student Services

University of Bridgeport

126 Park Avenue, G 133

Bridgeport, CT 06604

Tel: (203) 576 4395

Fax: (203) 576 4461

Email: hvarnet@bridgeport.edu

The information transmitted is intended solely for the person or entity to which it is addressed and may contain confidential and/or privileged material. Any review, retransmission, dissemination or other use of, or taking of any action in reliance upon this information by persons or entities other than the intended recipient is prohibited. If you receive this email in error, please contact the sender or delete the material from your computer. Dear Engineering student,

This is a reminder about Summer 2104 and Fall 2014 registration. The scheduled starts as follows:

Grads, Athletes, Seniors, Current Honors Program Students March 31, 8am

Juniors April 3, 8am

Sophomores April 7, 8am

Freshman April 10, 8am

Please make sure that you meet your advisor if you have any questions. Also, it is very important to register as early as possible to make sure that you reserve your seats in the classes you are interested in. You will not be able to register in closed classes. Also, classes with low registrations are subject to cancelations.

Please contact your academic advisor if you have any questions.

Best regards Dear students,

At this point in time the class is way over subscribed. Please remember that

you need to have completed all the 400 level classes and six upper level

classes before taking this course thanks.

Also, I may want to offer a second session on Saturday (9:00am 1:15pm). Would

any of you be interested in this section (please e mail me)?

Best,

A

Attachments:

need financial statement calculations for the following scenario blb enterprises is 689006

Need Financial Statement Calculations for the following scenario:

BLB Enterprises is located in France and has been in business for several years. They use IFRS to prepare their financial statements and have a calendar year accounting period (January 1 – December 31). Ending balances of the accounts at September 30, 2014 are as follows:

Account

Balance

Notes

Accounts Payable

€15,000

Accounts Receivable

€28,000

Accumulated Depreciation Equipment

€21,350

Advertising Expense

€0

Cash

€24,300

Common Stock

€150,000

Cost of Goods Sold

€0

Depreciation Expense

€0

Dividends

€0

Equipment

€99,600

Consists of: €31,000 equipment with €1,000 salvage value and 5 year estimated life; €40,000 equipment with €7,000 salvage value and 4 year estimated life; and €28,600 equipment with €2,500 salvage value and 3 year estimated life

Gain on sale of assets

€0

Insurance Expense

€0

Income Tax Expense

€0

Income Tax Payable

€12,000

Income Taxes relating to third quarter that won’t be paid until future months.

Interest Expense

€0

Interest Payable

€0

Inventory

€46,320

Land

€20,000

Loss on sale of assets

€0

Machine Rental Expense

€0

Note Payable to Bank

€0

Other operating expenses

€0

Prepaid Advertising

€0

Prepaid Insurance

€0

Prepaid Machine Rent

€6,000

Paid €12,000 in June for the months of July December

Retained Earnings

€25,670

Sales Revenue

€0

Supplies

€4,800

Supplies were counted at the end of September

Supplies Expense

€0

Unearned Revenue

€0

Wages Expense

€0

Wages Payable

€5,000

September wages won’t be paid until October

In October 2014, the company had the following transactions:
1. Made sales on account of €180,000 and cash sales of €15,000. The cost of the inventory sold was

€125,000.

  1. Purchased €135,500 of inventory on account.

  2. Purchased €2,500 of supplies for cash

  3. Made cash payments of €15,000 to employees for wages. This €15,000 includes the €5,000 owed

    to employees for September wages.

  4. On October 1, borrowed €50,000 from the bank. The principal of the note (the €50,000) is due in

    two years. The note has a 6% interest rate and interest is due quarterly (first payment is due

    January 1, 2015).

  5. The company declared and paid a cash dividend of €25,000

  6. The company paid €18,000 for insurance. The policy covers the company from October 1, 2014

    through March 31, 2015.

  7. The company received €10,000 from a customer in advance as a down payment. The sale will not

    take place until November.

  8. The company paid vendors €140,200 for payment of accounts payable.

  9. The company collected €175,000 from customers on account.

  10. The company sold the land they owned. They received €18,000 cash from the sale of the land.

  11. The company paid €27,550 for other operating expenses.

  12. The company issued additional common stock in exchange for €20,000

  13. The company purchased €60,000 of new equipment for cash. The estimated salvage value is

    €9,000 and the estimated life is 4 years.

  14. The company paid €5,000 for radio advertising. Half of the advertising relates to October

    advertising and half of the advertising will take place in November.

Use the information above (including beginning balance notes) to prepare adjusting journal entries. Additional information you may need:

a. €3,200 of wages for the last 3 days of October will be paid in November.
b. A physical count of supplies indicated that, at the end of October, there were €6,100 of supplies on

hand.
c. The company has €7,000 of income tax expenses relating to October which will not be paid until a

future month.

automatic versus manual processing photo station company operates a printing service 689023

Automatic versus Manual Processing
Photo Station Company operates a printing service for customers with digital cameras. The current service, which requires employees to download photos from customer cameras, has monthly operating costs of $6,000 plus $0.20 per photo printed. Management is evaluating the desirability of acquiring a machine that will allow customers to download and make prints without employee assistance. If the machine is acquired, the monthly fixed costs will increase to $10,000 and the variable costs of printing a photo will decline to $0.04 per photo.
(a) Determine the total costs of printing 20,000 and 50,000 photos per month.

Units Current Process Proposed Process
20,000 $Answer $Answer
50,000 $Answer $Answer

(b) Determine the monthly volume at which the proposed process becomes preferable to the current process.
Answer

units

the following data relate to the operations of lim corporation a wholesale distribut 689093

The following data relate to the operations of Lim Corporation, a wholesale distributor of consumer goods:

Current assets as of December 31:

Cash………………………. $6,000

Accounts receivable…………… $36,000

Inventory…………………… $9,800

Buildings and equipment, net ……… $110,885

Accounts payable ……………… $32,550

Common shares……………….. $100,000

Retained earnings ……………… $3

Document Preview:

PROBLEM 1 Completing a Master Budget [L04] The following data relate to the operations of Lim Corporation, a wholesale distributor of consumer goods: Current assets as of December 31: Cash………………………. $6,000 Accounts receivable…………… $36,000 Inventory…………………… $9,800 Buildings and equipment, net ……… $110,885 Accounts payable ……………… $32,550 Common shares……………….. $100,000 Retained earnings ……………… $30,135 a. The gross margin is 30% of sales. (In other words, cost of goods sold is 70% of sales.) b. Actual and budgeted sales data are as follows: December (actual) …… $60,000 January ………….. $70,000 February………….. $80,000 March……………. $85,000 April …………….. $55,000 c. Sales are 40% for cash and 60% on credit. Credit sales are collected in the month following sale. The accounts receivable at December 31 are the result of December credit sales. d. Each month’s ending inventory should equal 20% of the following month’s budgeted cost of goods sold. e. One quarter of a month’s inventory purchases is paid for in the month of purchase; the other three quarters is paid for in the following month. The accounts payable at December 31 are the result of December purchases of inventory. f. Monthly expenses are as follows: commissions, $12,000; rent, $1,800; other expenses (excluding depreciation), 8% of sales. Assume that these expenses are paid monthly. Depreciation is $2,400 for the quarter and includes depreciation on new assets acquired during the quarter. g. Equipment will be acquired for cash: $3,000 in January and $8,000 in February. h. Management would like to maintain a minimum cash balance of $5,000 at the end of each month. The company has an agreement with a local bank that allows the company to borrow in increments of $ 1,000 at the beginning of each month, up to a total loan balance of $50,000. The…

Attachments:

new tire retreading equipment acquired at a cost of 140 000 at the beginning of a fi 689095

New tire retreading equipment, acquired at a cost of $140,000 at the beginning of a fiscal year, has an estimated useful life of four years and an estimated residual value of $10,000. The manager requested information regarding the effect of alternative methods on the amount of depreciation expense each year. On the basis of the data presented to the manager, the double declining balance method was selected.

In the first week of the fourth year, the equipment was sold for $23,300.

 

Instructions

1. Determine the annual depreciation expense for each of the estimated four years of use, the accumulated depreciation at the end of each year, and the book value of the equipment at the end of each year by (a) the straight line method and (b) the double declining balance method. The following columnar headings are suggested for each schedule:

Year     Depreciation Expense    Accumulated Depreciation, End of Year    Book value, End of Year

 

2. Illustrate the effects on the accounts and financial statements of the sale.

3. Illustrate the effects on the accounts and financial statements of the sale, assuming a sale price of $15,250 instead of $23,300.

 

2. Data related to the acquisition of timber rights and intangible assets of Gemini Company during the current year ended December 31 are as follows:

a. On December 31, Gemini Company determined that $3,000,000 of goodwill was impaired.

b. Governmental and legal costs of $920,000 were incurred by Gemini Company on June 30 in obtaining a patent with an estimated economic life of 8 years.

Amortization is to be for one half year.

c. Timber rights on a tract of land were purchased for $1,350,000 on March 6.

The stand of timber is estimated at 15,000,000 board feet. During the current year, 3,300,000 board feet of timber were cut and sold.

Instructions

1. Determine the amount of the amortization, depletion, or impairment for the current year for each of the foregoing items.

2. Illustrate the effects on the accounts and financial statements of the adjustments for each item.

you are currently planning the audit of rc pty ltd you have 689114

You are currently planning the audit of RC Pty Ltd. You have calculated the following ratios in order to assist you in identifying potential audit risk areas:

Ratio Unaudited 30/6/X4 Audited 30/6/X3 Industry average
Cost of goods sold/sales (%) 73.27 79.34 69.24
Operating expenses/sales (%) 12.96 16.25 14.81
Selling and Administrative expenses/sales (%) 4.00 4.51 4.90
Interest expense/sales (%) 3.31 3.44 5.23
Total cost/sales (%) 92.99 97.40 93.25
Profit/sales (%) 4.47 3.91 5.15
Inventory turnover (times per year) 1.44 1.95 2.15
Accounts receivable turnover (days) 80.00 95.00 75.00
Current assets/current liabilities 1.37 1.20 1.45
Receivables/current liabilities 0.25 0.25 0.40
Profit/capital (% per year) 13.89 15.81 18.19
Times dividend earned 1.60 2.00 1.80

In addition you have the following information:

  • RC Pty Ltd is a large proprietary company involved in property development and residential construction in the Parramatta region. It experienced losses a few years ago but in recent years has improved its performance and returned to profitability.
  • Current year operating profit has increased by nearly 50% while sales have fallen slightly from last year’s levels.
  • Eight months ago, RC Pty Ltd became involved in a consortium building a large residential project on the out skirts of Parramatta. The project is expected to take 7 years to finish, with completed dwellings being sold in stages ‘off the plan’. Off the plan sales have become increasingly popular with consumers as only a small deposit is required to secure the property, with the balance not payable until completion (often up to 12 months later)
  • In order to finance participation in the residential project, RC Pty Ltd doubled its bank loan.
  • Industry average data was obtained from the Australian Bureau of Statistics, and is calculated using figures from all listed property development companies in Australia.

Your assistant has reviewed the data and made the following notes:

  1. Cost of goods/sales, operating expenses/sales, and selling and admin expenses /sales have all fallen, indicating an improvement in management efficiency. In addition, these ratios are all lower than industry averages, indicating RC Pty Ltd has good cost control measures in place.
  2. The inventory turnover ratio has fallen, indicating that inventory is held for less time than prior years. This should lessen our concerns regarding obsolescence.
  3. Debtors are now being collected 15 days (or 16%) faster than the prior year, reducing cash flow concerns and lessening pressure on the provision for doubtful debts.
  4. Current assets to current liabilities has increased, indicating an improvement in RC Pty Ltd’s liquidity position. In addition, this ratio is above industry average.

Required

  • Examine the validity of your assistant’s comments, and where appropriate provide an alternative interpretation and analysis.
  • Evaluate the implications of your analysis on the year end audit testing.

Adapted from the Professional Year Programme of The Institute of Chartered Accountants in Australia 2000 Accounting 2 Module

Question 2(5 Marks)

Case Scenario

Your audit manager has asked you to do the following:

  • Select a sample of creditors from the year end creditors ledger:
  • Vouch each creditor’s balance to selected invoices and subsequent cash payments: and
  • Agree the total of the creditors’ ledger to the trial balance and general ledger.

Required

She now wants to see if you understand the reasoning behind these activities and asks you to prepare a memo that:

  1. Identifies the audit assertion each of the above procedures is directed towards
  2. Identifies any assertions that you consider should be further tested (do not consider disclosure issues).
  3. In relation to the assertions identified in (2), she asks you to identify additional procedures you would perform to gather sufficient appropriate evidence.

Note: Please refer also to ASA 315(ISA 315) and ASA 500 (ISA 500)

Adapted from the Professional Year Programme of The Institute of Chartered Accountants in Australia 1999 Accounting 2 Module

Question 3(7.5 Marks)

If the auditor concludes that unethical behaviour has occurred, they need to consider whether it is necessary to whistleblow on the offender and, if so to whom to report. Auditors in practice need to be able to make a judgement as to when to whistleblow. Completing the requirement below will help you make that judgement.

Required

Develop a set of guidelines that you would use to determine if it was appropriate to whistleblow in a situation where you considered there may be unethical conduct. Explain in a paragraph the reason for each of the guidelines that you have developed.

This could be set out in a table as below:

Guideline Explanation

Rationale

Content assessed Topics 2, 6, 7, 8, 9, 10

This assessment has been designed to develop your ability to:

  • be able to describe risk management methodologies and the role of internal controls in an audit context
  • be able to explain and apply procedures involved in the audit process
  • be able to exercise critical and reflective judgement and appreciate the value of ethical practice.

Marking criteria

Question 1

Criteria HD/ Distinction 5.6 7.5 Credit 4.9 5.55 Pass 3.75 4.8
Use of the formulae which underpin the information provided by the assistant to check the statements made by the assistant for any errors and if errors found to provide the correct interpretation
(2 Marks)
All errors in assistant’s judgement identified. Correct interpretation provided Most errors in assistant’s judgement identified. Most alternative interpretations correct Demonstration of understanding of some of the formula. Some errors in assessing the assistant’s judgement. Some errors in alternative interpretation
Correlation of correct interpretation with additional information provided
(2 Marks)
All additional information taken into consideration when developing correct interpretation of ratios Most additional information taken into consideration when developing correct interpretation of ratios Some additional information taken into consideration when developing correct interpretation of ratios
Identification of year end audit tests that will need to be completed to minimise audit risk identified through the analysis of the ratios and the additional information (2 Marks) All appropriate year end audit tests identified with a clear statement as to how these will minimise audit risk Many appropriate year end audit tests identified. Evidence of some understanding as to how these will minimise audit risk Some appropriate year end audit tests identified.
Basic understanding of how these will minimise audit risk
Professional communication
(Note: you are required to space between paragraphs; use Arial 10pt or TNR 12 pt; use 1.5 or double line spacing)
(1 Mark)
Work contains distinct understandable statements with no errors.
Extremely well organised.
Content is structured in a manner that facilitates the reader’s understanding.
Minor spelling, grammar and punctuation errors. Work shows evidence of proofreading.
Well structured with one main idea or argument provided per paragraph arguments/ideas.
Some spelling, grammar and punctuation errors found but the work is readable and structured.
Work may include too many ideas in one paragraph.
Appropriate resources including references to the audit standards and correct referencing
(.5 Mark)
Used two or more additional sources in addition to the text and annual reports
All work has been referenced correctly as per APA (6th edn) requirements
Used one additional source in addition to the text and annual reports.
All work has been referenced within the body of the answer and in the reference list, with some omissions or errors in terms
No additional sources used other than text and annual reports.
Others’ work is not always acknowledged and there are a number of errors or non compliance with the APA (6th edn)

Question 2

Criteria HD/Distinction (3.75 5) Credit (3.25 3.7) Pass (2.5 3.2)
Correct identification of audit assertions relevant to the tests undertaken
(.5 Mark)
All assertions relevant to the tests undertaken correctly identified Most assertions relevant to the tests undertaken correctly identified Some assertions relevant to the tests undertaken
Correct identification of additional audit assertions for creditors
(1 Mark)
All additional audit assertions correctly identified Most additional audit assertions correctly identified Some additional audit assertions correctly identified
Identification of all appropriate procedures additional to those already identified by the audit manager. Explanation provided as to how these provide sufficient appropriate evidence
(2 Marks)
All appropriate procedures additional to those identified by the audit manager clearly identified. Clear explanation as to how these provide sufficient appropriate audit evidence Most appropriate procedures additional to those identified by the audit manager clearly identified. Some explanation as to how these provide sufficient appropriate audit evidence Some appropriate procedures additional to those identified by the audit manager clearly identified.
Professional communication using a memo format
(Note: you are required to space between paragraphs; use Arial 10pt or TNR 12 pt; use 1.5 or double line spacing)
(1 Mark)
Work contains distinct understandable statements with no errors.
Extremely well organised.
Content is structured in a memo format in a manner that facilitates the reader’s understanding.
Minor spelling, grammar and punctuation errors. Work shows evidence of proofreading.
Well structured in a memo format with one main idea or argument provided per paragraph arguments/ideas.
Some spelling, grammar and punctuation errors found but the work is readable and in a memo structure.
Work may include too many ideas in one paragraph.
Appropriate resources and correct referencing. References to the auditing standards where appropriate
(.5 Mark)
Used two or more additional sources in addition to the text and annual reports
All work has been referenced correctly as per APA (6th edn) requirements
Used one additional source in addition to the text and annual reports.
All work has been referenced within the body of the answer and in the reference list, with some omissions or errors in terms
No additional sources used other than text and annual reports.
Others’ work is not always acknowledged and there are a number of errors or non compliance with the APA (6th edn)

Question 3

Criteria HD/ Distinction (5.6 7.5) Credit (4.9 5.55) Pass (3.75 4.8)
Identification of guidelines that help user distinguish between general unethical acts and fraud including a clear explanation of the reason for the guidelines
(2 Mark)
Clear and complete set of guidelines and reasons identified.
Lucid explanation of the reason for each of the guidelines
Most of the guidelines which help the user distinguish between general unethical acts and fraud.
Most explanations of the reason for the guidelines good but some not clear.
Some attempt to identify guidelines that help determine course of action when the act is a fraud. Some attempt to explain the reason for the guidelines
Identification of guidelines that help determine course of action when the act is a fraud including a clear explanation of the reason for the guidelines
(2 Marks)
Clear and complete set of guidelines and reasons identified.
All relevant sections of law and accounting standards identified
Most of the guidelines that help determine course of action when the act is a fraud identified. including a clear explanation of the reason for the guidelines
Most relevant sections of law and accounting standards identified
Some attempt to identify guidelines that help determine course of action when the act is a fraud. Some attempt to explain the reason for the guidelines
Some relevant sections of law and accounting standards identified
Identification of guidelines that help determine course of action when the act is a general unethical act and not a fraud including a clear explanation of the reason for the guidelines
(2 Marks)
Clear set of guidelines and reasons identified. Guidelines provide assistance in identifying, if appropriate, the people to whom the unethical act should be reported. Most of the guidelines and reasons identified. Some of the guidelines and reasons identified
Professional communication using a clear system to identify the guideline and the reason for the guideline.
(Note: you are required to space between paragraphs; use Arial 10pt or TNR 12 pt; use 1.5 or double line spacing)
(1 Mark)
Work contains distinct understandable statements with no errors.
Extremely well organised.
Content is structured in a manner that facilitates the reader’s understanding.
Minor spelling, grammar and punctuation errors. Work shows evidence of proofreading.
Well structured with one main idea or argument provided per paragraph arguments/ideas.
Some spelling, grammar and punctuation errors found but the work is readable and structured.
Work may include too many ideas in one paragraph.
Appropriate resources and correct referencing. References to the law and auditing standards where appropriate
(.5 Mark)
Used two or more additional sources in addition to the text and annual reports
All work has been referenced correctly as per APA (6th edn) requirements
Used one additional source in addition to the text and annual reports.
All work has been referenced within the body of the answer and in the reference list, with some omissions or errors in terms
No additional sources used other than text and annual reports.
Others’ work is not always acknowledged and there are a number of errors or non compliance with the APA (6th edn)

Attachments:

please do this auditing memo for 800 words 688150

Background: Mr David Buttoner the owner operator of Buttons by David Pty Ltd. runs a number of button lines. His favourite is a cut stag horn button that has gold filigree. Using a variable costing approach, the revenues and costs per 100 buttons is: Table 1: Stag Horn Button Costs per 100 Buttons
Sales Revenue $200.00 VC of materials — Stag horn $40.00 — Gold wire 50.00 $90.00 VC of labour — Cutting $ 5.00 — Wire inlay 35.00 — Polishing 8.00 48.00 VC of MO/H — Indirect Materials $ 2.00 — Indirect Labour 5.00 — Other MO/H 15.00 22.00 VC of SG&A 5.00 $165.00 Contribution Margin $ 35.00 FC of MO/H $ 100.00 FC of SG&A 25.00 $125.00 Profit/(loss) before Taxes ($90.00)
However, that line is not popular with the customers and generates a substantial loss for the company. The stag horn button inventory has built up over the past year to 30,000 buttons. Required: As the External Auditor and using a standard format memo (i.e. a short report of 1/2 to 1 1/2 pages) please explain to Mr David Buttoner: 1) Whether the staghorn inventory meets the definition of an Inventory (cite the AASB Handbook section), 2) The current full absorption amount of the staghorn button inventory in the Books of Account.
3) Why and by how much the staghorn inventory needs to be written down (please justify the write down; e.g. the Lower of Cost or Market; cite the AASB Handbook section; and show calculations to justify your amount). This is to be answered in your groups of 3 6 students, where each group will submit one report with the names and student numbers of all group members in the heading. The assignment is due 14 May/14, in class.
NB: you need to identify the key question and to reorganise the rest of the memo to resolve that question.

myob 688157

cash flow statements

Document Preview:

MAA261 Financial Accounting ASSIGNMENT TWO Trimester 2, 2014 Part Three Proforma Family name:?First names:??Deakin ID:??Deakin Username:?? SPECIAL INSTRUCTIONS: Question 3 must be completed and submitted on this proforma. Please ensure that you understand the University rules on plagiarism – it must be your own work. Include your answer in sections provided You can create more lines on the journal entry tables etc. by placing the cursor at the end of a row and pressing return. Insert your name and Student ID in the footer. You are NOT to submit a hard copy (online submission only). TONY’S TELEVISIONS AN ACCOUNTING PRACTICE SET Tony’s Televisions Unadjusted Trial Balance As at 30th June 2014 Account Name?DR?CR??Cash at Bank?40,902???Office Supplies on Hand?1,790???Accounts Receivable?69,080???Allowance for Doubtful Debts????Inventory?105,400???Prepaid Advertising?1,200???Prepaid Insurance?4,000???Prepaid Rent?2,200???GST Paid?10,115???Motor Vehicles – at Cost?150,000???Accumulated Depreciation – Motor Vehicles????Furniture & Fittings?6,400???Accumulated Depreciation – Furn & Fittings??640??Office Equipment – as Cost?7,500???Accumulated Depreciation – Office Equipment??350??Accounts Payable??64,020??Telephone Payable????Electricity Payable????Interest Payable??250??GST Collected??17,160??PAYG Payable??900??Wages Payable????Bank Loan??30,000??Capital – T. Tomkins??147,282??Drawings – T. Tomkins?96,000???Income Summary Account????Sales??686,400??Sales Returns and Allowances?10,296???Discounts Allowed?12,355???Profit on Sale of Assets??3,000??Cost of Goods Sold?343,200???Discount Received??6,864??Inventory Losses????Advertising?19,600???Sub Total?880,038?956,866?? Tony’s Televisions Unadjusted Trial Balance Continued As at 30th June 2014 Account Name?DR?CR??Bad & Doubtful Debts Expense????Depreciation Expense – Motor Vehicles?6,000???Depreciation Expense – Furniture & Fittings????Depreciation Expense – Office…

Attachments:

bob ltd is finalising its financial statements for the reporting period 688159

  • Question 1 [15 marks]

Events after the reporting period

Bob Ltd is finalising its financial statements for the reporting period ending 30 June 2014.On 21 July 2014, before the financial statements have been finalised and authorised for issue, the company’s directors became aware of the following situations:

a) 2 July 2014: The directors proposed a dividend of $10,000.

b) 3 July 2014: The directors approved the sale of an off shore agency to another entity for a profit of $30,000.

c) 4 July 2014: The company received an invoice from a supplier for $85,000 for goods delivered in June; the goods were included in closing inventory at an estimated cost of $100,000.

d) 5 July 2014: The company executed a guarantee in favour of the banks for an outstanding loan of $1,000,000 that the bank made to X Ltd, the company’s major supplier, in January of that year; the guarantee was executed because the bank was demanding payment, which would have disrupted inventory supplies.

e) 6 July 2014: An agreement was signed to take over a production facility in Adelaide at a cost of $5,000,000, which will be paid for using a long term finance lease.

f) 7 July 2014: The Australian Taxation Office waived fines for the inclusion of incorrect information in the company’s 2012 income tax return; the adjusted return was reflected in the company’s financial statements and the fine of $30,000 was recognised as an expense and liability at reporting date.

Required:

i) Given that financial statements are prepared for the financial period up to the reporting date, explain why there is a need for a standard that refers to events occurring after the reporting date.

(3 marks)

ii) Explain whether the above events will be classified as either adjusting or non adjusting events after the end of the reporting period (assuming the amount is material), providing reasons for your decision.State the appropriate accounting treatment for each event in Bob Ltd’s 2014 financial statements.

(12 marks)

(Source: Adapted from Deegan, C. (2010). Australian financial accounting. (6th edition) Sydney: McGraw Hill.)

Marking Guide Question 1 Max. marks awarded
i)
Discussion re the need for a standard that refers to events occurring after the reporting date. 3
ii)
Classification and justification/discussion of events 6
Stating the appropriate accounting treatment for each event 6
Total 15

Question 2 [15 marks]

Accounting for share capital

The constitution of Henrietta Sweeney Ltd indicated that the company could issue up to 5,000,000 ordinary shares and 1,000,000 preference shares.Prospectuses had been published offering 1,000,000 preference shares at $1.50 payable in full on application by 31 March 2014, and 2,000,000 ordinary shares at $1.20 with 50% due on application by 31 March 2014, 25% due on allotment, and 25% due on a call to be made by the directors at a later date.

By 31 March 2014, the company had received amounts due on 800,000 of the preference shares and on applications for 2,400,000 ordinary shares.On 15 April 2014, the ordinary and preference shares were allotted.The ordinary shares were allotted to applicants on a pro rata basis and the amounts received in excess of that due were to be credited against amounts due on allotment.The amount due on allotment of the ordinary shares was due by 15 May 2014 and this was received on all shares.

The directors made the call on the ordinary shares on 31 August 2014, with amounts due by 30 September.By this date, amounts due on 1,997,000 ordinary shares had been received.On 15 October 2014, the shares on which call money was not received were forfeited and sold as fully paid.An amount of $0.75 was received for each share sold.Costs of the forfeiture and reissue amounted to $800, and were paid.The constitution does not provide for refund of any balance in the forfeited shares account after reissue to former shareholders.

Required:

Prepare the journal entries to record the transactions of Henrietta Sweeney Ltd up to and including that which took place on 15 October 2014.Show all relevant dates, narrations and workings.

(Source: Adapted from Dagwell, R., Wines, G., Lambert, C. (2012). Corporate Accounting in Australia. (1st edition) Sydney: Pearson Australia.)

Marking Guide Question 2 Max. marks awarded
Journal entries 11.5
Dates 1
Narrations 1
Workings 1.5
Total 15

Question 3 [15 marks]

Accounting for income tax

Twinkle Ltd commences operations on 1 July 2013 and presents its first Statement of Profit or Loss and Other Comprehensive Income, and first Statement of Financial Position on 30 June 2014.The statements are prepared before considering taxation.The following information is available:

Statement of Profit or Loss and Other Comprehensive Income for the year ended 30 June 2014
$ $
Gross profit 420,000
Royalty revenue (exempt income) 30,000
Expenses:
Administration expenses 75,000
Salaries 150,000
Long service leave 15,000
Warranty expenses 20,000
Depreciation expense plant 80,000
Insurance 30,000 370,000
Accounting profit before tax 80,000

Assets and liabilities as disclosed in the Statement of Financial Position as at 30 June 2014
$ $
Assets
Cash 10,000
Inventory 110,000
Accounts receivable 40,000
Prepaid insurance 15,000
Goodwill 20,000
Plant – cost 400,000
Less: accumulated depreciation 80,000 320,000
Total assets 515,000
Liabilities
Accounts payable 35,000
Provision for warranty expenses 10,000
Loan payable 225,000
Provision for long service leave 15,000
Total liabilities 285,000
Net assets 230,000

Other information:

  • All administration and salaries expenses incurred have been paid as at year end.
  • None of the long service leave expense has actually been paid.It is not deductible until it is actually paid.
  • Warranty expenses were accrued and, at year end, actual payments of $10,000 had been made.Deductions are available only when the amounts are paid and not as they are accrued.
  • Actual amounts paid for insurance are allowed as a tax deduction.
  • Amounts received from sales, including those on credit terms, are taxed at the time the sale is made.
  • The plant is depreciated over five years for accounting purposes, but over four years for taxation purposes.
  • The tax rate is 30%.

Required:

i) Determine the balance of any current and deferred tax assets and liabilities (using appropriate worksheets) as at 30 June 2014, in accordance with AASB 112.Show all necessary workings.

(9 marks)

ii) Prepare the journal entries to record the current tax liability and movements in deferred tax assets and liabilities.

(2 marks)

iii) What would your answer for part (a) if the following items on the statement of profit or loss and other comprehensive income were changed: ‘Gross profit’ was $360,000 (instead of $420,000) and the ‘Royalty revenue (exempt income)’ was $90,000 (instead of $30,000).Show all calculations and necessary workings.

(4 marks)

(Source:Adapted from Deegan, C. (2010). Australian financial accounting. (6th edition) Sydney: McGraw Hill.)

Marking Guide Question 3 Max. marks awarded
i)
Determination of taxable income and current tax liability, and workings 4
Determination of deferred tax balances 5
ii)
Journal entries 2
iii)
Determination of impact on current and deferred tax balances, and workings 4
Total 15

Question 4 [15 marks]

Property, plant and equipment

Petersen Ltd has the following land and buildings in its accounts as at 30 June 2014:

$
Residential land, at cost 1,000,000
Factory land, at valuation 2011 900,000
Buildings, at valuation 2010 800,000
Accumulated depreciation (100,000)

At 30 June 2014, the balance of the revaluation surplus is $200,000, of which $100,000 relates to the factory land and $100,000 to the buildings.On this same date, independent valuations of the land and building are obtained.In relation to the above assets, the assessed fair values at 30 June 2014 are:

$
Residential land, previously recorded at cost 1,100,000
Factory land, previously revalued in 2011 700,000
Buildings, previously revalued in 2010 900,000

The company has adopted fair value for the valuation of non current assets.

The company tax rate is 30%.

Required:

i) Prepare journal entries to record the revaluations on 30 June 2014.Petersen Ltd classifies the residential land and factory land as different classes of assets.

(12 marks)

ii) The directors of Petersen Ltd are now concerned about the impact of reporting the decline in the fair value of the factory land in the company’s financial statements.They have now asked you (the company accountant) to use the 2011 valuation for the 2014 financial statements, stating that the decline in value of the factory land is only temporary and will increase again in the near future, after a nearby multi million dollar development is approved.You need to prepare a response to the directors’ request.Provide references to AASB 116 to support your answer.(Word limit: 200 words)

(3 marks)

(Source: Adapted from Deegan, C. (2010). Australian financial accounting. (6th edition) Sydney: McGraw Hill.)

Marking Guide Question 4 Max. marks awarded
i)
Journal entries 12
ii)
Response to directors’ request 3
Total 15

Attachments:

bob ltd is finalising its financial statements for the reporting period ending 30 ju 688160

  • Question 1 [15 marks]

Events after the reporting period

Bob Ltd is finalising its financial statements for the reporting period ending 30 June 2014.On 21 July 2014, before the financial statements have been finalised and authorised for issue, the company’s directors became aware of the following situations:

a) 2 July 2014: The directors proposed a dividend of $10,000.

b) 3 July 2014: The directors approved the sale of an off shore agency to another entity for a profit of $30,000.

c) 4 July 2014: The company received an invoice from a supplier for $85,000 for goods delivered in June; the goods were included in closing inventory at an estimated cost of $100,000.

d) 5 July 2014: The company executed a guarantee in favour of the banks for an outstanding loan of $1,000,000 that the bank made to X Ltd, the company’s major supplier, in January of that year; the guarantee was executed because the bank was demanding payment, which would have disrupted inventory supplies.

e) 6 July 2014: An agreement was signed to take over a production facility in Adelaide at a cost of $5,000,000, which will be paid for using a long term finance lease.

f) 7 July 2014: The Australian Taxation Office waived fines for the inclusion of incorrect information in the company’s 2012 income tax return; the adjusted return was reflected in the company’s financial statements and the fine of $30,000 was recognised as an expense and liability at reporting date.

Required:

i) Given that financial statements are prepared for the financial period up to the reporting date, explain why there is a need for a standard that refers to events occurring after the reporting date.

(3 marks)

ii) Explain whether the above events will be classified as either adjusting or non adjusting events after the end of the reporting period (assuming the amount is material), providing reasons for your decision.State the appropriate accounting treatment for each event in Bob Ltd’s 2014 financial statements.

(12 marks)

(Source: Adapted from Deegan, C. (2010). Australian financial accounting. (6th edition) Sydney: McGraw Hill.)

Marking Guide Question 1 Max. marks awarded
i)
Discussion re the need for a standard that refers to events occurring after the reporting date. 3
ii)
Classification and justification/discussion of events 6
Stating the appropriate accounting treatment for each event 6
Total 15

Question 2 [15 marks]

Accounting for share capital

The constitution of Henrietta Sweeney Ltd indicated that the company could issue up to 5,000,000 ordinary shares and 1,000,000 preference shares.Prospectuses had been published offering 1,000,000 preference shares at $1.50 payable in full on application by 31 March 2014, and 2,000,000 ordinary shares at $1.20 with 50% due on application by 31 March 2014, 25% due on allotment, and 25% due on a call to be made by the directors at a later date.

By 31 March 2014, the company had received amounts due on 800,000 of the preference shares and on applications for 2,400,000 ordinary shares.On 15 April 2014, the ordinary and preference shares were allotted.The ordinary shares were allotted to applicants on a pro rata basis and the amounts received in excess of that due were to be credited against amounts due on allotment.The amount due on allotment of the ordinary shares was due by 15 May 2014 and this was received on all shares.

The directors made the call on the ordinary shares on 31 August 2014, with amounts due by 30 September.By this date, amounts due on 1,997,000 ordinary shares had been received.On 15 October 2014, the shares on which call money was not received were forfeited and sold as fully paid.An amount of $0.75 was received for each share sold.Costs of the forfeiture and reissue amounted to $800, and were paid.The constitution does not provide for refund of any balance in the forfeited shares account after reissue to former shareholders.

Required:

Prepare the journal entries to record the transactions of Henrietta Sweeney Ltd up to and including that which took place on 15 October 2014.Show all relevant dates, narrations and workings.

(Source: Adapted from Dagwell, R., Wines, G., Lambert, C. (2012). Corporate Accounting in Australia. (1st edition) Sydney: Pearson Australia.)

Marking Guide Question 2 Max. marks awarded
Journal entries 11.5
Dates 1
Narrations 1
Workings 1.5
Total 15

Question 3 [15 marks]

Accounting for income tax

Twinkle Ltd commences operations on 1 July 2013 and presents its first Statement of Profit or Loss and Other Comprehensive Income, and first Statement of Financial Position on 30 June 2014.The statements are prepared before considering taxation.The following information is available:

Statement of Profit or Loss and Other Comprehensive Income for the year ended 30 June 2014
$ $
Gross profit 420,000
Royalty revenue (exempt income) 30,000
Expenses:
Administration expenses 75,000
Salaries 150,000
Long service leave 15,000
Warranty expenses 20,000
Depreciation expense plant 80,000
Insurance 30,000 370,000
Accounting profit before tax 80,000

Assets and liabilities as disclosed in the Statement of Financial Position as at 30 June 2014
$ $
Assets
Cash 10,000
Inventory 110,000
Accounts receivable 40,000
Prepaid insurance 15,000
Goodwill 20,000
Plant – cost 400,000
Less: accumulated depreciation 80,000 320,000
Total assets 515,000
Liabilities
Accounts payable 35,000
Provision for warranty expenses 10,000
Loan payable 225,000
Provision for long service leave 15,000
Total liabilities 285,000
Net assets 230,000

Other information:

  • All administration and salaries expenses incurred have been paid as at year end.
  • None of the long service leave expense has actually been paid.It is not deductible until it is actually paid.
  • Warranty expenses were accrued and, at year end, actual payments of $10,000 had been made.Deductions are available only when the amounts are paid and not as they are accrued.
  • Actual amounts paid for insurance are allowed as a tax deduction.
  • Amounts received from sales, including those on credit terms, are taxed at the time the sale is made.
  • The plant is depreciated over five years for accounting purposes, but over four years for taxation purposes.
  • The tax rate is 30%.

Required:

i) Determine the balance of any current and deferred tax assets and liabilities (using appropriate worksheets) as at 30 June 2014, in accordance with AASB 112.Show all necessary workings.

(9 marks)

ii) Prepare the journal entries to record the current tax liability and movements in deferred tax assets and liabilities.

(2 marks)

iii) What would your answer for part (a) if the following items on the statement of profit or loss and other comprehensive income were changed: ‘Gross profit’ was $360,000 (instead of $420,000) and the ‘Royalty revenue (exempt income)’ was $90,000 (instead of $30,000).Show all calculations and necessary workings.

Attachments:

hii i am an accounting student need your help to solve this for me on saasu accounti 688164

hii.. I am an accounting student need your help to solve this for me on SAASU accounting site

Document Preview:

Assignment 2 Questions ACC539 2014 60 UPDATED (Note: the total marks of 35 will be adjusted to a final result of 15 marks) Q 1: Cloud based Accounting (“Saasu”) – practical task (15 marks)?Details of tasks and input data are provided in Resources in Interact.  Q 2: Systems Planning & Implementation (10 marks) (word limit guide – 300 words)?The Queensland Health Payroll System project (2007 2011) has received adverse media attention relating to significant issues and failures during the project. Undertake some research and of this information systems project to undertake the following task:?Using the Systems Development Lifecycle process (figure 19.1 on page 612 of the Romney text) as a guide, identify 5 deficiencies in the Queensland Health Payroll System project, and provide recommendations on how these deficiencies may be overcome in future projects. Structure your answer using headings and subheadings to address each of the issues and recommendations that you identify. Ensure you provide referencing where appropriate. Q 3: Rickard’s Rural – Prepare a Dashboard / Pivot Tables (5 marks) Q 4: Rickard’s Rural – Prepare a 3D Bar Chart (5 marks) Dashboards for Rickard’s Rural, an agricultural supply chain in the Central West of New South Wales Using the data in the Agricultural Sales spreadsheet provided in the Resources section prepare the following Pivot Tables and Pivot Charts. Each Pivot Chart and its related pivot table should be on a worksheet together. Each worksheet tab should be labelled identifying the contents of the worksheet. The initial data should be labelled Data. The sales in total for each product type for the quarter by each type of customer. Use a pie chart to show the percentage of each product type that is sold by cash for the quarter. Copy the results into a word document. Briefly state your findings from looking at the dashboard. (5 Marks) The sales by cash and account for each month for each product type. Use a 3D bar chart to show the…

instructions city of bingham computerized cumulative problem 688193

INTRODUCTION AND USE OF THE CITY OF SMITHVILLE/CITY OF BINGHAM SOFTWARE Welcome Thank you for purchasing the City of Smithville/City of Bingham Governmental Accounting Software. This software is designed to be used with the McGraw Hill/Irwin textbook Accounting for Governmental and Nonprofit Entities, 16 th Edition. System Requirements To use the City of Smithville/City of Bingham accounting software application, you must have the following: ? Operating System: ? Microsoft® Windows 2000 or later ? Mac OS X v10.4 or 10.5 ? Linux Red Hat® Enterprise Linux (RHEL) 5 ? Hardware: ? Windows ®: Intel® Pentium® II 450MHz or faster processor (or equivalent) ? Macintosh: PowerPC® G3 500MHz or faster processor; Intel Core™ Duo 1.33GHz or faster processor ? Recommended screen resolution of 1024 x 768 minimum ? 128MB of RAM minimum ? Spreadsheet program (in order to easily view the exported CSV files) ? Excel 2003 or later (in order to view the exported Excel files) ? Internet Explorer 7 or later, Mozilla Firefox 3.x or later, Safari 3.x or later ? Adobe Flash Player version 10.1 or above ? Adobe Reader 9 or later to read saved report files Special instruction for MAC and Linux users: Users have to manually open the programl file that resides in the Smithville_Bingham folder because start instructor is an .exe file and will run only in Windows environment.

Document Preview:

Instructions City of Bingham Computerized Cumulative Problem For use with McGraw Hill/Irwin Accounting for Governmental & Nonprofit Entities th 16 Edition By Jacqueline L. Reck, Suzanne L. Lowensohn, and Earl R. Wilson 1Table of Contents Chapter 1 Introduction and Use of the City of Smithville/City of Bingham Software …………………………….. 3 Project Instructions and Information ……………………………….. 13 Chapter 2 Opening the Books ……………………………………… 14 Chapter 3 Recording the Annual Budget …………………………. 19 Chapter 4 Recording Operating Transactions the General Fund and Governmental Activities at the Government wide Level ……………………………………. 21 Chapter 5 Recording Capital Assets and Capital Projects Transactions ………………………………………. 27 Chapter 6 Transactions Affecting General Long term Liabilities and Debt Service …………………………… 30 Chapter 7 Recording Transactions Affecting the Enterprise Fund and Business type Activities …………………………………………………………… 34 Chapter 8 Recording Transactions Affecting a Fiduciary Fund—an Investment Trust Fund ……………………… 38 Chapter 9 Adjusting and Closing Entries for Governmental Activities, Government wide Level; Preparation of Government wide and Major Fund Financial Statements …………………………………. 41 Chapter 10 Analysis of Financial Condition ………………………. 43 Chapter 11 Preparation of Audit Report …………………………… 43 Appendix A ………………………………………………………… 44 2CHAPTER 1 INTRODUCTION AND USE OF THE CITY OF SMITHVILLE/CITY OF BINGHAM SOFTWARE Welcome Thank you for purchasing the City of Smithville/City of Bingham Governmental Accounting Software. This software is designed to be used with the McGraw Hill/Irwin textbook Accounting for Governmental and Nonprofit Entities, th 16 Edition. System Requirements To use the City of Smithville/City of Bingham accounting software application, you must have the following: ? Operating System: ? Microsoft® Windows 2000 or later ? Mac OS X v10.4 or 10.5 ?…

port accounting performs two types of services audit and tax port s overhead costs c 688287

Port Accounting performs two types of services, Audit and Tax. Port’s overhead costs consist of computer support, $240,000; and legal support, $120,000. Information on the two services is:

Audit Tax
Direct labor cost $50,000 $100,000
CPU minutes 40,000 10,000
Legal hours used 200 800

Overhead applied to audit services using traditional costing is

for this organisation you may identify ethical issues or aspects from a range of are 688322

Select an organisation that you are either familiar with (you may, or may have, worked there) or have researched. Ideally, choose an organisation within the professional services sector (eg: Financial, Legal, insurance, medical, etc). You may choose to focus on an organization in another industry but you should seek advice from your lecturer first.

For this organisation, you may identify ethical issues or aspects from a range of areas covered in the readings and lecture notes. These include:

1. Ethics at the Individual Level

2. The dynamic of Groups and Ethics in the organisation

3. The Organisational Ethics Construct (How does the organisation ensure ethical behaviours?)

In your response you should:

A. Identify and define the Stakeholders and their stake.

B. Determine the key issues facing management when managing these stakeholders.

C. Evaluate the Stakeholder value management response. This may include:

(i) rational, process and transactional levels

(ii) stakeholder inclusiveness and relationships

(iii) stakeholder power: awareness, knowledge, admiration, action

(iv) monitoring and communicating with stakeholders

(v) winning stakeholder support

(vi) tangible and intangible resources to deliver stakeholder value

D. For the following Stakeholders, identify, discuss and evaluate the organisation’s ethical construct from a range of stakeholders which may include:

(i) Shareholders

(ii) Customers

(iii) Employees

(iv) Suppliers

(v) Community (Publics)

(vi) Determine associated accounting constraints and enabling processes (Hint: Enron insights).

You may decide to take on the role of a consultant and conduct an ‘ethical health check’ of the organisation incorporating the above elements.

Another approach may be to identify a ‘star performing’ organisation or Strategic Business Unit within the organiastion and use it to benchmark the industry.

Make appropriate recommendations based on your findings.

You are required to prepare a report based on the above incorporating relevant theory and additional readings when analysing your chosen company.

Your work will be assessed progressively.

In Week 6 you are to demonstrate an overview, supported by research you have gathered of your chosen organisation. You should search for company and industry reports as well as news and business articles about the company and the industry. You MUST also include at least three academic journal articles sources from the eResources. Failure to do this will result in a penalty of 10% of the available mark.

In Week 10, you will be required to present the class a 5 10 minute (maximum) summary of your progress. This may be in the form of a presentation or simply a discussion within the class. Failure to do this will result in a penalty of 20% of the available mark.

Prepare a six) page report (using standard formatting*) outlining your findings and appropriate recommendations based on the evidence you provide and the conclusions you draw. The six pages do not include Title Page, Table of Contents, Executive Summary, Recommendations or Appendices (if used).

Ensure your report is appropriately reference using the Harvard style of referencing and uploaded on the Turnitin link provided in the Week 11 area.

• The report is due at 18:00 of Week 11 on the day following of your scheduled lecture (upload on moodle turnitin). Team leader only to submit, including details of all team members.

The turnitin link will be provided to help you ensure you avoid plagiarism issues. You should use this to check your work and upload your work as many times as you wish up to the due date after which your submission is locked in.

*Standard Formatting

2.5cm margins, left, right, top and bottom

12pt Arial font

1.5 line spacing

YOU MUST INCLUDE A COVERSHEET WITH YOUR SUBMISSION.

ASSIGNMENTS NOT SUBMITTED WILL BE DEEMED INCOMPLETE AND PENALTIES FOR LATE ASSIGNMENTS WILL APPLY.

ONLY TEAM LEADER TO SUBMIT ASSIGNMENT ENSURING THAT DETAILS OF ALL TEAM MEMEBERS ARE INCLUDED ON THE COVER SHEET

ASSIGNMENTS WHICH ARE SUBMITTED LATE WILL BE SUBJECT TO A LATE PENALTY OF 10% OF THE AVAILABLE MARK FOR EACH DAY THE ASSIGNMENT IS LATE UNLESS PRIOR APPROVAL IS OBTAINED.

Attachments:

love theatre inc owns and operates movie theaters throughout new mexico and utah lov 688329

Love Theatre Inc. owns and operates movie theaters throughout New Mexico and Utah. Love Theatre has declared the following annual dividends over a six year period: 2009, $48,000; 2010, $144,000; 2011, $216,000; 2012, $288,000; 2013, $336,000; and 2014, $432,000. During the entire period ending December 31 of each year, theoutstanding stockof the company was composed of 30,000 shares of cumulative, 4% preferred stock, $100par, and 100,000 shares of common stock, $15 par.

Required:

1. Calculate the total dividends and the per share dividends declared on each class ofstockfor each of the six years. There were no dividendsin arrearson January 1, 2009. Summarize the data in tabular form. If required, round your answers to two decimal places. If the amount is zero, please enter “0”.


2.
Calculate the average annual dividend per share for each class of stock for the six year period. If required, round your answers to two decimal places.

Average annual dividend for preferred

$ per share

Average annual dividend for common

$ per share

you were given an escrow statement your employer just gave you this assignment you a 688336

You were given an escrow statement. Your employer just gave you this assignment. You are still on probation and want to do a good job. They want you to prepare a single, proper journal entry from the attached escrow statement on a property that they purchased. Deposits had been made previously by the client and recorded. The client only wants one journal entry for the purchase. No one else can assist you in the office. Show all work. Use Excel and proper journal entry format. Attach the escrow statement to your journal entry. Put your name on everything that you will turn in .

Document Preview:

Homework – due Sept 22nd You were given an escrow statement.  Your employer just gave you this assignment.  You are still on probation and want to do a good job.  They want you to prepare a single, proper journal entry from the attached escrow statement on a property that they purchased.  Deposits had been made previously by the client and recorded. The client only wants one journal entry for the purchase. No one else can assist you in the office.  Show all work. Use Excel and proper journal entry format. Attach the escrow statement to your journal entry. Put your name on everything that you will turn in .

Attachments:

powersys is an electricity distribution company based in a large capital city its bu 688421

Powersys is an electricity distribution company based in a large capital city. Its business is to manage the electricity assets, including poles, wires and other equipment, that are used to deliver electricity to more than 500 000 retail and business customers in the city. Pole, wire and substation maintenance and improvements are a large part of the company’s operations and teams of highly trained technicians are used for both planned work and emergency response activities. Emergency response is required when storms or fires bring down power lines, the power must be turned off at the direction of police, or the electricity supply fails for any reason.

Each team comprises several vehicles (vans and trucks) and uses additional heavy equipment, such as cherry pickers, cranes and diggers, as required. Each vehicle carries a core set of specialized parts and tools and additional items are obtained as required from the stores, located in a large warehouse in the northern suburbs. The warehouse is staffed on a 24 hour basis to assist night maintenance (designed to minimize disruption to business customers) and emergency response.

Required

(a) Make a list of the potential problems that could occur in Powersys’ maintenance and improvements program. (20 marks)

(b) Suggest ways that good internal control over parts, equipment and labour could help Powersys avoid these problems. (20 marks)

lt p gt post journal entries on ledger accounts 2 prepare a trial balance adjustmen 688430

post journal entries on ledger accounts.2.prepare a trial balance, adjustment , income, and balance statements. month end adjustments: (1) supplies inventory, 3,/31 , 2,100..prepare income statement4. statement retained earnings5. balance sheetprepare an adjusting journal/ entry, and closing journal8. prepare closing trial balance

open the books for the following period

Document Preview:

Caroline Landscaping Service General Journal Date Description Post. Ref. Debit Credit 2013 March. 1 Capital Stock 20,000 Cash 20,000 March. 1 Prepaid Rent 2,100 Cash 2,100 March. 1 Equipment 1,600 Capital Stock 1,600 March. 1 Automobile Expense 2,160 Accounts payable 2,160 March. 1 Equipment 8,500 Cash 4,000 Accounts payable 4,500 March. 1 Accounts receivable 1,225 Fees earned 1225 March. 10 Prepaid insurance 1,800 Cash Prepaid 3 months 1,800 March. 13 Supplies 200 Cash 80 Accounts payable 120 Bought Supplies March. 14 Supplies 2,500 Accounts payable 2,500 March. 15 Accounts receivable 4,845 Fees earned 4,845 Sold services March. 17 Gas and oil equipment 84 Cash 84 March. 19 Cash 2,700 Fees Earned 2,700 March. 22 Accounts Payable 600 Cash 600 Paid Creditors on Acct March. 24 Cash 500 Account receivable 500 Receivables from account March. 28 Accounts receivable 1,626 Fees Earned 1,626 March. 29 utilities expense 200 Accounts payable 200 March. 30 wages expense 3,200 Cash 3,200 Paid salaries March. 31 Withdrew/ dividend 1,300 Cash 1,300 Withdrew for personal use

Attachments:

assume that hyatt magic uses the lifo method of inventory costing 688543

1 UNIVERSITY OF THE WEST INDIESOpenCampusACCT 1003 Intro. toCost& ManagementAccountingWorksheet 2: Inventory Valuation and ControlStudyQuestionsQuestion 1Hyatt Magic carries an inventory of putters and other gulf clubs. Hyatt uses the FIFO method and a perpetual inventory system. Company records indicate the following for a particular line of Hyatt Magic putters:DateItemQuantityUnit CostNovember 1November 6November 8November 17November 30Balance………….Sale………………Purchase………….Sale……………….Sale……………….531045$70$79i)Prepare a perpetual inventory record for the putters using the FIFO method. What amounts would Hyatt report for ending inventory and cost of goods sold?ii)Assume that Hyatt Magic uses the LIFO method of inventory costing, what would be the cost of ending inventory and cost of goods sold?iii)Prepare Hyatt Magic perpetual inventory record assuming the company uses the average cost method. Round average cost per unit to the nearest cent and all other amountsto the nearest dollar.iv)After preparing the FIFO perpetual inventory record in part (i), journalize Hyatt Magic’s November 8 purchase of inventory on account and November 17thcash sale (sale price of each putter was $120).Question 2a)Leather Goods Company began the year with inventory of $50,000 and purchased $250,000 worth of goods during the year. Sales for the year are $500,000, and Leather Goods gross profit percentage is 55% of sales. Compute the estimated cost of ending inventory by the gross profit method.b)See Through Incretails hand blown glass vasesand uses a perpetual inventory system. A statement of their purchases and sales of these vases for the month of June 20X9 is given below.June 1Opening stock of 30 vases valued at a total costof $27,000.June 3Purchased 45 vases at a cost of $980 each.June 5Sold 55 vases at $1,600 each.June 6Purchased 70 vases at $1,200 each but a trade discount of 3% was received.June 10Sold 60 vases for $114,000.June 14Purchased 80 vases at $1,100 each but additionally there was a shipping cost of $200 per vase.June 18Sold 65 vases for $2,300 each.June 23Purchased 75 vases at a total cost of $108,750.June 255 of the vases last sold were returned, as the customer purchased an incorrect quantity.June 27Sold 112 vases for $280,000.June 30An actual count of vases was carried out which revealed that there wereonly 9 hand blown glass vases in the store room.

2 i)Prepare the inventory record for See Through Inc. using the LIFO method.ii)Determine the gross profit earned by See Through Inc. for the period.iii)State the journal entries necessary to record the transactions on June 6 & 18 under the perpetual inventory system periodic inventory systemQuestion 3(i)Electronics Ltdmanufactures air conditioners. It purchases 200,000 units of a particular type of compressor part, CU30, each year at a cost of $64 per unit. Annual carrying cost (for insurance, material handling, breakage etc) per unit is 5% of the unit purchase price. Currently Electronics Ltd places 50 orders for 4,000 units each year and ordering costs per purchase order are $18.a)Calculate the current annual ordering cost.b)Calculate the current annual carrying cost.c)What is the current total stock administration cost?d)Calculate theEOQ.e)What is the minimum ordering cost?f)What is the minimum storage cost?g)What is the minimum total stock administration cost?h)How much savings would Electronics Ltd. make using the EOQ policy, rather than the current policy of purchasing 4,000 units per order. (ii)Maximum usage for Electronics Ltd. For any one week is 1,000 units and the minimum usage 400 units. Suppliers take anywhere from 2 to 4 weeks to deliver supplies after the order is placed. Using the EOQ policy, determine the re order level, minimumlevel & maximum level of inventory.DiscussionQuestions(Kindly prepare these questions before each tutorialsession)Question 1Hobart Sign Company began the month of June with an inventory of 50 signs that cost a totalof $1,500. Hobart purchased and sold merchandise on accountas follows:June 3Purchase ………………………………..60 signs @ $35eachJune 8Sale ……………………………………….100 signs @ $60eachJune 18Purchase ………………………………..90 signs @ $40eachJune28Sale ……………………………………….70 signs @ $70eachHobart uses the FIFO cost method. Cash payments,on account,totaled $5,000. Operating expenses were $2,700; Hobart paid two thirds in cash and accrued the rest as Accounts Payable.i)Prepare a perpetual inventory record, at FIFO cost, for this merchandise.ii)Make journal entries to record the company’s transactionsiii)Prepare an income statement for Hobart Sign Company for the month ended June 30.Question 2a)Matthew Perry a merchandiser recently attended a seminar where he was informed that he should maintain proper stock control records and be able to calculate the various stock levels. Identify these stock levels, their purpose(s) and the factors to be considered in setting them.

Attachments:

prepare a report for your client your report should include a a description of the c 688554

Prepare a report for your client. Your report should include:

a. A description of the core business of the company including full details of its operating activities.

b. A discussion on any significant issues emerging from the Chairman’s Report.

c. A discussion on any significant issues emerging fromthe Director’s Report.

d. A discussion on company’s Corporate Governance Statement.

e. A calculation of the key financial ratios for 2013.

f. An overall evaluation of the company and your recommendation on investing inthe company.

Please note the following:

· Format: Business report

· Contribution to overall assessment: 20%

· Length: 2000 – 2200 words

· Your work must comply with the University’sGeneral Guide for the Presentation of Academic Work.

· Two Useful links.

§ http://federation.edu.au/students/assistance support and services/academic support/learning and study/resources

Document Preview:

BUACC5930 Accounting Concepts and Practices Semester 2, 2014 Group Assignment You are an accountant at West Group Accountants & Investment Advisers. You have been approached by a group of investors for your professional advice on investing in Harvey Norman Holdings Limited. Your client is a strong believer in supporting Australian made products and Socially Responsible Investing. The latter would mean adopting an ? HYPERLINK “http://en.wikipedia.org/wiki/Investment” o “Investment” ?investment? ? HYPERLINK “http://en.wikipedia.org/wiki/Strategy” o “Strategy” ?strategy? which seeks to consider both ? HYPERLINK “http://en.wikipedia.org/wiki/Financial_return” o “Financial return” ?financial return? and ? HYPERLINK “http://en.wikipedia.org/wiki/Social_good” o “Social good” ?social good?. Required: Go to: ? HYPERLINK “http://www.harveynormanholdings.com.au/pdf_files/2013_Annual_Report.pdf” ?http://www.harveynormanholdings.com.au/pdf_files/2013_Annual_Report.pdf? You +1’d this publicly. ? HYPERLINK “http://www.google.com.au/search?q=woolworths&rls=com.microsoft:en au&ie=UTF 8&oe=UTF 8&startIndex=&startPage=1&rlz=1I7ADFA_enAU443&redir_esc=&ei=jniGT6H0FcGZiAfe3eTGBw” ?Undo? and access the company’s annual report for 2013. Prepare a report for your client. Your report should include: A description of the core business of the company including full details of its operating activities. A discussion on any significant issues emerging from the Chairman’s Report. A discussion on any significant issues emerging from the Director’s Report. A discussion on company’s Corporate Governance Statement. A calculation of the key financial ratios for 2013. An overall evaluation of the company and your recommendation on investing in the company. Please note the following: Format: Business report Contribution to overall assessment: 20% Length: 2000 – 2200 words Due date: ?? Your work must comply with the University’s General Guide for the Presentation of Academic Work. Two…

Attachments:

develop a time sheet entry customer account management software system using relatio 688559

Develop a time sheet entry/customer account management software system using relational database software and prepare an associated report detailing the technical and learning issues encountered.

Document Preview:

School of Business BUACC5937: Information Systems Design and Development for Accountants Mudusu Calling Cards System (MCCS) Invoice Learning About Relational Database By using Microsoft Access 2010 Acknowledgements: Microsoft Windows, Microsoft Word, and Microsoft Access are registered trademarks of Microsoft Corporation. Images on the front page are adapted from Microsoft Word clipart gallery.BUACC5937 Assignment 2: Term02 2014 This assessment addresses the following criteria from the course profile: Knowledge ? Understand the principles of data management and relational databases. Skills ? Develop a time sheet entry/customer account management software system using relational database software and prepare an associated report detailing the technical and learning issues encountered. ? Work effectively as a team member. Values ? Appreciate the evolving nature of Accounting Information Systems, and how these are reshaping the practice of Accounting. This assignment is designed to help you to understand how data is stored and information is retrieved in Information Systems. Working together in teams of two, you will develop skills with Microsoft ™ Access and Word. It contributes 20% towards the overall assessment in the unit. It is best if you: ? Read through the entire assignment before you commence work; ? Prepare your report at the same time as you create your software; ? Learn how to capture ‘screen shots’, trim the part you want, and then place these screen shots into a Word document before you begin preparing your report; ? Understand that this assignment is not only about learning MS Access, it is also about the approach to developing a new skill. Relax and enjoy doing this assignment. You can learn a lot. If you make mistakes, that’s great! It is by correcting mistakes, or deleting part of the work already done and re doing it that we learn. In IT attention to detail is most important. This…

Attachments:

kathy and james mohr local golf stars opened the chip shot driving range on march 1 688606

Kathy and James Mohr, local golf stars, opened the Chip Shot Driving Range on March 1, 2014, by investing $25,000 of their cash savings in the business. A caddy shack was constructed for cash at a cost of $8,000, and $800 was spent on golf balls and golf clubs. The Mohrs leased five acres of land at a cost of $1,000 per month and paid the first month’s rent. During the first month, adver¬tising costs totaled $750, of which $150 was unpaid at March 31, and $400 was paid to members of the high school golf team for retrieving golf balls. All revenues from customers were deposited in the company’s bank account. On March 15, Kathy and James withdrew a total of $1,000 in cash for personal living expenses. A $ 100 utility bill was received on March 31 but was not paid. On March 31, the balance in the company’s bank account was $18,900.Kathy and James thought they had a pretty good first month of operations. But, their estimates of profitability ranged from a loss of $6,100 to net income of $2,450.

hi i need help on my accounting homework this is the problem the following trial bal 688747

Hi. I need help on my accounting homework.

This is the problem:

The following trial balance was prepared for Village Cycle Sales and Service on December 31, 2013, after the closing entries were posted:

Debit Credit

Cash $46,200

Accounts Receivable $21,300

Allowance for Doubtful Accounts $1,350

Inventory $85,600

Accounts Payable $28,000

Common Stock $80,000

Retained Earnings $43,750

Total $153,100 $153,100

Village Cycle had the following transaction in 2014:

1. Purchased merchandise on account for $260,000.

2. Sold merchandise that cost $243,000 on account for $340,000.

3. Performed $80,000 of services for cash.

4. Sold merchandise for $60,000 to credit card customers. The merchandise cost $41, 250. The credit card company charges a 5 percent fee.

5. Collected $348,000 cash from accounts receivable.

6. Paid $265,000 cash on accounts payable.

7. Paid $115,000 cash for selling and administrative expenses.

8. Collected cash for the full amount due from the credit card company. (see item 4).

9. Loaned $50,000 to Lee Supply. The note had a 9 percent interest and a one year term to maturity.

10. Wrote off $830 of accounts as uncollectible.

11. Made the following adjusting entries:

(a) Recorded three months’ interest on the note at December 31, 2014 (see item 9)

(b) Estimated uncollectible accounts expense to be .5 percent of sales on account.

Thanks so much for your time.

2 journalize the necessary entries a that increase cash and b that decrease cash the 688754

Bank Reconciliation and Entries

The cash account for Remedy Medical Co. at April 30, 2014, indicated a balance of $18,885. The bank statement indicated a balance of $23,775 on April 30, 2014. Comparing the bank statement and the accompanying canceled checks and memos with the records revealed the following reconciling items:

  1. Checks outstanding totaled $7,840.
  2. A deposit of $3,580, representing receipts of April 30, had been made too late to appear on the bank statement.
  3. The bank collected $3,780 on a note left for collection. The face of note was $3,600.
  4. A check for $770 returned with the statement had been incorrectly recorded by Remedy Medical Co. as $700. The check was for the payment of an obligation to Copelin Co. for a purchase on account.
  5. A check drawn for $330 had been erroneously charged by the bank as $3,300.
  6. Bank service charges for April amounted to $110.

Required:

1.Prepare a bank reconciliation

2.Journalize the necessary entries (a.) that increase cash and (b.) that decrease cash. The accounts have not been closed. For a compound transaction, if an amount box does not require an entry, leave it blank.

3.If a balance sheet were prepared for Remedy Medical Co. on April 30, 2014, what amount should be reported as cash?

 

ayb221 accounting systems technologies 688774

QUEENSLAND UNIVERSITY OF TECHNOLOGY

QUT Business School

School of Accountancy
AYB221 Accounting Systems & Technologies
SEMESTER 2 2014

MYOB Assignment

(Weighting Total – 20%)

Due: 24 September, 2014 at 8pm

Submission: Electronic assignment submission via Blackboard under Assessment & Assignment Minder, both before due date.

Type: Individual

Length: See Submission requirements below This assessment item assesses the following learning outcomes:

Have knowledge and skills pertinent to a particular discipline (KS)

1.2 Incorporate the use of technology appropriate to the discipline and context

This assignment is an individual assignment.
This means that you cannot collaborate with anyone in
any way in
the completion of the MYOB workbook and this assignment.
Submission Requirements:

1. Your final MYOB data file uploaded into the AYB221 Blackboard site via the electronic assignment submission process, and

2. Various MYOB reports and the MYOB assignment criteria sheet submitted through Assignment Minder.

** Please see page 7 of this document for full details regarding all documents required **

Academic staff will not discuss or provide assistance with this assignment in any manner.

The total assignment
will be marked out of 20 and
is worth 20%.

Late Assessment and Extensions

Assessment submitted after the
due date & time without an approved extension will not be marked and will receive a grade of 1 or 0%. If special circumstances prevent you from meeting the assessment due date, you can apply for an extension. If you don’t have an approved extension you should submit the work you have completed by the due date and it will be marked against the assessment criteria.

The Assignment

You will use your MYOB data file from the end of the MYOB Workbook to enter further transactions and some month end balance day adjustments for e Generation. You will then be requested to
produce various financial reports as at the end of
February 2015
.

You
should not
commence this assignment until you
have reviewed and confirmed that your
End of Workbook
file is correct. Any errors carried over into the assignment from the various textbook exercises will be penalised accordingly.

The
fundamental basis of this assignment is the accuracy of accounting information, in particular, the entry of the correct $ amounts to the correct debit and the correct credit accounts for each transaction. The marking for this assignment will be based on the accuracy of the accounting information contained in your submission.

The marking of this MYOB Assignment is based on this real world expectation. Please see the criteria sheet for further details regarding the marking process.

If in the processing of any of these transactions you make an error and you then correct that error so that the year to date balances are correct, then there will be no penalty. Errors also occur in practice and are tolerated if corrected appropriately.

Transactions – February 2015 Transactions for e Generation for the month of February 2015 are detailed below. Buying and selling prices for inventory items and freight charges do not include GST (add the GST) unless otherwise stated. Accept the next purchase order and invoice number displayed.

Date Transaction
2015
Feb
1 Reverse wages & salaries accrued at the end of January ($300.00).
2 Received a cheque for $10403.25 from Pickwick & Partners CPA which has not yet been banked.
2 Sent a purchase order to Manx Ltd for:
10 x Blueberry Playbook (BBP) @ $360.00 each
20 x Blueberry Smartphone (BBS) @ $270.00 each.
3 Received the rent bill (invoice # ER2/15) of $1320.00 including GST, from Glenelg Realty and paid (cheque # 1222).
3 Received a cheque for $869.00 from Seaview Medical Practice which has not yet been banked.
4 Received all the items ordered from Manx Ltd on 30/01/15 together with their invoice # 7020 for $5984.00 including GST. The price charged for the Blueberry Smartphones was $270.00 each, and a freight charge of $40.00 was added.
4 Credit sale to Glenview High School (PO # GHS16029) of 10 x ePad at a special price of $450.00 each.
5 Deposited $11547.25 in the Bank of Adelaide cheque account.
6 Cash sale of 1 x Blueberry Playbook (BBP) and 1 x Smart Cover Plastic (SCP). The customer paid by EFTPOS.
6 Wrote cheque # 1223 to Emma Tomlinson for $602.00, to pay net wages (Gross wages $650 and PAYG Withholding $48).
9 Paid $223.85 to Australian Super with cheque # 1224 for superannuation payable to date.
10 Cash received from sales of the following items (all prices include GST) was banked today.
2 x EPU ePad Camera USB @ $22.00 each
3 x EPS ePod Shuttle @ $55.00 each
1 x BBC Blueberry Charger @ $66.00 each
2 x EAR Earphones @ $22.00 each
11 Received a bill (invoice # 123990) from Express Deliveries ABN 32 123 990 666 for $296.00 including GST for deliveries to customers. (Easy add and use the service layout)

Date Transaction
Feb 12 Purchased staff amenities (tea, milk & biscuits) from Glenelg Supermarket for $22.50. The biscuits were $7.50 including GST the other items (tea & milk) were GST free. This was paid from petty cash.
13 Received all the items ordered from Manx Ltd together with their invoice # 7022 for $9955.00 including GST. The invoice included a freight charge of $55.00 including GST .
16 Cashed cheque # 1225 for $2500.00 to reimburse petty cash (imprest[1] amount $100) with the remainder for the owners drawings.
17 Sent cheque # 1226 for $9548.00 to Visual Corporation in payment of the amount owing to them.
18 Credit sale on terms of net 7 days to Seaside Art Gallery, 29 Marine Terrace, Glenelg SA 5045 (PO # 3373) of:
8 x ePad (EPD)
8 x Smart Cover Leather (SCL)
8 x ePad Power Adapter (EPA)
20 Wrote cheque # 1227 to Emma Tomlinson for $500.00, to pay net wages (Gross wages $545 and PAYG Withholding $45).
20 Paid Manx Ltd with cheque # 1228, the amount owing to them, less prompt payment discount where applicable.
23 Seaside Art Gallery have returned 3 x EPA after discovering that they were not required. Prepare and send them an adjustment note for these items[2].
24 Received and banked a cheque for $6875.00 from Mt Gilmore Mining Corp.
25 Credit sale to Mt Gilmore Mining Corp (PO # 19103) of 10 x Blueberry Smartphone (BBS) @ $310.00 each. Add a freight charge of $40.00.
26 Received the electricity bill from Eagle Power for $692.80 including GST. This was paid today with cheque # 1229.
27 Glenelg Glass repaired a broken window in the shop. Their invoice #7966 for $396.00 including GST was paid today with cheque # 1230.
28 Cash sales (including GST) for:
1 x ePod Classic (EPC) @ $297.00 each
2 x ePod Shuttle @ $55.00 each
were paid by EFTPOS.

Record end of month general journal entries (no GST in these transactions unless stated)
Date Transaction
2015
Feb 28 Prepaid insurance consumed for the month is $300.00.
Wages payable at the end of the month are $280.00.
Record the amount of superannuation payable for the month: $110.55.
Record depreciation & amortisation expense for the month.

  • Furniture & Equipment: $196
  • Leasehold Improvements: $120
Interest charged on the bank loan for the month was $113.50 (GST free)

The Bank of Adelaide cheque account statement for February is shown below. Record any missing transactions and prepare a bank reconciliation statement.

Bank of Adelaide
ABN 25 289 475 123
e Generation BSB No: 610 110
720 Anzac Highway Account No: 346790
Glenelg SA 5045 Page: 1 of 1
Statement Date: 28 February 2015
Debit Credit Balance
1/02/15 Opening balance 33 907.66 Cr
2/02/15 Bank fees 9.70
Cheque 1220 12 390.00 21 507.96 Cr
5/02/15 Deposit 11 547.25
Cheque 1222 1 320.00 31 735.21 Cr
6/02/15 Deposit 610.50
Cheque 1223 602.00 31 743.71 Cr
10/02/15 Deposit 319.00
Cheque 1224 223.85 31 838.86 Cr
16/02/15 Cheque 1225 2 500.00 29 338.86 Cr
20/02/15 Cheque 1227 500.00 28 838.86 Cr
23/02/15 Cheque 1228 15 739.90 13 098.96 Cr
24/02/15 Deposit 6 875.00 19 973.96 Cr
26/02/15 Cheque 1226 9 548.00 10 425.96 Cr
27/02/15 Cheque 1229 692.80
Loan Repayment 1 000.00 8 733.16 Cr

Record the transactions for February and prepare the end of month bank reconciliation.

MYOB Assignment – Assignment Submission Processes:

You are required to
submit the following before 8pm on 24 September 2014

Step #1
: A copy of your final MYOB data file through the Electronic Assignment Submission process on the AYB221 Blackboard site.

When you upload this file at the submission point you will be agreeing that:

The file submitted is your own work and that you have not colluded or collaborated in the completion of this assignment, nor have you provided any material or assistance in the completion of another’s assignment.

Th
is file should be the
final .MYO
data file
that represents the reports that you are
submit
t
ing
for this assignment
.
Please note that you will be penalised for a submitting an incorrect file and also penalised if you do not submit a file at all.

Step #2
: The various documents listed below through the Assignment Minder System:

  1. The following reports as at 28 February 2015 in the order specified below, all stapled together in the top left hand corner:
  1. Trial Balance (do not print zero balances or account numbers)
  2. Audit Report from 1/1/2010 until 1/12/2015

Select Reportsè Accountsè Audit Trailè Audit Trail Report è and select Customise to enter the date range è Print

  1. Reconciliation Report for ONLY the bank account
  2. Aged Receivables (Detail report)
  3. Aged Payables (Detail report)
  4. Balance Sheet (Standard report)
  5. Profit & Loss report for the month of February 2015 including year to date figures

Please note that you if you do not submit any of the above reports or they are not stapled in the correct order, your assignment will be considered incomplete and you will be penalised accordingly.

You will receive zero marks if you don’t submit the audit report, or if the audit was not turned on at the start of the workbook or has been turned off at any time or you don’t do the complete assignment under your MYOB User Id. You will also receive zero marks if you don’t submit your .MYO file electronically

Submission of plagiarised reports and/or contributed to the plagiarism of another student will be considered academic misconduct and reported accordingly to the university.

  1. The MYOB Assignment Assessment Criteria Sheet which is available from the MYOB Assignment folder on the AYB221 Blackboard site.

Students will need to write their name and student number in the space provided at the top of this document and staple this sheet to the front of the MYOB reports.

Hints to maximise your assignment mark

A good check is to do a session report every time you exit MYOB and then compare the journals in the session report with the transaction list on Page 3&4 of this document.

OR if you really want to be sure, do the full assignment manually and compare with the MYOB reports and identify and fix mistakes.

Make Sure:

  • All of the Dr and Cr balances in the Trial Balance are on the correct side, ie Assets Dr balance, Liabilities Cr balance, etc. and they look reasonable.
  • The Bank Reconciliation process has been fully completed and that it correctly summarises the Cash at Bank ledger balance, all relevant reconciliation adjustments, and that the calculated “Expected Balance on Statement” actually matches the Ending Balance on the Bank Statement.

[1] Imprest means bring the Petty Cash balance up to $100 [2] The sales return is a similar approach to the purchase return

Attachments:

hi 5001 assessment 2 marking sheet trimester 2 2014 685245

HI 5001 Assessment 2 Marking sheet – Trimester 2_2014 Student ID: Student Name: Max Marks 30 Assessor’s Comments: PART Assessment criteria Assessable Mark Actual Mark Total a. What is the interest expense for 2013 2 b. How much equipment was purchased during the year? 2 c. What was the depreciation expense for 2013? 2 d. Were any share issues? If any, calculate the value. 2 e. How much in dividend was paid during the year 20123 2 f. How much cash was received from customers during the year? 2 g. How much was paid in tax? 2 14 Part B Marks Scenario 1 a) Prepare the income statement of Deagan Lawn Service for the four months 2 b) Prepare the classified balance sheet of Deagan Lawn Service at 31 August 2 c) Was Deagan’s summer work successful? Give the reason for your answer 1 Scenario 2 Whether the Report Address the following? a) Will you use the cash basis or the accrual basis of accounting? Give a complete description of your reasoning. 2 b) How often do you want financial statements? Why? Discuss how you will use each financial statement 2 c) What kind of revenue will you earn? When will you record it as revenue? How will you decide when to record the revenue? 1 Scenario 3 On behalf of Brent and Den write a loan request include the followings: The request should specify all the details of Brent’s and Den’s plan that will motivate the bank to grant the loan. Include a budgeted income statement for the first six months of the copy centre’s operation. 6 TOTAL 16 Net Total 30%

Document Preview:

HI 5001 Assessment 2 Marking sheet – Trimester 2_2014 Student ID: Student Name: Max Marks 30 Assessable Actual Total Assessment criteria PART Mark Mark What is the interest expense for 2013 a. 2 How much equipment was purchased during the year? b. 2 What was the depreciation expense for 2013? c. 2 Were any share issues? If any, calculate the value. d. 2 How much in dividend was paid during the year e. 2 20123 How much cash was received from customers during f. 2 the year? How much was paid in tax? g. 2 14 Part B Marks Scenario 1 Prepare the income statement of Deagan Lawn a) 2 Service for the four months Prepare the classified balance sheet of Deagan Lawn b) 2 Service at 31 August Was Deagan’s summer work successful? Give the c) 1 reason for your answer Whether the Report Address the following? Scenario 2 Will you use the cash basis or the accrual basis of accounting? Give a complete description of your a) reasoning. 2 How often do you want financial statements? Why? Discuss how you will use each financial b) 2 statement What kind of revenue will you earn? When will you record it as revenue? How will you decide c) when to record the revenue? 1 On behalf of Brent and Den write a loan request Scenario 3 include the followings: The request should specify all the details of Brent’s and Den’s plan that will motivate the bank to grant the loan. Include a budgeted income statement for the first 6 six months of the copy centre’s operation. TOTAL 16 Net Total 30% Assessor’s Comments: 1 HI5001 Group Assignment Marking Criteria T2_2014

Attachments:

lena gold began a professional practice on june 1 and plans to prepare financial sta 685311

Lena Gold began a professional practice on June 1 and plans to prepare financial statements at the end of each month. During June, Gold (the owner) completed these transactions:

a. Owner invested $50,000 cash in the company along with equipment that had a $10,000 market value.

b. The company paid $1,600 cash for rent of office space for the month.

c. The company purchased $12,000 of additional equipment on credit (payment due within 30 days).

d. The company completed work for a client and immediately collected the $2,000 cash earned.

e. The company completed work for a client and sent a bill for $7,000 to be received within 30 days.

f. The company purchased additional equipment for $8,000 cash.

g. The company paid an assistant $2,400 cash as wages for the month.

h. The company collected $5,000 cash as a partial payment for the amount owed by the client in transaction e.

i. The company paid $12,000 cash to settle the liability created in transaction c.

j. Owner withdrew $500 cash from the company for personal use.

Required

Create a table, using the following headings for columns: Cash; Accounts Receivable; Equipment; Accounts Payable; L. Gold, Capital; L. Gold, Withdrawals; Revenues; and Expenses. Then use additions and subtractions to show the effects of the transactions on individual items of the accounting equation. Show new balances after each transaction.

ken age 31 and amy age 28 booth have brought you the following information regarding 685336

Ken (age 31) and Amy (age 28) Booth have brought you the following information regarding thei income, expenses, and withholding for the year. They are unsure which of these items must be used to calculate taxable income. Income: Ken’s salary $27,200 Amy’s wages 18,200 Insurance reimbursement for repairs from and auto accident 500 Gift from uncle George 2000 Interest income from Lodge state bank 640 Federal income taxes withheld: from ken’s salary 1200 from amy’s wage 650 Amy owns and operates a computer bookstore named “The Disk Drive” The store is located at 2000 park ave, Bellview, ME 04104. During 2012, Amy had the following income and expenses: sales of books $320,400 Expenses: Store rental 15000 office expense 6000 advertising 14000 city business license 1000 payroll 89000 payroll taxes 9100 utilities 8000 other 3000 Inventory: Jan. 1 250,000 Dec. 31 235,000 purchases during the year 165,000 last years amy’s first year of operating the booksotre. amy and ken elected to carry forward a $5000 net operating loss from the first year of business into 2012. This year amy loaned a friend $20,000 so that he could make an investment. instead of making the investment the friend lost all of the money in gambling and left for parts unknown. amy has no hope of ever collecting on this bad debt. Ken won $25,000 at a casino birthday party for a friend. this amount should be reported on the other income line. The booths support kens parents Rod (social security # 124809050) and Mary (social security # 489376676) Booth who live in their own home. ken and amy live at 2345 park ave. bellview ME 04104 and their social security numbers are ken 343753456 and amy 123457890 Complete the federal tax return for 2012. use form 1040, schedule c, schedule d, and form 8949.

comperhensive budgeting activity based costing propleme operating and financial budg 685338

comperhensive budgeting , activity based costing propleme, operating and financial budgets: Tyva makes a very popular undyed cloth sandal in one style, but in Regular and Deluxe. The Regular sandals have cloth sols and the Deluxs sandls have cloth coverd wooden soles……….Please see the ttached PDF for the full question case number 12 36 and the excel sheet.

Document Preview:

5000 2000 3000 18 5.25 1.3 1.5 120 195 1.8 7.5 0 2 400 600 1 1.35 250 650 15 5 7 23250 92625 2 3 0.08 15 610 800 50 386 295 40 3219 6060 2000 120 240000 3000 195 585000 18 43 2 1548 2150 1.3 0 2795 0 825000 18 59 3 3186 2950 1.5 2 4425 5900 1.8 31400 56520 7220 5900 1.35 5100 6885 68139 3219 6060 2000 3000 2150 50 43 400 600 2950 50 59 7220 610 6610 5.25 34702.5 2400 3600 5100 5900 800 5100 7.5 38250 250 650 37921.5 44310 82231.5 2150 2950 7220 5900 386 295 7606 6195 2150 5 10750 15 161250 610 800 2950 7 20650 15 309750 6996 5395 31400 471000 6996 5.25 36729 5395 7.5 40462.5 36729 40462.5 77191.5 5.25 1.3 6.8250000000000002 1.5 7.875 7.5 0 0 2 15 15 5 75 7 105 23250 92625 115875 18 0.04 0.72 0.06 1.08 82231.5 1.8 5 9 7 12.6 471000 1.35 1 1.35 1 1.35 68139 92.894999999999996 142.905 621370.5 737245.5 2 50 0.04 122906 3 50 0.06 614339.5 386 5.25 2026.5 825000 0.08 66000 295 7.5 2212.5 4239 5000 40 15 1875 400 92.894999999999996 37160 600 142.905 85746 122906 67875 127145 9435 0.6 825000 495000 0.38 495000 0.02 825000 16500 495000 504435 0.8 77192 0.2 0 471000 0 0 68139 0.7 471000 67875 0.9 750 32685 150000 0.06 12 750 825000 1 0 825000 1 16500 750 17250 2.0909090909090908E 2 807750 0.97909090909090912 32685 4.5231191795198807E 2 0 825000 0.4 16500 16500 2.2833552535437673E 2 0 0 870000 136335 163564.20000000001 706435.8 0.97760236074023887 68139 0.3 20441.7 722620.8 1 67875 0.1 6787.5 163564.20000000001 0 0 77192 0.2 750 1.0378887516108034E 3 150000 0.20757775032216066 0 1506654 2.0849856522258974 698904 0 807750 1506654 1657404 2.2936012912996691 Input Prices Cloth Wood per yard per board foot Direct materials: Direct manufacturing labor: yards Regular Deluxe hours Setup hours per batch Input Quantities per Unit of Output (per pair of sandals) board ft Beginning inventory Target ending inventory Cost of…

1 the management of baggerly corporation would like to investigate the possibility o 685385

1.The management of Baggerly Corporation would like to investigate the possibility of basing its

predetermined overhead rate on activity at capacity. The company’s controller has provided an example to

illustrate how this new system would work. In this example, the allocation base is machine hours, and the

estimated amount of the allocation base for the upcoming year is 81,000 machine hours. In addition,

capacity is 95,000 machine hours, and the actual level of activity for the year is 84,900 machine hours. All

of the manufacturing overhead is fixed and is $6,617,700 per year. For simplicity, it’s assumed that this is

the estimated manufacturing overhead for the year as well as the manufacturing overhead at capacity. It’s

further assumed that this is also the actual amount of manufacturing overhead for the year. If the company

bases its predetermined overhead rate on capacity, by how much was manufacturing overhead

underapplied or overapplied?

A.$318,630 underapplied

B.$703,566 overapplied

C.$703,566 underapplied

D.$318,630 overapplied

Use the following information to answer this question.

The following data (in thousands of dollars) have been taken from the accounting records of Karlana

Corporation for the just completed year.

Sales $910

Raw materials, inventory, beginning $80

Raw materials, inventory, ending $20

Purchases of raw materials $100

Direct labor $130

Manufacturing overhead $200

Administrative expenses $160

Selling expenses $140

Work in process inventory, beginning $40

Work in process inventory, ending $10

Finished goods inventory, beginning $130

Finished goods inventory, ending $150

2.The cost of the raw materials used in production during the year (in thousands of dollars) was

A.$40.

B.$120.

C.$160.

D.$180.

3.When would the direct method and the step down method of service department cost allocation result

in identical allocations being made to the operating departments?

A.The only time is when all costs in the service departments are fixed costs.

B.That can happen if there is only one service department or, if the company has more than one service department, if all the

costs in those departments are fixed costs.

C.The only time is when there is just one service department.

D.That can happen only if there’s an equal amount of service departments and operating departments.

4.Assume there’s no beginning work in process inventory and that the ending work in process inventory

is 100% complete with respect to materials costs. The number of equivalent units with respect to materials

costs under the weighted average method is

A.less than the number of units put into production.

B.less than the number of units completed.

C.the same as the number of units put into production.

D.the same as the number of units completed.

Use the following information to answer this question.

Abis Corporation uses the weighted average method in its process costing system. This month, the

beginning inventory in the first processing department consisted of 800 units. The costs and percentage

completion of these units in beginning inventory were

A total of 9,200 units were started, and 8,200 units were transferred to the second processing department

during the month. The following costs were incurred in the first processing department during the month:

The ending inventory was 80% complete with respect to materials and 20% complete with respect to

conversion costs.

Note:Your answers may differ from those offered below due to rounding error. In all cases, select the

answer that’s the closest to the answer you computed. To reduce rounding error, carry out all

computations to at least three decimal places.

Cost Percent Complete

Material costs $6,000 50%

Conversion costs $9,900 30%

Cost

Material costs $113,900

Conversion costs $322,500

5.The cost per equivalent unit for materials for the month in the first processing department isclosestto

A.$12.44.

B.$11.99.

C.$11.82.

D.$11.39.

Use the following information to answer this question.

Abis Corporation uses the weighted average method in its process costing system. This month, the

beginning inventory in the first processing department consisted of 800 units. The costs and percentage

completion of these units in beginning inventory were

A total of 9,200 units were started, and 8,200 units were transferred to the second processing department

during the month. The following costs were incurred in the first processing department during the month:

The ending inventory was 80% complete with respect to materials and 20% complete with respect to

conversion costs.

Note:Your answers may differ from those offered below due to rounding error. In all cases, select the

answer that’s the closest to the answer you computed. To reduce rounding error, carry out all

computations to at least three decimal places.

Cost Percent Complete

Material costs $6,000 50%

Conversion costs $9,900 30%

Cost

Material costs $113,900

Conversion costs $322,500

6.The cost per equivalent unit for conversion costs for the first department for the month isclosestto

A.$33.24.

B.$38.83.

C.$40.77.

D.$37.68.

7.Which situationalwaysresults in underapplied overhead?

A.Estimated overhead is greater than actual overhead.

B.Actual overhead is less than applied overhead.

C.Actual overhead is greater than applied overhead.

D.Estimated overhead is less than actual overhead.

8.Dewey Company uses the weighted average method in its process costing system. The first processing

department, the Welding Department, started the month with 15,000 units in its beginning work in process

inventory that were 20% complete with respect to conversion costs. The conversion cost in this beginning

work in process inventory was $19,200. An additional 86,000 units were started into production during

the month. There were 13,000 units in the ending work in process inventory of the Welding Department

that were 60% complete with respect to conversion costs. A total of $575,360 in conversion costs were

incurred in the department during the month.

The cost per equivalent unit for conversion costs isclosestto

A.$6.690.

B.$6.206.

C.$6.400.

D.$5.812.

9.Malaviya Corporation uses the FIFO method in its process costing system. Operating data for the

Casting Department for the month of September appear below:

According to the company’s records, the conversion cost in the beginning work in process inventory was

$63,104 at the beginning of September. Additional conversion costs of $654,240 were incurred in the

department during the month. What would be the cost per equivalent unit for conversion costs for

September? (Round off to three decimal places.)

Units

Percent Complete

with Respect to

Conversion

Beginning work in process inventory 17,000 40%

Transferred in from the prior department

during September 72,000

Ending work in process inventory 18,000 30%

A.$9.400

B.$8.060

C.$9.280

D.$9.087

Use the following information to answer this question.

The following data (in thousands of dollars) have been taken from the accounting records of Karlana

Corporation for the just completed year.

Sales $910

Raw materials, inventory, beginning $80

Raw materials, inventory, ending $20

Purchases of raw materials $100

Direct labor $130

Manufacturing overhead $200

Administrative expenses $160

Selling expenses $140

Work in process inventory, beginning $40

Work in process inventory, ending $10

Finished goods inventory, beginning $130

Finished goods inventory, ending $150

10.The net operating income for the year (in thousands of dollars) was

A.$40.

B.$410.

C.$180.

D.$110.

11.Becky works on the assembly line of a manufacturing company where she installs a component part

for one of the company’s products. She’s paid $16 per hour for regular time, and time and a half for all

work in excess of 40 hours per week. Becky’s employer offers fringe benefits that cost the company $3

for each hour of employee time (both regular and overtime). During a given week, Becky works 42 hours

but is idle for 3 hours due to material shortages. The company treats all fringe benefits relating to direct

labor as added direct labor cost and the remainder as part of manufacturing overhead. The allocation of

Becky’s wages and fringe benefits for the week between direct labor cost and manufacturing overhead

would be which of the following?

A.Direct Labor:$741/ Manufacturing Overhead:$73

B.Direct Labor:$672/ Manufacturing Overhead:$142

C.Direct Labor:$688/ Manufacturing Overhead:$126

D.Direct Labor:$624/ Manufacturing Overhead:$190

Use the following information to answer this question.

The following data (in thousands of dollars) have been taken from the accounting records of Karlana

Corporation for the just completed year.

Sales $910

Raw materials, inventory, beginning $80

Raw materials, inventory, ending $20

Purchases of raw materials $100

Direct labor $130

Manufacturing overhead $200

Administrative expenses $160

Selling expenses $140

Work in process inventory, beginning $40

Work in process inventory, ending $10

Finished goods inventory, beginning $130

Finished goods inventory, ending $150

12.The cost of goods sold for the year (in thousands of dollars) was

A.$500.

B.$670.

C.$650.

D.$540.

13.Which of the following isnotone of the five steps in the lean thinking model discussed in the text?

A.Automate the business process.

B.Organize work arrangements around the flow of the business process.

C.Identify the business process that delivers value.

D.Create a pull system that responds to customer orders.

14.Assume there’s no beginning work in process inventory and the ending work in process inventory is

70% complete with respect to conversion costs. Under the weighted average method, the number of

equivalent units of production with respect to conversion costs would be

A.the same as the units completed.

B.the same as the units started during the period.

C.less than the units completed.

D.less than the units started during the period.

Use the following information to answer this question.

The Lee Company uses a job order costing system. The following data were recorded for June:

Overhead is charged to production at 80% of direct materials cost. Jobs 235, 237, and 238 were

completed during June and transferred to finished goods. Jobs 235 and 238 have been delivered to

customers.

Added During June

Job

Number

June 1

Work in

Process

Inventory

Direct

Materials

Direct

Labor

235 $2,500 $600 $400

236 $1,500 $800 $1,000

237 $1,000 $1,200 $1,750

238 $800 $1,500 $2,250

15.Lee Company’s work in process inventory balance on June 30 was

A.$4,100.

B.$9,450.

C.$3,940.

D.$3,300.

Use the following information to answer this question.

The following cost data pertain to the operations of Brentwood Store, Inc., for the month of December.

Corporate legal office salaries $74,000

Shoe Department cost of sales,

Brentwood Store

$35,000

Corporate headquarters building lease $78,000

Store manager’s salary

Brentwood Store

$14,000

Shoe Department sales commissions,

Brentwood Store

$5,000

Store utilities,

Brentwood Store

$14,000

Shoe Department manager’s salary,

Brentwood Store

$3,000

Central warehouse lease cost $10,000

Janitorial costs, Brentwood Store $8,000

The Brentwood Store is just one of many stores owned and operated by the company. The Shoe

Department is one of many departments at the Brentwood Store. The central warehouse serves all

of the company’s stores.

16.What is thetotalamount of the costs listed above that are direct costs of the Shoe Department?

A.$79,000

B.$35,000

C.$40,000

D.$43,000

Use the following information to answer this question.

The following data (in thousands of dollars) have been taken from the accounting records of Karlana

Corporation for the just completed year.

Sales $910

Raw materials, inventory, beginning $80

Raw materials, inventory, ending $20

Purchases of raw materials $100

Direct labor $130

Manufacturing overhead $200

Administrative expenses $160

Selling expenses $140

Work in process inventory, beginning $40

Work in process inventory, ending $10

Finished goods inventory, beginning $130

Finished goods inventory, ending $150

17.The cost of goods manufactured (finished) for the year (in thousands of dollars) was

A.$460.

B.$500.

C.$530.

D.$520.

18.Which person would occupy a line position in a department store?

I. Sales manager

II. Manager, furniture department

III. Manager, advertising department

IV. Manager, personnel department

A.Only I, II, III

B.Only I and II

C.I, II, III, IV

D.Only I

Use the following information to answer this question.

The following cost data pertain to the operations of Brentwood Stores., for the month of December.

Corporate legal office salaries $74,000

Shoe Department cost of sales,

Brentwood Store

$35,000

Corporate headquarters building lease $78,000

Store manager’s salary

Brentwood Store

$14,000

Shoe Department sales commissions,

Brentwood Store

$5,000

Store utilities,

Brentwood Store

$14,000

Shoe Department manager’s salary,

Brentwood Store

$3,000

Central warehouse lease cost $10,000

Janitorial costs, Brentwood Store $8,000

The Brentwood Store is just one of many stores owned and operated by the company. The Shoe

Department is one of many departments at the Brentwood Store. The central warehouse serves all of

the company’s stores.

19.What is thetotalamount of the costs listed above that are not direct costs of the Brentwood

Store?

A.$43,000

B.$36,000

C.$162,000

D.$78,000

Use the following information to answer this question.

Abis Corporation uses the weighted average method in its process costing system. This month, the

beginning inventory in the first processing department consisted of 800 units. The costs and

percentage completion of these units in beginning inventory were

Cost Percent Complete

Material costs $6,000 50%

Conversion costs $9,900 30%

acc200 introduction 10 management accounting pe 685540

ACC200 INTRODUCTION 10 MANAGEMENT ACCOUNTING

‘Pe

CASSIGNMENT worth 25% 1

You have just been hired as an accountant by New Wave Furniture, a manufacturer of specialised furniture based in South Australia.The furniture is high quality and is priced accordingly The owner of the company Bradley Price, believes in pricing his products based on how his larger competitors have priced their products as he believes his firm produces furniture which is as good ,if not better than the mass producers of furniture

When you arrived at work on your first day you learned that although the company has been in existence for eight years ,it has never had a full time accountant .The accounts had been prepared by Joanne Day, the secretary of Bradley Price with Tim Smythe a part time accountant who would come in once or twice a month to help prepare the accounts.Tim has informed Bradley that he could no longer spare the time to come in and so he has suggested that the firm needs a full time accountant which is why you have been hired.Bradley Price however is still not convinced of the need for a full time accountant .”Look why do we need a full time accountant ? At the end of the day we just total up the revenues ,total up the expenses and the difference is the profit .Do I need to understand my product costs ?Why? I cannot lower my prices if my product costs are lower as I just follow the major retailers in the market ,Hardly Normal and King Lee , and price my products based on their prices’

Joanne who has been the secretary cum bookkeeper since the company started has prepared some information for you .Trying to be helpful she has listed the accounts in alphabetical order but she has admitted to not knowing a much about accounting but says “Tim ,the part time accountant ,has said that we need a Cost of Goods Manufactured Schedule and a Cost of Goods Sold Schedule .1 have last years accounts for you so you can prepare these schedules for Bradley”.The list of accounts in alphabetical order is in Appendix A

After four days into your work there was a major fire overnight in the main office that stored the accounts .The manufacturing facility was not affected and work could continue but most of the information about the current years accounts has been damaged and only part of the information is readable.This is shown in Appendix B .Your work on last years accounts has not been affected as you had the information available already however the information about the current month has been affected and not all of it is available.

definition of breakeven analysis is also known as cost volume profit analysis 685552

Costing: Breakeven Analysis Part I: Background 1.1 Definition of Breakeven Analysis is also known as cost volume profit analysis. 3) It is the study of the relationship between selling prices, sales volumes, fixed costs, variable costs and profits at various levels of activity. 1.2 Application of breakeven analysis 3) Breakeven analysis can be used to determine a company’s breakeven point (BEP). CP Breakeven point is a level of activity at which the total revenue is equal to the total costs. At this level, the company makes no profit. With reference to the breakeven point, the managers can set their sales goals and target profits. 1.3 Assumption of breakeven analysis The model of breakeven analysis is developed on the following assumptions: ¦ Relevance range—It is assumed that a company is operating within a relevant range. The relevant range is the range of an activity over which the fixed cost will remain fixed in total and the variable cost per unit will remain constant. ¦ Fixed costs—Total fixed costs are assumed to be constant in total. Fixed costs per unit will decrease with the increasing number of units produced. ¦ Variable costs—Variable costs per unit are assumed to be constant. Total variable costs will increase with the increasing number of units produced. ¦ Sales revenue —Sales revenue per unit is assumed to be constant and the total revenue will increase with the increasing number of units produced. 3) These assumptions are illustrated in the following diagrams:

do you believe that the iasb always act in an independent manner 685580

Assessment item 3

Length: 2000 words

Content assessed: Accounting regulation and politics and standard setting process

Key generic skills: Research, critical thinking and communication.

Subject Learning Outcome for Question 1: Be able to understand and explain the process of standards setting process and existing doubt on independence and faithful representation of the process.

Question 1 Short report

10 marks ( 1000 words excluding table of content and references)

Refer to the article Cortese, C. (2013). Politicization of the international accounting standard setting process: evidence from the extractive industries. Journal of New Business Ideas & Trends 11(2), pp. 48 57

Do you believe that the IASB always act in an ‘independent’ manner. Further, can the ‘engagement with powerful stakeholders around the world’ have potential implications for claims that IFRS are developed to provide financial information that is representationally faithful? Discuss this statement referring to the Cortese, C. 2013 article.

Subject Learning Outcome for Question 2: Be able to critically analyse complex issues in theory development based on established body of knowledge.

Question 2 Short report

10 marks ( 1000 words excluding table of content and references)

Search for 5 comment letters from IASB AASB and FASB websites, received on a current exposure draft or proposal for a new accounting standard. You may find comment letters from a range of respondents, for example; Accounting bodies, industries, companies or corporate bodies. If you are having trouble with finding suitable comment letters, then contact Sarath Ukwatte (sukwatte@csu.edu.au). Once you find suitable comment letters you wish to discuss, complete the following tasks:

a) Describe in your own words the issues dealt by the exposure draft/proposal and comment letters.

b) Is there agreement among the various groups? Describe the issues where there is agreement/disagreement and provide examples.

c) In your opinion, which of the three theories of regulation (eg. public interest theory, private interest or capture theory) best explains the comment letters? Justify your answer.

Rationale

This assessment is designed to assist you:

* to gather and integrate knowledge on the theories which you have learned from topics 4 to 7.

* to develop your learning through application of the concepts covered in topics 4 to 7

* allow you to demonstrate your level of understanding and the level of your research efforts.

pat corporation paid 5 000 000 for saw corporation s voting common stock on january 687792

Pat Corporation paid $5,000,000 for Saw Corporation’s voting common stock on January 2, 2011, and Saw was dissolved. The purchase price consisted of 100,000 shares of Pat’s common stock with a market value of $4,000,000, plus $1,000,000 cash. In addition, Pat paid $100,000 for registering and issuing the 100,000 shares of common stock and $200,000 for other costs of combination. Balance sheet information for the companies immediately before the acquisition is summarized as follows (in thousands):



REQUIRED

1. Prepare journal entries for Pat Corporation to record its acquisition of Saw Corporation, including all allocations to individual asset and liability accounts.

2. Prepare a balance sheet for Pat Corporation on January 2, 2011, immediately after the acquisition and dissolution ofSaw.

transactions completed during the year ended 30 june 2012 are summarised below 687851

Financial Accounting Practice (BBAC301)

Group Assessment (4 Students per group) DUE Session 6.1 Week 6

Learning Outcome Assessed..i

Weighting: 15

Transactions completed during the year ended 30 June 2012 for Johnson 8 Co are surnmained as follow

Johnson & Co Trial Balance As at 30 lune, 2011

.c,c,i Account no Debit (5) Credit (5) Case a: ta ,. 1100 9200 Accounts recery ab 10 1101 5900 Prepaid rent 1102 760 GST outlays 1105 1400 Office supplies 1106 Furniture and equipment 1110 17950 I Accum Dep lumiture 8 equip 1111 6080 ‘Accounts payable 2200 3400 i I Salaries payables 2201 260 [ GST collections 2203 3200 Johnson. capital 3000 23 130 , 536070 $36070

Transactions completed during the year ended 30 June 2012 are summarised below(ignoreGST):

1. Collections on accounts receivable totalled 551 820 2 Consulting fees of $55 720 were receivable during the year. Clients are invoiced after ,en.¦ces are provided and are given 30 days in whKh to pay. 3 Rent paid in advance was $9120 4 Office supplies were purchased during the year for 740.n cash & 260 on credit S. Johnson withdrew $16000 for private use 6. Salary payments amounted to $19 960, of which $260 was for salaries accrued to the end of the year ending 30 June 2011 7. Advertrsing totalling $2100 was purchased on credit 8 Electricrty expense of 52250 was pad 9 Accounts payable of $1800 were paid

Attachments:

the following data in millions are taken from recent financial statements of nike in 687940

The following data (in millions) are taken from recent financial statements of Nike Inc.:

a. Determine the amount of change (in millions) and percent of change in net income for Year 2. Round percent to one decimal place.

Amount of change $ million
Percent of change in net income %

b. Determine the percentage relationship between net income and net sales (net income divided by net sales) for Year 2 and Year 1. Round to one decimal place.

Year 2: %
Year 1: %

analyzing interpret the following information regarding westlake corporation s cash 687944

Please provide an interpretation of the given statement of cash flows.

(Analyzing a statement of cash flows) Interpret the following information regarding Westlake Corporations cash flows.

 

Net income $680
Depreciation expense 125
Profits before depreciation 805
Increase in accounts receivable 200
Increase in inventories 240
Increase in accounts payable 120
Increase in accrued expenses 81
Cash flow from operations $566
Investment activity  
Change in fixed assets 1,064
Financing activity  
Increase in long term debt $640
Common stock dividends 0 20)
Total financing activities $520
Change in cash $22
Beginning cash 500
Ending cash $522

the following facts relate to an actual embezzlement case someone stole more than 40 687967

The following facts relate to an actual embezzlement case. Someone stole more than $40,000 from a small company in less than two months. Your job is to study the following facts, try to figure out who was responsible for the theft, how it was perpetrated, and (most important) suggest ways to prevent something like this from happening again. Facts Location of company: a small town on the eastern shore of Maryland. Type of company: crabmeat processor, selling crabmeat to restaurants located in Maryland. Characters in the story (names are fictitious): _ John Smith, president and stockholder (husband of Susan). Susan Smith, vice president and stockholder (wife of John). Tommy Smith, shipping manager (son of John and Susan). Debbie Jones, office worker. She began working part time for the company six months before the theft. (At that time, she was a high school senior and was allowed to work afternoons through a school internship program.) Upon graduation from high school (several weeks before the theft was discovered), she began working full time. Although she is not a member of the family, the Smiths have been close friends with Debbie’s parents for more than 10 years.

Accounting Records

All accounting records are maintained on a microcomputer. The software being used consists of the following modules:

1. A general ledger system, which keeps track of all balances in the general ledger accounts and produces a trial balance at the end of each month.

2. A purchases program, which keeps track of purchases and maintains detailed records of accounts payable.

3. An accounts receivable program, which keeps track of sales and collections on account and maintains individual detailed balances of accounts receivable.

4. A payroll program. The modules are not integrated (that is, data are not transferred automatically between modules). At the end of the accounting period, summary information generated by the purchases, accounts receivable, and payroll programs must be entered into the general ledger program to update the accounts affected by these programs. Sales The crabmeat processing industry in this particular town was unusual in that selling prices for crabmeat were set at the beginning of the year and remained unchanged for the entire year. The company’s customers, all restaurants located within 100 miles of the plant, ordered the same quantity of crabmeat each week. Because prices for the crabmeat remained the same all year and the quantity ordered was always the same, the weekly invoice to each customer was always for the same dollar amount. Manual sales invoices were produced when orders were taken, although these manual invoices were not pre numbered. One copy of the manual invoice was attached to the order shipped to the customer. The other copy was used to enter the sales information into the computer. When the customer received the order, the customer would send a check to the company for the amount of the invoice. Monthly bills were not sent to customers unless the customer was behind in payments (that is, did not make a payment for the invoiced amount each week). Note: The industry was unique in another way: many of the companies paid their workers with cash each week (rather than by check). Therefore, it was not unusual for companies to request large sums of cash from the local banks. When Trouble Was Spotted Shortly after the May 30 trial balance was run, Susan began analyzing the balances in the various accounts. The balance in the cash account agreed with the cash balance she obtained from a reconciliation of the company’s bank account. However, the balance in the accounts receivable control account in the general ledger did not agree with the total of the accounts receivable subsidiary ledger (which shows a detail of the balances owed by each customer). The difference was not very large, but the balances should be in 100 percent agreement. At this point, Susan hired a fraud auditor to help her locate the problem. In reviewing the computerized accounts receivable subsidiary ledger, the auditor noticed the following: 1. The summary totals from this report were not the totals that were entered into the general ledger program at month end. Different amounts had been entered. No one could explain why this had happened. 2. Some sheets in the computer listing had been ripped apart at the bottom. (In other words, the listing of the individual accounts receivable balances was not a continuous list but had been split at several points.) 3. When an adding machine tape of the individual account balances was run, the individual balances did not add up to the total at the bottom of the report. Susan concluded that the accounts receivable program was not running properly. The auditor’s recommendation was that an effort be made to find out why the accounts receivable control account and the summary totals per the accounts receivable subsidiary ledger were not in agreement and why there were problems with the accounts receivable listing. Because the accounts receivable subsidiary and accounts receivable control account in the general ledger had been in agreement at the end of April, the effort should begin with the April ending balances for each customer by manually updating all of the accounts. The manually adjusted May 30 balances should then be compared with the computer generated balances and any differences investigated. After doing this, Susan and John found several differences. The largest difference was the following: Although they found the manual sales invoice for Sale 2, Susan and John concluded (based on the computer records) that Sale 2 did not take place. The auditor was not sure and recommended that they call this customer and ask him the following: 1. Did he receive this order?

2. Did he receive an invoice for it?

3. Did he pay for the order?

4. If so, did he have a copy of his canceled check?

Although John thought that this would be a waste of time, he called the customer. He received an affirmative answer to all of his questions. In addition, he found that the customer’s check was stamped on the back with an address stamp giving only the company’s name and city rather than the usual ‘‘for deposit only’’ company stamp. When questioned, Debbie said that she sometimes used this stamp. Right after this question, Debbie, who was sitting nearby at the computer, called Susan to the computer and showed her the customer’s account. She said that the payment for $5,000 was in fact recorded in the customer’s account. The payments were listed on the computer screen like this:

Amount

Date of Payment

$5,000

May 3

$5,000

May 17

$5,000

May 23

$5,000

May 10

The auditor questioned the order of the payments—why was a check supposedly received on May 10 entered in the computer after checks received on May 17 and 23? About 30 seconds later, the computer malfunctioned and the accounts receivable file was lost. Every effort to retrieve the file gave the message ‘‘file not found.’’ About five minutes later, Debbie presented Susan with a copy of a bank deposit ticket dated May 10 with several checks listed on it, including the check that the customer said had been sent to the company. The deposit ticket, however, was not stamped by the bank (which would have verified that the deposit had been received by the bank) and did not add up to the total at the bottom of the ticket (it was off by 20 cents). At this point, being very suspicious, the auditor gathered all of the documents he could and left the company to work on the problem at home, away from any potential suspects. He received a call from Susan about four hours later saying that she felt much better. She and Debbie had gone to RadioShack (the maker of their computer program) and RadioShack had confirmed Susan’s conclusion that the computer program was malfunctioning. She and Debbie were planning to work all weekend reentering transactions into the computer. She said that everything looked fine and not to waste more time and expense working on the problem. The auditor felt differently. How do you feel?

Performance of Key Functions by Individual(s)

Individual(s) Performing Task

Most of the Time

Sometimes

1. Receiving order from customers

John

All others

2. Overseeing production of crabmeat

John or Tommy

3. Handling shipping

Tommy

John

4. Billing customers (entering sales into accounts receivable program)

Debbie

Susan

5. Opening mail

John

All others

6. Preparing bank deposit tickets and making bank deposits

Susan or Debbie

All others

7. Recording receipt of cash and checks (entering collections of accounts receivable into accounts receivable program)

Debbie

Susan

8. Preparing checks (payroll checks and payments of accounts payable)

Susan or Debbie

9. Signing checks

John

10. Preparing bank reconciliations

John

11. Preparing daily sales reports showing sales by type of product

Susan

12. Summarizing daily sales reports to obtain monthly sales report by type of product

Susan or Debbie

13. Running summaries of AR program, AP program, and payroll program at month end and inputting summaries into GL program

Susan or Debbie

14. Analyzing trial balance at month end and analyzing open balances in accounts receivable and accounts payable

Susan

CUSTOMER ACCOUNT PER MANUAL RECONSTRUCTION

Dr.

Cr.

Sale 1

5,000

Pmt. #1

5,000

Sale 2

5,000

Pmt. #2

5,000

Sale 3

5,000

Pmt. #3

5,000

Sale 4

5,000

Ending Balance

5,000

CUSTOMER ACCOUNT PER MANUAL RECONSTRUCTION

Dr.

Cr.

Sale 1

5,000

Pmt. #1

5,000

Sale 2

5,000

Pmt. #2

5,000

Sale 3

5,000

Pmt. #3

5,000

Ending balance

0

Required

a. If you were asked to help this company, could you conclude from the evidence presented that embezzlement took place? What would you do next?

b. Who do you think was the embezzler?

c. How was the embezzlement accomplished?

d. What improvements would you recommend in internal control to prevent this from happening again? In answering this question, try to identify at least one suggestion from each of the six classes of internal control activities discussed in this chapter (in the Control Activities section): transaction authorization, segregation of duties, supervision, accounting records, access control, and independent verification.

e. Would the fact that the records were maintained on a microcomputer aid in this embezzlement scheme?

global bike inc has a pragmatic design philosophy that comes from its deep roots in 687978

Global Bike Inc. has a pragmatic design philosophy that comes from its deep roots in both the offroad trail racing and long distance road racing sports. Nearly 20 years ago, its founders designed

their first bikes out of necessity—they had races to win and the bikes that were available at the time

did not perform to their extremely high standards. So, they took matters into their own hands and

built legendary bikes that would outlast and outperform the competition. From these humble origins,

Global Bike Incorporated was born and continues to deliver innovative high performance bicycles to

the world’s most demanding riders.

This heritage of entrepreneurial spirit and quest for design perfection is still the cornerstone of GBI’s

corporate philosophy. GBI produces bikes for the most demanding competitors—whether the

competition is on pavement or dirt, for money, fame or just bragging rights.

John Davis earned his racing scars in the mountain racing circuit in America, where he won numerous

downhill and cross country championships. Early on, John realized that the mass produced bicycles

available were inadequate in many ways for the type of racing he was doing. So, John stripped four

of his old bikes down to the bare metal and rebuilt them into a single “Frankenstein” bike that he rode

to win the national championship. Once news of his Frankenstein bike got out, John’s friends and

even his competitors began asking him to build them a Frankenstein bike too. While recovering from

an injury in 1990, John started producing the first series of Frankenstein bikes in his garage—each

one custom built from cannibalized parts from other bikes. As more and more orders came in, John

successfully expanded Frankenstein Bikes from his garage operations into a full blown manufacturing

facility in Dallas and began producing custom trail bikes which he sold through a network of

specialized bike dealers throughout the country.

Document Preview:

CASE STUDY Financial Accounting Review Practice Set Assignment Overview This assignment is a review of general financial accounting principles and procedures. In this assignment you will create general journal entries for a series of transactions. You may optionally show the impact of these transactions on a set of t accounts and you must create a trial balance and adjusted trial balance. You will then close out the income statement accounts as you would at year end. In a continuation of this assignment later this semester (see Module #10), you will explore how your journal entries might be created in an automated system by entering your “manual” journal entries into the SAP ERP system. The SAP system will then be used to produce a set of financial statements (balance sheet and income statement). If you fail to successfully complete the Excel portion of this assignment it will be impossible to complete the SAP portion. Global Bike Inc. (GBI) We will be working with a company called Global Bike Inc., (GBI). Information regarding GBI follows. Company History Global Bike Inc. has a pragmatic design philosophy that comes from its deep roots in both the off road trail racing and long distance road racing sports. Nearly 20 years ago, its founders designed their first bikes out of necessity—they had races to win and the bikes that were available at the time did not perform to their extremely high standards. So, they took matters into their own hands and built legendary bikes that would outlast and outperform the competition. From these humble origins, Global Bike Incorporated was born and continues to deliver innovative high performance bicycles to the world’s most demanding riders. This heritage of entrepreneurial spirit and quest for design perfection is still the cornerstone of GBI’s corporate philosophy. GBI produces bikes for the most demanding competitors—whether the competition is on pavement or dirt, for money, fame or…

Attachments:

prepare a report for your client your report should include a description of the cor 687981

Prepare a report for your client. Your report should include:

  1. A description of the core business of the company including full details of its operating activities.

  2. A discussion on any significant issues emerging from the Chairman’s Report.

  3. A discussion on any significant issues emerging from the Director’s Report.

  4. A discussion on company’s Corporate Governance Statement.

  5. A calculation of the key financial ratios for 2013.

  6. An overall evaluation of the company and your recommendation on investing inthe company.

Document Preview:

BUACC5930 Accounting Concepts and Practices Semester 2, 2014 Group Assignment You are an accountant at West Group Accountants & Investment Advisers. You have been approached by a group of investors for your professional advice on investing in Harvey Norman Holdings Limited. Your client is a strong believer in supporting Australian made products and Socially Responsible Investing. The latter would mean adopting an ? HYPERLINK “http://en.wikipedia.org/wiki/Investment” o “Investment” ?investment? ? HYPERLINK “http://en.wikipedia.org/wiki/Strategy” o “Strategy” ?strategy? which seeks to consider both ? HYPERLINK “http://en.wikipedia.org/wiki/Financial_return” o “Financial return” ?financial return? and ? HYPERLINK “http://en.wikipedia.org/wiki/Social_good” o “Social good” ?social good?. Required: Go to: ? HYPERLINK “http://www.harveynormanholdings.com.au/pdf_files/2013_Annual_Report.pdf” ?http://www.harveynormanholdings.com.au/pdf_files/2013_Annual_Report.pdf? You +1’d this publicly. ? HYPERLINK “http://www.google.com.au/search?q=woolworths&rls=com.microsoft:en au&ie=UTF 8&oe=UTF 8&startIndex=&startPage=1&rlz=1I7ADFA_enAU443&redir_esc=&ei=jniGT6H0FcGZiAfe3eTGBw” ?Undo? and access the company’s annual report for 2013. Prepare a report for your client. Your report should include: A description of the core business of the company including full details of its operating activities. A discussion on any significant issues emerging from the Chairman’s Report. A discussion on any significant issues emerging from the Director’s Report. A discussion on company’s Corporate Governance Statement. A calculation of the key financial ratios for 2013. An overall evaluation of the company and your recommendation on investing in the company. Please note the following: Format: Business report Contribution to overall assessment: 20% Length: 2000 – 2200 words Due date: Friday of Week 10 Your work must comply with the University’s General Guide for the Presentation of Academic…

Attachments:

short run market supply 8 points 4 points each carolina textiles inc is a small manu 687985

Short Run Market Supply (8 Points – 4 points each). Carolina Textiles, Inc. is a small manufacturer of cotton linen bed products for the hospital industry that it sells in a perfectly competitive market. Given $100,000 in fixed costs per day, the daily total cost function for this product is described by:

TC = $100,000 + $2Q + $0.0625Q2

where Q is units of cotton linen produced per day. Assume that MC > AVC at every point along the firm’s marginal cost curve, and that total costs include a normal profit.

Derive the firm’s supply curve, expressing quantity as a function of price (remember that

P = MR in a perfectly competitive market).

Develop the firm’s total variable cost (TVC) and average variable cost (AVC) equations?

Carolina Textiles, Inc. is a small manufacturer of cotton linen bed products for the hospital industry that it sells in a perfectly competitive market. Given $100,000 in fixed costs per day, the daily

shepherd cycles sells a variety of bicycles the business which uses the fifo method 688021

Shepherd Cycles sells a variety of bicycles. The business which uses the FIFO method, began the last quarter of 2013 (October 1 to December 31) with 20 “Iron Man 2” bicycles at a total cost of $122,000. During the quarter, the company completed the following transactions.October 8 105 bicycles were purchased at a cost of $5,920 each but in addition there was a freight charge of $480 on each bicycle. October 31 The sales for October were 85 bicycles which yielded total sales revenue of $761,600. ( 15 of these units were sold on account) November 4 A new batch of 60 bicycles was purchased at a total cost of $408,000November 10 5 of the bicycles purchased on November 4 were returned to the supplier, as they were defective. November 30 During the month 57 bicycles were sold at a price of $9,520 each. December 4 A customer, to whom 9 bicycles were sold during the first business day of November, returned 4 of the cycles. December 10 Owing to an increased demand, a further 115 bicycles were purchased at a cost of $7,600 each; these were subject to a trade discount of 5% each. December 31 122 bicycles were sold during December at a unit selling price of $10,100.December 31 An actual count of inventory was carried out which revealed that there were 28 units of the merchandise in the store room.

colonial house furniture company 688040

The Colonial House Furniture Company manufactures four drawer oak filing cabinets in six stages. In the first stage, the boards forming the walls of the cabinet are cut; in the second stage, the front drawer panels are woodworked; in the third stage, the boards are sanded and finished; in the fourth stage, the boards are cleaned, stained, and painted with a clear finish; in the fifth stage, the hardware for pulls, runners, and fittings is installed; and in the final stage, the cabinets are assembled. Inspection occurs at each stage of the process, and the average percentages of good quality units are as follows.

Stage

Average Percentage Good Quality

1

87%

2

91%

3

94%

4

93%

5

93%

6

96%

The cabinets are produced in weekly production runs with a product input for 300 units.

a.

Determine the weekly product yield of good quality cabinets.

b.

What would weekly product input have to be in order to achieve a final weekly product yield of 300 cabinets?

the stockholders equity accounts of karp company at january 1 2014 688094

The stockholders’ equity accounts of Karp Company at January 1, 2014, are as follows. Preferred Stock,6%,$50par $615,000 Common Stock,$4par 632,000 Paid in Capital in Excess of Par—Preferred Stock 203,000 Paid in Capital in Excess of Par—Common Stock 300,500 Retained Earnings 799,400 There were no dividends in arrears on preferred stock. During 2014, the company had the following transactions and events. July 1 Declared a$0.60cash dividend per share on common stock. Aug. 1 Discovered$29,000understatement of 2013 depreciation on equipment. (Ignore income taxes.) Sept.

Document Preview:

The stockholders’ equity accounts of Karp Company at January 1, 2014, are as follows.? Preferred Stock, 6%, $50 par??$615,000??Common Stock, $4 par??632,000??Paid in Capital in Excess of Par—Preferred Stock??203,000??Paid in Capital in Excess of Par—Common Stock??300,500??Retained Earnings??799,400???There were no dividends in arrears on preferred stock. During 2014, the company had the following transactions and events.? July 1??Declared a $0.60 cash dividend per share on common stock.??Aug. 1??Discovered $29,000 understatement of 2013 depreciation on equipment. (Ignore income taxes.)??Sept. 1??Paid the cash dividend declared on July 1.??Dec. 1??Declared a 10% stock dividend on common stock when the market price of the stock was $19 per share.??15??Declared a 6% cash dividend on preferred stock payable January 15, 2015.??31??Determined that net income for the year was $310,800.??31??Recognized a $216,200 restriction of retained earnings for plant expansion.?? Requirements: Journalize the transactions, events, and closing entries for net income and dividends. (Credit account titles are automatically indented when amount is entered. Do not indent manually. If no entry is required, select “No Entry” for the account titles and enter 0 for the amounts.) Journalize the transactions, events, and closing entries for net income and dividends. (Credit account titles are automatically indented when amount is entered. Do not indent manually. If no entry is required, select “No Entry” for the account titles and enter 0 for the amounts.) Date?Account Titles and Explanation?Debit?Credit??July 1??????????Aug. 1??????????Sept. 1??????????Dec. 1???????????????Dec. 15??????????Dec. 31???????????(To close net income)???????????????(To close cash dividends)???????????????(To close stock dividends)??????? ? HYPERLINK “http://edugen.wileyplus.com/edugen/shared/assignment/test/qview.uni?id=quest2424825entrance1&selected_question=quest2424825&operation=take question” o “Show List of…

accounting 688122

You need to calculate the depreciation for each of the method and then recommend which method will give you the best tax benefit.You also need to tell us what is the extra depreciation Alaska Airlines will be able to deduct if it uses the favored tax benefit.

COMPUTING FIRT YEAR DEPRECIATION AND BOOOK VALUE (10 MINS). At the beginning of the year,Alaska Freight Airlines expects the plane to remain useful for five years, ($4,000,000 miles) and to have a residual value of $7,000,000. The company expects the plane to be flown, 1,4000,000 miles the first year. Selecting the best depreciation method for tax purpoe for tax purposes (10 mins). this exercise use the Alaska Freight Airlines data from short execercise 9 3. Alaska Freight Airlines is deciding which depreciation method to use for income tax purposes.

Requirement; 1. which depreciation method offers the tax advantage for the first year? describe the nature of the tax advantage. 2. how much extra depreciation will Alaska Freight get to deduct for the first year as compared with the straight line method?

question 2.

honeysuckle corporation has provided the following data for the month of january inv 682768

Honeysuckle Corporation has provided the following data for the month of January:

Inventories

Beginning

Ending

Raw materials

$40,000

$23,000

Work In process

$9,000

$13,000

Finished goods

$52,000

$45,000

Additional Information

Raw material purchases

$68,000

Direct labor costs

$90,000

Manufacturing overhead cost incurred

$44,000

Indirect materials included in manufacturing overhead costs incurred

$8,000

Manufacturing overhead cost applied to work in process

$39,000

financial management 682782

hi there, i have attached my question and assignment help document which is very useful. hope you will do your best.

Document Preview:

Assessment item 3 Task Question 1 (15 marks, 7.5 marks each) Write a short essay of 400 500 words for each of the following questions. Where possible, illustrate with an appropriate example in your answer. You must support your discussion with appropriate references. ?a. ‘Evaluating mutually exclusive projects using the IRR and NPV approaches can be problematic’. Discuss this statement. b. Is it possible for an ordinary annuity to have the same present value as perpetuity if the cash flows and discount rates are identical? Explain. Question 2 (15 marks, 5 marks each)??a. An investment will pay $200 at the end of each of the next 3 years, $250 at the end of year 4, $300 at the end of year 5 and $500 at the end of year 6. Given that other investments of equal risk earn 8% per annum, calculate the present value and future value of this investment. b. An investment firm A pays 9% interest per annum, compounded on a quarterly basis. To remain competitive, the investment manager of another firm (firm B) is willing to match the interest rate offered by firm A, but interest will be compounded on a monthly basis. What nominal rate of interest must firm B offer to its clients? c. A loan of $200,000 with an interest rate of 8% per annum is to be paid off by 20 equal quarterly payments; the first payment is due today. Find the size of the quarterly payment. Question 3 (30 marks) The Metro Glass Company (MGC) is proposing the construction of a new plant that manufactures glass products for large buildings and office blocks. The plant has an annual capacity of 100,000 tons and will cost $400 million to build. Profits on the plant will be taxed at a rate of 30 per cent. The company expects its new plant to produce 60,000 tons of plate glass per year. The annual revenues of $42.0 million based on an anticipated selling price of $700 per ton will allow the company to gain 12 per cent market share in the first year of operation. Fixed costs are expected to average $15 million…

nick stannos arrives in australia from a european country on 26 january 684813

Nick Stannos arrives in Australia from a European country on 26 January. He rents a small apartment and accepts two jobs. By 30 June he has saved $10,000 and decided to return to Europe permanently.
1. For the relevant income year, will Stannos be regarded as a resident or non resident? In any event, what difference does it make?
2. If Stannos had remained in Australia indefinitely and was regarded as a resident of Australia from 26 January, how would his European salary income of A$20,000 (derived in the period 1 July to 24 January) be taxed? Assume ex Australia tax of A$3,000 has been paid in respect of this income.
Income from Property
1. George owns a large cattle firm, parts of which are heavily timbered. He agrees to allow Mr Smith to enter his land for the purpose of felling timber in return for a payment of $20 per tree removed. During the first tax year of his agreement, George receives $10,000 from Smith. What are the tax consequences of this transaction? Would your answer differ if Smith had promised to remove the timber?
2. The following year, George contracts to sell Smith 9,000 trees for $90,000. Smith is to enter George’s land for the purpose of felling and removing the trees, and is to pay the money by monthly instalments regardless of whether the timber is removed or not. The agreement provides that if 9,000 suitable trees are not found upon the land, then there shall be a pro rata decrease in contract price.
3. In the following year, George decides to sell his farm to Brown for the sum of $200,000 plus “royalty” of $10 for every tree removed from the land for the next 10 years.
Advise George in each of the above situations as to his tax liabilities. Also advise him as to whether his receipts are income or capital on general principles. (Note that these cases are governed by CGT event DI)

Attachments:

schedule of cost of goods manufactured income statement the following information wa 684816

Scheduleof cost of goods manufactured, income statement

The following information was taken from the ledger of Jefferson Industries, Inc.:

Direct labor

$85,000

Administrative expenses

$59,000

Selling expenses

34,000

Work in. process:

Sales

300,000

Jan. 1

29,000

Finished goods

Dec. 31

21,000

Jan. 1

115,000

Direct material purchases

88,000

Dec. 31

131,000

Depreciation: factory

18,000

Raw (direct) materials on hand

Indirect materials used

10,000

Jan. 1

31,000

Indirect labor

24,000

Dec. 31

40,000

Factory taxes

8,000

Factory utilities

11,000

Prepare the following:

a. A schedule of cost of goods manufactured for the year ended December 31.

b. An income statement for the year ended December 31.

for the relevant income year will stannos be regarded as a resident or non resident 684830

Taxation LAW 2014

1..Residence and Source 250 words

Nick Stannos arrives in Australia from a European country on 26 January. He rents a small apartment and accepts two jobs. By 30 June he has saved $10,000 and decided to return to Europe permanently.

1.For the relevant income year, will Stannos be regarded as a resident or non resident? In any event, what difference does it make?

2.If Stannos had remained in Australia indefinitely and was regarded as a resident of Australia from 26 January, how would his European salary income of A$20,000 (derived in the period 1 July to 24 January) be taxed? Assume ex Australia tax of A$3,000 has been paid in respect of this income.

2.
.Income from Property 250 words

1.George owns a large cattle firm, parts of which are heavily timbered. He agrees to allow Mr Smith to enter his land for the purpose of felling timber in return for a payment of $20 per tree removed. During the first tax year of his agreement, George receives $10,000 from Smith. What are the tax consequences of this transaction? Would your answer differ if Smith had promised to remove the timber?

2.The following year, George contracts to sell Smith 9,000 trees for $90,000. Smith is to enter George’s land for the purpose of felling and removing the trees, and is to pay the money by monthly instalments regardless of whether the timber is removed or not. The agreement provides that if 9,000 suitable trees are not found upon the land, then there shall be a pro rata decrease in contract price.

3.In the following year, George decides to sell his farm to Brown for the sum of $200,000 plus “royalty” of $10 for every tree removed from the land for the next 10 years.

Advise George in each of the above situations as to his tax liabilities. Also advise him as to whether his receipts are income or capital on general principles. (Note that these cases are governed by CGT event DI)

3..
Week 4 Income Classification 250 words

Briefly discuss the income tax implications of the following, stating which sections of the ITAA 1997 or ITAA 1936, if any, are most relevant.

1.A $10,000 bonus paid by the Australian Cricket Control Board to the captain of the Australian cricket team for outstanding leadership during a successful tour of England.

2.A boat valued at $35,000 given to an amateur footballer to turn professional.

3.Profit of $25,000 made by a trucking company on the disposal of one of the 30 trucks it has leased to carry on its business.

4.An exchange gain of $500,000 made by a manufacturer in respect of money borrowed in 1997 and used to finance construction of a new building.

5.Gift and payments made by a football club and its supporters to a star professional player, largely in their delighted response to his being selected to play for Australia. The club gave him a car valued at $25,000; supporters, through a collection at one game, gave him $2,425.

4.. Week 5 Capital Gains Tax 250 words

Because of his wife’s ill health, Brain sold his gift shop and family home in Victoria and moved to WA on 20 June this year. Brain had acquired the vacant premises 10 years ago for $750,000 and established the business on that date. He sold the business on 20 May this year for a net consideration of $1,880,000. This was made up as follows:

Items AUD $
1 Goodwill 440,000
2 Trading Stock 60,000
3 Fittings 120,000
4 Shop and Land 1,360,000
5 Less debt taken over secured over stock and fittings (100,000)

In addition, Brain received a further $20,000 for signing a contract not to open another business within a 10 Km radius for the next five years.

The turnover of the shop for the previous financial year was $540,000.

Brain’s home is valued at $1.8m. He also has a 45% interest in a property development company which has assets of $5.4m. His wife also has a 5% interest in that company. The turnover of the property development company last year was $1.2m.

Advise Brain of the tax consequences arising from the sale.

5.. Week 7 Capital Allowance 250 words

Explain tax consequences for following term:

•Pooling of Assets

•Low – value pool assets

•Software development pool

•;Small business concessions

Attachments:

course project 1 consists of two parts a and b respectively there are 10 requirement 684843

Course Project #1 consists of two parts, A and B, respectively. There are 10 Requirements for you to complete in this exercise, Part A has 1 3 andPart B has 4 10. Part A is due at the end of Week 3. Part B is due at the end of Week 5. See Syllabus/”Due Dates for Assignments & Exams” for due date information. All of the information you need to complete Course Project #1 is located in this Workbook.
• There are8 worksheets in the workbook you will need to complete for Parts A & B.
• A list ofOctober transactions
• AChart of Accounts reference sheet
• AGrading Rubric to help explain what is expected.
• Each worksheet has the Check Figures embedded as a comment.

describe the substantive auditing procedures brown may consider performing with comp 684885

Required Describe the substantive auditing procedures Brown may consider performing with computerized audit software using Big Z’s two computer files and Brown’s computer file of test counts. The substantive auditing procedures described may indicate the reports to be printed out for Brown’s followup by subsequent application of manual procedures. Do not describe subsequent manual auditing procedures. Group the procedures by those using (a) the perpetual inventory file and (b) the physi cal inventory and test count files. (Hint: You may wish to refer to Chapter 12 as well as this chapter in answering this question.) AICPA

Attachments:

jodie products limited is a merchandising company that sells stationery and other sc 684896

Jodie Products Limited is a merchandising company that sells stationery and other school supplies. The company is planning its cash needs for 2014/2015. In the past Jodie Products had to borrow money in order to support peak sales of back to school material, which occur during January. The following information has been provided to assist in preparing a cash budget for the period:

Budgeted monthly income statement for December 2014 to March 2015

December

January

February

March

N$

N$

N$

N$

Sales

40 000

70 000

50 000

45 000

Cost of sales

24 000

42 000

30 000

27 000

Gross margin

16 000

28 000

20 000

18 000

Operating expenses

Selling expenses

7 200

11 700

8 500

7 300

Administrative expenses

5 600

7 200

6 100

5 900

Net income

3 200

9 100

5 400

4 800

Note the following additional information:

• Administrative expenses include N$2 000 depreciation each month.

• 20 % of sales are cash sales. Credit sales are collected over a three month period in the ratio of 10% in the month of sale; 70% in the month following sale and 20% in the second month following sale. October sales amounted to N$30 000, and November sales to N$36 000.

• Inventory purchases are paid for within 15 days. Therefore 50% of a month’s inventory purchases are paid for in the month of purchase. The remaining 50% is paid in the following month.

• Purchases amount to 75% of the following month’s sales.

• Dividends of N$1 000 will be declared and paid in February.

• The cash balance on 30 November was N$8 000. The company must maintain a cash balance of at least N$8 000 at all times.

• The company can borrow cash from the bank as needed. Loans are received in multiples of N$1 000. Interest paid on loans are 15% per annum, payable monthly.

Prepare a cash budget for December, January and February. When will Jodie products be able to repay the loan?

alladin company purchased machine 201 on may 1 2014 the following information relati 684904

Alladin Company purchased Machine #201 on May 1, 2014. The following information relating to Machine #201 was gathered at the end of May.

Price $136,850
Credit terms 2/10, n/30
Freight in $ 1,288
Preparation and installation costs $ 6,118
Labor costs during regular production operations $16,905

It is expected that the machine could be used for 10 years, after which the salvage value would be zero. Alladin intends to use the machine for only 8 years, however, after which it expects to be able to sell it for $2,415. The invoice for Machine #201 was paid May 5, 2014. Alladin uses the calendar year as the basis for the preparation of financial statements.

Collapse question part

(a)

Compute the depreciation expense for the years indicated using the following methods. (Round answers to 0 decimal places, e.g. 45,892.)

Depreciation Expense
(1) Straight line method for 2014 $Entry field with incorrect answer
(2) Sum of the years’ digits method for 2015 $Entry field with incorrect answer
(3) Double declining balance method for 2014

q during 2011 comet cares inc decided to sell an unprofitable segment of its busines 684917

Q). During 2011, Comet Cares, Inc. decided to sell an unprofitable segment of its business. The sale of this segment qualifies as a discontinued operation for financial reporting purposes. However, at the end of 2011, Comet had yet to sell the segment. On December 31, 2011 the segment assets had a fair value minus anticipated costs to sell of $3,500,000 and a book value of $3,700,000. For the year, the segment reported an operating loss of $500,000. In January of 2012, Comet Cares sold the segment for $3,600,000. Operating losses in the first month of 2012 amounted to $45,000. Assume a 40% tax rate in both 2011 and 2012.

a) What is the after tax dollar value impact of the discontinued operation on 2011 Net Income (use ( ) for a decrease)?

$_________________________________

b) What is the after tax dollar value impact of the discontinued operation on 2012 Net Income (use ( ) for a decrease)?

$__________________________________

which regression equation is best why explain 684922

Sorrentino CompanySorrentino Company, which has been in business for one year, manufactures specialty Italian pastas. The pasta products start in the mixing department, where durum flour, eggs, and water are mixed to form dough.The dough iskneaded, rolled flat, and cut into fettucine or lasagna noodles, then dried and packaged.Paul Gilchrist, controller for Sorrentino Company, is concerned because the company has yet to make a profit. Sales were slow in the first quarter butreally picked up by the end of the year. Over the course of the year, 718,200 boxes were sold. Paul is interested in determining how many boxes must be sold to break even. He has begun to determine relevant fixed and variable costs and has accumulated the following per unit data:Price $0.90Direct materials 0.35Direct labor 0.25He has had more difficulty separating overhead into fixed and variable components. In examining overhead related activities, Paul has noticed that machine hour appear to be closely correlated with units in that 100 boxes of pasta can be produced per machine hour. Setups are important batch level activity. Paul has accumulated the following information on overhead costs, number of setups, and machine hours for the past 12 months.MonthOverheadNumber of SetupsMachine HoursJanuary$5,700 18595February4,5006560March4,89012575April5,50015615May6,24020660June5,10010552July5,53216630August5,40912602September5,30011635October5,00012550November5,35014593December5,47014615Selling and administrative expenses, all fixed, amounted to $185,000 last year.In the second year of operations, Sorrentino Company has decided to expand into the production of sauces to top its pastas. Sauces are also started in the mixing department, using the same equipment. The sauces are mixed, cooked, and packaged into plastic containers. One jar of sauce is priced at $2 and required $0.75 of direct materials and $0.50 of direct labor. 50jars of sauce can be produced per machine hour. The production manager believes that with careful scheduling, he can keep the total number of setups (for both pasta and sauce) to the same number as used last year. The marketing director

expects to increase selling expense by$25,000 per year to promote the new product and believes Sorrentino Company can sell twoboxes of pasta for every one jar of sauce.Required:1.Separate overhead into fixed and variable components using regression analysis. Run three regressions, using the following independence variables: (a) number of setups, (b) number of machine hours, and (c) a multiple regression using both number of setups and machines. Which regression equation is best? Why?2.Using the results from (1) above, calculate the number of boxes of pasta which must be sold to break even beforethe expansion into theproduction of sauces.3.Now consider the production of sauces. Using the results of multiple regression equation, calculate the break even number of boxes of pasta and jars of sauce.4.Suppose that the production manager is wrong and that the number of setups doubles. Calculate the new break even number of boxes of pasta and jars of sauce.5.Refer to the original data. Suppose that only 30 jars of sauce can be produced per machine hour and thatSorrentino Company willsell3boxes of pastafor every one jar of sauce. Calculate the new break even number of boxes ofpasta and jars of sauce.6.Explain howuncertainty in the sales mix and in cost estimates affectthe break even points for Sorrentino Company.

Attachments:

hello how are you i need your help its a cost management assignment so i want the so 684931

hello, how are you. i need your help . its a cost management assignment. so, i want the solution of this assignment. could you help me ? i need this by 23 sep . file has been attached.my e mail id is smooth_as_velvet007@hotmail.com

Document Preview:

PROJECT COST MANAGEMENT 642 ASSIGNMENT NO 2 (Value 40%) LIFE CYCLE COSTING THIS ASSIGNMENT IS TO BE UNDERTAKEN INDIVIDUALLY Assignment Aim To understand and apply the processes for life cycle costing. Assignment Details You want to purchase (not rent) a post 2013 released computer system and associated software solely for use in your university PM course. The computer must be capable of creating, reading and manipulating word documents, pdf files, spreadsheets, detailed statistical programs (such as SPSS), email and internet communication and provide stable access to and use of University programs such as Turnitin, Blackboard, etc. It must also have sufficient processing power and data storage to cope with large and complex data. You are to choose TWO significantly different computer options (ie desktop PC, desktop MAC, laptop, tablet,etc) and perform a Life Cycle Costing calculation using the Net Present Value method to determine their life cycle costs, over a period of 3 years. The computers are NOT to be obtained by leasing or by loan. You are also to ignore any tax implications in your life cycle costing, but must consider depreciation and inflation. As a minimum you must provide the following information: ? a detailed description of ALL your assumptions ? a detailed explanation and breakdown of all costs and values used in your costing (do NOT use published all inclusive rates for running expenses, nor information directly from any website) ? sources for ALL the data used (with sources, properly referenced). ? a comprehensive explanation (with sources, properly referenced) for your choice of discount rate and method for handling inflation ? a single A4 spreadsheet for each computer LCC which shows all the relevant costs and values for the full life cycle as well as the discounting calculations. Each spreadsheet is to be in landscape layout, titled and use the following general layout for information: Years …

Attachments:

annexure a questions for man acc 1b assignment 1 question 1 60 of technical mark aca 684953

ANNEXURE A

QUESTIONS FOR MAN ACC 1B ASSIGNMENT 1

Question 1 (60% of technical mark)

Acacia Beverages (AB) bottles two soft drinks under license to Cadbury Schweppes at its Rehoboth plant. All inventory is in direct materials and finished goods at the end of each working day. There is no work in process inventory.

The two soft drinks bottled by AB are lemonade and diet lemonade. The syrup for both soft drinks is purchased from Cadbury Schweppes.

AB uses a batch size of 1 000 cases as the unit of analysis in its budgeting. (Each case contains 24 bottles.) Direct materials are expressed in terms of batches, in which one batch of direct materials is the input necessary to yield one batch (1 000 cases) of beverage. The following purchase prices are forecast for direct materials in 2015:

Lemonade

Diet Lemonade

Syrup

N$1 200 per batch

N$1 100 per batch

Containers (bottles, caps, etc)

N$1 000 per batch

N$1 000 per batch

Packaging

N$800 per batch

N$800 per batch

All direct material purchases are on account. The soft drinks are bottled using the same equipment. The only difference in the bottling process for the two soft drinks is the syrup.

Summary data used in developing budgets for 2015 are:

1. Sales

a. Lemonade, 1 080 batches @ N$9 000 selling price per batch

b. Diet Lemonade, 540 batches @ N$8 500 selling price per batch

c. All sales are on account

2. Beginning inventory of direct materials (1 Jan 2015)

a. Syrup for lemonade: 80 batches at N$1 100 purchase price per batch

b. Syrup for diet lemonade: 70 batches at N$1 000 purchase price per batch

c. Containers: 200 batches at N$950 purchase price per batch

d. Packaging: 400 batches at N$900 purchase price per batch

3. Beginning inventory of finished goods (1 Jan 2015)

a. Lemonade: 100 batches at N$5 300 per batch

b. Diet Lemonade: 50 batches at N$5 200 per batch

4. Target ending inventory of direct materials (31 Dec 2015)

a. Syrup for lemonade: 30 batches

b. Syrup for diet lemonade: 20 batches

c. Containers: 100 batches

d. Packaging: 200 batches

5. Target ending inventory of finished goods

a. Lemonade: 20 batches

b. Diet Lemonade: 10 Batches

6. Each batch requires 20 direct manufacturing labour hours at the 2015 budgeted rate of N$25 per hour. Direct manufacturing labour costs are paid at the end of each month.

7. Variable manufacturing overhead is forecast to be N$600 per hour of bottling time; bottling time is the time the filling equipment is in operation. It takes two hours to bottle one batch of lemonade and two hours to bottle one batch of diet lemonade. Assume all variable manufacturing costs are paid during the same month when incurred. Fixed manufacturing overhead is forecast to be N$1 200 000 for 2015. Included in the fixed manufacturing overhead forecast is N$400 000 for depreciation. All manufacturing costs are paid as incurred.

8. Hours of budgeted bottling time is the sole cost allocation base for all fixed manufacturing overhead.

9. Administration costs are forecast to be 10% of the cost of goods manufactured for 2015. Marketing costs are forecast to be 12% of revenues for 2015. Distribution costs are forecast to be 8% of revenues for 2015. All these costs are paid during the month when incurred. Assume that there is no depreciation costs included in these estimates.

10. Budgeted beginning balances on 1 January 2015:

a. Accounts receivables N$550 000

b. Accounts payables N$300 000

c. Cash N$100 000

11. Budgeted ending balances on 31 December 2015:

a. Accounts receivables (from sales) N$600 000

b. Accounts payables (for direct materials) N$400 000

12. Budgeted equipment purchase in May 2015: N$1 350 000

13. Estimated income tax expense for 2015 payable on 31 December 2015 amounts to N$625 000.

Assume AB uses the first in first out method for costing all inventories. On the basis of the preceding data, prepare the following budgets for the budget meeting in October 2014:

1.1

Sales budget (in N$)

1.2

Production budget (in units)

1.3

Direct materials usage budget (in units and N$)

1.4

Direct material purchases budget

1.5

Direct manufacturing labour budget

1.6

Manufacturing overhead costs budget

1.7

Ending finished goods inventory budget

1.8

Cost of goods sold budget

1.9

Distribution cost budget

1.10

Administration cost budget

1.11

Budgeted statement of comprehensive income

become an expert join solution inn sign in home portfolio experts about us products 684972

Solution inn, the source of academic solution!

Become expert! Become an expert Join now! Join Solution Inn Sign in!Sign in
  • Home
  • Portfolio
    • Experts
    • About us
  • Products and Services
    • Subjects
    • Services
  • My Account
    • Report Problem
    • Sign in
  • Shopping Cart

Home > Business > Managerial Accounting > Capital Budgeting Decisions

The budget director of Feathered Friends Inc., with the assistan

  • Question
  • Reviews

The budget director of Feathered Friends Inc., with the assistance of the controller, treasurer, production manager, and sales manager, has gathered the following data for use in developing the budgeted income statement for December 2010:

a. Estimated sales for December:

Bird House . . . . . . . . . . . . . . . . . . . . 32,500 units at $50 per unit

Bird Feeder. . . . . . . . . . . . . . . . . . . . 21,300 units at $85 per unit

b. Estimated inventories at December 1:

Direct materials: Finished products:

Wood. . . . . . . . . 2,400 ft. Bird House . . . . . . . . . 3,100 units at $26 per unit

Plastic. . . . . . . . . 3,600 lbs. Bird Feeder . . . . . . . . . 1,900 units at $40 per unit

c. Desired inventories at December 31:

Direct materials: Finished products:

Wood. . . . . . . . . 2,900 ft. Bird House . . . . . . . . . 3,600 units at $27 per unit

Plastic . . . . . . . . 3,400 lbs. Bird Feeder. . . . . . . . . 1,800 units at $41 per unit

d. Direct materials used in production:

In manufacture of Bird House: In manufacture of Bird Feeder:

Wood . . . . . 0.80 ft. per unit of product Wood . . . . . 1.20 ft. per unit of product

Plastic . . . . 0.50 lb. per unit of product Plastic . . . . 0.75 lb. per unit of product

e. Anticipated cost of purchases and beginning and ending inventory of direct materials:

Wood . . . . . . . $6.00 per ft. Plastic . . . . . . $0.80 per lb.

f. Direct labor requirements:

Bird House:

Fabrication Department . . . . . . . . . 0.20 hr. at $15 per hr.

Assembly Department . . . . . . . . . . 0.30 hr. at $11 per hr.

Bird Feeder:

Fabrication Department . . . . . . . . . 0.40 hr. at $15 per hr.

Assembly Department . . . . . . . . . . 0.35 hr. at $11 per hr.

g. Estimated factory overhead costs for December:

Indirect factory wages $750,000 Power and light $47,000

Depreciation of plant and equipment 185,000 Insurance and property tax 15,400

h. Estimated operating expenses for December:

Sales salaries expense $645,000

Advertising expense 149,700

Office salaries expense 211,100

Depreciation expense—office equipment 5,200

Telephone expense—selling 4,800

Telephone expense—administrative 1,500

Travel expense—selling 41,200

Office supplies expense 3,500

Miscellaneous administrative expense 5,000

i. Estimated other income and expense for December:

Interest revenue $16,900

Interest expense 11,600

j. Estimated tax rate: 35%

Instructions

1. Prepare a sales budget for December.

2. Prepare a production budget for December.

3. Prepare a direct materials purchases budget for December.

4. Prepare a direct labor cost budget for December.

5. Prepare a factory overhead cost budget for December.

6. Prepare a cost of goods sold budget for December. Work in process at the beginning of December is estimated to be $27,000, and work in process at the end of December is estimated to be $32,400.

7. Prepare a selling and administrative expenses budget for December.

8. Prepare a budgeted income statement for December.

$1.99


33
Purchases

0
Comments

1005
Views

Buyer’s Ratings

(Be the first one to rate)Social connections

Created

Compatibility

Files Included

September 5, 2011

MS excel

xls

Customers who bought this product also purchased…

Van Gogh Frame Company prepared the following sales budget for Steelcase Inc. is one of the largest manufacturers Pet Joy Wholesale Inc., a pet wholesale supplier, was organized
Audio Mechanics, Inc. manufactures two models of speakers, DL an Levi Strauss & Co. manufactures slacks and jeans under Venus Candy Company budgeted the following costs for anticipated production for

Why Solution Inn?

We at solution Inn provide eclectic range of subjects’ solutions ranging from college to university level books. Most of the questions at the back of each chapter of the book are difficult to solve and have no solution in the book. We have taken this challenge as we solved most of the questions and our online database contains almost thousand of questions taken from those books.

Need help in:BusinessChemistryComputer SciencesEssaysEngineeringMathematicsPhysicsSecurity at Solution Inn
Click to verify



Home | Terms Of Use | Privacy policy | About | FAQ | Our Team | Copyright | Testimonials | Feedback | Whose Using it | Why Solutioninn | Contact

Copyright (c) 2014 Solution Inn All rights reserved.

question 5 the comparative statements of osborne company are 684980

Question 5 The comparative statements of Osborne Company are presented here. OSBORNE COMPANY Income Statements For the Years Ended December 31 2014 2013 Net sales $1,896,390 $1,756,350 Cost of goods sold 1,064,390 1,011,850 Gross profit 832,000 744,500 Selling and administrative expenses 505,850 484,850 Income from operations 326,150 259,650 Other expenses and losses Interest expense 23,785 21,785 Income before income taxes 302,365 237,865 Income tax expense 93,785 74,785 Net income $ 208,580 $ 163,080 OSBORNE COMPANY Balance Sheets December 31 Assets 2014 2013 Current assets Cash $ 60,100 $ 64,200 Debt investments (short term) 74,000 50,000 Accounts receivable 123,650 108,650 Inventory 127,785 117,285 Total current assets 385,535 340,135 Plant assets (net) 662,341 533,641 Total assets $1,047,876 $873,776 Liabilities and Stockholders’ Equity Current liabilities Accounts payable $ 165,850 $151,250 Income taxes payable 45,285 43,785 Total current liabilities 211,135 195,035 Bonds payable 233,341 213,341 Total liabilities 444,476 408,376 Stockholders’ equity Common stock ($5 par) 290,000 300,000 Retained earnings 313,400 165,400 Total stockholders’ equity 603,400 465,400 Total liabilities and stockholders’ equity $1,047,876 $873,776 All sales were on account. Net cash provided by operating activities for 2014 was $241,920. Capital expenditures were $136,400, and cash dividends were $60,580. Compute the following ratios for 2014. (Round all answers to 2 decimal places, e.g. 1.83 or 12.61%.) (a) Earnings per share $ (b) Return on common stockholders’ equity % (c) Return on assets % (d) Current ratio :1 (e) Accounts receivable turnover times (f) Average collection period days (g) Inventory turnover times (h) Days in inventory days (i) Times interest earned times (j) Asset turnover times (k) Debt to assets % (l) Current cash debt coverage times (m) Cash debt coverage times (n) Free cash flow $ SHOW SOLUTION LINK TO TEXT LINK TO VIDEO

early in 2014 chien company switched to a just in time inventory system its sales co 685028

Early in 2014, Chien Company switched to a just in time inventory system. Its sales, cost of goods sold, and inventory amounts for 2013 and 2014 are shown below.

2013 2014
Sales $3,112,256 $3,750,000
Cost of goods sold 1,213,556 1,444,633
Beginning inventory 181,776 212,251
Ending inventory 212,251 108,987
Collapse question part

(a1)

Determine the inventory turnover for 2013 and 2014. (Ro

our apples because anthony s orchard is a family run orchard we grow every apple to 685039

UNIT TWO

Financial Analysis

Read these:

#1: Review the Anthony’s Orchard.

Our Apples: Because Anthony’s Orchard is a family run orchard, we grow every apple to be delicious and healthy for you and your family just like we would for ours. We choose only the highest quality trees and carefully select varieties that thrive in Wenatchee Valley.

Anthony’s Orchard currently features:

Braeburn: These beautiful red orange apples burst with flavors that are both sweet and crisp. One bite is all you’ll need to discover that Braeburn apples make a perfect snack for you or the kids. A hybrid descended from the Granny Smith variety, they also retain some of that apple’s signature tartness and are ideal for cooking in desserts and applesauce.

  • Chomp or cook: perfect for both!
  • Season: October July

Honeycrisp: Intense flavors and unique colors combine in this versatile apple. A mix of green and red colors complement the mix of sweet and tart flavors. Although small in size, Honeycrisp lives up to its namesake with a sweet and crunchy experience waiting in every bite. The tart flavors and dense texture also work great in your favorite pies or applesauce.

All Anthony’s Orchard apple pies feature fresh and delicious Honeycrisp apples right from our orchard.

  • Chomp or cook: perfect for both!
  • Season: October May
  • Prepared Apple Products Just Like Mom’s: At Anthony’s Orchard, we use time honored family recipes to make all of our apple products. The result? Simply amazing pies and applesauce made with fresh and natural ingredients. The only thing missing is mom’s oven.
  • Apple Pie Filling: Treat your family to a hot, fresh apple pie without the hassle using our prepared apple pie filling. We start with our freshly picked Honeycrisp apples; wash, core, and dice them; and prepare the filling with our secret family recipe, all in our own kitchen. Available now in our orchard store and at local retailers.
  • Applesauce: Hot or cold, applesauce is a year round treat. Our Braeburn and Honeycrisp apples give just enough sweet and tart flavors for a refreshing applesauce that’s perfect for breakfast, lunch, dinner, or dessert. Available now in our orchard store and at local retailers.
  • Where to Find Our Apple Products: Anthony’s Orchard works with a several national retail outlets. Just look for our logo next time you’re in the apple aisle at your local market!

Pick Your Own Apples: Bring the whole family and pick as many apples as you can carry. The orchard is open during apple season 8 a.m. to 6 p.m. daily. We sell all of our apple varieties by the pound. Bring your own bag and we offer a discount. Thanks to all the wonderful folks who continue visiting our Orchard. See you soon!

CSA Program, May – August: Our Community Supported Agriculture (CSA) program is a win win for you and for us. You get the best selection of our apples, picked and delivered weekly right to your door. And you help Anthony’s Orchard by becoming a loyal patron of our hard work.

We’ll deliver your CSA every Wednesday through our peak season, May – June for our Honeycrisp apples and July – August for our Braeburn apples. You can participate in our CSA as a full share member (for a family of four) or half share member (for a family of two):

  • Full share: 2 dozen per week, £10
  • Half share: 1 dozen per week, £5

For more information visit our store or look for a flyer at your local market

Fall Harvest Festival, October 1 – 31: Join us for our annual Fall Harvest Festival during the month of October with tricks and treats for the young and the young at heart. Take a spooktacular hayride through the orchard and over to our pumpkin patch, where you can pick out your very own future jack o lantern. Just watch out for our very own phantom of the orchard, old Apple Lloyd Webber!

Educational Programs: Turn our orchard into your classroom with special programs designed to enhance learning and build appreciation for farming and agriculture. Guided tours through the orchard and production facilities offer students an opportunity to learn more about where their food comes from and how it moves from the field to the market. We’ll also work with you to tailor the lessons to meet the needs of your class’ age group and your curriculum.

Welcome to the Apple Store: You’ll find plenty of our fresh apples, apple products, and other items like kitchen tools designed to help you turn apples into awesome.

Products:

  • Fresh Braeburn and Honeycrisp apples from our orchard (available year round)
  • Apple pie filling
  • Applesauce
  • Cookbooks
  • Apple presses, peelers, and corers
  • Other apple merchandise

Note: exact product selection may vary.

Store Hours: 7 days a week, 8 a.m. to 6 p.m

Location: Once you arrive at the orchard, turn left and follow the road to the big red barn. Look for the red door on the right.

About Anthony’s Orchard: Anthony’s Orchards is a 3rd generation, family owned 6,000 acre orchard situated in Wenatchee Valley, the heart of apple producing in Washington State. Surrounded by gently rolling mountains Anthony’s Orchard has been growing apples in the valley for over 60 years. Our waterfront orchards line the region’s rushing rivers. The nutrient rich soil and our advanced growing practices provide the right mix for producing some of the world’s top quality apples. Washington State has been known for its apples and produces a wide variety. At Anthony’s Orchard, we have focused on two varieties of apples: Braeburn and Honeycrisp.

Mission Statement: Our mission is to continually enhance consumers’ awareness of apples and apple orchards.

Vision Statement: Our vision is to provide the best apple products and services to our customers and generate returns for our shareholders, while enhancing the wellbeing of our community.

Careers at Anthony’s Orchard: Looking for a fulfilling career in apples? It may not be the Big Apple, but in addition to a great benefits package, Anthony’s Orchard offers all the clean air and fresh fruit you could ever want.

I
nvestor Information: Crunch into the financial information on our Investor Relations page. You’ll find the data as crisp and fresh as this morning’s harvest.

Careers at Anthony’s Orchard

Anthony’s Orchard employs about 300 year round staff plus several hundred more during peak harvest seasons. We are currently seeking candidates for the following positions:

Retail Store Sales Manager

The Sales Manager is directly responsible for overseeing the supervision and delegation of associates and staff, managing expenses, achieving profitability goals, and overseeing the corporate merchandising, and the overall management of the retail store while overseeing customer satisfaction.

For a full job description please contact Tara Olsen in our HR Department

Lead Outside Sales Rep

The Lead Outside Sales Rep is responsible for leading and overseeing sales operations with our retail partner accounts, managing inventory logistics, and managing staff scheduling, training, and development.

For a full job description please contact Tara Olsen in our HR Department

Field Trip Leader (Community Outreach Program)

Love working with kids? We need a rock star field trip leader to provide excellent customer service to our school group guests and lead on site learning programs.

For a full job description please contact Tara Olsen in our HR Department

Organizational Structure

  • Executive Committee:
    • Bob Frost, CEO
    • Mary Scott, CFO of Finance
    • John Daniels, SVP of Sales
    • Samir Saeed, SVP of Operations
  • Departmental Units:
    • Human Resources
      • Leonard Pulanski, Director of Human Resources
        • Chuck West, Benefits Manager
        • Brenda Norris, Payroll Manager
        • Tara Olsen, Recruiting Manager
          • Heather Mullins, Seasonal Labor Coordinator
        • Sean Manning, Training Manager
    • Marketing
      • Christine Miller, Director of Marketing
        • Darrius Mason, Marketing Manager
          • Kat Williams, Marketing Coordinator
          • Trevor Lewis, Market Research Analyst
          • Evan Beam, Marketing Intern
          • Jim Wiliams, Marketing Intern
          • Jack Walker, Marketing Intern
      • Bill Owens, Director of Communications
    • Sales
      • Director of Sales
        • Lead Outside Sales Rep (open)
          • Neil Faraday, Northeast US Sales Rep
          • Max Curie, Southeast US Sales Rep
          • Marie Planck, Central US Sales Rep
          • Michael Bohr, Western US Sales Rep
    • Operations & Processing
      • Allison Sinclair, Director of Operations
        • Ben Arshavin, Processing Manager
          • Felicia Jennings, Processing Floor Supervisor
          • Todd Driver, Processing Floor Supervisor
            • Processing staff (seasonal)
        • Retail Store Sales Manager (open)
          • Sun Kim, Retail Sore Supervisor
          • Taressa Johnson, Retail Store Supervisor
            • Retail Staff (seasonal)
          • CSA Supervisor
        • Shaleish Gupta, Educational Programs Manager
          • Field Trip Leader (open)
        • Shamus McFadden, Orchard Manager
          • Leo Richards, Orchard Supervisor
          • Don Smith, Orchard Supervisor
          • Mike Jones, Orchard Supervisor
          • Raphael Zirpolo, Orchard Supervisor
            • Orchard workers (seasonal)
        • AJ Martin, Purchasing Manager
        • Gabriella Nickel, Warehouse Manager
    • Finance & Accounting
      • Jeanine Lister, Director of Finance
        • Pamela Kirk, Accounting Manager
          • Scott Michaels, Accounts Payable Manager
          • Marsha Birch, Accounts Receivable Manager
    • Information Technology
      • Ted Schaeffer, Director of Information Technology
        • Bob O’Malley, Systems Manager
        • Martin Erlich, Website Manager

Note: seasonal staffing levels vary to meet business needs

Webcasts and Presentations

Earnings calls are designed to provide disclosure and financial results for a certain reporting period. In addition, earnings calls provide transparency and can help with forecasting and planning.

Listen to the latest investor call using the media player below.

Strategic Plan & Growth Initiatives

Please see below for excerpts from our Long Term Vision and strategic planning sessions:

  • In 2012, the Company is hoping to purchase an apple press, which would allow us to start a new line of prepared apple products – apple juice. Our plans include an estimated $95,000 net income per year and an estimated $90,000 of cash inflows per year from the new line of apple juice. We expect a 10% return on this investment.
  • We continue to search for new orchards throughout the country so that we may diversify the types of apples we produce. While we feel that Braeburn and Honeycrisp are among the best, we understand that you the customers might like additional variety.
  • Our 2015 revenue goal is to exceed $25 million dollars, which will involve either acquiring new orchards, or expanding our line of products.

Financial Information and Charts

  • Budget Overview
  • Anthony’s Orchard begins budget planning each year in August and finalized by the start of our fiscal year in October. Each departmental director, as defined by the company’s organizational structure, provides labor and materials cost forecasts. The Senior VP of Operations compiles these numbers to estimate total costs. The Senior VP of Sales forecasts sales based on fruit yield estimates from the Operations division, and the Senior VP of Finance prepares the final annual budget for approval by the Executive Committee and Board of Directors.
  • Anthony’s Orchard uses a FIFO accounting method to minimize crop waste. Factory overhead uses a flexible budget to respond to business demands and scale with crop yield. Variable overhead varies with the number of direct labor hours in operations areas, including processing, retail store, and harvesting.






Previous Year Financial Statement


Current Year Financial






Auditing Overview

Anthony’s Orchard has made a request to the Securities and Exchange Commission (SEC) to change their fiscal year from September to December. The request has been granted and a national accounting audit firm, Ethical and Integrity (E & I), has been engaged to provide an opinion on the financial statements ending December 2009.

Under the Sarbanes Oxley Act of 2002, Anthony’s Orchard Audit Committee, which is comprised of the Internal Auditing Department, is responsible for the oversight of the independent auditor. In addition, The Audit Committee is responsible for obtaining the appropriate input from management and considers the type and scope of work to be performed by the auditor and the audit fees associated with the audit.

The Audit Committee must comply and follow the SEC’s rules on auditor independence. Anthony’s Orchard personnel must ensure not to impair or interfere with the auditor’s independence. Personnel must not engage, in any circumstance, in services that are not permissible.

The Audit Committee has attested to the internal controls over financial reporting as of year end, section 404.

#2: Familiarise yourself with the Anthony’s Orchard company and its current situation; this can be done by exploring each of the tabs across the top of the screen in the Anthony’s Orchard case study media.
Hint: You should focus on the financial information.

Here is what the CEO says “Anthony’s Orchard is a third generation, family owned orchard. We’ve been in business well over 60 years, having been established in the years following World War II. Throughout our history, the company has experienced periods of tremendous growth, as well as a few very lean years. I think our longevity can be attributed to a number of factors. In agriculture, so much is dependent on production. We have always enjoyed a geographic advantage, as our orchards are located in some of the best regions in the world for growing apples. However—and I think this is vitally important to note—we have never been satisfied to rely on that single advantage alone. Historically, our corporate culture has always been one of fostering innovation to support our inherent advantages. We have repeatedly shown a willingness to invest in technology aimed at improving the quality and volume of our products. Since I joined the company in 1990, we have made it a priority to invest in technologies aimed at improving our growing practices. I think this practice has proven to be an invaluable tool in helping us address fluctuations in weather conditions, competition, and a variety of other threats to our market and our organization. We are excited about our future, and have aggressive plans for it. Among these plans is expansion into a new product line. We hope to be producing and distributing Anthony’s Orchard apple juice within the next 3 to 5 years. Ultimately, these and other strategic plans will depend on current and projected business conditions. We must remain mindful of cash flow, operating costs, and the threats associated with the natural environmental factors that affect our product line. We feel good about business conditions, but it is also very important to continually monitor those conditions and to consider them when developing and executing strategic plans.”

Exercise Questions:

  1. Review the Financial Statements: Analyse the current financial state of Anthony’s Orchard and evaluate the impact of a major customer cancelling their expected order.

This analysis should include the following:

  • Your view of the current financial health of the company
  • Your assessment of the materials and labour costs
  • Your assessment of the income statement
  • Your assessment of the cash flow statement
  • Your assessment of each of the business units in Anthony’s Orchard
  • C V P analysis of the current year’s financials
  1. Conduct a What If Analysis: This what if analysis concerns an unforeseen circumstance that could impact the company’s current health as well as its future plans. A major customer is considering cancelling their order for prepared apple products. This customer accounts for 25% of the prepared apple product revenue. Evaluate the impact of this on the budgeted statements contained in the case study.

Notes:

You should fully state and justify any assumptions that you choose to make in relation to the financial data you use. For example, if you make assumptions about the state of the economy or the industry over the period you cover in your analysis, be sure to state these and identify why you believe they are worth consideration here.

Be sure to include references to any sources you use as well, including external sources of economic data or industry statistics.

accounting information systems your task you are required to submit a feasibility re 685047

HA2042 – Accounting Information Systems

You are to prepare a submission to satisfy your client’s requirements as outlined below. You are

required to investigate the issues at Pressure Hydraulics and provide a feasibility study into the

improvement to their current business processes. You will need to conduct an investigation into the

client’s problems and evaluate a solution to meet their business needs.

Overview – Pressure Hydraulics

Pressure Hydraulics is a locally owned business that currently has three service centres; Newcastle,

Toronto and Maitland. Each service centre provides maintenance and specialised servicing of

hydraulic systems as used in cars, trucks and earth moving equipment. The Maitland service centre

also provides a specialty service to the mining industry where two purpose built trucks go onsite to

service a range of mining equipment.

The business has become quite profitable in the past years and its owner, Allan Taylor, has devised

plans to expand by opening service centres at Coffs Harbour and Gosford. Allan has future plans for

other service centres along the east coast of NSW. He also feels that the time is right to look at how IT

can support the existing business and enable his future business plans.

Currently, Allan spends a portion of each day at each service centre to monitor its operations. This is

leaving little time to continue developing his business and he realises he will not be able to spend the

same sort of time in the Coffs Harbour and Gosford service centres.

Existing System

Each service centre operates as an independent business, with eight technicians in the workshop (one

of whom is a workshop foreman) and one office assistant. The office assistant takes phone calls from

people requesting quotes for work or to have work done. For requests on quotes, the office assistant

looks up a hard copy of a price book (known as The Price Book) and gives a verbal quote. Allan is

unhappy with the time being taken to update the Price Book as the office assistant is often too busy to

update the prices from the supplier invoices. Sometimes, this time delay results in either lost sales if

the price has gone down or lost income if the price has gone up.

If a customer phones to make a booking for work to be done, the office staff assistant writes down the

details in a journal (Works Book). Any variations to the Works Book by a customer requesting to

change a date/time results in crossed out entries. There have been times when this has been the cause

of misinterpretation as to when a job is booked in. Other problems with the system include occasional

overbooking or slack periods for the workshop staff.

When a customer arrives at a service centre for work to be done, the office staff member writes out a

Job Card with the customer details along with the price from The Price Book for the job. There is

sometimes a discrepancy between what the Price Book shows and what the customer says they were

quoted for that particular work. When completed, the Job Card is passed to the workshop foreman

who assigns the job to a technician. When the job is completed, the technician initials the Job Card

and gives it back to the workshop foreman, who returns it to the office. The office assistant writes out

an invoice and collects payment from the customer; cash, EFTPOS and credit card are acceptable

forms of payment. Once per week, the office assistant uses the Job Cards to prepare an order for stock

replacement.

Allan has developed a business relationship with several transport companies where two technicians

will go to the transport companies’ central site and service the hydraulics on a truck. As the parts

required to perform the service can be varied, a special van equipped with basic workshop equipment

and a full range of component parts is used. The time taken for these jobs can be varied and unpredictable. Upon returning from such a job, the technicians will alert the workshop foreman that

the job has been completed. The office assistant is then informed and an invoice written out and

mailed to the transport company. Carbon copies are used to track such invoices. The Maitland service

centre provides a service to the mining industry and there have been a few repeated issues with the

incorrect component parts being taken onsite for servicing the machinery. Such mistakes not only

costs a loss in terms of travel time back to the service centre to collect the correct part, but the mining

companies have very tight maintenance schedules. Machinery has to be available and delays are not

tolerated.

Each fortnight, Allan contacts the workshop foremen and checks the hours worked by staff at the

service centre. He then prepares the payroll and writes cheques for each employee.

Each day, the office assistant banks the day’s takings and gives the bank receipt to Allan when he

makes his daily call. Usually this will be the following day from the bank deposit. Allan then does a

reconciliation of money banked with the previous day’s takings.

Apart from the issues identified above, Allan wants to achieve the following:

Ensuring repeat business from mining companies.

Improve control over stock ordering. A new system is required to manage this aspect of the

business. The ability to order stock for the three businesses instead of individual ordering

would ensure economies not being realised at the moment.

Ability to quickly determine if a required component part is in stock.

Ability to generate an invoice off data held within the system.

Improved payroll process to reduce the need to contact staff for details. Allan would prefer a

system that recorded the working hours daily and also supported staff being paid directly

through his bank system.

Tracking customers for repeat business. Allan wants a system that will track customers and

contact them at specified times with special offers.

Price Book issues. A system to have timely production of price variations to arrive at all

service centres on the same day.

Technology to support these changes.

After an initial investigation, Allan has asked your group to develop a feasibility report on providing a

suitable solution for the accounting and payroll problems. At a minimum level, your feasibility report

should contain the following generic sections:

Your Task:

You are required to submit a feasibility report.

Attachments:

cash budget for jodie products edlimit 685189

Jodie Products Limited is a merchandising company that sells stationery and other school supplies. The company is planning its cash needs for 2014/2015. In the past Jodie Products had to borrow money in order to support peak sales of back to school material, which occur during January. The following information has been provided to assist in preparing a cash budget for the period: Budgeted monthly income statement for December 2014 to March 2015 Budgeted monthly income statement for December 2014 to March 2015 Budgeted monthly income statement for December 2014 to March 2015 Budgeted monthly income statement for December 2014 to March 2015 Budgeted monthly income statement for December 2014 to March 2015 December January February March N$ N$ N$ N$ Sales 40 000 70 000 50 000 45 000 Cost of sales 24 000 42 000 30 000 27 000 Gross margin 16 000 28 000 20 000 18 000 Operating expenses Selling expenses 7 200 11 700 8 500 7 300 Administrative expenses 5 600 7 200 6 100 5 900 Net income 3 200 9 100 5 400 4 800 Note the following additional information: ? Administrative expenses include N$2 000 depreciation each month. ? 20 % of sales are cash sales. Credit sales are collected over a three month period in the ratio of 10% in the month of sale; 70% in the month following sale and 20% in the second month following sale. October sales amounted to N$30 000, and November sales to N$36 000. ? Inventory purchases are paid for within 15 days. Therefore 50% of a month?s inventory purchases are paid for in the month of purchase. The remaining 50% is paid in the following month. ? Purchases amount to 75% of the following month?s sales. ? Dividends of N$1 000 will be declared and paid in February. ? The cash balance on 30 November was N$8 000. The company must maintain a cash balance of at least N$8 000 at all times. ?

The company can borrow cash from the bank as needed. Loans are received in multiples of N$1 000. Interest paid on loans are 15% per annum, payable monthly. Prepare a cash budget for December, January and February. When will Jodie products be able to repay the loan?

a distribution center for a chain of electronics supply stores fills and ships order 685203

A distribution center for a chain of electronics supply stores fills and ships orders to retail outlets. A random sample of orders is selected as they are received and the dollar amount of the order (in thousands of dollars) is recorded, and then the time (in hours) required to fill the order and have it ready for shipping is determined. A scatterplot showing the times as the response variable and the dollar amounts (in thousands of dollars) as the predictor shows a linear trend. The least squares regression line is determined to be: yˆ= 0.76 +1.8x. A plot of the residuals versus the dollar amounts showed no pattern, and the following values were reported: Correlation r + 0.90; R2 = 0.81; standard deviation of the residuals is 0.48. What percentage of the variation in the times required to prepare an order for shipping is accounted for by the fitted line

international financial reporting standards ifrs are designed as a common global la 682115

Financial Accounting Theory (BBAC502)

Group Assignment (Group of Four) 25%

(Due Session 6.1)

“International Financial Reporting Standards (IFRS) are designed as a common global language for business affairs so that company accounts are understandable and comparable across international boundaries. They are a consequence of growing international shareholding and trade and are particularly important for companies that have dealings in several countries. They are progressively replacing the many different national accounting standards. The rules to be followed by accountants to maintain books of accounts which is comparable, understandable, reliable and relevant as per the users internal or external”. (Wikipedia)

Based on the above statement, Students are required to do research and write an essay of 2500 words (with proper references) which demonstrates their understanding about the Topic, outline the justifications and controversy of IFRSs. Journal Articles, Newspapers, Magazines and Websites will be valuable sources of information for this assignment

Document Preview:

Financial Accounting Theory (BBAC502) Group Assignment (Group of Four) 25% (Due Session 6.1) “International Financial Reporting Standards (IFRS) are designed as a common global language for business affairs so that company accounts are understandable and comparable across international boundaries. They are a consequence of growing international shareholding and trade and are particularly important for companies that have dealings in several countries. They are progressively replacing the many different national accounting standards. The rules to be followed by accountants to maintain books of accounts which is comparable, understandable, reliable and relevant as per the users internal or external”. (Wikipedia) Based on the above statement, Students are required to do research and write an essay of 2500 words (with proper references) which demonstrates their understanding about the Topic, outline the justifications and controversy of IFRSs. Journal Articles, Newspapers, Magazines and Websites will be valuable sources of information for this assignment Created with an evaluation copy of Aspose.Words. To discover the full versions of our APIs please visit: https://products.aspose.com/words/ ? FILENAME * MERGEFORMAT ?Case Study Analysis Assignment_25%? Page ? PAGE * MERGEFORMAT ?2?

Attachments:

hi this is my assessment for management accounting 2 i need to be done this assessme 682121

hi this is my assessment for management accounting 2 i need to be done this assessment

Document Preview:

BUACC 2614 – Management Accounting 2 Semester 2, 2014 Group Assignment According to Martin and Steele (2010, p.13), “The two principal professional associations in Australia – CPA Australia (the CPA) and the Institute of Chartered Accountants in Australia (the Institute) have indicated their awareness of the significance of issues of sustainability reporting and development of appropriate skill sets in word and in deed. The commitment of both organisations to sustainability principles has been shown by their adoption of, and support for, sustainability focused reporting approaches and by their opting to take up membership of the Accounting for Sustainability Forum”. According to Sikka (2008, p.291), “The burgeoning corporate social responsibility literature rarely focuses on the anti social practices of accounting firms…Accountancy firms are on a collision course with civil society”. Consider the above excerpts. In a two part essay: Discuss the stance and initiatives of the Australian accounting profession on corporate social responsibility (CSR) and sustainability. Include your views on the role of accounting and the professional accountants on CSR and sustainability. Discuss the way in which BHP Billiton has demonstrated its social and environmental accountability. BHP has been publishing Sustainability Reports or alternatively ? HYPERLINK “http://www.bhpbilliton.com/home/aboutus/sustainability/reports/Pages/Roll%20up%20Pages/2001%20Annual%20Health%20Safety%20Environment%20and%20Community%20Report%20documents.aspx” ?Annual Health, Safety, Environment and Community Report?s for over a decade. For BHP’s Sustainability Report for 2013, please go to: ? HYPERLINK “http://www.bhpbilliton.com/home/aboutus/sustainability/reports/Documents/2013/BHPBillitonSustainabilityReport2013_Interactive.pdf” ?http://www.bhpbilliton.com/home/aboutus/sustainability/reports/Documents/2013/BHPBillitonSustainabilityReport2013_Interactive.pdf? For BHP related issues in the…

Attachments:

according to the textbook the world adds about of co2 to the atmosphere from fossil 682145

According to the textbook, the world adds about __________ of CO2 to the atmosphere from fossil fuel combustion each year, whereas sustainable, stable amounts would be about __________.

Document Preview:

Online Exam 3_06 ? HYPERLINK “https://study.ashworthcollege.edu/portal/tool/9961487d 0d79 4318 9f6f 8d9718682e88/jsf/delivery/beginTakingAssessment” o “Display Table of Contents” ?Table of Contents? ?Time Remaining: 22:46:27 Part 1 of 1 ?? Question 1 of 20?5.0 Points??According to the textbook, there is a finite amount of time that our civilization can exist based on which of the following? ??   A. Our civilization is based on non renewable resources.???  B. Our production of nuclear weapons will lead to mass destruction.???  C. At the present birth and death rates, our population will outgrow its capacity to feed itself.???  D. Plagues and diseases from non industrialized countries will kill the population.???? HYPERLINK “https://study.ashworthcollege.edu/portal/tool/9961487d 0d79 4318 9f6f 8d9718682e88/jsf/delivery/beginTakingAssessment” ?Reset Selection??? Mark for Review? HYPERLINK “https://study.ashworthcollege.edu/portal/tool/9961487d 0d79 4318 9f6f 8d9718682e88/jsf/delivery/beginTakingAssessment” o “What’s This?” ? What’s This????Question 2 of 20?5.0 Points??Behind the United States, the world’s second largest emitter of energy related emissions is: ??   A. Mexico.???  B. China.???  C. The Soviet Union.???  D. Europe.???? HYPERLINK “https://study.ashworthcollege.edu/portal/tool/9961487d 0d79 4318 9f6f 8d9718682e88/jsf/delivery/beginTakingAssessment” ?Reset Selection??? Mark for Review? HYPERLINK “https://study.ashworthcollege.edu/portal/tool/9961487d 0d79 4318 9f6f 8d9718682e88/jsf/delivery/beginTakingAssessment” o “What’s This?” ? What’s This????Question 3 of 20?5.0 Points??In the year 2000, fossil fuels accounted for __________ of energy use while new renewable sources accounted for only __________. ??   A. 50%; 12%???  B. 70%; 5%???  C. 77%; 2%???  D. 97%; 3%???? HYPERLINK “https://study.ashworthcollege.edu/portal/tool/9961487d 0d79 4318 9f6f 8d9718682e88/jsf/delivery/beginTakingAssessment” ?Reset Selection??? Mark for Review? HYPERLINK…

Attachments:

analyzing manufacturing cost accounts jagger and richards company manufactures custo 682169

Analyzing Manufacturing Cost Accounts

Jagger and Richards Company manufactures custom guitars in a wide variety of styles. The following incomplete ledger accounts refer to transactions that are summarized for October:

In addition, the following information is available:

  1. Materials and direct labor were applied to six jobs in October:
  2. Factory overhead is applied to each job at a rate of 60% of direct labor cost.
  3. The October Work in Process balance consisted of two jobs, as follows:
  4. Customer jobs completed and units sold in October were as follows:

Instructions:

1. Determine the missing amounts associated with each letter by completing the table below. If an amount is zero, enter in “0”. Enter all amounts as positive numbers.

Job No. Style Quan
tity
Oct. 1
Work in
Process
Direct Materials Direct Labor Factory Overhead Total Cost Unit Cost Units Sold Cost of Goods Sold
No. 101 X 1 150 $ $37,500 $27,000 $ $ $ $
No. 102 X 3 175 48,000 33,000
No. 103 X 2 225 60,000 50,000
No. 104 S 1 175 30,000 18,000
No. 105 S 2 300 54,000 30,000
No. 106 X 4 150 30,000 14,000
Total 1,175 $ $259,500 $172,000 $ $ $

a. $

b. $

c. $

d. $

e. $

f. $

g. $

h. $

2. Determine the October 31 balances for each of the inventory accounts and factory overhead.

Materials: $
Work in Process: $
Finished Goods: $
Factory Overhead: $

bdcbcsd sdsdsdisd sslknskls zdkcnk 682242

bdcbcsd sdsdsdisd sslknskls Zdkcnk

Document Preview:

24210 34230 58440 17625 39775 19885 37515 20925 15500 960 600 17060 25700 1800 10000 750 9500 47750 43885 35.805954825462017 75.094113620807676 Trading & Profit & Loss Account for the period ended 31st December 2008 New Electrics Sales Revenue Particulars Details ($) Amount ($) Washing Machines Cookers Cost of Goods Sold Opening Stock (Add) Purchases (Less) Closing Stock Gross Profit Expenses Adversitnment & Marketing Expenses Sales Assisstant Salary Advertisnment Leaflets New Paper Advertisnment Administrative Expenses Salary & Wages (Fitter ) Salary & Wages (Part Time Accountant ) Rent Expenses Insurance Expenses Vehicale Expenses Net Profit / (Loss) (b) Gross Profit Margin Gross Profit / Sales Revenue Net Profit / Net Loss Margin Net Loss / Sales Revenue %

Attachments:

i have 2 memo each memo we need 500 word my due date is 24 09 2014 682254

Holmes Institute HA3032 Memo 02 – Semester 02, 2014

Background:

Mr. Howe, a Junior Partner of the CPA firm Dewey, Cheatem, & Howe (DCH), after noting that there is a proposal to limit Auditor liability is very excited and has announced that the DCH Audit firm will soon be able to take on much, much riskier audits with little risk of DCH going bankrupt or losing the right to audit.

Required:

Mr. Tu Dewie has asked you to evaluate Mr. Howe’s assertions (use the lectures, tutorials and the internet (i.e. http://www.clmr.unsw.edu.au/article/compliance/internal risk management/institute chartered accountants stripped liability cap). In writing your memo, please remember that while Mr. Dewie is the senior partner of DCH, Mr. Howe (as a junior partner) can hurt you if you are not polite and considerate.

A Full Service Creative Accounting Firm Loopholes Located Books Juggled Taxes Evaded Funds Diverted Money Laundered Shortages Hidden Pensions Plundered Credit Denied Parking Validated

Bankruptcies our Specialty!Holmes Institute HA3032 Memo 02 – Semester 02, 2014

Background:

Mr. Howe, a Junior Partner of the CPA firm Dewey, Cheatem, & Howe (DCH), after noting that there is a proposal to limit Auditor liability is very excited and has announced that the DCH Audit firm will soon be able to take on much, much riskier audits with little risk of DCH going bankrupt or losing the right to audit.

Required:

Mr. Tu Dewie has asked you to evaluate Mr. Howe’s assertions (use the lectures, tutorials and the internet (i.e. http://www.clmr.unsw.edu.au/article/compliance/internal risk management/institute chartered accountants stripped liability cap). In writing your memo, please remember that while Mr. Dewie is the senior partner of DCH, Mr. Howe (as a junior partner) can hurt you if you are not polite and considerate.

A Full Service Creative Accounting Firm Loopholes Located Books Juggled Taxes Evaded Funds Diverted Money Laundered Shortages Hidden Pensions Plundered Credit Denied Parking Validated

Bankruptcies our Specialty!

Attachments:

analyzing manufacturing cost accounts jagger and richards company manufactures custo 682268

Analyzing Manufacturing Cost Accounts

Jagger and Richards Company manufactures custom guitars in a wide variety of styles. The following incomplete ledger accounts refer to transactions that are summarized for October:

In addition, the following information is available:

  1. Materials and direct labor were applied to six jobs in October:
  2. Factory overhead is applied to each job at a rate of 60% of direct labor cost.
  3. The October Work in Process balance consisted of two jobs, as follows:
  4. Customer jobs completed and units sold in October were as follows:

Instructions:

1.Determine the missing amounts associated with each letter by completing the table below. If an amount is zero, enter in “0”. Enter all amounts as positive numbers.

Job No. Style Quan
tity
Oct. 1
Work in
Process
Direct Materials Direct Labor Factory Overhead Total Cost Unit Cost Units Sold Cost of Goods Sold
No. 101 X 1 150 $ $37,500 $27,000 $ $ $ $
No. 102 X 3 175 48,000 33,000
No. 103 X 2 225 60,000 50,000
No. 104 S 1 175 30,000 18,000
No. 105 S 2 300 54,000 30,000
No. 106 X 4 150 30,000 14,000
Total 1,175 $ $259,500 $172,000 $ $ $

a.$

b.$

c.$

d.$

e.$

f.$

g.$

h.$

2.Determine the October 31 balances for each of the inventory accounts and factory overhead.

Materials: $
Work in Process: $
Finished Goods: $
Factory Overhead: $

i have 14 framed hand colored prints from the series the paths of learning strewed w 682277

I have 14 framed hand colored prints from the series “The Paths of Learning Strewed with Flowers OR English Grammar Illustrated 1826.” The images are approx. 4 x2 3/4″ in 5×6″ wood frames.

I’ve had them since the 1950s, when they were hung on the wall. They’ve been boxed up for decades. Is there a market for such arcane children’s art?

ALSO…. I have a dozen silver discs that are music awards issued by Playboy magazine to the winners of its Pop and Jazz Readers’ Poll. There are medals from 1971, 72 and 73, plus 2 blank ones. They are in black presentation cases. Many are stamped with the word ‘sterling’ on the back, most are engraved with the winner’s name and category e.g. Paul McCartney, Bass, 1972. The ones that aren’t stamped ‘sterling’ are quite tarnished; the others are not (strange!). My husband worked at Playboy magazine and retrieved these items from the trash in the 1980s. Clearly the winners had not bothered to pick up their awards so…some 10 years later, they were tossed. The discs are quite heavy, but ….are they really silver? If so, there would be salvage value, if nothing else. But where to start in evaluating them?

write a report to the board of hardhat ltd setting out the financial effects of the 682290

Write a report to the Board of Hardhat Ltd setting out the financial effects of the various proposals and make recommendations as to what price Hardhat should charge next year and in the longer run.

Your report and presentation should cover:

(a) the sales price needed to earn the target profit, using the information compiled by the budget committee;

(b) the number of units that would have to be sold at the old price to meet the target profit, and whether this seems feasible;

(c) what the profit would be if the sales price calculated in (a) were adopted, but sales volume only rose by 10%, or at best 20%;

(d) any other factors you think should be taken into account when making decisions about the price to be charged next year, such as any change in risk involved in the cost—volume—profit structure you propose; the link between short and long run prices; and the interactions between acquisitions policy, financing decisions and pricing decisions.

grand valley appareal 682330

ntries and schedules for unfinished jobs and completed jobs

Grand Valley Apparel Co. uses a job order cost system. The following data summarize the operations related to production for May 2010, the first month of operations:

a. Materials purchased on account, $68,000.

b. Materials requisitioned and factory labor used:

managerial accounting

c. Factory overhead costs incurred on account, $2,750.

d. Depreciation of machinery and equipment, $1,870.

e. The factory overhead rate is $25 per machine hour. Machine hours used:

management accounting

f. Jobs completed: 401, 402 , 403, and 405.

g. Jobs were shipped and customers were billed as follows: Job 401, $26,000; Job 402, $33,400; Job 405, $23,400.

1. Journalize the entries to record the summarized operations. For a compound transaction,

ultra leather products sells leather clothing at both wholesale and retail the compa 682354

Ultra Leather Products sells leather clothing at both wholesale and retail. The company has found that there is a higher rate of uncollectible accounts from retail credit sales than from wholesale credit sales.

Ultra computes its estimate loss from uncollectible account at the end of the year. The amount is based on the rates of loss that the firm has developed from experience for each division. A separate computation is made for each of the two types of sales. The firm uses the percentage of net credit sales method. As of December 31 , 2010 Accounts Reseivable has a balance of $392,000, and allowance for Doubtful Accounts has a debit balance of 326.80. The following table provides a breakdown of the credit sales for the year 2010 and the estimate rate of loss. Category Amount Estimate rate of loss Whole sale $2,040,000 0.5% Retail 548,600 1.2% Instruction 1. Compute the estimate amount of uncollectible accounts expence for each of the two categories of net credit sales for the year.

for the year ended december 31 2014 the job cost sheets of cinta company contained t 682419

For the year ended December 31, 2014, the job cost sheets of Cinta Company contained the following data.

Job
Number
Explanation Direct
Materials
Direct
Labor
Manufacturing
Overhead
Total
Costs
7640 Balance 1/1 $33,500 $32,160 $38,592 $104,252
Current year’s costs 40,200 48,240 57,888 146,328
7641 Balance 1/1 14,740 24,120 28,944 67,804
Current year’s costs 57,620 64,320 77,184 199,124
7642 Current year’s costs 77,720 73,700 88,440 239,860

Other data:

1. Raw materials inventory totaled $20,100 on January 1. During the year, $187,600 of raw materials were purchased on account.
2. Finished goods on January 1 consisted of Job No. 7638 for $116,580 and Job No. 7639 for $123,280.
3. Job No. 7640 and Job No. 7641 were completed during the year.
4. Job Nos. 7638, 7639, and 7641 were sold on account for $710,200.
5. Manufacturing overhead incurred on account totaled $160,800.
6. Other manufacturing overhead consisted of indirect materials $18,760, indirect labor $24,120, and depreciation on factory machinery $10,720.

make sure this will be 100 palagirisim free work and i need 2000 words my deadline i 682433

make sure this will be 100% palagirisim free work and i need 2000 words my deadline is 21 september

Document Preview:

Financial Accounting Theory (BBAC502) Group Assignment (Group of Four) 25% (Due Session 6.1) “International Financial Reporting Standards (IFRS) are designed as a common global language for business affairs so that company accounts are understandable and comparable across international boundaries. They are a consequence of growing international shareholding and trade and are particularly important for companies that have dealings in several countries. They are progressively replacing the many different national accounting standards. The rules to be followed by accountants to maintain books of accounts which is comparable, understandable, reliable and relevant as per the users internal or external”. (Wikipedia) Based on the above statement, Students are required to do research and write an essay of 2500 words (with proper references) which demonstrates their understanding about the Topic, outline the justifications and controversy of IFRSs. Journal Articles, Newspapers, Magazines and Websites will be valuable sources of information for this assignment Created with an evaluation copy of Aspose.Words. To discover the full versions of our APIs please visit: https://products.aspose.com/words/ ? FILENAME * MERGEFORMAT ?Case Study Analysis Assignment_25%? Page ? PAGE * MERGEFORMAT ?2?

Attachments:

the following information was taken from the records of roland carlson inc for the y 682476

Financial Reporting I

E4 17 (Various reporting formats) The following information was taken from the records of Roland Carlson Inc for the year 2014. Income Tax applicable to income from continuing operations $187,000; income tax applicable to loss on discontinued operations $25,500; income tax applicable to extraordinary gain $32,300; income tax applicable to extraordinary loss $20,400; and unrealized holding gain on available for sale securities $15,000.

Extraordinary gain                                  $95,000                                             

Cash Dividend Declared                         150,000

Loss on discontinued operations             75,000                                               

Retained Earnings January 1,2014        600,000

Administrative expenses                        240,000                                              

Cost of Goods Sold                                850,000

Rent revenue                                           40,000                                               

Selling expenses                                   300,000

Extraordinary Loss                                   60,000                                              

Sales Revenue                                   1,900,000

Shares Outstanding during 2014 were 100,000.

Instructions:

a. Prepare single step income statement.

b. Prepare comprehensive income statement for 2014, using two statement format.

c. Prepare a retained earnings statement for 2014.

collect the following data pertaining to your branch or any nearby branch each parti 682583

Collect the following data pertaining to your branch or any nearby branch. Each participant is expected to provide data for his own branch or any branch of the ICICI bank which provides lockers.

(the type is only illustrative and you are to fill in the number of each sizes and type based on the classification at the branch)Cost of each type of locker

You are required to:

a. Calculate the profitability of the locker facility of that particular bank branch at current level of occupancy. (6 marks)

b. How many lockers must be sold in order to break even (as far as lockers are concerned)? Specify the break up of number of lockers type wise. E g Type A5 lockers, Type B 15 lockers etc (6 marks)

c. If the bank wants to achieve a minimum profit of Rs. 50,000, how many lockers must be rented out? (6 Marks)

d. Suggest measure to increase the profitability of the locker facility? (12 marks)

consolidation adjustment elimination journal entries 682607

200109 Corporate Accounting Systems

QUESTION

Using the information below and on the next two pages, prepare the following as at 30th June 2014:

PART A: Consolidation adjustment/elimination journal entries that are required at the above financial year end date (i.e. for one year only); and Non controlling Journal entries also.

THE FOLLOWING EVENTS OCCURRED
:

During the year ended 30 June 2012:

  1. On 17 October 2011 Sydney Ltd created a group entity when it purchased 90% of the issued capital of Tower Ltd for $289,980 cash. On acquisition, Tower Ltd’s accounts showed: Share capital $200,000 and Retained earnings $58,000. All assets and liabilities appearing in Tower Ltd’s financial statements were fairly valued, except:
  • One of their blocks of land was recorded at $100,000 when its fair value was judged by the group to be $130,000. During the following financial year this land was sold for $140,000 cash.
  • An item of plant was undervalued by $50,000. At that time it had a remaining life of 5 years and accumulated depreciation of $36,000. The plant is still an asset of Tower Ltd at 30 June 2014.
  • A contingent liability relating to an unsettled legal claim with a fair value of $50,000 was recorded in the notes to the financial statements. This amount will be tax deductible when paid. The court case is still in progress at 30 June 2014.

During the year ended 30 June 2013:

  1. On 1 July 2012 Tower Ltd sold an item of plant to Sydney Ltd for $59,000. This was financed by a short term interest free loan from Tower Ltd that was repaid 14 months later. The plant had cost $64,000 when purchased on 13 November 2011. It’s expected useful life was originally 5 years and this original estimate is still considered to be valid. The plant is still an asset of Sydney Ltd at 30 June 2014.
  2. During the year Sydney Ltd made sales of inventory to Tower Ltd of $569,600. The inventory balance of Tower Ltd at the end of the year included stock of $84,000 acquired from Sydney Ltd. Sydney Ltd declared and paid dividends of $90,000 for the year. Tower Ltd did not declare or pay any dividends for the year.

During the year ended 30 June 2014:

  1. On 23 December 2013 Sydney Ltd sold an item of plant to Tower Ltd for $100,000 when its carrying value in Sydney’s books was $170,000 (original cost $212,500 and original estimated life of 10 years). The plant is still an asset of Tower Ltd at 30 June 2014.
  2. During the year Tower Ltd made sales of inventory to Sydney Ltd of $88,200. The inventory balance of Sydney Ltd at the end of the year included stock of $54,300 acquired from Tower Ltd.
  3. The management of Sydney Ltd believes that the goodwill acquired on acquisition of Tower Ltd was impaired by $5,000 in the current year. This is in addition to a total of $8,000 of impairment in previous years.
  4. Sydney Ltd charged management fees to Tower Ltd.
  5. Dividends were declared/paid by both companies.
  6. Non controlling interests in Tower Ltd to be recognised. This is the only subsidiary in the group.

ADDITIONAL INFORMATION:

  • The company tax rate is currently 30% and it has been this rate for many years.
  • Sydney Ltd has the following accounting policies for the group:

(i) Revaluation adjustments on acquisition are to be made on consolidation only, not in the books of any subsidiary;

(ii) Non controlling interests are measured at fair value;

(iii) Intragroup sales of inventory to be at a selling price of cost plus a mark up of 50%;

(iv) Plant is depreciated using the straight line method with no residual value. For part years, depreciation is to be calculated on the number of days the asset is held in the relevant year, with the day of acquisition counting as one day while the day of disposal does not count; and

(v) All calculated amounts are to be rounded to the nearest whole dollar. Companies in the group do not show cents in any journals, worksheets, or financial statements.

NOTE:

  • You MUST number your journal entries and present them in the order as they relate to the number given for each “event”. Where more than one journal is needed for an “event” to be completely accounted for add the letters a,b,c,…etc to them as necessary. [For example, if three separate journal entries are required to fully record the information detailed in point number 1, then the first journal will be 1a and the second is to be 1b and the third 1c.] Short narrations are expected for each journal entry. Marks will be lost if journals are not presented in a clear and professional manner (i.e. poor or unclear presentation can include showing the debit entry on one page but the credit entry on another, or not clearly distinguishing between debit and credit entries).

AT 30 JUNE 2014 SYDNEY LTD TOWER LTD
$ $
INCOME STATEMENTS
Sales revenue 1,365,300 992,200
Cost of goods sold 692,000 618,500
Gross profit 673,300 373,700
Other income
Management fee revenue 12,900
Dividend revenue 87,960
Expenses
Depreciation expense (133,300) (59,000)
Management fee expense (12,900)
Loss on sale of asset (70,000)
Other expenses (426,200) (163,400)
Profit before tax 144,660 138,400
Income tax expense (17,010) (41,520)
Profit for the year after tax 127,650 96,880
Retained earnings at start of year 159,220 134,320
Dividend paid/declared (75,000) (96,400)
Retained earnings at end of year 211,870 134,800
BALANCE SHEETS
Equity
Share capital 450,000 200,000
Retained earnings 211,870 134,800
Current Liabilities
Accounts payable 210,280 148,180
Income tax payable 10,910 42,420
Dividends payable 37,500 48,200
Non Current Liabilities
Bank Loans 710,000 650,000
Provision for employee benefits 29,100 14,300
Deferred tax liability 6,100
1,665,760 1,237,900
Current Assets
Accounts receivable 176,800 98,700
Allowance for doubtful debts (22,100) (9,500)
Dividends receivable 43,980
Inventory 98,300 121,200
Non Current Assets
Land and buildings 790,000 910,800
Plant – at cost 449,700 301,200
Accumulated depreciation – plant (169,400) (185,400)
Deferred tax asset 900
Shares in The Rocks Pty Ltd 8,500
Investment in Tower Ltd 289,980
1,665,760 1,237,900

Attachments:

e9 20 computing depreciation three methods 10 15 min papa s fried chicken bought equ 682679

E9 20 Computing depreciation—three methods [10–15 min]Papa’s Fried Chicken bought equipment on January 2, 2012, for $39,000. The equipment was expected to remain in service for four years and to perform 11,000 fry jobs.At the end of the equipment’s useful life, Papa’s estimates that its residual value will be $6,000. The equipment performed 1,100 jobs the first year, 3,300 the second year, 4,400 the third, and 2,200 the fourth year.Requirements1. Prepare a schedule of
depreciation expenseper year for the equipment under the three depreciation methods. After two years under double declining balance depreciation, the company switched to the straight line method. Show your computations.
Note: Three depreciation schedules must be prepared.2. Which method tracks the wear and tear on the equipment most closely?

waterways have a sales mix of sprinklers valves and controllers as follows annual ex 682689

Waterways have a sales mix of sprinklers, valves, and controllers as follows:

· Annual expected sales:

Sale of sprinklers 460,000 units at $26.50

Sale of valves 1,480,000 units at $11.20

Sale of controllers 60,000 units at $42.50

· Variable manufacturing cost per unit:

Sprinklers $13.75

Valves $ 7.95

Controllers $29.75

· Fixed Manufacturing overhead cost (total) $760,000

· Variable selling and administrative expenses per unit:

Sprinklers $1.30

Valves $0.50

Controllers $3.41

· Fixed selling and administrative expenses (total) $1,600,000

Instructions

(a) Determine the sales mix based on unit sales for each product.

(b) Using the annual expected sales for these products, determine the weighted average unit contribution margin for these three products (Round to two decimal places.)

Assuming the sales mix remains the same, what is the break even point in units for these products

assignment 682704

Exercise 3 12 Preparing a classified balance sheet L.O. C4

Account Title Debit Credit
Cash $ 7,600
Accounts receivable 14,500
Office supplies 7,590
Trucks 186,000
Accumulated depreciation%u2014Trucks $ 38,316
Land 48,000
Accounts payable 11,600
Interest payable 9,000
Long term notes payable 53,000
Common stock 17,000
Retained earnings 154,078
Dividends 36,000
Trucking fees earned 128,000
Depreciation expense%u2014Trucks 24,714
Salaries expense 64,722
Office supplies expense 10,000
Repairs expense Trucks 11,868




Totals $ 410,994 $ 410,994









P.S: the company is authorized 20,000 shares of Common stock but issued 17,000 shares.

The common stock per is $1.

__________________

a. Use the above adjusted trial balance to prepare Webb Trucking Company classified balance sheet as of December 31, 2011.

b. Use the above adjusted trial balance to prepare Webb Trucking Company classified balance sheet as of December 31, 2011, knowing that the company was authorized 5,000 shares of preferred stock, of which it issued 1,000 shares during 2011; the par value of the preferred stock is $5 and was sold for $20.

california surplus inc qualifies to use the installment sales method for tax purpose 682723

California Surplus Inc. qualifies to use the installment sales method for tax purposes and sold an investment on an installment basis. The total gain of $75000 was reported for financial reporting purposes in the period of sale. The installment period is 3 years; one third of the sale price is collected in 2012 and the rest in 2013. The tax rate was 35% in 2012, and 30% in 2013 and 30% in 2014. The accounting and tax data is shown below.

Financial Accounting Tax Return
2012 (40% tax rate)
Income before temporary difference $ 175,000 $ 175,000 
Temporary difference $ 75,000 $ 25,000 
Income $ 250,000 $ 200,000 

2013 (35% tax rate)
Income before temporary difference $ 200,000 $ 200,000 
Temporary difference $ $ 25,000 
Income $ 200,000 $ 225,000 

2014 (35% tax rate)
Income before temporary difference $ 180,000 $ 180,000 
Temporary difference $ $ 25,000 
Income $ 180,000 $ 205,000 

Required:

1) Prepare the journal entries to record the income tax expense, deferred income taxes, and the income taxes payable for 2012, 2013, and 2014. No deferred income taxes existed at the beginning of 2012.
2) Explain how the deferred taxes will appear on the balance sheet at the end of each year. (Assume Installment Accounts Receivable is classified as a current asset.)
3) Show the income tax expense section of the income statement for each year, beginning with “Income before income taxes.”

credit terms purchases made on credit are due in full by the end of the billing peri 681680

Credit terms Purchases made on credit are due in full by the end of the billing period. Many firms extend a discount for payment made in the first part of the billing period. The original invoice contains a type of “short hand” notation that explains the credit terms that apply.

a. Write the short hand expression of credit terms for each of the following.

 

Cash

 

Beginning of

Cash discount

discount period

Credit period

credit period

1%

15 days

45 days

date of invoice

2

10

30

end of month

2

7

28

date of invoice

1

10

60

end of month

 

b. For each of the sets of credit terms in part a, calculate the number of days until full payment is due for invoices dated March 12.

c. For each of the sets of credit terms, calculate the cost of giving up the cash discount.

d. If the firm’s cost of short term financing is 8%, what would you recommend in regard to taking the discount or giving it up in each case?

cash discount decisions prairie manufacturing has four possible suppliers all of who 681682

Cash discount decisions Prairie Manufacturing has four possible suppliers, all of whom offer different credit terms. Except for the differences in credit terms, their products and services are virtually identical. The credit terms offered by these suppliers are shown in the following table.

Supplier

Credit terms

J

1/10 net 30 EOM

K

2/20 net 80 EOM

L

1/20 net 60 EOM

M

3/10 net 55 EOM

a. Calculate the approximate cost of giving up the cash discount from each supplier.

b. If the firm needs short term funds, which are currently available from its commercial bank at 16%, and if each of the suppliers is viewed separately, which, if any, of the suppliers’ cash discounts should the firm give up? Explain why.

c. What impact, if any, would the fact that the firm could stretch its accounts payable (net period only) by 30 days from supplier M have on your answer in part b relative to this supplier?

spontaneous sources of funds accruals when tallman haberdashery inc merged with meye 681684

Spontaneous sources of funds, accruals When Tallman Haberdashery, Inc., merged with Meyers Men’s Suits, Inc., Tallman’s employees were switched from a weekly to a bi weekly pay period. Tallman’s weekly payroll amounted to $750,000. The cost of funds for the combined firms is 11%. What annual savings, if any, are realized by this change of pay period? 15–8 Cost of bank loan Data Back Up Systems has obtained a $10,000, 90 day bank loan at an annual interest rate of 15%, payable at maturity. (Note: Assume a 360 day year.)

a. How much interest (in dollars) will the firm pay on the 90 day loan?

b. Find the effective 90 day rate on the loan.

c. Annualize your result in part b to find the effective annual rate for this loan, assuming that it is rolled over every 90 days throughout the year under the same terms and circumstances.

integrative mdash comparison of loan terms cumberland furniture wishes to establish 681687

Integrative—Comparison of loan terms Cumberland Furniture wishes to establish a prearranged borrowing agreement with its local commercial bank. The bank’s terms for a line of credit are 3.30% over the prime rate, and each year the borrowing must be reduced to zero for a 30 day period. For an equivalent revolving credit agreement, the rate is 2.80% over prime with a commitment fee of 0.50% on the average unused balance. With both loans, the required compensating balance is equal to 20% of the amount borrowed. The prime rate is currently 8%. Both agreements have $4 million borrowing limits. The firm expects on average to borrow $2 million during the year no matter which loan agreement it decides to use.

a. What is the effective annual rate under the line of credit?

b. What is the effective annual rate under the revolving credit agreement?

c. If the firm does expect to borrow an average of half the amount available, which arrangement would you recommend for the borrower? Explain why.

accounts receivable as collateral kansas city castings kcc is attempting to obtain t 681689

Accounts receivable as collateral Kansas City Castings (KCC) is attempting to obtain the maximum loan possible using accounts receivable as collateral. The firm extends net 30 day credit. The amounts that are owed KCC by its 12 credit customers, the average age of each account, and customer’s average payment period are as shown in the following table.

 

Account

Average age

Average payment

Customer

receivable

of account

period of customer

A

$37,000

40 days

30 days

B

42,000

25

50

C

15,000

40

60

D

8,000

30

35

E

50,000

31

40

F

12,000

28

30

G

24,000

30

70

H

46,000

29

40

I

3,000

30

65

J

22,000

25

35

K

62,000

35

40

L

80,000

60

70

a. If the bank will accept all accounts that can be collected in 45 days or less as long as the customer has a history of paying within 45 days, which accounts will be acceptable? What is the total dollar amount of accounts receivable collateral? (Note: Accounts receivable that have an average age greater than the customer’s average payment period are also excluded.)

b. In addition to the conditions in part a, the bank recognizes that 5% of credit sales will be lost to returns and allowances. Also, the bank will lend only 80% of the acceptable collateral (after adjusting for returns and allowances). What level of funds would be made available through this lending source?

accounts receivable as collateral cost of borrowing maximum bank has analyzed the ac 681691

Accounts receivable as collateral, cost of borrowing Maximum Bank has analyzed the accounts receivable of Scientific Software, Inc. The bank has chosen eight accounts totaling $134,000 that it will accept as collateral. The bank’s terms include a lending rate set at prime?3% and a 2% commission charge. The prime rate currently is 8.5%. a. The bank will adjust the accounts by 10% for returns and allowances. It then will lend up to 85% of the adjusted acceptable collateral. What is the maximum amount that the bank will lend to Scientific Software?

b. What is Scientific Software’s effective annual rate of interest if it borrows $100,000 for 12 months? For 6 months? For 3 months? (Assume that the prime rate remains at 8.5% during the life of the loan.)

inventory financing raymond manufacturing faces a liquidity crisis mdash it needs a 681693

Inventory financing Raymond Manufacturing faces a liquidity crisis—it needs a loan of $100,000 for 30 days. Having no source of additional unsecured borrowing, the firm must find a secured short term lender. The firm’s accounts receivable are quite low, but its inventory is considered liquid and reasonably good collateral. The book value of the inventory is $300,000, of which $120,000 is finished goods.

(1) City Wide Bank will make a $100,000 trust receipt loan against the finished goods inventory. The annual interest rate on the loan is 12% on the outstanding loan balance plus a 0.25% administration fee levied against the $100,000 initial loan amount. Because it will be liquidated as inventory is sold, the average amount owed over the month is expected to be $75,000.

(2) Sun State Bank will lend $100,000 against a floating lien on the book value of inventory for the 30 day period at an annual interest rate of 13%.

(3) Citizens’ Bank and Trust will lend $100,000 against a warehouse receipt on the finished goods inventory and charge 15% annual interest on the outstanding loan balance. A 0.5% warehousing fee will be levied against the average amount borrowed. Because the loan will be liquidated as inventory is sold, the average loan balance is expected to be $60,000.

a. Calculate the dollar cost of each of the proposed plans for obtaining an initial loan amount of $100,000.

b. Which plan do you recommend? Why?

c. If the firm had made a purchase of $100,000 for which it had been given terms of 2/10 net 30, would it increase the firm’s profitability to give up the discount and not borrow as recommended in part b? Why or why not?

selecting kanton company rsquo s financing strategy and unsecured short term borrowi 681694

Selecting Kanton Company’s Financing Strategy and Unsecured Short Term Borrowing Arrangement

Morton Mercado, the CFO of Kanton Company, carefully developed the estimates of the firm’s total funds requirements for the coming year. These are shown in the following table.

Month

Total funds

Month

Total funds

January

$1,000,000

July

$6,000,000

February

1,000,000

August

5,000,000

March

2,000,000

September

5,000,000

April

3,000,000

October

4,000,000

May

5,000,000

November

2,000,000

June

7,000,000

December

1,000,000

In addition, Morton expects short term financing costs of about 10% and long term financing costs of about 14% during that period. He developed the three possible financing strategies that follow:

Strategy 1—Aggressive: Finance seasonal needs with short term funds and permanent needs with long term funds.

Strategy 2—Conservative: Finance an amount equal to the peak need with long term funds and use short term funds only in an emergency.

Strategy 3—Tradeoff: Finance $3,000,000 with long term funds and finance the remaining funds requirements with short term funds.

Using the data on the firm’s total funds requirements, Morton estimated the average annual short term and long term financing requirements for each strategy in the coming year, as shown in the following table.

 

Average annual financing

Strategy 1

Strategy 2

Strategy 3

Type of financing

(aggressive)

(conservative)

(tradeoff)

Short term

$2,500,000

$ 0

$1,666,667

Long term

1,000,000

7,000,000

3,000,000

To ensure that, along with spontaneous financing from accounts payable and accruals, adequate short term financing will be available, Morton plans to establish an unsecured short term borrowing arrangement with its local bank,

Third National. The bank has offered either a line of credit agreement or a revolving credit agreement. Third National’s terms for a line of credit are an interest rate of 2.50% above the prime rate, and the borrowing must be reduced to zero for a 30 day period during the year. On an equivalent revolving credit agreement, the interest rate would be 3.00% above prime with a commitment fee of 0.50% on the average unused balance. Under both loans, a compensating balance equal to 20% of the amount borrowed would be required. The prime rate is currently 7%. Both the line of credit agreement and the revolving credit agreement would have borrowing limits of $1,000,000. For purposes of his analysis, Morton estimates that Kanton will borrow $600,000 on the average during the year, regardless which financing strategy and loan arrangement it chooses.

Required

a. Determine the total annual cost of each of the three possible financing strategies.

b. Assuming that the firm expects its current assets to total $4 million throughout the year, determine the average amount of net working capital under each financing strategy.

c. Using the net working capital found in part b as a measure of risk, discuss the profitability–risk tradeoff associated with each financing strategy. Which strategy would you recommend to Morton Mercado for Kanton Company? Why?

d. Find the effective annual rate under:

(1) The line of credit agreement.

(2) The revolving credit agreement.

e. If the firm does expect to borrow an average of $600,000, which borrowing arrangement would you recommend to Kanton? Explain why.

in january 2004 teresa leal was named treasurer of casa de dise ntilde o she decided 681695

Case de Diseño

In January 2004, Teresa Leal was named treasurer of Casa de Diseño. She decided that she could best orient herself by systematically examining each area of the company’s financial operations. She began by studying the firm’s short term financial activities. Casa de Diseño is located in southern California and specializes in a furniture line called “Ligne Moderna.” Of high quality and contemporary design, the furniture appeals to the customer who wants something unique for his or her home or apartment. Most Ligne Moderna furniture is built by special order, because a wide variety of upholstery, accent trimming, and colors are available. The product line is distributed through exclusive dealership arrangements with well established retail stores. Casa de Diseño’s manufacturing process virtually eliminates the use of wood. Plastic and metal provide the basic framework, and wood is used only for decorative purposes. Casa de Diseño entered the plastic furniture market in late 1998. The company markets its plastic furniture products as indoor–outdoor items under the brand name “Futuro.” Futuro plastic furniture emphasizes comfort, durability, and practicality and is distributed through wholesalers. The Futuro line has been very successful, accounting for nearly 40 percent of the firm’s sales and profits in 2003. Casa de Diseño anticipates some additions to the Futuro line and also some limited change of direction in its promotion in an effort to expand the applications of the plastic furniture. Ms. Leal has decided to study the firm’s cash management practices. To determine the effects of these practices, she must first determine the current operating and cash conversion cycles. In her investigations, she found that Casa de Diseño purchases all of its raw materials and production supplies on open account. The company is operating at production levels that preclude volume discounts. Most suppliers do not offer cash discounts, and Casa de Diseño usually receives credit terms of net 30. An analysis of Casa de Diseño’s accounts payable showed that its average payment period is 30 days. Leal consulted industry data and found that the industry average payment period was 39 days. Investigation of six California furniture manufacturers revealed that their average payment period was also 39 days. Next, Leal studied the production cycle and inventory policies. Casa de Diseño tries not to hold any more inventory than necessary in either raw materials or finished goods. The average inventory age was 110 days. Leal determined that the industry standard, as reported in a survey done by Furniture Age, the trade association journal, was 83 days. Casa de Diseño sells to all of its customers on a net 60 basis, in line with the industry trend to grant such credit terms on specialty furniture. Leal discovered, by aging the accounts receivable, that the average collection period for the firm was 75 days. Investigation of the trade association’s and California manufacturers’ averages showed that the same collection period existed where net 60 credit terms were given. Where cash discounts were offered, the collection period was significantly shortened. Leal believed that if Casa de Diseño were to offer credit terms of 3/10 net 60, the average collection period could be reduced by 40 percent. Casa de Diseño was spending an estimated $26,500,000 per year on operating cycle investments. Leal considered this expenditure level to be the minimum she could expect the firm to disburse during 2004. Her concern was whether the firm’s cash management was as efficient as it could be. She knew that the company paid 15 percent annual interest for its resource investment. For this reason, she was concerned about the financing cost resulting from any inefficiencies in the management of Casa de Diseño’s cash conversion cycle.

Required

a. Assuming a constant rate for purchases, production, and sales throughout the year, what are Casa de Diseño’s existing operating cycle (OC), cash conversion cycle (CCC), and resource investment needs?

b. If Leal can optimize Casa de Diseño’s operations according to industry standards, what will Casa de Diseño’s operating cycle (OC), cash conversion cycle (CCC), and resource investment need be under these more efficient conditions?

c. In terms of resource investment requirements, what is the cost of Casa de Diseño’s operational inefficiency?

d. (1) If in addition to achieving industry standards for payables and inventory, the firm can reduce the average collection period by offering credit terms of 3/10 net 60, what additional savings in resource investment costs will result from the shortened cash conversion cycle, assuming that the level of sales remains constant?

(2) If the firm’s sales (all on credit) are $40,000,000 and 45% of the customers are expected to take the cash discount, by how much will the firm’s annual revenues be reduced as a result of the discount?

(3) If the firm’s variable cost of the $40,000,000 in sales is 80%, determine the reduction in the average investment in accounts receivable and the annual savings that will result from this reduced investment, assuming that sales remain constant. (Assume a 360 day year.)

(4) If the firm’s bad debt expenses decline from 2% to 1.5% of sales, what annual savings will result, assuming that sales remain constant?

(5) Use your findings in parts (2) through (4) to assess whether offering the cash discount can be justified financially. Explain why or why not.

e. On the basis of your analysis in parts a through d, what recommendations would you offer Teresa Leal?

f. Review for Teresa Leal the key sources of short term financing, other than accounts payable, that she may consider in order to finance Casa de Diseño’s resource investment need calculated in part b. Be sure to mention both unsecured and secured sources.

what is the contribution margin per unit 2 what is the contribution margin ratio 681806

(Answer each question independently and always refer to the original data unless instructed otherwise.) 1. What is the contribution margin per unit? 2. What is the contribution margin ratio? 3. What is the variable expense ratio? 4. If sales increase to 1.001 units, what would be the increase in net operating income? 5. If sales decline to 900 units. what would he the net operating income? 6. If the selling price increases by $2 per unit and the sales volume decrea.ses by 100 units, what would be the net operating income? 7. If the variable cost per unit increases by SI, spending on advertising increases by 51.500. and unit sales increase by 250 units, what would he the net operating income? 8. What is the break even point in unit sales? 9. What is the break even point in sales dollars? 10. How many units must be sold to achieve a target profit of $5.01)0? 11. What is the margin of safety in dollars? What is the margin of safety percentage? 12. What is the degree of operating leverage? 13. Using the degree of operating leverage, what is the estimated percent increase in net operating income of a 5% increase in sales? 14. Assume that the amounts of the company’s total variable expenses and total fixekexpensts were reversed In other words. assume that the total variable expenses arc S6,000 and the total fixed expenses arc 512.000. Under this scenario and assuming that total sales remain the same. what is the degree of operating leverage? 15. Using the degree of operating leverage that you computed in the previous question, what is the estimated percent increase in net operating income of a 5% increase in sales?

Document Preview:

The Foundational 15 Sales Contribution Margin Net operating income Units 3 Fixed Expenses Variable expenses 1. Total contribution margin Total units sold Contribution margin per unit units per unit 2. Total contribution margin Total sales Contribution margin ratio Total variable expenses Variable expense ratio 4 Increase in unit sales Increase in net operating income unit 5. Contribution margin Fixed expenses Per unit 6. 7 8 Profit = Unit CM X Q Fixed expenses 9. Profit CM ration X Sales Fixed Expenses 10 11 Break even sales at units Margin of safety (in dollars) Margin of safety percentage % per unit $ Units sold 12 Degree of operating leverage 13 % increase in sales % increase in net operating income 14 15 $20,000.00 1.00 $20.00 1000.00 $12,000.00 0.60 $12.00 $8,000.00 0.40 $8.00 $6,000.00 $2,000.00 The Foundational 15 Sales Contribution Margin Net operating income Units 3 Fixed Expenses Variable expenses 1. Total contribution margin Total units sold Contribution margin per unit units per unit 2. Total contribution margin Total sales Contribution margin ratio Total variable expenses Variable expense ratio 4 Increase in unit sales Increase in net operating income unit 5. Contribution margin Fixed expenses Per unit 6. 7 8 Profit = Unit CM X Q Fixed expenses 9. Profit CM ration X Sales Fixed Expenses 10 11 Break even sales at units Margin of safety (in dollars) Margin of safety percentage % per unit $ Units sold 12 Degree of operating leverage 13 % increase in sales % increase in net operating income 14 15 $20,000.00 1.00 $20.00 1000.00 $12,000.00 0.60 $12.00 $8,000.00 0.40 $8.00 $6,000.00 $2,000.00

par corporation acquired a 70 percent interest in sul corporation s outstanding voti 681811

Par Corporation acquired a 70 percent interest in Sul Corporation’s outstanding voting common stock on January 1, 2011, for $490,000 cash. The stockholders’ equity (book value) of Sul on this date consisted of $500,000 capital stock and $100,000 retained earnings. The differences between the fair value of Sul and the book value of Sul were assigned $5,000 to Sul’s undervalued inventory, $14,000 to undervalued buildings, $21,000 to undervalued equipment, and $40,000 to previously unrecorded patents. Any remaining excess is goodwill. The undervalued inventory items were sold during 2011, and the undervalued buildings and equipment had remaining useful lives of seven years and three years, respectively. The patents have a 40 year life. Depreciation is straight line. At December 31, 2011, Sul’s accounts payable include $10,000 owed to Par. This $10,000 account payable is due on January 15, 2012. Separate financial statements for Par and Sul for 2011 are summarized as follows (in thousands):



REQUIRED: Prepare consolidation workpapers for Par Corporation and Subsidiary for the year ended December 31, 2011. Use an unamortized excessaccount.

the global organizational hierarchy goh 681815

Practice Case Workbook
Exercise 1: TeamRisk

Watch the TeamRisk demonstration video and take notes. Use the information provided in the video to answer the following questions and critical thinking task.

Multiple Choice

  1. The Global Organizational Hierarchy (GOH) can include all of the following except:
    1. Divisions
    2. Risks
    3. Business Units
    4. Activities
  2. An entity’s contributors may include:
    1. Auditors but not contacts
    2. Contacts but not auditors
    3. Neither auditor nor contacts, only TeamRisk administrators
    4. Both auditors and contacts
  3. Which of the following risk assessment elements are not stored in TeamStore?
    1. Risks
    2. Objectives
    3. Entities
    4. Controls
  4. Which of the following statements regarding the self assessment process is incorrect?
    1. Self assessment allows both auditors and contacts to provide input for a risk assessment.
    2. Self assessments are performed using a web based interface called TeamRisk Web.
    3. Self assessment scores automatically update the final risk score for the entity without any action required by the audit department.
    4. Self assessments can allow for the contact or audit partner to identify additional risks.
  5. When building an internal audit plan in TeamRisk, the system provides all of the following information except:
    1. Risk score by entity
    2. The estimated start date of the audit
    3. Cycle driven requirement status
    4. Whether or not the entity is already linked to an existing internal audit engagement

True/False

  1. The Risk and Controls Library is static and cannot be changed once it is setup.
    1. True
    2. False
  2. Classification of Risks is limited to the following types: Operational, Financial, Compliance, and Strategic.
    1. True
    2. False
  3. When identifying entities for the internal audit plan, only one entity may be linked to each audit engagement for the year.
    1. True
    2. False

Discussion Questions

  1. Explain the difference between evaluating inherent risks and residual risks, and how TeamRisk facilitates the evaluation process.
  2. Explain the purpose of the Global Organizational Hierarchy and its relationship with the COSO ERM framework.

Critical Thinking Task

  1. Create a Global Organizational Hierarchy for Champion Pharmaceutical, Inc. (CPI) using the information about the company provided in the Introduction to the TeamMate Practice Case. Then choose one entity, define two objectives for that entity, and describe two risks that threaten the achievement of each objective.
  2. Using the HeatMap reports in Appendix A and Appendix B, identify the three highest priority internal audit engagements. Justify your reasoning for each engagement identified.

Appendix A – HeatMap Inherent Risk Scores

Appendix B – HeatMap Residual Risk Scores

Attachments:

you are an accountant at west group accountants investment advi 681887

You are an accountant at West Group
Accountants &
Investment Advisers. You have been approached by a group of investors for your professional advice on investing in Harvey Norman Holdings Limited. Your client is a strong believer in supporting Australian made products and
Socially Responsible Investing. The latter would mean adopting an investment strategy which seeks to consider both financial return and social good.

Required:

Go to:

http://www.harveynormanholdings.com.au/pdf_files/2013_Annual_Report.pdf

You +1’d this publicly.Undo

and access the company’s annual report for 2013.

Prepare a report for your client. Your report should include:

  1. A description of the core business of the company including full details of its operating activities.
  2. A discussion on any significant issues emerging from the Chairman’s Report.
  3. A discussion on any significant issues emerging from the Director’s Report.
  4. A discussion on company’s Corporate Governance Statement.
  5. A calculation of the key financial ratios for 2013.
  6. An overall evaluation of the company and your recommendation on investing in the company.

Please note the following:

  • Format: Business report
  • Contribution to overall assessment: 20%
  • Length: 2000 – 2200 words
  • Due date: ??
  • Your work must comply with the University’s General Guide for the Presentation of Academic Work.
  • Two Useful links.
  • http://federation.edu.au/students/assistance support and services/academic support/learning and study/resources
  • http://www.federation.edu.au/current students/assistance, support and services/academic support/learning and study/resources/general guide for the presentation of academic work
  • http://unilearning.uow.edu.au/essay/2cl
  • This is a group assignment. Each group needs to have 2 to 3 members in it. Please organise yourselves into groups.
  • Please make sure that names and ID numbers of all group members are stated on the cover sheet of your submission.

As this is a group assignment, each member of your group is awarded the same mark. Working in groups has its pros and cons. I am sure that you will hold constructive and energetic discussions on the issues at hand. In case of any disagreements, you will be able to resolve them in a democratic and rational way. There will be times when you may have to agree to disagree with each other. Invariably different group members bring different skills to a project; it is up to you to make the best of it. I believe one can learn a lot by discussing the issues with one’s colleagues.

If you happen to find your group members are “not pulling their weight” or there are problems with any member’s commitment, then please try to resolve those issues amongst yourselves. Open and honest communication always helps. If you are unable to resolve these issues, you are most welcome to see me and we will try to sort out the problems together. Do this as soon as possible and certainly before the due date.

Attachments:

analyse how applying these voluntary guidelines might alter information presented in 681932

Task
In recent years directors and CEOs of companies have been placing more importance on holistic reporting of company activities rather than simply reporting what is required by the Accounting Standards, the Corporations Act and other legislation.
The Global Reporting Initiative is an international not for profit organisation that has pioneered and developed the world’s most widely used voluntary Sustainability Reporting Framework.(http://www.asx.com.au/documents/public consultations/global reporting initiative and sba submission 15Nov13.pdf)
In 2013 the fourth iteration of the Sustainability Reporting Framework guidelines were released.These can be accessed via the link below.
https://www.globalreporting.org/resourcelibrary/GRIG4 Part1 Reporting Principles and Standard Disclosures.pdf
Required
Referring specifically to section 5.1 and 5.2 of the above report, analyse how applying these voluntary guidelines might alter the information presented in the Flight Centre 2013 Annual report. Apply section 5.1 and 5.2 to triple bottom line reporting and balance score card under sustainability report.
Structure of Report
Your assignment needs to be presented as a standard business report including an

  • Executive summary
  • Table of contents
  • Introduction (Plz mention the sustainability report which FLIGHT CENTRE has followed)
  • Body (detailed explanation of section 5.1 and 5.2 to Triple Bottom Line and Balance score card under Sustainability report and how it affect Flight Centre)
  • Conclusion and Recommendations
  • References (at least 15 APA journal articles)
Document Preview:

Sustainability Reporting Framework? Length: 2500 words ?Task In recent years directors and CEOs of companies have been placing more importance on holistic reporting of company activities rather than simply reporting what is required by the Accounting Standards, the Corporations Act and other legislation.  The Global Reporting Initiative is an international not for profit organisation that has pioneered and developed the world’s most widely used voluntary Sustainability Reporting Framework. (? HYPERLINK “http://www.asx.com.au/documents/public consultations/global reporting initiative and sba submission 15Nov13.pdf” ?http://www.asx.com.au/documents/public consultations/global reporting initiative and sba submission 15Nov13.pdf?) ???In 2013 the fourth iteration of the Sustainability Reporting Framework guidelines were released. These can be accessed via the link below. ? HYPERLINK “https://www.globalreporting.org/resourcelibrary/GRIG4 Part1 Reporting Principles and Standard Disclosures.pdf” ?https://www.globalreporting.org/resourcelibrary/GRIG4 Part1 Reporting Principles and Standard Disclosures.pdf?? Required   Referring specifically to section 5.1 and 5.2 of the above report, analyse how applying these voluntary guidelines might alter the information presented in the Flight Centre 2013 Annual report. Apply section 5.1 and 5.2 to triple bottom line reporting and balance score card under sustainability report. Structure of Report Your assignment needs to be presented as a standard business report including an Executive summary Table of contents Introduction (Plz mention the sustainability report which FLIGHT CENTRE has followed) Body (detailed explanation of section 5.1 and 5.2 to Triple Bottom Line and Balance score card under Sustainability report and how it affect Flight Centre) Conclusion and Recommendations References (at least 15 APA journal articles) NOTE: Please make sure there is no plagiarism as we are not going to submit hard copy; please include at…

Attachments:

response to directors shows little or limited understanding of topic 681949

hi.i want u to make my finanacial accounting assignmnet which is given in the attached files.Also aal the requriemnts which u ave to keep in mind during makiing this assignmnet has been given in Requiement of marking crietera file.plz also make all the assignmnet best of ur quality and write it on ur own words.do no copy and paste the data bcoz i hav to put this assignmenet in turnitin to verify if it is copied or not.AND plz give me the correct and detailed answers of all the numeriacls and theory questions.for 1st theory question u have to conult the book of financial accounting which the text book name is given in question 1 requirement.thanks

Assignment 2

Value: 15%

Task

This assessment task consists of four (4) questions. A total of 60 marks are allocated to the questions below, which will then be converted to a mark out of 15%.

Rationale

This assessment task is designed to assess your understanding of topics 3 to 6.

Question 1 [15 marks]

Events after the reporting period

Bob Ltd is finalising its financial statements for the reporting period ending 30 June 2014. On 21 July 2014, before the financial statements have been finalised and authorised for issue, the company’s directors became aware of the following situations:

a) 2 July 2014: The directors proposed a dividend of $10,000.

b) 3 July 2014: The directors approved the sale of an off shore agency to another entity for a profit of $30,000.

c) 4 July 2014: The company received an invoice from a supplier for $85,000 for goods delivered in June; the goods were included in closing inventory at an estimated cost of $100,000.

d) 5 July 2014: The company executed a guarantee in favour of the banks for an outstanding loan of $1,000,000 that the bank made to X Ltd, the company’s major supplier, in January of that year; the guarantee was executed because the bank was demanding payment, which would have disrupted inventory supplies.

e) 6 July 2014: An agreement was signed to take over a production facility in Adelaide at a cost of $5,000,000, which will be paid for using a long term finance lease.

f) 7 July 2014: The Australian Taxation Office waived fines for the inclusion of incorrect information in the company’s 2012 income tax return; the adjusted return was reflected in the company’s financial statements and the fine of $30,000 was recognised as an expense and liability at reporting date.

Required:

i) Given that financial statements are prepared for the financial period up to the reporting date, explain why there is a need for a standard that refers to events occurring after the reporting date.

(3 marks)

ii) Explain whether the above events will be classified as either adjusting or non adjusting events after the end of the reporting period (assuming the amount is material), providing reasons for your decision. State the appropriate accounting treatment for each event in Bob Ltd’s 2014 financial statements.

(12 marks)

(Source: Adapted from Deegan, C. (2010). Australian financial accounting. (6th edition) Sydney: McGraw Hill.)

Marking Guide Question 1 Max. marks awarded

i)

Discussion re the need for a standard that refers to events occurring after the reporting date. 3

ii)

Classification and justification/discussion of events 6

Stating the appropriate accounting treatment for each event 6

Total 15

Question 2 [15 marks]

Accounting for share capital

The constitution of Henrietta Sweeney Ltd indicated that the company could issue up to 5,000,000 ordinary shares and 1,000,000 preference shares. Prospectuses had been published offering 1,000,000 preference shares at $1.50 payable in full on application by 31 March 2014, and 2,000,000 ordinary shares at $1.20 with 50% due on application by 31 March 2014, 25% due on allotment, and 25% due on a call to be made by the directors at a later date.

By 31 March 2014, the company had received amounts due on 800,000 of the preference shares and on applications for 2,400,000 ordinary shares. On 15 April 2014, the ordinary and preference shares were allotted. The ordinary shares were allotted to applicants on a pro rata basis and the amounts received in excess of that due were to be credited against amounts due on allotment. The amount due on allotment of the ordinary shares was due by 15 May 2014 and this was received on all shares.

The directors made the call on the ordinary shares on 31 August 2014, with amounts due by 30 September. By this date, amounts due on 1,997,000 ordinary shares had been received. On 15 October 2014, the shares on which call money was not received were forfeited and sold as fully paid. An amount of $0.75 was received for each share sold. Costs of the forfeiture and reissue amounted to $800, and were paid. The constitution does not provide for refund of any balance in the forfeited shares account after reissue to former shareholders.

Required:

Prepare the journal entries to record the transactions of Henrietta Sweeney Ltd up to and including that which took place on 15 October 2014. Show all relevant dates, narrations and workings.

(Source: Adapted from Dagwell, R., Wines, G., Lambert, C. (2012). Corporate Accounting in Australia. (1st edition) Sydney: Pearson Australia.)

Marking Guide Question 2 Max. marks awarded

Journal entries 11.5

Dates 1

Narrations 1

Workings 1.5

Total 15

Question 3 [15 marks]

Accounting for income tax

Twinkle Ltd commences operations on 1 July 2013 and presents its first Statement of Profit or Loss and Other Comprehensive Income, and first Statement of Financial Position on 30 June 2014. The statements are prepared before considering taxation. The following information is available:

Statement of Profit or Loss and Other Comprehensive Income for the year ended 30 June 2014

$ $

Gross profit 420,000

Royalty revenue (exempt income) 30,000

Expenses:

Administration expenses 75,000

Salaries 150,000

Long service leave 15,000

Warranty expenses 20,000

Depreciation expense plant 80,000

Insurance 30,000 370,000

Accounting profit before tax 80,000

Assets and liabilities as disclosed in the Statement of Financial Position as at 30 June 2014

$ $

Assets

Cash 10,000

Inventory 110,000

Accounts receivable 40,000

Prepaid insurance 15,000

Goodwill 20,000

Plant – cost 400,000

Less: accumulated depreciation 80,000 320,000

Total assets 515,000

Liabilities

Accounts payable 35,000

Provision for warranty expenses 10,000

Loan payable 225,000

Provision for long service leave 15,000

Total liabilities 285,000

Net assets 230,000

Other information:

• All administration and salaries expenses incurred have been paid as at year end.

• None of the long service leave expense has actually been paid. It is not deductible until it is actually paid.

• Warranty expenses were accrued and, at year end, actual payments of $10,000 had been made. Deductions are available only when the amounts are paid and not as they are accrued.

• Actual amounts paid for insurance are allowed as a tax deduction.

• Amounts received from sales, including those on credit terms, are taxed at the time the sale is made.

• The plant is depreciated over five years for accounting purposes, but over four years for taxation purposes.

• The tax rate is 30%.

Required:

i) Determine the balance of any current and deferred tax assets and liabilities (using appropriate worksheets) as at 30 June 2014, in accordance with AASB 112. Show all necessary workings.

(9 marks)

ii) Prepare the journal entries to record the current tax liability and movements in deferred tax assets and liabilities.

(2 marks)

iii) What would your answer for part (a) if the following items on the statement of profit or loss and other comprehensive income were changed: ‘Gross profit’ was $360,000 (instead of $420,000) and the ‘Royalty revenue (exempt income)’ was $90,000 (instead of $30,000). Show all calculations and necessary workings.

(4 marks)

(Source:Adapted from Deegan, C. (2010). Australian financial accounting. (6th edition) Sydney: McGraw Hill.)

Marking Guide Question 3 Max. marks awarded

i)

Determination of taxable income and current tax liability, and workings 4

Determination of deferred tax balances 5

ii)

Journal entries 2

iii)

Determination of impact on current and deferred tax balances, and workings 4

Total 15

Question 4 [15 marks]

Property, plant and equipment

Petersen Ltd has the following land and buildings in its accounts as at 30 June 2014:

$

Residential land, at cost 1,000,000

Factory land, at valuation 2011 900,000

Buildings, at valuation 2010 800,000

Accumulated depreciation (100,000)

At 30 June 2014, the balance of the revaluation surplus is $200,000, of which $100,000 relates to the factory land and $100,000 to the buildings. On this same date, independent valuations of the land and building are obtained. In relation to the above assets, the assessed fair values at 30 June 2014 are:

$

Residential land, previously recorded at cost 1,100,000

Factory land, previously revalued in 2011 700,000

Buildings, previously revalued in 2010 900,000

The company has adopted fair value for the valuation of non current assets.

The company tax rate is 30%.

Required:

i) Prepare journal entries to record the revaluations on 30 June 2014. Petersen Ltd classifies the residential land and factory land as different classes of assets.

(12 marks)

ii) The directors of Petersen Ltd are now concerned about the impact of reporting the decline in the fair value of the factory land in the company’s financial statements. They have now asked you (the company accountant) to use the 2011 valuation for the 2014 financial statements, stating that the decline in value of the factory land is only temporary and will increase again in the near future, after a nearby multi million dollar development is approved. You need to prepare a response to the directors’ request. Provide references to AASB 116 to support your answer. (Word limit: 200 words)

(3 marks)

(Source: Adapted from Deegan, C. (2010). Australian financial accounting. (6th edition) Sydney: McGraw Hill.)

Marking Guide Question 4 Max. marks awarded

i)

Journal entries 12

ii)

Response to directors’ request 3

Total 15

Marking criteria

The assessment rubric for this assessment task is provided below. The detailed allocation of marks for each question has been provided above for your information.

Criteria Exceeds expectation (HD/D)

Meets expectation (CR/PS) Fails to meet expectation (FL)

Question 1:

Apply relevant accounting principles to events occurring after reporting date.

correctly explain why there is a need for a standard that refers to events occurring after the reporting date;

classify adjusting and non adjusting events and state appropriate accounting treatment without flaw/with minor flaws;

explanations shown are exemplary and clear. correctly explain why there is a need for a standard that refers to events occurring after the reporting date;

classify adjusting and non adjusting events and state appropriate accounting treatment with some errors;

explanations shown are adequate. fails to accurately explain why there is a need for a standard that refers to events occurring after the reporting date;

fails to classify most of the adjusting and non adjusting events and state appropriate accounting treatment;

explanations shown are inadequate.

Question 2:

Prepare journal entries to account for share issue transactions.

all entries made are accurate/with minor flaws;

dates shown are correct for the transactions;

narrations are shown.

workings shown are logical and well presented. most of the entries made are correct with some errors;

dates shown are mostly correct for the transactions;

narrations are shown.

workings shown are logical and well presented. most of the entries made are incorrect;

most of the dates shown are incorrect for the transactions;

narrations are not shown.

workings shown are illogical and ill presented.

Question 3:

Apply relevant accounting principles in recognising and measuring income tax.

determine current and deferred tax balances without flaw/with minor flaws;

workings shown are logical and well presented;

all journal entries made are accurate/with minor flaws. determine current and deferred tax balances with some errors;

workings shown are logical and well presented;

journal entries made are accurate with some errors. fails to determine current and deferred tax balances;

workings shown are inadequate;

most of the journal entries made are incorrect.

Question 4:

Apply relevant accounting principles in the asset revaluation model. all entries made are accurate/with minor flaws;

response to directors is clear and shows mastery of the topic.

all entries made are mostly correct with some errors;

response to directors is clear and shows adequate understanding of the topic. most of the entries made are incorrect;

response to directors shows little or limited understanding of topic.

in recent years directors and ceos of companies have been placing more importance 681950

Task

In recent years directors and CEOs of companies have been placing more importance on holistic reporting of company activities rather than simply reporting what is required by the Accounting Standards, the Corporations Act and other legislation.

The Global Reporting Initiative is an international not for profit organisation that has pioneered and developed the world’s most widely used voluntary Sustainability Reporting Framework.(http://www.asx.com.au/documents/public consultations/global reporting initiative and sba submission 15Nov13.pdf)

In 2013 the fourth iteration of the Sustainability Reporting Framework guidelines were released.These can be accessed via the link below.

https://www.globalreporting.org/resourcelibrary/GRIG4 Part1 Reporting Principles and Standard Disclosures.pdf

Required

Referring specifically to section 5.1 and 5.2 of the above report, analyse how applying these voluntary guidelines might alter the information presented in the Flight Centre 2013 Annual report. Apply section 5.1 and 5.2 to triple bottom line reporting and balance score card under sustainability report.

Document Preview:

Sustainability Reporting Framework? Length: 2500 words ?Task In recent years directors and CEOs of companies have been placing more importance on holistic reporting of company activities rather than simply reporting what is required by the Accounting Standards, the Corporations Act and other legislation.  The Global Reporting Initiative is an international not for profit organisation that has pioneered and developed the world’s most widely used voluntary Sustainability Reporting Framework. (? HYPERLINK “http://www.asx.com.au/documents/public consultations/global reporting initiative and sba submission 15Nov13.pdf” ?http://www.asx.com.au/documents/public consultations/global reporting initiative and sba submission 15Nov13.pdf?) ???In 2013 the fourth iteration of the Sustainability Reporting Framework guidelines were released. These can be accessed via the link below. ? HYPERLINK “https://www.globalreporting.org/resourcelibrary/GRIG4 Part1 Reporting Principles and Standard Disclosures.pdf” ?https://www.globalreporting.org/resourcelibrary/GRIG4 Part1 Reporting Principles and Standard Disclosures.pdf?? Required   Referring specifically to section 5.1 and 5.2 of the above report, analyse how applying these voluntary guidelines might alter the information presented in the Flight Centre 2013 Annual report. Apply section 5.1 and 5.2 to triple bottom line reporting and balance score card under sustainability report. Structure of Report Your assignment needs to be presented as a standard business report including an Executive summary Table of contents Introduction (Plz mention the sustainability report which FLIGHT CENTRE has followed) Body (detailed explanation of section 5.1 and 5.2 to Triple Bottom Line and Balance score card under Sustainability report and how it affect Flight Centre) Conclusion and Recommendations References (at least 15 APA journal articles) NOTE: Please make sure there is no plagiarism as we are not going to submit hard copy; please include at…

Attachments:

costing and control 681963

i have attached the word file. plz make it according to instructions

Document Preview:

Costing and Control?The purpose of assessment item 2 continues to develop skills in costing and emphasises the role of control in managing the production of goods and services efficiently in the workplace. Each question builds on the knowledge gained through the first assessment to develop the capacity of managing control through costing. Each question uses realistic data and the professional practices similar to that found in workplaces. ?  Note all questions except for Question 9 are from the textbook: Fatseas, Victor & Williams, John, Cost Management (2013) 3rd edition, McGraw Hill QUESTION 1    Must be spreadsheeted ?P5 6  Weighted average cost: opening and closing WIP, and WIP ledger account Paris Paints Company produces a single lacquer paint which passes through two production departments. The following information concerns the first of those production departments. Direct materials are added at the beginning of the production process and conversion costs are incurred uniformly throughout the process. Total conversion costs are computed as direct labour costs, plus 50% of direct labour costs for the allocation of overhead costs.           The following information is available for May:        ?Opening work in process:?     Number of units?     Degree of completion?     Material costs?     Conversion costs??3 000?40%?$5 000?$7 200??        ?May production:?      Units started?     Units completed and transferred to Process 2?     Current material costs?     Current direct labour costs??30 000?32 000?$65 000?$40 000??        ?Closing work in process: ?      Degree of completion??80%??The company uses weighted average costing. There is no spoilage or wastage in the production process.           Required (a)    Prepare a Production Cost Report for May, showing the allocation of costs to closing work in process and to production transferred to the next department. (b)    Prepare the work in process T account for the first production department for May. QUESTION…

Attachments:

accounting 682015

Deliverable Length: 1,500–1,750 words

The President of EEC recently called a meeting to announce that one of the firm’s largest suppliers of component parts has approached EEC about a possible purchase of the supplier. The President has requested that you and your staff analyze the feasibility of acquiring this supplier. Discuss the following:

•What information will you and your staff need to analyze this investment opportunity?

•How will you go about making the decision?

•Discuss and evaluate the different techniques that could be used in capital budgeting decisions. ?Specifically, discuss how time value of money affects capital budgeting. Capital budgeting differs from regular budgeting in that capital budgeting is for large investment decisions like plant expansion. The regular budgeting is for your day to day operations decisions.

?Which do you think EEC should use? Why?

ha2042 accounting information systems assignment 2 682081

HA2042 – Accounting Information Systems Assignment 2 Due: 5pm Friday, Week 11 You are to prepare a submission to satisfy your client’s requirements as outlined below. You are required to investigate the issues at Pressure Hydraulics and provide a feasibility study into the improvement to their current business processes. You will need to conduct an investigation into the client’s problems and evaluate a solution to meet their business needs. Overview – Pressure Hydraulics Pressure Hydraulics is a locally owned business that currently has three service centres; Newcastle, Toronto and Maitland. Each service centre provides maintenance and specialised servicing of hydraulic systems as used in cars, trucks and earth moving equipment. The Maitland service centre also provides a specialty service to the mining industry where two purpose built trucks go onsite to service a range of mining equipment. The business has become quite profitable in the past years and its owner, Allan Taylor, has devised plans to expand by opening service centres at Coffs Harbour and Gosford. Allan has future plans for other service centres along the east coast of NSW. He also feels that the time is right to look at how IT can support the existing business and enable his future business plans. Currently, Allan spends a portion of each day at each service centre to monitor its operations. This is leaving little time to continue developing his business and he realises he will not be able to spend the same sort of time in the Coffs Harbour and Gosford service centres. Existing System Each service centre operates as an independent business, with eight technicians in the workshop (one of whom is a workshop foreman) and one office assistant. The office assistant takes phone calls from people requesting quotes for work or to have work done. For requests on quotes, the office assistant looks up a hard copy of a price book (known as The Price Book) and gives a verbal quote. Allan is unhappy with the time being taken to update the Price Book as the office assistant is often too busy to update the prices from the supplier invoices. Sometimes, this time delay results in either lost sales if the price has gone down or lost income if the price has gone up. If a customer phones to make a booking for work to be done, the office staff assistant writes down the details in a journal (Works Book). Any variations to the Works Book by a customer requesting to change a date/time results in crossed out entries. There have been times when this has been the cause of misinterpretation as to when a job is booked in. Other problems with the system include occasional overbooking or slack periods for the workshop staff. When a customer arrives at a service centre for work to be done, the office staff member writes out a Job Card with the customer details along with the price from The Price Book for the job. There is sometimes a discrepancy between what the Price Book shows and what the customer says they were quoted for that particular work. When completed, the Job Card is passed to the workshop foreman who assigns the job to a technician. When the job is completed, the technician initials the Job Card and gives it back to the workshop foreman, who returns it to the office. The office assistant writes out an invoice and collects payment from the customer; cash, EFTPOS and credit card are acceptable forms of payment. Once per week, the office assistant uses the Job Cards to prepare an order for stock replacement. Allan has developed a business relationship with several transport companies where two technicians will go to the transport companies’ central site and service the hydraulics on a truck. As the parts required to perform the service can be varied, a special van equipped with basic workshop equipment and a full range of component parts is used. The time taken for these jobs can be varied and unpredictable. Upon returning from such a job, the technicians will alert the workshop foreman that the job has been completed. The office assistant is then informed and an invoice written out and mailed to the transport company. Carbon copies are used to track such invoices. The Maitland service centre provides a service to the mining industry and there have been a few repeated issues with the incorrect component parts being taken onsite for servicing the machinery. Such mistakes not only costs a loss in terms of travel time back to the service centre to collect the correct part, but the mining companies have very tight maintenance schedules. Machinery has to be available and delays are not tolerated. Each fortnight, Allan contacts the workshop foremen and checks the hours worked by staff at the service centre. He then prepares the payroll and writes cheques for each employee. Each day, the office assistant banks the day’s takings and gives the bank receipt to Allan when he makes his daily call. Usually this will be the following day from the bank deposit. Allan then does a reconciliation of money banked with the previous day’s takings. Apart from the issues identified above, Allan wants to achieve the following: Ensuring repeat business from mining companies. Improve control over stock ordering. A new system is required to manage this aspect of the business. The ability to order stock for the three businesses instead of individual ordering would ensure economies not being realised at the moment. Ability to quickly determine if a required component part is in stock. Ability to generate an invoice off data held within the system. Improved payroll process to reduce the need to contact staff for details. Allan would prefer a system that recorded the working hours daily and also supported staff being paid directly through his bank system. Tracking customers for repeat business. Allan wants a system that will track customers and contact them at specified times with special offers. Price Book issues. A system to have timely production of price variations to arrive at all service centres on the same day. Technology to support these changes. After an initial investigation, Allan has asked your group to develop a feasibility report on providing a suitable solution for the accounting and payroll problems. At a minimum level, your feasibility report should contain the following generic sections: Your Task: You are required to submit a feasibility report. At a minimum level, your feasibility report should contain the following generic sections: ? Executive Summary ? Description of the problem ? Solution objectives ? Constraints ? Development plans ? Potential solutions ? Recommendations Your appendix should also contain ? Any brochures (scans), websites, photographs and prices you have researched to complete your assignment. A group evaluation sheet is also to be submitted by each member of the group on Blackboard by the end of week 12. Any student not submitting this form will not receive a mark. Feasibility Report Marking Guidelines Executive Summary …/2 Description of the problem …/3 Solution objectives …/3 Constraints …/1 Development plans …/3 Potential solutions …/2 Recommendations …/2 Presentation, clarity of discussion …/2 Logic and evidence of research …/2 Total Mark …/20 Your total mark will be added to your group presentation mark out of 5 for a total mark out of 25.

Attachments:

bankruptcies our specialty 682095

Holmes Institute HA3032 Memo 02 – Semester 02, 2014

Background:

Mr. Howe, a Junior Partner of the CPA firm Dewey, Cheatem, & Howe (DCH), after noting that there is a proposal to limit Auditor liability is very excited and has announced that the DCH Audit firm will soon be able to take on much, much riskier audits with little risk of DCH going bankrupt or losing the right to audit.

Required:

Mr. Tu Dewie has asked you to evaluate Mr. Howe’s assertions (use the lectures, tutorials and the internet (i.e. http://www.clmr.unsw.edu.au/article/compliance/internal risk management/institute chartered accountants stripped liability cap). In writing your memo, please remember that while Mr. Dewie is the senior partner of DCH, Mr. Howe (as a junior partner) can hurt you if you are not polite and considerate.

A Full Service Creative Accounting Firm Loopholes Located Books Juggled Taxes Evaded Funds Diverted Money Laundered Shortages Hidden Pensions Plundered Credit Denied Parking Validated

Bankruptcies our Specialty!

Document Preview:

Holmes Institute HA3032 Memo 02 – Semester 02, 2014 Background: Mr. Howe, a Junior Partner of the CPA firm Dewey, Cheatem, & Howe (DCH), after noting that there is a proposal to limit Auditor liability is very excited and has announced that the DCH Audit firm will soon be able to take on much, much riskier audits with little risk of DCH going bankrupt or losing the right to audit. Required: Mr. Tu Dewie has asked you to evaluate Mr. Howe’s assertions (use the lectures, tutorials and the internet (i.e. http://www.clmr.unsw.edu.au/article/compliance/internal risk management/institute chartered accountants A Full Service Creative Accounting Firm stripped liability cap). In writing your memo, please Loopholes Located Books Juggled Taxes Evaded remember that while Mr. Dewie is the senior partner of Funds Diverted Money Laundered Shortages Hidden DCH, Mr. Howe (as a junior partner) can hurt you if you Pensions Plundered Credit Denied Parking Validated are not polite and considerate. Bankruptcies our Specialty!

Attachments:

cash disbursements schedule maris brothers inc needs a cash disbursement schedule fo 681281

Cash disbursements schedule Maris Brothers, Inc., needs a cash disbursement schedule for the months of April, May, and June. Use the format of Table 3.9 and the following information in its preparation.

Sales: February = $500,000; March = $500,000; April = $560,000; May = $610,000; June = $650,000; July = $650,000

Purchases: Purchases are calculated as 60% of the next month’s sales, 10% of purchases are made in cash, 50% of purchases are paid for 1 month after purchase, and the remaining 40% of purchases are paid for 2 months after purchase.

Rent: The firm pays rent of $8,000 per month.

Wages and salaries: Base wage and salary costs are fixed at $6,000 per month plus a variable cost of 7% of the current month’s sales. Taxes: A tax payment of $54,500 is due in June.

Fixed asset outlays: New equipment costing $75,000 will be bought and paid for in April.

Interest payments: An interest payment of $30,000 is due in June. Cash dividends: Dividends of $12,500 will be paid in April.

Principal repayments and retirements: No principal repayments or retirements are due during these months.

cash budget mdash advanced the actual sales and purchases for xenocore inc for septe 681283

Cash budget—Advanced The actual sales and purchases for Xenocore, Inc., for September and October 2003, along with its forecast sales and purchases for the period November 2003 through April 2004, follow.

Year

Month

Sales

Purchases

2003

September

$210,000

$120,000

2003

October

250,000

150,000

2003

November

170,000

140,000

2003

December

160,000

100,000

2004

January

140,000

80,000

2004

February

180,000

110,000

2004

March

200,000

100,000

2004

April

250,000

90,000

The firm makes 20% of all sales for cash and collects on 40% of its sales in each of the 2 months following the sale. Other cash inflows are expected to be $12,000 in September and April, $15,000 in January and March, and $27,000 in February. The firm pays cash for 10% of its purchases. It pays for 50% of its purchases in the following month and for 40% of its purchases 2 months later. Wages and salaries amount to 20% of the preceding month’s sales. Rent of $20,000 per month must be paid. Interest payments of $10,000 are due in January and April. A principal payment of $30,000 is also due in April. The firm expects to pay cash dividends of $20,000 in January and April. Taxes of $80,000 are due in April. The firm also intends to make a $25,000 cash purchase of fixed assets in December. a. Assuming that the firm has a cash balance of $22,000 at the beginning of November, determine the end of month cash balances for each month, November through April. b. Assuming that the firm wishes to maintain a $15,000 minimum cash balance, determine the required total financing or excess cash balance for each month, November through April. c. If the firm were requesting a line of credit to cover needed financing for the period November to April, how large would this line have to be? Explain your answer.

cash flow concepts the following represent financial transactions that johns field a 681284

Cash flow concepts The following represent financial transactions that Johns field & Co. will be undertaking in the next planning period. For each transaction, check the statement or statements that will be affected immediately.

 

Statement

Transaction

Cash budget

Pro forma income
statement

Pro forma balance
sheet

Cash sale

     

Credit sale

 

   

Accounts receivable are collected

 

 

 

Asset with 5 year life is purchased

 

 

 

Depreciation is taken

 

 

 

Amortization of goodwill is taken

 

 

 

Sale of common stock

 

 

 

Retirement of outstanding bonds

 

 

 

Fire insurance premium is paid for the next 3 years

 

 

 

cash budget mdash sensitivity analysis trotter enterprises inc has gathered the foll 681285

Cash budget—Sensitivity analysis Trotter Enterprises, Inc., has gathered the following data in order to plan for its cash requirements and short term investment opportunities for October, November, and December. All amounts are shown in thousands of dollars.

 

October

November

December

 

Pessi

Most

Opti

Pessi

Most

Opti

Pessi

Most

Opti

 

mistic

likely

mistic

mistic

likely

mistic

mistic

likely

mistic

Total cash

 

 

 

 

 

 

 

 

 

receipts

$260

$342

$462

$200

$287

$366

$191

$294

$353

Total cash

 

 

 

 

 

 

 

 

 

disbursements

285

326

421

203

261

313

287

332

315

a. Prepare a sensitivity analysis of Trotter’s cash budget using $20,000 as the beginning cash balance for October and a minimum required cash balance of $18,000.

b. Use the analysis prepared in part a to predict Trotter’s financing needs and investment opportunities over the months of October, November, and December. Discuss how knowledge of the timing and amounts involved can aid the planning process.

multiple cash budgets mdash sensitivity analysis brownstein inc expects sales of 100 681286

Multiple cash budgets—Sensitivity analysis Brownstein, Inc., expects sales of $100,000 during each of the next 3 months. It will make monthly purchases of $60,000 during this time. Wages and salaries are $10,000 per month plus 5% of sales. Brownstein expects to make a tax payment of $20,000 in the next month and a $15,000 purchase of fixed assets in the second month and to receive $8,000 in cash from the sale of an asset in the third month. All sales and purchases are for cash. Beginning cash and the minimum cash balance are assumed to be zero.

a. Construct a cash budget for the next 3 months.

b. Brownstein is unsure of the sales levels, but all other figures are certain. If the most pessimistic sales figure is $80,000 per month and the most optimistic is $120,000 per month, what are the monthly minimum and maximum ending cash balances that the firm can expect for each of the 1 month periods?

c. Briefly discuss how the financial manager can use the data in parts a and b to plan for financing needs.

pro forma income statement the marketing department of metro line manufacturing esti 681287

Pro forma income statement The marketing department of Metro line Manufacturing estimates that its sales in 2004 will be $1.5 million. Interest expense is expected to remain unchanged at $35,000, and the firm plans to pay $70,000 in cash dividends during 2004. Metro line Manufacturing’s income statement for the year ended December 31, 2003, is given below, along with a breakdown of the firm’s cost of goods sold and operating expenses into their fixed and variable components.

Metroline Manufacturing

Income Statement

for the Year Ended December 31, 2003

Sales revenue

$1,400,000

Less: Cost of goods sold

910,000

Gross profits

$490,000

Less: Operating expenses

120,000

Operating profits

$370,000

Less: Interest expense

35,000

Net profits before taxes

$335,000

Less: Taxes (rate _ 40%)

134,000

Net profits after taxes

$201,000

Less: Cash dividends

66,000

To retained earnings

$135,000

 

Metroline Manufacturing
Breakdown of
Costs and Expenses
into Fixed and Variable
Components for the
Year Ended December 31, 2003

Cost of goods sold

 

Fixed cost

$210,000

Variable cost

700,000

Total cost

$910,000

Operating expenses

 

Fixed expenses

$36,000

Variable expenses

84,000

Total expenses

$120,000

a. Use the percent of sales method to prepare a pro forma income statement for the year ended December 31, 2004.

b. Use fixed and variable cost data to develop a pro forma income statement for the year ended December 31, 2004.

c. Compare and contrast the statements developed in parts a and b. Which statement probably provides the better estimate of 2004 income? Explain why.

pro forma income statement mdash sensitivity analysis allen products inc wants to do 681288

Pro forma income statement—Sensitivity analysis Allen Products, Inc., wants to do a sensitivity analysis for the coming year. The pessimistic prediction for sales is $900,000; the most likely amount of sales is $1,125,000; and the optimistic prediction is $1,280,000. Allen’s income statement for the most recent year follows.

Allen Products, Inc.

Income Statement for the

Year Ended December 31, 2003

Sales revenue

$937,500

Less: Cost of goods sold

421,875

Gross profits

$515,625

Less: Operating expenses

234,375

Operating profits

$281,250

Less: Interest expense

30,000

Net profits before taxes

$251,250

Less: Taxes (rate =  25%)

62,813

Net profits after taxes

$188,437

a. Use the percent of sales method, the income statement for December 31, 2003, and the sales revenue estimates to develop pessimistic, most likely, and optimistic pro forma income statements for the coming year.

b. Explain how the percent of sales method could result in an overstatement of profits for the pessimistic case and an understatement of profits for the most likely and optimistic cases.

c. Restate the pro forma income statements prepared in part a to incorporate the following assumptions about costs:

$250,000 of the cost of goods sold is fixed; the rest is variable.

$180,000 of the operating expenses is fixed; the rest is variable.

All of the interest expense is fixed.

d. Compare your findings in part c to your findings in part a. Do your observations confirm your explanation in part b?

pro forma balance sheet mdash basic leonard industries wishes to prepare a pro forma 681289

Pro forma balance sheet—Basic Leonard Industries wishes to prepare a pro forma balance sheet for December 31, 2004. The firm expects 2004 sales to total $3,000,000. The following information has been gathered.

(1) A minimum cash balance of $50,000 is desired.

(2) Marketable securities are expected to remain unchanged.

(3) Accounts receivable represent 10% of sales.

(4) Inventories represent 12% of sales.

(5) A new machine costing $90,000 will be acquired during 2004. Total depreciation for the year will be $32,000.

(6) Accounts payable represent 14% of sales.

(7) Accruals, other current liabilities, long term debt, and common stock are expected to remain unchanged.

(8) The firm’s net profit margin is 4%, and it expects to pay out $70,000 in cash dividends during 2004. (9) The December 31, 2003, balance sheet follows.

Leonard Industries

Balance Sheet

December 31, 2003

Assets

Liabilities and Stockholders’ Equity

Cash

$45,000

Accounts payable

$395,000

Marketable securities

15,000

Accruals

60,000

Accounts receivable

255,000

Other current liabilities

30,000

Inventories

340,000

Total current liabilities

$485,000

Total current assets

$655,000

Long term debt

$350,000

Net fixed assets

$600,000

Common stock

$200,000

Total assets

$1,255,000

Retained earnings

$220,000

 

 

Total liabilities and

 

 

 

stockholders’ equity

$1,255,000

a. Use the judgmental approach to prepare a pro forma balance sheet dated December 31, 2004, for Leonard Industries.

b. How much, if any, additional financing will Leonard Industries require in 2004? Discuss.

c. Could Leonard Industries adjust its planned 2004 dividend to avoid the situation described in part b? Explain how.

pro forma balance sheet peabody amp peabody has 2003 sales of 10 million 681290

Pro forma balance sheet Peabody & Peabody has 2003 sales of $10 million. It wishes to analyze expected performance and financing needs for 2005—2 years ahead. Given the following information, respond to parts a and b.

(1) The percents of sales for items that vary directly with sales are as follows:

Accounts receivable, 12%

Inventory, 18%

Accounts payable, 14%

Net profit margin, 3%

(2) Marketable securities and other current liabilities are expected to remain unchanged.

(3) A minimum cash balance of $480,000 is desired.

(4) A new machine costing $650,000 will be acquired in 2004, and equipment costing $850,000 will be purchased in 2005. Total depreciation in 2004 is forecast as $290,000, and in 2005 $390,000 of depreciation will be taken.

(5) Accruals are expected to rise to $500,000 by the end of 2005.

(6) No sale or retirement of long term debt is expected.

(7) No sale or repurchase of common stock is expected.

(8) The dividend payout of 50% of net profits is expected to continue.

(9) Sales are expected to be $11 million in 2004 and $12 million in 2005.

(10) The December 31, 2003, balance sheet follows.

Peabody & Peabody

Balance Sheet

December 31, 2003

($000)

Assets

Liabilities and Stockholders’ Equity

Cash

$400

Accounts payable

$1,400

Marketable securities

200

Accruals

400

Accounts receivable

1,200

Other current liabilities

80

Inventories

1,800

Total current liabilities

$1,880

Total current assets

$3,600

Long term debt

$2,000

Net fixed assets

$4,000

Common equity

$3,720

Total assets

$7,600

Total liabilities and

 

 

 

stockholders’ equity

$7,600

a. Prepare a pro forma balance sheet dated December 31, 2005.

b. Discuss the financing changes suggested by the statement prepared in part a.

integrative mdash pro forma statements red queen restaurants wishes to prepare finan 681291

Integrative—Pro forma statements Red Queen Restaurants wishes to prepare financial plans. Use the financial statements and the other information provided in what follows to prepare the financial plans.

 

Red Queen Restaurants
Income Statement for the
Year Ended December 31, 2003

Sales revenue

$800,000

Less: Cost of goods sold

600,000

Gross profits

$200,000

Less: Operating expenses

100,000

Net profits before taxes

$100,000

Less: Taxes (rate = 40%)

40,000

Net profits after taxes

$60,000

Less: Cash dividends

20,000

To retained earnings

$40,000

 

 

Red Queen Restaurants

Balance Sheet

December 31, 2003

Assets

Liabilities and Stockholders’ Equity

Cash

$32,000

Accounts payable

$100,000

Marketable securities

18,000

Taxes payable

20,000

Accounts receivable

150,000

Other current liabilities

5,000

Inventories

100,000

Total current liabilities

$125,000

Total current assets

$300,000

Long term debt

$200,000

Net fixed assets

$350,000

Common stock

$150,000

Total assets

$650,000

Retained earnings

$175,000

Total liabilities and

 

 

 

stockholders’ equity

$650,000

The following financial data are also available:

(1) The firm has estimated that its sales for 2004 will be $900,000.

(2) The firm expects to pay $35,000 in cash dividends in 2004.

(3) The firm wishes to maintain a minimum cash balance of $30,000.

(4) Accounts receivable represent approximately 18% of annual sales.

(5) The firm’s ending inventory will change directly with changes in sales in

2004.

(6) A new machine costing $42,000 will be purchased in 2004. Total depreciation for 2004 will be $17,000.

(7) Accounts payable will change directly in response to changes in sales in 2004.

(8) Taxes payable will equal one fourth of the tax liability on the pro forma income statement.

(9) Marketable securities, other current liabilities, long term debt, and common stock will remain unchanged.

a. Prepare a pro forma income statement for the year ended December 31, 2004, using the percent of sales method.

b. Prepare a pro forma balance sheet dated December 31, 2004, using the judgmental approach.

c. Analyze these statements, and discuss the resulting external financing required.

preparing martin manufacturing rsquo s 2004 pro forma financial statements 681293

Preparing Martin Manufacturing’s 2004 Pro Forma Financial Statements

To improve its competitive position, Martin Manufacturing is planning to implement a major equipment modernization program. Included will be replacement and modernization of key manufacturing equipment at a cost of $400,000 in 2004. The planned program is expected to lower the variable cost per unit of finished product. Terri Spiro, an experienced budget analyst, has been charged with preparing a forecast of the firm’s 2004 financial position, assuming replacement and modernization of manufacturing equipment. She plans to use the 2003 financial statements presented on pages 92 and 93, along with the key projected financial data summarized in the following table.

Martin Manufacturing Company

Key Projected Financial Data (2004)

Data item

Value

Sales revenue

$6,500,000

Minimum cash balance

$25,000

Inventory turnover (times)

7.0

Average collection period

50 days

Fixed asset purchases

$400,000

Dividend payments

$20,000

Depreciation expense

$185,000

Interest expense

$97,000

Accounts payable increase

20%

Accruals and long term debt

Unchanged

Notes payable, preferred and common stock

Unchanged

Required

a. Use the historical and projected financial data provided to prepare a pro forma income statement for the year ended December 31, 2004.

b. Use the projected financial data along with relevant data from the pro forma income statement prepared in part a to prepare the pro forma balance sheet at December 31, 2004.

c. Will Martin Manufacturing Company need to obtain external financing to fund the proposed equipment modernization program? Explain.

use this calculator to determine the straight line declining balance using 200 and m 681294

Use this calculator to determine the straight line, declining balance (using 200%), and MACRS depreciation schedules for the following items, using half year averaging (the half year convention).

Item

Date placed in service

Cost

Office furnishings

2/15/2002

$22,500

Laboratory equipment

5/27/2001

$14,375

Fleet vehicles

9/5/2000

$45,863

Make a chart comparing the depreciation amounts that these three methods yield for the years 2002 to 2007. Discuss the implications of these differences.

seven years ago after 15 years in public accounting stanley booker cpa resigned his 681295

Track Software, Inc.

Seven years ago, after 15 years in public accounting, Stanley Booker, CPA, resigned his position as Manager of Cost Systems for Davis, Cohen, and O’Brien Public Accountants and started Track Software, Inc. In the 2 years preceding his departure from Davis, Cohen, and O’Brien, Stanley had spent nights and weekends developing a sophisticated cost accounting software program that became Track’s initial product offering. As the firm grew, Stanley planned to develop and expand the software product offerings—all of which would be related to streamlining the accounting processes of medium to large sized manufacturers. Although Track experienced losses during its first 2 years of operation— 1997 and 1998—its profit has increased steadily from 1999 to the present (2003). The firm’s profit history, including dividend payments and contributions to retained earnings, is summarized in Table 1. Stanley started the firm with a $100,000 investment—his savings of $50,000 as equity and a $50,000 long term loan from the bank. He had hoped to maintain his initial 100 percent ownership in the corporation,

 

Table 1

Track Software, Inc.

Profit, Dividends, and Retained Earnings, 1997–2003

 

Net profits after taxes

 

Contribution to

Year

(1)

Dividends paid

(2)

retained earnings [(1) (2)]

(3)

1997

($50,000)

$ 0

($50,000)

1998

( 20,000)

0

( 20,000)

1999

15,000

0

15,000

2000

35,000

0

35,000

2001

40,000

1,000

39,000

2002

43,000

3,000

40,000

2003

48,000

5,000

43,000

but after experiencing a $50,000 loss during the first year of operation (1997), he sold 60 percent of the stock to a group of investors to obtain needed funds. Since then, no other stock transactions have taken place. Although he owns only 40 percent of the firm, Stanley actively manages all aspects of its activities; the other stockholders are not active in management of the firm. The firm’s stock closed at $4.50 per share in 2002 and at $5.28 per share in 2003. Stanley has just prepared the firm’s 2003 income statement, balance sheet, and statement of retained earnings, shown in Tables 2, 3, and 4 (on pages 143–145), along with the 2002 balance sheet. In addition, he has compiled the 2002 ratio values and industry average ratio values for 2003, which are applicable to both 2002 and 2003 and are summarized in Table 5 (on page 145). He is quite pleased to have achieved record earnings of $48,000 in 2003, but he is concerned about the firm’s cash flows. Specifically, he is finding it more and more difficult to pay the firm’s bills in a timely manner and generate cash flows to investors—both creditors and owners. To gain insight into these cash flow problems, Stanley is planning to determine the firm’s 2003 operating cash flow (OCF) and free cash flow (FCF).

Table 2

Track Software, Inc.
Income Statement ($000)
for the Year Ended December 31, 2003

Sales revenue

 

 

Less: Cost of goods sold

 

 

Gross profits

 

 

Less: Operating expenses

 

1,030

Selling expense

$150

$520

General and administrative expense

270

 

Depreciation expense

11

 

Total operating expense

 

431

Operating profits (EBIT)

 

$89

Less: Interest expense

 

29

Net profits before taxes

 

$60

Less: Taxes (20%)

 

12

Net profits after taxes

 

$48

Table 3

Track Software, Inc.
Balance Sheets ($000)

 

December 31

Assets

2003

2002

Current assets

 

Cash

$12

$31

Marketable securities

66

82

Accounts receivable

152

104

Inventories

191

145

Total current assets

$421

$362

Gross fixed assets

$195

$180

Less: Accumulated depreciation

63

52

Net fixed assets

$132

$128

Total assets

$553

$490

Liabilities and Stockholders’ Equity

 

 

Current liabilities

 

 

Accounts payable

$136

$126

Notes payable

200

190

Accruals

27

25

Total current liabilities

$363

$341

Long term debt

$38

$40

Total liabilities

$401

$381

Stockholders’ equity

 

 

Common stock (50,000 shares outstanding

 

 

at $0.40 par value)

$20

$20

Paid in capital in excess of par

30

30

Retained earnings

102

59

Total stockholders’ equity

$152

$109

Total liabilities and stockholders’ equity

$553

$490

       

Table  4

Track Software, Inc.

Statement of Retained Earnings ($000)

for the Year Ended December 31, 2003

Retained earnings balance (January 1, 2003)

$ 59

Plus: Net profits after taxes (for 2003)

48

Less: Cash dividends on common stock (paid during 2003)

(5 )

Retained earnings balance (December 31, 2003)

$ 102

Table 5

 

Actual

Industry average

Ratio

2002

2003

Current ratio

1.06

1.82

Quick ratio

0.63

1.10

Inventory turnover

10.40

12.45

Average collection period

29.6 days

20.2 days

Total asset turnover

2.66

3.92

Debt ratio

0.78

0.55

Times interest earned ratio

3.0

5.6

Gross profit margin

32.1%

42.3%

Operating profit margin

5.5%

12.4%

Net profit margin

3.0%

4.0%

Return on total assets (ROA)

8.0%

15.6%

Return on common equity (ROE)

36.4%

34.7%

Price/earnings (P/E) ratio

5.2

7.1

Market/book (M/B) ratio

2.1

2.2

Stanley is further frustrated by the firm’s inability to afford to hire a software developer to complete development of a cost estimation package that is believed to have “blockbuster” sales potential. Stanley began development of this package 2 years ago, but the firm’s growing complexity has forced him to devote more of his time to administrative duties, thereby halting the development of this product. Stanley’s reluctance to fill this position stems from his concern that the added $80,000 per year in salary and benefits for the position would certainly lower the firm’s earnings per share (EPS) over the next couple of years. Although the project’s success is in no way guaranteed, Stanley believes that if the money were spent to hire the software developer, the firm’s sales and earnings would significantly rise once the 2 to 3 year development, production, and marketing process was completed. With all of these concerns in mind, Stanley set out to review the various data to develop strategies that would help to ensure a bright future for Track Software. Stanley believed that as part of this process, a thorough ratio analysis of the firm’s 2003 results would provide important additional insights.

Required

a. (1) Upon what financial goal does Stanley seem to be focusing? Is it the correct goal? Why or why not?

(2) Could a potential agency problem exist in this firm? Explain.

b. Calculate the firm’s earnings per share (EPS) for each year, recognizing that the number of shares of common stock outstanding has remained unchanged since the firm’s inception. Comment on the EPS performance in view of your response in part a.

c. Use the financial data presented to determine Track’s operating cash flow (OCF) and free cash flow (FCF) in 2003. Evaluate your findings in light of Track’s current cash flow difficulties.

d. Analyze the firm’s financial condition in 2003 as it relates to (1) liquidity, (2) activity, (3) debt, (4) profitability, and (5) market, using the financial statements provided in Tables 2 and 3 and the ratio data included in Table 5. Be sure to evaluate the firm on both a cross sectional and a time series basis.

e. What recommendation would you make to Stanley regarding hiring a new software developer? Relate your recommendation here to your responses in part a.

in the preceding example we saw that lawrence industries could take the cash discoun 681671

In the preceding example, we saw that Lawrence Industries could take the cash discount on its February 27 purchase by paying $980 on March 10. If Lawrence gives up the cash discount, payment can be made on March 30. To keep its money for an extra 20 days, the firm must give up an opportunity to pay $980 for its $1,000 purchase. In other words, it will cost the firm $20 to delay payment for 20 days. Figure 15.1 shows the payment options that are open to the company. To calculate the cost of giving up the cash discount, the true purchase price must be viewed as the discounted cost of the merchandise, which is $980 for

 width=

Lawrence Industries. The annual percentage cost of giving up the cash discount can be calculated using Equation 15.1:1

 width=

where

CD = stated cash discount in percentage terms

= number of days that payment can be delayed by giving up the cash discount

Substituting the values for CD (2%) and (20 days) into Equation 15.1 results in an annualized cost of giving up the cash discount of 36.73% [(2% ÷ 98%) x (360 ÷ 20)]. A 360 day year is assumed.2 A simple way toapproximate the cost of giving up a cash discount is to use the stated cash discount percentage, CD, in place of the first term of Equation 15.1:

 

 width=

The smaller the cash discount, the closer the approximation to the actual cost of giving it up. Using this approximation, the cost of giving up the cash discount for Lawrence Industries is 36% [2% x (360 ÷ 20)].

mason products a large building supply company has four possible suppliers each offe 681672

Mason Products, a large building supply company, has four possible suppliers, each offering different credit terms. Otherwise, their products and services are identical. Table 15.1 presents the credit terms offered by suppliers A, B, C, and D and the cost of giving up the cash discounts in each transaction. The approximation method of calculating the cost of giving up a cash discount (Equation 15.2) has been used. The cost of giving up the cash discount from supplier A is 36%; from supplier B, 8%; from supplier C, 21.6%; and from supplier D, 28.8%. If the firm needs short term funds, which it can borrow from its bank at an interest rate of 13%, and if each of the suppliers is viewed separately, which (if any) of the suppliers’ cash discounts will the firm give up? In dealing with supplier A, the firm takes the cash discount, because the cost of giving it up is 36%, and then borrows the funds it requires from its bank at 13% interest. With supplier B, the firm would do better to give up the cash discount, because the cost of this action is less than the cost of borrowing money from the bank (8% versus 13%). With either supplier C or supplier D, the firm should take the cash discount, because in both cases the cost of giving up the discount is greater than the 13% cost of borrowing from the bank.

Equation 15.2

 width=

 

 

TABLE 15.1

Cash Discounts and

Associated Costs for

Mason Products

 

 

Approximate

 

 

cost of giving up

Supplier

Credit terms

a cash discount

A

2/10 net 30 EOM

36.0%

B

1/10 net 55 EOM

8.0

C

3/20 net 70 EOM

21.6

D

4/10 net 60 EOM

28.8

wooster company a manufacturer of athletic apparel wants to borrow 10 000 at a state 681675

Wooster Company, a manufacturer of athletic apparel, wants to borrow $10,000 at a stated annual rate of 10% interest for 1 year. If the interest on the loan is paid at maturity, the firm will pay $1,000 (0.10 $10,000) for the use of the $10,000 for the year. Substituting into Equation 15.3 reveals that the effective annual rate is therefore

 width=

If the money is borrowed at the same stated annual rate for 1 year but interest is paid in advance, the firm still pays $1,000 in interest, but it receives only $9,000 ($10,000 $1,000). The effective annual rate in this case is

 width=

Paying interest in advance thus makes the effective annual rate (11.1%) greater than the stated annual rate (10.0%).

Equation 15.3

 width=

estrada graphics a graphic design firm has borrowed 1 million under a line of credit 681676

Estrada Graphics, a graphic design firm, has borrowed $1 million under a line of credit agreement. It must pay a stated interest rate of 10% and maintain, in its checking account, a compensating balance equal to 20% of the amount borrowed, or $200,000. Thus it actually receives the use of only $800,000. To use that amount for a year, the firm pays interest of $100,000 (0.10 x $1,000,000). The effective annual rate on the funds is therefore 12.5% ($100,000 ÷ $800,000), 2.5% more than the stated rate of 10%. If the firm normally maintains a balance of $200,000 or more in its checking account, the effective annual rate equals the stated annual rate of 10% because none of the $1 million borrowed is needed to satisfy the compensating balance requirement. If the firm normally maintains a $100,000 balance in its checking account, only an additional $100,000 will have to be tied up, leaving it with $900,000 of usable funds. The effective annual rate in this case would be 11.1% ($100,000 ÷ $900,000). Thus a compensating balance raises the cost of borrowing only if it is larger than the firm’s normal cash balance.

reh company a major real estate developer has a 2 million revolving credit agreement 681677

REH Company, a major real estate developer, has a $2 million revolving credit agreement with its bank. Its average borrowing under the agreement for the past year was $1.5 million. The bank charges a commitment fee of 0.5%. Because the average unused portion of the committed funds was $500,000 ($2 million $1.5 million), the commitment fee for the year was $2,500 (0.005 x $500,000). Of course, REH also had to pay interest on the actual $1.5 million borrowed under the agreement. Assuming that $160,000 interest was paid on the $1.5 million borrowed, the effective cost of the agreement was 10.83% [($160,000 + $2,500)/$1,500,000]. Although more expensive than a line of credit, a revolving credit agreement can be less risky from the borrower’s viewpoint, because the availability of funds is guaranteed.

cash discount decisions the credit terms for each of three suppliers are shown in th 681679

Cash discount decisions The credit terms for each of three suppliers are shown in the following table.

Supplier

Credit terms

X

1/10 net 55 EOM

Y

2/10 net 30 EOM

Z

2/20 net 60 EOM

a. Determine the approximate cost of giving up the cash discount from each supplier.

b. Assuming that the firm needs short term financing, indicate whether it would be better to give up the cash discount or take the discount and borrow from a bank at 15% annual interest. Evaluate each supplier separatelyusing your findings in part a.

c. What impact, if any, would the fact that the firm could stretch its accounts payable (net period only) by 20 days from supplier Z have on your answer in part b relative to this supplier?

finding operating and free cash flows consider the balance sheets and selected data 681279

Finding operating and free cash flows Consider the balance sheets and selected data from the income statement of Keith Corporation that follow.

a. Calculate the firm’s accounting cash flow from operations for the year ended December 31, 2003, using Equation 3.1.

Equation 3.1

Cash flow from operations=

Net profits after taxes + Depreciation and other noncash charges

b. Calculate the firm’s operating cash flow (OCF) for the year ended December 31, 2003, using Equation 3.2.

Equation 3.2

OCF=EBIT Taxes+ Depreciation

c. Calculate the firm’s free cash flow (FCF) for the year ended December 31, 2003, using Equation 3.3.

d. Interpret, compare, and contrast your cash flow estimates in parts a, b, and c.

Equation 3.3

FCF= OCF Net fixed asset investment (NFAI)

Net current asset investment (NCAI)

Keith Corporation
Balance Sheets

 

December 31

Assets

2003

2002

 

Cash

$1,500

$1,000

 

Marketable securities

1,800

1,200

 

Accounts receivable

2,000

1,800

 

Inventories

2,900

2,800

 

Total current assets

$8,200

$6,800

 

Gross fixed assets

$29,500

$28,100

 

Less: Accumulated depreciation

14,700

13,100

 

Net fixed assets

$14,800

$15,000

 

Total assets

$23,000

$21,800

 

Liabilities and Stockholders’ Equity

 

 

 

Accounts payable

 

$1,600

$1,500

Notes payable

 

2,800

2,200

Accruals

 

200

300

Total current liabilities

 

$4,600

$4,000

Long term debt

 

$5,000

$5,000

Common stock

 

$10,000

$10,000

Retained earnings

 

3,400

2,800

Total stockholders’ equity

 

$13,400

$12,800

Total liabilities and stockholders’ equity

 

$23,000

$21,800

Income Statement Data (2003)

 

 

 

Depreciation expense

 

$11,600

 

Earnings before interest and taxes (EBIT)

 

2,700

 

Taxes

 

933

 

Net profits after taxes

 

1,400

 

ginger s flower company was started in 2012 when it acquired 80 000 cash from the is 678558

Basic transactions for three accounting cycles: perpetual system

Ginger’s Flower Company was started in 2012 when it acquired $80,000 cash from the issue of common stock. The following data summarize the company’s first three years’ operating activities. Assume that all transactions were cash transactions.

 

2012

2013

2014

Purchases of inventory

$ 60,000

$ 90,000

$ 130,000

Sales

102,000

146,000

220,000

Cost of goods sold

54,000

78,000

140,000

Selling and administrative expenses

40,000

52,000

72,000

Required

Prepare an income statement (use multistep format) and balance sheet for each fiscal year. (Hint: Record the transaction data for each accounting period in the accounting equation before preparing the statements for that year.)

why it is necessary to meet the designated timeline for preparing and submitting the 678688

Why it is necessary to meet the designated timeline for preparing and submitting the reports such as BUSINESS ACTIVITYSTATEMENT(BAS)?

2. Iif you are tasked with maintaining financial records for a company what are the organisation requirements that you should consider?

Document Preview:

DIPLOMA OF ACCOUNTING Why it is necessary to meet the designated timeline for preparing and submitting the reports such as BUSINESS ACTIVITYSTATEMENT(BAS)? Iif you are tasked with maintaining financial records for a company what are the organisation requirements that you should consider?

Attachments:

explain the concept of departmentalization and describes that how the various factor 678703

Explain the concept of departmentalization and describes that how the various Factory overheads have been assigned to different departments.?

Document Preview:

ALLAMA IQBAL OPEN UNIVERSITY Directorate of Admissions (COMPLAINTS MANAGEMENT SECTION) ***** CONFIRMATION OF ADMISSION Program ?MBA(Finance)??Semester?1st Autum??Course Codes?8501,8502,8507,8570,8506? MERGEFIELD Ccodes ????Student Name?Muhammad Jamil??Father’s Name?Gulab Din??Address?House No 394 Street No 3 Jinnah Abad Abbottabad??Cell Phone No.?03215100312??Email Address?? MERGEFIELD Email ??jamil_zobi@yahoo.com??Bank Chalan No./ Bank Name*?784345 First Women Bank ltd Abbottabad ??Roll No. (For Continuing Students)???Registration No. (If registered already)?03nma0861??If you received Objection letter from Admission deptt.? ? Yes ? No If yes, write Sr. No. of objection letter: ? MERGEFIELD Obj_No ???????*Note: Please attach copy of Bank chalan Remarks by concerned Section (for admission deptt. only): ? MERGEFIELD Remarks ?? ____________________ Signature Dealing Official

Attachments:

please help me 678729

TASK 2: Dropping or Retaining a Product Henry Douglas is the owner and managing director of Heritage Furniture, Ltd., a South African company that makes museum quality reproductions of antique outdoor furniture. Ms. Douglas would like advice concerning the advisability of eliminating the model C3 lawn chair. These lawnchairs have been among the company’s best selling products, but they seem to be unprofitable. A condensed absorption costing income statement for the company and for the model C3 lawnchair for the quarter ended June 30 follows: All Products Model C3 Lawnchair Sales. . . .

Document Preview:

TASK 2: Dropping or Retaining a Product Henry Douglas is the owner and managing director of Heritage Furniture, Ltd., a South African company that makes museum quality reproductions of antique outdoor furniture. Ms. Douglas would like advice concerning the advisability of eliminating the model C3 lawn chair. These lawnchairs have been among the company’s best selling products, but they seem to be unprofitable. A condensed absorption costing income statement for the company and for the model C3 lawnchair for the quarter ended June 30 follows: All Products Model C3 Lawnchair Sales. . . . . . . . . . . . . . . . . . . . . R2,900,000 R300,000 Cost of goods sold: Direct materials. . . . . . . . . . . . . . . . . 759,000 122,000 Direct labor . . . . . . . . . . . . . . . . . . . . 680,000 72,000 Fringe benefits (20% of direct labor) . . . . . 136,000 14,400 Variable manufacturing overhead . . . . . . . . 28,000 3,600 Building rent and maintenance . . . . . . 30,000 4,000 Depreciation . . . . . . . . . . . . . . . . . . . 75,000 19,100 Total cost of goods sold . . . . . . . . . 1,708,000 235,100 Gross margin. . . . . . . . . . . . . . . . . . 1,192,000 64,900 Selling and administrative expenses: Product managers’ salaries . . . . . . . . . . . 75,000 10,000 Sales commissions (5% of sales). . . .. . . . 145,000 15,000 Fringe beneftts (20% of salaries and commissions) 44,000 5,000 Shipping. . . . . . . . . . .. . . . . . . . . . . . 120,000 10,000 General administrative expenses . . . . . . . 464,000 48,000 Total selling and administrative expenses . . 848,000 88,000 Net operating income (loss) . . . . . . . . . .. R344,000 …

Attachments:

bmac5203 accounting for business decision making 678730

SECOND SEMESTER (INTAKE MARCH 2014) BMAC5203 – ACCOUNTING FOR BUSINESS DECISION MAKING ASSIGNMENT (60%) OBJECTIVE: To assess students’ ability to ethically use accounting information for costing, decision making, planning and control. INSTRUCTIONS: 1. Assignment Format: a. Use 12 point of Arial font. The line spacing should be 1.5 for the main text and tables. b. Provide references. c. References should use the American Psychological Association (APA) format. References should be up to date (year 2005 and onwards). 2. Notes: a. Assignments should be submitted according to the fixed date. b. Plagiarism is not acceptable. If you are not sure what is meant by plagiarism, refer to the various websites which discuss this matter, e.g. owl.english.purdue.edu/handouts. 3. The assignment consists of FIVE (5) unrelated tasks. Answer all. The details of the tasks are explained in pages 2 – 6. ASSIGNMENT – BMAC5203 Page 2 TASK 1: CVP Analysis Success Company’s contribution format income statement for the most recent month is given below: Sales (40,000 units) . . . . . . . . . . . . . . . . . . $800,000 Variable expenses . . . . . . . . . . . . . . . . . . . 560,000 Contribution margin . . . . . . . . . . . . . . . . . . 240,000 Fixed expenses . . . . . . . . . . . . . . . . . . . . . 192,000 Net operating income . . . . . . . . . . . . . . . . . $ 48,000 The industry in which Success Company operates is quite sensitive to cyclical movements in the economy. Thus, profits vary considerably from year to year according to general economic con ditions. The company has a large amount of unused capacity and is studying ways of improving profits. Required: 1. New equipment has come on the market that would allow F rieden Company to automate a portion of its operations. Variable expenses would be reduced by $6 per unit. However, fixed expenses would increase to a total of $432,000 each month. Prepare two contribution format income statements, one showing present operations and one showing how operations would appear if the new equipment is purchased. Show an Amount column, a Per Unit column, and a Percent column on each statement. Do not show percentages for the fixed expenses. 2. Refer to the income statements in (1) above. For both present operations and the proposed new operations, compute ( a) the degree of operating leverage, ( b) the break even point in dol lars, and ( c ) the margin of safety in both dollar and percentage terms. 3. Refer again to the data in (1) above. As a manager, what factor would be paramount in your mind in deciding whether to purchase the new equipment? (Assume that ample funds are available to make the purchase.) 4. Refer to the original data. Rather than purchase new equipment, the marketing manager argues that the company’s marketing strategy should be changed. Instead of paying sales commissions, which are included in variable expenses, the marketing manager suggests that salespersons be paid fixed salaries and that the company invest heavily in ASSIGNMENT – BMAC5203 Page 3 advertising. The marketing manager claims that this new approach would increase unit sales by 50% without any change in selling price; the company’s new monthly fixed expenses would be $240,000; and its net operating income would increase by 25%. Compute the break even point in dollar sales for the company under the new marketing strategy. Do you agree with the marketing manager’s proposal? [TOTAL: 25 MARKS] TASK 2: Dropping or Retaining a Product Henry Douglas is the owner and managing director of Heritage Furniture, Ltd., a South African company that makes museum quality reproductions of antique outdoor furniture. Ms. Douglas would like advice concerning the advisability of eliminating the model C3 lawn chair. These lawnchairs have been among the company’s best selling products, but they seem to be unprofitable. A condensed absorption costing income statement for the company and for the model C3 lawnchair for the quarter ended June 30 follows: All Products Model C3 Lawnchair Sales. . . . . . . . . . . . . . . . . . . . . . . . . . . . . . . . . . . . . . . . . . . R2,900,000 R300,000 Cost of goods sold: Direct materials. . . . . . . . . . . . . . . . . . . . . . . . . 759,000 122,000 Direct labor . . . . . . . . . . . . . . . . . . . . . . . . . . . . . . . . . . .680,000 72,000 Fringe benefits (20% of direct labor) . . . . . . . . . . . . . . .136,000 14,400 Variable manufacturing overhead . . . . . . . . . . . . . . . . . .28,000 3,600 Building rent and maintenance . . . . . . . . . . . . . . . . . . . . 30,000 4,000 Depreciation . . . . . . . . . . . . . . . . . . . . . . . . . . . . . . . . . . .75,000 19,100 Total cost of goods sold . . . . . . . . . . . . . . . . . . . . . . . 1,708,000 235,100 Gross margin. . . . . . . . . . . . . . . . . . . . . . . . . . . . . . . . 1,192,000 64,900 Selling and administrative expenses: Product managers’ salaries . . . . . . . . . . . . . . . . . . . . . . 75,000 10,000 Sales commissions (5% of sales). . . . . . . . . . . . . . . . . 145,000 15,000 Fringe beneftts (20% of salaries and commissions) . . . . 44,000 5,000 Shipping. . . . . . . . . . . . . . . . . . . . . . . . . . . . . . . . . . . . . 120,000 10,000 General administrative expenses . . . . . . . . . . . . . . . . . 464,000 48,000 Total selling and administrative expenses . . . . . . . . . . . 848,000 88,000 Net operating income (loss) . . . . . . . . . . . . . . . . . . . . . . . . . . R344,000 R(23,100) ASSIGNMENT – BMAC5203 Page 4 The currency in South Africa is the rand, denoted here by R. The following additional data have been supplied by the company: a. Direct labor is a variable cost. b. All of the company’s products are manufactured in the same facility and use the same equipment. Building rent and maintenance and depreciation are allocated to products using various bases. The equipment does not wear out through use; it eventually becomes obsolete. c. There is ample capacity to fill all orders. d. Dropping the model C3 lawnchair would have no effect on sales of other product lines. e. Work in process and finished goods inventories are insignificant. f. Shipping costs are traced directly to products. g. General administrative expenses are allocated to products on the basis of sales. There would be no effect on the total general administrative expenses if the model C3 lawnchair were dropped. h. If the model C3 lawnchair were dropped, the product manager would be laid off. Required: 1. Given the current level of sales, would you recommend that the model C3 lawnchair be dropped? Prepare appropriate computations to support your answer. 2. What would sales of the model C3 lawnchair have to be, at minimum, in order to justify retaining the product? Explain. [TOTAL: 25 MARKS] TASK 3: Cash budget Natural Care Corp., a distributor of natural cosmetics, is ready to begin its third quarter, in which peak sales occur. The company has requested a $60,000, 90 day loan from its bank to help meet cash requirements during the quarter. Because Natural Care has experienced difficulty in paying off its loans in the past, the bank’s loan officer has asked the company to prepare a cash budget for the quarter. In response to this request, the following data have been assembled: a. On July 1, the beginning of the third quarter, the company will have a cash balance of $43,000. b. Actual sales for the last two months and budgeted sales for the third quarter follow (all sales are on account): May (actual) . . . . . . . . . . . . . . . . . . . . . . . . . $360,000 June (actual) . . . . . . . . . . . . . . . . . . . . . . . . . $280,000 ASSIGNMENT – BMAC5203 Page 5 July (budgeted) . . . . . . . . . . . . . . . . . . . . . . . $350,000 August (budgeted) . . . . . . . . . . . . . . . . . . . . $420,000 September (budgeted) . . . . . . . . . . . . . . . . . $360,000 Past experience shows that 25% of a month’s sales are collected in the month of sale, 70% in the month following sale, and 2% in the second month following sale. The remainder is uncollectible. c. Budgeted merchandise purchases and budgeted expenses for the third quarter are given below: July August September Merchandise purchases . . . . . . . . . . . . $170,000 $155,000 $165,000 Salaries and wages . . . . . . . . . . . . . . . $70,000 $70,000 $65,000 Advertising . . . . . . . . . . . . . . . . . . . . . . $80,000 $90,000 $100,000 Rent payments . . . . . . . . . . . . . . . . . . . $30,000 $30,000 $30,000 Depreciation . . . . . . . . . . . . . . . . . . . . . $40,000 $40,000 $40,000 Merchandise purchases are paid in full during the month following purchase. Accounts payable for merchandise purchases on June 30, which will be paid during July, total $160,000. d. Equipment costing $25,000 will be purchased for cash during July. e. In preparing the cash budget, assume that the $60,000 loan will be made in July and repaid in September. Interest on the loan will total $2,000. Required: 1. Prepare a schedule of expected cash collections for July, August, and September and for the quarter in total. 2. Prepare a cash budget, by month and in total, for the third quarter. 3. If the company needs a minimum cash balance of $20,000 to start each month, can the loan be repaid as planned? Explain. [TOTAL: 25 MARKS] TASK 4: Budgetary Control Systems Facilitator Corp. is a company that acts as a facilitator in tax favored real estate swaps. Such swaps, known as 1031 exchanges, permit participants to avoid some or all of the capital gains taxes that would otherwise be due. The bookkeeper for the company has been asked to prepare a report for the company to help its owner/manager analyze performance. The first such report appears below: Facilitator Corp ASSIGNMENT – BMAC5203 Page 6 Analysis of Revenues and Costs For the Month Ended May 31 Planning Budget Actual Unit Variances Unit Revenues Revenues and Costs and Costs Exchanges completed . . . . . . . . 20 25 Revenue . . . . . . . . . . . . . . . . . . . $550 $500 $50 U Expenses: Legal and search fees . . . . . . 155 161 6 U Office expenses . . . . . . . . . . . 209 172 37 F Equipment depreciation . . . . . 30 24 6 F Rent . . . . . . . . . . . . . . . . . . . . 75 60 15 F Insurance . . . . . . . . . . . . . . . . 15 12 3 F Total expense . . . . . . . . . . . . . . . 484 429 55 F Net operating income . . . . . . . . . $ 66 $ 71 $ 5 F Note that the revenues and costs in the above report are unit revenues and costs. For example, the average office expense is $209 per exchange completed on the planning budget; whereas, the average actual office expense is $172 per exchange completed. Legal and search fees is a variable cost; office expenses is a mixed cost; and equipment depre ciation, rent, and insurance are fixed costs. In the planning budget, the fixed component of office expenses was $4,100. All of the company’s revenues come from fees collected when an exchange is completed. Required: 1. Evaluate the report prepared by the bookkeeper. 2. Prepare a performance report that would help the owner/manager assess the performance of the company in May. 3. Using the report you created, evaluate the performance of the company in May.

Attachments:

the assumptions underlying cost volume profit analysis make it a difficult method t 680777

“The assumptions underlying Cost Volume Profit Analysis make it a difficult method to apply in real world settings”.

</pclass=”msonormal”>Discuss this statement and conclude on the extent to which you agree with it. Use examples to illustrate your answer.

“For organisations that provide services rather than tangible goods or products, setting selling prices can be very difficult. This is because a high proportion of the costs of such organisations are usually indirect. This makes it difficult to determine reliably the cost of service provision and apply pricing related models such as cost plus pricing and target costing.”

Discuss this statement and conclude on the extent to which you agree with it. Use examples to illustrate your answer.

</pclass=”msonormal”></pclass=”msonormal”>

baker corporation acquired for an installed cost of 40 000 a machine having a recove 681262

Baker Corporation acquired, for an installed cost of $40,000, a machine having a recovery period of 5 years. Using the applicable percentages from Table 3.2, Baker calculates the depreciation in each year as follows:

 

 

Percentages

Depreciation

 

Cost

(from Table 3.2)

[(1) x (2)]

Year

(1)

(2)

(3)

1

$40,000

20%

$ 8,000

2

40,000

32

12,800

3

40,000

19

7,600

4

40,000

12

4,800

5

40,000

12

4,800

6

40,000

5

2,000

Totals

 

100%

$40,000

Column 3 shows that the full cost of the asset is written off over 6 recovery years.

substituting the values for baker corporation from its income statement table 3 4 in 681263

Substituting the values for Baker Corporation from its income statement (Table 3.4) into Equation 3.2, we get

OCF=$370 $120+ $100=$350

Baker Corporation during 2003 generated $350,000 of cash flow from producing and selling its output. Because Baker’s operating cash flow is positive, we can conclude that the firm’s operations are generating positive cash flows.

TABLE 3.4

Baker Corporation Income
Statement ($000) for the
Year Ended December 31,
2003

Sales revenue

$1,700

Less: Cost of goods sold

1,000

Gross profits

$700

Less: Operating expenses

 

Selling expense

$70

General and administrative expense

120

Lease expensea

40

Depreciation expense

100

Total operating expense

330

Earnings before interest and taxes (EBIT)

$370

Less: Interest expense

70

Net profits before taxes

$300

Less: Taxes (rate40%)

120

Net profits after taxes

$180

Less: Preferred stock dividends

10

Earnings available for common stockholders

$170

Earnings per share (EPS)b

$1.70

Equation 3.2

OCF=EBIT Taxes+ Depreciation

using the baker corporation rsquo s balance sheets in table 3 5 we see that its chan 681264

Using the Baker Corporation’s balance sheets in Table 3.5, we see that its change in net fixed assets between 2002 and 2003 was+$200 ($1,200 in 2003 $1,000 in 2002). Substituting this value and the $100 of depreciation for 2003 into Equation 3.4, we get Baker’s net fixed asset investment (NFAI) for 2003:

NFAI=$200+$100=$300

Baker Corporation therefore invested a net $300,000 in fixed assets during 2003. This amount would, of course, represent a net cash outflow to acquire fixed assets during 2003.

TABLE 3.5

 

Baker Corporation Balance
Sheets ($000)

 

December 31

Assets

2003

2002

Current assets

 

 

Cash

$400

$300

Marketable securities

600

200

Accounts receivable

400

500

Inventories

600

900

Total current assets

$2,000

$1,900

Gross fixed assets (at cost)

 

 

Land and buildings

$1,200

$1,050

Machinery and equipment

850

800

Furniture and fixtures

300

220

Vehicles

100

80

Other (includes certain leases)

50

50

Total gross fixed assets (at cost)

$2,500

$2,200

Less: Accumulated depreciation

1,300

1,200

Net fixed assets

$1,200

$1,000

Total assets

$3,200

$2,900

Liabilities and Stockholders’ Equity

 

 

Current liabilities

 

 

Accounts payable

$700

$500

Notes payable

600

700

Accruals

100

700

Total current liabilities

$1,400

200

Long term debt

$600

$1,400

Total liabilities

$2,000

$1,800

Stockholders’ equity

 

 

Preferred stock

$100

$100

Common stock—$1.20 par, 100,000 shares

 

 

outstanding in 2003 and 2002

120

120

Paid in capital in excess of par on common stock

380

380

Retained earnings

600

500

Total stockholders’ equity

$1,200

$1,100

Total liabilities and stockholders’ equity

$3,200

$2,900

Equation 3.4

NFAI=Change in net fixed assets + Depreciation

coulson industries a defense contractor is developing a cash budget for october nove 681266

Coulson Industries, a defense contractor, is developing a cash budget for October, November, and December. Coulson’s sales in August and September were $100,000 and $200,000, respectively. Sales of $400,000, $300,000, and $200,000 have been forecast for October, November, and December, respectively. Historically, 20% of the firm’s sales have been for cash, 50% have generated accounts receivable collected after 1 month, and the remaining 30% have generated accounts receivable collected after 2 months. Bad debt expenses (uncollectible accounts) have been negligible.5 In December, the firm will receive a $30,000 dividend from stock in a subsidiary. The schedule of expected cash receipts for the company is presented in Table 3.8. It contains the following items:

Forecast sales This initial entry is merely informational. It is provided as an aid in calculating other sales related items.

Cash sales The cash sales shown for each month represent 20% of the total sales forecast for that month.

Collections of A/R These entries represent the collection of accounts receivable (A/R) resulting from sales in earlier months.

 

TABLE 3.8

A Schedule of Projected Cash Receipts

for Coulson Industries ($000)

 

Aug.

Sept.

Oct.

Nov.

Dec.

Forecast sales

$100

$200

$400

$300

$200

Cash sales (0.20)

$20

$40

$ 80

$ 60

$ 40

Collections of A/R:

 

 

 

 

 

Lagged 1 month (0.50)

 

50

100

200

150

Lagged 2 months (0.30)

 

 

30

60

120

Other cash receipts

 

 

____

____

30

Total cash receipts

 

 

$ 210

$320

$ 340

Lagged 1 month These figures represent sales made in the preceding month that generated accounts receivable collected in the current month. Because 50% of the current month’s sales are collected 1 month later, the collections of A/R with a 1 month lag shown for September represent 50% of the sales in August, collections for October represent 50% of September sales, and so on.

Lagged 2 months These figures represent sales made 2 months earlier that generated accounts receivable collected in the current month. Because 30% of sales are collected 2 months later, the collections with a 2 month lag shown for October represent 30% of the sales in August, and so on.

Other cash receipts These are cash receipts expected from sources other than sales. Interest received, dividends received, proceeds from the sale of equipment, stock and bond sale proceeds, and lease receipts may show up here. For Coulson Industries, the only other cash receipt is the $30,000 dividend due in December.

Total cash receipts This figure represents the total of all the cash receipts listed for each month. For Coulson Industries, we are concerned only with October, November, and December, as shown in Table 3.8.

coulson industries has gathered the following data needed for the preparation of a c 681267

Coulson Industries has gathered the following data needed for the preparation of a cash disbursements schedule for October, November, and December.

Purchases The firm’s purchases represent 70% of sales. Of this amount, 10% is paid in cash, 70% is paid in the month immediately following the month of purchase, and the remaining 20% is paid 2 months following the month of purchase.6

Rent payments Rent of $5,000 will be paid each month.

Wages and salaries Fixed salary cost for the year is $96,000, or $8,000 per month. In addition, wages are estimated as 10% of monthly sales.

Tax payments Taxes of $25,000 must be paid in December.

Fixed asset outlays New machinery costing $130,000 will be purchased and paid for in November.

Interest payments An interest payment of $10,000 is due in December.

Cash dividend payments Cash dividends of $20,000 will be paid in October.

Principal payments (loans) A$20,000 principal payment is due in December.

Repurchases or retirements of stock No repurchase or retirement of stock is expected between October and December. The firm’s cash disbursements schedule, using the preceding data, is shown in Table 3.9. Some items in the table are explained in greater detail below.

Purchases This entry is merely informational. The figures represent 70% of the forecast sales for each month. They have been included to facilitate calculation of the cash purchases and related payments.

Cash purchases The cash purchases for each month represent 10% of the month’s purchases.

Payments of A/P These entries represent the payment of accounts payable (A/P) resulting from purchases in earlier months.

Lagged 1 month These figures represent purchases made in the preceding month that are paid for in the current month. Because 70% of the firm’s purchases are paid for 1 month later, the payments with a 1 month lag shown for September represent 70% of the August purchases, payments for October represent 70% of September purchases, and so on.

Lagged 2 months These figures represent purchases made 2 months earlier that are paid for in the current month. Because 20% of the firm’s purchases are paid for 2 months later, the payments with a 2 month lag for October represent 20% of the August purchases, and so on.

TABLE 3.9

A Schedule of Projected Cash

Disbursements for Coulson

Industries ($000)

 

Aug.

Sept.

Oct.

Nov.

Dec.

Purchases (0.70 x sales)

$70

$140

$280

$210

$140

Cash purchases (0.10)

$7

$14

$ 28

$ 21

$ 14

Payments of A/P:

 

 

 

 

 

Lagged 1 month (0.70)

 

49

98

196

147

Lagged 2 months (0.20)

 

 

14

28

56

Rent payments

 

 

5

5

5

Wages and salaries

 

 

48

38

28

Tax payments

 

 

 

 

25

Fixed asset outlays

 

 

 

130

 

Interest payments

 

 

 

 

10

Cash dividend payments

 

 

20

 

 

Principal payments

 

 

____

____

20

Total cash disbursements

 

 

$213

$418

$305

Wages and salaries These amounts were obtained by adding $8,000 to 10% of the sales in each month. The $8,000 represents the salary component; the rest represents wages. The remaining items on the cash disbursements schedule are self explanatory.

in october the firm will have an excess cash balance of 22 000 which can be held in 681268

Table 3.10 presents Coulson Industries’ cash budget, based on the data already developed. At the end of September, Coulson’s cash balance was $50,000, and its notes payable and marketable securities equaled $0.7 The company wishes to maintain, as a reserve for unexpected needs, a minimum cash balance of $25,000.

 width= For Coulson Industries to maintain its required $25,000 ending cash balance, it will need total borrowing of $76,000 in November and $41,000 in December. In October the firm will have an excess cash balance of $22,000, which can be held in an interest earning marketable security. The required total financing figures in the cash budget refer to how much will be owed at the end of the month; they do not represent the monthly changes in borrowing. The monthly changes in borrowing and in excess cash can be found by further analyzing the cash budget. In October the $50,000 beginning cash, which becomes $47,000 after the $3,000 net cash outflow, results in a $22,000 excess cash balance once the $25,000 minimum cash is deducted. In November the $76,000 of required total financing resulted from the $98,000 net cash outflow less the $22,000 of excess cash from October. The $41,000 of required total financing in December resulted from reducing November’s $76,000 of required total financing by the $35,000 of net cash inflow during December. Summarizing, the financial activities for each month would be as follows:

October: Invest the $22,000 excess cash balance in marketable securities.

November: Liquidate the $22,000 of marketable securities and borrow $76,000 (notes payable).

December: Repay $35,000 of notes payable to leave $41,000 of outstanding required total financing.

the most likely estimate is based on the expected outcomes presented earlier during 681269

Table 3.11 presents the summary of Coulson Industries’ cash budget prepared for each month of concern using pessimistic, most likely, and optimistic estimates of total cash receipts and disbursements. The most likely estimate is based on the expected outcomes presented earlier. During October, Coulson will, at worst, need a maximum of $15,000 of financing and, at best, will have a $62,000 excess cash balance. During November, its financing requirement will be between $0 and $185,000, or it could experience an excess cash balance of $5,000. The December projections show maximum borrowing of $190,000 with a possible excess cash balance of

TABLE 3.11

A Sensitivity Analysis of Coulson Industries’ Cash Budget ($000)

 

October

 

 

November

 

 

December

 

Pessi

Most

Opti

Pessi

Most

Opti

Pessi

Most

Opti

mistic

likely

mistic

mistic

likely

mistic

mistic

likely

mistic

Total cash

 

 

 

 

 

 

 

 

 

receipts

$160

$210

$285

$210

$320

$ 410

$275

$340

$422

Less: Total cash

 

 

 

 

 

 

 

 

 

disbursements

200

213

248

380

418

467

280

305

320

Net cash flow

($ 40)

($ 3)

$ 37

($170)

($ 98)

($ 57)

($ 5)

$ 35

$102

Add: Beginning

 

 

 

 

 

 

 

 

 

cash

50

50

50

10

47

87

(160)

(51)

30

Ending cash

$ 10

$ 47

$ 87

($160)

($ 51)

$ 30

($165)

($ 16)

$132

Less: Minimum

 

 

 

 

 

 

 

 

 

cash balance

25

25

25

25

25

25

25

25

25

Required total

 

 

 

 

 

 

 

 

 

financing

$ 15

$185

$ 76

$190

$ 41

Excess cash

 

 

 

 

 

 

 

 

 

balance

$ 22

$ 62

$ 5

$107

$107,000. By considering the extreme values in the pessimistic and optimistic outcomes, Coulson Industries should be better able to plan its cash requirements. For the 3 month period, the peak borrowing requirement under the worst circumstances would be $190,000, which happens to be considerably greater than the most likely estimate of $76,000 for this period.

cash budget and pro forma balance sheet inputs jane mcdonald a financial analyst for 681272

Cash budget and pro forma balance sheet inputs Jane McDonald, a financial analyst for Carroll Company, has prepared the following sales and cash disbursement estimates for the period February–June of the current year.

 

 

Cash

Month

Sales

disbursements

February

$500

$400

March

600

300

April

400

600

May

200

500

June

200

200

Ms. McDonald notes that historically, 30% of sales have been for cash. Of credit sales, 70% are collected 1 month after the sale, and the remaining 30% are collected 2 months after the sale. The firm wishes to maintain a minimum ending balance in its cash account of $25. Balances above this amount would be invested in short term government securities (marketable securities), whereas any deficits would be financed through short term bank borrowing (notes payable). The beginning cash balance at April 1 is $115.

a. Prepare a cash budget for April, May, and June.

b. How much financing, if any, at a maximum would Carroll Company require to meet its obligations during this 3 month period?

c. A pro forma balance sheet dated at the end of June is to be prepared from the information presented. Give the size of each of the following: cash, notes payable, marketable securities, and accounts receivable.

pro forma income statement euro designs inc expects sales during 2004 to rise from t 681273

Pro forma income statement Euro Designs, Inc., expects sales during 2004 to rise from the 2003 level of $3.5 million to $3.9 million. Because of a scheduled large loan payment, the interest expense in 2004 is expected to drop to $325,000. The firm plans to increase its cash dividend payments during 2004 to $320,000. The company’s year end 2003 income statement follows.

Euro Designs, Inc.

Income Statement

for the Year Ended December 31, 2003

Sales revenue

$3,500,000

Less: Cost of goods sold

1,925,000

Gross profits

$1,575,000

Less: Operating expenses

420,000

Operating profits

$1,155,000

Less: Interest expense

400,000

Net profits before taxes

$755,000

Less: Taxes (rate 40%)

302,000

Net profits after taxes

$453,000

Less: Cash dividends

250,000

To retained earnings

$3,500,000

a. Use the percent of sales method to prepare a 2004 pro forma income statement for Euro Designs, Inc.

b. Explain why the statement may underestimate the company’s actual 2004 pro forma income.

depreciation on march 20 2003 norton systems acquired two new assets asset a was res 681274

Depreciation On March 20, 2003, Norton Systems acquired two new assets. Asset A was research equipment costing $17,000 and having a 3 year recovery period. Asset B was duplicating equipment having an installed cost of $45,000 and a 5 year recovery period. Using the MACRS depreciation percentages in Table 3.2 on page 100, prepare a depreciation schedule for each of these assets.

TABLE 3.2

Rounded Depreciation

Percentages by Recovery Year

Using MACRS for First Four

Property Classes

 

Percentage by recovery yeara

Recovery year

3 years

5 years

7 years

10 years

1

33%

20%

14%

10%

2

45

32

25

18

3

15

19

18

14

4

7

12

12

12

5

 

12

9

9

6

 

5

9

8

7

 

 

9

7

8

 

 

4

6

9

 

 

 

6

10

 

 

 

6

11

___

___

___

4

Totals

100%

100%

100%

100%

macrs depreciation expense and accounting cash flow pavlovich instruments inc a make 681276

MACRS depreciation expense and accounting cash flow Pavlovich Instruments, Inc., a maker of precision telescopes, expects to report pre tax income of $430,000 this year. The company’s financial manager is considering the timing of a purchase of new computerized lens grinders. The grinders will have an installed cost of $80,000 and a cost recovery period of 5 years. They will be depreciated using the MACRS schedule.

a. If the firm purchases the grinders before year end, what depreciation expense will it be able to claim this year? (Use Table 3.2 on page 100.)

b. If the firm reduces its reported income by the amount of the depreciation expense calculated in part a, what tax savings will result?

c. Assuming that Pavlovich does purchase the grinders this year and that they are its only depreciable asset, use the accounting definition given in Equation 3.1 to find the firm’s cash flow from operations for the year.

Equation 3.1

Cash flow from operations=

Net profits after taxes + Depreciation and other noncash charges

depreciation and accounting cash flow a firm in the third year of depreciating its o 681277

Depreciation and accounting cash flow A firm in the third year of depreciating its only asset, which originally cost $180,000 and has a 5 year MACRS recovery period, has gathered the following data relative to the current year’s operations.

Accruals

$ 15,000

Current assets

120,000

Interest expense

15,000

Sales revenue

400,000

Inventory

70,000

Total costs before depreciation, interest, and taxes

290,000

Tax rate on ordinary income

40%

a. Use the relevant data to determine the accounting cash flow from operations (see Equation 3.1) for the current year.

Equation 3.1

Cash flow from operations=

Net profits after taxes + Depreciation and other noncash charges

b. Explain the impact that depreciation, as well as any other noncash charges, has on a firm’s cash flows.

classifying inflows and outflows of cash classify each of the following items as an 681278

Classifying inflows and outflows of cash Classify each of the following items as an inflow (I) or an outflow (O) of cash, or as neither (N).

Item

Change ($)

Item

Change ($)

Cash

+100

Accounts receivable

700

Accounts payable

1,000

Net profits

+600

Notes payable

+500

Depreciation

+100

Long term debt

2,000

Repurchase of stock

+600

Inventory

+200

Cash dividends

+800

Fixed assets

+400

Sale of stock

+1,000

stock dividends the company with the common equity accounts shown here has declared 677929

Stock Dividends. The company with the common equity accounts shown here has declared a 10 percent stock dividend at a time when the market value of its stock is $48 per share. What effects on the equity accounts will the distribution of the stock dividend have?

Common stock ($1 par value)

$ 425.000

Capital surplus

1,430.000

Retained earnings

3,250.000

Total owners” equity

$5.105,000

residual dividend policy lunch special inc predicts that earnings in the coming year 677933

Residual Dividend Policy. Lunch Special. Inc., predicts that earnings in the coming year will be $43 million. There are eight million shares, and Lunch Special maintains a debt equity ratio of .9.

a. Calculate the maximum investment funds available without issuing new equity and the increase in borrowing that goes along with it.

b. Suppose the firm uses a residual dividend policy. Planned capital expenditures  total $50 million. Based on this information, what will the dividend per share be”?

c. In part (b). how much borrowing will take place”? What is the addition to planned earnings”?

d. Suppose the company plans no capital outlays for the coming year. What will the dividend be under a residual policy? What will new borrowing he?

dividends and taxes as discussed in the text in the absence of market imperfections 677937

Dividends and Taxes. As discussed in the text, in the absence of market imperfections and tax effects, we would expect the share price to decline by the amount of the dividend payment when the stock goes ex dividend. Once we consider the role of taxes, however, this is not necessarily true. One model has been proposed that incorporates tax effects into determining the ex dividend price:

(P0  PX,)/D = (1  TP)/( 1 TG)

where PI! is the price just before the stock goes ex, P, is the ex dividend share price, D is the amount of the dividend per share, 7~, is the relevant marginal personal tax rate on dividends, and T(, is the effective marginal tax rate on capital gains.

a, If Tp TG = 0, how much will the share price fall when the stock goes ex”?

b, If Tp  = 15 percent and TG= 0, how much will the share price fall?

c. If Tp = 15 percent and TG= 30 percent, how much will the share price fall?

d, Suppose the only owners of stock are corporations. Recall that corporations get at least a 70 percent exemption from taxation on the dividend income they receive. but they do not get such an exemption on capital gains. If the corporation”s income and capital gains tax rates arc both 35 percent. what does this model predict the ex dividend share price will be?

c, What does this problem tell you about real world tax considerations and the dividend policy of the firm?

electronic timing inc eti is a small company founded 15 years ago by electronics eng 677941

ELECTRONIC TIMING, INC.

Electronic Timing. Inc. (ETI), is a small company founded 15 years ago by electronics engineers Tom Miller and Jessica Kerr ETI manufactures integrated circuits to capitalize on the complex mixed signal design technology and has recently entered the market for frequency timing generators, or silicon timing devices. which provide the timing signals or “clocks” necessary to synchronize electronic systems. Its clock products originally were used in PC video graphics applications, but the market subsequently expanded to include motherboards, PC peripheral devices, and other digital consumer electronics, such as digital television boxes and game consoles. ETI also designs and markets custom application specific integrated circuits (ASICs) for industrial customers. The ASIC”s design combines analog and digital. or mixed signal. technology. In addition to Tom and Jessica, Nolan Pittman, who provided capital for the company, is the third primary owner. Each owns 25 percent of the 1 million shares outstanding. The company has several other individuals. including current employees, who own the remaining shares.

Recently. the company designed a new computer

motherboard. The company”s design is both more

efficient and less expensive to manufacture, and the

 ETI design is expected to become standard in many personal computers. After investigating the possibility of manufacturing the new motherboard, ETI determined that the costs involved in building a new plant would be prohibitive. The owners also decided that they were unwilling to bring in another large outside owner. Instead, ETI sold the design to an outside firm. The sale of the motherboard design was completed for an aftertax payment of $30 million

1. Tom believes the company should use the extra cash to pay a special one time dividend. How will this proposal affect the stock price? How will it affect the value of the company?

2. Jessica believes the company should use the extra cash to pay off debt and upgrade and expand its existing manufacturing capability. How would Jessica”s proposals affect the company?

3. Nolan favors a share repurchase. He argues that a repurchase will increase the company”s PIE ratio, return on assets, and return on equity. Are his arguments correct? How will a share repurchase affect the value of the company?

4. Another option discussed by Tom. Jessica, and Nolan would be to begin a regular dividend payment to shareholders. How would you evaluate this proposal?

5 One way to value a share of stock is the dividend growth. or growing perpetuity, model. Consider the following: The dividend payout ratio is 1 minus b, where is the “retention” or “plowback” ratio. So, the dividend next year will be the earnings next year, E” times 1 minus the retention ratio. The most commonly used equation to calculate the sustainable growth rate is the return on equity times the retention ratio. Substituting these relationships into the dividend growth model. we get the following equation to calculate the price of a share of stock today:

 width= 

What are the implications of this result in terms of whether the company should pay a dividend or upgrade and expand its manufacturing capability? Explain.

6. Does the question of whether the company should pay a dividend depend on whether the company is organized as a corporation or an LLC?

job order costing part b questions 678087

Problem 2 4A

Agassi Company uses a job order cost system in each of its three manufacturing departments. Manufacturing overhead is applied to jobs on the basis of direct labor cost in Department D, direct labor hours in Department E, and machine hours in Department K.

In establishing the predetermined overhead rates for 2014, the following estimates were made for the year.

Department

D E K

Manufacturing overhead $1,065,000 $1,525,000 $840,000

Direct labor costs $1,500,000 $1,625,000 $585,000

Direct labor hours 130,000 125,000 52,000

Machine hours 520,000 650,000 120,000

During January, the job cost sheets showed the following costs and production data.

Department

D E K

Direct materials used $182,000 $163,800 $101,400

Direct labor costs $156,000 $143,000 $48,750

Manufacturing overhead incurred $128,700 $161,200 $102,700

Direct labor hours 10,400 14,300 4,550

Machine hours 44,200 58,500 10,300

LINK TO TEXT

LINK TO TEXT

Compute the predetermined overhead rate for each department.

Overhead rate

Department D (Round answers to 2 decimal places, e.g. 10.25%.)

%

Department E (Round answers to 2 decimal places, e.g. $10.25.) $

per direct labor hour

Department K (Round answers to 2 decimal places, e.g. $10.25.) $

per machine hour

LINK TO TEXT

LINK TO TEXT

Compute the total manufacturing costs assigned to jobs in January in each department.

Manufacturing Costs

Department D $

Department E $

Department K $

LINK TO TEXT

LINK TO TEXT

Compute the under or overapplied overhead for each department at January 31. (Enter all amounts as positive numbers and select appropriate option from the list.)

Manufacturing Overhead

Department D $

Department E $

Department K $

2 9 part B Prepare an income statement for May through gross profit.

MANTLE COMPANY

(Partial) Income Statement

For the Month Ended May 31, 2014

$

$

:

$

2 11 PART B

Determine the balance of the Service Contracts in Process account.

Balance of Service Contracts in Process account. $

Click if you would like to Show Work for this question: Open Show Work

2 13 PART B

Prepare the entry to apply the overhead for the year. (Credit account titles are automatically indented when amount is entered. Do not indent manually.)

Account Titles and Explanation Debit Credit

Click if you would like to Show Work for this question: Open Show Work

Attachments:

lee nicholas has been the principal stockholder and has operated world com advertisi 678100

Lee Nicholas has been the principal stockholder and has operated World.com Advertising, Inc., since its beginning 10 years ago. The company has prospered. Recently, Nicholas mentioned that he would sell the business for the right price. Assume that you are interested in buying World.com Advertising. You obtain the most recent monthly trial balance, which follows. Revenues and expenses vary little from month to month, and January is a typical month. The trial balance shown is a preliminary or unadjusted trial balance. The controller informs you that the necessary accrual adjustments should include revenues of $3,800 and expenses of $1,100. Also, if you were to buy World.com Advertising, you would hire a manager so you could devote your time to other duties. Assume that this person would require a monthly salary of $5,000.

Requirements
1. Assume that the most you would pay for the business is 20 times the monthly net income you could expect to earn from it. Compute this possible price.
2. Nicholas states the least he will take for the business is an amount equal to the business’s stockholders’ equity balance on January 31. Compute this amount.
3. Under these conditions, how much should you offer Nicholas? Give your reason.

locate a copy of enron the smartest guys in the room video prepare a paper discussin 678108

Paper due Week 2:Locate a copy of Enron:The Smartest Guys In The Room video. Prepare a paper of three pages or less discussing the implications of the documentary.

Document Preview:

Paper due Week 2:Locate a copy of Enron:The Smartest Guys In The Room video. Prepare a paper of three pages or less discussing the implications of the documentary.

Attachments:

financial accounting 678131

RESEARCH ESSAY TOPIC

“It is surprising and distressing to find so little discussion of the total asset figure in the accounting literature. Since total assets is one of the central concepts of accounting, one would expect to find a good deal of explanation and interpretation.”

Document Preview:

4RESEARCH ESSAY – SEMESTER 2, 2014 INFORMATION Your research essay must be submitted at the beginning of your seminar in week 10. Penalties will apply for late submission. The following matters should be given particular attention: Writing assignments must not exceed the word counts indicated. Double space your pages, use a 12 pt Times New Roman font, use 2 cm margins on all four sides of your page. Your essay must include an abstract/synopsis, introduction, essay body that clearly addresses the problem areas, a conclusion and a properly referenced bibliography. (refer to the research essay marking rubric for further guidance) Evidence of extensive research beyond the prescribed text is required. Ensure these are referenced appropriately in your bibliography. Refer to the statement regarding plagiarism. NO extensions will be granted unless supported by appropriate documentation prior to the due date. This assignment must be handed in for successful completion of the course and will count 20 marks towards the final mark. The word count for the research essay is 3000 words. Please refer to the Research Essay Marking Rubric for the specific allocation of word count for each specific section of your research essay (refer point 2 above). Points (fractional marks) have also been allocated to each specific section of your research essay. Student must select a company (Caltex group Australia) from the Australian Securities Exchange (ASX) website and download the 2014 annual report. Students can not do the same company and preferably from different ASX sectors and should discuss their selection prior to commencing their research essay. No banks allowed. Company selected: (Caltex group Australia) RESEARCH ESSAY TOPIC “It is surprising and distressing to find so little discussion of the total asset figure in the accounting literature. Since total assets is one of the central concepts of accounting, one would expect to find a good deal of explanation and…

Attachments:

howie stars produces starts for elementary teachers to reward their students howie s 678179

Howie Stars produces starts for elementary teachers to reward their students. Howie Stars’ trial balance on June 1 follows:
HOWIE STARS
Trial Balance
June 1, 2012
Balance
Account Title Debit Credit
Cash $14,000
Accounts receivable 1,55,000
Inventories:
Materials 5,700
Work in process 39,400
Finished goods 20,400
Plant assets 2,00,000
Accumulated depreciation $72,000
Accounts payable 1,27,000
Wages payable 1,700
Common stock 1,42,000
Retained earnings 91,800
Sales revenue
Cost of goods sold
Manufacturing overhead
Marketing and general expenses
Total $4,34,500 4,34,500
June 1 balances in the subsidiary ledgers were as follows:
– Materials subledger: Paper, $4,700; indirect materials, $1,000
– Work in process subledger: Job 120, $39,400; $0 for Job 121
– Finished goods subledger: Large Stars, $9,400; Small Stars, $11,000
June transactions are summarized as follows:
a. Collections on account, $152,000.
b. Marketing and general expenses incurred and paid, $28,000.
c. Payments on account, $36,000.
d. Materials purchases on credit: Paper, $22,900; indirect materials, $3,800.
e. Materials used in production (requisitioned):
– Job 120: paper , $850
– Job 121: paper, $7,650
– Indirect materials, $1,000
f. Wages incurred and assigned during June, $35,000. Labor time records for the month: Job 120, $3,500; Job 121, $16,600; indirect labor, $14,900.
g. Wages paid in June include the balance in the Wages payable account at May 31 and $32,200 of wages incurred during June.
h. Depreciation on plant and equipment, $2,600.
i. Manufacturing overhead was allocated at the predetermined rate of 50% of direct labor cost.
j. Jobs completed during the month: Job 120, 300,000 Large Stars at total cost of $45,500.
k. Credit sales on account: all of Job 120 for $111,000.
l. Closed the Manufacturing overhead account to Cost of goods sold.
Requirements:
1. Journalize the transactions for the company. Howie uses a perpetual inventory system.
2. Open T-accounts for the general ledger, the Materials ledger, the Work in process ledger, and the Finished goods ledger. Insert each account balance as given, and use the reference Bal. Post the journal entries to the T-accounts using the transaction letters as a reference.
3. Prepare a trial balance at June 30, 2012.
4. Use the Work in process inventory T-account to prepare a schedule of cost of goods manufactured for the month of June.
5. Prepare an income statement for the month of June.

Attachments:

you are to prepare a submission to satisfy your client s requirements as outlined be 678207

You are to prepare a submission to satisfy your client’s requirements as outlined below. You arerequired to investigate the issues at Pressure Hydraulics and provide a feasibility study into theimprovement to their current business processes. You will need to conduct an investigation into theclient’s problems and evaluate a solution to meet their business needs.Overview – Pressure HydraulicsPressure Hydraulics is a locally owned business that currently has three service centres; Newcastle,Toronto and Maitland. Each service centre provides maintenance and specialised servicing ofhydraulic systems as used in cars, trucks and earth moving equipment. The Maitland service centrealso provides a specialty service to the mining industry where two purpose built trucks go onsite toservice a range of mining equipment.The business has become quite profitable in the past years and its owner, Allan Taylor, has devisedplans to expand by opening service centres at Coffs Harbour and Gosford. Allan has future plans forother service centres along the east coast of NSW. He also feels that the time is right to look at how ITcan support the existing business and enable his future business plans.Currently, Allan spends a portion of each day at each service centre to monitor its operations. This isleaving little time to continue developing his business and he realises he will not be able to spend thesame sort of time in the Coffs Harbour and Gosford service centres.Existing SystemEach service centre operates as an independent business, with eight technicians in the workshop (oneof whom is a workshop foreman) and one office assistant. The office assistant takes phone calls frompeople requesting quotes for work or to have work done. For requests on quotes, the office assistantlooks up a hard copy of a price book (known as The Price Book) and gives a verbal quote. Allan isunhappy with the time being taken to update the Price Book as the office assistant is often too busy toupdate the prices from the supplier invoices. Sometimes, this time delay results in either lost sales ifthe price has gone down or lost income if the price has gone up.If a customer phones to make a booking for work to be done, the office staff assistant writes down thedetails in a journal (Works Book). Any variations to the Works Book by a customer requesting tochange a date/time results in crossed out entries. There have been times when this has been the causeof misinterpretation as to when a job is booked in. Other problems with the system include occasionaloverbooking or slack periods for the workshop staff.When a customer arrives at a service centre for work to be done, the office staff member writes out aJob Card with the customer details along with the price from The Price Book for the job. There issometimes a discrepancy between what the Price Book shows and what the customer says they werequoted for that particular work. When completed, the Job Card is passed to the workshop foremanwho assigns the job to a technician. When the job is completed, the technician initials the Job Cardand gives it back to the workshop foreman, who returns it to the office. The office assistant writes outan invoice and collects payment from the customer; cash, EFTPOS and credit card are acceptableforms of payment. Once per week, the office assistant uses the Job Cards to prepare an order for stockreplacement.Allan has developed a business relationship with several transport companies where two technicianswill go to the transport companies’ central site and service the hydraulics on a truck. As the partsrequired to perform the service can be varied, a special van equipped with basic workshop equipmentand a full range of component parts is used. The time taken for these jobs can be varied and unpredictable. Upon returning from such a job, the technicians will alert the workshop foreman thatthe job has been completed. The office assistant is then informed and an invoice written out andmailed to the transport company. Carbon copies are used to track such invoices. The Maitland servicecentre provides a service to the mining industry and there have been a few repeated issues with theincorrect component parts being taken onsite for servicing the machinery. Such mistakes not onlycosts a loss in terms of travel time back to the service centre to collect the correct part, but the miningcompanies have very tight maintenance schedules. Machinery has to be available and delays are nottolerated.Each fortnight, Allan contacts the workshop foremen and checks the hours worked by staff at theservice centre. He then prepares the payroll and writes cheques for each employee.Each day, the office assistant banks the day’s takings and gives the bank receipt to Allan when hemakes his daily call. Usually this will be the following day from the bank deposit. Allan then does areconciliation of money banked with the previous day’s takings.Apart from the issues identified above, Allan wants to achieve the following: Ensuring repeat business from mining companies. Improve control over stock ordering. A new system is required to manage this aspect of thebusiness. The ability to order stock for the three businesses instead of individual orderingwould ensure economies not being realised at the moment. Ability to quickly determine if a required component part is in stock. Ability to generate an invoice off data held within the system. Improved payroll process to reduce the need to contact staff for details. Allan would prefer asystem that recorded the working hours daily and also supported staff being paid directlythrough his bank system. Tracking customers for repeat business. Allan wants a system that will track customers andcontact them at specified times with special offers. Price Book issues. A system to have timely production of price variations to arrive at allservice centres on the same day. Technology to support these changes.After an initial investigation, Allan has asked your group to develop a feasibility report on providing asuitable solution for the accounting and payroll problems. At a minimum level, your feasibility reportshould contain the following generic sections:Your Task:You are required to submit a feasibility report.At a minimum level, your feasibility report should contain the following generic sections:? Executive Summary? Description of the problem? Solution objectives? Constraints? Development plans? Potential solutions? RecommendationsYour appendix should also contain? Any brochures (scans), websites, photographs and prices you have researched to complete yourassignment.A group evaluation sheet is also to be submitted by each member of the group on Blackboard bythe end of week 12. Any student not submitting this form will not receive a mark. Feasibility Report Marking GuidelinesExecutive Summary …/2Description of the problem …/3Solution objectives …/3Constraints …/1Development plans …/3Potential solutions …/2Recommendations …/2Presentation, clarity of discussion …/2Logic and evidence of research …/2Total Mark …/20Your total mark will be added to your group presentation mark out of 5 for a total mark out of 25.

Attachments:

harrelson company manufactures pizza sauce through two production departments cookin 678216

Harrelson Company manufactures pizza sauce through two production departments: Cooking and Canning. In each process, materials and conversion costs are incurred evenly throughout the process. For the month of April, the work in process accounts show the following debits.

Document Preview:

Chapter 3A 1 Exercise 3 2 Harrelson Company manufactures pizza sauce through two production departments: Cooking and Canning. In each process, materials and conversion costs are incurred evenly throughout the process. For the month of April, the work in process accounts show the following debits. ??Cooking??Canning??Beginning work in process??$0??$4,840??Materials??24,200??10,100??Labor??9,500??7,790??Overhead??33,750??28,800??Costs transferred in????55,620???Journalize the April transactions. (Credit account titles are automatically indented when amount is entered. Do not indent manually.) Date?Account Titles and Explanation?Debit?Credit??April 30????????????????(To record materials used.)????30????????????????(To assign factory labor to production.)????30????????????????(To assign overhead to production.)????30???????????(To record costs transferred in.)????Click if you would like to Show Work for this question:?? HYPERLINK “javascript:void();” ?Open Show Work??? ??? ??? HYPERLINK “http://edugen.wileyplus.com/edugen/shared/assignment/test/qview.uni?id=quest2089316entrance1&selected_question=quest2089316&operation=take question” o “Link to Text 1” ?LINK TO TEXT??????? Excersice 3 4Exercise 3 4 Schrager Company has two production departments: Cutting and Assembly. July 1 inventories are Raw Materials $4,510, Work in Process—Cutting $3,810, work in Process—Assembly $11,510, and Finished Goods $32,550. During July, the following transactions occurred. 1.??Purchased $63,850 of raw materials on account.??2.??Incurred $60,240 of factory labor. (Credit Wages Payable.)??3.??Incurred $72,330 of manufacturing overhead; $42,370 was paid and the remainder is unpaid.??4.??Requisitioned materials for Cutting $17,150 and Assembly $9,560.??5.??Used factory labor for Cutting $34,790 and Assembly $25,450.??6.??Applied overhead at the rate of $19 per machine hour. Machine hours were Cutting 1,720 and Assembly 1,780.??7.??Transferred goods costing $69,540 from the Cutting…

Attachments:

customers owed abc 45 000 at the beginning of 2013 and 55 000 at the end of 2013 678254

ACCT 3331 Name _________________________________ Quiz 1 – Chapters 1, 2, and 3 (10 pts.) DUE: September 11, 2014 Section ________________________________ 1. Listed below are several terms and phrases associated with the FASB’s conceptual framework. Pair each item from List A with the item from List B that is most appropriately associated with it. List A List B 1. Relevance a. Important for making inter firm comparisons 2. Timeliness b. Applying the same accounting practices over time 3. Faithful Representation c. Information is available when investors need it 4. Materiality d. Pertinent to the decision at hand 5. Comparability e. Implies consensus among different measurers 6. Neutrality f. The absence of bias 7. Consistency g. The financial statements do not contain material mistakes 8. Verifiability h. Agreement between a measure and the phenomenon it purports to represent 9. Free from error i. Concerns the relative size of an item and its effects on decisions 2. ABC Company uses cash basis accounting for its records. During 2013, ABC collected $400,000 from its customers, made payments of $300,000 to its suppliers for inventory (e.g., Purchases), and paid $140,000 for operating costs. ABC wants to prepare accrual basis financial statements. In gathering information for the accrualbasis financial statements, ABC discovered the following: ? Customers owed ABC $45,000 at the beginning of 2013 and $55,000 at the end of 2013. ? ABC owed suppliers $20,000 at the beginning of 2013 and $28,000 at the end of the 2013. ? ABC’s beginning inventory was $40,000 and its ending inventory was $44,000. What should ABC’s Gross Profit be on the accrual basis? $______________________________ 3. At the end of 2013, ABC did NOT make the adjusting entries indicated below. Indicate the effect of the error on 2013 Net Income, Assets, Liabilities, and Owner’s Equity (on December 31, 2013). Use O for overstate, U for understate, and NE for no effect. Assume each error is independent of the others. (You do not need to specify the dollar effect of the error.) Error Net Income Assets Liabilities Owner’s Equity Entry to adjust supplies expense for the year. The company charges the cost of supplies purchased throughout the year to a nominal account. Entry to record depreciation of equipment. Entry to record interest revenue on a short term Note Receivable, with all interest and principle received in four months. Entry to record the expired portion of a two year life insurance policy paid for on July 1, 2013, and charged to a permanent account. Entry to record accrued salaries and wages earned by employees at fiscal year end. 4. Comet Graphics Company was organized on January 1, 2013. The trial balance before adjustment at December 31, 2014 contained the following account balances: Debits Credits Cash $9,500 Accounts Receivable 14,000 Supplies 2,700 Prepaid Insurance 1,800 Equipment 45,000 Accounts Payable 9,000 Notes Payable 17,000 Common Stock 15,000 Retained Earnings (Beginning) 12,000 Dividend 2,000 Graphic Fees Revenue 52,100 Consulting Fees Revenue 5,000 Salaries Expense 30,000 Advertising Expense 1,900 Rent Expense 1,500 Utilities Expense 1,700 $110,100 $110,100 Analysis reveals the following additional data: 1. The $2,700 balance in Supplies represents supplies purchased in January. At December 31, there was $1,500 of supplies on hand. 2. The note payable was issued on September 1. It is a 12%, 6 month note, with principle and interest due at maturity. 3. The balance in Prepaid Insurance is the premium on a one year policy, dated March 1, 2014. 4. Consulting Fees are credited to revenue when received. At December 31, consulting fees of $1,000 contracted for January 2015 have yet to be performed. All other consulting projects have been finished. [Hint: Have they recognized revenue at the correct time?] 5. The equipment was purchased on January 1, 2014. It has a 10 year useful life and no salvage value. The company uses the straight line depreciation method. Required: Determine Ending Retained Earnings at December 31, 2014 (after adjusting entries and closing) $ ____________________

Attachments:

auditing 678285

Powersys is an electricity distribution company based in a large capital city. Its business is to manage the electricity assets, including poles, wires and other equipment, that are used to deliver electricity to more than 500 000 retail and business customers in the city. Pole, wire and substation maintenance and improvements are a large part of the company’s operations and teams of highly trained technicians are used for both planned work and emergency response activities. Emergency response is required when storms or fires bring down power lines, the power must be turned off at the direction of police, or the electricity supply fails for any reason.

Each team comprises several vehicles (vans and trucks) and uses additional heavy equipment, such as cherry pickers, cranes and diggers, as required. Each vehicle carries a core set of specialized parts and tools and additional items are obtained as required from the stores, located in a large warehouse in the northern suburbs. The warehouse is staffed on a 24 hour basis to assist night maintenance (designed to minimize disruption to business customers) and emergency response.

Required

(a) Make a list of the potential problems that could occur in Powersys’ maintenance and improvements program. (20 marks)

(b) Suggest ways that good internal control over parts, equipment and labour could help Powersys avoid these problems. (20 marks)

why should short term prepaid expenses be classified as current assets 678342

Assignment 2 Value: 35% Return date: 05 Oct 2014 Submission method options Alternative submission method Task Please note this assignment consists of questions from Horngren, C.T., Sundem, G.L., Stratton, W.O. Burgstahler, D., & Shatzberg, J. Introduction to management accounting, (15th edition 2014). You are required to complete all questions and submit all necessary workings. Follow the assignment requirements (refer below to the Requirements section) very closely. Content Assessed?Topics 2, 3, and 4?? Question 1 16 2, 16 6, 16 33 (illustrate your explanation with a numerical example).?Source: Horngren, et al. (2014). Introduction to management accounting (16th ed., p. 719). Pearson. 16 2. Why should short term prepaid expenses be classified as current assets? 16 6. What factors influence the estimate of useful life in depreciation accounting? 16 33. Study Appendix 16A. Suppose prices are rising and inventories are increasing. Which of the four generally accepted inventory accounting methods will usually result in the highest net income? Explain. Illustrate your explanation with a numerical example. Question 2 16 A1 Reigle Balance Sheet and Income Statement?Source: Horngren, et al. (2014). Introduction to management accounting (16th ed., p. 716). Pearson. The Reigle Company had the following items on its December 31, 20X0 balance sheet and 20X0 income statement (in dollars except for the number of shares outstanding): Cash and equivalents $ 60,000 Revenues $ 790,000 Notes Payable $ 51,000 Long term debt, excluding current portion $ 210,000 Accounts Receivable, net $ 48,000 Provision for income taxes $ 60,000 Other long term assets $ 110,000 Interest expense $ 55,000 Deferred income tax liability – long term $ 44,000 Retained earnings $ 204,000 Income taxes payable $ 37,000 Cost of sales $ 470,000 Inventories $ 36,000 Prepaid expenses $ 15,000 Common stock (50,000 shares) $ 25,000 Property, Plant and equipment, at cost $ 580,000 Accounts payable $ 43,000 Interest income $ 15,000 Goodwill, patents and trademarks $ 75,000 Current portion of long term debt $ 16,000 Less: Accumulated depreciation $ 170,000 Selling and administrative expenses $ 150,000 Additional paid in capital ? Prepare in proper form the December 31, 20X0 balance sheet and the 20X0 income statement for Reigle Company Solve using a spreadsheet following the assignment requirements listed in the ‘Requirements’ section below, noting the checklist for all spreadsheets: checklist: separate data/report, paste normal view and formula view, display row and column headings, IF functions and submit the doc file with the pasted views AND the spreadsheet file to turnitin. Create a second version or your spreadsheet, changing the following values: ? Cash & equivalents $305,000 ? Revenues $1,180,000 ? Notes payable $150,500 ? Long term debt, excluding current portion $500,000 How much is the additional paid in capital??Paste this second version. There is no need to paste the formula view again. Question 3 16 56 LIFO and FIFO?Source: Horngren, et al. (2014). Introduction to management accounting (16th ed., p. 724). Pearson. Study Appendix 16A. The inventory of West Virginia Coal Company on June 30 shows 500 tons at $62 per ton. A physical inventory on July 31 shows a total of 600 tons on hand. Revenue from sales of coal for July totals $105,000. The following purchases were made during July: July 5, 1,000 tons at $64 per ton; July 15, 250 tons at $66 per ton; July 25, 300 tons at $69 per ton. 1. Compute the inventory value as of July 31, using (a) FIFO and (b) LIFO. 2. Compute the gross profit, using each method. Solve using a spreadsheet. Check spreadsheet instructions. Ensure that your data entry and report areas are separate. Question 4 Watch the ABC Learning videos (see Interact) and answer the following questions. (about 300 400 words in total) 1. What went wrong at ABC Learning? 2. What are the major financial reports? What is the purpose of each? 3. Why are profits important? Why is cash flow important? 4. What are the characteristics of a good internal control system? Use the Internet as a resource for this part of the question. Reference your sources. These videos are also available on YouTube http://www.youtube.com/watch?v=YYF6JW9vJKo Question 5 17 18 (also explain which 3 of these ratios is, in your opinion, the most useful to management)?Source: Horngren, et al. (2014). Introduction to management accounting (16th ed., p. 761). Pearson. 17 18. Name two short term ratios and five profitability ratios. Explain which 3 of these ratios is, in your opinion, the most useful to management. Question 6 17 47 Financial Ratios?Source: Horngren, et al. (2014). Introduction to management accounting (16th ed., p. 769). Pearson. Honda Motor Company is a major Japanese company with sales equivalent to more than $100 billion. The company’s income statement and balance sheet for the year ended, March 31, 2011, are shown in Exhibit 17 10 and 17 11. Monetary amounts are in Japanese Yen (¥). Net sales ¥ 8,937 Cost of sales 6,497 Gross profit 2,440 Selling and administrative 1,383 Research and development 487 Operating Income 570 Other income (expense) Interest income 23 Interest expense (9) Equity in income of affiliates 140 Other, net 46 Income before income taxes 770 Income tax expense 207 Net income 563 Amounts per share Net income (EPS) 296 Cash dividends 54 Exhibit 17 10 Honda Motor Company, Ltd Income Statement for the Year Ended March 31, 2011 (in billions, except per share amounts) Assets Current Assets Cash and cash equivalents ¥1,279 Receivables 1,919 Inventories 900 Other 592 Total Current Assets 4,690 Property, plant and equipment 3,297 Investments 640 Other assets 2,944 Total assets ¥11,571 Liabilities and Shareholder’s Equity Current liabilities ¥1,095 Short term debt 717 Payables 525 Accrued Expenses 962 Current portion of long term debt 269 Other 3,568 Total current liabilities Long term liabilities Long term debt 2,043 Other 1,377 Total long term liabilities 3,420 Stockholder’s equity Common stock 86 Additional paid in capital 173 Retained earnings 5,667 Other (1,476) Total Honda Motor Company shareholder’s equity 4,450 Noncontrolling interests 133 Total stockholder’s equity 4,583 Total liabilities and stockholder’s equity ¥11,571 Exhibit 17 11 Honda Motor Company, Ltd Balance Sheet March 31, 2011 (in billions) 1. Prepare a common size income statement, that is, one show component percentages. 2. Compute the following ratios: a. Current ratio b. Total debt to equity c. Gross profit rate d. Return on stockholder’s equity (March 31, 2010 stockholder’s equity was ¥4,456 billion). e. P/E ratio (the market price on March 31, 2011, was approximately ¥2,207 per share) f. Dividend payout ratio 3. What additional information would help you interpret the percentages and ratios you calculated? Answer all sections, noting the following: ? For section 1, prepare a spreadsheet solution which complies with the assignment requirements. Then incorporate an appropriate graph style in your spreadsheet to show the component parts (and their relative size) of total assets, total liabilities, and total stockholder’s equity. The purpose of this part of the question is for you to demonstrate graphing/charting skills using Excel. ? For section 2 use a spreadsheet which is linked to the data in your section 1 spreadsheet. Question 7 Prepare a business report (as if for senior management) on two of the following. Use the Internet as a resource. Length about 300 400 words each not counting appendices. Use correct referencing and include a bibliography. Relate your report to the function of accounting. Refer to the earlier ‘Assessment Information’ section for essay/report writing resources. The Internet can also be search for advice on business report structures. a. Digital Dashboards. Include in your report definitions, types, history, benefits, disadvantages and screenshots of examples. b. Bounded rationality in decision making. c. ‘Predictably irrational’ Dan Ariely d. The Checklist Manifesto Gawande??e. Accounting and Impression Management? Rationale This assignment allows students to demonstrate their understanding of Topics 2, 3, and 4, and seeks to assess your ability to be able to construct and analyse accounting reports, research accounting issues, demonstrate spreadsheet skills and apply critical thinking in communicating results. In doing this it is aligned with the following subject learning outcomes: ? To be able to discuss and apply basic accounting ideas that underlie and limit the usefulness of accounting information; ? To be able to explain and analyse the nature, and measurement, of assets, equities, revenues, expenses and income; ? To be able to apply computer software to quantitative business decisions; ? To be able to implement and critique analytic and interpretative techniques used in accounting; and ? To be able to explain and implement financial analysis and management accounting procedures. Marking criteria Your assignment will be marked as a whole with recognition being given to demonstration of knowledge, quality thinking, application of skills and critical thinking. For some questions students have a chance to demonstrate some creativity, or provide their own examples or give evidence of wider reading or to structure a spreadsheet in an innovative manner. We also need to avoid the snowballing effects of cumulative errors that is, an error early in a practical question which carries through. Thus a grading system is used with deductions for errors and omissions. Full marks are a possibility for well structured, complete, quality answers. Students will be provided with a grade based on the work submitted and will also be provided with suggested solutions or guidelines. Feedback to students will normally consist of the grade, comments and the suggested solutions. Penalties will be applied to poor presentation including poor grammar, punctuation and spelling, and for poor use of software. Criteria High Distinction Distinction Credit Pass Fail In respect to topics 2, 3, and 4 of this subject, students will be required to undertake theory based questions including discussion questions, and problem based questions utilising spreadsheet skills, to demonstrate their ability to be able to: · discuss and apply basic accounting ideas that underlie and limit the usefulness of accounting information; · explain and analyse the nature, and measurement, of assets, equities, revenues, expenses and income; · be able to apply computer software to quantitative business decisions; · implement and critique analytic and interpretative techniques used in accounting and finance; · be able to explain and implement financial analysis and management accounting procedures. To meet this level you will attain a cumulative mark between 85% 100%. A mark in this range (no less than 29.6 marks) indicates you have demonstrated an exceptional and a consistently high level of knowledge and understanding in topics 2, 3, and 4 of this subject. This mark also indicates your assignment will: · be professionally presented with correct spelling and grammar · be the result of your original work · include complete references and acknowledgment all sources used · be complete without any omissions · be correct · discuss comprehensively the issues raised in questions · provide examples where appropriate, and · comply with all spreadsheet requirements. To meet this level you will attain a cumulative mark between 75% 84%. A mark in this range (26.1 – 29.5) indicates you have demonstrated a comprehensive and high level of knowledge and understanding in topics 2, 3, and 4 of this subject. This mark also indicates your assignment will: · be neat and legibly presented with very few spelling and grammatical errors · be the result of your original work · include references, acknowledging all sources used, · be mostly complete without any omissions · be mostly correct · discuss many of the issues raised in questions · provide examples where appropriate, and · comply with most spreadsheet requirements. To meet this level you will attain a cumulative mark between 65% 74%. A mark in this range (22.6 – 26) indicates you have demonstrated a sound knowledge and understanding in topics 2, 3, and 4 of this subject. This mark also indicates your assignment will: · be neat and legibly presented with few spelling and grammatical errors · be the result of your original work · include references, acknowledging most sources used, · be mostly complete without any omissions · be mostly correct with only a few errors · discuss many of the issues raised in questions · provide some examples where appropriate, and · comply with most spreadsheet requirements. To meet this level you will attain a cumulative mark between 50% 64%. A mark in this range (17.5 – 22.5) indicates you have demonstrated a basic knowledge and understanding in topics 2, 3, and 4 of this subject. This mark also indicates your assignment will: · be legibly presented · be the result of your original work · include some references and acknowledge most sources used · be mostly complete without too many omissions · be mostly correct with a few errors · discuss some of the issues raised in questions · provide some examples where appropriate, and · comply with most spreadsheet requirements. At this level you will attain a cumulative mark between 0% 49%. A mark in this range (less than 17.5) indicates you will not have demonstrated a basic knowledge and understanding in topics 2, 3, and 4 of this subject. This mark also indicates your assignment will: · lack some legibility · have multiple spelling and grammatical errors · present a few examples of non original work · include inadequate references · have significant missing work · have many errors · lack discussion of many of the issues raised in questions · lack examples where required, and · have significant flaws in spreadsheet presentation. Presentation Solutions to questions should vary in length according to the nature of the question and be in a format similar to that demonstrated in the text and study guide. Choose your own examples in discussion questions, and ensure you adhere to the spreadsheet requirements (refer below to the Requirements section) carefully. Your turnitin submission will consist of two files a Word doc (or pdf) and an Excel file. Submit one after the other. Paste the spreadsheet solutions and a formula view of each spreadsheet in the Word doc displaying row and column headings. Thus your Word doc will provide a complete answer to every question. The separate Excel file will provide a clear demonstration of correct spreadsheet structures. The formula view may need to have adjusted column widths to display formulas correctly. Use portrait orientation wherever possible. Name the two files with your family name plus assignment number (eg WilsonAss1.docx and WilsonAss1.xlsx) as the file name.? Requirements The work you submit must be your own. In the past a few students have received heavy penalties for submitting assignments which were not entirely their own work. 1. Submit through turnitin. 2. The first page of your assignment should consist of the following parts: a. subject code and name;? b. your name and student ID number;? c. list of questions attempted and a list of questions not attempted;? d. dates and details of any extension granted;? e. an academic integrity statement that the work submitted is your own and that all sources used have been acknowledged. The following statement can be used: THE WORK IN THIS ASSIGNMENT IS MY OWN WORK, AND HAS NOT BEEN PLAGIARISED. 3. Record your name, student number and page number as a footer on every page of your assignment. 4. Present assignment solutions in sequence. Provide references throughout (question by question if appropriate) and include a bibliography of all sources used including internet resources, at the end of your assignment. If using internet resources, include the URL and date of access. 5. Utilise the online forum to clarify any assignment format issues. Single spacing and small margins are preferred. 6. Keep backup copies of all your work. Make copies of your work progressively in case your computer has an accident. Remember that assignments will NOT be returned to students but will be retained by staff to facilitate discussion between students and staff whereby each can look at the same work; further, the retention of assignments by staff may assist in marginal grade decisions and in resolving any potential plagiarism. 7. Please check for viruses. Do not use macros in your spreadsheet files. 8. Students can discuss their work with each other on the forum prior to submission. However, please do not post complete versions of answers to the forum. Do not leave assignments until the last minute. 9. Plagiarism. Refer to the section in this Subject Outline and to the University website. Ensure that you include the academic integrity statement at the beginning of your assignment. 10. Penalties will apply for late submission at 10% of the total mark available per day late. 11. It is important that submitted assignments are complete; missing answers will result in mark deductions. 12. Refer to the University regulations contained in the handbook for matters pertaining to special consideration, misadventure and extenuating circumstances, academic misconduct, code of conduct for users of electronic facilities, assessment regulation, grading scale, progression and academic performance, grade point average, examination regulations and other matters pertaining to assessment. The regulations can be found at the University’s web site: http://www.csu.edu.au. Spreadsheet requirements 1. Your turnitin submission will consist of two files a Word doc (or pdf) and an Excel file. Submit one after the other. Paste the spreadsheet solutions and a formula view of each spreadsheet in the Word doc displaying row and column headings. Thus your Word doc will provide a complete answer to every question. The separate Excel file will provide a clear demonstration of correct spreadsheet structures. The formula view may need to have adjusted column widths to display formulas correctly. Use portrait orientation wherever possible. Name the two files with your family name plus assignment number (eg WilsonAss1.docx and WilsonAss1.xlsx) as the file name. 2. For good spreadsheet design it is very important that you have completely separate data entry and solution/report areas. A good spreadsheet solution format is to key in the question in a structure which allows the solution to be completely formula driven. There should be no data in the report/solution area. 3. Create all spreadsheets as separate spreadsheets in one workbook. Name the tags at the bottom of the computer screen with the question number. See examples in the spreadsheet advice section in Interact Resources. 4. Where appropriate, use the IF function to provide built in checks of balances, net income/net loss, favourable/unfavourable variances etc.etc. 5. How can you display negative numbers in brackets? Google the question! Hint. Format / Cells / Number and select Custom. Enter this: #,##0;(#,##0);0. 6. Save your workbook with the cursor in cell A1 of the first spreadsheet. 7. We have provided spreadsheet examples to guide you in the Spreadsheet Advice document in Interact it is important that you read these examples. Also consider using the spreadsheet templates available with your text and other online spreadsheet advice.

Attachments:

rabid roberts is the manager of a firm quiktax which specialises in the preparation 678468

Rabid Roberts is the manager of a firm, QuikTax, which specialises in the preparation of income tax returns. The firm offers two basic products: the preparation of income tax returns for wage and salary earners and the preparation of income tax returns for small businesses. Any clients requiring more complex services are referred to Rabid’s brother Roger, who is a partner in a large firm of chartered accountants. The processing of wage and salary tax returns is quite straightforward, and the firm uses a software package to process data and print the return. A software package is also used to prepare returns for small businesses, although more information is required, particularly about business expenses. Rabid has only recently joined QuikTax and he is concerned about the firm’s pricing policy, which sets a flat fee of $40 per return for wage and salary clients and $200 for small businesses. He decides to use activity based costing to estimate the costs of providing each of these services. At the end of the year, Rabid reviewed the company’s total costs and activities, resulting in the following list:

Activity Activity cost Interview salaried client S 40 000 Interview business client 50 000 Obtain missing data 400 000 Input data 80 000 Print return 60 000 Verify return with client 120 000 Rectify errors 60 000 Submit return 20 000 Total costs S830 000

Activity driver No. of wage and salary clients No. of business clients No. of follow up calls No. of data entries No. of returns No. of hours No. of errors No. of returns

Quantity of activity driver 8 000 2 000 8 000 400 000 10000 6 000 6 000 10 000

In identifying the activities required for each type of return, Rabid noted the following: (a) Clients are interviewed only once per return. (b) All follow up calls to obtain missing data relate to business returns; on average, each business tax return requires four follow up calls. (c) Processing a wage and salary tax return requires 20 data entries, whereas a business return requires 120 data entries. (d) On average, it takes 22.5 minutes to verify a wage and salary tax return, whereas it takes one and a half hours to verify a business return. (e) All errors relate to business returns; on average, there are 3 errors per business return. Required: 1 Use activity based costing to estimate the cost of preparing: (a) a wage and salary tax return (b) a business tax return. 2 In the light of your answers to requirement 1, evaluate the firm’s pricing policy. Peas Activity based costing; analysis of operations: service firm Clark and sniffer perform consulting services related to e commerce consulting and information I.1061 systems in Perth. The firm, which bills 5125 per hour for services performed, is in a very b ytigyht lo cal labour market and is having difficulty finding quality staff. The labour cost per hour paid and Shifter for professional staff time is 545. Selected information follows: Clark (a) Billable hours to clients for the year totalled 5000, consisting of: information systems services, 3100; e commerce consulting. 1900. (b) Administrative cost of 5342000 was (and continues to be) allocated to both consulting services based on billable hours. These costs consist of staff support, $180000; in house computing, $136400; and miscellaneous office charges. $25600.

Attachments:

the fast food industry has been dramatically altered through it enablement 678497

Case study:

The fast food industry has been dramatically altered through IT enablement. However, IT enablement has not completely eliminated manual processes in fast food franchise restaurants such as McDonald’s, Wendy’s, and Burger King.

Required:

Using your experience in visiting fast food restaurants, answer the two questions that follow:

a. List and describe four different activities that are manual parts of business processes at any of these restaurants

b. List and describe four different activities that are IT enabled parts of business processes at these restaurants

Turner, Leslie (2013 01 01). Accounting Information Systems: The Processes and Controls, 2nd Edition (Page 31).Wiley Higher Ed. Kindle Edition.

Note: Your grade will be based on the quality of both content and presentation, considering the following:

  • Discussion of all required points related to the question.
  • Ability to apply concepts from this course in order to design a solution.
  • Ability to conduct the appropriate analysis based upon the limited information that has been provided.
  • The creativeness of your solution.
  • Formatting considerations, such as
  • Well written and free of grammatical and typographical errors Professional, neat appearance
Document Preview:

Case study: The fast food industry has been dramatically altered through IT enablement. However, IT enablement has not completely eliminated manual processes in fast food franchise restaurants such as McDonald’s, Wendy’s, and Burger King. Required: Using your experience in visiting fast food restaurants, answer the two questions that follow: a. List and describe four different activities that are manual parts of business processes at any of these restaurants b. List and describe four different activities that are IT enabled parts of business processes at these restaurants Turner, Leslie (2013 01 01). Accounting Information Systems: The Processes and Controls, 2nd Edition (Page 31).Wiley Higher Ed. Kindle Edition. Note: Your grade will be based on the quality of both content and presentation, considering the following: Discussion of all required points related to the question. Ability to apply concepts from this course in order to design a solution. Ability to conduct the appropriate analysis based upon the limited information that has been provided. The creativeness of your solution. Formatting considerations, such as Well written and free of grammatical and typographical errors Professional, neat appearance

Attachments:

copy and 10 00 points southworth company uses a job order costing system and applies 678498

Copy and 10.00 points

Southworth Company uses a job order costing system and applies manufacturing overhead cost to jobs on the basis of the cost of direct materials used in production. Its predetermined overhead rate was based on a cost formula that estimated $248,000 of manufacturing overhead for an estimated allocation base of $155,000 direct material dollars.

The following transactions took place during the year (all purchases and services were acquired on account):

a. Raw materials purchased, $142,000.
b. Raw materials requisitioned for use in production (all direct materials), $150,000.
c. Utility bills incurred in the factory, $21,000.
d. Costs for salaries and wages were incurred as follows:

Direct labor $ 216,000
Indirect labor $ 90,000
Selling and administrative salaries $ 145,000

e. Maintenance costs incurred in the factory, $15,000.
f. Advertising costs incurred, $130,000.
g.

Depreciation recorded for the year, $50,000 (90% relates to factory assets, and the remainder relates to selling and administrative assets).

h.

Rental cost incurred on buildings, $90,000 (80% of the space is occupied by the factory, and 20% is occupied by sales and administration).

i. Miscellaneous selling and administrative costs incurred, $17,000.
j. Manufacturing overhead cost was applied to jobs, $ ? .
k.

Cost of goods manufactured for the year, $590,000.

l.

Sales for the year (all on account) totaled $1,000,000. These goods cost $600,000 according to their job cost sheets.

The balances in the inventory accounts at the beginning of the year were as follows:

Raw materials $ 18,000
Work in process $ 24,000
Finished Goods $ 35,000

Required:
1. Prepare journal entries to record the above data.

General Journal Debit Credit
a.
b.
c.
d.
e.
f.
g.
h.
i.
j.
k.
l

2.

Post your entries to T accounts. (Don’t forget to enter the opening inventory balances above.) Determine the ending balances in the inventory accounts and in the Manufacturing Overhead account. (Record the transactions in the given order.)

Accounts Receivable


Raw Materials


Bal.


Bal.


Work in Process


Bal.


Bal.


Finished Goods


Bal.


Bal.


Manufacturing Overhead




Bal.


Accounts Payable




Bal.


Accumulated Depreciation


Depreciation Expense


Salaries & Wages Payable


Selling and Administrative Salaries


Miscellaneous Expense


Advertising Expense


Rent Expense


Cost of Goods Sold


Sales



3.

Prepare a schedule of cost of goods manufactured. (Input all amounts as positive values.)

Southworth Company
Schedule of Cost of Goods Manufactured
Direct materials:
$
:

:

Materials used in production $

Total manufacturing cost
:

:

Cost of goods manufactured $



4.

Prepare a journal entry to close any balance in the Manufacturing Overhead account to Cost of Goods Sold. Prepare a schedule of cost of goods sold. (Input all amounts as positive values.)

General Journal Debit Credit

Schedule of cost of goods sold:
$
:

:

:

Adjusted cost of goods sold $



5.

Prepare an income statement for the year. (Input all amounts as positive values.)

Southworth Company
Income Statement
$

Selling and administrative expenses:
$


$



6.

Job 218 was one of the many jobs started and completed during the year. The job required $3,600 in direct materials and 400 hours of direct labor time at a rate of $11 per hour. If the job contained 500 units and the company billed at 75% above the unit product cost on the job cost sheet, what price per unit would have been charged to the customer? (Round your answer to 2 decimal places.)

Price charged for Job 218 $ per unit

paste your question here…

three years ago carrie dungy and her brother in law luke barber opened fedco departm 678501

Three years ago, Carrie Dungy and her brother in law Luke Barber opened FedCo Department Store. For the first two years, business was good, but the following condensed income results for 2011 were disappointing.

FEDCO DEPARTMENT STORE

Income Statement

For the Year Ended December 31, 2011

Net sales

$700,000

Cost of goods sold

553,000

Gross profit

147,000

Operating expenses

Selling expenses

$100,000

Administrative expenses

20,000

120,000

Net income

$ 27,000

Carrie believes the problem lies in the relatively low gross profit rate (gross profit divided by net sales) of 21%. Luke believes the problem is that operating expenses are too high. Carrie thinks the gross profit rate can be improved by making both of the following changes. She does not anticipate that these changes will have any effect on operating expenses.

1. Increase average selling prices by 17%. This increase is expected to lower sales volume so that total sales will increase only 6%.

2. Buy merchandise in larger quantities and take all purchase discounts. These changes are expected to increase the gross profit rate by 3 percentage points.

Luke thinks expenses can be cut by making both of the following changes. He feels that these changes will not have any effect on net sales.

1. Cut 2011 sales salaries of $60,000 in half and give sales personnel a commission of 2% of net sales.

2. Reduce store deliveries to one day per week rather than twice a week; this change will reduce 2011 delivery expenses of $30,000 by 40%.

Carrie and Luke come to you for help in deciding the best way to improve net income.

Instructions

With the class divided into groups, answer the following.

(a) Prepare a condensed income statement for 2012, assuming (1) Carrie’s changes are implemented and (2) Luke’s ideas are adopted.

(b) What is your recommendation to Carrie and Luke?

(c) Prepare a condensed income statement for 2012, assuming both sets of proposed changes are made.

the haines corp shows the following financial data for 2012 and 2013 2012 2013 sales 678537

The Haines Corp. shows the following financial data for 2012 and 2013.

2012 2013
Sales $ 3,160,000 $ 3,860,000
Cost of goods sold 2,360,000 2,570,000




Gross profit $ 800,000 $ 1,290,000
Selling & administrative expense 287,000 223,000




Operating profit $ 513,000 $ 1,067,000
Interest expense 41,200 52,900




Income before taxes $ 471,800 $ 1,014,100
Taxes (35%) 165,130 354,935




Income after taxes $ 306,670 $ 659,165









For each year, compute the following ratios and indicate how the change in each ratio will affect profitability in 2013. (Input your answers as a percent rounded to 2 decimal places.)

2012 2013 Profitability
a. Cost of goods sold to sales % %
b. Selling and administrative expense to sales % %
c. Interest expense to sales % %

this is the assignment that is due september 4th at midnight 675670

This is the assignment that is due September 4
th at midnight

Case Study Presentation

Objectives:

  • 1.1
  • 1.2
  • 1.3
  • more
  • Instructions
  • Assignment Files
  • Grading

Resources:Case Study Learning Team Assignments from Weeks Two, Three, or Four

Preparea 7 to 9 slide Microsoft
®PowerPoint
®presentation illustrating your responses to the questions posed by the assigned case study.

Listmajor points in the slides. Include detailed explanations in the
speaker notes section that correlate to each point.

Citeyour sources and format your text consistent with APA guidelines.

Clickthe Assignment Files tab to submit your assignment.

Below is the list of topics for each week. I will attach copies of the assignments separately.

Week1. Fundamentals of Managerial and Cost Accounting. Aug 05 Aug 11

Week1

Fundamentals of Managerial and Cost Accounting

Aug 05 Aug 11
4 / 14 points

Tasks

Review the Week One Student Guide.

Objectives/Competencies

    • 1.1Identify key components and terminology utilized within cost accounting.
    • 1.2Compare and contrast process and job cost systems.
    • 1.3Examine ethical issues in managerial accounting.
    • 1.4Differentiate between direct and indirect costs.

Learning Activities

Week2

Cost Allocation

Aug 12 Aug 18
15 / 18 points

Tasks

Review the Week Two Student Guide.

Objectives/Competencies

    • 2.1Assess the advantages and disadvantages of an activity based costing (ABC) system.
    • 2.2Identify cost drivers for manufacturing and service activities.
    • 2.3Calculate overhead rates using cost drivers.
    • 2.4Compute allocation of overhead costs to cost pools, end products, and services.

Week3

Cost Volume Profit (CVP)

Aug 19 Aug 25
11.03 / 12 points

Tasks

Review the Week Three Student Guide.

Objectives/Competencies

    • 3.1Distinguish among fixed, mixed, and variable costs.
    • 3.2Calculate a break even point using the elements of cost volume profit (CVP).
    • 3.3Prepare income statements using both variable and absorption costing.

Week4

Variance Analysis

Aug 26 Sep 01

/ 18 points

Tasks

Review the Week Four Student Guide.

Objectives/Competencies

    • 4.1Explain the advantages and disadvantages of using standard costing.
    • 4.2Calculate price, usage, and overhead variances.
    • 4.3Determine the causes of overapplied or underapplied overhead variances.

Learning Activities

Week5

Pricing Decisions and Budgeting

Sep 02 Sep 08

/ 38 points

Tasks

Review the Week Five Student Guide.

Objectives/Competencies

    • 5.1List the components of an operating and financial budget.
    • 5.2Determine pricing for external and internal sales.

introduction to management accounting assignment 675686

Copy and paste your question here…

You are required to complete all questions and submit all necessary workings. Follow the assignment requirements all the requirements have been attached in the file (refer below to the Requirements section) very closely.

Document Preview:

Assignment 2 Value: 35% Return date: 05 Oct 2014 Submission method options Alternative submission method Task Please note this assignment consists of questions from Horngren, C.T., Sundem, G.L., Stratton, W.O. Burgstahler, D., & Shatzberg, J. Introduction to management accounting, (16th edition 2014). You are required to complete all questions and submit all necessary workings. Follow the assignment requirements (refer below to the Requirements section) very closely. Content Assessed?Topics 2, 3, and 4?? Question 1 16 2, 16 6, 16 33 (illustrate your explanation with a numerical example).?Source: Horngren, et al. (2014). Introduction to management accounting (16th ed., p. 719). Pearson. 16 2.   Why should short term prepaid expenses be classified as current assets? 16 6.   What factors influence the estimate of useful life in depreciation accounting? 16 33. Study Appendix 16A. Suppose prices are rising and inventories are increasing. Which of the four generally accepted inventory accounting methods will usually result in the highest net income? Explain. Illustrate your explanation with a numerical example.       Question 2 16 A1 Reigle Balance Sheet and Income Statement?Source: Horngren, et al. (2014). Introduction to management accounting (16th ed., p. 716). Pearson. The Reigle Company had the following items on its December 31, 20X0 balance sheet and 20X0 income statement (in dollars except for the number of shares outstanding):  Cash and equivalents?$   60,000??Revenues?$ 790,000??Notes Payable?$   51,000??Long term debt, excluding current portion?$ 210,000??Accounts Receivable, net?$ 48,000??Provision for income taxes?$ 60,000??Other long term assets?$ 110,000??Interest expense?$ 55,000??Deferred income tax liability – long term?$ 44,000??Retained earnings?$ 204,000??Income taxes payable?$ 37,000??Cost of sales?$ 470,000??Inventories?$ 36,000??Prepaid expenses?$ 15,000??Common stock (50,000 shares)?$ 25,000??Property, Plant and equipment, at cost?$…

Attachments:

hickory company manufactures two products 675731

Available with McGraw Hill’s Connect. Accounting.

Hickory Company manufactures two products 14,000 units of Product Y and 6,000 units of Product Z. The company uses a plantwidc overhead rate based on direct labor hours. It is considering implementing an activity based costing (ABC) system that allocates all of its manufacturing overhead to four cost pools. The following additional information is available for the company as a whole and for Products Y and 1:

Estimated Activity Cost Pool Activity Measure Overhead Cost Expected Activity Machining Machine hours 5200,000 10,000 MHs Machine setups Number of setups 5100,000 200 setups Production design Number of products S84.000 2 products General factory Direct labor hours $300,000 12,000 DLHs

Activity Measure Machining Number of setups Number of products Direct labor hours

ProductY Product Z

7,000 3,000 50 150 1 1 8,000 4,000

1. What is the company’s plantwide overhead rate? 2. Using the plantwide overhead rate, how much manufacturing overhead cost is allocated to Product Y? How much is allocated to Product Z?

the following data were taken from the records of clarkson company for the fiscal ye 675785

The following data were taken from the records of Clarkson Company for the fiscal year ended June 30,2014.

Raw Materials

Inventory 7/1/13 $48,000

Factory Insurance $4,600

Raw Materials

Inventory 6/30/2014 $39,600

Factory Machinery

Depreciation $16,000

Finished Goods

Inventory 7/1/13 $96,000

Factory Utilities $27,600

Finished Goods

Inventory 6/30/14 $75,900

Office Utilities Expense $8,650

Work in Progress

Inventory 7/1/13 $19,800

Sales Revenue $534,000

Work In Progress

Inventory 6/30/14 $18,600

Sales Discounts $4,200
Direct Labor $139,250 Plant Managers Salary $58,000
Indirect Labor $24,460 Factory Property Taxes $9,600
Accounts Receivable $27,000 Factory Repairs $1,400
  Raw Materials Purchases $96,400
  Cash $32,000
   

A) Prepare a cost of goods manufactured schedule (Assume all raw materials used were direct materials).

B) Prepare an income statement through gross profit

C)Prepare the current assets section of the balance sheet at June 30, 2014

stake holders and ethics grocery market in australia 675795

STAKEHOLDER VALUE AND ETHICS

TERM 2 2014 ASSESSMENT ONE DETAILS

Individual Assignment (Weight 10%)

Woolworths and Coles dominate the food grocery market in Australia. Both these organisations have come in for what might be described as their share of criticism. A number of practices have been described as predatory in the past by the ACCC, however the previous chairman, Graeme Samuel, defended both these organisations from the onslaught of negative publicity from commentators and consumer groups alike.

Required:

Choose one of the organisations and examine their commercial practices in light of ethical behavioural theory you have learnt in this session so far.

Write a paper of no more than three typed pages* addressing the following task:

1. Examine the role of the ACCC is this issue and whether is appropriate for it to adopt the role have in this debate. Justify your position (20% of the available mark).

2. Make recommendations for the organisation of your choice (Woolworths or Coles) in terms of an ethics policy based on your findings. You will need to investigate subsequent practices to adequately address this question (30% of the available mark).

3. In your response, you need to show evidence of appropriate research into the organisation’s practices and wider reading of corporate ethical behaviour. Your focus will be on the first 3 weeks of this course, though higher level responses will demonstrate critical evaluation incorporating broader concepts (40% of the available mark)

4. Harvard Referencing must be used (10% of the available mark).

Due: Week 5 at the commencement of you allocated class.

Post your response on Moodle at the indicated link.

* 2.5cm margins, left, right, top and bottom

12pt Arial or Times Roman Font

1.5 line spacing

HINT SOME KEY CONCEPTS TO EXPLORE

The following areas for discussion are provided to assist you in developing your response to this task.

The list is not intended to be exhaustive nor do you need to include all areas. These are possible areas/concepts to explore as they may relate to your chosen organisation.

? Price Gouging

? Predatory Pricing

? Geographic Price Discrimination

? Undue Influence on Suppliers

? Uncompetitive Behaviour

? Discounted Petrol Effect on Competition (Recent Development)

o (also discuss recent developments and changes effective as at 1 January 2014)

? Cross Promotion of Liquor and Fuel

? Loss Leading Pricing

? Obligation to stakeholders (who are they?)

part 1 case studyjanet taxpayer residing in australia is named as the sole beneficia 675796

Part 1: Case studyJanet (taxpayer) residing in Australia is named as the sole beneficiary of aproperty (1.85 hectares) with a large homestead as a result of the deathof a relative on 7/10/2010. The property is not used for commercialpurposes and at the date of death, the property was valued at$1.45million. Settlement took place on 21/12/2010. After moving into thehomestead shortly after taking ownership, she planned to take a one yeartrip which she had been planning for some time in late 2011. The taxpayerfelt that the homestead was far too large for her (she is single), applied tothe ATO for an exemption for ABN registration and some fourteen monthslater (16/2/2012), she obtained council approval to subdivide the propertyinto three, with the intention of building three units, one she will take up asher own residence, the other two will be sold. Work commenced someweeks after approval and on 12th December that same year, thetaxpayer returned and moved into one of the apartments. The other twowere sold in March/April in 2013, one selling for $1.35m (24/3/2013), theother for $1.45m (9/4/2013).You are to consider the CGT implications both from the relevant sections(ITAA), rulings, etc. and from the values (if/where applicable). Assume thatthe blocks are subdivided equally. For each determination that you make,you should clarify. You should also clarify what Capital Gains and CGT is inyour answer (15 marks)Part 2: QuestionExplain using examples and relevant sections of the act, what thedifferences between Ordinary Income and Statutory income are. Use yourown examples (not from MTG or Barkoczy text) (5 marks)

part 1 case studyjanet taxpayer residing in australia is named as the sole beneficia 675797

Part 1: Case studyJanet (taxpayer) residing in Australia is named as the sole beneficiary of aproperty (1.85 hectares) with a large homestead as a result of the deathof a relative on 7/10/2010. The property is not used for commercialpurposes and at the date of death, the property was valued at$1.45million. Settlement took place on 21/12/2010. After moving into thehomestead shortly after taking ownership, she planned to take a one yeartrip which she had been planning for some time in late 2011. The taxpayerfelt that the homestead was far too large for her (she is single), applied tothe ATO for an exemption for ABN registration and some fourteen monthslater (16/2/2012), she obtained council approval to subdivide the propertyinto three, with the intention of building three units, one she will take up asher own residence, the other two will be sold. Work commenced someweeks after approval and on 12th December that same year, thetaxpayer returned and moved into one of the apartments. The other twowere sold in March/April in 2013, one selling for $1.35m (24/3/2013), theother for $1.45m (9/4/2013).You are to consider the CGT implications both from the relevant sections(ITAA), rulings, etc. and from the values (if/where applicable). Assume thatthe blocks are subdivided equally. For each determination that you make,you should clarify. You should also clarify what Capital Gains and CGT is inyour answer (15 marks)Part 2: QuestionExplain using examples and relevant sections of the act, what thedifferences between Ordinary Income and Statutory income are. Use yourown examples (not from MTG or Barkoczy text) (5 marks)

create the accounts to demonstrate the ability to add delete modify accounts until c 675868

Tasks Your group are establishing a small retail shop to sale different products to the customers. The products that you sell are similar with those found in Minimart, 7 Eleven store. In the operation of your business, you will need to record data in the accounting software to reflect different accounting operation. These operations are creating the account, purchasing products from suppliers, selling products to customers and producing financial reports. Your group has chosen to use the current MYOB (version 19) to assist you to record the data about your company, account, inventory, purchase, sales and to producing the financial reports. In order to accomplish these operations, your groups will proceed with the following tasks with MYOB. Task 1 (5 marks) Create the accounts to demonstrate the ability to add, delete, modify accounts until checking balance. Task 2 (5 marks) Record the purchase process (buying items from the suppliers) (3 marks) and produce two financial reports of purchasing activity (2 marks). Task 3 (5 marks) Record the sale process (selling items to the customers) (3 marks) and produce two financial reports of sale activity (2 marks). Task 4 Presentation (10 marks) 4.1 Explain the processes of modifying account (2 marks). 4.2 Explain the purchase and sale process (5 marks). 4.3 Suggest three improvements of MYOB that will make the current accounting operation more efficient and effective (3 marks, 1 marks for each improvement).

Document Preview:

BAC11 IT for Accountants, Semester 2/2014 Assignment 1 (25%) This assignment aim is to demonstrate your understanding for the implementation of modifying accounts, checking balances, creating purchase and sales, producing financial report and to the ability to suggest the improvements of the current accounting software. The assignment requires the students to work in a group of 3 4 students. Due Date & Submission Requirement The assignment tasks consist of presenting and submitting the MYOB works. The due date for presenting the MYOB work (task 4 below) is during the normal lecture hrs in week9 (the week beginning of Monday, September 15th, 2014). Each group will be presenting the work only to the lecturer for the length of 10 15 minutes. Please note that the presentation marks will only be given to the students who present during the presentation. Any student who does not present will obtain no presentation mark. After the presentation, the students may take comments from the lecturer to further refine the MYOB works and submit task1, task2 and task3. The due date for submitting the MYOB files is by Friday, September 19th, 2014 at 17.00. Please note that late assignment will be deducted by 10% per day (counting from the due submission day/time). If you submit on the following Monday, this will be counted as 3 days late and the assignment will not be marked if submitted for more than 5 days late. Only one student will be required to submit the group assignment through Moodle. The students will need to submit the zip file containing student ID and name of every group member in MYOB zip files (separate each person with _ like 1103456Sara_1109876Manish). Without providing information of your student id and name, your assignment will not be marked. 1Tasks Your group are establishing the small retail shop to sale different products to the customers. The products that you sell are similar with those found in Minimart,…

Attachments:

develops an opening paragraph that introduces the case and for the most part fulfill 675903

Brief 2 Rubric

Total 50 points

Criteria Unacceptable Acceptable Proficient Student Score and Comments

Accurately completes the problem worksheet.

Does not accurately complete the problem worksheet.

(0 3 points) For the most part, accurately completes the problem worksheet and fulfills assignment requirements.

(4 points) Accurately completes the problem worksheet and demonstrates solid ability to accomplish the assignment.

(5 points)

Applies the 5 step critical thinking decision making tool (appendix). Vaguely applies the 5 step critical thinking decision making tool.

(0 3 points) Applies the 5 step critical thinking decision making tool and, for the most part, fulfills assignment requirements.

(4 points) Applies the 5 step critical thinking decision making tool and demonstrates solid ability to accomplish the assignment.

(5 points)

Develops an opening paragraph that introduces the case.

Vaguely develops an opening paragraph that introduces the case or does not have an opening paragraph.

(0 3 points) Develops an opening paragraph that introduces the case and, for the most part, fulfills assignment requirements.

(4 points) Develops an opening paragraph that introduces the case and demonstrates solid ability to accomplish the assignment.

(5 points)

Identifies the relevant costs that impact the volume production decisions(s) at the two locations.

Vaguely identifies the relevant costs that impact the volume production decisions(s) at the two locations or completely off the topic.

(0 3 points) Identifies the relevant costs that impact the volume production decisions(s) at the two locations and, for the most part, fulfills assignment requirements.

(4 points) Identifies the relevant costs that impact the volume production decisions(s) at the two locations and demonstrates solid ability to accomplish the assignment.

(5 points)

Discusses the contribution margin impact for each location and why.

Vaguely discusses the contribution margin impact for each location and why or completely off the topic.

(0 3 points) Discusses the contribution margin impact for each location and why and, for the most part, fulfills assignment requirements.

(4 points) Discusses the contribution margin impact for each location and why and demonstrates solid ability to accomplish the assignment.

(5 points)

Recommends the production split that the company should consider for optimizing plant production and total operating income.

Vaguely recommends the production split that the company should consider for optimizing plant production and total operating income or completely off the topic.

(0 3 points) Recommends the production split that the company should consider for optimizing plant production and total operating income and, for the most part, fulfills assignment requirements.

(4 points) Recommends the production split that the company should consider for optimizing plant production and total operating income and demonstrates solid ability to accomplish the assignment.

(5 points)

Integrates established accounting principles into the discussion.

Rarely integrates established accounting principles into the discussion.

(0 3 points) For the most part, does a good job of integrating established accounting principles into the discussion.

(4 points) Consistently does a good job of integrating established accounting principles into the discussion.

(5 points)

Synthesizes relevant information and materials to provide evidence of critical thought.

Synthesizes information at a minimal level.

(0 3 points) For the most part, effectively synthesizes information, which supports main ideas.

(4 points) Consistently and effectively synthesizes information, which provides strong support to main ideas.

(5 points)

Uses supporting documentation that has been properly references and cited.

Inadequate or minimal use of supporting documentation or not properly referenced or cited.

(0 3 points) For the most part, uses supporting documentation that is properly referenced and cited.

(4 points) Consistently uses supporting documentation that is properly referenced and cited.

(5 points)

Considered holistically, demonstrates the ability to write at the graduate level.

Considered holistically, the student demonstrates an inadequate ability to write at the graduate level.

(0 3 points) Considered holistically, the student demonstrates an acceptable ability to write at the graduate level.

(4 points) Considered holistically, the student demonstrates a proficient ability to write at the graduate level.

(5 points)

SubTotal Points

Turnitin.com Did not turn paper into Turnitin.com

Minus 5 points

Revisions based on the originality Report Did not revise paper based on the originality report

Minus 5 points

Grammarly Did not revise paper based on the recommendations from Grammarly.com.

Minus 5 points

Total Points =

are you legally obliged to compensate golden egg for the 3 675922

Commercial Law

Assignment

(BULAW5914)

The Business Faculty

CRICOS Provider No. 00103D

(VIC); 01266K (NSW); 02235J

(SA)

Semester Two 2014

Purpose

This is a significant task that requires forward planning and adequate time for research, reading and reflecting. It comprises 35% of your assessment in this subject. You should begin researching early to gather information and establish a plan as soon as possible.

The purposes of the assignment are to enable you to:

? develop your law reading, research and writing skills;

? enhance your understanding of law as a dynamic process;

? learn how to independently research a particular aspect of the law;

? reflect on and consider particular legal issues;

? demonstrate understanding of the legal environment, including relevant laws as well as economic, ethical, social/cultural, international and political issues;

? develop your knowledge about the subject area of your research;

? demonstrate the ability to investigate, synthesise and analyse;

? communicate the findings in a formal piece of work and meet a deadline;

? enhance your analytical and written communication skills; and

? apply your legal skills.

Your work must comply with the format outlined in the University’s General Guide to the Presentation of Academic Work publication, available online at: http://federation.edu.au/library/assignment and research help/referencing

Assignments must be accompanied by a signed official cover sheet (‘Plagiarism Declaration Form’), available online at http://federation.edu.au/faculties and schools/faculty of business/the business school/current students/student forms and resources

On the cover sheet, please state your name, and student number. Also please include the word count for your assignment, and a signed declaration that you are aware of Federation University’s policies on plagiarism, and that this submission is entirely your own work free of plagiarism and collaboration.

The total length of the assignment must not be more than 2,500 words; do not include the references or bibliography in the word count. Submission of the assignment is due in teaching week 9. Specific due date, time and procedure of submission will be advised by your lecturer. Please note that late penalties apply to assignments where no alternative arrangements have been made with lecturing staff (subject to the discretion of the course coordinator).

Commercial Law

Assignment

(BULAW5914)

The Business Faculty

CRICOS Provider No. 00103D

(VIC); 01266K (NSW); 02235J

(SA)

Assignment Assessment Criteria

You will be assessed on the extent to which you have:

? answered the set question;

? used good English expression, correct spelling, good punctuation and grammar and have proof read your work;

? used good logic and structure;

? been able to identify, set out and discuss relevant legal issues, legal principles and cases, as appropriate;

? used statutes and cases to support your arguments, as appropriate;

? analysed, argued or discussed as required by the task questions; and

? found, read and used other resources.

Task

Read and consider the following five references, the scenario which follows and then answer the set question. Note that while the question relates to the references, good answers will go beyond those references and evidence further reading and research.

References

1. Paul Latimer, Australian Business Law (a 2011 2014 edition), ¶9 770 (fiduciary relationships) and chapter 10 (partnership).

2. Polkinghorne v Holland [1934] HCA 28 (the joint judgment of Rich, Dixon, Evatt and McTiernan JJ) available at: http://www.austlii.edu.au/au/cases/cth/HCA/1934/28l

3. National Commercial Banking Corporation of Australia v Batty [1986] HCA 21 (the leading judgment of Gibbs CJ) available at: http://www.austlii.edu.au/au/cases/cth/HCA/1986/21l

4. Strother v 3464920 Canada Inc 2007 SCC 24, (the majority/leading judgment of Binnie J) available at:

http://scc csc.lexum.com/scc csc/scc csc/en/item/2363/index.do

5. Accounting Professional & Ethical Standards Board 110 Code of Ethics for Professional Accountants, Section 140 (confidentiality) and Section 220 (conflicts of interest) available at:

http://www.apesb.org.au/attachments/Compiled%20APES%20110%20Code%20of%20Ethics%20for%20Professional%20Accountants%20 %20Nov%202013.pdf

Commercial Law

Assignment

(BULAW5914)

The Business Faculty

CRICOS Provider No. 00103D

(VIC); 01266K (NSW); 02235J

(SA)

Scenario

You are an accountant who practiced in partnership with Alex Wolfe under the firm name Y & W Partners. Y & W Partners provided auditing services and taxation advice but did not provide strategic investment advice. Your expertise involved the provision of audit services. Wolfe’s expertise involved taxation advice. One of Wolfe’s clients was Golden Egg Pty Ltd which advises high wealth individuals on their investment strategies. Based on Wolfe’s taxation advice Golden Egg has developed a range of tax effective investment strategies for its high wealth clients.

Several years ago, without your knowledge and without the knowledge of Golden Egg, Wolfe set up a rival business to Golden Egg called Superb Growth Pty Ltd. In setting up Superb Growth, Wolfe used both the confidential taxation advice that he had given to Golden Egg and Golden Egg’s confidential client database that Wolfe had received in the course of giving that advice.

Armed with that advice and the database, Superb Growth competed successfully against Golden Egg in the provision of strategic investment advice to high wealth individuals. Superb Growth won over many of Golden Egg’s best clients. As a direct result of this competition Golden Egg’s fee income decreased substantially.

Having recently discovered Wolfe’s involvement in Superb Growth, Golden Egg confronted him by email. Upon receiving the email Wolfe promptly ceased the operation of Superb Growth, quit the Y & W partnership, converted most of his wealth into diamonds and permanently left Australia with the diamonds. His country of residence is unknown.

The fee income that Golden Egg has lost directly as a result of the competition from Superb Growth is $3 million.

Question

Are you legally obliged to compensate Golden Egg for the $3 million in lost fee income? Confine your answer to the legal obligations that arise in fiduciary and partnership law. (35 marks)

Attachments:

problem 1 4a debra menge started her own consulting firm menge consulting on may 1 2 675936

Problem 1 4A

Debra Menge started her own consulting firm, Menge Consulting, on May 1, 2014. The following transactions occurred during the month of May.

May 1 Debra invested $7,000 cash in the business.
2 Paid $900 for office rent for the month.
3 Purchased $600 of supplies on account.
5 Paid $125 to advertise in the County News.
9 Received $4,000 cash for services performed.
12 Withdrew $1,000 cash for personal use.
15 Performed $5,400 of services on account.
17 Paid $2,500 for employee salaries.
20 Paid for the supplies purchased on account on May 3.
23 Received a cash payment of $4,000 for services performed on account on May 15.
26 Borrowed $5,000 from the bank on a note payable.
29 Purchased equipment for $4,200 on account.
30 Paid $275 for utilities.

in january 2012 the management of sarah company concludes that it has sufficient cas 675938

In January 2012, the management of Sarah Company concludes that it has sufficient cash to purchase some short term investments in debt and stock securities. During the year, the following transactions occurred.

Feb.

1

Purchased 1,200 shares of NJF common stock for $50,600 plus brokerage fees of $1,000.

Mar.

1

Purchased 500 shares of SEK common stock for $18,000 plus brokerage fees of $500.

Apr.

1

Purchased 70 $1,000, 8% CRT bonds for $70,000 plus $1,200 brokerage fees. Interest is payable semiannually on April 1 and October 1.

July

1

Received a cash dividend of $0.80 per share on the NJF common stock.

Aug.

1

Sold 200 shares of NJF common stock at $42 per share less brokerage fees of $350.

Sept.

1

Received $2 per share cash dividend on the SEK common stock.

Oct.

1

Received the semiannual interest on the CRT bonds.

Oct.

1

Sold the CRT bonds for $77,000 less $1,300 brokerage fees.

At December 31, the fair values of the NJF and SEK common stocks were $39 and $30 per share, respectively.

Journalize investment transactions, prepare adjusting entry, and show financial statement presentation.

Instructions

  1. 1. Journalize the transactions and post to the accounts Debt Investments and Stock Investments. (Use the T account form.)
  2. 2. Prepare the adjusting entry at December 31, 2012, to report the investments at fair value. All securities are considered to be trading securities.
  3. 3. Show the balance sheet presentation of investment securities at December 31, 2012.

4. Identify the income statement accounts and give the statement classification of each account.


a job order cost sheet for lowry company is shown below 675953

Exercise 2 3

A job order cost sheet for Lowry Company is shown below.

Job No. 92 For 2,000 Units

Date Direct

Materials Direct

Labor Manufacturing

Overhead

Beg. bal. Jan. 1 6,100 6,500 4,940

8 6,500

12 9,100 7,644

25 2,700

27 5,100 4,284

15,300 20,700 16,868

Cost of completed job:

Direct materials $15,300

Direct labor 20,700

Manufacturing overhead 16,868

Total cost $52,868

Unit cost ($52,868/2,000) $26.43

(a) On the basis of the foregoing data, answer the following questions.

(1) What was the balance in Work in Process Inventory on January 1 if this was the only unfinished job?

Balance in Work in Process Inventory on January 1 $

(2) If manufacturing overhead is applied on the basis of direct labor cost, what overhead rate was used in each year?

Last year Current year

Overhead rate

%

%

(b) Prepare summary entries at January 31 to record the current year’s transactions pertaining to Job No. 92. (Credit account titles are automatically indented when amount is entered. Do not indent manually.)

Date Account Titles and Explanation Debit Credit

Jan. 31

(To record raw materials used)

31

(To record factory labor used)

31

(To record manufacturing overhead)

31

(To record job completed)

Click if you would like to Show Work for this question: Open Show Work

SHOW LIST OF ACCOUNTS

Document Preview:

Exercise 2 3 A job order cost sheet for Lowry Company is shown below. Job No. 92? ? ? ? ? ?For 2,000 Units???Date? ?Direct?Materials? ?Direct?Labor? ?Manufacturing?Overhead??Beg. bal. Jan. 1? ?6,100? ?6,500? ?4,940??8? ?6,500? ?? ? ??12? ?? ?9,100? ?7,644??25? ?2,700? ?? ? ??27? ? ? ?5,100? ?4,284?? ? ?15,300? ?20,700? ?16,868?? Cost of completed job:????   Direct materials??$15,300??   Direct labor??20,700??   Manufacturing overhead??16,868??Total cost??$52,868??Unit cost ($52,868/2,000)??$26.43???(a) On the basis of the foregoing data, answer the following questions.??(1) What was the balance in Work in Process Inventory on January 1 if this was the only unfinished job? Balance in Work in Process Inventory on January 1??$???(2) If manufacturing overhead is applied on the basis of direct labor cost, what overhead rate was used in each year? ??Last year??Current year???Overhead rate???%??%???(b) Prepare summary entries at January 31 to record the current year’s transactions pertaining to Job No. 92. (Credit account titles are automatically indented when amount is entered. Do not indent manually.) Date?Account Titles and Explanation?Debit?Credit??Jan. 31???????????(To record raw materials used)????31???????????(To record factory labor used)????31???????????(To record manufacturing overhead)????31???????????(To record job completed)????Click if you would like to Show Work for this question:?? HYPERLINK “javascript:void();” ?Open Show Work??? ??? ? HYPERLINK “http://edugen.wileyplus.com/edugen/shared/assignment/test/qview.uni?id=quest2089057entrance1&selected_question=quest2089057&operation=take question” o “Show List of Accounts” ?SHOW LIST OF ACCOUNTS???

in an excel spreadsheet configured similarly to the journal shown below prepare jour 675960

In an Excel spreadsheet configured similarly to the journal shown below, prepare journal entries (in journal entry form) for the following transactions. If no entry is required, write “no entry.” Omit explanations.

March 1

Jean Kelly opened a dance school, called Jean’s Dance Studio, by depositing $15,000 into a business bank account.

March 2

Paid three months’ rent in advance, $1,800.

March 4

Hired a part time assistant, to be paid $250 per week, starting next week.

March 6

Purchased sound equipment for $2,000. Paid $400 in cash, the remainder to be paid in installments of $800 every two weeks.

March 8

Signed up five students, who will begin lessons on March 10, at $80 per week per student.

March 17

Received the first week’s tuition from four students; the fifth student will remit payment in three days.

March 17

Paid the assistant his first week’s wages.

March 20

Received payment from the fifth student.

March 21

Paid the first installment on the sound equipment purchased on March 6.

March 23

Received an electric bill of $100, to be paid April 1.

repurchase versus cash dividend trantor corporation is deciding whether to payout 30 677915

Repurchase versus Cash Dividend. Trantor Corporation is deciding whether to payout $300 in excess cash in the form of an extra dividend or a share repurchase. Current earnings are $1.50 per share. and the stock sells for $15. The market value balance sheet before paying out the $300 is as follows:

Market Value Balance Sheet

(before paying out excess cash)

Excess cash

$ 300

Debt

$ 400

Other assets

1,600

Equity

1,500

Total

$1,900

Total

$1,900

Evaluate the two alternatives in terms of the effect on the price per share of the stock, the EPS, and the PE ratio.

the market value of the equity is 1 500 the price per share is 15 so there are 100 s 677916

The market value of the equity is $1,500. The price per share is $15, so there are 100 shares outstanding. The cash dividend would amount to $3001100 = $3 per share. When the stock goes ex dividend, the price will drop by $3 per share to $I2. Put another way, the total assets decrease by 5300, so the equity value goes down by this amount 10 $1,200. With 100 shares, the new stock price is $12 per share. After the dividend, EPS will be the same, S I.50, but the PE ratio will be $12/1.50 ~ 8 times.

With a repurchase, 5300115 = 20 shares will be bought up, leaving 80. The equity will again be worth $1.200 total. With 80 shares, this is $1,200/80 = $15 per share. so the price doesn’t  change. Total earnings for Trantor must be $1.50 X  100 = $150. After the repurchase, EPS will be higher at $150/80 = $1.875. The PE ratio. however, will still be $15/1.875 = 8 times.

dividend reinvestment plans the drk corporation has recently developed a dividend re 677920

Dividend Reinvestment Plans. The DRK Corporation has recently developed a dividend reinvestment plan (DRIP). The plan allows investors to reinvest cash dividends automatically in DRK in exchange for new shares of stock. Over time. investors in DRK will be able to build their holdings by  reinvesting dividends to purchase additional shares of the company.

Over 1,000 companies offer dividend reinvestment plans. Most companies with DRIPs charge no brokerage or service fees. In fact. the shares of DRK will he purchased at a 10 percent discount from the market price. A consultant for DRK estimates that about 75 percent of DRK”s shareholders will take part in this plan. This is somewhat higher than the average. Evaluate DRK”s dividend reinvestment plan. Will it increase shareholder wealth? Discuss the advantages and disadvantages involved here.

stock dividends the owners equity accounts for trans world international are shown h 677923

Stock Dividends. The owners” equity accounts for Trans World International are shown here:

Common stock ($1 par value)

$ 25,000

Capital surplus

170,000

Retained earnings

530,000

Total owners” equity

$725,000

a. If Trans world stock currently sells for $35 per share and a 10 percent stock dividend is declared, how many new shares will be distributed? Show how the equity accounts would change.

b. If Trans World declared a 25 percent stock dividend, how would the accounts change?

regular dividends the balance sheet for impossible odds inc is shown here in market 677926

Regular Dividends. The balance sheet for Impossible Odds, Inc., is shown here in market value terms. There are 35,000 shares of stock outstanding.

Market Value Balance Sheet

Cash

$ 90,000

 

 

Fixed assets

325,000

Equity

$415,000

Total

$415,000

Total

$415,000

The company has declared a dividend of $1.10 per share. The stock goes ex dividend tomorrow. Ignoring any tax effects, what is the stock sel1ing for today? What will it sell for tomorrow? What will the balance sheet look like after the dividends are paid?

stock dividends the market value balance sheet for ewe manufacturing is shown here e 677928

Stock Dividends. The market value balance sheet for Ewe Manufacturing is shown here. Ewe has declared a 20 percent stock dividend. The stock goes ex dividend tomorrow (the chronology for a stock dividend is similar to that for a cash dividend). There are 15,000 shares of stock outstanding. What will the ex dividend price be?

Market Value Balance Sheet

Cash

$130,000

Debt

$110,000

Fixed assets

535,000

Equity

555,000

Total

$665,000

Total

$665.000

you are given the following figures 674745

You are given the following figures:

Current ratio

2.5

Liquidity ratio

1.5

Net working capital

Rs.3,00,000

Fixed assets turnover ratio (on cost of sales)

2 times

Average debt collection period

2 months

Stock turnover ratio (cost of sales/closing stock)

6 times

Gross profit ratio

20%

Fixed assets/shareholders net worth

0.80

Reserve and surplus/capital

0.50

Draw up the balance sheet of the company.

from the following information prepare balance sheet 674746

1. From the following information prepare balance sheet:

 

 

Rs.

Current ratio

2.5 Working capital

60,000

Liquidity ratio

1.5 Reserves and surplus

40,000

Proprietary ratio

 

 

(fixed assets/

 

 

proprietory fund)

0.75 Bank overdraft

10,000

There is no long term loan or fictitious asset.

Syntex Limited?s financial statements contain the following information:

 

31.3.2010

31.3.2011

Cash

2,00,000

1,60,000

Sundry debtors

3,20,000

4,00,000

Temporary investments

2,00,000

3,20,000

Stock

18,40,000

21,60,000

Prepaid expenses

28,000

12,000

Total current assets

25,88,000

30,52,000

Total assets

56,00,000

64,00,000

Current liabilities

6,40,000

8,00,000

10% debentures

16,00,000

16,00,000

Equity share capital

20,00,000

20,00,000

Retained earnings

4,68,000

8,12,000

Statement of Profit for the year ended 31st March, 2011

Sales

40,00,000

Less: Cost of goods sold

      28,00,000

Less: Interest

      1,60,000

Net profit

10,40,000

Less: Taxes @ 50%

      5,20,000

Profit after taxes

5,20,000

Dividends declared on equity shares

2,20,000

From the above, appraise the financial position of the company from the points of view of: (i) liquidity, (ii) solvency, (iii) profitability, and (iv) activity.

the balance sheet of major ltd as on 31st march 2010 is as under 674747

The balance sheet of Major Ltd. as on 31st March, 2010 is as under:

Liabilities

Rs.

Assets

Rs.

Share capital:

 

Fixed assets:

 

2,000 equity shares of

 

At cost

5,00,000

Rs.100 each fully paid

2,00,000

Less: Depreciation

1,60,000 3,40,000

8% preference shares

1,00,000

 

 

General reserve

60,000

Current assets:

 

12% debentures

60,000

Stock

60,000

Current liabilities:

 

Debtors

80,000

Sundry creditors

80,000

Bank

20,000

 

5,00,000

 

5,00,000

The company wishes to forecast balance sheet as on 31st March, 2011. The following additional particulars are available:

(i) Fixed assets costing Rs.1,00,000 have been installed on 1st April, 2010 but the payment will be made on 31st March, 2011.

(ii) The fixed assets turnover ratio on the basis of gross value of fixed assets would be 1.5.

(iii) The stock turnover ratio would be 14.4 (calculated on the basis of average stock).

(iv) The break up of cost and profit would be as follows:

Material

40%

 Labour  

25%

 Manufacturing expenses  

10%

 Office and selling expenses  

10%

 Depreciation  

5%

 Profit  

10%

 

100%

The profit is subject to interest and taxation at 50%.

(v) Debtors would be 1/9 of sales.

(vi) Creditors would be 1/5 of material consumed.

(vii) In March 2011 a dividend @ 10% on equity capital would be paid.

(viii) 12% debentures for Rs.25,000 have been issued on 1st April, 2010.

Prepare the forecast balance sheet as on 31st March, 2011 and show the following resultant ratios:

(a) Current ratio;

(b) Fixed assets/net worth ratio; and

(c) Debt equity ratio

ldquo inter firm comparison is carried out with the help of ratios although they ar 674751

1. Write short notes on:

(a) Liquidity test ratio

(b) Acid test ratio

(c) Profitability test ratios

(d) Turnover ratios.

2. “Inter firm comparison is carried out with the help of ratios although they are not exclusive and conclusive indicators of performance”. Examine.

3. Prepare a proforma income statement for the month of April, May and June for Eastern Ltd. from the following information’s:

(i) Sales are projected at Rs.4,50,000, Rs.4,80,000 and Rs.4,30,000 for April, May and June respectively.

(ii) Cost of goods sold is Rs.1,00,000 plus 30% of selling price per month.

(iii) Selling expenses are 4% of sales.

(iv) Rent Rs.15,000 per month.

(v) Administrative expenses for April are expected to be Rs.1,20,000 but are expected to rise 2% per month over the previous month’s expenses.

(vi) The company has Rs.5,00,000 of 12% loan, interest payable monthly.

(vii) Corporate tax expected is 40%.

the newman company ltd is in the midst of a promotional campaign to boost sales in 2 674752

The Newman Company Ltd. is in the midst of a promotional campaign to boost sales. In 2010 11 an additional Rs.70,000 was spent on advertising. Presented below are revenue and expense data for the company.

 

2010 11

2009 10

 

Rs.

Rs.

Sales

8,16,000

6,56,500

Sales returns and allowances

16,000

6,500

Cost of goods sold

4,00,000

3,12,000

Selling expenses

2,00,000

1,30,000

General expenses

1,20,000

78,000

Miscellaneous income

6,400

6,500

Income tax

32,000

67,600

You are required to prepare a comparative statement for the year 2010 11 and 2009 10 for the company. Also comment on the relationships revealed in the comparative income statements.

the profit and loss and balance sheet of happy ltd is given below 674754

The profit and loss and balance sheet of Happy Ltd. is given below:

Profit & Loss Account for the year ended 31st March, 2011

 

Rs

 

Rs

To Opening stock

90,000

By Sales

9,00,000

To Purchases

5,60,000

By Closing stock

90,000

To Wages

2,14,000

 

 

To Gross profit

1,26,000

 

 

 

9,90,000

 

9,90,000

To Salaries

16,000

By Gross profit

1,26,000

To Electricity

10,000

 

 

To Miscellaneous expenses

10,000

 

 

To Depreciation

30,000

 

 

To Net profit

60,000

 

 

 

1,26,000

 

1,26,000

Balance Sheet as on 31.3.2011

 

Rs

 

Rs

Share capital:

 

Fixed assets

5,40,000

Equity shares

1,80,000

Less: Depreciation

1,50,000

3,90,000

Reserve and surplus

1,20,000

 

 

Secured loans

2,10,000

Current assets:

 

 

 

Stock

90,000

Current liabilities:

 

Sundry debtors

1,05,000

Sundry creditors

90,000

Cash

15,000

2,10,000

 

6,00,000

 

6,00,000

Discuss under the following important functional groupings the usual ratios and comment on the financial strength of the company:

(i) Liquidity and solvency test ratios;

(ii) Profitability test ratios; and

(iii) Overall measures ratios.

prepare balance sheet and profit and loss account from the following information 674755

Prepare Balance Sheet and Profit and Loss Account from the following information:

 

Rs

Capital

4,00,000

Working capital

1,80,000

Bank overdraft

30,000

There is no fictitious asset. Current assets contain only stock, debtors and cash. The following additional data is also available:

(i) Closing stock is 20% higher than opening stock

(ii) Current ratio 2.5

(iii) Quick ratio 2.0

(iv) Proprietary ratio 0.6 (Fixed assets: Proprietary fund)

(v) Gross profit ratio 20% (of sales)

(vi) Stock velocity 5

(vii) Debtor’s velocity 73 days

(viii) Net profit ratio 10% (to average capital employed).

the following are the summarized profit and loss account and balance sheet of waldo 674756

The following are the summarized profit and loss account and balance sheet of Waldo Company Ltd., for the year ending 31st March, 2011.

Profit and Loss Account

 

Rs

 

Rs

To Opening stock

9,950

By Sales

85,000

To Purchases

54,525

By Closing stock

14,900

To Incidental expenses

1,425

 

 

To Gross profit

34,000

 

 

 

99,900

 

99,900

To Operating expenses:

 

By Gross profit

34,000

Selling and distribution

3,000

By Non operating

 

Administration

16,500

income interest

300

To Non operating expenses:

 

By Profit on sale of shares

600

Loss on sale of assets

400

 

 

To Net profit

15,000

 

 

 

34,900

 

34,900

Balance Sheet

 

Rs

 

RS

Issued Capital

 

Land and building

15,000

2,000 equity shares of

 

Plant and machinery

8,000

Rs.10 each

20,000

Stock in trade

14,900

Reserves

9,000

Sundry debtors

7,100

Profit and loss account

6,000

Cash and bank balance

3,000

Current liabilities

13,000

 

 

 

48,000

 

48,000

You are required to calculate:

(a) Current ratio

(b) Operating ratio

(c) Stock turnover ratio

(d) Return on total resources

(e) Turnover of fixed assets.

from the following information you are required to calculate i sales ii sundry debto 674757

From the following information you are required to calculate (i) Sales; (ii) Sundry Debtors; (iii) Sundry Creditors; (iv) Closing stock;

Debtors velocity ratio

3 months

Stock velocity ratio

6 months

Creditors velocity ratio

2 months

Gross profit ratio

25%

The gross profit for the year ended 31st March 2011 was 5,00,000. Stock for the same period was 20,000 more than it was in the beginning of the year. Bills receivable and bills payable were Rs.1,50,000 and Rs.83,333 respectively.

calculate working capital turnover ratio from the following information 674758

Calculate working capital turnover ratio from the following information:

Current ratio

=

5:3

Quick ratio

=

3:5

Inventory turnover ratio

=

5 times

Closing Stock was Rs.1,92,000 less than opening stock

Gross profit

=

25% on cost

Average debt collection period

=

3 months

Cash sales

=

25% of Total sales

Opening debtors

=

Rs.2,80,000

Closing debtors

=

Rs.3,20,000

from the information as contained in the income statement and the balance sheet of a 674759

From the information as contained in the income statement and the balance sheet of Ashok Ltd., you are required to prepare a cash flow statement using (i) Direct Method and (ii) Indirect Method.

A. Income Statement and Reconciliation of Earnings for the year ended 31.3.20 11

     Rs.

Rs.

 Net Sales  

   

25,20,000  

 Less: Cost of sales  

 

19,80,000  

 

 Depreciation  

 

60,000

 

 Salaries and wages  

 

2,40,000  

 

 Operating expenses  

 

80,000

 

 Provision for taxation  

 

88,000

24,48,000  

 Net operating profit  

   

72,000

Non recurring income:

     

Profit on sale of equipment

 

 

12,000

     

84,000

 Retained earnings (balance in profit and loss account brought forward)  

 

 1,51,800  

 

   

 2,35,800  

 Dividend declared and paid during the year  

 

72,000

 Profit and loss account balance as on 31.3.2011  

 

 1,63,800  

B. Comparative Balance Sheets

 

As at 31.3.2010

As at 31.3.2011

 

Rs

Rs

Capital

3,60,000

4,44,000

Surplus in profit and loss A/c

1,51,800

 1,63,800

Sundry creditors

2,40,000

 2,34,000

Outstanding expenses

24,000

 48,000

Income tax payable

12,000

 13,200

Accumulated depreciation on building and equipments

1,20,000

 1,32,000

 

9,07,800

 10,35,000

Fixed assets

 

 

Land

48,000

96,000

Building and equipments

3,60,000

5,76,000

Current assets

 

 

Cash

60,000

72,000

Debtors

1,68,000

1,86,000

Stock

2,64,000

96,000

Advances

7,800

9,000

 

9,07,800

10,35,000

Cost of equipment sold was 72,000.

from the following balance sheets of xyz ltd for the year ended 31st march 2010 and 674760

From the following balance sheets of XYZ Ltd., for the year ended 31st March, 2010 and 2011, prepare a cash flow statement.

   

31.3.2010

 

31.3.2011

   

Rs.

 

Rs.

 Liabilities  

 

 

 

 

 Equity Share of Rs.20 each  

 

 3,00,000  

 

 4,00,000  

 Share premium  

 

 

 

 10,000  

 Profit and loss appropriation A/c  

 1,00,000  

 

 1,00,000  

 Profit for the year  

 

 

 

 2,00,000  

 6% Debentures  

 

 1,50,000  

 

 1,00,000  

 Profit on Redemption of Debentures  

 

 

 2,000  

 Sundry creditors  

 

 1,40,000  

 

 1,10,000  

 Provision for taxation  

 

 50,000  

 

 1,00,000  

 Proposed dividend  

 

 15,000  

 

 20,000  

 

 

 7,55,000  

 

 10,42,000  

 Assets  

 

 

 

 

 Property  

 

 2,00,000  

 

 2,50,000  

 Plant and machinery  

 4,00,000  

 

 4,50,000  

 

 Less: Depreciation  

 1,40,000  

 2,60,000  

 1,50,000  

 3,00,000  

 Loans to subsidiary Co.  

 

 

 

 15,000  

 Share in subsidiary Co.  

 

 20,000  

 

 20,000  

 Stock in trade  

 

 1,40,000  

 

 1,50,000  

 Debtors  

 

 1,00,000  

 

 1,50,000  

 Bank  

 

 35,000  

 

 1,57,000  

 

 

 7,55,000  

 

 10,42,000  

Additional information:

During the year plant costing Rs.50,000 was sold for Rs.10,000. Accumulated depreciation on this plant was Rs.30,000. Loss on sale of plant was charged to profit and loss account. Income tax paid during the year was Rs.60,000.

from following balance sheet of mahendra ltd prepare cash flow statement for the yea 674761

From following balance sheet of Mahendra Ltd. prepare cash flow statement for the year ended 31.3.2011 by Indirect Method.

 

31.3.2010

31.3.2011

 

Rs

Rs

Liabilities

 

 

Equity share capital

3,00,000

4,00,000

8% Preference shares

1,50,000

 1,00,000

Capital reserve

 

20,000

General reserve

40,000

 50,000

Profit and Loss account

30,000

48,000

Proposed dividend

42,000

50,000

Sundry creditors

25,000

47,000

Bills payable

20,000

16,000

Liability for expenses

30,000

36,000

Provision for taxation

40,000

50,000

 

6,77,000

8,17,000

Assets

 

 

Goodwill

1,00,000

80,000

Land and building Plant

2,00,000

80,000

1,70,000 2,00,000

Investment

20,000

30,000

Sundry debtors

1,40,000

1,70,000

Stock

 

77,000

 1,09,000

Bills receivable

20,000

30,000

Cash in hand

15,000

10,000

Cash at bank

10,000

8,000

Preliminary expenses

15,000

6,77,000

10,000 8,17,000

Additional information’s:

(i) A piece of land has been sold during the year and the profit on sale has been credited to capital reserve. Depreciation charged on building during the year is Rs.5,000; no additions under this head during the year.

(ii) A machine was sold for Rs.10,000. The written down value of the machine was Rs.12,000. Depreciation of Rs.10,000 is charged on plant in 2010 11.

(iii) Investments are trade investments. Rs.3,000 by way of dividend is received including Rs.1,000 from pre acquisition profit which has been credited to investment account.

(iv) An interim dividend of Rs.20,000 has been paid in 2010 11.

from the following information you are required to prepare cash flow statement of xy 674766

From the following information you are required to prepare cash flow statement of XYZ Ltd., for the year ended 31st March, 2011 :

Liabilities

31.3.2010

31.3.2011

Assets

31.3.2010

31.3.2011

 

Rs

Rs

 

Rs

Rs

 Share capital  

 70,000  

 74,000  

 Bank balance  

 9,000  

 7,800  

 Bonds  

 12,000  

 6,000  

 Accounts receivable  

 14,900  

 17,700  

 Accounts payable  

 10,360  

 11,840  

 Inventories  

 49,200  

 42,700  

 Provision for  

 

 

 Land  

 20,000  

 30,000  

 doubtful debts  

 700  

 800  

 Goodwill  

 10,000  

 5,000  

 Reserves and  

 

 

 

 

 

 Surplus  

 10,040  

 10,560  

 

   
 

1,03,200

1,03,200

 

1,03,200

1,03,200

Following additional information has also been supplied to you:

(i) Dividends of Rs.3,500 thousand were paid during the year 2010 11.

the balance sheets of narula ltd as at the end of march 2010 and 2011 were given bel 674767

The balance sheets of Narula Ltd. as at the end of March 2010 and 2011 were given below:

 

31.3.2010

31.3.2011

 

Rs

Rs

Share capital

1,00,000

1,50,000

Share premium

 

5,000

General reserve

50,000

60,000

Profit & Loss A/c

10,000

17,000

8% Debentures

70,000

50,000

Depreciation reserve:

 

 

Plant

50,000

56,000

Furniture

5,000

6,000

Provision of tax

20,000

30,000

Sundry creditors

86,000

95,000

Reserve for bad debts

2,000

5,000

 

3,93,000

4,74,000

Assets

 

 

Freehold land

1,00,000

1,00,000

Plant at cost

1,04,000

1,00,000

Furniture at cost

7,000

9,000

Investment at cost

60,000

80,000

Debtors

32,000

75,000

Stock

60,000

65,000

Cash

30,000

45,000

 

3,93,000

4,74,000

A plant purchased for Rs.4,000 (Depreciation Rs.2,000) was sold for Rs.800 on 31st December, 2010. On October 2010 an item of furniture was purchased for Rs.2,000. Depreciation on plant was provided at 8 per cent on cost (excluding sold out item) and on furniture at 12.5 per cent on average cost. A dividend of 22.5 per cent on original shares was paid.

Prepare a cash flow statement for 2010 11.

the financial position of xyz ltd on 1st april 2010 and 31st march 2011 was as follo 674768

The financial position of XYZ Ltd. on 1st April, 2010 and 31st March, 2011 was as follows:

 

 

1.4.2010

 

31.3.2011

 

 

Rs

 

Rs

Liabilities

 

 

 

 

Current liabilities

 

72,000

 

82,000

Loan from associate company

 

 

 

40,000

Loan from bank

 

60,000

 

50,000

Capital and reserve

 

2,96,000

 

2,98,000

 

 

4,28,000

 

4,70,000

Assets

 

 

 

 

Cash

 

8,000

 

7,200

Debtors

 

70,000

 

76,800

Stock

 

50,000

 

44,000

Land

 

40,000

 

60,000

Buildings

 

1,00,000

 

1,10,000

Machinery cost

2,14,000

 

2,44,000

 

Less: Prov. for

 

 

 

 

depreciation

54,000

1,60,000

72,000

1,72,000

 

 

4,28,000

 

4,70,000

During the year Rs.52,000 were paid as dividends.

You are required to prepare cash flow statements for the year ended 31.3.2011.

the following balance sheets have been prepared from the books of taj limited as app 674769

The following balance sheets have been prepared from the books of Taj Limited as appearing on 31.3.2010 and 31.3.2011.

 

31.3.2010

31.3.2011

 

Rs

Rs

 Liabilities  

 

 

 Equity capital  

 4,00,000  

 6,00,000  

 Share premium  

 1,00,000  

 1,10,000  

 General reserve  

 2,00,000  

 2,20,000  

 Debenture redemption reserve  

 1,00,000  

 1,10,000  

 Debentures  

 3,00,000  

 2,90,000  

 Taxation provision  

 40,000  

 35,000  

 Secured loan (short term)  

 2,00,000  

 1,00,000  

 Current liabilities  

 24,000  

 30,000  

 

 13,64,000  

 14,95,000  

 Assets  

 

 

 Buildings  

 5,70,000  

 5,00,000  

 Plant and machinery  

 3,60,000  

 3,51,000  

 Furniture  

 90,000  

 81,000  

 Cash in hand  

 5,000  

 8,000  

 Stock  

 1,55,000  

 1,25,000  

 Debtors  

 1,80,000  

 1,80,000  

 Investments (short term)  

 

 2,10,000  

 Bills receivable  

4,000

40,000

 

13,64,000 

14,95,000 

(i) During 2010 11, the company paid 12% dividend on its equity share capital of Rs.4,00,000.

(ii) The shares are of Rs.10 each fully paid.

(iii) Taxation provision of 2009 10 was utilised to the extent of Rs.30,000 for income tax paid in 2010 11.

(iv) Depreciation was charged on building at 5%; on plant and machinery at 10% and on furniture at 10% for a full one year.

(v) A building worth Rs.70,000 was sold on 1.4.2010 at Rs.60,000 and a new building was constructed at a value of 25,000 on 31.3.2011.

(vi) A machine was purchased at a cost of Rs.40,000 on 1.4.2010 while a machine having a book value of Rs.10,000 was sold on 1.10.2010 at Rs.20,000.

Prepare a cash flow statement for the year ended 31.3.2011.

the following information is available from the books of exclusive ltd for the year 674771

The following information is available from the books of Exclusive ltd. For the year ended 31st March 2011:

(i) Cash sales for the year were Rs.10,00,000 and sales on account Rs.12,00,000

(ii) Payments on accounts payable for inventory totaled Rs.7,80,000

(iii) Collection against accounts receivable were Rs.7,60,000

(iv) Rent paid in cash Rs.2,20,000, outstanding rent being Rs.20,000

(v) 4,00,000 equity shares of Rs.10 par value were issued for Rs.48,00,000

(vi) Equipment was purchased for cash Rs.16,80,000

(vii) Dividend amounting to Rs.10,00,000 was declared but yet to be paid.

(viii) Rs.4,00,000 of dividends declared in the previous year were paid.

(ix) Equipment having a book value of Rs.1,60,000 was sold for Rs.2,40,000

(x) The cash account was increased by Rs.37,20,000

Prepare a cash flow statement using direct method.

in a simple binary communication system during every t seconds one of two possible s 675646

In a simple binary communication system, during every T seconds, one of two possible signals s0(t) and s1(t) is transmitted. Our two hypotheses are

H0: s0(t) was transmitted.

H1 : s1(t) was transmitted.

We assume that

s0(t) = 0

and

s1(t)= 1

0 < t < T

The communication channel adds noise n(t), which is a zero mean normal random process with variance 1. Let x(t) represent the received signal :

We observe the received signal x(t) at some instant during each signaling interval. Suppose that we received an observation x = 0.6.

(a) Using the maximum likelihood test, determine which signal is transmitted.

(b) Find P1 and Pn .

a company lsquo s production for the year ending 30 3 2011 is given below 674681

A company‘s production for the year ending 30.3.2011 is given below:

Items

Production Departments

Office

Stores

Work shop

Total

 

P1

P2

P3

       

Direct Wages

Rs.20,000

25,000

30,000

 

 

 

75,000

Direct Materials

Rs.30,000

35,000

45,000

 

 

 

1,10,000

Indirect Materials

Rs.2,000

3,000

3,000

1,000

2,000

2,000

13,000

Indirect Wages

Rs.3,000

3,000

4,000

10,000

10,000

5,000

35,000

Area in Square Meters

200

250

300

150

100

250

1,250

Book value of Machinery

Rs.30,000

35,000

25,000

 

 

15,000

1,05,000

Total H.P. of Machinery

15

20

25

 

 

5

65

Machine Hours Worked

10,000

20,000

15,000

 

 

5,000

50,000

General Expenses:

(i) Rent

Rs.12,500

(ii) Insurance

Rs.1,050

(iii) Depreciation

15% of value of machinery

(iv) Power

Rs.3,800

(v) Light

Rs.1,250

You are required to prepare an overhead analysis sheet for the departments showing clearly the basis of apportionment when necessary.

a factory has two service departments p and q and three production departments a b a 674682

A factory has two service departments P and Q and three production departments A, B, and C. You are supplied with the following information:

Particulars

Total

Production Departments

Service Departments

   

A

B

C

P

Q

 

Rs

Rs

Rs

Rs

Rs

Rs

 Rent  

12,000

2,400

4,800

2,000

2,000

800

 Electricity  

4,000

800

2,000

500

400

300

 Indirect labour  

6,000

1,200

2,000

1,000

800

1,000

 Depreciation of machinery  

5,000

2,500

1,600

200

500

200

 Sundries  

4,500

910

2,143

847

300

300

Estimated working hours

 

1,000

2,000

1,400

   

Expenses of Service Departments P and Q are apportioned as under:

 

 A  

 B  

 C  

 P  

 Q  

 P  

30%

40%

20%

 

10%

 Q  

10%

20%

50%

20%

 

You are required to show the apportionment of overheads under different methods of apportioning inter service departments overheads and also to work out the production hour rate recovery of overheads in departments A, B and C.

the following information has been collected from the cost records of a small compan 674683

The following information has been collected from the cost records of a small company for the year ended 31st March:

 

Rs

 Direct Materials  

 2,50,000  

 Direct Labour  

 2,00,000  

 Direct Expenses  

20,000

 Works Overheads  

 1,60,000  

 Office Expenses  

94,500

The total number of direct labour hours were 1,00,000 involving 40,000 machine hours. What should be the price quoted for a job involving 2,000 labour hours @ Rs.3 per hour, 1,000 machine hours and Rs.10,000 in direct materials if the profit desired is 20% on the selling price?

the machine occupies 1 4th of the total area of the shop the supervisor is expected 674684

Calculate the machine hour rate from the following:

 

Rs

 Cost of machine  

18,000

 Cost of installation  

2,000

 Scrap value after 10 years  

2,000

 Rates and rent for a quarter for the shop  

600

 General lighting  

 200 p.m.  

 Shop supervisor‘s salary  

Rs.6,000 per quarter  

 Insurance premium for a machine  

 120 p.a.  

 Estimated repair  

 200 p.a.  

 Power 2 units per hour @ Rs.150 per 100 units  

 

 Estimated working hours p.a. 2,000  

 

The machine occupies 1/4th of the total area of the shop. The supervisor is expected to devote 1/6th of his time for supervising the machine. General lighting expenses are to be apportioned on the basis of floor area.

the following information is obtained from the records of a factory regarding the ex 674687

The following information is obtained from the records of a factory regarding the execution of two orders for the same quantity of a commodity:

 

Materials

Wages

Sale Price

Percentage of Profit on Cost of Production %

 

Rs

Rs

Rs

 

 First order  

25,000

20,000

85,800

10

 Second order  

36,000

28,000

1,23,760

12

Find out the percentage of Factory Overheads and Office Overheads.

hind private ltd manufactures four sizes of the product modern model lsquo called a 674688

Hind Private Ltd. manufactures four sizes of the product `Modern Model‘ called A, B, C, and D in the Department. The workers are paid the piece rate of Re. 1.00, Rs.1.50, Rs.2.00, Rs.3.00 per unit of the product sizes A, B, C and D respectively. Dearness allowance paid to the workers is 4.00 per day. Miscellaneous payments are 20% of the basic wages. From the following information for the month of January, you are required to find the total cost per unit of each size of the product `Modern Model‘:

   

Product

   

Size A

Size B

Size C

Size D

 Direct Labour (Days)  

 

104

78

52

52

 Production (Units) Direct Material ( )  

 

320

150

70

55

 Overhead Expenses:  

 

250

150

100

125

 

Rs

       

 Indirect Material  

500

       

 Indirect Labour  

572

       

 Indirect Expenses  

429

       

a company is having three production departments x y and z and two service departmen 674694

A company is having three production departments X, Y and Z and two service departments boiler house and pump room. The boiler house has to depend upon the pump room for supply of water and pump room in its turn is dependent on the boiler house for supply of steam power for driving the pump. The expenses incurred by the production departments are: X

Rs.6,00,000; Y – Rs.5,25,000; and Z – Rs.3,75,000. The expenses for boiler house are Rs.1,75,500 and pump room are 2,25,000.

The expenses of the boiler house and pump room are apportioned to the production departments on following basis:

 

DEPARTMENT

Boiler

Pump

 

X

Y

Z

House

Room

Expenses of boiler house

20%

40%

30%

10%

Expenses of pump room

40%

20%

20%

20%

Show clearly as to how the expenses of boiler house and pump room would be apportioned to X, Y and Z departments?

the budgeted working conditions of a cost centre are as follows 674695

The budgeted working conditions of a cost Centre are as follows:

Normal working per week 

 42 hours 

 No. of machines 

14

 Normal weekly loss of hours on 

 

 maintenance etc. 

 5 hours per machine 

 No. of weeks worked per year 

48

 Estimated annual overheads 

2,48,640

 Estimated direct wage rate

 8 per hour 

Actual results in respect of a week period are:

 

Wages incurred

18,000

Overheads incurred

20,400

Machine hours produced

2,000

You are required to calculate:

(i) The overhead rate per machine hour; and

(ii) The amount of under or over absorption of wages and overheads.

the following particulars have been extracted from the books of a manufacturing comp 674696

The following particulars have been extracted from the books of a manufacturing company for the month of March, 2011:

 

Rs

 Stock of materials as on 1st March, 2011  

47,000

 Stock of materials as on 31st March, 2011  

50,000

 Materials purchased during the month  

2,08,000

 Drawing office salaries  

9,600

 Counting house salaries  

14,000

 Carriage on purchases  

8,200

 Carriage on sales  

5,100

 Cash discount allowed  

3,400

 Bad debts written off  

4,700

 Repairs of plant, machinery and tools  

10,600

 Rent, rates, taxes and insurance (factory)  

3,000

 Rent, rates, taxes and insurance (office)  

1,000

 Travelling expenses  

3,100

 Travellers‘ salaries and commission  

8,400

 Productive wages  

1,40,000

 Depreciation written off on plant, machinery and tools  

7,100

 Depreciation written off on office furniture  

600

 Directors‘ fees  

6,000

 Gas and water charges (factory)  

1,500

 Gas and water charges (office)  

300

 General charges  

5,000

 Manager‘s salary  

12,000

Out of 48 working hours in a week, the time devoted by the Manager to the factory and office was on an average 40 hours and 8 hours respectively throughout the month. 1,00,000 units were produced and sold; there was no opening or closing stock of it.

Prepare a cost sheet showing the following:

(i) Cost of Materials Consumed;

(ii) Prime Cost;

(iii) Works Overhead;

(iv) Works Cost;

(v) Office and Administration Overhead;

(vi) Cost of Production;

(vii) Selling and Distribution Overhead; and

(viii) Total Cost or Cost Sales.

the following information has been obtained from the records of abc co ltd for the m 674697

The following information has been obtained from the records of ABC Co. Ltd. For the month of January, 2011:

 

Rs

 Cost of raw materials on 1/01/2011  

30,000

 Purchase of raw materials during the month  

4,50,000

 Wages paid  

2,30,000

 Factory overheads  

92,000

 Cost of work in progress on 1/01/2011  

12,000

 Cost of raw materials on 30 /01/2011  

25,000

 Cost of work in progress on 30 /01/2011  

15,000

 Cost of stock of finished goods on 1 /01/2011  

60,000

 Cost of stock of finished goods on 30 /01/2011  

55,000

 Administration overheads  

30,000

 Selling and distribution overheads  

20,000

 Sales  

9,00,000

Prepare: (i) Cost sheet showing the cost of production of goods manufactured, and (ii) Statement showing the cost of sales and the profit earned.

the following balances were extracted from the books of a building contract on 31st 674698

The following balances were extracted from the books of a building contract on 31st March, 2011 regarding Contract No. 123:

 

Rs

Materials issued to site

6,27,200

Wages Paid

7,34,550

Wages outstanding on 31.3.2011

7,200

Plant issued to site

60,000

Direct charges paid

25,150

Direct charges outstanding on 31.3.2011

2,100

Establishment charges

56,500

Stock of materials at site on 31.3.2011

12,000

Value of work certified on 31.3.2011

16,50,000

Cost of work not yet certified

35,000

Cash received on account of architect‘s

 

certificate after deduction by customer

 

of 5 percent retention money

14,10,750

The work was commenced on April 1, 2010 and the contract price agreed at Rs.24,50,000.

Prepare contract account for the year providing for depreciation of plant of 25 per cent. Calculate the Profit or Loss in the contract to date and make such provision in the contract account as you consider desirable. Set out also contractor‘s balance sheet so far as it relates to the contract.

xyz contractors obtained a contract to construct a house for rs 8 00 000 work was st 674700

XYZ contractors obtained a contract to construct a house for Rs.8,00,000. Work was started on 1st January, 2010 and it was estimated that contract would take 15 months to complete. Work is proceeding as per schedule and the details upto 31st December 2010 are as follows:

 

Rs

Materials and stores

1,87,000

Wages Paid

2,70,000

Plant hire charges and other expenses

60,000

Establishment charges

54,000

Material unused

11,000

Work Certified

6,00,000

Cash received

5,40,000

Work not yet certified (at cost)

20,000

It is further estimated that the following further expenses will be required to complete the work:

Additional material: Rs.25,000; Wages : Rs.20,000; Sub Contract cost : Rs.50,000;

Plant hire charges : Rs.10,000; Establishment Expenses : Rs.11,800; and provision for contingencies : 5% of total cost

You are required to calculate the value of Work in Progress as on 31st December 2010 taking credit for a reasonable profit and also show the contract account.

explain the terms a escalation clause and b cost plus contract 674701

1. Explain the terms: (a) escalation clause and (b) cost plus contract.

2. The cost of sales of product A is made up as follows:

 

Rs

Materials used in manufacturing

5,500

Materials used in packing materials

1,000

Materials used in selling the product

150

Materials used in the factory

75

Materials used in office

125

Labour required in production

1,000

Labour required for supervision of the management of factory

200

Expenses direct, factory

500

Expenses indirect, factory

100

Expenses office

125

Depreciation office building and equipment

75

Depreciation factory

175

Selling expenses

350

Freight

500

Advertising

125

Assuming that all the products manufactured are sold, what should be the selling price to obtain a profit of 25% on selling price?

Illustrate in a chart form for presentation to your manager, the division of costs for Product  width=

from the following particulars of a manufacturing firm prepare a statement showing a 674702

From the following particulars of a manufacturing firm, prepare a statement showing: (a) Cost of Materials Used, (b) Prime Cost, (c) Works Cost, (d) Cost of Production, (e) Cost of Sales, and (f) Profit Earned.

 

Rs

Stocks of materials on 1st January, 2011

40,000

Purchase of materials in January, 2011

11,00,000

Stock of Finished goods on 1st January, 2011

50,000

Stock of work in progress on 1st January, 2011

35,000

Productive wages

5,00,000

Works overhead charges

1,50,000

Office and administration overheads

90,000

Selling and distribution overheads

60,000

Stock of materials on 31st January, 2011

1,40,000

Stock of finished goods on 31st January, 2011

60,000

Stock of work in progress on 31st January, 2011

25,000

Finished Goods sold in January, 2011

22,50,000

in a factory two types of radios are manufactured viz orient and sujan models from t 674703

In a factory, two types of radios are manufactured viz. Orient and Sujan Models. From the following particulars, prepare a statement showing cost and profit per radio sold. There is no opening or closing stock.

 

Orient

Sujan

 

Rs

Rs

 Direct materials  

27,300

1,08,680

 Direct labour  

15,600

62,920

Works overhead is charged @ 80% on labour and office overhead is taken at 15% on works cost. The selling price of both the radios is 1,000 each. 78 Orient radios and 286 Sujan radios were sold.

a firm of building contractors began to trade on 1st january 2010 during the year th 674704

A firm of building contractors began to trade on 1st January, 2010. During the year, the company was engaged on only one contract. The contract price was Rs.50,00,000.

Of the plant and materials charged to the contract, the plant which cost Rs.50,000 and materials which cost Rs.40,000 were lost in an accident.

On December 31, 2011, the plant which cost Rs.50,000 was returned to the stores the cost of work done but uncertified was Rs.20,000 and the materials costing Rs.40,000 were in hand on site.

Charge 10% depreciation of the plant, carry forward by way of reserve one third of the profit received and compile the Contract Account and the Balance Sheet from the following Trial Balance on December 31, 2011.

 

Rs

Rs

Share capital

 

12,00,000

Creditors

 

1,00,000

Cash received on contract (80% of work certified)

 

20,00,000

Land, buildings, etc.

4,30,000

 

Bank balance

2,50,000

 

Charged to contract

   

Materials

9,00,000

 

Plant

2,50,000

 

Wages

14,00,000

 

Expenses

70,000

 
 

33,00,000

33,00,000

abc ltd produces and sells a single product sales budget for the calendar year 2011 674707

ABC Ltd. produces and sells a single product. Sales budget for the calendar year 2011 for each quarter is as under:

Quarter

No. of Units to be Sold

I

12,000

II

15,000

III

16,500

IV

18,000

The year 2011 is expected to open with an inventory of 4,000 units of finished product and close with an inventory of 6,500 units. Production is customarily scheduled to provide for two thirds of the current quarter‘s demand plus one third of the following quarter‘s demand. Thus production anticipates sales volume by about one month. The standard cost details for one unit of the product is as follows:

— Direct materials 10 Kgs. @ Rs.50 paise per kg.

— Direct labour 1 hour 30 minutes @ Rs.4 per hour.

— Variable overheads 1 hour 30 minutes @ Rs.1 per hour.

— Fixed overheads 1 hour 30 minutes @ Rs.2 per hour based on a budgeted production volume of 90,000 direct labour hours for the year.

Answer the following:

(i) Prepare a production budget for the year 2011 by quarters, showing the number of units to be produced.

(ii) If the budgeted selling price per unit is Rs.17, what would be the budgeted profit for the year as a whole?

(iii) In which quarter of the year the company is expected to break even? 

abc ltd a newly started company wishes to prepare cash budget from january prepare a 674710

ABC Ltd. a newly started company wishes to prepare cash budget from January. Prepare a cash budget for the first six months from the following estimated revenue and expenses.

Month

Total Sales Rs.

Materials Rs.

Wages Rs.

Overheads

       

Production Rs.

Selling & Distribution Rs.

 Jan. 

 20,000 

 20,000 

 4,000 

 3,200 

 800 

 Feb. 

 22,000 

 14,000 

 4,400 

 3,300 

 900 

 March 

 28,000 

 14,000 

 4,600 

 3,400 

 900 

 April 

 36,000 

 22,000 

 4,600 

 3,500 

 1,000 

 May 

 30,000 

 20,000 

 4,000 

 3,200 

 900 

 June 

 40,000 

 25,000 

 5,000 

 3,600 

 1,200 

Cash balance on Ist January was Rs.10,000. New machinery is to be installed at Rs.20,000 on credit, to be repaid by two equal installments in March and April.

Sales commission at @ 5% on total sales is to be paid within a month following actual sales.

Rs.10,000 being the amount of 2nd call may be received in March. Share premium amounting to

Rs.2,000 is also obtainable with the 2nd call.

Period of credit allowed by suppliers

2 months

Period of credit allowed to customers

1 month

Delay in payment of overheads

1 month

Delay in payment of wages

1/2 month

Assume cash sales to be 50% of total sales.

the cost of an article at capacity level of 5 000 units is given under a below for a 674711

The cost of an article at capacity level of 5,000 units is given under A below. For a variation of 25% in capacity above or below this level, the individual expenses vary as indicated under B below:

 

A

B

 

Rs

 

Material cost

25,000

(100% varying)

Labour cost

15,000

(100% varying)

Power

1,250

(80% varying)

Repairs and maintenance

2,000

(75% varying)

Stores

1,000

(100% varying)

Inspection

500

(20% varying)

Depreciation

10,000

 (100% fixed)  

Administration overheads

5,000

 (25% varying)  

Selling overheads

3,000

 (50% varying)  

 

62,750

 

Cost per unit

12.55

 

Find the unit cost of the product under each individual expense at production levels of 4,000 units and 6,000 units.

determine which company is more profitable 674743

1. Determine which company is more profitable:

 

A Ltd.

B Ltd.

Net Profit Ratio

3%

4%

Sales/Capital Employed

5 times

3 times

2. From the following information, calculate, debtors turnover ratio and average collection period.

 

Rs.

Total debtors (opening balance)

2,00,000

Cash sales

1,50,000

Credit sales

10,00,000

Cash collected

7,80,000

Sales returns

60,000

Bad debts

40,000

Discount allowed

20,000

Provision for bad debts

25,000

No. of days in a year 360

 

using the following data compute i closing inventory and ii cost of sales under bdqu 674647

Using the following data, compute (i) Closing Inventory and (ii) Cost of sales under „current purchasing power? (CPP) method assuming that the firm is following LIFO method of inventory valuation:

Inventory as on 1/04/2010:

Rs.2,40,000

Purchases during 2010:

Rs.14,40,000

Inventory as on 31/03/2011:

Rs.3,60,000

Price index as on 01/04/2010:

100

Price index as on 31/03/2011:

130

Average price Index for 2010:

120

a company draws up the standard cost of a product as follows 674649

A company draws up the standard cost of a product as follows:

 

Rs

Rs

Rs

Direct Materials

 

12

 

Direct Wages:

     

 Dept. A 3 hours  

15

   

 Dept. B 2 hours  

12

   

 Dept. C 5 hours  

20

47

 

Factory Overhead:

     

 Dept. A  

18

   

 Dept. B  

18

   

 Dept. C  

40

76

 

Factory Cost

   

135

Administration Cost

   

12

Selling Cost

   

15

Distribution cost

   

18

Total

   

180

Net Profit

   

20

Selling Price

   

200

Factory overhead is absorbed by means of departmental hour rates. Analysis of these overheads reveal that in each department a rate of Rs.2 per hour is required to absorb the variable portion, the balance being of a fixed nature. As a general rule, all production is of first class quality.

After a batch of 1,000 units has been processed through all three departments, inspection reveals that half are faulty. The faulty products can be rectified by completely re processing through departments B and C. Alternatively, they can be sold for Rs.20 each.

Present figures which indicate to management the most economic method of dealing with the faulty products. 

from the following records regarding material calculate i the re order level ii the 674651

From the following records regarding material calculate (i) the re order level, (ii) the maximum stock level, and (iii) the minimum stock level.

Re order quantity

6,000 units

Minimum stock (for emergencies)

5 weeks

Average delivery time

4 weeks

Maximum stock level

20 weeks

Average consumption per week

400 units

Minimum consumption in 4 weeks

1,200 units

calculate for each components 1 the re order level 2 the minimum level 3 the maximum 674652

Two components X and Y are used as follows:

Normal usage

50 units each per week

Minimum usage

25 units each per week

Maximum usage

75 units each per week

Re order quantity

X: 400 units; Y: 600 units

Re order period

X: 4 to 6 weeks; Y: 2 to 4 weeks.

Calculate for each components: (1) the re order level, (2) the minimum level, (3) the maximum level, and (4) the average stock level.

from the following transactions prepare separately the stores ledger accounts using 674653

From the following transactions, prepare separately the Stores Ledger Accounts, using the following pricing methods: (i) the FIFO, (ii) the LIFO.

January

1

Opening balance 100 units @ Rs.5 each

January

5

Received 500 units @ Rs.6 each

January

20

Issued 300 units

February

5

Issued 200 units

February

6

Received 600 units @ Rs.5 each

March

10

Issued 300 units

March

12

Issued 250 units

the following receipts and issues of materials were made during the month of january 674654

The following receipts and issues of materials were made during the month of January.

January 1

Opening stock 80 units @ Rs.1.00 each

January 7

Received from vendors 40 units @ Rs.1.10 each

January 12

Received from vendors 60 units @ Rs.1.20 each

January 22

Received from vendors 72 units @ Rs.1.25 each

January 4

Issued 60 units

January 9

Issued 40 units

January 14

Issued 40 units

January 30

Issued 80 units

Prepare the Stores Ledgers maintained under (i) the FIFO, (ii) the LIFO methods.

the following transactions took place in respect of a material item during the month 674655

The following transactions took place in respect of a material item during the month of March:

Date

Receipt Qty.

Rate

Issue Qty.

March 2

200

2.00

 

March 10

300

2.40

 

March 15

 

 

250

March 18

250

2.60

 

March 20

 

 

200

Prepare the Stores Ledger Sheet, pricing the issue at the simple average rate and the weighted average rate.

stocks are issued at the standard price and debit balance of variance amount before 674656

Stocks are issued at the standard price and debit balance of variance amount before transfer to Costing Profit and Loss Account was Rs.500. The following

purchases and issues were made during the month of April:

April 1

Opening Balance

100 units @ Rs.90.00 per unit

April 5

Purchased

500 units @ Rs.85.00 per unit

April 6

Issued

60 units

April 12

Issued

375 units

April 23

Issued

65 units

April 30

Purchased

250 units @ Rs.80.00

Find out the standard price for the issue and prepare the Stores Ledger Account. Also calculate the material price variance.

after inviting tenders two quotations are received as follows 674657

After inviting tenders two quotations are received as follows:

(a) Rs.1.20 per unit.

(b) Rs.1.10 per unit plus Rs.3,000 fixed charges to be added irrespective of units ordered.

Advise with your arguments on whom orders should be placed and what quantity is to be ordered.

The following additional information may be of interest:

Present Stock

35,000

Average Monthly Requirement

10,000

Maximum Level

 80,000

Minimum Level

30,000

(Sales tax to be ignored in both cases).

 

a consignment consisted of two chemicals a and b the invoice gave the following data 674660

A consignment consisted of two chemicals A and B. The invoice gave the following data:

Chemical A 4,000 kgs. @ Rs.2.50 per/kg.

10,000

Chemical B 3,200 kgs. @ Rs.3.25 per/kg.

10,400

Sales Tax

816

Railway Freight

384

Total cost

21,600

A shortage of 200 kgs. in A and 128 kgs. in B was noticed due to breakage.

What is the stock rate you would adopt for pricing issues assuming a provision of 5% towards further deterioration?

the following is an extract of the record of receipt and issues of sulphur in a chem 674661

The following is an extract of the record of receipt and issues of sulphur in a chemical factory during February:

1 Opening balance 500 tons @ Rs.200

3 Issued 70 tons

4 Issued 100 tons

8 Issued 80 tons

13 Received from supplier 200 tons @ Rs.190

14 Returned from Deptt. 15 tons

16 Issued 180 tons

20 Received from supplier 240 tons @ Rs.190

24 Issued 300 tons

25 Received from supplier 320 tons @ Rs.190

26 Issued 115 tons

27 Returned from Deptt. 35 tons

28 Received from supplier 100 tons @ Rs.190

Issues are to be priced on the principle of „First in First out?. The stock verifiers of the factory had found a shortage of 10 tons on the 22nd and left a note accordingly. Draw up a priced stores ledger card for the material showing the above transactions.

x ltd has purchased and issued the materials in the following order 674662

X Ltd. has purchased and issued the materials in the following order:

   

Unit

Unit Cost

     

Rs

 1st January  

 Purchased  

300

3

 4th January  

 Purchased  

600

4

 6th January  

 Issued  

500

 

 10th January  

 Purchased  

700

4

 15th January  

 Issued  

800

 

 20th January  

 Purchased  

300

5

 23rd January  

 Issued  

100

 

Ascertain the quantity of closing stock as on 31st January and state what would be its value (in each case) if issues were made under the following methods:

(i) Average cost.

(ii) First in First out.

(iii) Last in First out.

three workers x y and z work in a factory the following particulars apply to them 674663

Three workers X, Y and Z work in a factory. The following particulars apply to them:

Normal rate per hour

4.00

Piece rate

3.00 per unit

Standard 2 units per hour

In a 40 hour week, the production of the workers is as follows:

X

50 units

Y

80 units

Z

120 units

Calculate the earnings of the workers under (a) Taylor differential piece rate system, (b) Merrick differential piece rate system, and (c) Gantt‘s task bonus system. Also show cost per unit under these methods.

in a factory bed aux point premium system is in operation the following are the part 674664

1. In a factory Bed aux Point Premium System is in operation. The following are the particulars with regard to a job in a factory:

Allowed time for the job 600 minutes (or B‘s)

Time taken 480 minutes (or B‘s)

Rate 12.00 per hour

Calculate bonus and earnings.

2. The following particulars apply to a particular work situation:

Standard time allowed 6 hours

Rate per hour Re. 10.00

Actual time taken by

Worker P

8 hours

Worker Q

6 hours

Worker R

4 hours

Calculate the wages of the workers under Barth Premium System. Also calculate labour cost per hour.

from the following information calculate the bonus and earnings under emerson effici 674665

1. From the following information, calculate the bonus and earnings under Emerson Efficiency Bonus Plan:

Standard output in 12 hours

48

Actual output in 12 hours

42

Time rate 7.50 per hour

If the actual output is 60 units, what will be amount of bonus and earnings?

2. The existing incentive system of a certain factory is:

Normal working week  

 5 days of 9 hours plus 3 late shifts of  

 

 3 hours each  

 Rate of payment  

 Day work =  Rs.10.00 per hour

 

 Late shift =  Rs.15.00 per hour

 Additional bonus payable  

 25.00 per day shift  

 

 15.00 per late shift  

Average output per operative for 54 hours week i.e., including 3 late shifts 120 articles

In order to increase output and eliminate overtime it was decided to switch on to a system of payment by results. The following information is obtained:

Time rate (as usual)

10.00 per hour 

Basic time allowed for 15 articles

5 hours

Piece work rate

Add: 20% to piece

Premium

Add: 50% to time

(i) hours worked;

(ii) weekly earnings;

(iii) number of articles produced; and

(iv) labour cost per article for one operative under the following systems:

(a) Existing time rate.

(b) Straight piece work.

(c) Rowan system.

(d) Halsey Weir.

Assume that 135 articles are produced in a 45 hour week under (b), (c) and (d) and that the worker earns half the time saved under the Halsey Weir System. The additional bonus under the existing system will be discontinued in the proposed incentive scheme.

find out whether the year 2011 showed an overall better performance or otherwise 674670

From the following comparative statements of the years 2010 and 2011:

(a) Find out whether the year 2011 showed an overall better performance or otherwise:

(b) Possible causes of difference:

 

2010

2011

Wages incurred

2,80,000

5,10,000

Units produced

16,000

25,000

Average number of workers

225

400

(Assume production of only one quality and same machinery conditions in both years).

the following particulars of soni amp co relate to the year ending 31st march 2011 f 674672

The following particulars of Soni & Co. relate to the year ending 31st March, 2011 for 30 workers :

 

Rs

 Basic wages  

50,000

 Dearness allowance  

25,000

 Night shift allowance  

9,600

 Overtime allowance  

7,000

 PF deposit  

12,000

 ESI contribution  

2,808

 Recovery towards house rent  

10,200

 Recoveries against supply of goods  

16,000

 Expenditure for employees‘ amenities  

4,730

PF is paid in equal share by the employer and employee. Contribution to ESI is in proportion of 7:5 by the employer and employee respectively. The workers are entitled to 5% of the total days worked as leave on full pay. The number of days worked in a year is 300. Normal idle time is 5%. Assuming that all the items are evenly spread over all the days in a year find out total wages, total cash payment to workers and per hour per labour wages. The daily working hours are 8.

x the proprietor of a small engineering workshop producing specialty product by empl 674677

X, the proprietor of a small engineering workshop producing specialty product by employing 5 skilled workers is considering the introduction of some incentive scheme either Halsey scheme or Rowan scheme of wage payment for increasing the labour productivity to cope with the increased demand for the product by about 25%. He feels that if the proposed incentive scheme could bring about an average 20% increase over the present earnings of the workers, it would act as a sufficient incentive for them to produce more and he has accordingly given this assurance to the workers.

As a result of this assurance, an increase in productivity has been observed as revealed from the following figures for the current month:

Hourly rate of wages (guaranteed)  

5

 Average time for producing 1 piece by one worker as  

 

 per the previous performance (X desires that this  

 

 time be considered as time allowed for the purpose  

 

 of incentive scheme)  

 2 hours  

 No. of working days in the month  

25

 No. of working hours per day for each worker  

8

 Actual production during the month  

 625 pieces  

You are required to:

(a) Calculate effective rate of earnings per hour under Halsey scheme and Rowan scheme.

(b) Calculate the savings to X in terms of direct labour cost per piece under the above schemes.

(c) Advise X about the selection of the scheme to fulfill his assurance.

calculate the normal and overtime wages payable to a workman from the following data 674678

Calculate the normal and overtime wages payable to a workman from the following data:

Days

Hours worked

Monday

8 hrs.

Tuesday

10 hrs.

Wednesday

9 hrs.

Thursday

11 hrs.

Friday

9 hrs.

Saturday

4 hrs.

Normal rate Rs. 5.00 per hour

Normal working hours 8 hours per day.

Overtime rate Upto 9 hours in a day at single rate and over 9 hours in a day

at double rate;

plot the above information on the graph to draw a 674680

Month  

 Output  Units 

 Indirect Expenses  ( Rs) 

 April  

 1,500  

 6,000  

 May  

 1,800  

 6,600  

 June  

 2,100  

 7,200  

 July  

 2,820  

 8,640  

 August  

 2,220  

 7,440  

Plot the above information on the graph to draw a  width= of best fit‘.

the company follows the practice of transferring 25 of profits to general reserve si 674621

Balance Sheet of Diamond Ltd. as on 31.3.2011:

Liabilities:

Rs

 Share capital:  

 

 20,000 shares of Rs.10 each  

 2,00,000  

 General reserve  

40,000

 Profit and loss account  

32,000

 Sundry creditors  

 1,28,000  

 Income tax reserve  

60,000

 

 4,60,000  

Assets:

Rs

 Land and buildings  

 1,10,000  

 Plant and machinery  

 1,30,000  

 Patents and trade marks  

20,000

 Stock  

48,000

 Debtors  

88,000

 Bank balance  

52,000

 Preliminary expenses  

12,000

 

 4,60,000  

The expert valuer valued the land and buildings at Rs.2,40,000; goodwill at Rs.1,60,000; and plant and machinery at Rs.1,20,000. Out of the total debtors, it is found that debtors of Rs.8,000 are bad. The profits of the company have been as follows:

 

Rs

31.3.2009

92,000

31.3.2010

88,000

31.3.2011

96,000

The company follows the practice of transferring 25% of profits to general reserve. Similar type of companies earn at 10% of the value of their shares. Ascertain the value of shares of the company under:

(i) intrinsic value method;

(ii) yield value method; and

(iii) fair value method.

from the following particulars calculate the value of share of z ltd on yield basis 674622

From the following particulars calculate the value of share of Z Ltd. on yield basis:

Balance Sheet of Z Ltd. as on 31st March, 2011

Liabilities

Rs

Assets

Rs

 80,000 equity shares  

 

Land and buildings

5,00,000

 of `Rs.10 each  

8,00,000

Plant and machinery

6,00,000

 4,000 9% preference  

 

Patents

2,00,000

shares of `Rs.100 each

4,00,000

 Sundry debtors  

3,00,000

10% Debentures

2,00,000

 Work in progress  

 

Reserves

4,00,000

 and stock  

5,00,000

Sundry creditors

4,00,000

 Cash at bank  

1,00,000

 

22,00,000

 

22,00,000

Land and buildings to be valued at 9,00,000. The company’s earnings were as follows:

Year ended 31st March

Profits before tax Rs

Tax paid Rs

2007

3,00,000

80,000

2008

4,00,000

1,60,000

2009

1,00,000

40,000 (Strike) .

2010

5,00,000

2,30,000

2011

5,50,000

3,00,000

The company paid managerial remuneration of Rs.60,000 per annum but it will become Rs.1,00,000 in future. There has been no change in capital employed. The company paid dividend of 90 paise per share and it will maintain the same in future. The company proposes to build up a plant rehabilitation reserve. Dividend rate in this type of company is fluctuating and the asset backing of an equity share is about 1 1/2 times. The equity shares with an average dividend of 8% sell at par. (Tax rate is assumed to be 40%).

 

the aforesaid figures relate to a company which has rs 10 00 000 on equity shares of 674623

Year ended

31st March

Average net worth

(excluding investment)

Adjusted taxed

profit

 

Rs

Rs

2009

18,50,000

1,80,000

2010

21,20,000

2,00,000

2011

21,30,000

2,30,000

The aforesaid figures relate to a company which has Rs.10,00,000 on equity shares of Rs.100 each and Rs.3,00,000 in 9% preference shares of Rs.100 each. The company has investments worth Rs.2,50,000 (at market value) on the valuation date the yield in respect of which has been excluded in arriving at the adjusted tax profit figures. It is usual for similar type of companies to set aside 25% of the taxed profit for rehabilitation and replacement purposes. On the valuation day the net worth (excluding investment) amounts to Rs.22,00,000. The normal rate of return expected is 9%. The company paid dividends consistently within a range of 8 to 10% on equity shares over the previous seven years and the company expects to maintain the same. Compute the value of each equity share on the basis of productivity.

 

from the following balance sheet of m p products ltd find out the values of equity s 674624

From the following balance sheet of M.P. Products Ltd., find out the values of equity shares and preference shares:

Balance Sheet of M.P. Products Ltd.

Liabilities

Rs

Assets

Rs

20,000 equity shares of

2,00,000

Goodwill

25,000

Rs.10 each

 

Machinery

1,60,000

8% 1,000 preference shares

 

Furniture

5,000

of Rs.100 each

1,00,000

Stock

80,000

Reserves

30,000

Debtors

1,50,000

Profit and loss account

10,000

Cash

2,000

Sundry creditors

60,000

Preliminary expenses

3,000

Other liabilities

12,000

   

Overdraft

5,000

   

Proposed preference dividend

8,000

   
 

4,25,000

 

4,25,000

Goodwill is valued at Rs.15,000. Stock is overvalued by Rs.10,000. Machinery is undervalued by Rs.15,000.

 

the debenture interest is owing for six months and dividends on preference shares ar 674625

The following is the balance sheet of Mark Ltd., as 31st March, 2011

Liabilities

Rs

Assets

Rs

Share Capital :

 

Sundry Assets

5,48,000

10,000 12% Preference Shares

 

Preliminary Expenses

5,000

of Rs.10 each fully paid

1,00,000

Discount on

 

30,000 Equity Shares of Rs.10

 

debentures

2,000

each fully paid

3,00,000

Profit & Loss Account

35,000

General Reserve

10,000

   

Debenture redemption fund

20,000

   

Depreciation fund

15,000

   

10% Debentures

50,000

   

Sundry Creditors

95,000

   
 

5,90,000

 

5,90,000

The debenture interest is owing for six months and dividends on preference shares are in arrears for one year. Assuming the assets are worth their book values, show the approximate value of preference and equity shares if :

(i) Preference shares are preferential as to capital and arrears are payable in a winding up; and:

(ii) Preference shares are preferential as to capital but arrears of preference dividends are not payable.

compute the values of equity shares of companies a and b on the basis of dividend an 674626

Compute the values of equity shares of companies A and B on the basis of dividend and that of yield on capital employed. The following information is provided:

 

Company A

Company B

 

Rs

Rs

Profit per year

1,00,000

1,00,000

7 ½% Preference capital

2,00,000

6,00,000

Equity capital ( Rs.100 each)

8,00,000

4,00,000

Assume that all the profits were distributed. Market expectation is 10%.

 

you as auditor are required to fix the fair value rsquo of the shares of t ltd on 31 674627

You, as Auditor, are required to fix the `fair value’ of the shares of T Ltd., on 31st March, 2011. The company’s position was as follows:

Liabilities

Rs

Rs

Assets

Rs

Rs

Capital 5,000 shares  of

 

 

Bldgs. at cost

 

80,000

Rs.100 each

 

 

Furniture at cost

 

3,000

fully paid

 

5,00,000

Stock in trade

 

 

Reserve fund

 

1,50,000

at market value

 

4,50,000

Depreciation Funds:

 

 

Investment at Cost:

 

 

Buildings

10,000

 

G.P. Notes for

 

 

Investments

45,000

55,000

Rs.2,00,000

1,80,000

1,80,000

Creditors

 

48,000

Indian Gold Loan

 

 

Bad debts reserve

 

20,000

Repayable 2014

2,00,000

3,80,000

Profit and Loss:

 

 

 

 

 

Balance from 2009 10

80,000

 

Books debts consi

 

 

Profit for 2010 11

4,30,000 

5,10,000

dered good

 

3,00,000

(subject to tax of 40%)

 

 

Cash and bank balance

 

70,000

 

12,83,000

 

 

 

12,83,000

You are given the following information:

(1) The company’s prospects for 2012 12 are equally good.

(2) Its buildings are now worth Rs.3,50,000.

(3) Public companies doing similar business show a profit earning capacity of 15 per cent on market value of their shares.

(4) Profits for the past three years have shown an increase of Rs.50,000 annually.

(5) Investments yield 8% net on the book value on the whole.

 

depreciation fund is in excess by rs 54 000 of the amount of actual depreciation fin 674628

Liabilities

Rs

Assets

Rs

Capital:

 

Land (at cost)

2,21,000

8,800 shares of 250

 

Building (at cost)

11,73,000

each fully paid up

22,00,000

Machinery etc. (at cost)

20,58,000

Reserve fund

8,24,000

Furniture

5,000

Profit and Loss account

36,12,000

Investments (at market value)

17,00,000

Workmen’s savings account

2,27,000

Stock in trade

 

Provident fund account

54,000

(at market value)

26,00,000

Depreciation fund account

4,63,000

Book Debt

3,35,000

Creditors

38,86,000

Cash and other balances

31,49,000

   

Debenture charges

25,000

 

1,12,66,000

 

1,12,66,000

Depreciation fund is in excess by Rs.54,000 of the amount of actual depreciation. Find out the intrinsic value of the share.

 

it is provided in the articles of association that on the death of a shareholder his 674629

It is provided in the Articles of Association that on the death of a shareholder, his shares shall be purchased by the remaining shareholders at a price to be settled by the Auditors, on the basis of the last balance sheet.

It is further provided that for this purpose, goodwill was to be of the value of three years’ purchase of the average annual profits for the last four years. The last balance sheet is as follows:

Liabilities

Rs

Assets

Rs

Capital

 

Goodwill

1,00,000

20,000 shares of Rs.10

 

Investment at cost

 

each fully paid

2,00,000

(market value

 

Reserve

1,00,000

Rs.1,25,000)

1,50,000

Debentures

2,00,000

Stock at cost

2,50,000

Sundry creditors

1,50,000

Debtors

1,50,000

Profit and Loss Account

35,000

Cash

35,000

 

6,85,000

 

6,85,000

The profits for the last four years were (after tax) Rs.15,000, 20,000, 25,000 and 40,000 respectively.

You are required to state with details of working the price which should be paid per share.

 

during the year 2009 10 pragati ltd starts developing a new production process durin 674630

(a) During the Year 2009 10, Pragati Ltd. starts developing a new production process. During the year, expenditure incurred was Rs.20 lakhs, of which Rs.18 lakhs was incurred before 1st March, 2010 and 2 lakhs was incurred between 1st March, 2010 and 31st March, 2010. The company demonstrated that on 1st March, 2010 the production process met the criteria for recognition as an intangible asset. The recoverable amount of the know how embodied in the process (including future cash outflows to complete the process before it was available for use) was estimated to be 10 lakhs.

(i) What is the value of the intangible asset as on 31st March, 2010?

(ii) What amount will be treated as an expense?

(b) Continuing the problem mentioned in part (a) above, suppose during the year ended 31st March, 2011 additional expenditure incurred on the new production process was Rs.40 lakhs. On 31st March, 2011 the recoverable amount of the know how embodied in the process (including future cash outflows to complete the process before it was available for use) was estimated to be Rs.38 lakhs.

(i) What is the total cost of the production process on 31st March, 2011?

(ii) What is the impairment loss?

(iii) When can impairment loss be reversed in a subsequent period?

 

define and explain the terms a cost centre and b cost unit 674635

1. Define and explain the terms (a) cost Centre and (b) cost unit.

2. You have been asked to design a system of cost accounting for installation in a factory. Describe the essentials that should be considered before you design such a system.

3. Explain briefly the meaning, nature and scope of management accounting.

4. Discuss the importance and limitations of management accounting for managerial decision making.

5. Explain the tools and techniques of management accounting.

6. Distinguish between management accounting, cost accounting and financial accounting.

7. Management accounting is concerned with accounting information which is useful to management?. Comment.

8. Explain briefly the role of a management accountant.

9. What are the limitations of management accounting? How can these limitations be eliminated?

one parcel containing two important materials was received by a factory and the invo 674636

One parcel containing two important materials was received by a factory and the invoice pertaining to the same discloses the following information:

 

 

Rs

Material I 500 kgs. @ Rs.2.00 per kg.

 

1,000.00

Material II 600 kgs. @ Rs.1.60 per kg.

 

960.00

Insurance

 

39.20

Sales Tax

 

98.00

Freight etc.

 

55.00

Due to mishandling in the factory’s store a loss of 10 units of material I and 6 units of material II was noted. What rate would you adopt, for issuing these vital components to the jobs? Also give your changed rate, if a provision of 10% is to be kept for probable risk of obsolescence.

a consignment was received from a foreign supplier containing four types of material 674637

A consignment was received from a foreign supplier, containing four types of material. The invoice reveals the following:

 

Rs

Material A 2,000 kgs. @ Rs.2.00 per kg.

4,000.00

Material B 1,000 kgs. @ Rs.3.00 per kg.

3,000.00

Material C 1,500 kgs. @ Rs.4.00 per kg.

6,000.00

Material D 500 kgs. @ Rs.4.50 per kg.

2,250.00

Insurance

152.50

Freight paid by supplier

1,000.00

Expenses incurred by importer

1,600.00

Duty paid by the importer

610.00

Loss due to breakage was recorded as follows:

Matl. A 20 kgs.; Matl. B 20 kgs.; Matl. C 45 kgs.; and Matl. D 10 kgs.

Provision of 10% is made for evaporation and minor losses due to seasonal variations. Calculate the rate at which these should be issued.

ace ltd manufactures a product and the following particulars are collected for the y 674639

Ace Ltd. manufactures a product and the following particulars are collected for the year ended March, 2011:

Monthly demand (units)

250

Cost of placing an order ( )

100

Annual carrying cost ( per unit)

15

Normal usage (units per week)

50

Minimum usage (units per week)

25

Maximum usage (units per week)

75

Re order period (weeks)

4–6

You are required to calculate:

(i) Re order quantity

(ii) Re order level

(iii) Minimum level

(iv) Maximum level

(v) Average stock level.

the following is a summary of the receipts and issues of materials in a factory duri 674641

The following is a summary of the receipts and issues of materials in a factory during a month:

Date

 

1

Opening balance 500 units at Rs.25 per unit.

3

Issue 70 units

4

Issue 100 units

8

Issue 80 units

13

Received 200 units @ Rs.24.50 per unit.

14

Returned to store 15 units @ Rs.24 per unit

16

Issue 180 units

20

Received 240 units @ Rs.24.75 per unit

24

Issue 304 units

25

Received 320 units @ Rs.24.50 per unit

26

Issue 112 units

27

Returned to store 12 units @ Rs.24.50 per unit

28

Received 100 units at Rs.25 per unit.

Work out on the basis of „FIFO?. It was revealed that on 15th there was a shortage of 5 units and on 27th of 8 units.

from the following you are required to prepare a statement showing the issues made u 674642

From the following you are required to prepare a statement showing the issues made under LIFO method:

Date

Opening Balance 100 units at 10 each

1

Received 200 units at Rs.10.50 each

2

Received 300 units at Rs.10.60 each

4

Issued 400 units to Job A vide MR No. 3

6

Issued 120 units to Job B vide MR No. 4

7

Received 400 units at Rs.11 each

10

Issued 200 units to Job C vide MR No. 5

12

Received 300 units at Rs.11.40 each

13

Received 200 units at Rs.11.50 each

15

Issued 400 units to Job D vide MR No. 6

prepare a statement showing the pricing of issues on the basis of a simple average a 674643

Prepare a statement showing the pricing of issues, on the basis of (a) Simple Average, and (b) Weighted Average Methods from the following information pertaining to material „X?.

Date

 

1

Purchased 100 units @ Rs.10.00 each.

2

Purchased 200 units @ Rs.10.20 each.

5

Issued 250 units to Job A vide MR No. 1

7

Purchased 300 units @ Rs.10.50 each

10

Purchased 200 units @ Rs.10.80 each

13

Issued 200 units to Job B vide MR No. 2

18

Issued 200 units to Job C vide MR No. 3

20

Purchased 100 units @ Rs.11.00 each.

25

Issued 150 units to Job D vide MR No.4.

purchased 500 units rs 65 each 674644

Date  

 

 

 2 April  

 Purchased 100 units @  Rs.55 each

 6 April  

 Issued 400 units  

 

 10 April  

 Purchased 600 units @  Rs.60 each

 13 April  

 Issued 500 units  

 

 20 April  

 Purchased 500 units @  Rs.65 each

 25 April  

 Issued 600 units  

 

 10 May  

 Purchased 800 units @  Rs.70 each

 12 May  

 Issued 500 units  

 

 13 May  

 Issued 200 units  

 

 15 May  

 Purchased 500 units @  Rs.75 each

 12 June  

 Issued 400 units  

 

 15 June  

 Purchased 300 units @  Rs.80 each

prepare stores ledger account showing pricing of material issues on replacement pric 674645

Prepare Stores Ledger Account showing pricing of material issues on Replacement Price basis, from the following particulars.

Opening Balance 400 units @ Rs.4 each.

10th March  

 Received 100 units @ Rs.4.10 each.  

 15th March  

 Issued 300 units to job XY vide M.R. No. 1  

 17th March  

 Received 200 units @ Rs.4.30 each.  

 20th March  

 Issued 250 units to job AB vide M.R. No. 2.  

 25th March  

 Received 400 units @ Rs.4.50 each.  

 26th March  

 Issued 200 units to job JK vide M.R. No. 3.  

 27th March  

 Received 100 units @ Rs.4.60 each.  

 30th March  

 Issued 300 units to job PQ vide M.R. N  

Replacement price on various dates: 15th March Rs.4.20; 20th March Rs.4.40; 26th March Rs.4.60 and 30th March Rs.4.80.

stocks are issued at standard price and the following transactions occurred in a spe 674646

Stocks are issued at standard price and the following transactions occurred in a specific material :

Date

     

April

     

1

 Stock  

 10 tons  

at Rs.240 per ton

4

 Purchased  

 5 tons  

at Rs.260 per ton

5

 Issued  

 3 tons  

 

12

 Issued  

 4 tons  

 

13

 Purchased  

 3 tons  

at Rs.250 per ton

19

 Issued  

 4 tons  

 

26

 Issued  

 3 tons  

 

30

 Purchased  

 4 tons  

at Rs.280 per ton

30

 Issued  

 3 tons  

 

The debit balance of price variation on 1st April was Rs.20. Show the stock account for the material for the month of April indicating how would you deal with the difference in material price variance, while preparing the Profit and Loss Account for the month.

a company incorporated on 1st may 2010 acquired a business as a going concern with e 674585

A company, incorporated on 1st May, 2010 acquired a business as a going concern with effect from 1st January, 2010. The first accounts were drawn up to September 30, 2010. The gross profit is Rs.2,24,000. The general expenses are Rs.56,880, directors remuneration Rs.4,000 p.m.; formation expenses amounted to Rs.6,000, rent which till June 30, 2010 was 400 p.m. was increased to Rs.12,000 per annum from July 1, 2010.

The manager of the earlier firm whose salary was Rs.2,000 p.m. was made as director upon the incorporation and his remuneration thereafter is included in the figure of Directors‘ remuneration given earlier. Prepare Profit and Loss Account for the period and find out the profits available for dividends and the profit prior to incorporation.

vijay ltd was incorporated on 1st march 2011 and received its certificate of commenc 674586

Vijay Ltd. was incorporated on 1st March, 2011 and received its certificate of commencement of business on 1st April, 2011. The company bought the business of M/s Small and Co. with effect from 1st November, 2010. From the following figures relating to the year ending October, 2011, find out the profit available for dividends:

(i) Sales for the year were Rs.6,00,000 out of which sales upto 1st March were Rs.2,50,000.

(ii) Gross profit for the year was Rs.1,80,000.

(iii) Expenses debited to the Profit and Loss account were:

 

RS

 Rent  

 9,000  

 Salaries  

 15,000  

 Directors‘ fees  

 4,800  

 Interest on debentures  

 5,000  

 Audit fees  

 1,500  

 Discount on sales  

 3,600  

 Depreciation  

 24,000  

 General expenses  

 4,800  

 Advertising  

 18,000  

 Stationery and printing  

 3,600  

 Commission on sales  

 6,000  

 Bad debts  

 1,500*  

 Interest to vendor on purchase consideration upto May 1, 2011  

 3,000  

*Rs.500 relates to debts created prior to incorporation.

the trial balance of simplex ltd as at 31st march 2011 shows the following items 674593

The Trial Balance of Simplex Ltd. as at 31st March, 2011 shows the following items:

 

Dr.

Cr.

 

Rs

Rs

Provision for Income tax Account (1.4.2010)

 

60,000

Advance Payment of Tax Account

1,10,000

 

You are also given the following information:

(1) Advance Payment of Tax Account includes Rs.70,000 for 2009 10.

(2) Actual tax liability for 2009 10 amounts to Rs.76,000 and no effect for the same has so far been given in accounts.

(3) Provision for Income tax has to be made for 2010 11 for Rs80,000. Prepare the various ledger accounts involved and also show how the relevant items will appear in the balance sheet of the company.

calculation of maximum remuneration where there is a managing director 674596

(Calculation of maximum remuneration where there is a managing director)

From the following particulars of G.G. Ltd. calculate the maximum remuneration payable to the managing director and other part time directors of the company:

 

Rs

 Net Profit before provision for income tax and managerial  

 

 remuneration, but after depreciation and provision for repairs  

 86,84,100  

 Depreciation provided in the books  

 32,00,000  

 Repairs for machinery provided for during the year  

 2,50,000  

 Actual expenditure incurred on repairs during the year  

 1,50,000  

slow and steady ltd has a manager who is entitled to get a monthly salary of rs 25 0 674597

Slow and Steady Ltd. has a manager who is entitled to get a monthly salary of Rs.25,000 per month and in addition to receive a commission of 1% of the net profits of the company before such salary or commission. The following is the Profit and Loss Account of the company for the year ended 31st March, 2011:

 

Rs

 

Rs

 To Staff Salaries 

 1,920 

 By Gross Profit b/d 

9,650

 To General Expenses 

 885 

 By Subsidy from the Government

600

 To Depreciation 

 820 

 By Profit on sale on land 

200

 To Manager‘s Salary 

 300 

 By Profit on sale of  machinery (cost  2,50,000 and written down value 1,80,000)

50

 To Commission to the Manager (on account)

 60 

   

 To Provision for Bad Debts 

 75 

   

 To Provision for income tax 

 1,800 

   

 To Proposed Dividends 

 1600 

 

 

 To Balance c/d 

 3,040 

 

 

 

 10,500

 

10,500 

Calculate the maximum remuneration payable to the Manager.

calculation of managerial remuneration on net profits after charging such remunerat 674598

1. (Calculation of managerial remuneration on net profits after charging such remuneration) Taking Illustration No. 6 above, suppose the manager is entitled to receive a commission of 1% of the net profits of the company after charging his salary and commission. Calculate the maximum remuneration payable to the manager.

2. Due to inadequacy of profit during the year, the company proposes to declare dividend out of the general reserves. From the following particulars, you are to ascertain the amount that can be drawn applying the Companies (Declaration of Dividend out of Reserves) Rules, 1975:

 

Rs

 (a) 17,500 9% preference shares of 100 each fully paid  

 17,50,000  

 (b) 7,00,000 equity shares of 10 each fully paid  

 70,00,000  

 (c) General reserves  

 21,00,000  

 (d) Capital reserves on revaluation of assets  

 3,50,000  

 (e) Share premium  

 3,50,000  

 (f) Profit and loss account credit balance  

 63,000  

 (g) Net profit for the year  

 3,57,000  

Average rate of dividend during the last five years: 15%.

below is the balance sheet of the excellent ltd as at 31st march 2011 674599

Below is the Balance Sheet of the Excellent Ltd. as at 31st March, 2011.

Liabilities  

Rs

 Assets  

Rs

 Share Capital:  

 

 Fixed Assets:  

 

 Authorised  

 15,00,000  

 Plant and Machinery  

 6,00,000  

 Issued and Subscribed:  

 

 Furniture and Fittings  

 75,000  

 2,000 8% Redeemable  

 

 Investments  

 1,00,000  

 Preference Shares of  

 

 

 

 100 each, fully  

 

 Current Assets, Loans  

 

 paid up  

 2,00,000  

 and Advances:  

 

 50,000 Equity Shares  

 

 (a) Current Assets  

 

 of 10 each fully  

 

 Stock  

 1,70,000  

 paid up  

 5,00,000  

 Debtors  

 65,000  

 Reserves and Surplus:  

 

 Cash at Bank  

 80,000  

 Capital Reserve  

 25,000  

 (b) Loans and Advances:  

 Nil  

 Securities Premium  

 30,000  

 

 

 General reserve  

 1,60,000  

   

 Profit and Loss Account  

 1,38,000  

   

 Current Liabilities and Provisions:

     

(A) Current liabilities: Sundry Creditors

37,000

   

(B) Provisions

     
 

10,90,000

 

10,90,000

On this date the company decided to redeem its preference shares at a premium of 5%. For this purpose, investments were sold for Rs.1,10,000 and Rs.10,000 equity shares of Rs.10 each were issued at a premium of Rs.1 each.

After redemption of preference shares, the company issued one bonus share of Rs.10 each for every five equity shares held.

Pass journal entries to record the above mentioned transactions and prepare the balance sheet as it would appear thereafter.

the following balances appeared in the books of the moon light co ltd as on 31st mar 674600

The following balances appeared in the books of the Moon Light Co. Ltd. as on 31st March, 2011:

 

Dr.

Cr.

Issued, Subscribed and paid up Capital:

 width=

 width=

60,00,000 Equity Shares of 100 each

 

6,00,000

General Reserve

 

2,50,000

Unclaimed Dividend

 

6,526

Trade Creditors

 

36,858

Buildings at cost

1,50,000

 

Purchases

5,00,903

 

Sales

 

10,83,947

Manufacturing Expenses

3,59,000

 

Establishment Charges

26,814

 

General Charges

31,078

 

Machinery at Cost

2,00,000

 

Motor Vehicle at Cost

30,000

 

Furniture at Cost

5,000

 

Opening Stock

1,72,058

 

Book Debts

2,23,380

 

Investments

2,88,950

 

Depreciation Reserve

 

71,000

Advance Payment of Income tax

50,000

 

Cash Balance

72,240

 

Directors‘ Fees

1,800

 

Interest on Investment

 

8,544

Profit and Loss Account

 

 

1st April, 2010

 

16,848

Staff Provident Fund

 

37,500

 

21,11,223

21,11,223

From these balances and the following information, prepare the Company‘s Balance Sheet as on 31st March, 2011 and its Profit and Loss Account for the year ended on that date:

(a) The stocks on 31st March, 2011 were valued at Rs.1,48,680 thousand.

(b) Provided Rs.10,000 thousand for depreciation on fixed assets, Rs.1,800 thousand for Managing Director‘s remuneration and Rs.6,200 thousand for the company‘s contribution to the Staff Provident Fund.

(c) Interest accrued on investment amounted to Rs.2,750 thousand.

(d) A provision of 50,000 thousand for taxes in respect of the profit for 2010 11 considered necessary.

(e) The directors propose a final dividend @ 8% after transfer to General Reserve Rs.30,000 thousand.

(f) A claim of Rs.2,500 thousand for workmen‘s compensation is being disputed by the company.

(g) The market value of investments as on 31.3.2011 amounts to Rs.3,02,500 thousand.

mutual engineers ltd have authorised capital of rs 50 lakhs divided into 5 00 000 eq 674601

Mutual Engineers Ltd. have authorised capital of Rs.50 lakhs, divided into 5,00,000 equity shares of Rs.10 each. Their books show the following balances as on 3 1.3.2011:

 

Rs

 

Rs

Stock 1.4.2010

6,65,000

Bank current account

20,000

Discount and rebates

30,000

Cash in hand

8,000

Carriage inwards

57,500

Debenture interest

 

Patterns

3,75,000

(for ½ year to 30.9.2010)

20,000

Rates, taxes and insurance

55,000

Interest banks (Dr.)

91,000

Furniture and fixtures

1,50,000

Preliminary expenses

10,000

Materials purchased

12,32,500

Calls in arrears

10,000

Wages

13,05,000

Equity share capital

 

Coal and coke

63,000

(2,00,000 share of

 

Freehold land

12,50,000

10 each)

20,00,000

Plant and machinery

7,50,000

8% Debentures

5,00,000

Engineering tools

1,50,000

Bank overdraft

7,57,000

Goodwill

3,75,000

Sundry creditors (for goods)

2,40,500

Sundry debtors

2,66,000

Sales

36,17,000

Bills receivable

1,34,500

Rents (Cr.)

30,000

Advertisement

15,000

Transfer fees

6,500

Commission and brokerage

67,500

Profit and Loss A/c (Cr.)

67,000

Business expenses

56,000

 

 

Repairs

46,500

 

 

Bad debts

25,500

 

 

The stock (valued at cost or market value whichever is lower) as on 31.3.2011 was Rs.7,08,000. Outstanding liability for wages Rs.25,000 and business expenses Rs.25,000.

Dividend declared @ 8% on paid up capital.

To charge depreciation: Plant and Machinery @ 15% Engineering Tools @ 20%, Patterns @ 10% and furniture and fixture @ 10%. Provide 2% on debtors as doubtful debts after writing off Rs.21,500 as bad debts. Write off preliminary expenses Rs.5,000 and create Debenture Redemption Reserve Rs.50,000. Provide Rs.1,30,000 for income tax.

Prepare Profit and Loss Account for the year ended 31.3.2011 and Balance Sheet, as on that date, in accordance with the Companies Act, 1956, giving as much information as necessary. Ignore previous year‘s figures.

the following balances have been extracted from the books of arts and crafts limited 674602

The following balances have been extracted from the books of Arts and Crafts Limited as on 31st March, 2011:

 

Rs

 

Rs

 Freehold land  

 28,000  

Income from Investment

1,200

 Buildings  

 7,500  

Provision for doubtful

 

 Furniture  

 2,000  

debts (1st April, 2010)

 200  

 Debtors  

 10,000  

Creditors

 2,000  

 Stock (31st March, 2011)  

 4,000  

 Provision for depreciation:  

 

 Cash at bank  

 500  

 (1st April, 2010)  

 

 Cash in hand  

 100  

 Building  

 500  

 Cost of goods sold  

 30,000  

 Furniture  

 400  

 Salaries and wages  

 1,500  

 Suspense  

 250  

 Miscellaneous expenses  

 800  

 Equity shares capital  

 36,750  

 Investment in shares  

 18,000  

 6% Cumulative pref. share  

 

 Interest  

 300  

 Capital  

 8,000  

 Bad debts  

 100  

 Securities Premium  

 1,000  

 Repairs and maintenance  

 150  

 Bank Overdraft  

 5,000  

 Advance payment of Income tax  

 600  

 Sales  

 48,000  

   

 Profit and Loss A/c (1st April, 2010)  

250

 

1,03,550

 

1,03,550

The following further particulars are available:

(1) The land was revalued on 1st January, 2011 at Rs.3, 50, 00,000 by an expert valuer, but no effect has been given in the books although the Directors have decided to adjust the relevant amount.

(2) Provision for doubtful debt is to be adjusted to 2.5% on the amount of debtors.

(3) Equity share capital is composed of Rs.10 shares, 36,40,000 fully paid and 50,000 on which final call of Rs.3 remains unpaid.

(4) Suspense amount represents money received from the new allottee for reissue of 50,000 shares forfeited during the year for non payment of the final call, but no entry for adjustment thereof has been passed.

(5) Provision for taxation is to be made at 40%.

(6) Market value of investments was Rs.1,85,00,000 on 31st March, 2011.

(7) The company is managed by the Directors who are entitled to a remuneration of 3% on the annual net profits.

(8) Depreciation to be charged on written down value of:

Building at 5%.

Furniture at 10%.

(9) The land and buildings of the company are mortgaged in favour of the bank as security for overdraft sanctioned up to a limit of 2,50,00,000.

(10) Dividend on cumulative preference shares were in arrears for 5 years upto 31st March, 2011. The directors have recommended payment of dividend for two years.

You are required to prepare the Profit and Loss Account for the year ended 31st March, 2011 and a Balance Sheet as on that date. Ignore previous year‘s figures.

the balance sheet of the h ltd and s ltd as on 31st march 2011 are given below 674606

The Balance Sheet of the H Ltd. and S Ltd. as on 31st March, 2011 are given below:

Balance Sheet

Liabilities  

H Ltd.

S Ltd.

Assets

H Ltd.

S Ltd.

 

Rs

Rs

 

Rs

Rs

 Share Capital:  

   

Sundry Assets

5,55,000

2,48,000

 Shares of  

   

Investments

   

 100 each  

6,00,000

2,00,000

Shares in S Ltd.

 

 

 Profit and Loss Account  

80,000

(at cost)

2,00,000

 Sundry Creditors  

75,000

48,000

     
 

7,55,000

2,48,000

 

7,55,000

2,48,000

Prepare the consolidated balance sheet of the holding company and its subsidiary as on 31st March, 2011.

the balance sheet of h ltd and s ltd as on 31st march 2011 are given below 674607

The Balance Sheet of H Ltd. and S Ltd. as on 31st March, 2011 are given below:

Balance Sheet

Liabilities  

H Ltd.

S Ltd.

Assets

H Ltd.

S Ltd.

 

Rs

Rs

 

Rs

Rs

 Share Capital:  

   

Sundry Assets

6,05,000

2,48,000

 Shares of  

   

Investments

   

 100 each  

6,00,000

2,00,000

1,500 shares in

 

 

 Profit and Loss Account  

80,000

S Ltd. (at cost)

1,50,000

 Sundry Creditors  

75,000

48,000

     
 

7,55,000

2,48,000

 

7,55,000

2,48,000

Prepare the consolidated balance sheet of H Ltd. and S Ltd. as on 31st March, 2011.

h ltd acquired shares in s ltd on 31st march 2011 prepare the consolidated balance s 674608

Liabilities  

H Ltd.

S Ltd.

Assets

H Ltd.

S Ltd.

 

Rs

Rs

 

Rs

Rs

 Share Capital: Equity shares of

   

Sundry Assets

6,55,000

2,88,000

Rs.100 each

6,00,000  

2,00,000

Investments:

   

 General Reserve  

 60,000  

25,000

1600 shares of

 

 

 Profit and Loss  

   

 Rs.100 each  

1,60,000

 Account  

80,000

15,000

     

 Creditors  

75,000

48,000

     
 

8,15,000

2,88,000

 

8,15,000

2,88,000

H Ltd. acquired shares in S Ltd. on 31st March, 2011. Prepare the Consolidated balance sheet of H Ltd. and S Ltd. as on that date.

the balance sheets of h ltd and s ltd as on 31st march 2011 are given below 674609

The Balance Sheets of H Ltd. and S Ltd. as on 31st March 2011 are given below :

Liabilities  

H Ltd.

S Ltd.

Assets

H Ltd.

S Ltd.

 

Rs

Rs

 

Rs

Rs

 Share Capital:  

   

 Sundry Assets  

5,91,000

3,18,000

 Shares of  

   

 Investments:  

   

 Rs.100 each  

 6,00,000  

 2,00,000  

 1,600 shares of  

 

 

 General Reserve  

 60,000  

 40,000  

 Rs.100 each  

2,24,000

 Profit & Loss Account  

 80,000  

 30,000  

     

 Creditors  

 75,000  

 48,000  

     
 

 8,15,000  

 3,18,000  

 

 8,15,000  

 3,18,000  

H Ltd. acquired the shares in S Ltd. on 31st March 2,011. The plant worth book value of Rs.60,000 included in sundry assets of S Ltd. was re valued at Rs.50,000 on this date.

Prepare the consolidated balance sheets of H Ltd. and S Ltd. as on that date.

you are required to prepare the consolidated balance sheet of h ltd with its subsidi 674612

Balance Sheet as on 31st March, 2011

Liabilities

H Ltd.

S Ltd.

 

Rs

Rs

 Share capital:  

 

 

 6% preference shares of 10 each

 —  

 1,60,000  

 Equity shares of 10 each

 6,00,000  

 2,00,000  

 General reserve  

 1,00,000  

80,000

 Profit and loss account  

 2,00,000  

90,000

 6% Debentures of  10 each

 —  

40,000

 Proposed dividend:  

 

 

 on equity shares  

60,000

20,000

 on preference shares  

 —  

9,600

 Debentures interest accrued  

 —  

2,400

 Sundry creditors  

 2,94,000  

 1,25,000  

 

 12,54,000  

 7,27,000  

Assets

   

 Fixed assets  

 5,00,000  

 4,40,000  

 15,000 equity shares in S Ltd.  

 3,30,000  

 —  

 12,000 preference shares in S Ltd.  

 1,20,000  

 —  

 1,000 6% debentures in S Ltd.  

10,000

 —  

 Current assets  

 2,94,000  

 2,87,000  

 

 12,54,000  

 7,27,000  

Other information is as under:

(i) The general reserve of S Ltd. as on 1.4.2010 was Rs.80,000.

(ii) H Ltd. acquired the shares in S Ltd. on 1.4.2010

(iii) The balance of profit and loss account of S Ltd. is made up as follows:

Balance as on 1.4.2010  

 56,000  

 Net profit for the year ended 31.3.2011  

 63,600  

 

 1,19,600  

Less: Provision for proposed dividend

 29,600  

 

 90,000  

(iv) The balance of profit and loss account of S Ltd. as on 1.4.2010 is after providing for preference dividend of Rs.9,600 and proposed dividend of Rs.10,000 both of which were subsequently paid and credited to profit and loss account of H Ltd.

(v) No entries have been made in the books of H Ltd. for debentures interest due from or proposed dividend of S Ltd. for the year ended on 31.3.2011.

(vi) S Ltd. has issued fully paid bonus shares of Rs.40,000 on 31.3.2011 among the existing shareholders by drawing upon the general reserve. The transaction has not been given effect to in the books of S Ltd.

You are required to prepare the consolidated balance sheet of H Ltd. with its subsidiary S Ltd. as on 31st March, 2011.

sun ltd owns 80 of issued capital of moon ltd and 90 of issued capital of star ltd t 674613

(More than one subsidiary company)

Sun Ltd. owns 80% of issued capital of Moon Ltd. and 90% of issued capital of Star Ltd. The following are the balances of all the companies as on 31.3.2011.

 

Sun Ltd.

Moon Ltd.

Star Ltd.

 

Rs

Rs

Rs

 Fixed Assets  

 1,70,000  

10,000

27,000

 Less: Provision for Depreciation  

70,000

6,000

9,900

 

 1,00,000  

4,000

18,000

 Current Assets  

 2,68,000  

50,000

50,000

 Investments  

 

 

 

 Shares in Moon Ltd.  

15,000

 

 

 Shares in Star Ltd.  

25,000

 

 

 Current Accounts:  

 

 

 

 Moon Ltd.  

20,000

 

 

 Star Ltd.  

20,000

   

 

 4,48,000  

54,000

68,000

 Share Capital  

 3,20,000  

20,000

25,000

 Current Liabilities  

40,000

6,000

10,000

 Current Accounts  

22,000

18,000

 Proposed Dividend  

20,000

2,500

 Revenue Reserve  

68,000

6,000

12,500

 

 4,48,000  

54,000

68,000

1. At the time of acquiring the shares the subsidiaries had the following Revenue Reserves:

Moon Ltd. Rs.6,000

Star Ltd. Rs.3,000

2. Neither of the subsidiaries has paid any dividend since acquisition of shares.

3. Payment of creditors of Moon Ltd. by Sun Ltd. to the extent of Rs.2,000 has not been considered in the books of Moon Ltd.

4. A remittance of Rs.2,000 by Star Ltd. to Sun Ltd. has not yet been adjusted in the books of Sun Ltd.

5. The Stock of Moon Ltd. includes Rs.3,000 purchased from Sun Ltd. which made 25% profit on cost. Sun Ltd.’s stock includes Rs.5,000 purchased from Star Ltd.’s which made 20% profit on sales.

Prepare the consolidated Balance Sheet of Sun Ltd. and its subsidiaries — Moon Ltd. and Star Ltd.

the following are the profit and loss accounts of h ltd and s ltd for the year ended 674614

The following are the Profit and Loss Accounts of H Ltd. and S Ltd. for the year ended 31st March, 2011.

 

H Ltd.

S Ltd.

 

H Ltd.

S Ltd.

 

Rs

Rs

 

Rs

Rs

To Opening Stock

1,00,000

By Sales

8,00,000

6,50,000

To Purchases

5,00,000

4,00,000

By Closing Stock

1,50,000

1,00,000

To Productive Wages

1,50,000

1,00,000

     

To Gross Profit c/d

2,00,000

2,50,000

     
 

9,50,000

7,50,000

 

9,50,000

7,50,000

 To Sundry Expenses To Debenture Interest 

75,000

1,00,000

 By Gross Profit b/d 

2,00,000

2,50,000

 To Provision for 

6,000

 By Debenture 

   

 Taxation 

60,000

70,000

 Interest 

3,000

 To Profit c/d 

68,000

74,000

     
 

2,03,000

2,50,000

 

2,03,000

2,50,000

 To Preference Dividend 

3,000

By Profit b/d

68,000

74,000

 To Proposed Dividend

20,000

20,000

     

To Tax on Distributed

         

 Profit @ 15%* 

3,000

3,450

     

 To Balance c/d 

45,000

47,550

     
 

68,000

74,000

 

68,000

74,000

You are also given the following additional information:

(1) H Ltd. holds 1,500 equity shares of Rs.100 each in S Ltd. whose capital consists of 2,000 equity shares of Rs.100 each and 6% 500 cumulative preference shares of Rs.100 each. S Ltd. has also issued 6% Debentures of Rs.1,00,000 out of which H Ltd. holds Rs.50,000.

(2) The shares in S Ltd. were acquired by H Ltd. on 1st July, 2010 but the debentures were acquired on 1st April, 2010. S Ltd. was incorporated on 1st April, 2 010.

(3) During the year S Ltd. sold to H Ltd. goods costing Rs.50,000 at the selling price of Rs.75,000. One fourth of the goods manufactured remained unsold on 31st March, 2011. The goods were valued at cost to the holding company for closing stock purposes.

Prepare a consolidated profit and loss account.

prepare a consolidated balance sheet as on 31st march 2011 workings will be part of 674616

Balance Sheet of Harry Co. Ltd. as on 31st March, 2011

Liabilities

Rs

Assets

Rs

 Capital  

 1,00,000  

 Goodwill  

20,000

 General Reserve  

34,000

 Land and Buildings  

76,000

 Profit and Loss A/c  

11,100

 Investments*  

28,800

 Bills Payable  

41,000

 Stock  

52,000

 Sundry Creditors  

63,900

 Sundry Debtors and Advances  

 

 

 

 (including loan to Suman  

 

 

 

 Ltd.: Rs.1000)  

58,000

 

 

 Cash and bank  

15,200

 

 2,50,000  

 

 2,50,000  

The investment consists of 2,400 shares of Rs.10 each fully paid in its subsidiary Suman Ltd. which was acquired on 1st July, 2010.

Balance Sheet of Suman Ltd. as on 31st March, 2011

Liabilities

Rs

Assets

Rs

Capital (d)

30,000

Goodwill

4,400

General Reserve

5,000

Plant and Machinery (a)

29,000

Profit and Loss A/c (e)

4,400

Stock

6,000

Loan

21,000

Sundry Debtors

12,500

Sundry Creditors

28,400

Bills Receivable (b)

30,000

 

 

Cash and Bank (c)

6,900

 

88,000

 

88,000

(a) On 1st April, 2010 the plant and machinery were revalued at Rs.32,000 which should be taken in the consolidated balance sheet. Ignore depreciation. There were no additions or deletions to plant during the year.

(b) Total bills receivable were Rs.41,000 (all accepted by Harry Co. Ltd.) of which bills of Rs.11,000 had been discounted with the banker and yet to be matured.

(c) Cash and bank balances were arrived at after sending a cheque for Rs.1,000 to Harry Co. Ltd., on account of repayment of loan.

(d) Capital account of Suman Ltd. consists of 3,000 ordinary shares of Rs.10 each.

(e) Balances as on 1st April, 2010

Profit and Loss A/c

Rs.1,200

General Reserve

Rs.4,000

Prepare a consolidated balance sheet as on 31st March, 2011. Workings will be part of your answer.

from the following balance sheets of vipul ltd and its subsidiary vedika ltd as on 3 674619

From the following balance sheets of Vipul Ltd. and its subsidiary Vedika Ltd. as on 31st March, 2011 and the additional information provided thereafter, prepare the consolidated balance sheet of the two companies as on that date:

 

Vipul Ltd.

Vedika Ltd.

Liabilities

Rs

Rs

 Share capital:  

   

 Shares of 10 each fully paid  

10,00,000

2,00,000

 General reserve  

3,10,000

––

Profit and loss account

1,50,000

40,000

Creditors

2,30,000

69,000

 

16,90,000

3,09,000

Assets

   

Fixed assets

   

70% Shares of Vedika Ltd. (at cost)

11,62,000

1,80,000

Current assets

1,42,000

––

Discount on issue of shares

3,86,000

1,24,000

 

5,000

 

16,90,000

3,09,000

On 31st December, 2010 Vipul Ltd. acquired the shares in Vedika Ltd. On 1st April, 2010, Vedika Ltd.’s profit and loss account showed a debit balance of Rs.8,000. On 31st March, 2011, Vedika Ltd. decided to revalue its fixed assets at Rs.2,00,000.

from the following figures calculate the value of a share of rs 10 on i dividend bas 674620

From the following figures calculate the value of a share of Rs.10 on (i) dividend basis, and (ii) return on capital employed basis, the market expectation being 12% .

Year ended

Capital Employed

Profit

Dividend (%)

31st March

Rs

Rs

 

2008

5,00,000

80,000

12

2009

8,00,000

1,60,000

15

2010

10,00,000

2,20,000

18

2011

15,00,000

3,75,000

20

give journal entries for the forfeiture and re issue of shares in the following case 674531

Give journal entries for the forfeiture and re issue of shares in the following cases:

(a) P Ltd. forfeited 300 shares of Rs.10 each, fully called up for non payment of final call money of Rs.4 per share. These shares were subsequently re issued by the company for Rs.10 per share as fully paid up.

(b) Q Ltd. forfeited 300 shares of Rs.10 each, fully called up for non payment of final call money of Rs.4 per share. These shares were subsequently re issued by the company for Rs.12 per share as fully paid up.

(c) R Ltd. forfeited 200 shares of Rs.10 each, Rs.8 per share being called up on which a shareholder paid application and allotment money of Rs.5 per share but did not pay the first call money of Rs.3 per share. Of these forfeited shares, 150 shares were subsequently re issued by the company as fully paid up for Rs.Rs.8 per share.

(d) S Ltd. forfeited 100 shares of Rs.10 each, Rs.8 per share being called up, which were issued at a discount of 10% for non payment of first call money of Rs.3 per share. Of these forfeited shares, 80 shares were subsequently reissued by the company at Rs.5 as Rs.8 paid up.

a limited company issued a prospectus inviting applications for 2 000 shares of rs r 674532

A limited company issued a prospectus inviting applications for 2,000 shares of Rs.Rs.10 each at a premium of Rs.Rs.2 per share payable as follows:

On Application

Rs.2

On Allotment

Rs.5 (including premium)

On First Call

Rs.3

On Second and Final Call

Rs.2

Applications were received for 3,000 shares and allotment was made pro rata to the applicants of 2,400 shares. Money overpaid on applications was employed on account of sum due on allotment.

Ramesh, to whom 40 shares were allotted, failed to pay the allotment money and on his subsequent failure to pay the first call, his shares were forfeited. Mohan, the holder of 60 shares failed to pay the two calls and his shares were forfeited after the second and final call.

Of the shares forfeited, 80 shares were sold to Krishna credited as fully paid for Rs.9 per share, the whole of Ramesh’s share being included.

Show journal and cash book entries and the Balance Sheet.

varun ltd was registered with a nominal capital of rs 20 00 000 in equity shares of 674533

Varun Ltd. was registered with a nominal capital of Rs.20,00,000 in equity shares of Rs.100 each. 10,000 of these shares were issued to the public at a premium of Rs.20 per share, payable as to Rs.20 on application, Rs.45 on allotment including premium, Rs.25 on first call and the balance on final call. Applications were received for 13,000 shares and allotment was made pro rata to the applicants of 12,000 shares. Money over paid on application was employed on account of sums due on allotment. Sudhir holding 200 shares paid the whole of the amount due on first call along with the allotment but failed to pay final call. Pradeep holding 300 shares failed to pay the two calls and Joshi holding 400 shares failed to pay the final call.

All these shares were forfeited after the final call. Of the shares forfeited, 200 shares belonging to Pradeep and 200 shares belonging to Joshi were sold to Rajesh as fully paid for Rs.90 per share.

Show the journal entries and cash book in the books Varun Ltd.

the balance sheet of power link ltd as on 31st march 2011 is as follows 674535

The balance sheet of Power link Ltd. as on 31st March, 2011 is as follows:

Liabilities

 

 

Assets

Share capital:

 

Fixed Assets

66,00,000

5,00,000 equity shares of

 

Investments

18,00,000

10 each, fully paid

50,00,000

Stock

11,87,000

General Reserve

6,50,000

Sundry debtors

9,60,000

Securities Premium

5,40,000

Cash and bank balance

7,10,000

Profit and Loss Account

3,75,000

 

 

12% Debentures

25,00,000

 

 

Term Loans

13,25,000

 

 

Sundry Creditors

7,42,000

 

 

Provision for tax

1,25,000

 

 

 

1,12,57,000

 

1,12,57,000

The shareholders adopted the resolution on the date of the abovementioned balance sheet to:

(i) buy back 20% of the paid up capital @ Rs.15 each.

(ii) issue 13% debentures of Rs.5,00,000 at a premium of 10% to finance the buy back of shares.

(iii) maintain a balance of Rs.3,00,000 in general reserve account, and

(iv) sell investments worth Rs.8,00,000 for Rs.6,50,000.

Pass necessary journal entries to record the above transactions and prepare the balance sheet immediately after the buy back.

the following is the balance sheet of oscar india ltd as on 31st march 201 1 674538

The following is the balance sheet of Oscar India Ltd. as on 31st March 201 1:

Liabilities

.

Assets

Preference share capital:

 

Fixed assets

6,00,000

2,500 shares of Rs.100

 

Investment

50,000

each fully called up

2,50,000

Bank

90,000

Less: Final call @ Rs.20

 

 

 

per share unpaid

2,000

 

 

 

2,48,000

 

 

Equity share capital:

 

 

 

30,000 shares of Rs.10

 

 

 

each fully paid up

3,00,000

 

 

Profit and loss A/c

1,50,000

 

 

Securities premium

15,000

 

 

Creditors

27,000

 

_______

 

7,40,000

 

7,40,000

On 30th June, 2011, the Board of directors decided to redeem the preference shares at a premium of 10% and to sell the investments at its market price of Rs.40,000. They also decided to issue sufficient number of equity shares of Rs.10 each at a premium of Re. 1 per share, required after utilising the profit and loss account leaving a balance of Rs.50,000. Premium on redemption is required to be set off against securities premium account.

Repayments on redemption were made in full except to one shareholder holding 50 shares only due to his leaving India for good.

You are required to show the journal entries and the balance sheet of the company after redemption. Assumption made should be shown in the working.

when redeemable preference shares are redeemed partly out of the profits of the com 674539

(When Redeemable Preference Shares are redeemed partly out of the profits of the company and partly out of the proceeds of fresh issue of shares made for the purpose).

The Balance Sheet of Producers Ltd. as at 31st March, 2011 is as follows:

Liabilities

Rs

Assets

Rs

Share Capital:

 

Fixed Assets:

 

Authorized Capital 40,000 Equity Shares of 10 each

4,00,000

Plant and Machinery

1,90,000

1,000 8% Preference Shares of 100 each

1,00,000

Furniture and Fixtures

20,000

 

5,00,000

Investments

60,000

Issued and Subscribed Capital: 25,000 Equity Shares of 10 each fully paid up

2,50,000

Current Assets, Loans and Advances:

 

1,000, 8% Preference Shares of 100 each fully paid up

1,00,000

A. Current Assets:

 

Reserves and Surplus:

 

Stock

1,30,500

Securities Premium Account

9,000

Debtors

49,550

Profit and Loss Account

55,000

Cash at Bank

4,950

Current Liabilities and Provisions

 

B. Loans and Advances:

 

A. Current Liabilities : Sundry Creditors

22,500

Prepaid expenses

1,000

B. Provisions: Provisions for taxation

19,500

   
 

4,56,000

 

4,56,000

In order to redeem its preference shares, the company issued 5,000 equity shares of Rs.10 each at a Premium of 10% and sold its investment of Rs.70,800. Preference shares were redeemed at a premium of 10%.

Show the necessary journal entries in the books of the company and prepare the balance sheet of the company immediately after redemption of preference shares.

when fresh issue of shares is made at a discount 674540

(When fresh issue of shares is made at a discount)

The Balance Sheet of Ultra Modern Ltd. as at 31st March, 2011 is as follows:

Liabilities

Rs

Assets

Rs

Share Capital

 

Fixed Assets:

 

Issued and Subscribed Capital:

 

Land and Building

2,00,000

1,000, 9% Redeemable

 

Plant and Machinery

60,000

Preference Shares of

 

Furniture and Fixtures

9,000

Rs.100 each

1,00,000

Current Assets:

 

Rs.18,000 Equity Shares of

 

Stock

60,000

Rs.10 each

1,80,000

Debtors

25,000

Reserves and Surplus:

 

Investments

54,000

Securities Premium Account

20,000

Bank

42,000

General Reserve Account

60,000

 

 

Profit and Loss Account

40,000

 

 

Current Liabilities:

 

 

 

Sundry Creditors

50,000

 

 

 

4,50,000

 

4,50,000

The Company decided to redeem its preference shares at a premium of 5% on 1st April, 2011.

A fresh issue of 3,000 equity shares of Rs.10 each was made at Rs.9 per share, payable in full on 1st April, 2011. These were fully subscribed and all moneys were duly collected. All the investments were sold for Rs.50,000 to provide cash for redemption of preference shares. The directors wish that only a minimum reduction should be made in the revenue reserves.

You are required to give the journal entries, including those relating to cash to record the above transactions and to draw up the balance sheet as it would appear after redemption of preference shares.

a company issued 10 000 shares of rs 10 each total applications were for 12 000 shar 674543

1. A company issued 10,000 shares of Rs.10 each. Total applications were for 12,000 shares; allotment was made pro rata. Application money was Rs.2 per share and allotment money Rs.3 per share. Rao failed to pay the allotment money on his 300 shares. How much is due from Rao for allotment?

2. A company issues 10,000 shares of Rs.10 each @ a premium of Rs.2 per share, payable as: on application Rs.4 (including premium), on allotment Rs.3 and the balance on calls. 8,000 shares were applied for. Which of the following entries is correct for application money:

(a) Bank

Dr

32,000

 

To Share Application A/c

   

16,000

To Securities Premium A/c

   

16,000

(b) Bank

Dr

32,000

 

To Share Application A/c

   

32,000

w ltd issued 2 000 14 debentures of rs 100 each at discount of 5 the 674552

W Ltd. issued 2,000, 14% Debentures of Rs.100 each at discount of 5% the

discount being adjustable on allotment. The debentures were payable as follows:

On Application

Rs.25

On Allotment

Rs.20

On First and Final Call

Rs.50

The debentures were fully subscribed and the moneys were duly received.

Show the cash book and journal entries and prepare the balance sheet of the company.

when sinking fund is created to redeem debentures at the end of the specified perio 674559

(When Sinking Fund is created to redeem debentures at the end of the specified period).

Steady Ltd. issued 2,000, 9% Debentures of Rs.100 each at par on 1st April 2006 repayable at the end of 5 years at a premium of 6%. It was decided to institute a Sinking Fund for the purpose, the investments being expected to yield 8% p.a. Sinking Fund tables show that Re. 1 per annum at 8% compound interest amounts to Rs.5.867 in 5 years. Investments were made in multiples of rupees ten only.

On 31st March, 2011 the investments realized Rs.1,75,000 and the debentures were redeemed. The bank balance as on that date was Rs.54,800.

You are required to show the journal entries relating to the creation of Sinking Fund and to prepare the relevant ledger accounts in the books of the company. Ignore debenture interest.

when debentures are redeemed out of the proceeds of fresh issue of shares or debent 674562

(When Debentures are redeemed out of the proceeds of fresh issue of shares or debentures).

The following is the Balance Sheet of Good Luck Ltd. as on 1st April, 2011

Liabilities

Rs

Assets

Rs

Share Capital:

 

Fixed Assets:

 

Authorised Capital:

 

Land and Building

2,00,000

1,00,000 Equity Shares

 

Plant and Machinery

2,00,000

of 10 each

10,00,000

Furniture and Fixtures

10,000

50,000 Equity shares of

 

Current Assets, Loans

 

10 each fully paid up

5,00,000

and Advances:

 

Reserves and Surplus:

 

A. Current Assets

 

Profit & Loss A/c

50,000

Stock in Trade

1,70,000

Secured Loans:

 

Sundry Debtors

2,00,000

1,000 12% Debentures of

 

Cash at Bank

20,000

100 each fully paid up

1,00,000

B. Loans and Advances

Nil

Current Liabilities and Provisions:

 

 

 

A. Current Liabilities

 

 

 

Creditors

1,50,000

 

 

B. Provisions

Nil

 

 

 

8,00,000

 

8,00,000

The Debenture Trust Deed provides that the company may redeem the debentures at a premium of 5% at any time before the maturity. In order to exercise this option, the directors decided to issue 10,000 equity shares of 10 each at 11 on this day and to redeem the debentures. All the shares were duly subscribed and the debentures were redeemed.

Show the journal entries in the books of the company. Also prepare the Balance Sheet after the redemption of debentures.

when debentures are purchased for immediate cancellation and there is no sinking fu 674563

(When debentures are purchased for immediate cancellation and there is no Sinking Fund)

Favourite Ltd. had 2,000, 12% Debentures of Rs.100 each as on 1st April, 2010. As per the terms of issue, the company purchased the following debentures in the open market for immediate cancellation:

1st May

400 Debentures at 98 cum interest

1st January

800 Debentures at 100.25 cum interest

1st March

98.50 ex interest

Assuming that debenture interest was payable half yearly on 30th September and 31st March and the Income tax was deductible at the rate of 10% at source. Show the journal entries in the books of the company and prepare the necessary ledger accounts. The company closes its books on 31st March.

when debentures are purchased for immediate cancellation and sinking fund exists 674564

(When debentures are purchased for immediate cancellation and Sinking Fund exists).

The following balances appeared in the books of Cheerful Ltd. as on 1st April, 2010:

9% Debentures (face value Rs.100)

Rs.1,50,000

Debenture Redemption Fund

Rs.75,000

Debenture Redemption Fund Investment

Rs.75,000

(in 8% Government Bonds of the face value of `90,000)

Interest on the debentures was payable on 30th September and 31st March and interest on Government Bonds was receivable on the same dates.

On 31st May, 2010 the company purchased for immediate cancellation 250 debentures in the market at Rs.95 each cum interest. The amount required for this was raised by selling 8% Government Bonds of the face value of Rs.27,000.

On 31st March, 2011 Rs.20,800 was appropriated for the Sinking Fund and on the same date 8% Government Bonds were acquired for the amount plus the interest on investments. The face value of the Government Bonds acquired was Rs.28,000. You are required to show the journal entries and ledger accounts in the books of the company. Ignore Income tax.

where debentures are purchased as investments and no sinking fund exists this also 674565

(Where debentures are purchased as investments and no Sinking Fund exists. This also includes treatment of interest on own debentures).

In the books of Joy Ltd., the 12% Debentures Account showed a credit balance of Rs.2,00,000 consisting of 2,000 debentures of Rs.100 each as on 1st April, 2010.

During the year debentures were purchased in the open market as follows:

1st August, 300 Debentures at Rs.95 ex interest.

1st November, 200 Debentures at Rs.98 cum interest.

The Debentures, thus, purchased were retained as investments of the company. Interest on debentures was payable half yearly on 30th September and 31st March every year.

You are required to show the journal entries and the ledger accounts in the books of the company. Ignore Income tax. Also show how the items would appear in the Balance Sheet.

cancellation of own debentures on a subsequent date where sinking fund does not exi 674566

1. (Cancellation of Own Debentures on a subsequent date where Sinking Fund does not exist)

Continuing Illustration No. 19, if the Debentures held by the company are cancelled on 31st March, 2011, show the necessary journal entries on cancellation and the effect of the same in the Balance Sheet of the company.

2. (When Debentures are purchased as investments of Sinking Fund).

Confident Ltd. had 2,000 12% Debentures of Rs.100 each outstanding as on 1st April 2010. The following other balances also appeared in the books of the company o n this date:

 

Rs

Debentures Redemption Fund Account

1,00,000

Debentures Redemption Fund Investments:

 

12% Port Trust Bonds (face value Rs.60,000)

55,000

Own Debentures (face value Rs.50,000)

45,000

Interest on the debentures was payable on 30th September and 31st March and interest on Port Trust Bonds was received on the same dates.

On 1st August, 2010, Rs.20,000, 12% Port Trust Bonds were sold at Rs.95 ex interest and the amount realised was invested in Own Debentures at Rs.97 cuminterest. During the year a sum of Rs.5,800 was appropriated for the Sinking Fund which together with the interest received on Sinking Fund during the year was invested in Own Debentures at Rs.95 each.

You are required to show the journal entries and ledger accounts in the books of the company. Also show how the items will appear in the Balance Sheet of the company. Ignore Income tax.

cancellation of own debentures on a subsequent date where sinking fund exists 674567

1. (Cancellation of Own Debentures on a subsequent date where Sinking Fund exists).

Continuing Illustration No. 21 if Own Debentures held by the company are cancelled on 31st March, 2011, show the necessary journal entries on cancellation and the effect of the same in the Balance Sheet of the Company.

2. On 1st April, 2010, Green Ltd. issued 2500 12% Debentures of Rs.100 each at Rs.95. Holders of these debentures have an option to convert their holdings into 14% Preference Shares of Rs.100 each at a Premium of Rs.25 per share at any time within three years.

On 31st March, 2011, holders of 500 Debentures notified their intention to exercise the option.

Show the journal entries relating to the issue and conversion of debentures in the books of the company. Also show how the items affected would appear in the company’s balance sheet.

the summarised balance sheet of swathi ltd as on 31st march 2011 stood as follows 674568

The summarised balance sheet of Swathi Ltd. as on 31st March, 2011 stood as follows:

Liabilities:  

Rs

 Share Capital 50,000 shares of   10 each 

5,00,000

 General reserve  

7,50,000

 Debenture Redemption Fund

5,00,000

12% Convertible debentures 

 

 10,000 debentures of 100 each  

10,00,000

 Unsecured Loan  

5,00,000

 Current liabilities  

12,50,000

 

45,00,000

Assets:

 

Net fixed assets

18,00,000

Debenture Redemption fund investment

4,00,000

Cash at bank

5,00,000

Other current assets

18,00,000

 

45,00,000

The debentures are due for redemption on 1st April, 2011. According to the terms of issue of debentures, they were redeemable at a premium of 5% and also conferred option to the debenture holders to convert 20% of their holdings into equity shares at a predetermined price of Rs.15.75 per share and the payment in cash.

Assuming that:

(i) Except for 100 debenture holders holding 2,500 debentures, the rest of them exercised the option for maximum conversion.

(ii) The investments realise Rs.4,40,000 on sale and

(iii) All transactions are put through, on 1st April, 2011.

You are required to redraft the balance sheet of the company as on 1st April, 2011 after giving effect to the redemption. Also show the number of equity shares to be allotted and the cash payment necessary. 

satellite ltd issued 12 10 000 preference shares of rs 10 each the issue was underwr 674579

1. Satellite Ltd., issued 12% 10,000 Preference Shares of Rs.10 each. The issue was underwritten as follows:

Apple 30%, Mango 30%, Orange 20%.

Application for 8,000 shares were received by the company in all. Determine the liability of the respective underwriters.

2. Emess Ltd. issued 40,000 shares which were underwritten as:

P: 24,000 shares Q: 10,000 shares and R: 6,000 shares. The underwriters made applications for firm underwriting as under:

P: 3,200 shares; Q: 1,200 shares; and R: 4,000 shares. The total subscriptions excluding firm underwriting (including marked applications) were 20,000 shares.

The marked applications were P: 4,000 shares; Q: 8,000 shares; and R: 2,000 shares.

Prepare a statement showing the net liability of underwriters.

woodlands ltd registered with a capital of rs 10 00 000 in equity shares of rs 10 ea 674583

(Where consideration is not given in the problem).

Woodlands Ltd., registered with a capital of Rs.10,00,000 in equity shares of Rs.10 each acquired the business of M/s A and B, the Balance Sheet of whom at the date o f acquisition was as follows:

Liabilities

Rs

Assets

Rs

Bills Payable

16,000

Cash at Bank

29,000

Sundry Creditors

30,000

Bills Receivable

13,000

Reserve

14,000

Sundry Debtors

48,000

Capital Accounts:

 

Stock

18,000

A 70,000

 

Furniture and Fixtures

2,000

B 70,000

1,40,000

Plant and Machinery

40,000

 

 

Land and Buildings

50,000

 

2,00,000

 

2,00,000

The assets and liabilities were subject to the following revaluation:

Plant and Machinery to be depreciated by 10%

Furniture and Fittings to be depreciated by 15%

Land and Buildings to be appreciated by 20%

A provision to be made for bad debts on debtors @ 2 1/2%

Goodwill of the firm was valued at Rs.24,000.

The consideration was to be discharged as follows:

(i) Allotment of 10,000 Equity Shares of Rs.10 each at Rs.12 each.

(ii) Allotment of 500 14% Debentures of Rs.100 each at a discount of 10%.

(iii) Balance in cash.

The cost of acquisition of the company amounted to Rs.5,000.

You are required to show the journal entries in the books of the company and prepare the opening balance sheet of the company after the acquisition.

smart ltd was incorporated on 1st august 2010 with an authorised capital of 5 00 000 674584

Smart Ltd. was incorporated on 1st August, 2010 with an authorised capital of 5,00,000 equity shares of Rs.10 each to acquire the business of Mr. Smart with effect from 1st April, 2010.

The purchase consideration was agreed at Rs.7,00,000 to be satisfied by the issue of 40,000 equity shares of Rs.10 each as fully paid up and 3,000, 9% debentures of Rs.100 each as fully paid up.

The entries relating to the transfer were not made in the books which were carried on without a break until 31st March, 2011. On 31st March, 2011 the trial balance extracted from the books showed the following:

 

Rs

Rs

 Sales  

 

 10, 43,700  

 Purchases  

 7,76,580  

 

 Advertising  

 37,800  

 

 Postage and Telegram  

 8,820  

 

 Rent and Rates  

 18,420  

 

 Packing Expenses  

 16,800  

 

 Office Expenses  

 12,540  

 

 Opening Stock as on 1.4.2010  

 1,05,220  

 

 Directors‘ fees  

 20,000  

 

 Debenture Interest  

 18,000  

 

 Land and Buildings  

 3,00,000  

 

 Plant and Machinery  

 1,80,000  

 

 Furniture and Fixture  

 20,000  

 

 Sundry Debtors  

 1,39,500  

 

 Cash at Bank  

 40,000  

 

 Cash in hand  

 4,900  

 

 Bills Payable  

 

 30,000  

 Sundry Creditors  

 

 53,240  

 Preliminary Expenses  

 7,360  

 

 Smart‘s Capital Account  

 

 5, 89,000  

 Smart‘s Drawings Account  

 10,000  

 

 

 17,15,940  

 17,15,940  

You are also given the following additional information:

(i) Stock on 31st March, 2011 amounted to Rs.98,920.

(ii) The average monthly sales for April, May and June were one half of those for the remaining months of the year and the gross profit margin was constant throughout the year.

You are required to prepare the Trading and Profit and Loss Account for the year ended 31st March, 2011 and the Balance Sheet of Smart Ltd. as on that date.

accounting 673771

20000 257500 19600 580000 420 37 420 26 1400 56 325 75 150 103 High Point produces fleece jackets. The company uses JIT costing for its JIT production system. High Point has two inventory accounts: Raw and in process inventory and Finished goods inventory. On February 1, 2012, the account balances were Raw and in process inventory, $7,000; Finished goods inventory, $2,200. The standard cost of a jacket is $37, comprised of $13 direct materials plus $24 conversion costs.

Document Preview:

20000 257500 19600 580000 420 37 420 26 1400 56 325 75 150 103 High Point produces fleece jackets. The company uses JIT costing for its JIT production system. High Point has two inventory accounts: Raw and in process inventory and Finished goods inventory. On February 1, 2012, the account balances were Raw and in process inventory, $7,000; Finished goods inventory, $2,200. The standard cost of a jacket is $37, comprised of $13 direct materials plus $24 conversion costs. Data for February’s activities follow: Number of jackets completed Number of jackets sold Direct materials purchased Requirements: 1. What are the major features of a JIT production system such as that of High Point? 2. Prepare summary journal entries for February. Under or over allocated conversion costs are closed to Cost of goods sold monthly. 3. Use a T account to determine the February 29, 2012, balance of Raw and in process inventory. Activity Inspection of incoming materials Inspection of finished goods 1. Calculate the predicted quality cost savings from the design engineering work. 2. Christi spent $103,000 on design engineering for the new skateboard. What is the net benefit of this “preventive” quality activity? 3. What major difficulty would Christi’s managers have in implementing this cost of quality approach? What alternative approach could they use to measure quality improvement? Predicted Reduction in Activity Units Activity Cost Allocation Rate Per Unit Number of defective units discovered in house Number of defective units discovered by customers Lost sales to dissatisfied customers Conversion costs incurred Christi, Inc., is using a costs of quality approach to evaluate design engineering efforts for a new skateboard. Christi’s senior managers expect the engineering work to reduce appraisal, internal failure, and external failure activities. The predicted reductions in activities over the 2 year life of the skateboards follow. Also shown are the cost…

the main focus of this course is to understand rules in international accounting 673804

The following is a list of the questions. Your project will need to be submitted as one MSWord document. If you use Excel spreadsheets to answer the questions, please copy and paste them into your Word document. You can only submit ONE document for the Portfolio Project, so all of your work needs to be contained in that one document. Please make sure to clearly label your answers and use APA for formatting and citation as required. PROBLEM The main focus of this course is to understand rules in International Accounting. One such focus is to understand the major differences in US GAAP and IFRS (International Financial Reporting Standards). The portfolio project is divided into two parts; provide detailed responses to both parts of the project. Part I 1. Provide a detailed history of the convergence project between US GAAP and IFRS. 2. Discuss in detail the major differences between US GAAP and IFRS applications. 3. Provide a list of countries that have adopted IFRS and how cultural differences can impact interpretation of IFRS. Part II 1. Explain Transfer Pricing in detail. 2. Provide a detailed example of how Transfer Pricing can be used successfully by Multinational Corporations. 3. Please discuss in detail various methods of translating foreign currency financial statements. For each of the two parts, discuss and cite at least two credible outside sources other than the assigned textbook. Your paper should be 8 10 pages in length, well written, and formatted in conformity with APA Style.

Document Preview:

The following is a list of the questions. Your project will need to be submitted as one MSWord document.  If you use Excel spreadsheets to answer the questions, please copy and paste them into your Word document.  You can only submit ONE document for the Portfolio Project, so all of your work needs to be contained in that one document. Please make sure to clearly label your answers and use APA for formatting and citation as required. PROBLEM The main focus of this course is to understand rules in International Accounting. One such focus is to understand the major differences in US GAAP and IFRS (International Financial Reporting Standards). The portfolio project is divided into two parts; provide detailed responses to both parts of the project. Part I Provide a detailed history of the convergence project between US GAAP and IFRS. Discuss in detail the major differences between US GAAP and IFRS applications. Provide a list of countries that have adopted IFRS and how cultural differences can impact interpretation of IFRS. Part II Explain Transfer Pricing in detail. Provide a detailed example of how Transfer Pricing can be used successfully by Multinational Corporations. Please discuss in detail various methods of translating foreign currency financial statements. For each of the two parts, discuss and cite at least two credible outside sources other than the assigned textbook. Your paper should be 8 10 pages in length, well written, and formatted in conformity with APA Style.

Attachments:

what are some of the problems an analyst might encounter in analyzing foreign financ 673806

• 1) Analyzing Foreign Financial Statements (50 Points)

What are some of the problems an analyst might encounter in analyzing foreign financial statements?

Your well written 2 3 page essay should outline these problems, citing real world examples from at least two outside sources. You are welcome to cite the textbook in addition. Ensure your paper is formatted per APA style. The CSU Global Library is a great place to find sources. You may also want to explore the following websites:

• www.ifrs.org

• www.fasb.org

• www.aicpa.org

• 2) Case 12 1: Litchfield Corporation (50 Points)

Please read through the following case study from the course text. Then complete the assignment outlined at the close of the study.

Case Study 12 1: Litchfield Corporation is a U.S. based manufacturer of fashion accessories that produces umbrellas in its plant in Roanoke, Virginia, and sells directly to retailers in the United States. As chief financial officer, you are responsible for all of the company’s finance, accounting, and tax related issues.

Sarah Litchfield, chief executive officer and majority shareholder, has informed you of her plan to begin exporting to the United Kingdom, where she believes there is a substantial market for Litchfield umbrellas. Rather than selling directly to British umbrella retailers, she plans to establish a wholly owned UK sales subsidiary that would purchase umbrellas from its U.S. parent and then distribute them in the United Kingdom. Yesterday, you received the following memo from Sarah Litchfield:

Memorandum

SUBJECT: Export Sales Prices

It has come to my attention that the corporate income tax rate in Great Britain is only 28 percent, as compared to the 35 percent rate we pay here in the United States. Since our average production cost is $15.00 per unit and the price we expect to sell to UK retailers is $25.00 per unit, why don’t we plan to sell to our UK subsidiary at $15.00 per unit? That way we make no profit here in the United States and $10.00 of profit in the United Kingdom, where we pay a lower tax rate. We have plans to invest in a factory in Scotland in the next few years anyway, so we can keep the profit we earn over there for that purpose. What do you think?

Draft a 1 2 page essay responding to Sarah Litchfield’s question by explaining U.S. income tax regulations related to the export sales described in her memo. Include a discussion of any significant risks associated with her proposal. Make a recommendation with respect to how the price for these sales might be determined. Ensure your essay is well written and formatted in conformity with standards for APA style.

Attachments:

a government financial reporting 50 points submit your responses to the following qu 673807

A) Government Financial Reporting50 Points) Submit your responses to the following questions in a 1 2 page summary MSWord document. Label each question clearly. For computations done in an Excel spreadsheet, please copy and paste your work into your MSWord document. For written answers, please make sure your responses are well written, use APA formatting, and have the proper citation, if needed. A city manager was overheard saying, “Since we don’t release them to the public, I don’t see any value in taking the time to prepare interim reports.

Document Preview:

A) Government Financial Reporting 50 Points) Submit your responses to the following questions in a 1 2 page summary MSWord document. Label each question clearly. For computations done in an Excel spreadsheet, please copy and paste your work into your MSWord document. For written answers, please make sure your responses are well written, use APA formatting, and have the proper citation, if needed. A city manager was overheard saying, “Since we don’t release them to the public, I don’t see any value in taking the time to prepare interim reports.” Explain why you agree or disagree with this statement. Describe how a taxpayer might assess the financial solvency and viability of a government using government wide financial statements prepared according to generally accepted accounting principles. B) Not For Profit Financial Reporting  Submit your responses to the following questions in a 1 2 page summary MSWord document. Label each question clearly. For computations done in an Excel spreadsheet, please copy and paste your work into your MSWord document. For written answers, please make sure your responses are well written, use APA formatting, and have the proper citation, if needed. Explain how organizations in the not for profit sector differ from organizations in the public sector or for profit business sector.  Provide an example of an entity in each sector. Which standards setting bodies are assigned responsibility for establishing accounting and financial reporting standards for not for profit organizations? What issues remain unfinished on the FASB’s technical agenda for not for profit organizations?

Attachments:

balance sheet 673849

fill out the balance sheet for a company

Document Preview:

Problem 24 1 8/30/14 7:28 AM Print by: Alexa Block 251113: 251113: Intermediate Accounting Theory and Practice (Online) X 120C (Summer 2014) / Chapter 24 Graded Problems *Problem 24 1 Your firm has been engaged to examine the financial statements of Almaden Corporation for the year 2014. The bookkeeper who maintains the financial records has prepared all the unaudited financial statements for the corporation since its organization on January 2, 2009. The client provides you with the information below. ALMADEN CORPORATION BALANCE SHEET DECEMBER 31, 2014 Assets Liabilities Current assets $1,884,860 Current liabilities $966,240 Other assets 5,192,850 Long term liabilities 1,493,060 Capital 4,618,410 $7,077,710 $7,077,710 An analysis of current assets discloses the following. Cash (restricted in the amount of $302,770 for plant expansion) $571,670 Investments in land 185,800 Accounts receivable less allowance of $30,560 481,920 Inventories (LIFO flow assumption) 645,470 $1,884,860 Other assets include: Prepaid expenses $63,290 Plant and equipment less accumulated depreciation of $1,443,300 4,137,400 Cash surrender value of life insurance policy 84,310 Unamortized bond discount 44,400 Notes receivable (short term) 162,520 Goodwill 253,870 Land 447,060 $5,192,850 Current liabilities include: Accounts payable $512,100 Notes payable (due 2017) 157,550 Estimated income taxes payable 145,590 Premium on common stock 151,000 $966,240 Long term liabilities include: Unearned revenue $490,790 Dividends payable (cash) 201,870 8% bonds payable (due May 1, 2019) 800,400 $1,493,060 Capital includes: Retained earnings $2,768,510 Capital stock, par value $10; authorized 200,000 shares, 184,990 shares 1,849,900 http://edugen.wileyplus.com/edugen/shared/assignment/test/qprint.uni Page 1 of 3Problem 24 1 8/30/14 7:28 AM Capital stock, par value $10; authorized 200,000 shares, 184,990 shares 1,849,900 issued $4,618,410 The supplementary…

Attachments:

delta 673957

Delta Company produces a single product. The cost of producing and selling a single unit of this product at the company’s normal activity level of 60,000 units per year is:

Direct materials ………………………………………………. $5.10

Direct labor …………………………………………………….. $3.80

Variable manufacturing overhead ……………………… $1.00

Fixed manufacturing overhead ………………………….. $4.20

Variable selling and administrative expense ……….. $1.50

Fixed selling and administrative expense …………… $2.40

The normal selling price is $21 per unit. The company’s capacity is 75,000 units per year. An order has been received from a mail order house for 15,000 units at a special price of $14.00 per unit. This order would not affect regular sales.

Required:

i. If the order is accepted, by how much will annual profit to be increased or decreased? (The order will not change the company’s total fixed costs.)

ii. Assume the company has 1,000 units of this product left over from last year that are inferior to the current model. The units must be sold through regular channels at reduced prices. What unit cost is relevant for establishing a minimum selling price for these units? Explain.

quality cost report do you believe this firm s quality initiatives have been success 673971

Consider the following quality cost report:

Year 1 Year 2 Year 3
Prevention $1,775 $1,650 $1,650
Appraisal $2,500 $2,500 $2,500
Internal failure $2,400 $2,600 $2,900
External failure $2,500 $2,900 $3,300
Total quality costs $9,175 $9,650 $10,350
Total revenues $400,000 $420,000 $445,000

Do you believe this firm’s quality initiatives have been successful?
Be sure to justify your opinion with specific information from the quality report.

Document Preview:

Question #5 Consider the following quality cost report: ?Year 1?Year 2?Year 3??Prevention ?$1,775 ?$1,650 ?$1,650??Appraisal?$2,500 ?$2,500 ?$2,500 ??Internal failure?$2,400 ?$2,600 ?$2,900 ??External failure?$2,500 ?$2,900 ?$3,300 ??Total quality costs?$9,175 ?$9,650 ?$10,350??Total revenues?$400,000 ?$420,000 ?$445,000 ?? Do you believe this firm’s quality initiatives have been successful? Be sure to justify your opinion with specific information from the quality report.

Attachments:

compute revenue and variable costs for each show 2 use the income statement equation 673986

1. Compute revenue and variable costs for each show. 2. Use the income statement equation approach to compute the number of shows England Productions must perform each year to break even. 3. Use the contribution margin approach to compute the number of shows needed each year to earn a profit of $5,687,500. Is this profit goal realistic? Give your reasoning. 4. Prepare England Productions’ contribution margin income statement for 150 shows performed in 2012.

Document Preview:

150000 41000 15000 4000 60000 90000 37800 12600 50400 39600 Requirements: 1. Compute revenue and variable costs for each show. 2. Use the income statement equation approach to compute the number of shows England Productions must perform each year to break even. 3. Use the contribution margin approach to compute the number of shows needed each year to earn a profit of $5,687,500. Is this profit goal realistic? Give your reasoning. 4. Prepare England Productions’ contribution margin income statement for 150 shows performed in 2012. Report only two categories of costs: variable and fixed. England Productions performs London shows. The average show sells 1,300 tickets at $60 per ticket. There are 150 shows a year. No additional shows can be held as the theater is also used by other production companies. The average show has a cast of 65, each earning a net average of $340 per show. The cast is paid after each show. The other variable cost is a program printing cost of $8 per guest. Annual fixed costs total $728,000. The contribution margin income statement of Delectable Donuts for August 2012 follows: Cost of goods sold DELECTABLE DONUTS Contribution Margin Income Statement For the Month of August 2012 Sales revenue Variable costs: Marketing costs General and administrative costs Contribution margin Fixed costs: Operating income 1. Calcuate the weighted average contribution margin. Delectable sells four dozen plain donuts for every dozen custard filled donuts. A dozen plain donuts sells for $4, with total variable cost of $1.60 per dozen. A dozen custard filled donuts sells for $5, with total variable cost of $2 per dozen. 2. Determine Delectable’s monthly breakeven point in dozens of plain donuts and custard filled donuts. Prove your answer by preparing a summary contribution.

your company has just purchased a large piece of property and multiple creditors hav 674012

Your company has just purchased a large piece of property, and multiple creditors have a security interest in the property. This security interest must then be perfected, which sets forth the rights of a secured creditor against other creditors who also have a security interest in your property.

  • In an Excel spreadsheet that is 2 pages in length, create 2 charts (1 chart on each page).
    • The guidelines for the first chart are the following:
      • Create a list of the priority of creditors with regard to security interests when there are conflicting claims.
      • Which creditor is given preference, and why?
      • Discuss the creditors that follow in the chain.
    • The guidelines for the second chart are the following:
      • Depict a scenario with 4 different creditors that have a security interest in the property of a company.
      • Be sure to list next to each creditor the date of perfection if the security interest is perfected.
      • List, in order, the creditors. Create a column in which you explain why that creditor is in that place in the chain of priority.
      • How will the lien on the property be shown in the notes to the financial statements?

click here to read the case study for keystone computers and network a partially com 674014

Click here to read the case study for Keystone Computers and Network. A partially completed analytical ratios working paper for Keystone Computers & Networks, Inc., is presented on page 239 of that case study. Please address the following in a 3 to 5 page paper.

  1. Complete the working paper by computing the financial ratios for 20X5 and provide these in your paper.
  2. After completing part (A), above, review the ratios and identify financial statement accounts that should be investigated because the related ratios are not comparable to prior year ratios, industry averages, or your knowledge of the company. Provide these in your paper.
  3. For each account identified in part (B), above, identify and discuss potential reasons for the unexpected account balances and related ratios.

i need this both assignment on 10th september 2014 can you please tell me what is th 674082

I need this both assignment on 10th September, 2014 . Can you please tell me what is the charges

Document Preview:

ACCM 4500 Management Accounting 2 Assignment Trimester 2, 2014 VALUE: 25% of OVERALL ASSESSMENT th DUE DATE: Friday 19 September by 11.59pm AEST. Adelaide students, please allow for time differences. The assignment comprises two components: Part A – Technical skills 15% Part B – Professional Competencies covering communication skills; both written and oral and teamwork 10% WORD LIMIT: 3,000 Words (including any appendices, excluding presentation) REQUIRED: Students are to form groups of 3 (you must obtain your lecturer’s approval for a group of 2 or 4 members). You must attach pages 3 & 4 of this document as pages 1 & 2 of your assignment. Each student’s name must be recorded on the coversheet otherwise that student will receive zero for the assignment. Only the group leader should submit to Turnitin otherwise penalties will apply. Submission is via Turnitin on the moodle site for the course. Please see the site for more details.Team Meetings The team should have a minimum of two meetings (the team may have as many meetings as they need to) and should maintain minutes of those meetings detailing – 1) Where the meeting was held (including online) 2) People present; absent; apologies 3) What was discussed and agreed on; 4) The follow up action relating to each member of the team; 5) Any other relevant matter, things you agreed or disagreed on etc. The team meeting minutes should be included with the hard copy of your assignment as an attachment. Please ensure that you follow the rules noted under presentation of written work in the subject outline. Your attention is also drawn to the academic integrity section of the subject outline. Assignment Task Please visit the website http://www.board.com/au, browse through the case studies and select a business to analyse. Please check with your lecturer that no other group has selected the same business. Answer the following questions: …

see attached 674225

ACT 5060 – Accounting for Decision Makers Extra Credit Directions: Each question is worth up to 5 points added to your mid term exam score. Please submit your work in Word or PDF formats only. You can submit an Excel file to support calculations, but please “cut and paste” your solutions into the Word or PDF file. Be sure to show how you did your calculations. Also, please be sure to include your name at the top of the first page of your file. You can use any sources you wish, except for other people. Please be sure to document any source you use.

Document Preview:

ACT 5060 – Accounting for Decision Makers Extra Credit Directions: Each question is worth up to 5 points added to your mid term exam score. Please submit your work in Word or PDF formats only. You can submit an Excel file to support calculations, but please “cut and paste” your solutions into the Word or PDF file. Be sure to show how you did your calculations. Also, please be sure to include your name at the top of the first page of your file. You can use any sources you wish, except for other people. Please be sure to document any source you use. The assignment is due by 5:00 PM on Friday September 5th. No late submissions will be accepted. If you have any questions, please e mail me at ? HYPERLINK “mailto:af878@nova.edu” ?af878@nova.edu? or ? HYPERLINK “mailto:andrew.felo@gmail.com” ?andrew.felo@gmail.com?. Good luck! Question #1 The following is budgeted information for the Christopher Corporation: ?Product 1?Product 2??Annual production & sales?50,000?75,000??Projected selling price?$40?$30??????Direct Production Cost Information????Materials (per unit)?$12?$8??Direct Labor (per unit)?$6?$5?? Additional information: Selling & administrative costs (a mixed cost) are budgeted to be $600,000 at the production and sales listed above. The variable component is $3 per unit (same for each product). Manufacturing overhead costs (a mixed cost) are budgeted to be $800,000 at the production and sales listed above. The fixed component is $300,000. Each product uses the same amount of variable manufacturing overhead per unit. Assuming the budgeted sales mix remains intact, how many units of each product does Christopher need to sell in order to earn a target operating income of $240,000? Question #2 Consider the following information, prepared based on a monthly capacity of 80,000 units: Category?Cost per Unit??Variable manufacturing costs ?$12.00??Fixed manufacturing costs?$3.00??Variable selling costs?$4.00??Fixed selling costs?$2.00?? Capacity cannot be added…

Attachments:

proprietorship attributes applying the entity concept and preparing financial statem 674258

Proprietorship attributes, applying the entity concept, and preparing financial statements

Sandy White is a realtor. She organized her business as a proprietorship, Sandy White, Realtor, by investing $27,000 cash.

The business gave capital to her. Consider the following facts at

May 31, 2012:

a. The business owes $62,000 on a note payable for land that the business acquired for a total price of $80,000.

b. The business spent $26,000 for a Minko Banker real estate franchise, which entitles the business to represent itself as a Minko Banker office. This franchise is a business asset.

c. White owes $70,000 on a personal mortgage for her personal residence, which she acquired in 2012 for a total price of $130,000.

d. White has $4,000 in her personal bank account, and the business has $13,000 in its bank account.

e. White owes $3,000 on a personal charge account with Chico”s.

f. The office acquired business furniture for $20,000 on May 25. Of this

amount, the business owes $5,000 on account at May 31.

g. Office supplies on hand at the real estate office total $1,100.

Requirements

1. White was concerned about taxes. Which propriertorship feature limits White”s business taxes?

2. Prepare the balance sheet of the real estate business of Sandy White, Realtor at May 31, 2012.

3. Identify the personal items that would not be reported on the business records.

proprietorship attributes applying the entity concept and preparing financial statem 674259

Proprietorship attributes, applying the entity concept, and preparing financial statements

Sandy White is a realtor. She organized her business as a proprietorship, Sandy White, Realtor, by investing $27,000 cash.

The business gave capital to her. Consider the following facts at

May 31, 2012:

a. The business owes $62,000 on a note payable for land that the business acquired for a total price of $80,000.

b. The business spent $26,000 for a Minko Banker real estate franchise, which entitles the business to represent itself as a Minko Banker office. This franchise is a business asset.

c. White owes $70,000 on a personal mortgage for her personal residence, which she acquired in 2012 for a total price of $130,000.

d. White has $4,000 in her personal bank account, and the business has $13,000 in its bank account.

e. White owes $3,000 on a personal charge account with Chico”s.

f. The office acquired business furniture for $20,000 on May 25. Of this

amount, the business owes $5,000 on account at May 31.

g. Office supplies on hand at the real estate office total $1,100.

Requirements

1. White was concerned about taxes. Which propriertorship feature limits White”s business taxes?

2. Prepare the balance sheet of the real estate business of Sandy White, Realtor at May 31, 2012.

3. Identify the personal items that would not be reported on the business records.

accounting eec has introduced a new 1 5 gigabyte gb computer memory chip should eec 674321

ACCT614 1403B 02 Applied Managerial Accounting
Task Name: Phase 2 Individual Project
Deliverable Length: 1,750–2,000 words (not including title and reference pages)
Details:

Weekly tasks or assignments (Individual or Group Projects) will be due by Monday and late submissions will be assigned a late penalty in accordance with the late penalty policy found in the syllabus. NOTE: All submission posting times are based on midnight Central Time.

EEC has introduced a new 1.5 gigabyte (GB) computer memory chip.

  • Should EEC use a job order costing system, which is a costing system where costs are collected and assigned to units of production for each individual job, or a process costing system, which is a costing system that accumulates production costs by process or department for a given period of time, to report the costs of this new product?
    • How are the systems different?
  • Describe how the selected system would work to track the costs of the product.
  • Identify the cost driver, and explain the process of tracking the costs.
  • Provide examples of products that EEC might offer for which a job order costing system would be appropriate.
  • Provide examples of products that EEC might offer for which a process costing system would be appropriate.

Please submit your assignment.

Click here if you need to watch a tutorial on submitting Individual Project files.

Document Preview:

ACCT614 1403B 02 Applied Managerial Accounting Task Name: Phase 2 Discussion Board Mayra V Cruz August 27,2014 Professor: The purpose of the Discussion Board is to allow students to learn through sharing ideas and experiences as they relate to course content and the DB question. Because it is not possible to engage in two way dialogue after a conversation has ended, no posts to the DB will be accepted after the end of each week. As EEC’s corporate business financial analyst, you will need to have a clear understanding of the different types of costs (variable, fixed, and mixed) that the company carries. Complete the following for this assignment: •Review EEC’s journal activity. •Define and identify its variable, fixed, and mixed costs. •Determine what affect a sales volume increase or decrease will have on unit fixed cost, unit variable cost, total fixed cost, and total variable cost. Click here to view EEC’s journal activity. In your own words, please post a response to the Discussion Board and comment on other postings. You will be graded on the quality of your postings.

Attachments:

unit name professional ethics and governance assignment answers for enron fraud plea 674404

Unit name: Professional Ethics And GovernanceAssignment: Answers for Enron fraud

Please follow the requirements from the files uploadedUse the theories and learning materials provided

If you need any question please do no hesitate to ask me

Thank you

Document Preview:

Assignment No. 2 MAA 350 Professional Ethics and Governance – Trimester 2, 2014 This is a group written assignment of two students, you need to form your own group and register yourself in a group on CloudDeakin by the end of week 2. A detailed marking rubric is provided and only one report per group is to be submitted. The Deakin Graduate Learning Outcomes which this assessment is desire to achieve includes: GLO 1 Discipline specific knowledge and capabilities; GLO 4 Critical thinking; GLO 5 Problem solving and GLO 8 Global citizenship. This assessment is intended to evaluate your knowledge and understanding of some topics in the unit such as: Topic 2 (corporate governance scandals and the need to restore public trust); Topic 4 (understanding the theories of ethics); Topic 5 (moral reasoning and ethical decision making) and Topic 6 (professional independence and the codes of ethics). Note: You are required to watch Enron MAA350 (screen during lecture in week 2) as background understanding and answer all the questions below. Assignment Questions: Q1: Analyse the corporate culture at Enron and its management’s behaviour. Include in your analysis, the normative theory of ethics which you would consider most relevant in driving the decision making at Enron. (7 Marks; approximately 1,200 words) Q2: Did Arthur Andersen maintain its professional independence in its dealings with Enron, why or why not? (7 Marks; approximately 1,200 words) Q3: At which stages on Kohlberg’s theory of cognitive moral reasoning and development do you consider Jeff Skilling, Ken Lay and the whistle blower Sherron Watkins are at? Justify your answer. (6 Marks; approximately 1,100 words) Marks: 20 in total (7+7+6=20 marks) Word limit: 3,500 words in total (not including references, bibliography). th Due date: Week 8, Friday 12 September 2014, by noon (12.00pm) 1Referencing: Harvard…

nu look ltd issued 1 00 000 equity shares of rs 10 each payable as follows 674523

Nu Look Ltd. issued, 1,00,000 Equity Shares of Rs.10 each payable as follows:

On Application (On 1st March, 2011)

Rs.4

On Allotment (On 1st April, 2011)

Rs.1

On First Call (On 1st August, 2011)

Rs.3

On Final Call (On 1st October, 2011)

Rs.2

Application were received for 2,60,000 shares. Of these 10,000 shares were in disorder; 40,000 shares in lots of 100 shares; 1,20,000 shares in lots of exceeding 100 but less than 500 shares; 60,000 shares in lots of exceeding 500 but less than 1,000 shares and the balance in lots of exceeding 1,000 shares.

Allotment was made as follows:

Application for the 10,000 shares in disorder were rejected.

Application for 100 shares in full, i.e. 100%

40,000

Application over 100 shares but not exceeding 500 shares 40%

48,000

Application over 500 shares but not exceeding 1,000 shares 15%

9,000

Applications over 1,000 shares 10%

3,000

Money received in excess on shares partially allotted were retained to the extent possible. Show the cash book and journal entries assuming that all the installments were duly received and interest was paid by  he directors on calls in advance @ 6% per annum on 1st October, 2011.

on 1st january 2011 new ventures ltd issued 1 00 000 equity shares of rs 10 each pay 674524

On 1st January, 2011, New Ventures Ltd. issued 1,00,000 equity shares of Rs.10 each payable as follows:

On application

Rs.3

On allotment

Rs.2

On 1st Call

Rs.2 (Payable after 2 months, from the date of allotment)

On Final Call

Rs.3 (Payable after 2 months from the date of 1st call)

Applications were received on 15th January, 2011 for 1,20,000 shares and allotment was made on 1st February, 2011. Applicants for 50,000 shares were allotted in full, those for 60,000 shares were allotted 50,000 shares and applications for 10,000 shares were rejected.

Balance of amount due on allotment was received on 15th February.

The calls were duly made on 1st March, 2011 and 1st April, 2011 respectively. One shareholder did not pay the 1st Call money on 3,000 shares which he paid with the final call together with interest at 5% p.a. Another shareholder holding 2,000 share did not pay the final call money till end of the accounting year which ends on 30th June. Show the Cash Book and Journal Entries.

wonder ltd issued 10 000 12 preference shares of rs 100 each at a premium of rs 10 p 674525

Wonder Ltd. issued 10,000, 12% Preference Shares of Rs.100 each at a premium of Rs.10 per share payable as follows:

On Application

Rs.30

On Allotment

Rs.30 (including premium

On First Call

Rs.25

On Final Call

Rs.25

The application were received for 12,000 shares and the directors allotted 10,000 shares and rejected 2,000 shares with the money received thereon refunded.

The allotment money was duly received while the first call money was received on 9,000 shares and the final call money on 8,000 shares.

Show the cash book and journal entries and prepare the balance sheet of the company.

elegant ltd issued at 25 000 equity shares of rs 10 each at a discount of 10 payable 674526

Elegant Ltd. issued at 25,000 equity shares of Rs.10 each at a discount of 10% payable as follows:

On Application

Rs.3.00 per share

On Allotment

Rs.1.00 per share

On First Call

Rs.2.50 per share

On Final Call

Rs.2.50 per share

Applications were received for 30,000 shares and the directors allotted 25,000 shares and refunded the excess application money for 5,000 shares.

The allotment money was duly received on all the shares. One shareholder holding 1,000 shares did not pay the first call money while another shareholder holding 200 shares paid the final call money along with the first call money. The company did not make the final call.

Show the Cash Book, Journal entries and prepare the Balance Sheet of the Company.

a nuclear power company is deciding whether or not to build a nuclear power plant at 672756

A nuclear power company is deciding whether or not to build a nuclear power plant at Diablo Canyon or at Roy Rogers City. The cost of building the power plant is $10 million at Diablo and $20 million at Roy Rogers City. If the company builds at Diablo, however, and an earthquake occurs at Diablo during the next five years, construction will be terminated and the company will lose $10 million (and will still have to build a power plant at Roy Rogers City). A priori, the company believes there is a 20% chance that an earthquake will occur at Diablo during the next five years. For $1 million, a geologist can be hired to analyze the fault structure at Diablo Canyon. He will either predict that an earthquake will occur or that an earthquake will not occur. The geologist’s past record indicates that he will predict an earthquake on 95% of the occasions for which an earthquake will occur and no earthquake on 90% of the occasions for which an earthquake will not occur. Should the power company hire the geologist? Also find EVSI and EVPI.

the government is attempting to determine whether immigrants should be tested for a 672763

The government is attempting to determine whether immigrants should be tested for a contagious disease. Let’s assume that the decision will be made on a financial basis. Assume that each immigrant who is allowed into the country and has the disease costs the United States $100,000, and each immigrant who enters and does not have the disease will contribute $10,000 to the national economy. Assume that 10% of all potential immigrants have the disease. The government may admit all immigrants, admit no immigrants, or test immigrants for the disease before determining whether they should be admitted. It costs $100 to test a person for the disease; the test result is either positive or negative. If the test result is positive, the person definitely has the disease. However, 20% of all people who do have the disease test negative. A person who does not have the disease always tests negative. The government’s goal is to maximize (per potential immigrant) expected benefits minus expected costs. Use a decision tree to aid in this undertaking. Also determine EVSI and EVPI.

many colleges face the problem of whether athletes should be tested for drug use def 672764

Many colleges face the problem of whether athletes should be tested for drug use. Define

c1 = Cost if athlete is falsely accused of drug use

c2 = Cost if a drug user is not identified

c3 = Cost due to invasion of privacy if a nonuser is tested

Suppose that 5% of all athletes are drug users, and that the test used is 90% reliable. This means that if an athlete uses drugs, there is a 90% chance that the test will detect it, and if the athlete does not use drugs, there is a 90% chance that the test will show no drug use.

a If c1 = 10, c2 = 5, and c3 = 1, should the college test athletes for drugs?

b Prove that if c1 > c2 > c3, then the college should not test for drugs.

keeney and raiffa 1976 discuss the assessment of a blood bank rsquo s multiattribute 672767

Keeney and Raiffa (1976) discuss the assessment of a blood bank’s multiattribute utility function. For simplicity, we assume that the blood bank must determine at the beginning of each week how many pints of blood should be ordered. Any blood left over at the end of the week spoils (it is outdated). For the blood bank, two attributes of interest are as follows:

Attribute 1 Number of pints of blood by which ordered blood falls short of the week’s demand (the weekly shortage). The weekly shortage is known to be always between 0 and 10 pints.

Attribute 2 Number of pints of blood that are outdated (known to be always between 0 and 10 pints) Assume that attributes 1 and 2 exhibit mui.

a Suppose the blood bank is indifferent between

 width=

Let x1 = value of attribute 1, and x2 = value of attribute 2. Also suppose that

 width=

Determine the blood bank’s multiattribute utility function.

b Suppose that each week there is a ½ chance that the demand for blood will be 25 pints and a ½ chance it will be 35 pints. Would the blood bank be better off ordering 28 pints, 30 pints, or 32 pints?

public service indiana psi is considering two sites for a nuclear power plant the fo 672769

Public service Indiana (PSI) is considering two sites for a nuclear power plant. The following two attributes will influence its determination about where to build the plant: Attribute 1 Cost of the plant (in millions of dollars)

Attribute 2 Acres of land damaged by building the plant

Assume that PSI’s multiattribute utility function is given by u1(x1, x2) = .70u1(x1) + .20u2(x2) + .10u1(x1)u2(x2), where u1(x1) = .1 + exp( .1x1) and u2(x2) 2.5 2.5 exp(.0006x2  .48).

Two locations for the power plant are under consideration. Location 1 is equivalent to the following lottery:

 width=

and location 2 is equivalent to the following lottery:

 width=

Which location should be chosen?

consider the four points a b c and d in figure 16 672770

Consider the four points A, B, C, and D in Figure 16.

 width= 

Assume that more of each attribute is desirable and that a decision maker’s utility function exhibits mui. Consider the following two lotteries:

 width=

a Show that if k3 > 0, then L1pL2.

b Show that if k3 < 0, then L2pL1.

c Show that if the decision maker exhibits additive independence (k3 = 0), then L1iL2.

d Let attribute 1 = performance of Germany on the eastern front near the end of World War II, and attribute 2 = performance of Germany on the western front. A high level of an attribute means that Germany did well, and a low level of an attribute means that Germany did poorly. Suppose that Germany will suffer defeat if it performs poorly on either front. If these attributes exhibit mui, what would be the sign of k3?

e General Motors has domestic and international divisions. Let attribute 1 = profits in the domestic division and attribute 2 = profits in the international division. Suppose General Motors is reasonably happy if at least one division has a good year but is very unhappy if both divisions have a bad year. If these attributes exhibit mui, what would be the sign of k3?

each professor rsquo s annual salary increase is determined by performance in three 672771

Each professor’s annual salary increase is determined by performance in three areas: teaching, research, and service to the university. The administration has come up with the following pairwise comparison matrix for these objectives:

 width=

The administration has compared two professors with regard to their teaching, research, and service over the past year. The pairwise comparison matrices are as follows. For teaching:

 width=

a Which professor should receive a bigger raise?

b Does the AHP indicate how large a raise each professor should be given?

c Check the pairwise comparison matrix for consistency.

a business is about to purchase a new personal computer three objectives are importa 672772

A business is about to purchase a new personal computer. Three objectives are important in determining which computer should be purchased: cost, user friendliness, and software availability. The pairwise comparison matrix for these objectives is as follows:

 width=

Three computers are being considered for purchase. The performance of each computer with regard to each objective is indicated by the following pairwise comparison matrices. For cost (low cost is good, high cost is bad!):

 width=

a Which computer should be purchased?

b Check the pairwise comparison matrices for consistency.

in determining where to invest my money two objectives mdash expected rate of return 672774

In determining where to invest my money, two objectives—expected rate of return and degree of risk—are considered equally important. Two investments (1 and 2) have the following pairwise comparison matrices: Expected return:

 width=

a How should I rank these investments?

b Now suppose another investment (investment 3) is available. Suppose the pairwise comparison matrices for these investments are as follows. Expected return:

 width=

c Observe that the entries in the comparison matrices for investments 1 and 2 have not changed. How should I now rank the investments? Contrast my ranking of investments 1 and 2 with the answer from part (a).

a consumer is trying to determine which type of frozen dinner to eat he considers th 672775

A consumer is trying to determine which type of frozen dinner to eat. He considers three attributes to be important: taste, nutritional value, and price. Nutritional value is considered to be determined by cholesterol and sodium levels. Three types of dinners are under consideration. The pairwise comparison matrix for the three attributes is as follows:

 width=

Between the three frozen dinners the pairwise comparison matrix for each attribute is as follows. For taste:

 width=

To determine how each dinner rates on nutrition you will need the following pairwise comparison matrix for cholesterol and sodium:

 width=

Which frozen dinner would he prefer?(Hint: Nutrition score for a dinner = (score of dinner on sodium) * (weight for sodium) + (score for dinner on cholesterol) * (weight for cholesterol).)

you are trying to determine which mba program to attend you have been accepted at tw 672776

You are trying to determine which MBA program to attend. You have been accepted at two programs: Indiana  and Northwestern. You have chosen three attributes to use in helping you make your decision:

Attribute 1 Cost

Attribute 1 Starting salary

Attribute 1 Ambience of school (can we party there?!!)

Your pairwise comparison matrix for these attributes is as follows:

 width=

For each attribute the pairwise comparison matrix for Indiana and Northwestern is as follows. For cost:

 width=

Which MBA program should you attend?

we have 1 000 to invest all the money must be placed in one of three investments gol 672779

We have $1,000 to invest. All the money must be placed in one of three investments: gold, stock, or money market certificates. If $1,000 is placed in an investment, the value of the investment one year from now depends on the state of the economy (see Table).

Value of $1,000

State 1

State 2

State 3

Money market

 

 

 

certificate

$1,100

$1,100

$1,100

Stock

$1,000

$1,100

$1,200

Gold

$1,600

$300

$1,400

Assume that each state of the economy is equally likely. For each of the following decision criteria, determine the optimal decision:

a maximin

b maximax

c minimax regret

d expected value

we have 1 000 to invest all the money must be placed in one of three investments gol 672780

We have $1,000 to invest. All the money must be placed in one of three investments: gold, stock, or money market certificates. If $1,000 is placed in an investment, the value of the investment one year from now depends on the state of the economy (see Table).

Value of $1,000

State 1

State 2

State 3

Money market

 

 

 

certificate

$1,100

$1,100

$1,100

Stock

$1,000

$1,100

$1,200

Gold

$1,600

$300

$1,400

Assume that each state of the economy is equally likely. For each of the following decision criteria, determine the optimal decision:

Suppose that the utility function for the value of the investment (x) one year from now is given by u(x) = ln x. Determine which investment we should choose. Could we have predicted this answer without a table of logarithms?

rollo megabux has 1 million to invest in stocks or bonds the percentage yield on eac 672783

Rollo Megabux has $1 million to invest in stocks or bonds. The percentage yield on each investment during the coming year depends on whether the economy has a good or a bad year (see Table).

 

Economy Has Good Year

Economy Has Bad Year

Yield on stocks

22%

10%

Yield on bonds

16%

14%

It is equally likely that the economy will have a good or a bad year.

a If Rollo is risk neutral, how should he invest his money?

b For $10,000, Rollo can hire a consulting firm to forecast the state of the economy. The consulting firm’s forecasts have the following properties:

P(good forecast|economy good) = .80

P(good forecast|economy bad) = .20

Should Rollo hire the consulting firm? What are EVSI and EVPI?

willy mutton has three potential bank robberies lined up his chance of success and t 672784

Willy Mutton has three potential bank robberies lined up. His chance of success and the size of the take are given in Table:

Robbery

Chance of Success

Size of Take (in millions of dollars)

1

.60

7

2

.80

6

3

.70

5

These robberies must be attempted in order; if you “pass” on a robbery you may not go on to the next robbery. If Willie is caught, he loses all his money. What strategy maximizes his expected “take”?

the pine valley board of education is trying to determine its multiattribute utility 672786

The Pine Valley Board of Education is trying to determine its multiattribute utility function with respect to the following attributes:

Attribute 1 Average score of students on an English achievement test

Attribute 2 Average score of students on a mathematics achievement test

The board believes that both attributes range between 70% and 90% correct answers. The board is indifferent between

 width=

For any level x2 of attribute 2, the board is also indifferent between

 width=

For any level x1 of attribute 1, the board is indifferent between

 width=

The board is also indifferent between

 width=

Finally, the board is indifferent between

 width=

The board must decide which of two instructional techniques should be utilized in the Pine Valley schools. Technique 1 is equivalent to the following lottery:

 width=

Technique 2 is equivalent to the following lottery:

 width=

Would the board prefer technique 1 or technique 2?

beattrop foods is trying to choose one of three companies to merge with in making th 672787

BeatTrop Foods is trying to choose one of three companies to merge with. In making this decision seven factors are important:

Factor 1 Contribution to profitability

Factor 2 Growth potential

Factor 3 Labor environment

Factor 4 R&D ability of company

Factor 5 Organizational fit

Factor 6 Relative size

Factor 7 Industry commonality

The pairwise comparison for these factors is as follows:

 width=

The three contenders for merger have the following pairwise comparison matrices for each factor:

 width=

Use the AHP to determine the company with which BeatTrop should prefer to merge.

you are trying to determine which city to live in new york and chicago are under con 672788

You are trying to determine which city to live in. New York and Chicago are under consideration. Four objectives will determine your decision: affordability of housing, cultural opportunities, quality of schools and universities, and crime level. The weight for each objective is in Table.

Affordability of housing

.50

Cultural opportunities

.10

Quality of schools and universities

.20

Crime level

.20

For each objective (except for quality of schools and universities) New York and Chicago scores are as given in Table.

 

New York

Chicago

Affordability of housing

.30

.70

Cultural opportunities

.70

.30

Crime level

.40

.60

Suppose that the score for each city on the quality of schools and universities depends on two things: a score on public school quality and a score on university quality. The pairwise comparison matrix for public school and university quality is as follows:

 width=

To see how each city scores on public school quality and university quality use the following pairwise comparison matrices. For public school quality:

 width=

For university quality:

 width=

You should now be able to come up with a score for each city on the quality of schools and universities objective. Now determine where you should live.

rollo megabux has 1 million to invest in stocks or bonds the percentage yield on eac 672789

Rollo Megabux has $1 million to invest in stocks or bonds. The percentage yield on each investment during the coming year depends on whether the economy has a good or a bad year (see Table).

 

Economy Has Good Year

Economy Has Bad Year

Yield on stocks

22%

10%

Yield on bonds

16%

14%

It is equally likely that the economy will have a good or a bad year.

Suppose Rollo cannot hire the consulting firm, and his utility function for ending cash position is u(x) = ln x. How much money should he invest in stocks and bonds?

question 2 consider the following information about a potential project 673757

Question #2 Consider the following information about a potential project: Investment required $15,000,000 Expected annual project revenue $17,000,000 Expected annual project expenses $14,500,000 Required rate of return 15% Current division return on investment 19% a) Calculate the project’s return on investment. b) Based solely on ROI, is this project in the firm’s best interests? Why or why not? c) Is this project in the division manager’s best interests? Why or why not? d) Perform DuPont Analysis on this project. e) What is the project’s residual income?

Document Preview:

Question #2 Consider the following information about a potential project: Investment required?$15,000,000??Expected annual project revenue?$17,000,000??Expected annual project expenses?$14,500,000??Required rate of return?15%??Current division return on investment?19%?? a) Calculate the project’s return on investment. b) Based solely on ROI, is this project in the firm’s best interests? Why or why not? c) Is this project in the division manager’s best interests? Why or why not? d) Perform DuPont Analysis on this project. e) What is the project’s residual income?

Attachments:

consider the following lease versus borrow and purchase problem 671268

Consider the following lease versus borrow and purchase problem:

• Borrow and purchase option:

1.Jensen Manufacturing Company plans to acquire sets of special industrial tools with a four year life and a cost of $200,000, delivered and installed. The tools will be depreciated by the MACRS three year classification.

2.Jensen can borrow the required $200,000 at a rate of 10% over four years. Four equal end of year annual payments would be made in the amount of The annual interest and principal payment schedule, along with the equivalent present worth of these payments, is as follows:

End of YearInterestPrincipal1$20,000$43,094215,96147,403310,95052,14445,73657,358

3.The estimated salvage value for the tool sets at the end of four years is $20,000.

4.If Jensen borrows and buys, it will have to bear the cost of maintenance, which will be performed by the tool manufacturer at a fixed contract rate of $10,000 per year.

• Lease option:

1.Jensen can lease the tools for four years at an annual rental charge of $70,000, payable at the end of each year.

2.The lease contract specifies that the lessor will maintain the tools at no additional charge to Jensen.

Jensen’s tax rate is 40%. Any gains will also be taxed at 40%.

(a) What is Jensen’s PW of after tax cash flow of leasing at i = 15%?

(b) What is Jensen’s PW of after tax cash flow of owning at i = 15%?

an asset is to be purchased for 25 000 the asset is expected to provide revenue of 1 671271

An asset is to be purchased for $25,000. The asset is expected to provide revenue of $10,000 a year and have operating costs of $2,500 a year. The asset is considered to be a seven year MACRS property. The company is planning to sell theasset at the end of year 5 for $5,000. Given that the company’s marginal tax rate is 30% and that it has a MARR of 10% for any project undertaken, answer the following questions:

(a) What is the net cash flow for each year, given that the asset is purchased with borrowed funds at an interest rate of 12%, with repayment in five equal endof year payments?

(b) What is the net cash flow for each year, given that the asset is leased at a rate of $3,500 a year (a financial lease)?

(c) Which method (if either) should be used to obtain the new asset?

national parts inc an auto parts manufacturer is considering purchasing a rapid prot 671275

National Parts, Inc., an auto parts manufacturer, is considering purchasing a rapid prototyping system to reduce prototyping time for form, fit, and function applicationsin automobile parts manufacturing. An outside consultant has been called in to estimate the initial hardware requirement and installation costs. He suggests the following:

• Prototyping equipment: $187,000.

• Posturing apparatus: $10,000.

• Software: $15,000.

• Maintenance: $36,000 per year by the equipment manufacturer.

• Resin: Annual liquid polymer consumption of 400 gallons at $350 per gallon.

• Site preparation: Some facility changes are required for the installation of the rapid prototyping system (e.g., certain liquid resins contain a toxic substance, so the work area must be well vented).

The expected life of the system is six years, with an estimated salvage value of $30,000. The proposed system is classified as a five year MACRS property. A group of computer consultants must be hired to develop customized software to run on the system. Software development costs will be $20,000 and can be expensed during the first tax year. The new system will reduce prototype development time by 75% and material waste (resin) by 25%. This reduction in development time and material waste will save the firm $114,000 and $35,000annually, respectively. The firm’s expected marginal tax rate over the next six years will be 40%. The firm’s interest rate is 20%.

(a) Assuming that the entire initial investment will be financed from the firm’sretained earnings (equity financing), determine the after tax cash flows over the life of the investment. Compute the NPW of this investment.

(b) Assuming that the entire initial investment will be financed through a localbank at an interest rate of 13% compounded annually, determine the net after tax cash flows for the project. Compute the NPW of the investment.

(c) Suppose that a financial lease is available for the prototype system at $62,560 per year, payable at the beginning of each year. Compute the NPW of the investment with lease financing.

(d) Select the best financing option, based on the rate of return on incremental investment.

suppose my utility function for asset position x is given by u x ln x 672720

Suppose my utility function for asset position x is given by u(x) = ln x.

a Am I risk averse, risk neutral, or risk seeking?

b I now have $20,000 and am considering the followingtwo lotteries:

L1:

With probability 1, I lose $1,000.

L2:

With probability .9, I gain $0.

 

With probability .1, I lose $10,000.

Determine which lottery I prefer and the risk premium of L2.

the allais paradox suppose we are offered a choice between the following two lotter 672726

(The Allais Paradox) Suppose we are offered a choice between the following two lotteries:

L1:

With probability 1, we receive $1 million.

L2:

With probability .10, we receive $5 million.

 

With probability .89, we receive $1 million.

 

With probability .01, we receive $0.

Which lottery do we prefer? Now consider the following two lotteries:

L3:

With probability .11, we receive $1 million.

 

With probability .89, we receive $0.

L4:

With probability .10, we receive $5 million.

 

With probability .90, we receive $0.

Which lottery do we prefer? Suppose (like most people), we prefer L1 to L2. Show that L3 must have a larger expected utility than L4.

although the von neumann ndash morgenstern axioms seem plausible there are many reas 672730

Although the Von Neumann–Morgenstern axioms seem plausible, there are many reasonable situations in which people appear to violate these axioms. For example, suppose a recent college graduate must choose between three job offers on the basis of starting salary, location of job, and opportunity for advancement. Given two job offers that are satisfactory with regard to all three attributes, the graduate will decide between two job offers by choosing the one that is superior on at least two of the three attributes. Suppose he or she has three job offers and has rated each one as shown in Table (E = excellent, G = good, and S = satisfactory).

 

Starting Salary

Location

Opportunity for Advancement

Job 1

E

S

G

Job 2

G

E

S

Job 3

S

G

E

Show that the graduate’s preferences among these jobs violate the Complete Ordering Axiom.

the allais paradox suppose we are offered a choice between the following two lotter 672732

(The Allais Paradox) Suppose we are offered a choice between the following two lotteries:

L1:

With probability 1, we receive $1 million.

L2:

With probability .10, we receive $5 million.

 

With probability .89, we receive $1 million.

 

With probability .01, we receive $0.

Which lottery do we prefer? Now consider the following two lotteries:

L3:

With probability .11, we receive $1 million.

 

With probability .89, we receive $0.

L4:

With probability .10, we receive $5 million.

 

With probability .90, we receive $0.

Explain how prospect theory and/or framing explainsthe Allais Paradox.

you are given a choice between lottery 1 and lottery 2 you are also given a choice b 672734

You are given a choice between lottery 1 and lottery 2. You are also given a choice between lottery 3 and lottery 4.

Lottery 1:

A sure gain of $240

Lottery 2:

25% chance to gain $1,000 and 75% chance to gain nothing

Lottery 3:

A sure loss of $750

Lottery 4:

A 75% chance to lose $1,000 and a 25% chance of losing nothing

84% of all people prefer lottery 1 over lottery 2, and 87% choose lottery 4 over lottery 3.

a Explain why the choice of lottery 1 over lottery 2 and lottery 4 over lottery 3 contradicts expected utility maximization. (Hint: Compare lottery 1 + lottery 4 to lottery 2 + lottery 3.)

b Can you explain this anomalous behavior?

tversky and kahneman asked 72 respondents to choose between lottery 1 and lottery 2 672735

Tversky and Kahneman asked 72 respondents to choose between lottery 1 and lottery 2 and lottery 3 and lottery 4.

Lottery 1:

A .001 chance at winning $5,000 and a .999 chance of winning $0

Lottery 2:

A sure gain of $5

Lottery 3:

A .001 chance of losing $5,000 and a .999 chance of losing $0

Lottery 4:

A sure loss of $5

More than 75% of all participants preferred lottery 1 to lottery 2 and lottery 4 to lottery 3.

a Which choices would be made by a risk averse decisionmaker?

b Which choices would be made by a risk seeking decision maker?

c How does the observed behavior of the participants contradict expected utility maximization?

d How does prospect theory resolve the contradiction?

during the summer olympic swimmer adam johnsonswims every day on sunny summer days h 672742

During the summer, Olympic swimmer Adam Johnsonswims every day. On sunny summer days, he goes to an outdoor pool, where he may swim for no charge. On rainy days, he must go to a domed pool. At the beginning of the summer, he has the option of purchasing a $15 season pass to the domed pool, which allows him use for the entire summer. If he doesn’t buy the season pass, he must pay $1 each time he goes there. Past meteorological records indicate that there is a 60% chance that the summer will be sunny (in which case there is an average of 6 rainy days during the summer) and a 40% chance the summer will be rainy (an average of 30 rainy days during the summer).

Before the summer begins, Adam has the option of purchasing a long range weather forecast for $1. The forecast predicts a sunny summer 80% of the time and a rainy summer 20% of the time. If the forecast predicts a sunny summer, there is a 70% chance that the summer will actually be sunny. If the forecast predicts a rainy summer, there is an 80% chance that the summer will actually be rainy. Assuming that Adam’s goal is to minimize his total expected cost for the summer, what should he do? Also find EVSI and EVPI.

erica is going to fly to london on august 5 and return home on august 20 it is now j 672744

Erica is going to fly to London on August 5 and return home on August 20. It is now July 1. On July 1, she may buy a one way ticket (for $350) or a round trip ticket (for $660). She may also wait until August 1 to buy a ticket. On August 1, a one way ticket will cost $370, and a round trip ticket will cost $730. It is possible that between July 1 and August 1, her sister (who works for the airline) will be able to obtain a free one way ticket for Erica. The probability that her sister will obtain the free ticket is .30. If Erica has bought a round trip ticket on July 1 and her sister has obtained a free ticket, she may return “half ” of her roundtrip to the airline. In this case, her total cost will be $330 plus a $50 penalty. Use a decision tree approach to determine how to minimize Erica’s expected cost of obtaining roundtrip transportation to London.

i am a contestant on the tv show remote jeopardy which works as follows i am first a 672745

I am a contestant on the TV show Remote Jeopardy, which works as follows. I am first asked a question about Stupid Videos. If I answer correctly, I earn $100. I believe that I have an 80% chance of answering such a question correctly. If I answer incorrectly, the game is over, and I win nothing. If I answer correctly, I may leave with $100 or go on and answer a question about Stupid TV Shows. If I answer this question correctly, I earn another $300, but if I answer incorrectly, I lose all previous earnings and am sent home. My chance of answering this question correctly is .60. If I answer the Stupid TV Shows question correctly, I may leave with my “earnings” or go on and answer a question about Statistics. If I answer this question correctly, I earn another $500, but if I answer it incorrectly, I lose all previous earnings and am sent home. My chance of answering this question correctly is .40. Draw a decision tree that can be used to maximize my expected earnings. What are my expected earnings?

the american chess master jonathan meller is playing the soviet expert yuri gasparov 672746

The American chess master Jonathan Meller is playing the Soviet expert Yuri Gasparov in a two game exhibition match. Each win earns a player one point, and each draw earns a half point. The player who has the most points after two games wins the match. If the players are tied after two games, they play until one wins a game; then the first player to win a game wins the match. During each game, Meller has two possible approaches: to play a daring strategy or to play a conservative strategy. His probabilities of winning, losing, and drawing when he follows each strategy are shown in Table.

Strategy

Win

Loss

Draw

Daring

.45

.55

0

Conservative

0

.10

.90

To maximize his probability of winning the match, what should the American do?

edwina a commodities broker has acquired an option to buy 1 000 oz of gold at 50 oz 672749

Edwina, a commodities broker, has acquired an option to buy 1,000 oz of gold at $50/oz. If she takes the option and if Congress relaxes import quotas, she can sell the gold for $80/oz. If she takes the option and Congress does not relax the import quotas, however, the company will lose $10/oz.

Edwina believes that there is a 50% chance that the government will relax the quota. She also has the option of waiting until Congress decides whether to relax the import quota. If she adopts this strategy, however, there is a 70% chance that some other broker will have already taken the option.

a If Edwina is risk neutral, what should she do?

b If Edwina’s utility function for a change x in her asset position is given by u(x) = (10,000 + x)1/2, what should she do?

we are going to see the movie fatal repulsion there are three parking lots we may pa 672750

We are going to see the movie Fatal Repulsion. There are three parking lots we may park in. One is one block east of the theater (call this lot 1); one lot is directly behind the theater (lot 0); and one lot is one block west of the theater (lot 1). We are approaching the theater from the east. There is an 80% chance that lot 1 will have a vacant space, a 60% chance that lot 0 will, and an 80% chance that lot 1 will. Once we pass a lot, we can’t go back to it. Assume that when we are at a given parking lot, we can determine whether it has any vacant spaces, but we can’t see any of the other lots. Our dates for the evening will assess us a penalty equal to the distance (in blocks) that we park from the theater. If we find no space, they will assess a penalty of 10 (and never go out with us again). What strategy minimizes our expected penalty? Answer the same question if there is a 70% chance that lot 0 has a vacant space.

a patient enters the hospital with severe abdominal pains based on past experience d 672751

A patient enters the hospital with severe abdominal pains. Based on past experience, Doctor Craig believes there is a 28% chance that the patient has appendicitis and a 72% chance that the patient has nonspecific abdominal pains. Dr. Craig may operate on the patient now or wait 12 hours to gain a more accurate diagnosis. In 12 hours, Dr. Craig will surely know whether the patient has appendicitis. The problem is that in the meantime, the patient’s appendix may perforate (if he has appendicitis), thereby making the operation much more dangerous. Again based on past experience, Dr. Craig believes that if he waits 12 hours, there is a 6% chance that the patient will end up with a perforated appendix, a 22% chance the patient will end up with “normal” appendicitis, and a 72% chance that the patient will end up with nonspecific abdominal pain. From past experience, Dr. Craig assesses the probabilities shown in Table of the patient dying.

Situation

Probability That Patient Will Die

Operation on patient

.0009

with appendicitis

 

Operation on patient

.0004

with nonspecific

 

abdominal pain

 

Operation on perforated

.0064

appendix

 

No operation on patient

0

with nonspecific

 

abdominal pain

 

Assume that Dr. Craig’s goal is to maximize the probability that the patient will survive. Use a decision tree to help Dr. Craig make the right decision.

suppose you are given a choice between the following options 672752

a Suppose you are given a choice between the following options:

A1:

Win $30 for sure

A2:

80% chance of winning $45 and 20% chance of A2: winning nothing

B1:

25% chance of winning $30

B2:

20% chance of winning $45

Most people prefer A1 to A2 and B2 to B1. Explain why this behavior violates the assumption that decision makers maximize expected utility.

b Now suppose you play the following game: You have a 75% chance of winning nothing and a 25% chance of playing the second stage of the game. If you reach the second stage, you have a choice of two options (C1 and C2), but your choice must be made now, before you reach the second stage.

C1: Win $30 for sure

C2: 80% chance of winning $45

Most people choose C1 over C2 and B2 to B1 (from part (a)). Explain why this again violates the assumption of expected utility maximization. Tversky and Kahneman (1981) speculate that most people are attracted to the sure $30 in the second stage, even though the second stage may never be reached! Note that B1 and C1 both give $30 with the same probability, and B2 and C2 both yield $45 with the same probability. It appears that people do not act very rationally!

 

you have just been chosen to appear on hoosier millionaire the rules are as follows 672753

You have just been chosen to appear on Hoosier Millionaire! The rules are as follows: There are four hidden cards. One says “STOP” and the other three have dollar amounts of $150,000, $200,000, and $1,000,000. You get to choose a card. If the card says “STOP,” you win no money. At any time you may quit and keep the largest amount of money that has appeared on any card you have chosen, or continue. If you continue and choose the stop card, however, you win no money. As an example, you may first choose the $150,000 card, then the $200,000 card, and then you may choose to quit and receive $200,000!

a If you goal is to maximize your expected payoff, what strategy should you follow?

b My utility function for an increase in cash satisfies u(0) = 0, u($40,000) = .25, u($120,000) = .50, u($400,000) = .75, and u($1,000,000) = 1. After drawing a curve through these points, determine a strategy that maximizes my expected utility. You might want to use your own utility function.

a customer has approached a bank for a 50 000 oneyear loan at 12 interest if the ban 672755

A customer has approached a bank for a $50,000 oneyear loan at 12% interest. If the bank does not approve the loan, the $50,000 will be invested in bonds that earn a 6% annual return. Without further information, the bank feels that there is a 4% chance that the customer will totally default on the loan. If the customer totally defaults, the bank loses $50,000. At a cost of $500, the bank can thoroughly investigate the customer’s credit record andsupply a favorable or unfavorable recommendation. Past experience indicates that

p(favorable recommendation| customer does not default) =  width=

p(favorable recommendation| customer defaults) =  width=

How can the bank maximize its expected profits? Also find EVSI and EVPI.

alcoa aluminum rsquo s mccook plant produces aluminum coils sheets and plates its an 671237

Project Cash Flows for a Cost Only Project

Alcoa Aluminum’s McCook plant produces aluminum coils, sheets, and plates. Its annual production runs at 400 million pounds. In an effort to improve McCook’s current production system, an engineering team, led by the divisional vice president, went on a fact finding tour of Japanese aluminum and steel companies to observe their production systems and methods. Cited among the team’s observations were large fans which the Japanese companies used to reduce the time that coils need to cool down after various processing operations. Cooling the hot process coils with the fans was estimated to significantly reduce the queue or work in process (WIP) inventory buildup allowed for cooling. The approach also reduced production lead time and improved delivery performance. The possibility of reducing production time and, as a consequence, the WIP inventory, excited the team members, particularly the vice president. After the trip, Neal Donaldson, the plant engineer, was asked toinvestigate the economic feasibility of installing cooling fans at the McCook plant. Neal’s job is to justify the purchase of cooling fans for his plant. He was given one week to prove that the idea was a good one. Essentially, all he knew was the number of fans, their locations, and the project cost. Everything else was left to Neal’s devices. Suppose that he compiled the following financial data:

• The project will require an investment of $536,000 in cooling fans now.

• The cooling fans would provide 16 years of service with no appreciable salvage values, considering the removal costs.

• It is expected that the amount of time required between hot rolling and the next operation would be reduced from five days to two days. Cold rolling queue time also would be reduced, from two days to one day for each cold roll pass. The net effect of these changes would be a reduction in the WIP inventory at a value of $2,121,000. Because of the lead time involved in installing the fans, as well as the consumption of stockpiled WIP inventory, this working capital release will be realized one year after the fans are installed.

• The cooling fans will be depreciated according to seven year MACRS.

• Annual electricity costs are estimated to rise by $86,000.

• The firm’s after tax required rate of return is known to be 20% for this type of cost reduction project. Develop the project cash flows over the service period of the fans, and determine whether the investment is a wise one, based on 20% interest.

langley manufacturing company lmc a manufacturer of fabricated metal products is con 671238

A Project Requiring Multiple Assets

Langley Manufacturing Company (LMC), a manufacturer of fabricated metal products, is considering purchasing a new computer controlled milling machine to produce a custom ordered metal product. The following summarizes the relevant financial data related to the project:

• The machine costs $90,000. The costs for its installation, site preparation, and wiring are expected to be $10,000. The machine also needs special jigs and dies, which will cost $12,000. The milling machine is expected to last 10 years, the jigs and dies 5 years. The machine will have a $10,000 salvage value at the end of its life. The special jigs and dies are worth only $1,000 as scrap metal at any time in their lives. The milling machine is classified as a 7 year MACRS property and the jigs and dies as a 3 year MACRS property.

• LMC needs to either purchase or build an8,000 ft2 warehouse in which to store the product before it is shipped to the customer. LMC has decided to purchase a building near the plant at a cost of $160,000. For depreciation purposes, the warehouse cost of $160,000 is divided into $120,000 for the building (39 year real property) and $40,000 for land. At the end of 10 years, the building will have a salvage value of $80,000, but the value of the land will have appreciated to $110,000.

• The revenue from increased production is expected to be $150,000 per year. The additional annual production costs are estimated as follows: materials, $22,000; labor, $32,000; energy $3,500; and other miscellaneous costs, $2,500.

For the analysis, a 10 year life will be used. LMC has a marginal tax rate of 40% and a MARR of 18%. No money is borrowed to finance the project. Capital gains will also be taxed at 40%.

consider again example 10 4 use the generalized cash flow approach to obtain the aft 671239

Generalized Cash Flow Approach

Consider again Example 10.4. Use the generalized cash flow approach to obtain the after tax cash flows:

TABLE 10.7 Cash Flow Statement for Example 10.4 Using the Generalized Cash Flow Approach

012345Investment$(125,000)  Net proceeds from sale  $43,387Investment in   working capital(23,331)  Recovery of   working capital  23,331(Revenue) (1 0.40)$ 60,000$ 60,000$ 60,000 $ 60,000 $ 60,000(Expenses) (1 0.40)(24,000)(24,000)(24,000) (24,000) (24,000)(Debt interest) (1 0.40)(3,750)(3,136)(2,460) (1,717) (899)(Depreciation)+(0.40)7,14512,2458,745 6,245 2,232Borrowed funds62,500  Repayment of principal(10,237)(11,261)(12,387) (13,626) (14,988)Net cash flow$ (85,831) $ 29,158$ 33,849$ 29,898 $ 26,903 $ 89,063       

you are considering a luxury apartment building project that requires an investment 671241

Generating Net Cash Flows

You are considering a luxury apartment building project that requires an investment of $12,500,000. The building has 50 units. You expect the maintenance cost for the apartment building to be $250,000 the first year and $300,000 the second year, after which it will continue to increase by $50,000 in subsequent years. The cost to hire a manager for the building is estimated to be $80,000 per year. After five years of operation, the apartment building can be sold for $14,000,000. What is the annual rent per apartment unit that will provide a return on investment of 15% after tax? Assume that the building will remain fully occupied during the five years. Assume also that your tax rate is 35%. The building will be depreciated according to 39 year MACRS and will be placed in service in January during the first year of ownership and sold in December during the fifth year of ownership.

a los angeles company is planning to market an answering device for people working a 671244

A Los Angeles company is planning to market an answering device for people working alone who want the prestige that comes with having a secretary, but who cannot afford one. The device, called Tele Receptionist, is similar to a voice mail system. It uses digital recording technology to create the illusion that a person is operating the switchboard at a busy office. The company purchased a 40,000 ft2   building and converted it to an assembly plant for $600,000 ($100,000 worth of land and $500,000 worth of building). Installation of the assembly equipment worth $500,000 was completed on December 31. The plant will begin operation on January 1. The company expects to have a gross annual income of $2,500,000 over the next 5 years. Annual manufacturing costs and all other operating expenses (excluding depreciation) are projected to be $1,280,000. For depreciation purposes, the assembly plant building will be classified as 39 year real property and the assembly equipment as a 7 year MACRS property. The property value of the land and the building at the end of year 5 would appreciate as much as 15% over the initial purchase cost. The residual value of the assembly equipment is estimated to be about $50,000 at the end of year 5. The firm’s marginal tax rate is expected to be about 40% over the project period. Determine the project’s after tax cash flows over the period of 5 years.

an automaker is considering installing a three dimensional 3 d computerized car styl 671249

An automaker is considering installing a three dimensional (3 D) computerized car styling system at a cost of $200,000 (including hardware and software). With the 3 D computer modeling system, designers will have the ability to view their design from many angles and to fully account for the space required for the engine and passengers. The digital information used to create the computer model can be revised in consultation with engineers, and the data can be used to run milling machines that make physical models quickly and precisely. The automaker expects to decrease the turnaround time for designing a new automobile model (from configuration to final design) by 22%. The expected savings in dollars is $250,000 per year. The training and operating maintenance cost for the new system is expected to be $50,000 per year. The system has a five year useful life and can be depreciatedaccording to five year MACRS class. The system will have an estimated salvage value of $5,000. The automaker’s marginal tax rate is 40%. Determine the annual cash flows for this investment. What is the return on investment for the project?

investment in working capital 671252

Investment in Working Capital

The Doraville Machinery Company is planning to expand its current spindle product line. The required machinery would cost $500,000. The building that will house the new production facility would cost $1.5 million. The land would cost $250,000, and $150,000 working capital would be required. The product is expected to result in additional sales of $675,000 per year for 10 years, at which time the land can be sold for $500,000, the building for $700,000, and the equipment for $50,000. All of the working capital will be recovered. The annual disbursements for labor, materials, and all other expenses are estimated to be $425,000. The firm’s income tax rate is 40%, and any capital gains will be taxed at 35%. The building will be depreciated according to a 39 year property class. The manufacturing facility will be classified as a 7 year MACRS. The firm’s MARR is known to be 15% after taxes.

(a) Determine the projected net after tax cash flows from this investment. Is the expansion justified?

(b) Compare the IRR of this project with that of a situation with no working capital.

delaware chemical corporation is considering investing in a new composite material 671254

Delaware Chemical Corporation is considering investing in a new composite material. R&D engineers are investigating exotic metal–ceramic and ceramic–ceramic composites to develop materials that will withstand high temperatures, such as those to be encountered in the next generation of jet fighter engines. The company expects a 3 year R&D period before these new materials can be applied to commercial products. The following financial information is presented for management review:

• R&D cost. $5 million over a 3 year period. Annual R&D expenditure of $0.5 million at the beginning of year 1, $2.5 million at the beginning of year 2, and $2 million at the beginning of year 3. For tax purposes, these R&D expenditures will be expensed rather than amortized.

• Capital investment. $5 million at the beginning of year 4. This investment consists of $2 million in a building and $3 million in plant equipment. The company already owns a piece of land as the building site.

• Depreciation method. The building (39 year real property class with the asset placed in service in January) and plant equipment (7 year MACRS recovery class).

• Project life. 10 years after a 3 year R&D period.

• Salvage value. 10% of the initial capital investment for the equipment and 50% for the building (at the end of the project life).

• Total sales. $50 million (at the end of year 4), with an annual sales growth rate of 10% per year (compound growth) during the next 5 years (year 5 through year 9) and 10% (negative compound growth) per year for the remaining project life.

• Out of pocket expenditures. 80% of annual sales.

• Working capital. 10% of annual sales (considered as an investment at the beginning of each production year and investments fully recovered at the end of the project life).

• Marginal tax rate. 40%.

• Determine the net after tax cash flows over the project life.

• Determine the IRR for this investment.

• Determine the equivalent annual worth for the investment at= 20%. 

in problem 10 2 to finance the industrial robot the company will borrow the entire a 671255

In Problem 10.2, to finance the industrial robot, the company will borrow the entire amount from a local bank, and the loan will be paid off at the rate of $37,000 per year, plus 10% on the unpaid balance. Determine the net after tax cash flows over the project life.

Cash Flow Statement012Operating activities   Net income 10,40012,019Depreciation 6,6664,445Investment activities   Investment 20,000  Salvage  8,000Gains tax (40%)   Financial activities   Borrowed funds10,000  Principal repayment0  Net cash flow $10,000  

consider the following financial information about a retooling project at a computer 671257

Consider the following financial information about a retooling project at a computer manufacturing company:

• The project costs $2 million and has a five year service life.

• The retooling project can be classified as seven year property under the MACRS rule.

• At the end of the fifth year, any assets held for the project will be sold. The expected salvage value will be about 10% of the initial project cost.

• The firm will finance 40% of the project money from an outside financial institution at an interest rate of 10%. The firm is required to repay the loan with five equal annual payments.

• The firm’s incremental (marginal) tax rate on the investment is 35%.

• The firm’s MARR is 18%.

• With the preceding financial information,

(a) Determine the after tax cash flows.

(b) Compute the annual equivalent worth for this project.

a construction company is considering acquiring a new earthmover the purchase price 671261

A construction company is considering acquiring a new earthmover. The purchase price is $100,000, and an additional $25,000 is required to modify the equipment for special use by the company. The equipment falls into the MACRS seven year classification (the tax life), and it will be sold after five years (the project life) for $50,000. The purchase of the earthmover will have no effect on revenues, but them a chine is expected to save the firm $60,000 per year in before tax operating costs, mainly labor. The firm’s marginal tax rate is 40%. Assume that the initial investment is to be financed by a bank loan at an interest rate of 10%, payable annually. Determine the after tax cash flows by using the generalized cash flow approach and the worth of the investment for this project if the firm’s MARR is known to be 12%.

air south a leading regional airline that is now carrying 54 of all the passengers t 671262

Air South, a leading regional airline that is now carrying 54% of all the passengers that pass through the Southeast, is considering adding a new long range aircraft to its fleet. The aircraft being considered for purchase is the McDonnell Douglas DC 9 532 “Funjet,” which is quoted at $60 million per unit. McDonnell Douglas requires a 10% down payment at the time of delivery, and the balance is to be paid over a 10 year period at an interest rate of 12% compounded annually. The actual payment schedule calls for only interest payments over the 10 year period, with the original principal amount to be paid off at the end of the 10th year. Air South expects to generate $35 million per year by adding this aircraft to its current fleet, but also estimates an operating and maintenance cost of $20 million per year. The aircraft is expected to have a 15 year service life with a salvage value of 15% of the original purchase price. If the aircraft is bought, it will be depreciated by the 7 year MACRS property classifications. The firm’s combined federal and state marginal tax rate is 38%, and its required minimum attractive rate of return is 18%.

(a) Use the generalized cash flow approach to determine the cash flow associated with the debt financing.

(b) Is this project acceptable?

the pittsburgh division of vermont machinery inc manufactures drill bits one of the 671263

Comparing Mutually Exclusive Alternatives

The Pittsburgh Division of Vermont Machinery, Inc., manufactures drill bits. One of the production processes of a drill bit requires tipping, whereby carbide tips are inserted into the bit to make it stronger and more durable. The tipping process usually requires four or five operators, depending on the weekly workload. The same operators are assigned to the stamping operation, in which the size of the drill bit and the company’s logo are imprinted into the bit. Vermont is considering acquiring three automatic tipping machines to replace the manual tipping and stamping operations. If the tipping process is automated, Vermont engineers will have to redesign the shapes of the carbide tips to be used in the machines. The new design requires less carbide, resulting in a savings of material. The following financial data have been compiled:

• Project life: six years.

• Expected annual savings: reduced labor, $56,000; reduced material, $75,000; other benefits (reduction in carpal tunnel syndrome and related problems), $28,000; reduced overhead, $15,000.

• Expected annual O&M costs: $22,000.

• Tipping machines and site preparation: equipment costs (three machines), including delivery, $180,000; site preparation, $20,000.

• Salvage value: $30,000 (three machines) at the end of six years.

• Depreciation method: seven year MACRS.

• Investment in working capital: $25,000 at the beginning of the project year, and that same amount will be fully recovered at the end of the project year.

• Other accounting data: marginal tax rate of 39% and MARR of 18%. To raise $200,000, Vermont is considering the following financing options:

Option 1. Use the retained earnings of the tipping machines to finance them.

Option 2. Secure a 12% term loan over six years (to be paid off in six equal annual installments).

Option 3. Lease the tipping machines. Vermont can obtain a six year financial lease on the equipment (with, however, no maintenance service) for payments of $55,000 at the beginning of each year.

(a) Determine the net after tax cash flows for each financing option.

(b) What is Vermont’s present value cost of owning the equipment by borrowing?

(c) What is Vermont’s present value cost of leasing the equipment?

(d) Recommend the best course of action for Vermont.

the jacob company needs to acquire a new lift truck for transporting its final produ 671266

Lease versus Buy Decisions

The Jacob Company needs to acquire a new lift truck for transporting its final product to the warehouse. One alternative is to purchase the truck for $40,000, which will be financed by the bank at an interest rate of 12%. The loan must be repaid in four equal installments, payable at the end of each year. Under the borrowto purchase arrangement, Jacob would have to maintain the truck at a cost of $1,200, payable at year end. Alternatively, Jacob could lease the truck under a four year contract for a lease payment of $11,000 per year. Each annual lease payment must be made at the beginning of each year. The truck would be maintained by the lessor. The truck falls into the five year MACRS classification, and it has a salvage value of $10,000, which is the expected market value after four years, at which time Jacob plans to replace the truck irrespective of whether it leases or buys. Jacob has a marginal tax rate of 40% and a MARR of 15%.

(a) What is Jacob’s cost of leasing, in present worth?

(b) What is Jacob’s cost of owning, in present worth?

(c) Should the truck be leased or purchased? This is an operating lease, so the truck would be maintained by the lessor.

janet wigandt an electrical engineer for instrument control inc ici has been asked t 671267

Janet Wigandt, an electrical engineer for Instrument Control, Inc. (ICI), has been asked to perform a lease–buy analysis of a new pin inserting machine for ICI’s PC board manufacturing.

Buy Option. The equipment costs $120,000. To purchase it, ICI could obtain a term loan for the full amount at 10% interest, payable in four equal end of year annual installments. The machine falls into a five year MACRS property classification.

· Annual revenues of $200,000 and operating costs of $40,000 are anticipated. The machine requires annual maintenance at a cost of $10,000. Because technology is changing rapidly in pin inserting machinery, the salvage value of the machine is expected to be only $20,000.

Lease Option. Business Leasing, Inc. (BLI), is willing to write a four year operating lease on the equipment for payments of $44,000 at the beginning of each year. Under this arrangement, BLI will maintain the asset, so that the annual maintenance cost of $10,000 will be saved. ICI’s marginal tax rate is 40%, and its MARR is 15% during the analysis period.

(a) What is ICI’s present value (incremental) cost of owning the equipment?

(b) What is ICI’s present value (incremental) cost of leasing the equipment?

(c) Should ICI buy or lease the equipment?

on january 2 2000 allen flour company purchased a new machine at a cost of 63 000 in 671229

On January 2, 2000, Allen Flour Company purchased a new machine at a cost of $63,000. Installation costs for the machine were $2,000. The machine was expected to have a useful life of 10 years, with a salvage value of $4,000. The company uses straight line depreciation for financial reporting. On January 3, 2003, the machine broke down, and an extraordinary repair had to be made to the machine at a cost of $6,000. The repair extended the machine’s life to 13 years, but left the salvage value unchanged. On January 2, 2006, an improvement was made to the machine in the amount of $3,000 that increased the machine’s productivity and increased the salvage value (to $6,000), but did not affect the remaining useful life. Determine depreciation expenses every December 31 for the years 2000, 2003, and 2006.

on march 17 2003 wildcat oil company began operations at its louisiana oil field the 671230

On March 17, 2003, Wildcat Oil Company began operations at its Louisiana oil field. The oil field had been acquired several years earlier at a cost of $11.6 million. The field is estimated to contain 4 million barrels of oil and to have a salvage value of $2 million both before and after all of the oil is pumped out. Equipment costing $480,000 was purchased for use at the oil field. The equipment will have no economic usefulness once the Louisiana field is depleted; therefore, it is depreciated on a units of production method. In addition,Wildcat Oil built a pipeline at a cost of $2,880,000 to serve the Louisiana field. Although this pipeline is physically capable of being used for many years, its economic usefulness is limited to the productive life of the Louisiana field; therefore, the pipeline has no salvage value. Depreciation of the pipeline is based on the estimated number of barrels of oil to be produced. Production at the Louisiana oil field amounted to 420,000 barrels in 2006 and 510,000 barrels in 2007.

(a) Compute the per barrel depletion rate of the oil field during the years 2006 and 2007.

(b) Compute the per barrel depreciation rates of the equipment and the pipeline during the years 2006 and 2007.
Q494

At the beginning of the fiscal year, Borland Company acquired new equipment at a cost of $65,000. The equipment has an estimated life of five years and an estimated salvage value of $5,000.

(a) Determine the annual depreciation (for financial reporting) for each of the five years of the estimated useful life of the equipment, the accumulated depreciation at the end of each year, and the book value of the equipment at the end of each year, all by (1) the straight line method, and (2) the double declining balance method.

(b) Determine the annual depreciation for tax purposes, assuming that the equipment falls into a seven year MACRS property class.

(c) Assume that the equipment was depreciated under seven year MACRS. In the first month of the fourth year, the equipment was traded in for similar equipment priced at $82,000. The trade in allowance on the old equipment was $10,000, and cash was paid for the balance. What is the cost basis of the new equipment for computing the amount of depreciation for income tax purposes?

electronic measurement and control company emcc has developed a laser speed detector 671231

Electronic Measurement and Control Company (EMCC) has developed a laser speed detector that emits infrared light, which is invisible to humans and radar detectors alike. For full scale commercial marketing, EMCC needs to invest $5 million in new manufacturing facilities. The system is priced at $3,000 per unit. The company expects to sell 5,000 units annually over the next five years. The new manufacturing facilities will be depreciated according to a seven year MACRS property class. The expected salvage value of the manufacturing facilities at the end of five years is $1.6 million. The manufacturing cost for the detector is $1,200 per unit, excluding depreciation expenses. The operating and maintenance costs are expected to run to $1.2 million per year. EMCC has a combined federal and state income tax rate of 35%, and undertaking this project will not change this current marginal tax rate.

(a) Determine, for the next five years, the incremental taxable income, income taxes, and net income due to undertaking this new product.

(b) Determine the gains or losses associated with the disposal of the manufacturing facilities at the end of five years.

diamonid is a start up diamond coating company that is planning to manufacture a mic 671232

Diamonid is a start up diamond coating company that is planning to manufacture a microwave plasma reactor which synthesizes diamonds. Diamonid anticipates that the industry demand for diamonds will skyrocket over the next decade, for use in industrial drills, high performance microchips, and artificial human joints, among other things. Diamonid has decided to raise $50 million through issuing common stocks for investment in plant ($10 million) and equipment ($40 million). Each reactor can be sold at a price of $100,000 per unit. Diamonid can expect to sell 300 units per year during the next 8 years. The unit manufacturing cost is estimated at $30,000, excluding depreciation. The operating and maintenance cost for the plant is estimated at $12 million per year. Diamonid expects to phase out the operation at the end of 8 years, revamp the plant and equipment, and adopt a new diamond manufacturing technology. At that time, Diamonid estimates that the salvage values for the plant and equipment will be about 60% and 10% of the original investments, respectively. The plant and equipment will be depreciated according to 39 year real property (placed in service in January) and 7 year MACRS, respectively. Diamonid pays 5% of state and local income taxes on its taxable income. (a) If the 2006 corporate tax system continues over the project life, determine the combined state and federal income tax rate each year.

(b) Determine the gains or losses at the time the plant is revamped.

(c) Determine the net income each year over the plant life.

suppose that in example 10 1 the tool manufacturing company rsquo s annual revenue p 671234

Working Capital Requirements

Suppose that in Example 10.1 the tool manufacturing company’s annual revenue projection of $100,000 is based on an annual volume of 10,000 units (or 833 units per month). Assume the following accounting information:

Price (revenue) per unit$10Unit variable manufacturing costs: Labor$2Material$1.20Overhead$0.80Monthly volume833 unitsFinished goods inventory to maintain2 month supplyRaw materials inventory to maintain1 month supplyAccounts payable30 daysAccounts receivable60 days

The accounts receivable period of 60 days means that revenues from the current month’s sales will be collected two months later. Similarly, accounts payable of 30 days indicates that payment for materials will be made approximately one month after the materials are received. Determine the working capital requirement for this operation.

calculating average cost per mile as a 671155

Calculating Average Cost per Mile as a

Function of Mileage

Table 8.2 itemizes the operating and ownership costs associated with driving a passenger car by fixed, variable, and mixed classes. Note that the only change from the preceding list is in the depreciation amount. Using the given data, develop a cost–volume chart and calculate the average cost per mile as a function of the annual mileage.

TABLE 8.2 Cost Classification of Owning and Operating a Passenger Car

Cost ClassificationReferenceCost
Variable costs:  Standard miles per gallon20 miles/gallon Average fuel price per gallon$1.939/gallon Fuel and oil per mile $0.085Maintenance per mile $0.058Tires per mile $0.007
Annual fixed costs:  Insurance (comprehensive) $1,195License, registration, taxes $390Finance charge $740
Mixed costs:Depreciation  Fixed portion per year  (15,000 miles) $4,005Variable portion per mile  (above 15,000 miles) $0.185

break even volume analysis 671157

Break Even Volume Analysis

Sandstone Corporation has one of its manufacturing plants operating on a singleshift five day week. The plant is operating at its full capacity (24,000 units of output per week) without the use of overtime or extra shifts. Fixed costs for single shift operation amount to $90,000 per week. The average variable cost is a constant $30 per unit, at all output rates, up to 24,000 units per week. The company has received an order to produce an extra 4,000 units per week beyond the current single shift maximum capacity. Two options are being considered to fill the new order:

• Option 1. Increase the plant’s output to 36,000 units a week by adding overtime, by adding Saturday operations, or both. No increase in fixed costs is entailed, but the variable cost is $36 per unit for any output in excess of 24,000 units per week, up to a 36,000 unit capacity.

• Option 2. Operate a second shift.

The maximum capacity of the second shift is 21,000 units per week. The variable cost of the second shift is $31.50 per unit, and the operation of a second shift entails additional fixed costs of $13,500 per week. Determine the range of operating volume that will make Option 2 profitable.

opportunity cost lost rental income opportunity cost 671158

Opportunity Cost: Lost Rental Income (Opportunity Cost)

Benson Company is a farm equipment manufacturer that currently produces 20,000 units of gas filters annually for use in its lawn mower production. The expected annual production cost of the gas filters is summarized as follows:

Variable costs: Direct materials$100,000Direct labor190,000Power and water35,000Fixed costs: Heating and light20,000Depreciation100,000Total cost$445,000

Tompkins Company has offered to sell Benson 20,000 units of gas filters for $17.00 per unit. If Benson accepts the offer, some of the manufacturing facilities currently used to manufacture the filters could be rented to a third party at an annual rent of $35,000.Should Benson accept Tompkins’s offer, and why?

marginal costs versus average costs 671159

Marginal Costs versus Average Costs

Consider a company that has an available electric load of 37 horsepower and that purchases its electricity at the following rates:

 width=

 

According to this rate schedule, the unit variable cost in each rate class represents the marginal cost per kilowatt hours (kWh). Alternatively, we may determine the average costs in the third column by finding the cumulative total cost and dividing it by the total number of kWh (X). Suppose that the current monthly consumption of electric power averages 3,200 kWh. On the basis of this rate schedule, determine the marginal cost of adding one more kWh and, for a given operating volume (3,200 kWh), the average cost per kWh.

the accompanying figures are a number of cost behavior patterns that might be found 671163

The accompanying figures are a number of cost behavior patterns that might be found in a company’s cost structure. The vertical axis on each graph represents total cost, and the horizontal axis on each graph represents level of activity (volume). For each of the situations that follow, identify the graph that illustrates the cost pattern involved. Any graph may be used more than once5.

(a) Electricity bill—a flat rate fixed charge, plus a variable cost after a certain number of kilowatt hours are used.

(b) City water bill, which is computed as follows:

First 1,000,000 gallons $1,000 flat or less rate

Next 10,000 gallons $0.003 per gallon used

Next 10,000 gallons $0.006 per gallon used

Next 10,000 gallons $0.009 per gallon used

Etc. etc.

(c) Depreciation of equipment, where the amount is computed by the straight line method. When the depreciation rate was established, it was anticipated that the obsolescence factor would be greater than the wear and tear factor.

(d) Rent on a factory building donated by the city, where the agreement calls for a fixed fee payment unless 200,000 labor hours or more are worked, in which case no rent need be paid.

(e) Cost of raw materials, where the cost decreases by 5 cents per unit for each of the first 100 units purchased, after which it remains constant at $2.50 per unit.

(f) Salaries of maintenance workers, where one maintenance worker is needed for every 1,000 machine hours or less (that is, 0 to 1,000 hours requires one maintenance worker, 1,001 to 2,000 hours requires two maintenance workers, etc.).

(g) Cost of raw materials used.

(h) Rent on a factory building donated by the county, where the agreement calls for rent of $100,000, less $1 for each direct labor hour worked in excess of 200,000 hours, but a minimum rental payment of $20,000 must be paid.

(i) Use of a machine under a lease, where a minimum charge of $1,000 must be paid for up to 400 hours of machine time. After 400 hours of machine time, an additional charge of $2 per hour is paid, up to a maximum charge of $2,000 per period.

 width=

harris company manufactures a single product costs for the year 2001 for output leve 671164

Harris Company manufactures a single product. Costs for the year 2001 for output levels of 1,000 and 2,000 units are as follows:

Units produced1,0002,000Direct labor$30,000$30,000Direct materials20,00040,000Overhead:  Variable portion12,00024,000Fixed portion36,00036,000Selling and administrative costs:  Variable portion5,00010,000Fixed portion22,00022,000

At each level of output, compute the following:

(a) Total manufacturing costs.

(b) Manufacturing costs per unit.

(c) Total variable costs.

(d) Total variable costs per unit.

(e) Total costs that have to be recovered if the firm is to make a profit.

cost ndash volume ndash profit relationships 671165

Cost–Volume–Profit Relationships

Bragg & Stratton Company manufactures a specialized motor for chain saws. The company expects to manufacture and sell 30,000 motors in year 2001. It can manufacture an additional 10,000 motors without adding new machinery and equipment. Bragg & Stratton’s projected total costs for the 30,000 units are as follows:

Direct Materials$150,000Direct labor300,000Manufacturing overhead: Variable portion100,000Fixed portion80,000Selling and administrative costs: Variable portion180,000Fixed portion70,000

The selling price for the motor is $80.

(a) What is the total manufacturing cost per unit if 30,000 motors are produced?

(b) What is the total manufacturing cost per unit if 40,000 motors are produced?

(c) What is the break even price on the motors?

the accompanying chart shows the expected monthly profit or loss of cypress manufact 671166

The accompanying chart shows the expected monthly profit or loss of Cypress Manufacturing Company within the range of its monthly practical operating capacity. Using the information provided in the chart, answer the following questions:

(a) What is the company’s break even sales volume?

(b) What is the company’s marginal contribution rate?

(c) What effect would a 5% decrease in selling price have on the break even point in (a)?

(d) What effect would a 10% increase in fixed costs have on the marginal contribution rate in (b)?

(e) What effect would a 6% increase in variable costs have on the break even point in (a)?

(f) If the chart also reflects $20,000 monthly depreciation expenses, compute the sale at the break even point for cash costs.

 width=

a cost ndash volume ndash profit cvp graph is a useful technique for showing relatio 671168

A cost–volume–profit (CVP) graph is a useful technique for showing relationships between costs, volume, and profits in an organization.

 width=

(a) Identify the numbered components in the accompanying CVP graph.

No.DescriptionNo.Description16273849510

(b) Using the typical CVP relationship shown, fill in the missing amounts in each of the following four situations (each case is independent of the others):

   Contribution    VariableMargin perFixedNet IncomeCase Units SoldSalesExpensesUnitExpenses(Loss)A9,000$270,000$162,000 $ 90,000 B $350,000 $15$170,000$ 40,000C20,000 $280,000$ 6 $ 35,000D5,000$100,000  $ 82,000($12,000)

the morton company produces and sells two products a and b financial data related to 671170

The Morton Company produces and sells two products: A and B. Financial data related to producing these two products are summarized as follows:

 
Product A
Product BSelling price$ 10.00$12.00Variable costs$ 5.00$10.00Fixed costs$ 2,000$ 600

(a) If these products are sold in the ratio of 4A for 3B, what is the break even point?

(b) If the product mix has changed to 5A for 5B, what would happen to the breakeven point?

(c) In order to maximize the profit, which product mix should be pushed?

(d) If both products must go through the same manufacturing machine and there are only 30,000 machine hours available per period, which product should be pushed? Assume that product A requires 0.5 hour per unit and B requires 0.25 hour per unit.

pearson company manufactures a variety of electronic printed circuit boards pcbs tha 671171

Pearson Company manufactures a variety of electronic printed circuit boards (PCBs) that go into cellular phones. The company has just received an offer from an outside supplier to provide the electrical soldering for Pearson’s Motorola product line (Z 7 PCB, slimline). The quoted price is $4.80 per unit. Pearson is interested in this offer, since its own soldering operation of the PCB is at its peak capacity.

Outsourcing option. The company estimates that if the supplier’s offer were accepted, the direct labor and variable overhead costs of the Z 7 slimline would be reduced by 15% and the direct material cost would be reduced by 20%.

In house production option. Under the present operations, Pearson manufactures all of its own PCBs from start to finish. The Z 7 slimlines are sold through Motorola at $20 per unit.

Fixed overhead charges to the Z 7 slimline total $20,000 each year. The further breakdown of producing one unit is as follows:

Direct materials$ 7.50Direct labor5.00Manufacturing overhead Total cost4.00 $16.00

The manufacturing overhead of $4.00 per unit includes both variable and fixed manufacturing overhead, based on a production of 100,000 units each year.

(a) Should Pearson Company accept the outside supplier’s offer?

(b) What is the maximum unit price that Pearson Company should be willing to pay the outside supplier?

the hamilton flour company is currently operating its mill six days a week 24 hours 671172

The Hamilton Flour Company is currently operating its mill six days a week, 24 hours a day, on three shifts. At current prices, the company could easily obtain a sufficient volume of sales to take the entire output of a seventh day of operation each week. The mill’s practical capacity is 6,000 hundredweight of flour per day. Note that

• Flour sells for $12.40 a hundredweight (cwt.) and the price of wheat is $4.34 a bushel. About 2.35 bushels of wheat are required per cwt. of flour. Fixed costs now average $4,200 a day, or $0.70 per cwt. The average variable cost of mill operation, almost entirely wages, is $0.34 per cwt.

• With Sunday operation, wages would be doubled for Sunday work, which would bring the variable cost of Sunday operation to $0.66 per cwt. Total fixed costs per week would increase by $420 (or $29,820) if the mill were to operate on Sunday.

(a) Using the information provided, compute the break even volumes for sixday and seven day operation.

(b) What are the marginal contribution rates for six day and seven day operation?

(c) Compute the average total cost per cwt. for six day operation and the net profit margin per cwt. before taxes.

(d) Would it be economical for the mill to operate on Sundays? (Justify your answer numerically.)

in january 2001 kendall manufacturing company purchased a new numerical control mach 671180

Depreciation Adjustment for an Overhauled Asset

In January 2001, Kendall Manufacturing Company purchased a new numerical control machine at a cost of $60,000. The machine had an expected life of 10 years at the time of purchase and a zero expected salvage value at the end of the 10 years.

•  For book depreciation purposes, no major overhauls had been planned over the 10 year period, and the machine was being depreciated toward a zero salvage value, or $6,000 per year, with the straight line method.

•  For tax purposes, the machine was classified as a 7 year MACRS property. In December 2003, however, the machine was thoroughly overhauled and rebuilt at a cost of $15,000. It was estimated that the overhaul would extend the machine’s useful life by 5 years.

(a) Calculate the book depreciation for the year 2006 on a straight line basis.

(b) Calculate the tax depreciation for the year 2006 for this machine.

a mail order computer company sells personal computers and peripherals the company l 671181

Corporate Taxes

A mail order computer company sells personal computers and peripherals. The company leased showroom space and a warehouse for $20,000 a year and installed $290,000 worth of inventory checking and packaging equipment. The allowed depreciation expense for this capital expenditure ($290,000) amounted to $58,000 using the category of 5 year MACRS. The store was completed and operations began on January 1. The company had a gross income of $1,250,000 for the calendar year. Supplies and all operating expenses, other than the lease expense, were itemized as follows:

Merchandise sold in the year$600,000Employee salaries and benefits150,000Other supplies and expenses90,000 $840,000

Compute the taxable income for this company. How much will the company pay in federal income taxes for the year?

a company buys a numerically controlled nc machine for 28 000 year 0 and uses it for 671186

Cash Flow versus Net Income

A company buys a numerically controlled (NC) machine for $28,000 (year 0) and uses it for five years, after which it is scrapped. The allowed depreciation deductionduring the first year is $4,000, as the equipment falls into the category of seven yearMACRS property. (The first year depreciation rate is14.29 %.) The cost of the goodsproduced by this NC machine should include a charge for the depreciation of the machine.Suppose the company estimates the following revenues and expenses, includingdepreciation, for the first operating year:

Gross income = $50,000,

Cost of goods sold = $20,000,

Depreciation on NC machine = $4,000,

Operating expenses = $6,000.

(a) If the company pays taxes at the rate of 40% on its taxable income, what is its net income from the project during the first year?

(b) Assume that (1) all sales are cash sales and (2) all expenses except depreciation were paid during year 1. How much cash would be generated from operations?

for each of four assets in the following table determine the missing amounts for ass 671206

For each of four assets in the following table, determine the missing amounts (for asset type III, the annual usage is 15,000 miles):

Types of AssetIIIIIIIVDepreciating    MethodsSLDDBUPMACRSEnd of year7434Initial cost ($)10,00018,00030,0008,000Salvage value ($)2,0002,00001,000   _ _1,382Book value ($)3,0002,320         _ _Depreciable life8 yr5 yr90,000 mi    Depreciable        _ _Amount ($)_ __ __ _    Accumulated    Depreciable ($)_ _15,680_ __ _

oklahoma oil company incurred acquisition exploration and development costs during 2 671210

Oklahoma Oil Company incurred acquisition, exploration, and development costs during 2006 as follows:

                                                                                SiteItems*Parcel AParcel BTotalAcquisition costs6410Exploration costs13922Development costs201131Recoverable oil   (millions of barrels)9514

The market price of oil during 2006 was $16 per barrel.

(a) Determine the cost basis for depletion on each parcel.

(b) During 2006, 1,200,000 barrels were extracted from parcel A at a production cost of $3,600,000. Determine the depletion charge allowed for parcel A.

(c) In (b), if Oklahoma Oil Company sold 1,000,000 of the 1,200,000 barrels extracted during 2006 at a price of $55 per barrel, the sales revenue would be $55,000,000. If it qualified for the use of percentage depletion (15%), what would be the allowed depletion amount for 2006?

(d) During 2006, 800,000 barrels were extracted from parcel B at a production cost of $3,000,000. Assume that during 2007 it is ascertained that the remaining proven reserves on parcel B total only 4,000,000 barrels, instead of the originally estimated 5,000,000. This revision in proven reserves is considered a change in an accounting estimate that must be corrected during the current and future years. (A correction of previous years’ depletion amounts is not permitted.) If 1,000,000 barrels are extracted during 2007, what is the total depletion charge allowed, according to the unit cost method?

on january 2 2004 hines food processing company purchased a machine that dispenses a 671214

On January 2, 2004, Hines Food Processing Company purchased a machine that dispenses a premeasured amount of tomato juice into a can. The machine cost $75,000, and its useful life was estimated at 12 years, with a salvage value of $4,500. At the time it purchased the machine, Hines incurred the following additional expenses:

Freight in$800Installation cost2,500Testing costs prior to regular operation1,200

Book depreciation was calculated by the straight line method, but for tax purposes, the machine was classified as a 7 year MACRS property. In January 2006, accessories costing $5,000 were added to the machine to reduce its operating costs. These accessories neither prolonged the machine’s life nor provided any additional salvage value.

(a) Calculate the book depreciation expense for 2007.

(b) Calculate the tax depreciation expense for 2007.

consider a five year macrs asset purchased at 60 000 note that a five year macrs pro 671218

Gains or Losses

Consider a five year MACRS asset purchased at $60,000. (Note that a five year MACRS property class is depreciated over six years, due to the half year convention.

TABLE P9.36

Manufacturing expenses (including depreciation)$450,000Operating expenses (excluding interest expenses)120,000A new short term loan from a bank50,000Interest expenses on borrowed funds (old and new)40,000Dividends paid to common stockholders80,000Old equipment sold60,000

The applicable salvage values would be $20,000 in year 3, $10,000 in year 5, and $5,000 in year 6.) Compute the gain or loss amounts when the asset is disposed of

(a) In year 3.

(b) In year 5.

(c) In year 6.

marginal tax rate in project evaluation 671221

Marginal Tax Rate in Project Evaluation

Boston Machine Shop expects to have an annual taxable income of $325,000 from its regular business over the next six years. The company is considering acquiring a new milling machine during year 0. The machine’s price is $200,000, installed. The machine falls into the MACRS five year class, and it will have an estimated salvage value of $30,000 at the end of six years. The machine is expected to generate additional before tax revenue of $80,000 per year.

(a) What is the total amount of economic depreciation for the milling machine if the asset is sold at $30,000 at the end of six years?

(b) Determine the company’s marginal tax rates over the next six years with the machine.

(c) Determine the company’s average tax rates over the next six years with the machine.

what is the profit maximizing strategy given the above rise in costs g how much prof 670306

1. Lizzie’s Lingerie started selling robes for $36, adding a 50 per cent mark up on cost. Costs were estimated at $24 each: the $10 purchase price of each robe, plus $6 in allocated variable overhead costs, plus an allocated fixed overhead charge of $8. Customer response was such that when Lizzie’s raised prices from $36 to $39 per robe, sales fell from 54 to 46 robes perweek.

a. Estimate the optimal (profit maximizing) pricing strategy assuming a linear demand curve.

b. Estimate the optimal pricing strategy assuming a power demand curve.

c. Explain why there is a difference between the above two strategies.

d. Estimate the size of the profit at both prices, assuming a power demand curve.

e. Estimate the optimal price if the cost of buying the robes rises from $10 to $11, assuming a power demand curve. 

2. Crystal Ball Corp. has estimated its demand and cost functions as follows: 

Q = 80 5P

C = 30 + 2.2Q + 0.5Q 2 

Where P is in $, Q is in thousands of units and C is in $,000.

a. Calculate the profit maximizing price and output.

b. Calculate the size of the above profit.

c. Calculate the price elasticity of demand at the above output; is demand elastic or inelastic here? What should it be?

d. Calculate the marginal cost at the above output.

e. If unit costs rise by $2 at all levels of output and the firm raises its price by the same amount, what profit is made?

f. What is the profit maximizing strategy given the above rise in costs?

g. How much profit is the firm forgoing by raising its price $2?

in eighteen trials of the experiment the average surplus gained was 36 per cent of t 670307

Case study 9.1: Experiments testing the Cournot equilibrium

An experiment was conducted in 1990 regarding the behaviour of people in a Cournot type situation. Participants were put into groups of eight players and each player was given ten tokens. Each token could be redeemed for 5 cents or it could be sold on a market. When tokens were sold on the market the price was determined by how many tokens were offered by all eight players, in the following equation: 

P = 23 – 0.25QT 

where QT is the total number of tokens put up for sale by all eight players. Players could choose howmany of their tokens to put up for sale and how many to redeemfor the fixedpriceof 5 cents. At the endof each trial the total surplus was calculated, being measured as the excess value received by all the players over the 5 cents per token redeemable value. For example, if a total of sixty tokens are sold, themarket price is 8 cents and the total surplus is 180 cents.

Questions

1. If the players collude, what will be the market price and the total surplus gained?

2. If the players act as in a Cournot oligopoly, what will be the market price and the total surplus gained?

3. In eighteen trials of the experiment the average surplus gained was 36 per cent of the maximum possible from collusion. Does this evidence support the existence of Cournot–Nash behaviour?

using the backward induction method analyse the game tree and explain the result obs 670308

Case study 9.2: Monetary policy in

Thailand The games people play11

Let’s have fun with game theory, which can shed some light on the outcome of the monetary policy dispute between Prime Minister Thaksin Shinawatra and former Bank of Thailand governor MR Chatu Mongol Sonakul.

Many might be perplexed by Chatu Mongol’s abrupt dismissal after he refused to cave in to the government’s demand to raise interest rates. But by applying game theory to analyse the jostling between the two, one may find a surprising answer and become more aware of the usefulness of the tool. We know that Thaksin and Chatu Mongol took polar positions on the issue and are by nature rather proud and stubborn. So let us begin by constructing what the payoff matrix for the interest rate policy would have been before Chatu Mongol was sacrificed.

Faced with Thaksin’s command to ‘review’ the central bank’s longstanding low interest rate policy, Chatu Mongol could do one of two things – concede to Thaksin, or not give way. Similarly, Thaksin had two options in dealing with the obstinate governor – either fire him or keep him. In order to keep the game simple, we rank the preferences for the possible outcomes from worst to best, and assign the respective payoffs the numbers 1 through to 4. Chatu Mongol had made it perfectly clear that he had no intention of changing the low interest rate policy. Therefore, the worst outcome for Chatu Mongol was to concede but then get fired, so that outcome would have a payoff of 1 for him.

The second worst outcome was to concede and not be fired, but that would leave Chatu Mongol with his integrity bruised and the central bank with its independence impaired.

The third worst outcome was not to concede, and get fired. Though he might lose his job, he could still maintain his integrity and time could prove his stance correct.

Chatu Mongol’s strongest preference was not to concede, but still keep his job. This outcome would have a payoff of 4 for him. This would mean he had beaten Thaksin in their two way gamesmanship. Meanwhile, the worst outcome for Thaksin would be for Chatu Mongol to defy his demand, but to keep the maverick as central bank governor.

The second worst option was for Chatu Mongol to make a concession, but for the PM to have to fire the governor anyway to avoid future trouble. The next worst scenario was for Thaksin to fire Chatu Mongol for his defiance. Thaksin’s highest preference was for Chatu Mongol to fully agree with his demand so that he would not have to get rid of him as governor.

Questions

1. Describe the type of game that is involved in the above situation.

2. Draw a game tree of the situation, with the appropriate payoffs.

3. Using the backward induction method, analyse the game tree and explain the result observed.

contrast the nio and chicago school theories of predatory pricing 4 what is lott rsq 670309

Case study 9.3: Credible commitments21

John Lott’s book Are Predatory Commitments Credible? Who Should the Courts Believe? (1999) is an attempt to test one of the implied assumptions of game theoretic models.

The key questions for NIO (new industrial organization) models developed by game theorists are: ‘Are CEOs hawks or doves, and how can an entrant tell the difference?’ Predatory pricing in the NIO arises when a dominant firm can credibly signal that it will price below cost if anyone enters the market. If the signal is credible, the entrant will not enter. The Chicago School had shown that predatory pricing would be costly to the dominant firm and argued that therefore it would be unlikely to be practiced. Proponents of the NIO agree that predatory pricing is costly, but they argue that to keep an entrant out, no firm need actually practice predatory pricing if the threat to do so is credible.

A ‘hawk’ is a firm that will actually cut prices to drive out an entrant. A ‘dove’ is a firm that will acquiesce to entry because it cannot bear the shortterm losses entailed by engagement in predatory pricing. Of course, doves threaten predatory pricing just as hawks do. How can the entrant discover who is a hawk and who a dove?

Lott’s answer is that a hawk CEO must have high job security. As the entrant enters, the hawk CEO goes to war against the entrant by driving down prices and thereby greatly reducing profits. If the CEO must answer to stockholders for declines in the price of stock, or if his own pay is tied to the stock price through options or other means, then he will be unwilling to prosecute the war. The signal required to make a predatory commitment credible is a system of corporate governance that allows the CEO more control over the corporation than stockholders would otherwise give him. In short, to signal credibly, the CEO must be a dictator rather than an elected representative. Dictatorship, of course, has its costs, in nations or in firms, so not every firm will want to be a hawk. Lott proposes that this difference in corporate governance presents an opportunity to test the NIO theory of credible commitments. He examines twenty eight firms accused of predatory pricing between 1963 and 1982. Is the corporate governance of these firms more hawklike than that of other firms? It is not. Lott finds few differences in CEO turnover, incorporation in a state with antitakeover provisions, stock ownership, or CEO pay sensitivity between the firms accused of predatory pricing and a control group. One of the key assumptions of the NIO is therefore wrong. The question remains, ‘Why would firms that had no better commitment strategy than a control group have been accused of predatory pricing?’ Although this question lies beyond the scope of Lott’s book, one can only conjecture that those firms were accused of predatory pricing not because they actually practiced it but because their competitors wanted to stop competition (see Fred S. McChesney and William F. Shughart II, The Causes and Consequences of Antitrust: The Public Choice Perspective, Chicago: University of Chicago Press, 1995).

If firms accused of predatory pricing do not seem to differ systematically from the control group, is any firm capable of following a predatory pricing strategy? In effect, could any organization commit to not maximizing profits, if only for a limited period of time? Lott’s answer is that one group of firms can make such a commitment: publicly owned firms. The basic idea comes from Niskanen’s model (William Niskanen, Bureaucracy and Representative Government, Chicago: Aldine Atherton, 1971): publicly owned firms maximize size rather than profit. Lott gives several examples, but none hits closer to home than the public university, which must maintain enrollment in order to maintain the size of the faculty and therefore sets prices considerably below costs.

Lott’s second type of evidence that publicly owned firms practice price predation is the fact that dumping cases – the international version of predatory pricing complaints – have been filed under the General Agreement on Tariffs and Trade more frequently against firms from communist countries than against firms from noncommunist countries. Lott shows, therefore, that the NIO theory of predatory pricing makes sound predictions (hawks practice predatory pricing more than doves), but it has limited application to the private enterprise system, to which its advocates intended it to apply.

Lott’s third argument supplements the theory of predatory pricing. He extends Jack Hirshleifer’s observation that inventors of public goods can internalize at least some of the value of theirinvention by taking long or short positions in assets whose price will change after the discovery is made public (see Jack Hirshleifer, ‘The private and social value of information and the reward to inventive activity’, American Economic Review, 61 (1971): 561–574). Lott extends this idea by arguing that an entrant facing an incumbent with a reputation for toughness should take a short position in the incumbent’s stock, enter, and reap trading profits. In effect, the incumbent firm with a reputation for toughness finances the entry of its own competitors. The entrant can also make profits by exiting. If the entrant enters and finds that it cannot withstand the attack of the hawk, it can take a long position in the incumbent’s stock, exit, and collect the trading profits. Either way, trading profits increase the incentive to enter because whether or not entry ultimately succeeds, trading profits allow the entrant to make a profit. As Lott puts it, ‘the more successfully a predator deters entry, the greater the return that trading profits create toward producing new entry. Creating a reputation to predate can thus be selfdefeating’ (p. 115).

Lott provides several anecdotes about the use of trading profits, but he admits he can find few recent examples. The problem, of course, is that a firm holding a short position in a competitor’s stock would not want to advertise that fact to the market. Therefore, we would expect such evidence to be thin. The trading profits idea does suggest that the threat to practice predatory pricing would be more successful when the incumbent firm was closely held, and therefore entrants could not easily buy shares of it. This relationship might make predatory pricing more likely in developing countries that are dominated by family run firms and that lack welldeveloped equity markets.

One of the basic insights of economics is that wellestablished markets threaten rents. Lott’s simple application of this wisdom ought to change the way economists think about antitrust cases and the way they are litigated both as private and as public cases. The notion that trading profits can mitigate or eliminate the private damage from predatory pricing should certainly give antitrust experts cause to worry about the efficiency of treble damages. I await the day when the defendant in an antitrust case will respond, ‘If my actions were predatory, why didn’t the plaintiff just buy my stock short and use the profits to stay in the market.’

Questions

1. What is meant by a hawk CEO?

2. Why should hawk CEOs need high job security?

3. Contrast the NIO and Chicago School theories of predatory pricing.

4. What is Lott’s conclusion relating to empirical evidence for the NIO?

5. Explain Lott’s theory of trading profits and how it relates to predatory pricing; how does the theory support his conclusion that ‘Creating a reputation to predate can thus be self defeating’?

examine the problem on property rights and fishing how is the situation affected if 670312

1. The cement making industry is a duopoly, with two firms, Hardfast and Quikrok, operating under conditions of Cournot competition. The demand curve for the industry is P¼200Q, where Q is total industry output in thousands of tons per day. Both firms have a marginal cost of £50 per ton and no fixed costs. Calculate the equilibrium price, outputs and profits of each firm.

2. A market consists of two firms, Hex and Oct, which produce a differentiated product. The firms’ demand functions are given by: 

QH = 100 2PH + PO

QO = 80 – 2.5PO + PH 

Hex has a marginal cost of £20, while Oct has a marginal cost of £15. Calculate the Bertrand equilibrium prices in this market.

3. Examine the problem on property rights and fishing; how is the situation affected if the cost of catching fish increases from £0.10 to £0.20 per fish?

assuming a linear market demand function and linear cost functions with no fixed cos 670313

1. Two banks are operating in a duopolistic market, and each is considering whether to cut their interest rates or leave them the same. They have the following payoff matrix: 

Bank

Maintain rate

Cut rate

Bank

Maintain rate Cut rate

(50, 50) (70, 20)

(20, 70) (30, 30)

a. Does either bank have a dominant strategy?

b. Does the above game represent a Prisoner’s Dilemma? Explain.

c. Is there any way in which the two banks can achieve co operation?

2. Assuming a linear market demand function and linear cost functions with no fixed costs, show the differences in output, price and profits between the Cournot and Stackelberg oligopoly models.

explain the implications of the saturation of the handset market for nokia rsquo s c 670314

Case study 10.1: Mobile phones – Nokia

Emergency Calls4

As bad news flows thick and fast, it has become clear that a vicious shake out is under way in the global telecoms industry, led by the mobile operations on which dozens of companies have staked their futures. Many will want to forget April 24th, a black Tuesday that saw a string of announcements typifying the problems faced by operators and manufacturers alike:

1 In Japan, NTT DoCoMo, the wireless arm of NTT, postponed the commercial launch of its thirdgeneration (3G) mobile network from next month until October. It said the system needed further tests before it would be robust enough. The blow is to DoCoMo’s prestige rather than its profits, as the firm is still reckoned to be a leader in 3G technology. But the news worried heavily indebted rivals that have looked to DoCoMo to show that 3G can be made to work.

2 Germany’s Deutsche Telekom announced a firstquarter net loss of e400m ($369m), and said that it had shifted its focus away from acquiring new mobile customers towards making more money fromexisting ones. T Mobile, its wireless arm, made pre tax profits of e590m, almost 70% more than in the sameperiod last year, and has justwon approval to merge with VoiceStream, an American mobile firm. But Telekom remains in trouble. It is shackled with e57 billion of debt and would like to float TMobile, but market conditions are too awful.

3 Motorola, an American mobile handset and equipment manufacturer, said it was closing its biggest factory in Britain, with the loss of more than 3,000 jobs. It blamed a sudden collapse in demand for mobile telephones and may have to hand back almost £17m ($25m) in state aid. Other equipment makers are having a torrid time, too. JDS Uniphase, a market leader in fibre optics for high speed telephony, announced a $1.3 billion loss for its third quarter and said it would cut its workforce by a fifth.

4 Lastly, Ericsson of Sweden, the world’s thirdbiggest maker of mobile handsets, announced a joint mobile phone venture with Sony. The new business, which will be launched in October, aims to develop new mobile consumer brands and to become a long term global competitor.

In normal times, Ericsson’s deal might have looked like the only piece of good news on an otherwise miserable day. But it came almost immediately after the company had announced a big retrenchment and 12,000 job losses. In the first quarter of this year, it made a SKr4.9 billion ($502m) pre tax loss, and it admitted that its mobile handset business is looking shaky. Its shares are worth around 70% less than they were a year ago, and have fallen by more than onethird since the beginning of March. So it is hardly surprising that Kurt Hellstrom, Ericsson’s chief executive, looked subdued at the press conference to announce the tie up with Sony. He is paying a heavy price for having missed the strategic shift as mobile phones became trendy consumer goods rather than purely functional items. Ericsson underinvested in design and has been eclipsed by niftier rivals, notably Nokia of Finland. Thus, the deal with Sony is better seen as a measure of how quickly Ericsson has fallen from grace. It has conceded 50% of the venture to the Japanese, even though it is far bigger, selling 43m phones last year against Sony’s 7.5m. In an industry that moves quickly, the new venture will produce its first products only some time next year. Mr Hellstrom might no longer be the boss by then.

All over the industry there are clear signs that the telecoms boom has ended. Dramatic retrenchment by top notch competitors may look panicky, but the truth is that most have little alternative. As the mobile sector has matured, its growth has inevitably begun to slow, leading investors to question the prospects of traditional operators that have placed huge bets on mobile technologies. Not only have capital markets become choosier about telecoms related financings, but weak equity markets have made it almost impossible to float off mobile ventures and pay back debt. The knock on effect for mobile handset manufacturers has become all too evident – not least because they have had to provide billions of dollars in ‘vendor financing’ so that mobile network operators can continue to buy their products.

A rare bright spot has been Nokia, the world’s biggest mobile maker. Total sales in 2000 increased by 54% over the previous year, to around e30 billion. Net profit also increased by 53%. Even in the turbulent first quarter of 2001 total sales increased 22%, while operating profit rose 8%. These first quarter results were better than expected and its share of the global handset market is edging up towards 40% as Ericsson’s falls. So are its shares doing well? Relatively, yes: at the end of April they were a mere 40% below their level of a year before.

Nokia succumbs5

Then on June 12th there was proof, if proof were still needed, that no technology company is bulletproof. Nokia, the world’s leading manufacturer of mobile telephones, gave warning that its second quarter profits would be lower than expected, and that annual sales growth, previously forecast at 20%, would in fact be less than 10%. Nokia’s shares fell by 23%, though they later recovered slightly. Other telecoms firms’ share prices suffered too, with the exception of BT, whose shares rose after news of a 3G network sharing deal with Deutsche Telekom.

Nokia’s announcement was portrayed by Jorma Ollila, the firm’s boss, as an indication that the slowdown in the American economy is having knockon effects in Europe. But this explanation is a red herring, says Mark Davies Jones, an analyst at Schroder Salomon Smith Barney. Although Nokia has been slightly affected by falling consumer demand in America, the real cause of its problems is that the market for handsets, which account for nearly threequarters of its sales, is saturated.

The problem is that people are neither buying new phones, nor upgrading their old ones, as often as they used to. In part this is because network operators, most of which are struggling with huge debts, feel less inclined to subsidise handsets. But it is also because there is no compelling reason to upgrade your phone once it is small and sexy enough. The days of double digit growth in handset sales are over: the number of handsets sold worldwide – 400m last year, of which 32% were Nokia’s – is not expected to rise this year. Any sales growth at Nokia this year will come from increasing its market share. In the UK there are now over 40 million mobile phone subscribers, and the market penetration rate is around 70%, one of the highest in the world.

What now? The industry has a plan, which is to introduce new mobile data services. Operators will benefit by being able to charge for these services (since revenues from voice traffic have stopped growing) and handset manufacturers will be able to sell everybody new phones. The problem is that the first incarnation of mobile data, Wireless Application Protocol (WAP) services, was a flop. So the industry’s hopes are now pinned on a new technology, General Packet Radio System (GPRS), which is faster than WAP and offers ‘always on’ connections.

Nokia has extensive 3G contracts in the UK, including three year agreements with Orange to deliver the radio access network from their 3G network, and a three year agreement with Hutchison to deliver a complete range of 3G mobile network infrastructure worth around e500 million.

On June 13th, the GSM Association, an industry standards setting body, announced a scheme called the ‘M Services Initiative’, which defines a standard way to offer graphics and other multimedia content on GPRS phones. The idea is that these new features will encourage users to upgrade their handsets, and thus plug the gap before the arrival of 3G phones in a couple of years’ time. The big operators and manufacturers, including Nokia, are backing the scheme, and the first handsets sporting graphics should be in the shops by Christmas. One way or another, this week could prove to be a turning point for the industry.

Questions

1. How would you describe Nokia’s competitive advantage?

2. Explain the implications of the saturation of the handset market for Nokia’s competitive advantage, making strategy recommendations.

 

evaluate palm rsquo s targeting strategy 670315

Handheld Computers – Palm

One Palm flapping6

Carl Yankowski is having a hard time turning Palm’s promise into profits

By rights, it should not have been this hard. Carl Yankowski, chief executive of Palm, the leading maker of handheld computers, is a former president of Sony’s American operations. He is a big, dressedin black ‘gadget guy’ with just the right combination of consumer marketing experience and technology savvy to straddle the gap between computing and consumer electronics. From its start in 1996, Palm single handedly generated the personal digital assistant (PDA) craze, succeeding where many had failed before, and it is now leading a drive into the new wireless world. Computing meets telecoms meets consumer electronics – the palmtop becomes the new desktop. And here, or so it might seem, in the right place at the right time, is Mr Yankowski.

Yet he is struggling. For all Palm’s success in defining and leading a booming new industry – its operating systemrunsmore than three quarters of the world’s PDAs, giving it an almost Microsoft like monopoly  the company itself is in a very un Microsoft like financial mess. Recently, it issued a warning that its losses in the current quarter would be twice what had been expected – as much as $190m. Its sinking fortunes also scuttled a planned mergerwith Extended Systems, a deal that was meant to take it to new heights in the corporate market. Palm’s shares have fallen by 90% over the past six months.

The company’s difficulties do not all lie at the feet of Mr Yankowski – he took over only a year and a half ago, as Palm was freeing itself from 3Com, its former corporate parent and the source of its risk averse corporate culture. But, like it or not, they are now his to solve. 

The biggest problem is that Palm is having a hard time finding the right strategy. At the moment, it follows a combination of Microsoft’s with Apple’s that ends up weaker than either. Like Apple, it makes its own hardware and software: a line of PDAs and the famed Palm operating system (OS) that is the secret of its success. Like Microsoft, it also licenses its OS to other companies, ranging from Handspring, which was started by Palm’s original founders, to Sony and several mobile phone makers.

The downside to this is that Palm’s licensees have proved all too good at making hardware. Today, a mere two years after it released its first PDA, Handspring is beating Palm in sales. And both Sony and Handspring have pushed their hardware further than Palm, introducing innovations such as expansion slots and add on devices from phones to music players. The result is that the PDA business is quickly taking on the savage character of the PC industry, with commodity products, falling margins and cut throat competition.

Mr Yankowski inherited most of this, and it had too muchmomentumfor him to change it quickly or easily. If Palm stops licensing its operating system, it risks losing out to OS competitors such as Microsoft and Psion. If it stops making hardware entirely, it would take the best known brand of PDA out of circulation.

The enthusiasm over Mr Yankowski’s arrival in late 1999 was based largely on his background, which suggested that there might be a third way for the company. Aside from his Sony experience, which placed him at the heart of the best consumerelectronics firm just as it was embracing digital technology, his career has been a virtual tour of great marketing firms: Reebok, General Electric, Pepsi, Memorex and Procter & Gamble. Starting it all was an engineering degree from MIT.

The hope was thatMr Yankowski could combine Sony’s design and marketing skills with Palm’s technology. His type of consumer marketing experience can make the difference between a niche gadget and a Walkman like hit. Which is why it is so puzzling that Palmhas changed so little since his arrival.

CORPORATE PRIORITY

Many expected him to push the company faster into the consumer market, with brightly coloured PDAs and extra consumer features such as MP3 and video. Instead, he has made his main priority the staid corporate market. At present, most Palms make it into the office thanks to somebody’s personal expense account. Mr Yankowski’s aim is to encourage IT managers to purchase them directly for employees, much as they buy PCs.

This is not a bad strategy – the corporate PDAmarket is about the same size as the consumer market, and both have lots of potential – but itmay be awaste of Mr Yankowski’s special talents. While he tries to sell his firm’s strait laced productivity tools, in black and grey, to corporate purchasing managers, his old company, Sony, is generating enviable buzz with a cool purple PDA that plays videos and has a headphone jack. Worse, the corporate market is the one in which Palm faces its toughest competition, in the formof Research in Motion’s Blackberry interactive pagers, which have generated Sony like excitement among the suits.

Palm’s recent results have at last provoked Mr Yankowski into thinking more broadly. He is now in management retreats with his staff and is expected to announce a new strategy soon. But his options get more limited by the day. Palm’s finances are too rocky to get into a consumer marketing race with Sony. Nor does it have the products to justify that. The first Palms designed on Mr Yankowski’s watch are now out. They do little more than add an expansion slot like the one Handspring has had for two years.

Meanwhile, the collapse of the plannedmergerwith Extended Systems, which has had success selling to IT managers, limits the push into the corporate sector. And abandoning hardware entirely would reduce Palmto a software and services firm– hardly the place for a consumer marketing guru. Mr Yankowski does not have much more time to find the right answer.

Questions

1. Describe Palm’s positioning in themarket, in terms of resources, capabilities, cost and benefit advantage.

2. What criteria can Palm use to segment its market?

3. Evaluate Palm’s targeting strategy.

explain the role of the internet in the pricing strategies above 670316

Case study 10.3: Airlines

Britain takes to the air8

Low cost carriers are transforming not just the travel business in Britain, but also the way people live The air of gloom surrounding much of European business made Ryanair’s results, announced on June 25th, particularly impressive. The low cost airline reported a 37% year on year increase in pre tax profits, and its chief executive, Michael O’Leary, said he expects business to grow by 25% over the next year. Thanks to Ryanair and its sort, Britons are beginning to hop on and off planes the way Americans do.

Air travel in and around Britain has grown by nearly 40% in the past five years, but the really spectacular growth has come from the low fare airlines, which have carried around 20m passengers in the past year. By spotting and satisfying the untapped demand for travel from and between the regions, they have fuelled the growth of Britain’s smaller airports and undermined Heathrow’s dominance.

EasyJet, the first of the low cost carriers, was set up in 1995 at Luton. Eastwards around the M25 at Stansted are Ryanair, Go, the low cost offshoot of British Airways (BA) sold to a management buy out earlier this year, and Buzz, the British arm of KLM, which uses the airline partly to feed its international hub at Amsterdam. While Heathrow has seen the number of passengers rise by about 19% over that period, traffic at Luton and Stansted has more than trebled. Traffic at Liverpool’s airport nearly quadrupled.

Demand for air travel is highly elastic. Bring down the price and sales rise sharply. The low fare carriers are often cheaper not just than the mainstream operators but also than the railways. While low fare airlines keep their fixed costs to a minimum, the railways are burdened by the need to maintain and improve their crumbling network. Last year was a disaster for them. A crash blamed on a cracked rail led to mass disruption as managers tried to locate and mend other dodgy rails. Delays drove passengers onto the airlines.

Low cost airlines fill their planes differently from mainstream carriers. BA, British Midland, Air France and Lufthansa aim to make their money out of business travellers who pay over the odds to enjoy meals and loads of drinks in the air and on the ground in exclusive lounges. The economy seats are sold off, discounted as need be, some in advance and some at the last minute. Cheap seats are made available through downmarket travel agencies which publicise their deals through newspapers’ classified columns.

The low cost carriers see their aircraft as a series of buckets. The first set of buckets are the lowest priced seats, with the eye catching prices. Once these are all sold, demand flows into the next, slightly more expensive, bucket of seats. As the flight’s departure approaches, seats get progressively more expensive. On a typical low cost flight there could be up to ten different price buckets, with one way fares ranging from £30 ($42) to £210. But even the most expensive tickets tend to be cheaper than for the mainstream airlines.

Early assumptions that the low cost carriers would struggle to make headway in the business market, because businessmen do not care how much their companies pay for their tickets, have turned out to be wrong. Stelios Haji Ioannou, EasyJet’s founder, says that one of the things he first noticed when the airline launched was how many business passengers he seemed to be carrying. Not that business travellers are set apart, since everybody piles into the same nofrills cabin, with free for all seating and pay for all drinks and sandwiches; but business travellers tend to book late (and so more expensively) and travel mid week. It turns out that businessmen are more price sensitive than had been assumed. Some, presumably, are running their own businesses, so have an interest in keeping costs down; others are responding to cost cutting memos from above.

WIRED FOR TAKE OFF

The Internet has also helped the low cost airlines. Airline tickets rival pornography as the hottest selling commodity on the Internet, with sales estimated at more than $5 billion worldwide. Mainstream airlines sell around 5%of tickets over theweb. EasyJet decided to focus on Internet sales, so it offers discounts for online booking and has built a site that is easy to use. These days, some 90% of EasyJet bookings are made online. Ray Webster, EasyJet’s chief executive, reckons that older or techno illiterate people get a younger or more wired friend to do it for them.

Some 65% of Ryanair’s bookings are made online. Even this figure is twice as high as the highest e booking airline in America, Southwest, the original low fare, no frills carrier which was the model for the British low cost operators.

The low cost airlines have not just brought down the price of flying. They have changed the way British people travel, and also where they live, holiday and work. Air travel no longer involves the crowded hell of scheduled flights at Heathrow or charter flight delays at Gatwick. Cheap fares and European second homes have almost replaced house prices and school fees as a topic for dinner party chat.

At four o’clock on a summer afternoon at Luton airport, a queue is forming for the 5.40 to Edinburgh. A holidaying couple are returning to Edinburgh from Spain. The EasyJet flight was so cheap that it was worth their while taking an Airtours package from Luton, rather than flying from Scotland. Behind them is a management consultant who uses EasyJet from Luton because he lives just three exits up the M1 and it is quicker than hacking round the M25 to Heathrow. A technology transfer specialist at the Medical Research Council in Edinburgh says EasyJet is a way of coming down to London once a month for a fraction of the fare on British Airways or BritishMidland. ‘It seems silly paying extra when you are dealing with public funds,’ she explains.

Businessmen from small and big companies alike are hopping onto cheap flights. Robert Jones, shipping and travel manager for Smit Land and Marine, a Liverpool based Anglo Dutch pipeline company, reckons he is saving £50,000 a year by using budget airlines. Half of his company’s regular 30 trips a month from Liverpool to Amsterdam are by EasyJet, at £120 each, instead of the £350 he would spend on a scheduled airline. Travelling to Spain, he saves around £500 a trip. And, he says, the low cost airlines make it easier to change passenger names if one employee has to substitute for another at a meeting. According to airport managers at Liverpool, since Ryanair and EasyJet have built up their flights from the city, the number of executive type cars parked at the airport has shot up.

Leisure travel is changing too. Until recently, most people flew once or twice a year, to Florida or the south of Spain. These days, people increasingly hop on planes several times a year. It’s no big deal any more. A salesman in a London electronics shop says his parents have a holiday house in the south of France, which he had stopped visiting after they stopped paying for his holidays. Recently, however, he has discovered that if he books ahead on the Internet, he can fly EasyJet to Nice and back for under £50, making a monthly visit an affordable treat.

Perhaps the most astounding change is the number of long distance commuters using the new airlines. Hang around long enough at Luton and you will meet a businessman or woman, usually middleaged owners of companies in the south east, who spend half their week as lotus eaters in Provence, nipping back for the other half to oversee their business being handled day to day by their staff. Most incredible of all, there is a resident of the Luton area who commutes to bustling Glasgow every morning, only to return to lovely Luton in the evening. That could be called some sort of progress.

Questions

1. What conditions make price discrimination possible in the airline industry, and what types of price discrimination are possible? Explain how different demand elasticities are relevant.

2. Explain the differences in pricing strategy between the mainstream airlines and the low cost airlines, in terms of the different types of price discrimination used.

3. Explain the role of the Internet in the pricing strategies above.

gmg a cinema complex is considering charging a different price for the afternoon sho 670320

1. GMG, a cinema complex, is considering charging a different price for the afternoon showings of its films compared with the evening ticket price for the same films. It has estimated its afternoon and evening demand functions to be: 

                         PA = 8.5 – 0.25QA

                         PE = 12.5 – 0.4QE 

where PA and PE are ticket prices (in £) and QA and QE are number of customers per week (in hundreds). GMG has estimated that its fixed costs are £2,000 per week, and that its variable costs are 50 pence per customer.

a. Calculate the price that GMG should charge if it does not use price discrimination, assuming its objective is to maximize profit.

b. Calculate the prices that GMG should charge if it does use price discrimination.

c. Calculate the price elasticities of demand in the case of price discrimination.

d. How much difference does price discrimination make to profit? 

2. TT Products produces two items, bibs and bobs, in a process that makes them joint products. For every bib produced two bobs are produced. The demand functions for the two products are: 

                          Bibs: P = 40 4Q

                          Bobs: P = 60 3Q

where P is price in £ and Q is units of each product. The total cost function is: 

                          C = 80 + 20Q + 4Q 2 

where Q represents a product bundle consisting of one bib and two bobs.

a. Calculate the prices and outputs of bibs and bobs that maximizes profit.

b. Calculate the size of the above profit.

brief 1 justin anson distillery inc balance sheet as of june 30 2011 and 2012 670575

Purpose

To assess your ability to:

· identify essential manufacturing costs for a finished good

· discuss why it is important to distinguish inventoriy costs from ‘Other Costs’ that are included in the Cost of Goods sold

· discuss what costs should be included in inventory when preparing financial statements

Justin Anson Distillery Exhibit 1 Justin Anson Distillery, Inc., Balance Sheet as of June 30, 2011 and 2012 ($ thousands) Current Assets Cash VI Accounts receivable trade Inventories: z,„. Bulk whiskey in barrels at average production cost (no excise tax included) Bottled and cased whiskey, 175,000 gallons in each year at an average cost of $24.75 per gallon (including excise tax) Inventory in process Raw materials and supplies Prepaid expenses Total current assets Fixed Assets Cost Accumulated T Depreciation 2011 2012 2011 2012 Land $ 66 $ 66 Buildinga 4,202 4,642 $1,760 $1,877 Factory equipment 158 158 57 83 Warehouse equipment 77 141 53 75 Total assets Current Liabilities Short term notes payable to banks Current maturities of long term debt Accounts payable Federal excise taxes payable Total current liabilities Noncurrent Liabilities Notes payable (9 1/2%) secured by deed of trust on warehouse property Stockholders’ Equity Common stock held principally by mem bers of the Anson family Earnings retained in the business Total liabilities and capital 2011 2012 $ 2,892 $ 792 3,139 4,028 9,914 11,067 4,331 4,331 222 222 880 519 970 854 22,348 21,813 Net 2011 2012 $ 66 $ 66 2,442 2,765 101 75 24 66 2,633 ? ,1 2,972 $24,981 $24,785 $ 2,420 $ 3,300 506 1,062 2,330 1,175 902 6,158 5,537 7,700 9,020 3,960 3,960 7,163 6,268 $24,981 $24,785 aln June 2012, payment was made for work that had been performed during the year in adding to and improving the warehousing space in the building owned by Justin Anson Distillery. Printed by Dhamn endra Singh (dharrnendra.singh@franklin.edu) on 8/26/2014 from 173.245.67.101 authorized to use until 7/28/2016. Use beyond the authorized user or valid subscription date represents a copyright violation.

the balance in an account as shown by the checkbook 670598

32. The balance in an account as shown by the checkbook on August 1 is

$3,980. The balance as reported by the bank statement for the same

Date is $4,320. The difference is most likely due to

  1. An error in the checkbook balance
  2. Checks outstanding which have not cleared the bank
  3. Interest earned but not yet recorded in the checkbook
  4. Deposits made before August 1 but not yet credited by the bank

33. The cashbook balance is $360. The bank statement issued on the

Same day shows a bank charge of $8. Unlearned checks amount to

$175 and the most recent deposit of $50 entered on the cash book

Has not been credited by the bank

What is the balance on the bank statement?

  1. $427
  2. $477
  3. $467
  4. $277

34. Using the information in the questions above, what cash balance

Should be carried forward into the next period?

  1. $344
  2. $352
  3. $360
  4. $402

Deception Debit Credit
(1)Equipment 2,600
Accounts Payable 2,000
Cash 600
(R. Bean Inc., Invoice #103)
(2) Cash 2,800
License Fees 2,800
(Fees for month)
(3) Accounts Payable 2,000
Rent Expense 2,000
(Rent for month)
(4) Salaries Expense 600
Cash
(Receipts for month)
(5) Accounts Payable 1,600
Cash 1,000
Equipment 1,000
(R. Bean Inc., payment on acct.)
  1. Equipment worth $2,600 was bought on account from R. Bean Inc. with a down payment of $600
  2. Received $2,800 in license fees for the month
  3. Paid accrued rent, $200
  4. Paid salaries, $600
  5. Paid $1,600 on account to R. Bean Inc.

43. Using the GENERAL JOURNAL, transaction 1, the journal entry for

“Accounts Payable”

  1. Is in the wrong column
  2. Is in the wrong amount
  3. Is for the wrong account
  4. A and C above
  5. None of these

44. Using the GENERAL JOURNAL, transaction 2, the journal entry for

The account “Cash”

  1. Is in the wrong column
  2. Is in the wrong amount
  3. Is for the wrong account
  4. A and C above
  5. None of these

45. Using the GENERAL JOURANL, transaction 2, the journal entry for

The account “License fees”

  1. Is in the wrong column
  2. Is in the wrong amount
  3. Is for the wrong account
  4. A and C above
  5. None of these

46. Using the GENERAL JORUANL, transaction 5, the journal entry for

The account “Cash”

  1. Is in the wrong column
  2. Is in the wrong amount
  3. Is for the wrong account
  4. B and C above
  5. None of these

47. Using the GENERAL JOURNAL, a trail balance prepared

Entries on this sheet

  1. Would contain errors for individual accounts
  2. Would be in balance
  3. Would not be in balance
  4. A and B above
  5. A and C above

48. $1,200 is borrowed at 6%. One and ½ years later, the note is paid in full with interest. What is the amount of the payment?

  1. $1,289.55
  2. $1,298.35
  3. $1,301.90
  4. $1,308.00

49. A school collected $420 in fees during the month of April, in May

Is collected a sum 17.5% greater than the collections for April

And in June, it collected a sum 20% greater than the average of the

previous 2 months’ collections. What was the total collected

during the quarter?

  1. $1,226.40
  2. $1,417.50
  3. $1,461.60
  4. $2,009.70

50. A district is considering a medical contract that requires at least

75% of the employees to enroll before the contract goes into

Effect. Sixty percent of the employees have agreed to sign and

More are needed. How many employees work in this district?

  1. 150
  2. 225
  3. 300
  4. 360

Attachments:

tco b heckaman corporation produces and sells a single product data concerning that 670606

(TCO B) Heckaman Corporation produces and sells a single product. Data concerning that product appear below.

Selling price per unit $230.00
Variable expense per unit $112.70
Fixed expense per month $239,292

Required:

Determine the monthly break even in unit sales. Show your work! (Points : 25)

3. (TCO G) (Ignore income taxes in this problem.) Axillar Beauty Products Corporation is considering the production of a new conditioning shampoo that will require the purchase of new mixing machinery. The machinery will cost $375,000, is expected to have a useful life of 10 years, and is expected to have a salvage value of $50,000 at the end of 10 years. The machinery will also need a $35,000 overhaul at the end of Year 6. A $40,000 increase in working capital will be needed for this investment project. The working capital will be released at the end of the 10 years. The new shampoo is expected to generate net cash inflows of $85,000 per year for each of the 10 years. Axillar’s discount rate is 16%.

Required:

a. What is the net present value of this investment opportunity?

b. Based on your answer to (a) above, should Axillar go ahead with the new conditioning shampoo?

(Points : 35)

question1 consider the following statement it is most sensible to start with the sal 670621

Question1

Consider the following statement: It is most sensible to start with the sales budget and develop the other budget from there. After what you have learned this week with regards to budget processes and procedures, analyse the validity of this statement. Do you agree with the statement? Justify your answer based on the week’s readings. How should sales revenues be considered when determining other costs?

Question 2

A consulting firm produces a service that requires the use of labor and materials. Each unit of service requires a standard labor time of 30 minutes (0.5 hours). The average pay rate for a labor hour is £20. The consulting firm considers all materials that are required for the service as variable overheads (OH), the cost of which is directly associated with the labor hours worked. It has been estimated that variable OH rate is £10 per service hour.

The budgeted and actual costs, revenue and units for the month November are given in the table below:

Original Budget

Actual

Units of Service

1,500

1,600

Sales Revenue

£120,000

£124,400

Labor hours

750

860

Labor cost

£15,000

£20,210

Variable OH costs

£7,500

£8,170

Fixed Cost

£68,000

£68,000

Total Cost

£90,500

£96,380

Operating Profit

£29,500

£28,020

1. Calculate the flexed budget and the key variances between budgeted and actual results.

2. Reconcile the original budget and present the relationship between the budgeted and the actual profit for the month November

3. Discuss the calculated variances, and provide suggestions for better cost management (target length 300 words).

Attachments:

for each independent case fill in the missing figures 670722

For each independent case, fill in the missing figures.

CASE A CASE B CASE C CASE D

………………………………………………………………………………………………………………………………………………………………………………………………………………………….

Units produced 1000 ? 240 1,500

Standard hours per unit 3.5 0.9 ? ?

Standard hours ? 900 600 ?

Standard rate per hour $7.25 ? $ 10.50 $ 7.00

Actual hours worked 3,400 975 ? 4,900

Actual labor cost ? ? $6,180 $ 31, 850

Labor rate variance $ 850 F $975 F $ 300 U ?

Labor efficiency variance ? $ 765 U ? $ 2,800 U

Document Preview:

For each independent case, fill in the missing figures. CASE A CASE B CASE C CASE D …………………………………………………………………………………………………………………………………………………………………………………………………………………………. Units produced 1000 ? 240 1,500 Standard hours per unit 3.5 0.9 ? ? Standard hours ? 900 600 ? Standard rate per hour $7.25 ? $ 10.50 $ 7.00 Actual hours worked 3,400 975 ? 4,900 Actual labor cost ? ? $6,180 $ 31, 850 Labor rate variance $ 850 F $975 F $ 300 U ? Labor efficiency variance ? $ 765 U ? $ 2,800 U

Attachments:

can i get the answer 670763

Task This assessment task consists of two (2) questions: a newsletter, and a question on financial statement presentation.  A total of 50 marks are allocated to the questions below, which will then be converted to a mark out of 15%. Rationale This assessment task is designed to assess your understanding of topics 1 and 2 of the subject. Marking criteria A detailed marking rubric has been provided in the ‘Requirements’ section below for each question.

Document Preview:

Task This assessment task consists of two (2) questions: a newsletter, and a question on financial statement presentation.  A total of 50 marks are allocated to the questions below, which will then be converted to a mark out of 15%. Rationale This assessment task is designed to assess your understanding of topics 1 and 2 of the subject. Marking criteria A detailed marking rubric has been provided in the ‘Requirements’ section below for each question.   Presentation Physical presentation of assignments It is essential that presentation of assignments adheres to accepted standards in relation to neatness and layout, as you are practising to present material in a work situation. Correct formatting and referencing procedures of material should be strictly adhered to for essays. You should submit a proper reference list (using APA referencing style) for all essay type assignments. A reference list contains only those works cited or quoted from in your essay. A bibliography is acceptable for practical type assignments.   For practical questions: all journal entries must include narrations unless otherwise specified; any ledger accounts should preferably be shown in ‘T’ account format and dates and descriptions be included; journal entries and ledger accounts must reflect the strict order of sequence of events; financial statements (including extracts) should include proper headings and accord with presentation standards. Penalties will be incurred if material is not correctly referenced and if presentation is not of an acceptable standard.   Please also note the following: Journal entries, ledger accounts, worksheets and financial statements should always balance. If you have to submit a piece of work that does not balance because you cannot detect your error please include some comment about the source of your problem so the marker can provide appropriate feedback. Include workings where appropriate. Partial marks can be allocated for workings where the final answer…

Attachments:

on may 31 the inventory balances of tog designs a manufacturer of high quality child 670791

On May 31, the inventory balances of Tog Designs, a manufacturer of high quality children’s clothing, were as follows: Materials Inventory, $21,360; Work in Process Inventory, $15, 112; and Finished Goods Inventory, $17, 120. Job order cost cards for jobs in process as of June 30 had the following totals: Job no. Direct Materials Direct labor Overhead 24 A $1593 $1290 $1677 24 B $1492 $1380 $1794 24 c $1987 $1760 $2288 24 d $1608 $1540 $2002 The predetermined overhead rate is 130 percent of direct labor cost. Materials purchased and received in June were as follows. June4 $33120 June 6 – $28, 600 June 22 – $31,920 Direct Labor Cost of June were as follows: June 15 payroll $23680 June 29 payroll $25,960 Direct Materials requested by production during June were as follows June 6 $37,240 June 23 $38,960 On June 30, Tog Designs sold on account finished goods with a cost of $183,000 for $320,000 Required 1. Using T accounts for Materials Inventory, Work In Process Inventory, Finished Goods Inventory, Overhead, Accounts Receivable, Payroll Payable, Sales, and Cost of Goods Sold, reconstruct the transaction in June, including applying overhead to production. 2. Compute the cost of Units completed during the month 3. Determine the ending inventory balances 4. Jobs 24 A and 24 C were completed during the first week of July. No additional materials cost were incurred, but Job 24 A required $960 more of direct labor, and Job 24 C needed an additional $1610 of direct labor. Job 24 A was composed of $1800 pairs of trousers; Job 24 C, of 900 shirts. Compute the product unit cost for each job.

manlius manufacturing co produces only one product you have obtained the following i 670813

Manlius Manufacturing Co. produces only one product. You have obtained the following information from the corporation’s books and records for the year ended December 31, 2013: 

a. Total manufacturing cost during the year was $1,000,000, including direct materials, direct labor, and factory overhead. 

b. Cost of goods manufactured during the year was $970,000. 

c. Factory Overhead charged to Work in Process was 75% of direct labor cost and 27% of the total manufacturing cost. 

d. The beginning Work in Process inventory, on January 1, was 40% of the ending Work in Process inventory, on December 31. 

Required: 

Prepare a statement of cost of goods manufactured for the year ended December 31 for Manlius Manufacturing.

pls provide a quote 670846

HA2042 – Accounting Information Systems Assignment 2 Due: 5pm Friday, Week 11 You are to prepare a submission to satisfy your client’s requirements as outlined below. You are required to investigate the issues at Pressure Hydraulics and provide a feasibility study into the improvement to their current business processes. You will need to conduct an investigation into the client’s problems and evaluate a solution to meet their business needs.

Document Preview:

HA2042 – Accounting Information Systems Assignment 2 Due: 5pm Friday, Week 11 You are to prepare a submission to satisfy your client’s requirements as outlined below. You are required to investigate the issues at Pressure Hydraulics and provide a feasibility study into the improvement to their current business processes. You will need to conduct an investigation into the client’s problems and evaluate a solution to meet their business needs. Overview – Pressure Hydraulics Pressure Hydraulics is a locally owned business that currently has three service centres; Newcastle, Toronto and Maitland. Each service centre provides maintenance and specialised servicing of hydraulic systems as used in cars, trucks and earth moving equipment. The Maitland service centre also provides a specialty service to the mining industry where two purpose built trucks go onsite to service a range of mining equipment. The business has become quite profitable in the past years and its owner, Allan Taylor, has devised plans to expand by opening service centres at Coffs Harbour and Gosford. Allan has future plans for other service centres along the east coast of NSW. He also feels that the time is right to look at how IT can support the existing business and enable his future business plans. Currently, Allan spends a portion of each day at each service centre to monitor its operations. This is leaving little time to continue developing his business and he realises he will not be able to spend the same sort of time in the Coffs Harbour and Gosford service centres. Existing System Each service centre operates as an independent business, with eight technicians in the workshop (one of whom is a workshop foreman) and one office assistant. The office assistant takes phone calls from people requesting quotes for work or to have work done. For requests on quotes, the office assistant looks up a hard copy of a price book (known as The Price Book) and gives a…

Attachments:

here is a look at why it costs 2 50 for a single dip ice cream cone at a typical sto 671154

Here is a look at why it costs $2.50 for a single dip ice cream cone at a typical store in Washington, DC. The annual sales volume (the number of ice cream cones sold) averages around 185,000 cones, bringing in revenue of $462,500. This is equivalent to selling more than 500 cones a day, assuming a seven day operation. The following table shows the unit price of an ice cream cone and the costs that go into producing the product:

ItemsTotal CostUnit Price*% of PriceIce cream (cream, sugar, milk, and milk solids)$120,250$0.6526%Cone9,2500.052Rent112,8500.6124Wages46,2500.2510Payroll taxes9,2500.052Sales taxes42,5500.239Business taxes14,8000.083Debt service42,5500.239Supplies16,6500.094Utilities14,8000.083Other expenses (insurance, advertising, fees, and heating and lighting for shop)9,2500.052Profit24,0500.135Total$462,500$2.50100

If you were to classify the operating costs into either product costs or period costs, how would you do it?

carrie is a licensed hairstylist and operates her own business 667678

Carrie A. Morgan, age 45, is single and lives with her dependent mother at 426 Grouse Avenue, Allentown, PA 18105. Her social security number is 111 11 1111. 1. Carrie is a licensed hairstylist and operates her own business. Located at 480 Laurel Street, Allentown, PA 18105, the business is conducted under the name of “Carrie’s Coiffures.” Carrie’s business activity code is 812112. In addition to 10 workstations (i.e., stylist chairs) and a small reception area, the shop has display and storage areas for the products Carrie sells (see item 2 below).

Document Preview:

Carrie A. Morgan, age 45, is single and lives with her dependent mother at 426 Grouse Avenue, Allentown, PA 18105. Her social security number is 111 11 1111. 1. Carrie is a licensed hairstylist and operates her own business. Located at 480 Laurel Street, Allentown, PA 18105, the business is conducted under the name of “Carrie’s Coiffures.” Carrie’s business activity code is 812112. In addition to 10 workstations (i.e., stylist chairs) and a small reception area, the shop has display and storage areas for the products Carrie sells (see item 2 below). During the year, Carrie leased nine of the stations to other hairstylists. As is common practice in similar businesses in the area, the other stylists are considered to be self employed. In fact, the IRS sanctioned the self employment classification for the stylists in an audit of one of Carrie’s prior tax returns. Each stylist pays Carrie a fixed rent for the use of a workstation, resulting in $68,000 of rents received during 2012. From her own station, Carrie earned $44,000 (including tips of $12,000) for the styling services she provided to her own clients. 2. Carrie’s Coiffures is the local distributor for several beauty products (e.g., conditioners, shampoos) that cannot be purchased anywhere else. Carrie buys these items from the manufacturers and sells them to regular patrons, walk in customers, and other beauticians (including those who lease chairs from her). Carrie’s Coiffures is also known for the selection and quality of its hairpieces (i.e., wigs, toupees). Through the shop, Carrie made the following sales during the year: Hairpieces and wigs $69,000 Beauty products 48,000 3. Although Carrie operates her business on a cash basis, she maintains inventory accounts for the items she sells as required by law. Relevant information about the inventories (based on lower of cost or market) is summarized below. 4. 12/31/11 12/31/12 Hairpieces and wigs $10,700 $12,600 Beauty products 11,400…

Attachments:

a hamburgers and more inc sells hamburgers drinks and fries the sales mix is 1 3 2 i 667688

a) Hamburgers and More, Inc., sells hamburgers, drinks, and fries. The sales mix is 1:3:2 (i.e., for every one hamburger sold, three drinks and two fries are sold). Using the contribution margin approach, find the breakeven point in units for each product. The company’s fixed costs are $2,040. Other information is as follows:

Selling Price

per Unit

Variable Costs

per Unit

Hamburgers

$0.99

$0.27

Drinks

0.99

0.09

Fries

0.99

0.15

illustration vikas sharma was carrying on business of hosiery goods on ist january 1 667708

ILLUSTRATION

Vikas Sharma was carrying on business of hosiery goods. On ist January 1978 his assets and liabilities were as follows:

Assets: plant and Machinary Rs 6000, Furniture Rs 2000, Stock of goods Rs 8000, Cash in hand Rs 1100, Cash at bank Rs 5000, Amount due from Sonnu Rs 2010, Amount due from Raunak Singh Rs 777.

Liabilities: Amount due to Batra & Co. Rs 400, Bhuller & Co Rs 308, and Bhatia & sons Rs 1616.

The following transactions took place during the month of January 1978:

1. Purchased goods from Batra & co. Rs 1800

2. Cash sales Rs 1350

3. Deposited in Bank Rs 400

4. Sold goods to sonnu Rs 750

5. Cash received from raunak Singh Rs 760

Discount allowed Rs 17

6. Paid cheque to Batra & co on account Rs 1000

7. Hoseiry goods taken for domestic use Rs 250

8. Bought Postage stamps rs 25

Bought National Plan certificates(by cheque) rs 3000

Paid for advertising expenses rs 275

9. Withdrawn from Bank for office use Rs 175

10. Withdrwan from Bank for domestic use Rs 211

11. Paid cheque to Bhuller & co. in full settlement rs 300

12. The above cheque is dishonoured because of wrong stamping, Cheque withdrawn and cash paid to Bhuller & co.

13. Purchased goods from Bhatia & sons rs 2600

14. Cash received from Sonnu on account Rs 1400

15. Goods destroyed by fire Rs 240

16. Old Furniture(Cost Rs 400) sold rs 280

17. Sonnu declared insolvent: Only 30% is received against amount due from him

18. Repairs to machinery Rs 155

19. Returned defective goods to Bhatia & sons rs 200

Received from Durga Dutt whose account was written off earlier

20. Provide for:

a. Interest on capital: rs 70

b. Rent due to landlord:rs 300

21. Paid Income tax( personal) rs 160

ILLUSTRATION

Journalise the following transactions in the books of sh V.K rao, post them into ledger and prepare a trial balance as at 30th june 2003.

1. Commenced business with a capital of Rs 10,000

2. Purchased office furniture for Rs 1500

3. Purchased goods from Mr Anil for Rs 3000

4. Sold goods to Mr Azad for Rs 2500

5. Received cash from Mr Azad Rs 2400 in full settlement

6. Paid to Mr Anil in cash Rs 2800 and he allowed discount Rs 200

7. Sold goods for cash Rs 800

8. Opened a bank account with Rs 1100

9. Paid salary to Mr Gokul Rs 400

10. Withdrew cash from bank for personal use Rs 300

JOURNALISE THE FOLLOWING TRANSACTIONS, POST THEM INTO LEDGER AND PREPARE A TRIAL BALANCE AS ON 30TH SEPTEMBER 2003.

1. SMITH COMMENCED BUSINESS WITH CASH RS 30,000

2. PURCHASED GOODS FOR CASH RS 1250

3. DEPOSITED INTO BANK RS 22,500

4. BOUGHT FURNITURE FOR OFFICE USE RS 3500

5. SOLD GOODS TO STEPHENS RS 1500

6. DREW FROM BANK FOR OFFICE USE RS 2500

7. BOUGHT GOODS FROM ROBERT RS 1025

8. RECEIVED CASH FROM STEPHENS RS RS 1475

ALLOWED HIM DISCOUNT RS 25

9. PAID FOR MISCELLANEOUS EXPENSES RS 250

10. PAID ROBERT IN FULL SETTLEMENT RS 1000

11. PAID SALARY TO KEITH RS 125

12. PAID RENT TO LANDLORD RS 250

13. ALLOW INTEREST ON CAPITAL RS 250

Attachments:

problem 9 2a accounting navaro corporation 667715

Problem 9 2A

At December 31, 2014, Navaro Corporation reported the following plant assets.

Land $ 5,151,000

Buildings $29,420,000

Less: Accumulated depreciation—buildings 20,475,225 8,944,775

Equipment 68,680,000

Less: Accumulated depreciation—equipment 8,585,000 60,095,000

Total plant assets $74,190,775

During 2015, the following selected cash transactions occurred.

Apr. 1 Purchased land for $3,777,400.

May 1 Sold equipment that cost $1,030,200 when purchased on January 1, 2008. The equipment was sold for $291,890.

June 1 Sold land for $2,747,200. The land cost $1,717,000.

July 1 Purchased equipment for $1,888,700.

Dec. 31 Retired equipment that cost $1,201,900 when purchased on December 31, 2005. No salvage value was received.

Journalize the transactions. Navaro uses straight line depreciation for buildings and equipment. The buildings are estimated to have a 40 year useful life and no salvage value; the equipment is estimated to have a 10 year useful life and no salvage value. Update depreciation on assets disposed of at the time of sale or retirement. (Record entries in the order displayed in the problem statement. Credit account titles are automatically indented when amount is entered. Do not indent manually.)

Date Account Titles and Explanation Debit Credit

(To record depreciation on equipment sold)

(To record depreciation on equipment retired)

SHOW LIST OF ACCOUNTS

LINK TO TEXT

LINK TO VIDEO

Record adjusting entries for depreciation for 2015. (Credit account titles are automatically indented when amount is entered. Do not indent manually.)

Date. Account Titles and Explanation Debit Credit

Dec. 31

(To record depreciation on buildings.)

31

SHOW LIST OF ACCOUNTS

LINK TO TEXT

LINK TO VIDEO

Prepare the plant assets section of Navaro’s balance sheet at December 31, 2015. (Hint: You may wish to set up T accounts, post beginning balances, and then post 2015 transactions.) (List Plant Assets in order of Land, Building and Equipment.)

NAVARO CORPORATION

Partial Balance Sheet

December 31, 2015

$

$

:

:

$

SHOW LIST OF ACCOUNTS

LINK TO TEXT

LINK TO TEXT

LINK TO VIDEO

By accessing this Question Assistance, you will learn while you earn points based on the Point Potential Policy set by your instructor.

Question Attempts: 0 of 3 used SAVE FOR LATER

SUBMIT ANSWER

Copyright © 2000 2014 by John Wiley & Sons, Inc. or related companies. All rights reserved.

Attachments:

define and identify its variable fixed and mixed costs 667716

The purpose of the Discussion Board is to allow students to learn through sharing ideas and experiences as they relate to course content and the DB question. Because it is not possible to engage in two way dialogue after a conversation has ended, no posts to the DB will be accepted after the end of each week.
As EEC’s corporate business financial analyst, you will need to have a clear understanding of the different types of costs (variable, fixed, and mixed) that the company carries. Complete the following for this assignment:
•Review EEC’s journal activity.
•Define and identify its variable, fixed, and mixed costs.
•Determine what affect a sales volume increase or decrease will have on unit fixed cost, unit variable cost, total fixed cost, and total variable cost.
Click here to view EEC’s journal activity.
In your own words, please post a response to the Discussion Board and comment on other postings. You will be graded on the quality of your postings.

Document Preview:

ACCT614 1403B 02 Applied Managerial Accounting Task Name: Phase 2 Discussion Board Mayra V Cruz August 27,2014 Professor: The purpose of the Discussion Board is to allow students to learn through sharing ideas and experiences as they relate to course content and the DB question. Because it is not possible to engage in two way dialogue after a conversation has ended, no posts to the DB will be accepted after the end of each week. As EEC’s corporate business financial analyst, you will need to have a clear understanding of the different types of costs (variable, fixed, and mixed) that the company carries. Complete the following for this assignment: •Review EEC’s journal activity. •Define and identify its variable, fixed, and mixed costs. •Determine what affect a sales volume increase or decrease will have on unit fixed cost, unit variable cost, total fixed cost, and total variable cost. Click here to view EEC’s journal activity. In your own words, please post a response to the Discussion Board and comment on other postings. You will be graded on the quality of your postings.

Attachments:

you are required to develop a cost analysis for a product or service using a job ord 667749

PART A: INTRODUCTION

You arerequired to develop a cost analysis for a product or service usinga job order costing systemfor a real orfictitious manufacturing or service entity. The unique feature of a job order costing system is that the product/service needs to bemultiple distinct productsor services. Examples of entities that use job order costing systems are special purpose machinery manufacturers, furniture makers, construction companies, motor car repair shops, advertising agencies, accounting firms (see example in the unit’s Cloud Deakinwebpage a video interview with the principal partner of LBW Chartered Accountants), consulting firms, financial advisors, healthcare centres, and law firms. You are free to use any entity as long as the products/services are distinct jobs and are costed using a job order costing system. It will make your work easier if you choose a real entity that you are familiar with. Even better, if you can visit the entity, talk to staff and gather data, you will enjoy the assignment and produce a costing analysis based on close to real data.

You have been hired asa consultant to a business requiringa job order costing system to be developed usingspreadsheetmodelling. You are required to develop thespreadsheetmodel (see detailed requirements of the consulting brief below),and once completed, prepare the manufacturing statements and income statement andtransmit your reportsto the business by:

Using a business report format

Include:

Acover letter

Table of contents

Executive Summary

Bibliography

BasicProductRequirements:

Your product or service must comprise:

Between 2 and 3differenttypes of Raw Material (if applicable)

Between 2 and 3 Cost Centres

Between 2 and 3 levels of Direct Labour skill

Selectinga real orcreating afictitiousentity

In selecting arealorcreatinga fictitiousentity, keep the following in mind:

Ensure you areable to understand the basic workings of the organisation relatively easily.

Choose a small or medium sizedorganisation oradepartment or production sub unit in a large organisation.

Limit the number of jobs to between 3 and 5.

Keep in mind that in doing this project, your maintaskis creating the data.The real challenges in management accounting are not computational but data insufficiency, unavailability and interpretation. It is important to learn how to address these challenges. Be confident that you will be able to estimateand develop information for cost analysis.

If in doubt,consultyour tutoras early as possibleabout thesuitability of theentity you are considering.

REQUIRED:

PART A SECTION 1:Write a Report Describingthe actual or fictional entity selected (1,000 Words worth 25% of the 25 marks allocated for the group work).

In Part A section 1, you arerequired to write a reportdescribing the actual or fictional entity selected(maximum 1,000 words).The report should bewell structuredand includethe following points:

Introduction (introducing the assignment and what you are going to accomplish);

A description of the entity that you have selected or created – the industry in which it operates, type of entity,organisational structure,location, number of employees, approximate annual turnover, major customers or markets, major competitors (do the research and/or be creative use your imagination!);

The cost objects, i.e., theproducts/servicesthatthe entityoffers;

Resources usedand major activities performed in producing thejobsexplaining the structure of the production process, what department produces what part of the job.

The cost structure of the entity – the relative proportion of thefixedcosts and variable costs;

A cost flow diagram (see Exhibit 3 5on page 94 of Garrisonfor an example);and

Why a job order costing system is appropriateforthe entity.

PART A SECTION 2:Perform a cost analysis (1,000 Words worth 60% of the 25 marks).

Part A Section 2 has 3 sub sections, with the 60% of the 25 marks allocated as follows:

Sub section 1:Data Section 20%of 25 Marks

Sub section 2:Computation Section 20%of 25 Marks

Sub section 3:Output section 20%of 25 Marks

In Part A section2, you are required to performa Cost Analysis of the organisation and products(maximum 1,000 words).

The report mustbewell structured, and MUST be done on aSpreadsheetusing the capabilities of thespreadsheetin good form – In other words, use formulae wherever possible and not absolute numbers, and make sure that the information is comprehensible to the reader – Guide your reader around thespreadsheet. Aspreadsheetpacked with numbers and no guidance is difficult to follow. We recommend that you become familiar with how data is presented inspreadsheets.

PartA Section 2is to be completed assuming you will be developing a job order cost analyses for an entity that has been in operation for some years. As such, you will have beginning balances of raw materials, work in process, finished goods; and also some incomplete jobs started in the previous year to be completed in the current year.If you choose a service entity such as an accounting firm, you will not haveraw materialsinventory.You are strongly advised to study the concepts and applications of job order costing before you begin working on this part of the assignment.

Sub Section1:Data section(20%of 25 Marks)

Beginning Inventories

Provide beginning balances of raw materials, work in process (for incomplete jobs carried over from the previous year, provide product/project/service names for the incomplete jobs e.g., Job A, Job B. Limit the number of incomplete jobs at the beginning of the year tobetween 3 and5) and finished goods (for jobs completed in the previous year but not yet transferred to the customers, provide product names for the completed jobs such as Job C, Job D). If your entity is a service organisation,finished goods inventory willnot be applicable.

Provide details of the cost components of theincomplete jobs at the beginning of the year(materials, labour and overhead).

Raw Materials (Between 2 and 3 Raw Materials, if applicable)

Raw materials purchased.

Raw materials used. Assume some of the materials available for use were not used, i.e., you have an ending raw materials inventory.

Direct Labour(Between 2 and 3 types of Direct Labour Skills)

Describe the Direct Labour Skills required.

Cost of Direct Labour

Manufacturing Overheads(Between 2 and 3 Cost Centres)

Usepre determined overhead rates.

Estimatethe totaloverheadbudget for the year.You need to consider fixed overhead required to complete the incomplete jobs, jobs that will be started and completed in the current year and jobs that will be started butnot completed at the end of the year. Your starting point should be to decide on the number of all the jobs and their estimated fixed overhead cost. Limit the number of jobs to between 3 and 5.

Estimate thequantity of the overhead application base for the year, explaining why you have chosen the particular base.

General

List the jobs startedduringthe year.Limit the number of new jobsstarted in the current year to between 3 and5.Give names for the jobs.

List of the jobscompletedand their associated costs (show clearly the materials, labour and overhead components)duringthe year.Limit the number of jobs completed and transferred to customers to between 3and5.

List of the jobs started but not completed at the end of the yearand their associated costs (show clearly the materials, labour and overhead components).Similar to the number of jobs at the beginning of the year, limit the number of jobs incomplete at the end of the year to between 3 and5.

Actual overhead incurred during the year (you need to show at least five overhead items that are incurred during the year, e.g., depreciation of equipment, electricityetc.).

Actual costs incurred during the year for expensesother than overhead. You need to have at least 10 expense items.See Review Problem: Job OrderCostingon p. 109 of your text book for examples of transactions.You are not required to prepare the journal entries. You only need tosummarise the expenses based ofthe transactions.

Summary of the revenue fromeach jobcompletedand sold.

Forms to be used:

Materials requisition form (see Exhibit 3 1, p. 86 in your text book).

Job cost sheet (see Exhibit 3 2, p. 87 in your text book).

Employee time ticket (see Exhibit 3 3, p. 88 in your text book).

Sub Section2: Computation Section 20% of 25 Marks

Cost of Raw Materials used in production.

Cost of Direct Labour used in production.

Predeterminedoverhead ratefor the year (including the overhead applied during the year to the different jobs.

Sales of manufactured products (Units and Revenue).

The unadjusted cost of goods sold.

The under or over applied overhead amount.

Dispose the under or over applied overhead amount to cost of goods sold, work in process and finished goods.

Dispose the under or over applied overhead amount tocost of goods soldonly.

The adjusted cost of goods sold assuming the under or over applied overhead is disposed tocost of goods soldonly.

The ending balances of materials, work in process and finished goods.

Sub Section3: Output section 20% of 25 Marks

A schedule ofthecost of goods produced (manufactured).

A schedule of the cost of goods sold.

An income statement for the year.

PART A SECTION 3:Evaluate the job order cost system(500 words worth15% of the 25 marks).

Write a report discussing the job order costing system developed. The report mustbewell structured and written and includethe following points:

The strengths and weaknesses of the systems.

Evaluate whether the present costing system reasonably accurately calculates the cost of products or jobs? Which products, if any, are under costed or over costed?

The ways management would use the information generated from the costing system.

Whether you would recommend that the entityshould considerimplementingan activity based costing system.

Some guidelinestodevelopingthe costing system

Begin the workas early as possible.

Choosetheentity you want todevelop the costing systemfor.

To be able to develop the job order costing system, you need to master the concepts and techniques from lectures, tutorials,text book, and any other relevant resource.

Use the lecture illustration and the examples in yourtext bookas guides.

Decide on the resources your entity is going to use human resources, building, equipment, transport, water, energy, communication, and any other resources. Decide on the costs of these resources wages, salaries, depreciation, maintenance, repairs, electricity, water, gas, rent, rates, telephone, fax, postage,stationery, outsourced costs etc.

The dollar costs do not have to beaccurate guesstimates are fineand you can change these amounts later if necessary.

Decide on the major activities your business is performing on the entire value chain to produce and market the products to its customers.

Adjustcosts and revenuesas necessary to produce a realistic costing system.

Check that all your formulaeand computations are accurate and that the results generated areplausible and realistic. If you are not happy with the final results or they appear to be incorrect or unrealistic, correct any errors and adjustthedata so that your end results are plausible and realistic.

Keep it simple. Do not unnecessarily complicate the system butat the same time, do not makeittoosimple and unrealistic.

PART B – THESELF REFLECTION REPORT(5/30)

Once PART A of the assignment is complete, you are required to prepare aself reflectionreportas follows:

Please reflect on the experience you had working on this assignment and answer the following questions as accurately and honestly as possible (limited to 500 words and wortha total of 5/30 of the assignment mark, and worth 5% of the overall assessment in this unit. Themark attained is awarded to the individual student).

What did you enjoy/not enjoy about the assignment?

Did you find the assignment challenging?

Were the materials provided in the unit useful in doing the assignment and to whatextent?

How proficient were you with Excel, and did this assignment help you develop these skills?

Did you find it to be a learning experience?

Why?

Why not?

Note: Please submit yourindividualreview onCloud Deakinat the ‘Reflection on assignment’ folder.

Attachments:

the explanatory notes to financial statements a should be referred to if more than a 667764

The explanatory notes to financial statementsa. should be referred to if more than a cursory, and perhaps misleading, impression of a firm’s financial position and its results of operations is to be achieved.b. are used by many entities to hide information from the reader of financial statements by including in the explanatory notes information that should be shown in detail on the financial statements themselves.

Recognition of revenue in accrual accounting requiresa. only that the amount of cash to be received from the sale of a product or service be knownb. that the revenue be realized or realizable, and earnedc. only that a product be delivered or a service be performed

Gains differ from revenues because gainsa. aren’t a result of the entity’s ongoing, central operations.b. don’t involve any offsetting costs or expensesc. don’t have to be realized

the government wants to encourage the use of lpg to protect the environment by reduc 670285

Case study 6.2: Converting to LPG – is it worth it?

Green fuel runs out of gas5

The cost of converting a car to run on liquefied petroleum gas (LPG) is about £1,500 in the UK, towards which a government grant would contribute about £700. From September 1 2004, LPG will on average cost 40.7p per litre, compared with 79.1p for ordinary unleaded petrol. However, LPG cars usually have slightly worse fuel consumption, losing about 13% in terms of miles per gallon.

Questions

Fast Trak Company owns a fleet of 20 cars, which are bought new and are used for 30,000 miles over two years before being sold off. The cars average 30 miles to the gallon (imperial) on petrol. The conversion to LPG does not affect the price in the second hand market.

1 Calculate the profit contribution per hundred miles of LPG compared with unleaded petrol, for one of Fast Trak’s cars.

2 Calculate the break even mileage for the cars with the LPG conversion.

3 Calculate the effect on the profit of Fast Trak of converting to LPG.

4 The government wants to encourage the use of LPG to protect the environment by reducing the break even mileage to 10,000 miles; how large a grant should it offer for the LPG conversion?

the management at easyloan bank is interested in offering a corporate fitness progra 670287

The management at Easyloan Bank is interested in offering a corporate fitness programme for its employees. It has decided to offer a 50 per cent subsidy to all who participate. The gym has agreed to offer the bank a 20 per cent discount on all fees for its employees, provided that they all have the same terms of membership. The bank’s management observe from the membership information (Table 6.3) that there are various membership options. From a survey of its employees it establishes that they would use the gym only. Of their visits, 75 per cent would be during weekdays and 25 per cent at weekends. Half their weekday visits would be before 12 noon and the other half would be after 5 p.m.

Questions

1 Identify the feasible membership options and express these as cost functions for the bank.

2 Draw a graph showing the above options and explain how the bank’s management should select the optimal one.

 width=

a firm can have diminishing returns and increasing returns at the same time but not 670288

1 Explain the difference between the explicit cost of buying a textbook on economics and the opportunity cost, stating any assumptions. How are these costs relevant for the decision to buy the book?

2 Explain the relationship between a firm’s short run production function and its short run cost function.

3 Explain whether the following statements are true or false:

a. In the long run a firm might choose to operate a larger plant at less than maximum efficiency rather than a smaller plant at maximum efficiency.

b. Maximum efficiency is achieved when AVC and MC are equal.

c. An improvement in technology will shift the LAC curve upwards.

d. If AVC and price stay the same when there is an increase in fixed costs the

BEO will decrease.

e. A firm can have diminishing returns and increasing returns at the same time but not economies and diseconomies of scale.

quikpak sells returnable containers to major food processors the price received for 670290

1. Quikpak sells returnable containers to major food processors. The price received for the containers is £2 per unit. Of this amount £1.25 is profit contribution. Quikpak is considering an attempt to differentiate its product through quality improvement at a cost of 5p. per unit. Current profits are £40,000 on sales of 100,000 units.

a. Assuming that average variable costs are constant at all output levels, find Quikpak’s total cost function before the proposed change.

b. Calculate the total cost function if the quality improvement is implemented.

c. Calculate Quikpak’s break even output before and after the change, assuming it cannot increase its price.

d. Calculate the increase in sales that would be necessary with the quality improvement to increase profits to £45,000.

2. Two business students are considering opening a business selling hamburgers next summer. The students view this as an alternative to taking summer employment with a local firm where they would each earn £3,000 during the three month summer period. It would cost £2,000 to obtain a licence to operate their stand, £1,000 per month to rent the stand with the necessary equipment and £100 per month for insurance. Petrol costs are estimated at £10 per day. Hamburger meat can be bought for £4.00 per kilo and buns cost £1.20 per dozen. The burgers would sell in 125 gram patties for £1.50 each.

a. Find the accounting cost function for the proposed business.

b. Find the economic cost function for the proposed business.

c. Calculate the level of output where the business would make normal profit.

d. Cheese slices for the burgers can be bought for £2.40 for twenty slices and it is estimated that 30 per cent of customers would ask for the cheeseburgers, with these selling for £1.95. Calculate the effect of this on the output necessary to make normal profit.

e. If the students can sell 150 burgers a day, 30 per cent with cheese, estimate the economic profit they would make. Advise them whether they should enter the business.

blunt corporation manufactures calculators involving a product line of two models th 670291

1. Blunt Corporation manufactures calculators, involving a product line of two models. The current price charged for its B1 model is £8, and profit contribution is 25 per cent. Sales have been disappointing for this model, so Blunt is considering a quality improvement at a cost of 50p. per calculator. Advertising would be increased by £40,000 to promote the improvement. Current profit is £60,000 on sales of 80,000 items per month. There is a relationship between sales of B1 and B2 models, such that for every four extra sales of a B1 model there is one less sale of a B2 model, as some consumers switch from one model to the other. The B2 models sell for £6 and have an average variable cost of £5. a. Determine the cost function for the B1 model with the changes that Blunt is considering. b. Calculate the increase in sales of B1 models necessary with the quality improvement in order to earn increased profits of £40,000, ignoring the effect on sales of B2 models. c. Calculate the increase in sales of B1 models necessary with the quality improvement in order to earn increased profits of £40,000, taking into account the effect on sales of B2 models.

2. Haedoo is presently struggling to survive in the motor car industry. Competition is increasing and the company is plagued by overcapacity. Its capacity is 2 million units per year, but it is currently operating at only 70 per cent of this level, and this is resulting in an annual loss of $480 million. Its profit contribution per unit is 25 per cent. Haedoo has now set targets for its performance in two years’ time: it aims to reduce its capacity to 1.5 million units and operate at 80 per cent of this level; it aims to have a profit of $800 million; and in order to achieve its target output and sell it, it aims to keep its prices the same, while reducing its level of unit variable costs to 90 per cent of their current level by rationalizing its supply procedures and standardizing components.

a. Calculate the target average level of price for Haedoo.

b. Calculate the target level of unit variable costs.

c. Calculate the target level of break even output.

what factors do you think might be responsible for the current trends of increasing 670293

Case study 7.1: Banking There are various theoretical reasons why economies of scale should occur in the banking industry:

1 Specialization of labour. There is considerable scope for this as cashiers, loan officers, account managers, foreign exchange managers, investment analysts and programmers can all increase their productivity with increased volume of output.

2 Indivisibilities. Banks make use of much computer and telecommunications technology. Larger institutions are able to use better equipment and spread fixed costs more easily.

3 Marketing. Much of this involves fixed costs, in terms of reaching a given size of market; large institutions can again spread these costs more easily.

4 Financial. Banks have to raise finance, mainly from depositors. Larger banks can do thismore easily and at lower cost, meaning that they can afford to offer their depositors lower interest rates. There are also reasons why banks should gain from economies of scope; many of their products are related and banks have increasingly tried to cross sell them. Examples are different types of customer

account, accounts and credit cards, accounts and mortgages or consumer loans, and even banking services and insurance.

There has also been a spate of bank mergers and acquisitions in recent years, often involving related institutions like building societies, investment banks and insurance companies. Many of these institutions have been very large in size, with assets in excess of $100 billion. Examples are Citibank and Travellers Insurance (now Citigroup), Bank of America and NationsBank, Chase Manhattan and J. P. Morgan, HSBC and Midland; both NatWest Bank and Abbey National Bank in the UK have been the object of recent takeover bids. This would tend to support the hypothesis that ‘bigger is better’.

The empirical evidence, however, is not supportive of the ‘bigger is better’ policy that many banks seem to be following. A number of empirical studies have been carried out regarding commercial banking and related activities, in both Europe and the United States. Some US studies in the early 1980s found diseconomies for banks larger than $25 million14 or $50 million15 in assets, a very small size compared with the current norm (the largest banks now have assets in excess of $500 billion). More recently a greater availability of data has enabled research to be carried out on much bigger banks, as deregulation in 1980 led to interstate banking in the United States. Shaffer and David16 examined economies of scale in ‘superscale’ banks, that is banks with assets ranging from $2.5 billion to $120 billion in 1984. They estimated that the minimum efficient scale of these banks was between $15 billion and $37 billion in assets, and that these larger banks enjoyed lower average costs than smaller banks.

Many of the studies have been summarized by Clark in the USA.17 In particular, Clark’s conclusions were that there are only significant economies of scale at low levels of output (less than $100 million in deposits). Furthermore, it appeared that economies of scope were limited to certain specific product categories, for example consumer loans and mortgages, rather than being generally applicable.

Questions

1 What shape of long run average cost curve appears to be appropriate for the commercial banking industry?

2 What mathematical form of cost function would be most appropriate to use to test the existence of economies of scale in banking?

3 What factors might cause the LAC curve to flatten out at high levels of output?

4 In view of the empirical evidence, what factors do you think might be responsible for the current trends of increasing size and mergers?

the airline industry has very marked differences in structure between the united sta 670294

Case study 7.2: Airlines

The airline industry has very marked differences in structure between the United States and Europe. US airlines were deregulated in 1978, leading to intense competition on many routes, bankruptcies and mergers. In Europe the airlines have remained highly regulated, with governments protecting their own largely state owned airlines. The deregulation in the US has led to much lower fares per passenger mile and has caused some radical changes in structuring and therefore unit costs. Economies of scale arise if airlines can make use of a single type of plane. For example, Southwest Airlines only flies the Boeing 737, while the struggling US Airways uses a number of different aircraft. This was largely because US Airways came into existence as a result of a series of mergers. Southwest can therefore reduce its maintenance costs, crew training and scheduling costs compared with US Airways. This was reflected in the fact that in 1993 its costs were only 7 cents per available seat mile, compared with over 11 cents for US Airways.

Economies of scope are also very important in the airline industry. Clearly it is cheaper for an airline to carry passengers and freight in the same aircraft than carry them in separate aircraft. However, the most important economy of scope arose from a change in routing: instead of having a large number of point to point services between different cities, airlines like Southwest and Continental switched to using a ‘hub and spoke’ system. Thismeant that customers flew from one city to a central hub, and then from there to their final destination, as shown in Figure 7.4. This gave these airlines a number of advantages. They could fly amuch larger number of routes, in terms of joining city pairs, with a given number of aircraft and miles flown. This would attractmore passengers. Since these passengers all landed at the hub, before being transferred to their final destination, it allowed the airlines to fill the flights more comprehensively. It also became possible to combine passengerswith different characteristicsmore effectively; thus routes that largely carried business passengers could now attract vacation customers.18 The above economies of scale and scope have not prevented many airlines from getting into serious financial difficulties recently. Although the events of 11 September 2001 did not help the industry, there were already many problems brewing in terms of overcapacity and industrial relations. Many airlines now appear likely to go out of business unless they receive government subsidies, a controversial issue discussed in more detail in Chapter 12. Questions

1 Explain how you would conduct an empirical study to investigate the availability of economies of scale in the airline industry.

2 What implications are there regarding mergers of different airlines?

3 What disadvantages might be incurred with the hub and spoke system compared with the pointto point system?

4 Explain why the hub and spoke system relates to economies of scope rather than economies of scale.

 width=

what is happening to unit costs for producing wind power 670295

Case study 7.3: Electricity generation

This industry has been the subject of many empirical studies in different countries, going back to the 1950s. These studies have been facilitated by three factors:

1 Output is easy to measure (electricity produced, usually in billions of kilowatt hours).

2 Different companies, or state utilities, have plants of different sizes.

3 Large amounts of data are available, since producers often have to record this for regulatory purposes.

Johnston19 estimated long run cost functions using both time series and cross sectional data, for the period 1928–47 and for twenty three firms. Although a cubic total cost function was specified, he found that the cubic term was not statistically significant in any of the regression equations; the quadratic term was found to be significant in only six of the equations, and was positive in three cases and negative in the other three cases. In general, the linear total cost model tended to fit best.

Two major studies were conducted in the United States in the 1970s. Christensen and Greene20 used a cross sectional study involving 1970 data for 114 firms, specifying a power model to test for the presence of economies and diseconomies of scale. They found significant economies of scale below about 20 billion kwh in output, relating to 85 per cent of the firms and nearly 49 per cent of total output. Between this output and 67 billion kwh they observed no significant economies of scale; this related to another 14 per cent of the firms and 45 per cent of output. Above this output they found that diseconomies of scale occurred for the one remaining firm, producing about 7 per cent of total output. This pattern of average costs is shown in Figure 7.5.

Another study by Huettner and Landon21 examined seventy four utility firms, using 1971 data, and specifying a cubic model. Their conclusions were broadly the same as for the previous study: the LAC curve was U shaped.

Since the 1970s there have been great changes in the electricity generation industry. Alternative sources of power have become more important, owing to a combination of market forces, environmental concern and improvements in technology. The fossil fuels, coal and oil, have declined in importance, while natural gas, a much cleaner fuel, has gained in importance. Unfortunately, for many utility companies who have had to buy electricity recently from gas powered sources, demand has far outstripped supply, and prices of gas have soared, in turn causing electricity prices on the wholesale market to soar. In northern

 width=

California, for example, electricity prices are now ten times their level at the beginning of 2000. Hydroelectric power, from dams, has also become more popular on a global basis, though this too has caused environmental and political problems.

When the electricity utility firmswere deregulated in the United States fromthe 1980s, deregulation posed problems for many of them. The traditional means of producing electricity, whether coal or oil fired power stations or hydroelectric plant, involved very high fixed costs. Being sunk costs, these costs then became ‘stranded’ after deregulation, since electricity buyers could then buy from out of state suppliers; this practice is known as ‘freewheeling’. Many utility firms lost customers to these competitive suppliers, which forced them to move to the left and upwards along their LAC curves, thereby increasing unit costs; this in turn made them even less competitive. These and other aspects of deregulation will be considered in another case study in Chapter 12.

Improvements in technology have led to cleaner methods of producing electrical power in recent years. Although still insignificant in terms of global market share, wind power is becoming more important, particularly in countries like Denmark and Germany. The cost structure of this technology is also different, and many of the indivisibilities of the older methods of production are avoided. A large 1.65 megawatt turbine now costs less than £1 million to build; although this is 20 times that of the 1980s version, it produces 120 times as much energy. Given the problems with other sources of energy, in terms of environmental pollution or volatile costs, wind power may prove to be a major source of energy in the future, along with other new technology sources like hydrogen power.

Questions

1. Interpret the results obtained by Johnston in terms of the quadratic terms of three equations being positive and the quadratic terms of the other three being negative.

2. Explain the relative advantages and disadvantages of using power or cubic cost functions to estimate LAC curves for electricity production.

3. Explain the meaning of ‘stranded costs’; why are they a problem?

4. What is happening to unit costs for producing wind power?

how many questionnaires will be processed by the end of tuesday 670299

MJ, a management consultant, is processing 500 questionnaires that the firm has received from respondents. MJ starts work on Monday at 9 a.m., takes an hour for lunch at 1p.m. and finishes at 5 p.m. The following progress is noted:

Time

Total questionnaires processed

since a.m.

9.30

 

10.00

 

10.30

15

11.00

24

a. Is there evidence of a learning curve? Give your reasons.

b. Estimate and interpret the learning rate.

c. How long did the first questionnaire take to process?

d. Estimate the day and time when MJ will finish processing all the questionnaires.

e. How long will the last questionnaire take to process?

f. How many questionnaires will be processed by the end of Tuesday?

how do you think changes in technology will affect the market structure of the power 670300

Case study 8.1: Electricity generation

Here and now8 Distributed power generation will end the long distance tyranny of the grid.

For decades, control over energy has been deemed too important to be left to the markets. Politicians and officials have been dazzled by the economies of scale promised by ever bigger power plants, constructed a long way from consumers. They have put up with the low efficiency of those plants, and the environmental harm they do, because they have accepted that the generation, transmission and distribution of power must be controlled by the government or another monopoly.

Yet in the beginning things were very different. When Thomas Edison set up his first heat and power co generation plant near Wall Street more than 100 years ago, he thought the best way to meet customers’ needs would be to set up networks of decentralised power plants in or near homes and offices. Now, after a century that saw power stations getting ever bigger, transmission grids spreading ever wider and central planners growing ever stronger, the wheel has come full circle. The bright new hope is micropower, a word coined by Seth Dunn of the WorldWatch Institute in an excellent report.* Energy prices are increasingly dictated by markets, not monopolies, and power is increasingly generated close to the end user rather than at distant stations. Edison’s dream is being revived.

The new power plants of choice the world over are using either natural gas or renewable energy, and are smaller, nimbler, cleaner and closer to the end user than the giants of yesteryear. That means power no longer depends on the vagaries of the grid, and is more responsive to the needs of the consumer. This is a compelling advantage in rich countries, where the digital revolution is fuelling the thirst for highquality, reliable power that the antiquated grid seems unable to deliver. California provides the best evidence: although the utilities have not built a single power plant over the past decade, individuals and companies have added a whopping 6gW of nonutility micropower over that period, roughly the equivalent of the state’s installed nuclear capacity. The argument in favour of micropower is even more persuasive in developing countries, where the grid has largely failed the poor.

This is not to say that the existing dinosaurs of power generation are about to disappear. Because the existing capital stock is often already paid for, the marginal cost of running existing power plants can be very low. That is why America’s coal fired plants, which produce over half the country’s power today, will go on until the end of their useful lives, perhaps decades from now – unless governments withdraw the concessions allowing them to exceed current emissions standards. While nobody is rushing to build new nuclear plants, old ones may have quite a lot of life left inthem if they are properly run, as the success of the Three Mile Island nuclear power plant in Pennsylvania attests. After the near catastrophic accident in 1979 that destroyed one of the plant’s two reactors, the remaining one now boasts an impressive safety and financial record. Safety and financial success are intimately linked, says Corbin McNeill, chairman of Exelon and the current owner of the revived plant. He professes to be an environmentalist, and accepts that nuclear power is unlikely to be the energy of choice in the longer term: ‘A hundred years from now, I have no doubt that we will get our energy using hydrogen.’ But he sees nuclear energy as an essential bridge to that future, far greener than fossil fuels because it emits no carbon dioxide.

GOOD OLD GRID

The rise of micropower does not mean that grid power is dead. On the contrary, argues CERA, a robust grid may be an important part of a micropower future. In poor countries, the grid is often so shoddy and inadequate that distributed energy could well supplant it; that would make it a truly disruptive technology. However, in rich countries, where nearly everyone has access to power, micropower is much more likely to grow alongside the grid. Not only can the owners of distributed generators tap into the grid for back up power, but utilities can install micropower plants close to consumers to avoid grid bottlenecks.

However, a lot of work needs to be done before any of this can happen. Walt Patterson of the Royal Institute of International Affairs, a British think tank, was one of the first to spot the trend toward micropower. He argues that advances in software and electronics hold the key to micropower, as they offer new and more flexible ways to link parts of electricity systems together. First, today’s antiquated grid, designed when power flowed from big plants to distant consumers, must be upgraded to handle tomorrow’s complex, multi directional flows. Yet in many deregulated markets, including America’s, grid operators have not been given adequate financial incentives to make these investments. To work effectively, micropower also needs modern command and communications software.

Another precondition is the spread of real time electricity meters to all consumers. Consumers who prefer stable prices will be able to choose hedged contracts; others can buy and sell power, much as day traders bet on shares today. More likely, their smart micropower plants, in cahoots with hundreds of others, will automatically do it for them.

In the end, though, it will not be the technology that determines the success of distributed generation, but a change in the way that people think about electricity. CERA concludes that for distributed energy, that will mean the transition from an equipment business to a service business. Already, companies that used to do nothing but sell equipment are considering rental and leasing to make life easier for the user. Forward looking firms such as ABB, a Swiss Swedish equipment supplier, are now making the shift from building centralised power plants to nurturing micropower. ABB is already working on developing ‘microgrids’ that can electronically link together dozens of micropower units, be they fuel cells or wind turbines.

Kurt Yeager of the Electric Power Research Institute speaks for many in the business when he sums up the prospects: ‘ Today ’s technological revolution in power is the most dramatic we have seen since Edison’s day, given the spread of distributed generation, transportation using electric drives, and the convergence of electricity with gas and even telecoms. Ultimately, this century will be truly the century of electricity, with the microchip as the ultimate customer.’

* ‘Micropower: the next electrical era’, by Seth Dunn. World Watch Institute, 2000.

Questions

1. Explain why power generation has traditionally been a monopoly in all developed countries.

2. What is meant by a transmission grid? How is this feature related to a monopolistic market structure?

3. What is meant by micropower? What are its implications for grid systems?

4. What are the implications of micropower for the environment?

5. How do you think changes in technology will affect the market structure of the power generation industry?

how does the rise of the internet affect this situation 670301

Case study 8.2: Price cuts for medicines

Chemists at risk as prices are slashed12

BY NIGEL HAWKES, HEALTH EDITOR

Big price cuts on a wide range of medicines and vitamins were promised by the supermarket chains yesterday as 30 years of price fixing were swept aside.

Many popular products, including painkillers, cough medicines, indigestion tablets and nutritional supplements are being halved in price from last night, with reductions of between 20 and 40 per cent on many others.

The Office of Fair Trading called it excellent news for consumers but the body representing small pharmacies said that many would close, threatening community services.

The big supermarkets trumpeted ‘millions of pounds worth of savings’ as they competed to offer the biggest reductions. At Asda, a packet of 16 regular Anadin will be 87p, instead of £1.75, and Nurofen tablets will cost £1.14 for 16, rather than £2.29.

Reductions at Tesco included a 40 per cent cut in Anadin Extra, to £1.29 for 16, while Sainsbury’s matched the Asda price for Nurofen, and reduced Seven Seas Evening Primrose Oil from £5.59 for a 60 pack to £2.79.

The cuts came after the Community Pharmacy Action Group, representing small retailers, withdrew its opposition to a High Court action brought by the Office of Fair Trading. The OFT had sought the abolition of resale price maintenance in the industry, exempted 30 years ago from general price fixing rules to try to ensure the survival of small pharmacies. There are 13,500 pharmacies in Britain, of which 9,000 are small shops serving local high streets and rural communities.

The action group backed out after Mr Justice Buckley said that he believed there was insufficient proof that a large number of independent pharmacies would close, or that the range of products would be reduced. But the group’s chairman, David Sharpe, said that the outcome would be a devastating blow. ‘Many pharmacists will simply not be able to survive given the buying power and aggressive pricing of the supermarkets’ he said. ‘It’s a sad day for Britain. The potential losers are the elderly, disabled and young mothers who rely on the free advice and range of services offered by the local pharmacist. We’ll fight on and hope the public will remain loyal.’

The changes will cover about 2,500 products sold without requiring a doctor’s prescription, and will have no effect on prescription drugs or on cosmetics sold by pharmacists.

Prices are likely to fall even lower as competition grows. In the United States, where prices are unregulated, comparable products are markedly cheaper.

Richard Hyman, chairman of the Verdict retail research consultancy, said: ‘This is a market made for supermarkets. Medicines are small, they fit on shelves and supermarkets are going to make a lot of noise about the great prices that they will be offering. Soon medicines will become like any other product and be part of the weekly shop.’

John Vickers, Director General of Fair Trading, said: ‘This is excellent news for consumers, who will now benefit from lower and more competitive prices for common household medicines. Consumers will save many millions of pounds a year.’

The Proprietary Association of Great Britain, which represents medicine and food supplement manufacturers, said it was disappointed.

Questions

1. What kind of market structure is involved for the sale of medicines and vitamins?

2. What can be said about barriers to entry in this market?

3. Might there be a change in market structure after the change in the law?

4. Explain the disadvantages of the abolition of resale price maintenance (RPM) for this market.

5. When RPM was abolished for book sales in 1995, the same concerns as those expressed in the above case were voiced. Since then, 10 per cent of bookshops have gone out of business. What conclusions might this help you to draw regarding the future of small pharmacies?

6. How does the rise of the Internet affect this situation?

describe the factors that are favourable to the formation of a successful cartel and 670303

Case study 8.4: Private school fees

Private schools in row over fee fixing16

Some of Britain’s top private schools stand accused of price fixing after meeting to plan steep increases in fees, which lawyers say could breach competition laws. Eton, Westminster and Marlborough are among the schools that appear to have colluded on the fees they charge. The Office of Fair Trading is now considering launching an investigation as parentsface record hikes in fees averaging 10%, four times the rate of inflation. In the past decade fees have risen by 56%.

Across the country, local and national groups of schools have ‘cartel style’ private meetings where they share sensitive financial information. The result is near identical increases in fees.

One bursar admitted last week that he had shared pricing information with other schools and compiled a dossier of his rivals’ future fees that would be presented to his governing body before finalising his own.

David Chaundler, bursar at Westminster school, said he acquired details of rivals’ fee proposals and costs from meetings of the Eton Group of 12 top private schools. At one meeting in February each bursar announced their school’s proposals for increasing fees.

‘We do compare school fees,’ Chaundler said. ‘If I went to my governors with a rise substantially above the others they might tell meto rethink.We do ensure we are pretty well in line.’

Competition lawyers believe the relationship could constitute a cartel. Jonathan Tatten, a partner at Denton Wilde Sapte, said the schools, which have charitable status and are non profit making, were not exempt from competition laws: ‘Showing confidential pricing information to competitors is a very serious breach of competition rules. You know where you can safely pitch your own fees and it’s a way of fixing the market.’

The maximum punishment if a cartel is found is a five year prison term, he added. In America, a pricefixing inquiry into Ivy League colleges ended without any principals going to jail but led to new rules banning discussion of fees with each other.

Westminster and the other schools say they still make independent decisions on the precise level of fees, and claim the prices are close because many schools have similar cost bases.

This year private schools face a financial crunch from higher salaries and pension payments for teachers, plus Gordon Brown’s rise in National Insurance contributions. Fearing a backlash from parents against big fee rises, this spring schools were particularly keen to present a united front. Top boarding schools are set to cross the £20,000 a year fees watershed for the first time.

On February 7 the Eton Group, including Westminster, Marlborough, King’s College school (London), Sherborne, Tonbridge and Bryanston, met at Dulwich College, south London. Each of the bursars outlined the fees they proposed to charge for the next year.

Andrew Wynn, of Eton, admitted: ‘We do meet and talk about fees to get some idea of what other schools are thinking. We are a co operative bunch, and we are not out to slit each other’s throats.’ Although their academic results vary, the group’s six provincial boarding schools are already closely aligned on fees of £6,300 to £6,445 a term. Its two major London boarding schools, Westminster and Dulwich, charge fees of more than £6,000 and are just £138 apart. Day school members Highgate and nearby University College school have charged exactly the same for the past two years.

A similar meeting held by a rival network, the Rugby Group, whose members include Winchester, Radley, Harrow, Clifton College and Shrewsbury, is also understood to have discussed plans for the first £20,000 annual fees.

William Organ, bursar of Winchester, said: ‘Sometimes schools feel they are too far ahead in fees and row back a bit, or the otherway round. They look at their competitors in the area and say: Gosh, we’re slipping behind in the fees league we’d better catch up.’

A network of six leading private day schools in Manchester, known as the Consortium, holds similar meetings. The schools including Manchester Grammar, William Hulme’s Grammar and Stockport Grammar, last year charged about £1,900 a term, with a difference of £131 between them.

Elizabeth Fritchley, William Hulme’s bursar, said the group met every term and phoned each other in March: ‘We decide what our increase is to be and then phone the other schools. If we are thinking of putting the fees up by, say, 15% and the rest were proposing far less, then it would make us rethink our strategy.’

Yesterday Mike Sant, general secretary of the Independent Schools’ Bursars Association, denied any cartels were operating: ‘Schools will decide where they want to be in the market and will be watching their competition and move fees accordingly. All the schools are so different they are just not incompetition. They do exchange information, but just to get a feel for what others are doing.’ Questions

1. If a group of schools simultaneously raises their fees by a similar amount, is this evidence of collusion? What other explanation might be possible?

2. If fees have risen much faster than the rate of inflation, 56 per cent over a decade, is this evidence of collusion, or are other explanations possible?

3. Describe the factors that are favourable to the formation of a successful cartel, and those that are unfavourable, for the elite private schools mentioned in the article. Use the statements in quotations as evidence.

what would happen to profit if the firm tried to pass on all the tax to the consumer 670305

1. An apartment block has seventy units of accommodation. It is estimated that it is possible to let them all if the rent is $2,000 per month, and for each $100 per month the rent is increased there would be one unit vacant. LG, the manager of the block, finds that a vacant unit costs $100 per month to maintain whereas an occupied one costs $300.

a. If profit from the lettings is measured as revenue minus maintenance costs, find an expression for profit in terms of the number of units let.

b. What rent should LG charge to maximize profit?

c. KA, a contractor, offers to be responsible for the maintenance of the entire block at a rate of $150 per unit, whether the units are occupied or not. Would it be more profitable for LG to employ KA? 

2. XL Corp has estimated its demand and cost functions to be as follows: 

P = 60 0:2Q

C = 200 + 4Q + 1.2Q 2 

where Q is in units, P is in $ and C is in $.

a. Calculate the profit maximizing price and output.

b. Calculate the size of the profit.

c. Calculate the price elasticity of demand at the above price.

d. If there is a $14 tax placed on the good, so that the producer has to pay the government $14 for every unit sold, calculate the new profitmaximizing price and output.

e. What would happen to profit if the firm tried to pass on all the tax to the consumer in the form of a higher price? f. If fixed costs rise by $200 how would this affect the firm’s situation?

the project has a negative npv and therefore it is not profitable 667177

1 – Short Answer

The responses to these questions should not require more than 100 150 words, and may be complete with far fewer words.

A. What should a change in the reported unit cost of a particular product or service imply about the economics of that product or service? Identify one (1) specific condition when a change in the reported unit cost of a particular product or service does not correspond to this economic implication.

B. INTENTIONALLY LEFT BLANK

C. Identify two
specific and distinct conditions when traditional full absorption cost allocation can be the best method for determining reported unit cost for a particular organization. These conditions should be about the organization, and not about the costs of changing the reporting system.

D. Explain why the following statement is false:

“The project has a negative NPV and therefore it is not profitable.”

2 – Cost Systems

Each of the companies described below currently uses a traditional single cost driver cost system. Indicate whether the benefits of a multiple cost driver system could be greater than the costs to each company by (1) entering either YES or NO and (2) providing a one sentence (or at most two sentence) explanation why.

A. Company A produces three furniture products, tables, desks, and credenzas, in a single production facility. Activities in the facility include cutting, sanding, assembling, and finishing. Each product requires approximately the same amount of time from the resources available for each activity. The market for these furniture products is very competitive.

B. Company B produces three highly specialized machines that other companies use to improve productivity. Company B has only one production facility. Each of the three products requires unique attention at various stages of the production process. Each product also faces little competition because of the many engineering patents Company B possesses.

C. Company C produces three generic pharmaceutical products in a single production facility. The products take turns in the various pieces of equipment. Activities in the production process include heating, cooling, stirring/mixing, testing, separating, and cleaning. One product is very difficult to clean out of the equipment, another product requires extensive testing before it can be approved, and all products require different amounts of heating, cooling, mixing, and separating. The market for these generic pharmaceutical products is very competitive.

D. Company D produces plain 12 ounce aluminum cans that it sells to various beverage producers, who then place their own logos and pictures on the cans and fill the cans with their respective beverages.

3 Micro Devices Division

Assume that the data in case Exhibit 8
(see first tab of Excel doc) are MDD’s budget for the coming year, and that all other information in the case and the preparation document holds unless stated explicitly otherwise in this problem.

The table below contains information about Product SCM DUN. These data are also in the “MDD Data” tab of the Exam Data spreadsheet.

A. 10 Points

Prepare an estimate of the reported unit cost of Product SCM DUN if MDD were to make Product SCM DUN in its production facility. Describe the steps you take and justify any choices you make.

You may use (citing appropriately) values from the in class analysis of the case, and from the follow up assignment analysis, without providing the underlying calculations.

B. 20 Points

The Handheld Products division of Chips ‘R’ Us (MDD’s parent company) currently purchases 500,000 units (die) of Product SCM DUN from Integrated Corporation, a third party producer of high volume chips. Handheld Products pays Integrated Corporation $33.00 per unit of Product SCM DUN.

Assume (1) a marginal corporate tax rate of 25%, (2) a five year window before MDD’s new product pipeline leads to a production plan with overall utilization at 80% without Product SCM DUN, and (3) a 7% cost of corporate capital.

Prepare a one page memo to the MDD Executive Committee regarding whether MDD should insource Product SCM DUN for the next several years.

NOTE: Assume that MDD can handle this volume of Product SCM DUN without approaching 80% utilization.

** SEE ANSWER FORMAT ON PAGE 3

3 – Micro Devices[30 Points]
A. 10 points

Reported Unit Cost of Product SCM DUN

Explanation and calculation(s):

3 – Micro Devices
B. 20 points

Date: Today

To: Micro Devices Division Executive Committee

From: Re: Insourcing Product SCM DUN
Executive Summary

Supporting Analysis

4 – MS/SCM Pictures

Assume you are the CEO of a MS/SCM Pictures, a small, publicly traded movie production company. While most of your studio’s projects are set for the coming year, there is enough money left in the budget for one additional film. Of the many scripts and ideas that writers and producers have pitched to you, you believe only two have any hope of generating a return for your shareholders. There would be no need to acquire additional equipment, facilities, or other committed resources for either film.

Finally, assume that tax rules permit straight line amortization1 of production costs over the estimated revenue producing life of the film (
all revenues, not just lease revenues).

a.
Death of a Professor is a biographical film about a renowned teacher of cost and managerial accounting. The script is well written and you expect the film to appeal to a more intellectual audience. While you would have to invest in some up front promotional materials, this type of film, when it succeeds, normally does so by word of mouth. Studios typically view this type of film as low risk, because their success does not depend on what else is released the same weekend or on when during the year the release occurs. The expected cash flows for
Death of a Professor ($ millions) are in Table A.

b.
Professor Stuart and the Lost Cost Allocation is an action adventure movie about a renowned teacher of cost accounting who happens to find himself, with alarming frequency, engaged in pitched battles with corporate raiders and venture capitalists. Arnold Schwarzenegger has expressed interest in playing the title role if the film is made. This type of film typically requires greater promotional outlays and production costs than average, but the chance for a blockbuster showing at the box office exists. Additionally, if the film does well it is quite likely that a sequel would also do well. Only 2% of the films that MS/SCM Picture has produced have these characteristics, and the company has yet to make a sequel. The expected cash flows for
Professor Stuart and the Lost Cost Allocation ($ millions) are in Table B.

If you choose to make a sequel, it would start production in year 3 and launch in year 4. Your industry sources suggest that the volatility (variation) of returns to such sequels is 120%. Action adventure sequels have cost, on average, $120 million to produce and promote in the year before release. Your best estimate of the present value of net cash flows
from the sequel in years 4 through 9 (sequel years 1 through 6), as of the end of Year 3, is $125 million.

A. 20 Points

Calculate the net present value of each movie (not including the sequel).

B. 5 Points

Using solely financial criteria, which movie should MS/SCM Pictures produce? “Neither” is not an acceptable response.

C. 5 Points

Explain why the NPV of
Professor Stuart and the Lost Cost Allocation does not capture the total value of the project.

D. 5 Points

Under what condition(s) would you recommend
Professor Stuart and the Lost Cost Allocation instead of
Death of a Professor?

1 “Amortization” is exactly like depreciation, only “amortization” is the term used when the asset involved is not a tangible facility or piece of equipment

Attachments:

surgical products produces latex surgical gloves machines perform the majority of th 667248

Surgical Products produces latex surgical gloves. Machines perform the majority of the processing for 1,000 pairs of gloves per hour. Each pair of gloves requires 0.85 square foot of latex, which has a standard price of $0.80 per square foot. Machine operators are considered direct labor and are paid $15 per hour.

During one week in May, Surgical Products produced 300,000 pairs of gloves and experienced a $1,440 unfavorable material quantity variance. The company had purchased 2,500 more square feet of material than had been used in production that week. The unfavorable material price variance for the week was $5,186. A $288 unfavorable total labor variance was generated based on 315 total actual labor hours to produce the gloves. Determine the following amounts:

a. Standard quantity of material for production achieved

b. Actual quantity of material used

c. Actual quantity of material purchased

d. Actual price of material purchased

e. Standard hours for actual production

f. Labor efficiency variance

g. Labor rate variance

h. Actual labor rate

choose a global business form the following 667249

Choose a global business form the following :

  • Milk industry (in Australia and New Zealand)
  • Car industry (especially Peugeot in China)
  • On line retailing especially Alibaba vs Amazon
  • Container shipping – Maersk)
  • Woolworths and Coles operating overseas markets
  • Organic food company trying to expand in international market
  • Health and fitness company
  • Any small to medium size company in Australia which is trying to expand

Note: You need to discuss your choice with your lecturer before you start your assignment.

  • Discussion on theory maximum 20%
  • This assignment is in report form with the following headings:
  • Executive summary
  • Apply “AAA” framework which includes Adaptation, Aggregation and Arbitrage
  • Porters 3 generic strategies (cost leadership, differentiation, focus and Hybrid)
  • Diversification and Integration
  • Competitive advantage
  • Absolute advantage
  • Your comments or any recommendations
  • References Harvard style
Document Preview:

HI6006 Competitive Strategy Tri1 2014 Group Assignment Guidelines Written document: A short report Marks: 20 Report Due: Week 11 Words limit: 2500 Choose a global business form the following : Milk industry (in Australia and New Zealand) Car industry (especially Peugeot in China) On line retailing especially Alibaba vs Amazon Container shipping – Maersk) Woolworths and Coles operating overseas markets Organic food company trying to expand in international market Health and fitness company Any small to medium size company in Australia which is trying to expand Note: You need to discuss your choice with your lecturer before you start your assignment. Discussion on theory maximum 20% This assignment is in report form with the following headings: Executive summary Apply “AAA” framework which includes Adaptation, Aggregation and Arbitrage Porters 3 generic strategies (cost leadership, differentiation, focus and Hybrid) Diversification and Integration Competitive advantage Absolute advantage Your comments or any recommendations References Harvard style Every member needs to bring at least 2 articles which the group would use and add that articles with your report with reference page. There is no need to submit the articles on BB. Students need to submit a soft copy on BB and hard copy on Lvl 6 by due date. To do well groups need to do a quality research and apply the above concepts from the subject.

Attachments:

a computer manufacturer produces three types of devices mobile phones tablets and co 667279

A computer manufacturer produces three types of devices:mobile phones, tablets, and computers. For the production of these three devices you have the following information:

Phone

Tablet

Computer

Material cost per unit

£90

£140

£315

Direct labor hours per unit

2

2.5

4

Budgeted units

1,500,000

900,000

1,200,000

Labor cost per hour

£8

Overhead costs per annum

Utilities

£20,000,000

Rent

£15,000,000

Audit and legal

£5,000,000

Administrative staff

£40,000,000

Total

£80,000,000

ABC analysis suggested that overhead costs are distributed to the three products according to the table below:

Overheads

Phone

Tablet

Computer

Utilities

£8,000,000

£5,000,000

£7,000,000

Rent

£8,250,000

£2,250,000

£4,500,000

Audit and legal

£2,900,000

£1,250,000

£850,000

Administrative staff

£23,200,000

£6,000,000

£10,800,000

For each of the three products, the company aims at a different percentage for profit. Under the full absorption costing method and the targeted profit percentage, the prices of the three products should be:

Phone

Tablet

Computer

Full costing price

£170.69

£233.87

£435.67

  1. Calculate the aimed profit percentages for the three products and under the full absorption costing method, with overhead costs absorbed on the basis of direct labour hours.
  2. Use the profit percentages that you derived in (1) and calculate the prices of the three products under the ABC system.
  3. Recommend a cost system and include any changes that you would suggest to the pricing strategy of the computer company

exercise 9 8 here are selected 2014 transactions of cleland corporation jan 1 retire 667292

Exercise 9 8

Here are selected 2014 transactions of Cleland Corporation.

Jan. 1 Retired a piece of machinery that was purchased on January 1, 2004. The machine cost $61,270 and had a useful life of 10 years with no salvage value.
June 30 Sold a computer that was purchased on January 1, 2012. The computer cost $35,200 and had a useful life of 4 years with no salvage value. The computer was sold for $5,100 cash.
Dec. 31 Sold a delivery truck for $9,190 cash. The truck cost $24,540 when it was purchased on January 1, 2011, and was depreciated based on a 5 year useful life with a $3,330 salvage value.

Journalize all entries required on the above dates, including entries to update depreciation on assets disposed of, where applicable. Cleland Corporation uses straight line depreciation.
(Record entries in the order displayed in the problem statement. Credit account titles are automatically indented when amount is entered. Do not indent manually.)

i need this 667305

3 Micro Devices Division

Assume that the data in case Exhibit 8
(see first tab of Excel doc)are MDD’s budget for the coming year, and that all other information in the case and the preparation document holds unless stated explicitly otherwise in this problem.

The table below contains information about Product SCM DUN. These data are also in the “MDD Data” tab of the Exam Data spreadsheet.

A. 10 Points

Prepare an estimate of the reported unit cost of Product SCM DUN if MDD were to make Product SCM DUN in its production facility. Describe the steps you take and justify any choices you make.

You may use (citing appropriately) values from the in class analysis of the case, and from the follow up assignment analysis, without providing the underlying calculations.

B. 20 Points

The Handheld Products division of Chips ‘R’ Us (MDD’s parent company) currently purchases 500,000 units (die) of Product SCM DUN from Integrated Corporation, a third party producer of high volume chips. Handheld Products pays Integrated Corporation $33.00 per unit of Product SCM DUN.

Assume (1) a marginal corporate tax rate of 25%, (2) a five year window before MDD’s new product pipeline leads to a production plan with overall utilization at 80% without Product SCM DUN, and (3) a 7% cost of corporate capital.

Prepare a one page memo to the MDD Executive Committee regarding whether MDD should insource Product SCM DUN for the next several years.

NOTE: Assume that MDD can handle this volume of Product SCM DUN without approaching 80% utilization.

** SEE ANSWER FORMAT ON PAGE 3

3 – Micro Devices[30 Points]
A. 10points

Reported Unit Cost of Product SCM DUN

Explanation and calculation(s):

3 – Micro Devices
B. 20 points

Date: Today

To: Micro Devices Division Executive Committee

From: Re: Insourcing Product SCM DUN
Executive Summary

Supporting Analysis

4 – MS/SCM Pictures

Assume you are the CEO of a MS/SCM Pictures, a small, publicly traded movie production company. While most of your studio’s projects are set for the coming year, there is enough money left in the budget for one additional film. Of the many scripts and ideas that writers and producers have pitched to you, you believe only two have any hope of generating a return for your shareholders. There would be no need to acquire additional equipment, facilities, or other committed resources for either film.

Finally, assume that tax rules permit straight line amortization1 of production costs over the estimated revenue producing life of the film (
all revenues, not just lease revenues).

a.
Death of a Professor is a biographical film about a renowned teacher of cost and managerial accounting. The script is well written and you expect the film to appeal to a more intellectual audience. While you would have to invest in some up front promotional materials, this type of film, when it succeeds, normally does so by word of mouth. Studios typically view this type of film as low risk, because their success does not depend on what else is released the same weekend or on when during the year the release occurs. The expected cash flows for
Death of a Professor ($ millions) are in Table A.

b.
Professor Stuart and the Lost Cost Allocation is an action adventure movie about a renowned teacher of cost accounting who happens to find himself, with alarming frequency, engaged in pitched battles with corporate raiders and venture capitalists. Arnold Schwarzenegger has expressed interest in playing the title role if the film is made. This type of film typically requires greater promotional outlays and production costs than average, but the chance for a blockbuster showing at the box office exists. Additionally, if the film does well it is quite likely that a sequel would also do well. Only 2% of the films that MS/SCM Picture has produced have these characteristics, and the company has yet to make a sequel. The expected cash flows for
Professor Stuart and the Lost Cost Allocation ($ millions) are in Table B.

If you choose to make a sequel, it would start production in year 3 and launch in year 4. Your industry sources suggest that the volatility (variation) of returns to such sequels is 120%. Action adventure sequels have cost, on average, $120 million to produce and promote in the year before release. Your best estimate of the present value of net cash flows
from the sequel in years 4 through 9 (sequel years 1 through 6), as of the end of Year 3, is $125 million.

A. 20 Points

Calculate the net present value of each movie (not including the sequel).

B. 5 Points

Using solely financial criteria, which movie should MS/SCM Pictures produce? “Neither” is not an acceptable response.

C. 5 Points

Explain why the NPV of
Professor Stuart and the Lost Cost Allocation does not capture the total value of the project.

D. 5 Points

Under what condition(s) would you recommend
Professor Stuart and the Lost Cost Allocation instead of
Death of a Professor?

1 “Amortization” is exactly like depreciation, only “amortization” is the term used when the asset involved is not a tangible facility or piece of equipment

Attachments:

data for mariner designs inc follow mariner designs inc comparative income statement 667307

Data for Mariner Designs, Inc., follow:
MARINER DESIGNS, INC.
Comparative Income Statement
Years Ended December 31, 2012 and 2011
2012 2011
Net sales revenue: $431,000 $372,350
Expenses:
Cost of goods sold $200,000 $187,550
Selling and general expenses 99,000 91,050
Other expense 8,350 6,850
Total expenses $307,350 $285,450
Net income $123,650 $86,900
Requirements:
1. Prepare a horizontal analysis of the comparative income statement of Mariner Designs, Inc. Round percentage changes to one decimal place.
2. Why did 2012 net income increase by a higher percentage than net sales revenue?

identify some companies who have been successful in following bit of flexibility in 667311

  • Students need to research and identify 2 to 3 industries and 2 to 3 companies who have been successful in creating a competitive advantage over their competition for longer period may be 5 to 10 years or more but finding difficult to maintain that.
  • Also identify some companies who have been successful in following bit of flexibility in their approach
  • You need to discuss both sides of the arguments whether there is such thing as sustainable competitive advantage or companies need to be flexible in their business strategies due to changing business environment.
  • There will always be some companies who have been changing part of their business model and still keeping something which has been helping them.
  • You can see the Article on line on “Competitive Advantage or flexibility” by Rita Gunther McGrath and also see how the shortening of the strategy life cycle and product life cycle are contributing towards this change.
  • Following concepts can be useful to analyse this research:
  • Porters 3 generic strategies
  • “AAA” framework to show where these changes are being adopted to become more flexible and/or sustainable competitive advantage
  • Every member needs to submit 2 articles each with their assignment (hard copy)
  • Assessment criteria has been given in the subject outline.
  • References Harvard style
Document Preview:

HI6006 Competitive Strategy Tri2 2014 Guidelines for Group Assignment 1 Written document: A short report Marks: 20 Report Due: Week 6 Words limit: 1200 Students need to research and identify 2 to 3 industries and 2 to 3 companies who have been successful in creating a competitive advantage over their competition for longer period may be 5 to 10 years or more but finding difficult to maintain that. Also identify some companies who have been successful in following bit of flexibility in their approach You need to discuss both sides of the arguments whether there is such thing as sustainable competitive advantage or companies need to be flexible in their business strategies due to changing business environment. There will always be some companies who have been changing part of their business model and still keeping something which has been helping them. You can see the Article on line on “Competitive Advantage or flexibility” by Rita Gunther McGrath and also see how the shortening of the strategy life cycle and product life cycle are contributing towards this change. Following concepts can be useful to analyse this research: Porters 3 generic strategies “AAA” framework to show where these changes are being adopted to become more flexible and/or sustainable competitive advantage Every member needs to submit 2 articles each with their assignment (hard copy) Assessment criteria has been given in the subject outline. References Harvard style

Attachments:

assume that you are about to graduate from your university and are deciding whether 667325

Assume that you are about to graduate from your university and are deciding whether to apply for graduate school or enter the job market.

To help make the decision, you have gathered the following data:

Costs incurred for the bachelor’s degree ……………..$163,000

Out of pocket costs for a master’s degree …………… $92,000

Estimated starting salary with B.A. ………………….. $48,400

Estimated starting salary with MA …………………… $66,800

Estimated time to complete master’s degree 2 years

Estimated time from the present to retirement 40 years

a. Which of these factors is relevant to your decision?

b. What is the opportunity cost associated with earning the master’s degree?

c. What is the out of pocket cost to obtain the master’s degree?

d. What other factors should you consider before making a decision?

individual assignment developing a balanced scorecard bsc in this unit you are explo 667376

Individual Assignment: Developing a Balanced Scorecard ( BSC )

In this unit, you are exploring the need for organisations to measure and manage performance against objectives, as well as the potential effectiveness of tools such as Balanced Scorecards and Strategy Maps as aids in this cause.
You will now develop a BSC for Anthony’s Orchard. The company has a number of strategic goals; measuring performance towards those goals will be critical to its sustained success.

To prepare for this Individual Assignment:

Visit the Anthony’s Orchard case study in the unit resources. Review again the current and historical financials. Consider that one of the company’s key goals in its strategic plan is to exceed revenue of $25 million dollars by the year 2015.

To complete this Individual Assignment:

§ Explain the potential value of a BSC to Anthony’s Orchard. Describe specific ways that the introduction of a BSC can contribute to this organisation.

§ Develop a BSC that is aligned to the key goal in the strategic plan, i.e. exceeding revenue of $25 million dollars by 2015. Develop, quantify and justify suitable key performance measurement criteria for Anthony’s Orchard in each of these four key areas:

§ Financial

§ Customer

§ Internal Business Processes

§ Learning and Growth

Notes:

You should fully state and justify any assumptions that you make in relation to the financial measurements you use. You should also make suitable use of appendices to include relevant financial performance measurement information you include. Be sure to include all references as well.

This Individual Assignment forms the basis of Section 3 of your Final Project. You will receive feedback on this work from your Faculty Member, and will be expected to incorporate any suggestions into the Final Project.
Your submission (excluding appendices) should be 1,000 words (+/ 10%) in length.

the last few years have been difficult economically but the owners of johnsons p l 667379

The last few years have been difficult economically but the owners of Johnsons P/L, a medium sized manufacturer of quality dining furniture is keen to grow the business. They have seen an increase in demand for their products from overseas and feel that they will need to increase their operation in order to continue to meet this demand. They are currently looking at a number of options to finance this expansion such as through debt and through equity raising (meaning they will need to “go public”). They have determined that they need to raise $60 million. Giving consideration to the various options, you have been requested to advise the owners of Johnsons what the various options are, outlining the positives and negatives of each. Required: Write a report (should be extensive) to the owners detailing ALL the different options and considerations that you feel the owners should consider raising the $60 million. Part B (600 words) (6 marks) Choose a Manufacturing Company listed in the Australian Stock Market (ASX 200). Discuss the following: 1. Nature of Business 2. If there are, mention and discuss two (2) subsidiaries of the chosen company. 3. Compare the latest Net Profit After Tax (NPAT) for two years and make some comments whether to invest or not. Explain why yes or not. 4. What kinds of shares are being issued? 5. Who is the external auditor? What is the role of the external auditor? HA2032 Corporate Accounting Assignment T2 14 Page 2 of 2 Part C (8 marks) Regardless of the advice you have given (Part A), the owners have decided to go “public” and issue an ‘IPO” They issue 30 million shares ($2.00), of which the payment on application is to $0.80 per share (closes 18th April 2013), $0.50 four weeks after allocation (allocation is 12th May 2013) and the remaining amount to be paid on 30th June 2013. The IPO attracts requests for 30.4 million shares. In this case, it exceeds the allowable number of shares and the directors decide to apply the “first come, first served” approach and return the excess back to the unlucky applicants Required: Journalise the events (including dates and notations). Assume that all monies were received on 18th April (applications). Use the proper journal for your answers.

Attachments:

problem 1 32 classifying events as asset source use or exchange lo 4 the following u 667554

Problem 1 32 Classifying events as asset source, use, or exchange [LO 4]

The following unrelated events are typical of those experienced by business entities.

1. Acquire cash by issuing common stock.
2. Purchase land with cash.
3. Purchase equipment with cash.
4. Pay monthly rent on an office building.
5. Hire a new office manager.
6. Borrow cash from a bank.
7. Pay a cash dividend to stockholders.
8. Pay cash for operating expenses.
9. Pay an office manager’s salary with cash.
10. Receive cash for services that have been performed.
11. Pay cash for utilities expense.
12. Acquire land by accepting a liability (financing the purchase).
13. Pay cash to purchase a new office building.
14. Discuss plans for a new office building with an architect.
15. Repay part of a bank loan.

Required:

Identify each of the events as an asset source, asset use, or asset exchange transaction. If an event would not be recorded under generally accepted accounting principles, identify it as not applicable (NA). Also indicate for each event whether total assets would increase, decrease, or remain unchanged. The first event is shown in the table as an example.

you are a member of a large accounting firm which is responsible for preparing finan 667575

You are a member of a large accounting firm which is responsible for preparing financial reports, including statements and notes to the accounts; and for advising staff in client firms who are responsible for preparing financial reports. The firm only deals with large Australian companies listed on the Australian Stock Exchange. One of your key tasks is to monitor the changes and developments in the financial reporting environment and summarise them in a regularly published newsletter.

Required:

Prepare a 2 page newsletter that identifies and summarises developments and changes in the financial reporting environment for the period 24 July 2014 22 August 2014, inclusive.

Document Preview:

Assessment item 2 Assignment 1 Value: 15% Due date: 29 Aug 2014 Return date: 19 Sep 2014 Submission method options Hand delivery (option applies to Internal only)?Alternative submission method Task This assessment task consists of two (2) questions: a newsletter, and a question on financial statement presentation.  A total of 50 marks are allocated to the questions below, which will then be converted to a mark out of 15%. Rationale This assessment task is designed to assess your understanding of topics 1 and 2 of the subject. Marking criteria A detailed marking rubric has been provided in the ‘Requirements’ section below for each question.   Presentation Physical presentation of assignments It is essential that presentation of assignments adheres to accepted standards in relation to neatness and layout, as you are practising to present material in a work situation. Correct formatting and referencing procedures of material should be strictly adhered to for essays. You should submit a proper reference list (using APA referencing style) for all essay type assignments. A reference list contains only those works cited or quoted from in your essay. A bibliography is acceptable for practical type assignments.   For practical questions: all journal entries must include narrations unless otherwise specified; any ledger accounts should preferably be shown in ‘T’ account format and dates and descriptions be included; journal entries and ledger accounts must reflect the strict order of sequence of events; financial statements (including extracts) should include proper headings and accord with presentation standards. Penalties will be incurred if material is not correctly referenced and if presentation is not of an acceptable standard.   Please also note the following: Journal entries, ledger accounts, worksheets and financial statements should always balance. If you have to submit a piece of work that does not balance because you cannot detect your error please include some…

Attachments:

evaluate accounting information system architecture 667578

hi.i m giving u the assignment of accounting informaton system.its a question of making a presentation and about ERP..actually my friend also give u the same assignment which u guys made it.so do not put that data into this question.make it totally differently and give me quality work.read all the instruction in this file and make it in that way.do not copy from internet or from my freind assignment not a single word.make it in ur own words. make it professionally.

Assignment # 1

Value: 15%

Task

The assessment is based on two blog entries, Blog 1 and Blog 4 which will be posted on the Resouce section under Assignment.

Assignment Question 1 (7.5%)

Create a PowerPoint Presentation to provide a succinct explanation of how the implementation of cloud computing relates to an organisations strategy and how it could affect a company’s value chain. Use a recent announcement by a company to illustrate your explanation. Your presentation should be a maximum of 10 slides. Please include any accompanying notes (in the note section of your presentation) you would use for a 10 minute presentation. Alternatively you can use Voice over PowerPoint to provide a 10 minute presentation. Ensure you refer to the marking criteria specified in the subject outline.

Assignment Question 2 (7.5%)

Your manager has requested a report explaining how ERP can value add. Include in your report any potential risks that should be avoided when implementing a cloud based ERP solution in an organisation. Use the blog post above as a starting point . Your report, including executive summary should be maximum length 1200 words.

Question 1 should be pasted in word and then uploaded onto Turnitin.

Rationale

This assignment is designed to enable you to demonstrate your ability to:

• understand and apply foundation principles relating to computerised information systems in contemporary organisations

• explain the role of data analysis tools and data mining;

• to illustrate typical network configurations and identify the components of a network, particularly identifying the role that cloud computing has in a network;

• evaluate accounting information system architecture, particularly identifying the role that cloud computing and ERP have as the basis for an accounting information system;

• identify organisational risk and the impact that cloud computing and ERP systems have on that risk

Attachments:

ha2032 corporate accounting assignment t2 14 667647

HA2032 CORPORATE ACCOUNTING ASSIGNMENT T2 14

This is an individual assignment. It is required to be submitted in both soft and hard copy by the Friday of Week 6. Total marks applied to this assessment are 20%.

Please ensure that you attach an assignment submission sheet to your hard copy only. Late submissions draw a penalty of 5% per day (this includes weekends) of the value of the assessment (1 mark in this case) up to a maximum of fourteen (14) days. After that date, your assessment may not be accepted unless prior and special consideration has been granted.

This is NOT a report but it is expected that your submission will be in an appropriate format. There is a word limit applied but you should ensure that each question is appropriately answered. Where references are used, ensure they are recognised (refer to student handbook or your lecturer if unsure).

Important instruction(s):

a) Do not submit Part C to the Safeassign System. b) Safeassign report matching percentage should not exceed 20%. c) Include the Safeassign report in your submission.

Part A (600 words) (6 marks)

The last few years have been difficult economically but the owners of Johnsons P/L, a medium sized manufacturer of quality dining furniture is keen to grow the business. They have seen an increase in demand for their products from overseas and feel that they will need to increase their operation in order to continue to meet this demand. They are currently looking at a number of options to finance this expansion such as through debt and through equity raising (meaning they will need to “go public”). They have determined that they need to raise $60 million.

Giving consideration to the various options, you have been requested to advise the owners of Johnsons what the various options are, outlining the positives and negatives of each.

Required: Write a report (should be extensive) to the owners detailing ALL the different options and considerations that you feel the owners should consider raising the $60 million.

Part B (600 words) (6 marks)

Choose a Manufacturing Company listed in the Australian Stock Market (ASX 200). Discuss the following:

1. Nature of Business 2. If there are, mention and discuss two (2) subsidiaries of the chosen company. 3. Compare the latest Net Profit After Tax (NPAT) for two years and make some comments whether to invest or not. Explain why yes or not. 4. What kinds of shares are being issued? 5. Who is the external auditor? What is the role of the external auditor?

HA2032 Corporate Accounting Assignment T2 14 Page 1 of 2

Attachments:

model x100 sells for 120 per unit whereas model x200 offers advanced features and se 667667

Model X100 sells for $120 per unit whereas Model X200 offers advanced features and sells for $500 per unit. Management expects to sell 50,000 units of Model X100 and 5,000 units of Model X200 next year. The direct material cost per unit is $50 for Model X100 and $220 for Model X200. The company’s total manufacturing overhead for the year is expected to be $1,995,000. A unit of Model X100 requires 2 direct labor hours and a unit of Model X200 requires 5 direct labor hours. The direct labor wage rate is $20 per hour.

Required:

i. The company currently applies manufacturing overhead to products using direct labor hours as the allocation base. Using this traditional approach, compute the product margins for X100 and X200.

ii. Management is considering an activity based costing system and would like to know what impact this would have on product costs. Preliminary analysis suggests that under activity based costing, a total of $1,000,000 in manufacturing overhead cost would be assigned to Model X100 and a total of $600,000 would be assigned to Model X200. In addition, a total of $150,000 in nonmanufacturing overhead would be applied to Model X100 and a total of $350,000 would be applied to Model X200. Using the activity based costing approach, compute the product margins for X100 and X200.

iii. Explain why the product margins computed in requirement (1) differ from those computed in requirement (2).

a hamburgers and more inc sells hamburgers drinks and fries the sales mix is 1 3 2 i 667670

a) Hamburgers and More, Inc., sells hamburgers, drinks, and fries. The sales mix is 1:3:2 (i.e., for every one hamburger sold, three drinks and two fries are sold). Using the contribution margin approach, find the breakeven point in units for each product. The company’s fixed costs are $2,040. Other information is as follows:

Selling Price

per Unit

Variable Costs

per Unit

Hamburgers

$0.99

$0.27

Drinks

0.99

0.09

Fries

0.99

0.15

lavage rapide is a canadian company that owns and operates a large automatic carwash 667672

Lavage Rapide is a Canadian company that owns and operates a large automatic carwash facility near Montreal. The following table provides data concerning the company’s costs:

Fixed Cost

per Month

Cost per

Car Washed

Cleaning supplies

$0.80

Electricity

$1,200

$0.15

Maintenance

$0.20

Wages and salaries

$5,000

$0.30

Depreciation

$6,000

Rent

$8,000

Administrative expenses

$4,000

$0.10

For example, electricity costs are $1,200 per month plus $0.15 per car washed. The company expects to wash 9,000 cars in August and to collect an average of $4.90 per car washed.

Required: Prepare the company’s planning budget for August.

indicate whether each of the following variances is favorable or unfavorable the fir 666942

Classifying variances as favorable or unfavorable

Required

Indicate whether each of the following variances is favorable or unfavorable. The first one has been done as an example.

Item to Classify

Standard

Actual

Type of Variance

Materials cost

$2.90 per pound

$3.00 per pound

Unfavorable

Materials usage

91,000 pounds

90,000 pounds

Labor cost

$10.00 per hour

$9.60 per hour

Labor usage

61,000 hours

61,800 hours

Fixed cost spending

$400,000

$390,000

Fixed cost per unit (volume)

$3.20 per unit

$3.16 per unit

Sales volume

40,000 units

42,000 units

Sales price

$3.60 per unit

$3.63 per unit

marlow manufacturing company established the following standard price and cost data 666944

Preparing master and flexible budgets

Marlow Manufacturing Company established the following standard price and cost data.

Sales price

$7.50 per unit

Variable manufacturing cost

3.00 per unit

Fixed manufacturing costs

3,000 total

Fixed selling and administrative costs

1,000 total

Marlow planned to produce and sell 1,100 units. Actual production and sales amounted to 1,300 units.

Required

a. Prepare the pro forma income statement in contribution format that would appear in a master budget.

b. Prepare the pro forma income statement in contribution format that would appear in a flexible budget.

comment on the usefulness of the variances with respect to performance evaluation an 666945

Determining sales and variable cost volume variances

Required

Use the information provided in Exercise 15 3.

a. Determine the sales and variable cost volume variances.

b. Classify the variances as favorable (F) or unfavorable (U).

c. Comment on the usefulness of the variances with respect to performance evaluation and identify the member of the management team most likely to be responsible for these variances.

d. Determine the amount of fixed cost that will appear in the flexible budget.

e. Determine the fixed cost per unit based on planned activity and the fixed cost per unit based on actual activity. Assuming Marlow uses information in the master budget to price the company’s product, comment on how the volume variance could affect the company’s profitability.

prepare in good form a budgeted and actual income statement for internal use separat 666950

Income statement for internal use

Harris Company has provided the following 2008 data.

Budget

Sales

$400,000

Variable product costs

155,000

Variable selling expenses

45,000

Other variable expenses

3,600

Fixed product costs

16,600

Fixed selling expenses

24,300

Other fixed expenses

2,200

Interest expense

800

Variances

Sales

8,600 U

Variable product costs

4,000 F

Variable selling expenses

2,500 U

Other variable expenses

1,200 U

Fixed product costs

220 F

Fixed selling expenses

390 F

Other fixed expenses

150 U

Interest expense

80 F

Required

a. Prepare in good form a budgeted and actual income statement for internal use. Separate operating income from net income in the statements.

b. Calculate variances and identify these as favorable (F) or unfavorable (U).

darden medical equipment company makes a blood pressure measuring kit dan bushaw is 666951

Evaluating a cost center including flexible budgeting concepts

Darden Medical Equipment Company makes a blood pressure measuring kit. Dan Bushaw is the production manager. The production department’s static budget and actual results for 2009 follow.

Static Budget

Actual Results

20,000 kits

21,000 kits

Direct materials

$154,000

$166,000

Direct labor

136,000

139,800

Variable manufacturing overhead

37,000

46,600

Total variable costs

327,000

352,400

Fixed manufacturing overhead

167,000

164,000

Total manufacturing cost

$494,000

$516,400

Required

a. Convert the static budget into a flexible budget.

b. Use the flexible budget to evaluate Mr. Bushaw’s performance.

c. Explain why Mr. Bushaw’s performance evaluation does not include sales revenue and net income.

using the concept of controllability advise ms kysar about the best performance meas 666952

Evaluating a profit center

Sharon Kysar, the president of Shortt Toys Corporation, is trying to determine this year’s pay raises for the store managers. Shortt Toys has seven stores in the southwestern United States. Corporate headquarters purchases all toys from different manufacturers globally and distributes them to individual stores. Additionally, headquarters makes decisions regarding location and size of stores. These practices allow Shortt Toys to receive volume discounts from vendors and to implement coherent marketing strategies. Within a set of general guidelines, store managers have the flexibility to adjust product prices and hire local employees. Ms. Kysar is considering three possible performance measures for evaluating the individual stores: cost of goods sold, return on sales (net income divided by sales), and return on investment.

Required

a. Using the concept of controllability, advise Ms. Kysar about the best performance measure.

b. Explain how a balanced scorecard can be used to help Ms. Kysar.

holligan publications established the following standard price and costs for a hardc 666957

Determining sales and variable cost volume variances

Holligan Publications established the following standard price and costs for a hardcover picture book that the company produces.

Standard price and variable costs:

Sales price

$37.00

Materials

8.70

Labor

4.30

Overhead

6.10

General, selling, and administrative Planned fixed costs:

6.50

Manufacturing

$128,000

General, selling, and administrative

49,000

Holligan planned to make and sell 30,000 copies of the book.

Required

a. Prepare the pro forma income statement that would appear in the master budget.

b. Prepare flexible budget income statements, assuming volumes of 29,000 and 31,000 units.

c. Determine the sales and variable cost volume variances, assuming volume is actually 31,000 units.

d. Indicate whether the variances are favorable (F) or unfavorable (U).

e. Comment on how Holligan could use the variances to evaluate performance.

use the standard price and cost data supplied in problem 15 18 assume that holligan 666958

Determining and interpreting flexible budget variances

Use the standard price and cost data supplied in Problem 15 18. Assume that Holligan actually produced and sold 31,000 books. The actual sales price and costs incurred follow.

Actual price and variable costs:

Sales price

$36.00

Materials

9.10

Labor

4.10

Overhead

6.20

General, selling, and administrative Actual fixed costs:

6.10

Manufacturing

$120,000

General, selling, and administrative

55,000

Required

a. Determine the flexible budget variances. Provide another name for the fixed cost flexible budget variance.

b. Indicate whether each variance is favorable (F) or unfavorable (U).

c. Identify the management position responsible for each variance. Explain what could have caused the variance.

the same consultant also suggests that if the firm raises its rate to 88 per hour th 666959

Flexible budget planning

Sam Yu, the president of Centech Computer Services, needs your help. He wonders about the potential effects on the firm’s net income if he changes the service rate that the firm charges its customers. The following basic data pertain to fiscal year 2009.

Standard rate and variable costs:

Service rate per hour

$83.00

Labor

40.00

Overhead

7.10

General, selling, and administrative

4.40

Expected fixed costs:

Facility repair

$500,000.00

General, selling, and administrative

130,000.00

Required

a. Prepare the pro forma income statement that would appear in the master budget if the firm expects to provide 27,000 hours of services in 2009.

b. A marketing consultant suggests to Mr. Yu that the service rate may affect the number of service hours that the firm can achieve. According to the consultant’s analysis, if Centech charges customers $78 per hour, the firm can achieve 33,000 hours of services. Prepare a flexible budget using the consultant’s assumption.

c. The same consultant also suggests that if the firm raises its rate to $88 per hour, the number of service hours will decline to 22,000. Prepare a flexible budget using the new assumption.

d. Evaluate the three possible outcomes you determined in Requirements a, b, and c and recommend a pricing strategy.

would the manager of the computer department be likely to conduct the operations of 666960

Different types of responsibility centers

First National Bank is a large municipal bank with several branch offices. The bank’s computer department handles all data processing for bank operations. In addition, the bank sells the computer department’s expertise in systems development and excess machine time to several small business firms, serving them as a service bureau. The bank currently treats the computer department as a cost center. The manager of the computer department prepares a cost budget annually for senior bank officials to approve. Monthly operating reports compare actual and budgeted expenses. Revenues from the department’s service bureau activities are treated as other income by the bank and are not reflected on the computer department’s operating reports. The costs of servicing these clients are included in the computer department reports, however. The manager of the computer department has proposed that bank management convert the computer department to a profit or investment center.

Required

a. Describe the characteristics that differentiate a cost center, a profit center, and an investment center from each other.

b. Would the manager of the computer department be likely to conduct the operations of the department differently if the department were classified as a profit center or an investment center rather than as a cost center? Explain.

angelo corporation operates three investment centers the following financial stateme 666961

Comparing return on investment and residual income

Angelo Corporation operates three investment centers. The following financial statements apply to the investment center named Owen Division.

Income Statement

For the Year Ended December 31, 2008

Sales revenue

$113,385

Cost of goods sold

(60,420)

Gross margin

52,965

Operating expenses

Selling expenses

(1,345)

Depreciation expense

(1,300)

Operating income

52,320

Nonoperating income

Gain on sale of land

4,460

Net income

$ 54,780

OWEN DIVISION
Balance Sheet
As of December 31, 2008

Assets

Cash

$18,303

Accounts receivable

37,032

Merchandise inventory

37,455

Equipment less accum. dep.

90,000

Non operating assets

10,000

Total assets

$192,790

Liabilities

Accounts payable

$6,700

Notes payable

67,310

Stockholders’ equity

Common stock

64,000

Retained earnings

54,780

Total liab. and stk. equity

$192,790

Required

a. Should operating income or net income be used to determine the rate of return (ROI) for the Owen investment center? Explain your answer.

b. Should operating assets or total assets be used to determine the ROI for the Owen investment center? Explain your answer.

c. Calculate the ROI for Owen.

d. Angelo has a desired ROI of 10 percent. Headquarters has $91,000 of funds to assign its investment centers. The manager of the Owen division has an opportunity to invest the funds at an ROI of 13 percent. The other two divisions have investment opportunities that yield only 12 percent. Even so, the manager of Owen rejects the additional funding. Explain why the manager of Owen would reject the funds under these circumstances.

e. Explain how residual income could be used to encourage the manager to accept the additional funds.

hugo corporation s balance sheet indicates that the company has 350 000 invested in 666962

Return on investment

Hugo Corporation’s balance sheet indicates that the company has $350,000 invested in operating assets. During 2008, Hugo earned operating income of $56,000 on $700,000 of sales.

Required

a. Compute Hugo’s margin for 2008.

b. Compute Hugo’s turnover for 2008.

c. Compute Hugo’s return on investment for 2008.

d. Recompute Hugo’s ROI under each of the following independent assumptions.

(1) Sales increase from $700,000 to $840,000, thereby resulting in an increase in operating income from $56,000 to $67,200.

(2) Sales remain constant, but Hugo reduces expenses resulting in an increase in operating income from $56,000 to $58,000.

(3) Hugo is able to reduce its invested capital from $350,000 to $240,000 without affecting operating income.

would return on investment or residual income be the better performance measure for 666963

Comparing return on investment and residual income

The manager of the Alston Division of Stanford Manufacturing Corporation is currently producing a 20 percent return on invested capital. Stanford’s desired rate of return is 16 percent. The Alston Division has $6,000,000 of capital invested in operating assets and access to additional funds as needed. The manager is considering a new investment in operating assets that will require a $1,500,000 capital commitment and promises an 18 percent return.

Required

a. Would it be advantageous for Stanford Manufacturing Corporation if the Alston Division makes the investment under consideration?

b. What effect would the proposed investment have on the Alston Division’s return on investment? Show computations.

c. What effect would the proposed investment have on the Alston Division’s residual income? Show computations.

d. Would return on investment or residual income be the better performance measure for the Alston Division’s manager? Explain.

assume the labor price variance is unfavorable was the labor usage variance favorabl 666964

Business Applications Case Static versus flexible budget variances

Justin Radeka is the manufacturing production supervisor for Clear View Optics Company. Trying to explain why he did not get the year end bonus he had expected, he told his wife, This is the dumbest place I ever worked. Last year the company set up this budget assuming it would sell 230,000 lenses. Well, it sold only 220,000. The company lost money and gave me a bonus for not using as much materials and labor as was called for in the budget. This year, the company has the same 230,000 goal and it sells 250,000. The company’s making all kinds of money. You’d think I’d get this big fat bonus. Instead, management tells me I used more materials and labor than was budgeted. They say the company would have made a lot more money if I’d stayed within my budget. I guess I gotta wait for another bad year before I get a bonus. Like I said, this is the dumbest place I ever worked.” Clear View Company’s master budget and the actual results for the most recent year of operating activity follow.

Master Budget

Actual Results

Variances

F or U

Number of units

230,000

250,000

20,000

Sales revenue

$4,600,000

$5,037,500

$437,500

F

Variable manufacturing costs Materials

(759,000)

(835,000)

76,000

U

Labor

(391,000)

(420,000)

29,000

U

Overhead

(414,000)

(442,500)

28,500

U

Variable general, selling, and admin. costs

(552,000)

(612,500)

60,500

U

Contribution margin

2,484,000

2,727,500

243,500

F

Fixed costs Manufacturing overhead

(1,570,000)

(1,585,700)

15,700

U

General, selling, and

admin. costs

(575,000)

(563,500)

11,500

F

Net income

$ 339,000

$ 578,300

$239,300

F

Required

a. Did Clear View increase unit sales by cutting prices or by using some other strategy?

b. Is Mr. Radeka correct in his conclusion that something is wrong with the company’s performance evaluation process? If so, what do you suggest be done to improve the system?

c. Prepare a flexible budget and recompute the budget variances.

d. Explain what might have caused the fixed costs to be different from the amount budgeted.

e. Assume that the company’s material price variance was favorable and its material usage variance was unfavorable. Explain why Mr. Radeka may not be responsible for these variances. Now, explain why he may have been responsible for the material usage variance.

f. Assume the labor price variance is unfavorable. Was the labor usage variance favorable or unfavorable?

study the following cash inflow streams expected from two different potential invest 666968

1. Study the following cash inflow streams expected from two different potential investments.

Year 1

Year 2

Year 3

Total

Alternative 1

$2,000

$3,000

$4,000

$9,000

Alternative 2

4,000

3,000

2,000

9,000

Based on visual observation alone, which alternative has the higher present value? Why?

2. To increase productivity, Wald Corporation is considering the purchase of a new machine that costs $50,000. Wald expects using the machine to increase annual net cash inflows by $12,500 for each of the next five years. Wald desires a minimum annual rate of return of 10 percent on the investment. Determine the net present value of the investment opportunity and recommend whether Wald should acquire the machine.

3. EZ Rentals can purchase a van that costs $24,000. The van has an expected useful life of three years and no salvage value. EZ expects rental revenue from the van to be $12,000 per year. Determine the payback period and the unadjusted rate of return.

suggested format residence of james principal matters 666972

BULAW 5916 – SUGGESTED FORMAT OF ASSIGNMENT PREPARATION AND PRESENTATION DIRECTION Semester 2 year 2014

PART A Required: (1400 words)

1 For taxation purposes, is James a resident of Australia? What is the source of his salary from Sales International

2 For taxation purposes, is James a resident of Australia? What is the source of his If James sold his house, would he be entitles to the main residence exemption in Subdiv 118 – B?

Suggested format Residence of James – principal matters

1 Introduction

2 Four tests [Two does not appear to apply – 183 days & super test]

3 Residence test [state the law (i) statute law, (ii) case law, and (iii) Tax ruling and apply to the facts of James]

4 Domicile test [state the law (i) statute law, (ii) case law, and (iii) Tax ruling and apply to the facts of James]

5 Period of residency + dual residency with a conclusion for James and

6 Write on tax implication of residency for James

Hints: at least read Sadiq, Barkoczy, Gilders and Woellner and advanced students also read Kenny, Coleman, Burgess and Kobetsky & the suggested ATO Rulings

Suggested Format source of salary income of James – principal matters

1 Introduction

2 Source of employment income

3 State the taxation law (i) statute law, (ii) case law, and (iii) Taxation Ruling, and apply the principles to the facts of James employment income

4 Write a brief conclusion on the assessability of employment income

Hints: at least read Sadiq, Barkoczy, Gilders and Woellner and advanced students also read Kenny, Coleman, Burgess and Kobetsky & the suggested ATO Rulings

Suggested format taxation of the disposal of main residence principal matters

1 Introduction

2 State the statute law including (i) basic rule, (ii) rules that may extend the exemption, (iii) rules that may limit the exemption, and (iv) partial exemption rule, and then apply the principles to the facts of James

3 Write a brief conclusion on the assessability of the net capital gain, if applicable

Hints: at least read Sadiq, Barkoczy, Gilders and Woellner

PART B Required: (600 words)

Should Lee return on a cash or accrual basis in 2012/13 and 2013/2014

Professional services (business) income

1 Derivation of income – an Introduction

2 State the law (i) statute law, (ii) case law, and (iii) Tax ruling (TR 98/1, especially the following aspects

· Factors that affect the choice of cash or accrual basis

· Does Lee have a choice of the basis he adopts

· Does the commissioner of taxation has a right to insist on a particular basis?

3 Apply these laws to the facts of Lee’s business as an architect and comment on whether Lee’s income should be assessed on the same basis in both years.

Hints: at least read Sadiq, Barkoczy, Gilders and Woellner and advanced students also read Kenny, Coleman, Burgess and Kobetsky & the suggested ATO Rulings

LIST OF REFERENCE

Basic resources:

1 Textbooks – SADIQ, WOELLNER AND GILDERS

Principles of Taxation Law – SADIQ

· Chapter 4 Residence and source pp 81 106

· Chapter 11 Capital gains (main residence exemption pp 326 331

· Chapter 16 Tax accounting (derivation of income pp 515 539)

Australian Taxation Law – WOELLNER

· Chapter 13 Tax accounting (derivation of income pp 651 702)

· Chapter 08 Capital gains (main residence exemption pp 355 365

· Chapter 24 International aspects (residence & source pp 1260 1289)

Understanding Taxation Law – GILDERS

· Chapter 4 Derivation & measurement of income

· Chapter 6 Capital gain (main residence exemption Pp 381 388

· Chapter 2 Residence and source Pp 76 88

2 Taxation case book – Krever

· Chapter 11 Tax accounting (derivation pp239 249)

· Chapter 17 International aspects (residence & source pp 325 338)

Principles cases – SADIQ, WOELLNER AND COLMAN

· Levene v IRC [1928]

· IRC v Lysaght [1928]

· Miller v FCT [1946)

· Joachim v FCT (2002) 50 ATR 1072

· FCT v Applegate (1979) 9 ATR 899

· FCT v Jenkins (1982) 12 ATR 745

· FCT v French (1957)98 CLR 398

· FCT v Efstathakis (1979) 9 ATR 867

· FCT v Mitchum (1965) 113 CLR 401

· Carden’s case (1938) 63 CLR 108

· Gulland v FCT ( ) 83 ATC 4352

· FCT v Firstenberg (1976) 76 ATC 4141

· FCT v Dunn (1989) 20 ATR 356

· FCT v Henderson (1970) 119 CLR 612

· Barratt v FCT (1992) 23 ATR 339

· Dormer v FCT (2002) 51 ATR 353

3 Taxation Rulings

· TR 98/17 Residency of individuals entering Australia

· IT 2650 Permanent place of Abode outside Australia

· TR 98/1 Determination of income: receipts versus earnings basis

Residence of BASIL YEAR 30 JUNE 2013

Tax residence is very important for many reasons for individual taxpayers like basil. First, the individual taxpayers get a tax free threshold of $18 200 for a full year of residence. Second they pay tax at a rate which is different from the rates applicable to that of non resident individual taxpayers. Third assessable income of resident individuals income all income both Australian and international and finally resident taxpayers are allowed many tax offsets.

Tax residence is different from the residency of individuals under the nationality Act. Tax residency is determined in accordance with the Taxation Statues. Section 6(1) of Income tax assessment Act 1936 defines a person as resident of Australia when he resides in Australia and includes a person (i) whose domicile is in Australia, unless the Commissioner is satisfied that his permanent place of abode is outside Australia, (ii) who has actually been in Australia, continuously or intermittently, during more than one half of the year of income, unless the Commissioner is satisfied that his usual place of abode is outside Australia and he does not intend to take up residence in Australia, and (iii) a member of the Commonwealth superannuation scheme. Therefore there are four separate and exhaustive tests for determining whether an individual is resident of Australia for tax purposes:

· Residence test

· Domicile test

· The 183 day test, and

· Superannuation test

PART B Required: (1000 words)

Should Lee return on a cash or accrual basis in 2012/13 and 2013/2014

Professional services (business) income

4 Derivation of income – an Introduction

5 State the law (i) statute law, (ii) case law, and (iii) Tax ruling (TR 98/1, especially the following aspects

· Factors that affect the choice of cash or accrual basis

· Does Lee have a choice of the basis he adopts

· Does the commissioner of taxation has a right to insist on a particular basis?

6 Apply these law to the facts of Lee’s business as an architect and comment on whether Lee’s income should be assessed on the same basis in both years.

Attachments:

what is the concept of current rate method of translation and temporal method of tra 666981

  1. Translations and Reporting Issues
  1. What is the concept of current rate method of translation and temporal method of translation? How does balance sheet exposure differ under these two methods?

Provide your reference in Apa Format and include any In text citation

  1. The 2010 financial statement of Child Co. Inc (Mexico), a subsidiary of Parent Co. Inc (United States), reveals the following information:
Beginning Inventory Peso 100,000
Purchases Peso 500,000
Ending Inventory Peso 150,000
COGS Peso 450,000
US dollar exchange rate for 1 Peso:
January 1, 2010 $0.45
Average, 2010 $0.42
December 31, 2010 $0.38
  • The beginning inventory was acquired when the exchange rate was $0.50 last quarter of 2009; ending inventory was acquired when the exchange rate was $0.40 last quarter of 2010.
  • Report amounts of ending inventory and cost of goods sold to be included in the consolidated financial statements under (1) Current rate method and (2) Temporal method.

  1. Analyzing Foreign Financial Statements
    1. What are some of the problems an analyst might encounter in analyzing foreign financial statements?

Your well written 2 3 page essay should outline these problems, citing real world examples from at least two outside sources. Please ensure your paper is formatted per according to the standards for APA style and provide in text citation. You may also want to explore the following websites:

  • www.ifrs.org
  • www.fasb.org
  • www.aicpa.org

Attachments:

phil thomas and mary martin sole proprietor of art innovations have decided to form 667052

Phil Thomas and Mary Martin, sole proprietor of Art Innovations, have decided to form a partnership. All the assets and liabilities of Art Innovations will be contributed by Mary to the new partnership. Phil is an art broker with valuable relationships, which he will bring to the new partnership, along with $100,000 in cash. The new partnership will continue doing business as Art Innovations.

Existing book and fair market values for the assets and liabilities are as follows:

Account Book Value Fair Market Value

Cash

$55,000.00

$55,000.00

Accounts Receivable

14,000.00

12,000.00

Allowance for Doubtful Accounts

1,100.00

Inventory

94,000.00

120,000.00

Equipment

70,000.00

12,000.00

Accumulated Depreciation

47,000.00

Accounts Payable

4,500.00

4,500.00

Loan Payable

28,500.00

28,500.00

Phil and Mary have agreed to use the bonus method to begin the partnership with each partner having an equal capital balance. Mary will contribute significantly more than Phil in physical assets and liabilities but Phil will bring his expertise and client relationships.

  1. Create an opening balance sheet for the new Art Innovations partnership. Assume the new partnership will be created on June 1, 2012.

  2. Identify the elements that should be addressed in the partnership agreement both from a business standpoint and from an accounting standpoint.

what is the basis of the new property in each of the following exchanges a apartment 667146

What is the basis of the new property in each of the following exchanges?

a. Apartment building held for investment (adjusted basis of $145,000) for office building to be held for investment (fair market value of $225,000).

b. Land and building used as a barbershop (adjusted basis of $190,000) for land and

building used as a grocery store (fair market value of $350,000).

c. Office building (adjusted basis of $45,000) for bulldozer (fair market value of

$42,000), both held for business use.

d. IBM common stock (adjusted basis of $20,000) for ExxonMobil common stock (fair
market value of $28,000).

e. Rental house (adjusted basis of $90,000) for mountain cabin to be held for rental

use (fair market value of $225,000).

f. General partnership interest (adjusted basis of $400,000) for a limited partnership

interest (fair market value of $580,000).

determine the amount of ending inventory the company will report on its pro forma ba 666911

Preparing an inventory purchases budget

Tucker Lighting Company sells lamps and other lighting fixtures. The purchasing department manager prepared the following inventory purchases budget. Tucker Lighting’s policy is to maintain an ending inventory balance equal to 10 percent of the following month’s cost of goods sold. April’s budgeted cost of goods sold is $85,000.

January

February

March

Budgeted cost of goods sold

$70,000

$74,000

$80,000

Plus: Desired ending inventory

7,400

?

?

Inventory needed

77,400

?

?

Less: Beginning inventory

18,000

?

?

Required purchases (on account)

$59,400

?

?

Required

a. Complete the inventory purchases budget by filling in the missing amounts.

b. Determine the amount of cost of goods sold the company will report on its first quarter pro forma income statement.

c. Determine the amount of ending inventory the company will report on its pro forma balance sheet at the end of the first quarter.

readers home buys books and magazines directly from publishers and distributes them 666912

Preparing a schedule of cash payments for inventory purchases

Readers’ Home buys books and magazines directly from publishers and distributes them to grocery stores. The wholesaler expects to purchase the following inventory.

April

May

June

Required purchases (on account)

$70,000

$90,000

$110,000

Readers’ Home’s accountant prepared the following schedule of cash payments for inventory purchases. Readers’ Home’s suppliers require that 90 percent of purchases on account be paid in the month of purchase; the remaining 10 percent are paid in the month following the month of purchase.

Schedule of Cash Payments for Inventory Purchases

April

May

June

Payment for current accounts payable

$63,000

?

?

Payment for previous accounts payable

4,000

?

?

Total budgeted payments for inventory

$67,000

?

?

Required

a. Complete the schedule of cash payments for inventory purchases by filling in the missing amounts.

b. Determine the amount of accounts payable the company will report on its pro forma balance sheet at the end of the second quarter.

determine the amount of cash payments budgeted for inventory purchases in july 666913

Determining the amount of expected inventory purchases and cash payments

Birchem Company, which sells electric razors, had $260,000 of cost of goods sold during the month of June. The company projects a 5 percent increase in cost of goods sold during July. The inventory balance as of June 30 is $28,000, and the desired ending inventory balance for July is $29,000. Birchem pays cash to settle 80 percent of its purchases on account during the month of purchase and pays the remaining 20 percent in the month following the purchase. The accounts payable balance as of June 30 was $35,000.

Required

a. Determine the amount of purchases budgeted for July.

b. Determine the amount of cash payments budgeted for inventory purchases in July.

complete the schedule of cash payments for s a expenses by filling in the missing am 666914

Preparing a schedule of cash payments for selling and administrative expenses

The budget director for Lenoir Window Cleaning Services prepared the following list of expected operating expenses. All expenses requiring cash payments are paid for in the month incurred except salary expense and insurance. Salary is paid in the month following the month in which it is incurred. The insurance premium for six months is paid on October 1. October is the first month of operations; accordingly, there are no beginning account balances.

October

November

December

Budgeted S&A Expenses

Equipment lease expense

$ 6,000

$ 6,000

$ 6,000

Salary expense

6,100

6,600

7,000

Cleaning supplies

2,800

2,730

3,066

Insurance expense

1,200

1,200

1,200

Depreciation on computer

1,800

1,800

1,800

Rent

1,700

1,700

1,700

Miscellaneous expenses

700

700

700

Total S&A expenses

$20,300

$20,730

$21,466

Schedule of Cash Payments for S&A Expenses

Equipment lease expense

?

?

?

Prior month’s salary expense, 100%

?

?

?

Cleaning supplies

?

?

?

Insurance premium

?

?

?

Depreciation on computer

?

?

?

Rent

?

?

?

Miscellaneous expenses

?

?

?

Total disbursements for S&A expenses

$18,400

$17,230

$18,066

Required

a. Complete the schedule of cash payments for S&A expenses by filling in the missing amounts.

b. Determine the amount of salaries payable the company will report on its pro forma balance sheet at the end of the fourth quarter.

c. Determine the amount of prepaid insurance the company will report on its pro forma balance sheet at the end of the fourth quarter.

executive officers of shavez company are wrestling with their budget for the next ye 666915

Preparing inventory purchases budgets with different assumptions

Executive officers of Shavez Company are wrestling with their budget for the next year. The following are two different sales estimates provided by two different sources.

Source of Estimate

First Quarter

Second Quarter

Third Quarter

Fourth Quarter

Sales manager

$380,000

$310,000

$280,000

$480,000

Marketing consultant

520,000

460,000

410,000

650,000

Shavez’s past experience indicates that cost of goods sold is about 60 percent of sales revenue. The company tries to maintain 10 percent of the next quarter’s expected cost of goods sold as the current quarter’s ending inventory. This year’s ending inventory is $29,000. Next year’s ending inventory is budgeted to be $30,000.

Required

a. Prepare an inventory purchases budget using the sales manager’s estimate.

b. Prepare an inventory purchases budget using the marketing consultant’s estimate.

determine the amount of depreciation expense the store will report on the income sta 666916

Determining the amount of cash payments and pro forma statement data for selling and administrative expenses

January budgeted selling and administrative expenses for the retail shoe store that May Rozell plans to open on January 1, 2009, are as follows: sales commissions, $18,000; rent, $15,000; utilities, $5,000; depreciation, $4,000; and miscellaneous, $2,000. Utilities are paid in the month following their incursion. Other expenses are expected to be paid in cash in the month in which they are incurred.

Required

a. Determine the amount of budgeted cash payments for January selling and administrative expenses.

b. Determine the amount of utilities payable the store will report on the January 31st pro forma balance sheet.

c. Determine the amount of depreciation expense the store will report on the income statement for the year 2009, assuming that monthly depreciation remains the same for the entire year.

the accountant for lori s dress shop prepared the following cash budget lori s desir 666917

Preparing a cash budget

The accountant for Lori’s Dress Shop prepared the following cash budget. Lori’s desires to maintain a cash cushion of $14,000 at the end of each month. Funds are assumed to be borrowed and repaid on the last day of each month. Interest is charged at the rate of 2 percent per month.

Cash Budget

July

August

September

Section 1: Cash receipts

Beginning cash balance

$43,000

$ ?

$ ?

Add cash receipts

183,000

196,000

240,200

Total cash available (a)

226,000

?

?

Section 2: Cash payments

For inventory purchases

163,646

139,900

172,474

For S&A expenses

54,000

62,060

61,536

For interest expense

0

?

?

Total budgeted disbursements (b)

217,646

?

?

Section 3: Financing activities

Surplus (shortage)

8,354

?

?

Borrowing (repayments) (c)

5,646

?

?

Ending cash balance (a b + c)

$14,000

$14,000

$14,000

Required

a. Complete the cash budget by filling in the missing amounts. Round all computations to the nearest whole dollar.

b. Determine the amount of net cash flows from operating activities Lori’s will report on the third quarter pro forma statement of cash flows.

c. Determine the amount of net cash flows from financing activities Lori’s will report on the third quarter pro forma statement of cash flows.

what amount will the restaurant report as interest expense on the july pro forma inc 666918

Determining amount to borrow and pro forma financial statement balances

Ellen Crawley owns a small restaurant in New York City. Ms. Crawley provided her accountant with the following summary information regarding expectations for the month of June. The balance in accounts receivable as of May 31 is $55,000. Budgeted cash and credit sales for June are $105,000 and $525,000, respectively. Credit sales are made through Visa and MasterCard and are collected rapidly. Ninety percent of credit sales is collected in the month of sale, and the remainder is collected in the following month. Ms. Crawley’s suppliers do not extend credit. Consequently, she pays suppliers on the last day of the month. Cash payments for June are expected to be $640,000. Ms. Crawley has a line of credit that enables the restaurant to borrow funds on demand; however, they must be borrowed on the last day of the month. Interest is paid in cash also on the last day of the month. Ms. Crawley desires to maintain a $30,000 cash balance before the interest payment. Her annual interest rate is 9 percent.

Disregard any credit card fees.

Required

a. Compute the amount of funds Ms. Crawley needs to borrow for June, assuming that the beginning cash balance is zero.

b. Determine the amount of interest expense the restaurant will report on the June pro forma income statement.

c. What amount will the restaurant report as interest expense on the July pro forma income statement?

jim denty the controller of grime corporation is trying to prepare a sales budget fo 666919

Preparing pro forma income statements with different assumptions

Jim Denty, the controller of Grime Corporation, is trying to prepare a sales budget for the coming year. The income statements for the last four quarters follow.

First

Second

Third

Fourth

Quarter

Quarter

Quarter

Quarter

Total

Sales revenue

$170,000

$200,000

$210,000

$260,000

$840,000

Cost of goods sold

102,000

120,000

126,000

156,000

504,000

Gross profit

68,000

80,000

84,000

104,000

336,000

Selling & admin. expense

17,000

20,000

21,000

26,000

84,000

Net income

$ 51,000

$ 60,000

$ 63,000

$ 78,000

$252,000

Historically, cost of goods sold is about 60 percent of sales revenue. Selling and administrative expenses are about 10 percent of sales revenue. Gene Moreno, the chief executive officer, told Mr. Denty that he expected sales next year to be 10 percent above last year’s level. However, Sarah Toole, the vice president of sales, told Mr. Denty that she believed sales growth would be only 5 percent.

Required

a. Prepare a pro forma income statement including quarterly budgets for the coming year using Mr. Moreno’s estimate.

b. Prepare a pro forma income statement including quarterly budgets for the coming year using Ms. Toole’s estimate.

c. Explain why two executive officers in the same company could have different estimates of future growth.

caine inc sells fireworks the company s marketing director developed the following c 666921

Preparing an inventory purchases budget and schedule of cash payments

Caine Inc. sells fireworks. The company’s marketing director developed the following cost of goods sold budget for April, May, June, and July.

April

May

June

July

Budgeted cost of goods sold

$70,000

$80,000

$90,000

$76,000

Caine had a beginning inventory balance of $3,500 on April 1 and a beginning balance in accounts payable of $15,100. The company desires to maintain an ending inventory balance equal to 10 percent of the next period’s cost of goods sold. Caine makes all purchases on account. The company pays 50 percent of accounts payable in the month of purchase and the remaining 50 percent in the month following purchase.

Required

a. Prepare an inventory purchases budget for April, May, and June.

b. Determine the amount of ending inventory Caine will report on the end of quarter pro forma balance sheet.

c. Prepare a schedule of cash payments for inventory for April, May, and June.

d. Determine the balance in accounts payable Caine will report on the end of quarter pro forma balance sheet.

top executive officers of leach company a merchandising firm are preparing the next 666922

Preparing pro forma income statements with different assumptions

Top executive officers of Leach Company, a merchandising firm, are preparing the next year’s budget. The controller has provided everyone with the current year’s projected income statement.

Current Year

Sales revenue

$2,600,000

Cost of goods sold

1,690,000

Gross profit

910,000

Selling & admin. expenses

325,000

Net income

$ 585,000

Cost of goods sold is usually 65 percent of sales revenue, and selling and administrative expenses are usually 10 percent of sales plus a fixed cost of $65,000. The president has announced that the company’s goal is to increase net income by 15 percent.

Required

The following items are independent of each other.

a. What percentage increase in sales would enable the company to reach its goal? Support your answer with a pro forma income statement.

b. The market may become stagnant next year, and the company does not expect an increase in sales revenue. The production manager believes that an improved production procedure can cut cost of goods sold by 2 percent. What else can the company do to reach its goal? Prepare a pro forma income statement illustrating your proposal.

c. The company decides to escalate its advertising campaign to boost consumer recognition, which will increase selling and administrative expenses to $405,000. With the increased advertising, the company expects sales revenue to increase by 15 percent. Assume that cost of goods sold remains a constant proportion of sales. Can the company reach its goal?

prepare a schedule of cash payments for selling and administrative expenses 666923

Preparing a schedule of cash payments for selling and administrative expenses

Wynn is a retail company specializing in men’s hats. Its budget director prepared the list of expected operating expenses that follows. All items are paid when the expenses are incurred except sales commissions and utilities, which are paid in the month after they are incurred. July is the first month of operations, so there are no beginning account balances.

July

August

September

Salary expense

$14,000

$14,000

$14,000

Sales commissions (4 percent of sales)

1,700

1,700

1,700

Supplies expense

360

390

420

Utilities

1,100

1,100

1,100

Depreciation on store equipment

2,800

2,800

2,800

Rent

6,600

6,600

6,600

Miscellaneous

690

690

690

Total S&A expenses before interest

$27,250

$27,280

$27,310

Required

a. Prepare a schedule of cash payments for selling and administrative expenses.

b. Determine the amount of utilities payable as of September 30.

c. Determine the amount of sales commissions payable as of September 30.

mcbride medical clinic has budgeted the following cash flows 666924

Preparing a cash budget

McBride Medical Clinic has budgeted the following cash flows.

January

February

March

Cash receipts

$101,000

$108,000

$125,000

Cash payments

For inventory purchases

92,000

72,000

83,000

For S&A expenses

30,000

32,000

26,000

McBride Medical had a cash balance of $7,000 on January 1. The company desires to maintain a cash cushion of $5,000. Funds are assumed to be borrowed, in increments of $1,000, and repaid on the last day of each month; the interest rate is 1 percent per month. McBride pays its vendor on the last day of the month also. The company had a $30,000 beginning balance in its line of credit liability account.

Required

Prepare a cash budget. (Round all computations to the nearest whole dollar.)

the company expects to collect 100 percent of the accounts receivable generated by c 666925

Preparing a master budget for a retail company with no beginning account balances

Resha Company is a retail company that specializes in selling outdoor camping equipment. The company is considering opening a new store on October 1, 2009. The company president formed a planning committee to prepare a master budget for the first three months of operation. He assigned you, the budget coordinator, the following tasks.

Required

a. October sales are estimated to be $120,000 of which 40 percent will be cash and 60 percent will be credit. The company expects sales to increase at the rate of 25 percent per month. Prepare a sales budget.

b. The company expects to collect 100 percent of the accounts receivable generated by credit sales in the month following the sale. Prepare a schedule of cash receipts.

c. The cost of goods sold is 60 percent of sales. The company desires to maintain a minimum ending inventory equal to 10 percent of the next month’s cost of goods sold. Ending inventory at December 31 is expected to be $12,000. Assume that all purchases are made on account. Prepare an inventory purchases budget.

d. The company pays 70 percent of accounts payable in the month of purchase and the remaining 30 percent in the following month. Prepare a cash payments budget for inventory purchases.

e. Budgeted selling and administrative expenses per month follow.

Salary expense (fixed)

$18,000

Sales commissions

5 percent of Sales

Supplies expense

2 percent of Sales

Utilities (fixed)

$1,400

Depreciation on store equipment (fixed)*

$4,000

Rent (fixed)

$4,800

Miscellaneous (fixed)

$1,200

Use this information to prepare a selling and administrative expenses budget.

f. Utilities and sales commissions are paid the month after they are incurred; all other expenses are paid in the month in which they are incurred. Prepare a cash payments budget for selling and administrative expenses.

g. Resha borrows funds, in increments of $1,000, and repays them on the last day of the month. The company also pays its vendors on the last day of the month. It pays interest of 1 percent per month in cash on the last day of the month. To be prudent, the company desires to maintain a $12,000 cash cushion. Prepare a cash budget.

h. Prepare a pro forma income statement for the quarter.

i. Prepare a pro forma balance sheet at the end of the quarter.

j. Prepare a pro forma statement of cash flows for the quarter.

if growth is actually 10 percent as usual how much bonus would everett division s ex 666926

Behavioral impact of budgeting

Zigler Corporation has three divisions, each operating as a responsibility center. To provide an incentive for divisional executive officers, the company gives divisional management a bonus equal to 20 percent of the excess of actual net income over budgeted net income. The following is Everett Division’s current year’s performance.

Current Year

Sales revenue

$3,200,000

Cost of goods sold

2,000,000

Gross profit

1,200,000

Selling & admin. expenses

640,000

Net income

$ 560,000

The president has just received next year’s budget proposal from the vice president in charge of Everett Division. The proposal budgets a 5 percent increase in sales revenue with an extensive explanation about stiff market competition. The president is puzzled. Everett has enjoyed revenue growth of around 10 percent for each of the past five years. The president had consistently approved the division’s budget proposals based on 5 percent growth in the past. This time, the president wants to show that he is not a fool. I will impose a 15 percent revenue increase to teach them a lesson!” the president says to himself smugly.

Assume that cost of goods sold and selling and administrative expenses remain stable in proportion to sales.

Required

a. Prepare the budgeted income statement based on Everett Division’s proposal of a 5 percent increase.

b. If growth is actually 10 percent as usual, how much bonus would Everett Division’s executive officers receive if the president had approved the division’s proposal?

c. Prepare the budgeted income statement based on the 15 percent increase the president imposed.

d. If the actual results turn out to be a 10 percent increase as usual, how much bonus would Everett Division’s executive officers receive since the president imposed a 15 percent increase?

e. Propose a better budgeting procedure for Zigler.

review the statements you prepared for the first requirement and prepare a list of r 666927

Business Applications Case Preparing and using pro forma statements

Rachael Moulton and Bobby Lagg recently graduated from the same university. After graduation they decided not to seek jobs in established organizations but to start their own small business. They hoped this would provide more flexibility in their personal lives for a few years. Since both of them enjoyed cooking, they decided on a business selling vegetarian wraps and fruit juices from a street cart near their alma mater. They bought a small enclosed cart for $5,000 that was set up for selling food. This cost, along with the cost for supplies to get started, a business license, and street vendor license, brought their initial expenditures to $6,500. They used $1,000 of their personal savings, and they borrowed $5,500 from Rachael’s parents. They agreed to pay interest on the outstanding loan balance each month based on an annual rate of 5 percent. They will repay the principal over the next two years as cash becomes available. After two months in business, September and October, they had average monthly revenues of $8,200 and out of pocket costs of $5,100 for ingredients, paper supplies, and so on, but not interest. Bobby thinks they should repay some of the money they borrowed, but Rachael thinks they should prepare a set of forecasted financial statements for their first year in business before deciding whether or not to repay any principal on the loan. She remembers a bit about budgeting from a survey of accounting course she took and thinks the results from their first two months in business can be extended over the next 10 months to prepare the budget they need. They estimate the cart will last at least three years, after which they expect to sell it for $1,000 and move on to something else in their lives. Rachael agrees to prepare a forecasted (pro forma) income statement, balance sheet, and statement of cash flows for their first year in business, which includes the two months already passed.

Required

a. Prepare the annual pro forma financial statements that you would expect Rachael to prepare based on her comments about her expectations for the business. Assume no principal will be repaid on the loan.

b. Review the statements you prepared for the first requirement and prepare a list of reasons why Bobby and Rachael’s business results probably will not agree with their budgeted statements.

select a representative from each section have the representatives supply the missin 666928

Group Assignment Master budget and pro forma statements

The following trial balance was drawn from the records of Havel Company as of October 1, 2010.

Cash

$ 16,000

Accounts receivable

60,000

Inventory

40,000

Store equipment

200,000

Accumulated depreciation

$ 76,800

Accounts payable

72,000

Line of credit loan

100,000

Common stock

50,000

Retained earnings

17,200

Totals

$316,000

$316,000

Required

a. Divide the class into groups, each with four or five students. Organize the groups into three sections. Assign Task 1 to the first section, Task 2 to the second section, and Task 3 to the third section.

Group Tasks

(1) Based on the following information, prepare a sales budget and a schedule of cash receipts for October, November, and December. Sales for October are expected to be $180,000, consisting of $40,000 in cash and $140,000 on credit. The company expects sales to increase at the rate of 10 percent per month. All of accounts receivable is collected in the month following the sale.

(2) Based on the following information, prepare a purchases budget and a schedule of cash payments for inventory purchases for October, November, and December. Cost of goods sold for October is expected to be $72,000. Cost of goods sold is expected to increase by 10 percent per month in November and December. Havel expects January cost of goods sold to be $89,000. The company desires to maintain a minimum ending inventory equal to 20 percent of the next month’s cost of goods sold. Seventy five percent of accounts payable is paid in the month that the purchase occurs; the remaining 25 percent is paid in the following month.

(3) Based on the following selling and administrative expenses budgeted for October, prepare a selling and administrative expenses budget for October, November, and December. Cash payments for sales commissions and utilities are made in the month following the one in which the expense is incurred. Supplies and other operating expenses are paid in cash in the month in which they are incurred. As of October 1, no amounts were payable for either commissions or utilities from the previous month.

Sales commissions (10% increase per month)

$ 7,200

Supplies expense (10% increase per month)

1,800

Utilities (fixed)

2,200

Depreciation on store equipment (fixed)

1,600

Salary expense (fixed)

34,000

Rent (fixed)

6,000

Miscellaneous (fixed)

1,000

b. Select a representative from each section. Have the representatives supply the missing information in the following pro forma income statement and balance sheet for the fourth quarter of 2010. The statements are prepared as of December 31, 2010.

Income Statement

Sales Revenue

$ ?

Cost of Goods Sold

?

Gross Margin

357,480

Operating Expenses

?

Operating Income

193,290

Interest Expense

(2,530)

Net Income

$190,760

Balance Sheet

Assets

Cash

$ 9,082

Accounts Receivable

?

Inventory

?

Store Equipment

$200,000

Accumulated Depreciation Store Equipment

?

Book Value of Equipment

118,400

Total Assets

$314,682

Liabilities

Accounts Payable

?

Utilities Payable

?

Sales Commissions Payable

?

Line of Credit

23,936

Equity

Common Stock

50,000

Retained Earnings

?

Total Liabilities and Equity

$314,682

c. Indicate whether Havel will need to borrow money during October.

assume you are toll brothers budget director write a memo to the management team exp 666930

Writing Assignment Continuous budgeting

Toll Brothers, Inc., is a large builder of luxury homes across the United States. From 2000 through 2006 it experienced continuous growth in revenues that averaged over 24 percent annually. Not only did it experience growth from year to year, but its revenue grew in each quarter of 2005 and 2006. Then things started to slow down. In 2007 its revenue dropped by 24 percent compared to 2006. Additionally, its revenue declined in two of the four quarters of 2007. Management of the company commented as follows in the company’s 2007 annual report.

Required

Assume you are Toll Brothers’ budget director. Write a memo to the management team explaining how the practice of continuous budgeting could overcome the shortcomings of an annual budget process in an uncertain market situation.

the static master budget of parcel inc called for a production and sales volume of 2 666931

1. The static (master) budget of Parcel, Inc., called for a production and sales volume of 25,000 units. At that volume, total budgeted fixed costs were $150,000 and total budgeted variable costs were $200,000. Prepare a flexible budget for an expected volume of 26,000 units.

2. Scott Company’s master budget called for a planned sales volume of 30,000 units. Budgeted direct materials cost was $4 per unit. Scott actually produced and sold 32,000 units with an actual materials cost of $3.90 per unit. Determine the volume variance for materials cost and identify the organizational unit most likely responsible for this variance. Determine the flexible budget variance for materials cost and identify the organizational unit most likely responsible for this variance.

the following financial statements apply to hola division one of three investment ce 666935

The following financial statements apply to Hola Division, one of three investment centers operated by Costa Corporation. Costa Corporation has a desired rate of return of 15 percent. Costa Corporation Headquarters has $80,000 of additional operating assets to assign to the investment centers.

HOLA DIVISION

Income Statement

For the Year Ended December 31, 2009

Sales revenue

$ 78,695

Cost of goods sold

(50,810)

Gross margin

27,885

Operating expenses

Selling expenses

(1,200)

Depreciation expense

(1,125)

Operating income

25,560

Nonoperating expense

Loss on sale of land

(3,200)

Net income

$ 22,360

HOLA DIVISION
Balance Sheet
As of December 31, 2009

Assets

Cash

$8,089

Accounts receivable

22,870

Merchandise inventory

33,460

Equipment less acc. dep.

77,581

Nonoperating assets

8,250

Total assets

$150,250

Liabilities

Accounts payable

$5,000

Notes payable

58,000

Stockholders’ equity

Common stock

55,000

Retained earnings

32,250

Total liab. and stk. equity

$150,250

Required

a. Should Costa use operating income or net income to determine the rate of return (ROI) for the Hola investment center? Explain.

b. Should Costa use operating assets or total assets to determine the ROI for the Hola investment center? Explain.

c. Calculate the ROI for Hola.

d. The manager of the Hola division has an opportunity to invest the funds at an ROI of 17 percent. The other two divisions have investment opportunities that yield only 16 percent. The manager of Hola rejects the additional funding. Why would the manager of Hola reject the funds under these circumstances?

e. Calculate the residual income from the investment opportunity available to Hola and explain how residual income could be used to encourage the manager to accept the additional funds.

hanley corporation is considering the elimination of one of its segments the segment 666885

Identifying avoidable cost of a segment

Hanley Corporation is considering the elimination of one of its segments. The segment incurs the following fixed costs. If the segment is eliminated, the building it uses will be sold.

Advertising expense

$ 92,000

Supervisory salaries

151,000

Allocation of companywide facility level costs

49,000

Original cost of building

110,000

Book value of building

50,000

Market value of building

80,000

Maintenance costs on equipment

70,000

Real estate taxes on building

6,000

Required

Based on this information, determine the amount of avoidable cost associated with the segment.

based on this information recommend whether to replace the equipment support your re 666887

Asset replacement decision

Elrod Company is considering replacement of some of its manufacturing equipment. Information regarding the existing equipment and the potential replacement equipment follows.

Existing Equipment

Replacement Equipment

Cost

$ 90,000

Cost

$95,000

Operating expenses*

115,000

Operating expenses*

95,000

Salvage value

10,000

Salvage value

12,000

Market value

60,000

Useful life

8 years

Book value

33,000

Remaining useful life

8 years

Required

Based on this information, recommend whether to replace the equipment. Support your recommendation with appropriate computations.

the old machine is more durable and is expected to have a remaining useful life of f 666888

Asset replacement decision

Philpot Company paid $73,000 to purchase a machine on January 1, 2007. During 2009, a technological breakthrough resulted in the development of a new machine that costs $120,000. The old machine costs $41,000 per year to operate, but the new machine could be operated for only $12,000 per year. The new machine, which will be available for delivery on January 1, 2010, has an expected useful life of four years. The old machine is more durable and is expected to have a remaining useful life of four years. The current market value of the old machine is $30,000. The expected salvage value of both machines is zero.

Required

Based on this information, recommend whether to replace the machine. Support your recommendation with appropriate computations.

determine the total cost of the lease over the four year contract based on your comp 666889

Annual versus cumulative data for replacement decision

Because of rapidly advancing technology, Newell Publications Inc. is considering replacing its existing typesetting machine with leased equipment. The old machine, purchased two years ago, has an expected useful life of six years and is in good condition. Apparently, it will continue to perform as expected for the remaining four years of its expected useful life. A four year lease for equipment with comparable productivity can be obtained for $14,000 per year. The following data apply to the old machine.

Original cost

$160,000

Accumulated depreciation

55,000

Current market value

74,000

Estimated salvage value

10,000

Required

a. Determine the annual opportunity cost of using the old machine. Based on your computations, recommend whether to replace it.

b. Determine the total cost of the lease over the four year contract. Based on your computations, recommend whether to replace the old machine.

describe two decision making contexts one in which depreciation on manufacturing equ 666890

Context sensitive relevance

Required

Respond to each requirement independently.

a. Describe two decision making contexts, one in which unit level materials costs are avoidable, and the other in which they are unavoidable.

b. Describe two decision making contexts, one in which batch level setup costs are avoidable, and the other in which they are unavoidable.

c. Describe two decision making contexts, one in which advertising costs are avoidable, and the other in which they are unavoidable.

d. Describe two decision making contexts, one in which rent paid for a building is avoidable, and the other in which it is unavoidable.

e. Describe two decision making contexts, one in which depreciation on manufacturing equipment is avoidable, and the other in which it is unavoidable.

assume that overby has decided to accept one of the two jobs identify the informatio 666891

Context sensitive relevance

Overby Construction Company is a building contractor specializing in small commercial buildings. The company has the opportunity to accept one of two jobs; it cannot accept both because they must be performed at the same time and Overby does not have the necessary labor force for both jobs. Indeed, it will be necessary to hire a new supervisor if either job is accepted. Furthermore, additional insurance will be required if either job is accepted. The revenue and costs associated with each job follow.

Cost Category

Job A

Job B

Contract price

$730,000

$650,000

Unit level materials

243,700

223,450

Unit level labor

249,150

253,750

Unit level overhead

18,000

12,600

Supervisor’s salary

116,670

116,670

Rental equipment costs

24,900

27,300

Depreciation on tools (zero market value)

19,900

19,900

Allocated portion of companywide facility sustaining costs

10,400

8,600

Insurance cost for job

18,200

18,200

Required

a. Assume that Overby has decided to accept one of the two jobs. Identify the information relevant to selecting one job versus the other. Recommend which job to accept and support your answer with appropriate computations.

b. Assume that Job A is no longer available. Overby’s choice is to accept or reject Job B alone. Identify the information relevant to this decision. Recommend whether to accept or reject Job B. Support your answer with appropriate computations.

describe the qualitative factors that awtrey quilting company should consider before 666892

Effect of order quantity on special order decision

Awtrey Quilting Company makes blankets that it markets through a variety of department stores. It makes the blankets in batches of 1,000 units. Awtrey made 20,000 blankets during the prior accounting period. The cost of producing the blankets is summarized here.

Materials cost ($21 per unit X 20,000)

$ 420,000

Labor cost ($22 per unit X 20,000)

440,000

Manufacturing supplies ($2 X 20,000)

40,000

Batch level costs (20 batches at $4,000 per batch)

80,000

Product level costs

160,000

Facility level costs

290,000

Total costs

$1,430,000

Cost per unit = $1,430,000 ÷ 20,000 = $71.50

Required

a. Sunny Motels has offered to buy a batch of 500 blankets for $51 each. Awtrey’s normal selling price is $90 per unit. Based on the preceding quantitative data, should Awtrey accept the special order? Support your answer with appropriate computations.

b. Would your answer to Requirement a change if Sunny offered to buy a batch of 1,000 blankets for $51 per unit? Support your answer with appropriate computations.

c. Describe the qualitative factors that Awtrey Quilting Company should consider before accepting a special order to sell blankets to Sunny Motels.

discuss the qualitative factors that faucette should consider before deciding to out 666893

Effects of the level of production on an outsourcing decision

Faucette Chemical Company makes a variety of cosmetic products, one of which is a skin cream designed to reduce the signs of aging. Faucette produces a relatively small amount (15,000 units) of the cream and is considering the purchase of the product from an outside supplier for $4.50 each. If Faucette purchases from the outside supplier, it would continue to sell and distribute the cream under its own brand name. Faucette’s accountant constructed the following profitability analysis.

Revenue (15,000 units X $9)

$135,000

Unit level materials costs (15,000 units X $1.40)

(21,000)

Unit level labor costs (15,000 units X $0.50)

(7,500)

Unit level overhead costs (15,000 X $0.10)

(1,500)

Unit level selling expenses (15,000 X $0.20)

(3,000)

Contribution margin

102,000

Skin cream production supervisor’s salary

(44,000)

Allocated portion of facility level costs

(11,300)

Product level advertising cost

(34,000)

Contribution to companywide income

$ 12,700

Required

a. Identify the cost items relevant to the make or outsource decision.

b. Should Faucette continue to make the product or buy it from the supplier? Support your answer by determining the change in net income if Faucette buys the cream instead of making it.

c. Suppose that Faucette is able to increase sales by 10,000 units (sales will increase to 25,000 units). At this level of production, should Faucette make or buy the cream? Support your answer by explaining how the increase in production affects the cost per unit.

d. Discuss the qualitative factors that Faucette should consider before deciding to outsource the skin cream. How can Faucette minimize the risk of establishing a relationship with an unreliable supplier?

holcombe bike company hbc makes the frames used to build its bicycles during 2009 hb 666894

Outsourcing decision affected by equipment replacement

Holcombe Bike Company (HBC) makes the frames used to build its bicycles. During 2009, HBC made 20,000 frames; the costs incurred follow.

Unit level materials costs (20,000 units X $40)

$ 800,000

Unit level labor costs (20,000 units X $51)

1,020,000

Unit level overhead costs (20,000 X $9)

180,000

Depreciation on manufacturing equipment

90,000

Bike frame production supervisor’s salary

70,000

Inventory holding costs

290,000

Allocated portion of facility level costs

500,000

Total costs

$2,950,000

HBC has an opportunity to purchase frames for $100 each.

Additional Information

1. The manufacturing equipment, which originally cost $550,000, has a book value of $450,000, a remaining useful life of four years, and a zero salvage value. If the equipment is not used to produce bicycle frames, it can be leased for $70,000 per year.

2. HBC has the opportunity to purchase for $910,000 new manufacturing equipment that will have an expected useful life of four years and a salvage value of $70,000. This equipment will increase productivity substantially, reducing unit level labor costs by 60 percent. Assume that HBC will continue to produce and sell 20,000 frames per year in the future.

3. If HBC outsources the frames, the company can eliminate 80 percent of the inventory holding costs.

Required

a. Determine the avoidable cost per unit of making the bike frames, assuming that HBC is considering the alternatives between making the product using the existing equipment and outsourcing the product to the independent contractor. Based on the quantitative data, should HBC outsource the bike frames? Support your answer with appropriate computations.

b. Assuming that HBC is considering whether to replace the old equipment with the new equipment, determine the avoidable cost per unit to produce the bike frames using the new equipment and the avoidable cost per unit to produce the bike frames using the old equipment. Calculate the impact on profitability if the bike frames were made using the old equipment versus the new equipment.

c. Assuming that HBC is considering to either purchase the new equipment or outsource the bike frame, calculate the impact on profitability between the two alternatives.

d. Discuss the qualitative factors that HBC should consider before making a decision to outsource the bike frame. How can HBC minimize the risk of establishing a relationship with an unreliable supplier?

confirm the conclusion you reached in requirement a by preparing income statements f 666895

Eliminating a segment

Levene Boot Co. sells men’s, women’s, and children’s boots. For each type of boot sold, it operates a separate department that has its own manager. The manager of the men’s department has a sales staff of nine employees, the manager of the women’s department has six employees, and the manager of the children’s department has three employees. All departments are housed in a single store. In recent years, the children’s department has operated at a net loss and is expected to continue doing so. Last year’s income statements follow.

Men’s

Women’s

Children’s

Department

Department

Department

Sales

$590,000

$420,000

$155,000

Cost of goods sold

(265,500)

(176,400)

(96,875)

Gross margin Department

324,500

243,600

58,125

manager’s salary Sales

(52,000)

(41,000)

(21,000)

commissions Rent on

(106,200)

(75,600)

(27,900)

store lease

(21,000)

(21,000)

(21,000)

Store utilities

(4,000)

(4,000)

(4,000)

Net income (loss)

$141,300

$102,000

$ (15,775)

Required

a. Determine whether to eliminate the children’s department.

b. Confirm the conclusion you reached in Requirement a by preparing income statements for the company as a whole with and without the children’s department.

c. Eliminating the children’s department would increase space available to display men’s and women’s boots. Suppose management estimates that a wider selection of adult boots would increase the store’s net earnings by $32,000. Would this information affect the decision that you made in Requirement a? Explain your answer.

based on the preceding information recommend whether to eliminate division b support 666896

Effect of activity level and opportunity cost on segment elimination decision

Walter Manufacturing Co. produces and sells specialized equipment used in the petroleum industry. The company is organized into three separate operating branches: Division A, which manufactures and sells heavy equipment; Division B, which manufactures and sells hand tools; and Division C, which makes and sells electric motors. Each division is housed in a separate manufacturing facility. Company headquarters is located in a separate building. In recent years, Division B has been operating at a loss and is expected to continue doing so. Income statements for the three divisions for 2009 follow.

Division A

Division B

Division C

Sales

$3,000,000

$ 850,000

$3,800,000

Less: Cost of goods sold

Unit level manufacturing costs

(1,800,000)

(510,000)

(2,280,000)

Rent on manufacturing facility

(410,000)

(225,000)

(300,000)

Gross margin

790,000

115,000

1,220,000

Less: Operating expenses

Unit level selling and admin. expenses

(187,500)

(42,500)

(237,500)

Division level fixed selling and

admin. expenses

(250,000)

(85,000)

(310,000)

Headquarters facility level costs

(150,000)

(150,000)

(150,000)

Net income (loss)

$ 202,500

$(162,500)

$ 522,500

Required

a. Based on the preceding information, recommend whether to eliminate Division B. Support your answer by preparing companywide income statements before and after eliminating Division B.

b. During 2009, Division B produced and sold 20,000 units of hand tools. Would your recommendation in response to Requirement a change if sales and production increase to 30,000 units in 2010? Support your answer by comparing differential revenue and avoidable costs for Division B, assuming that it sells 30,000 units.

c. Suppose that Walter could sublease Division B’s manufacturing facility for $375,000. Would you operate the division at a production and sales volume of 30,000 units, or would you close it? Support your answer with appropriate computations.

the company s chief accountant recently prepared the following income statement show 666897

Comprehensive problem including special order, outsourcing, and segment elimination decisions

McKay Inc. makes and sells state of the art electronics products. One of its segments produces The Math Machine, an inexpensive calculator. The company’s chief accountant recently prepared the following income statement showing annual revenues and expenses associated with the segment’s operating activities. The relevant range for the production and sale of the calculators is between 30,000 and 60,000 units per year.

Revenue (40,000 units X $16)

$ 640,000

Unit level variable costs

Materials cost (40,000 X $4)

(160,000)

Labor cost (40,000 X $2)

(80,000)

Manufacturing overhead (40,000 X $1)

(40,000)

Shipping and handling (40,000 X $0.50)

(20,000)

Sales commissions (40,000 X $2)

(80,000)

Contribution margin

260,000

Fixed expenses

Advertising costs

(40,000)

Salary of production supervisor

(120,000)

Allocated companywide facility level expenses

(160,000)

Net loss

$ (60,000)

Required (Consider each of the requirements independently.)

a. A large discount store has approached the owner of McKay about buying 5,000 calculators. It would replace The Math Machine’s label with its own logo to avoid affecting McKay’s existing customers. Because the offer was made directly to the owner, no sales commissions on the transaction would be involved, but the discount store is willing to pay only $9.00 per calculator. Based on quantitative factors alone, should McKay accept the special order? Support your answer with appropriate computations. Specifically, by what amount would the special order increase or decrease profitability?

b. McKay has an opportunity to buy the 40,000 calculators it currently makes from a reliable competing manufacturer for $9.80 each. The product meets McKay’s quality standards. McKay could continue to use its own logo, advertising program, and sales force to distribute the products. Should McKay buy the calculators or continue to make them? Support your answer with appropriate computations. Specifically, how much more or less would it cost to buy the calculators than to make them? Would your answer change if the volume of sales were increased to 60,000 units?

c. Because the calculator division is currently operating at a loss, should it be eliminated from the C company’s operations? Support your answer with appropriate computations. Specifically, by what amount would the segment’s elimination increase or decrease profitability?

assume some of the workers being terminated are assembly employees and that they are 666898

Business Application Case Analyzing costs reductions at Dell

On May 31, 2007, Dell, Inc. announced it was making several changes to the way it did business in order . . . to restore competitiveness to the core business, re ignite growth, and build solutions critical to customer needs.” As one of the changes the company

Required

a. Other than the obvious reduction in salary and wages expenses, identify some costs savings Dell might realize by reducing its workforce by 10 percent.

b. Assume some of the workers being terminated are assembly employees and that they are being replaced by new robotic assembly machines. Explain how this might affect Dell’s unit level, batch level, and/or facility level costs.

c. Consider the additional information presented below, which is hypothetical. All dollar amounts are in thousands, unit amounts are not. Assume that Dell decides to eliminate one product line, Delta, for one of its segments that currently produces three products. As a result the following are expected to occur.

(1) The number of units sold for the segment is expected to drop by only 40,000 because of the elimination of Delta, since most customers are expected to purchase an Alpha or Beta product instead. The shift of sales from Delta to Alpha and Beta is expected to be evenly split. In other words, the sales of Alpha and Beta will each increase by 80,000 units.

(2) Rent is paid for the entire production facility, and the space used by Delta cannot be sublet.

(3) Utilities costs are expected to be reduced by $18,000.

(4) The supervisors for Delta will all be terminated. No new supervisors will be hired for Alpha or Beta.

(5) The equipment being used to produce Delta is also used to produce the other two products. The company believes that as a result of eliminating Delta it can eliminate some equipment that has a remaining useful life of five years and a projected salvage value of $20,000. Its current market value is $30,000.

(6) Facility level costs will continue to be allocated between the product lines based on the number of units produced.

Product line Earnings Statements

Annual Costs of Operating Each Product Line

Alpha

Beta

Delta

Total

Sales in units

400,000

400,000

200,000

1,000,000

Sales in dollars

$400,000

$400,000

$200,000

$1,000,000

Unit level costs:

Cost of production

40,000

40,000

22,000

102,000

Sales commissions

5,000

5,000

2,000

12,000

Shipping and handling

9,000

8,000

4,000

21,000

Miscellaneous

3,000

2,000

2,000

7,000

Total unit level costs

57,000

55,000

30,000

142,000

Product level costs:

Supervisors’ salaries

4,000

3,000

1,000

8,000

Facility level costs:

Rent

40,000

40,000

20,000

100,000

Utilities

50,000

50,000

25,000

125,000

Depreciation on equipment

160,000

160,000

80,000

400,000

Allocated companywide expenses

10,000

10,000

5,000

25,000

Total facility level costs

260,000

260,000

130,000

650,000

Total product cost

321,000

318,000

161,000

800,000

Profit on products

$ 79,000

$ 82,000

$ 39,000

$ 200,000

Prepare revised product line earnings statements based on the elimination of Delta. It will be necessary to calculate some per unit data to accomplish this.

maccoa soft a division of zayer software company produces and distributes an automat 666899

Group Assignment Relevance and cost behavior

Maccoa Soft, a division of Zayer Software Company, produces and distributes an automated payroll software system. A contribution margin format income statement for Maccoa Soft for the past year follows.

Revenue (12,000 units X $1,200)

$14,400,000

Unit level variable costs

Product materials cost (12,000 X $60)

(720,000)

Installation labor cost (12,000 X $200)

(2,400,000)

Manufacturing overhead (12,000 X $2)

(24,000)

Shipping and handling (12,000 X $25)

(300,000)

Sales commissions (12,000 X $300)

(3,600,000)

Nonmanufacturing miscellaneous costs (12,000 X $5)

(60,000)

Contribution margin (12,000 X $608)

7,296,000

Fixed costs

Research and development

(2,700,000)

Legal fees to ensure product protection

(780,000)

Advertising costs

(1,200,000)

Rental cost of manufacturing facility

(600,000)

Depreciation on production equipment (zero market value)

(300,000)

Other manufacturing costs (salaries, utilities, etc.)

(744,000)

Division level facility sustaining costs

(1,730,000)

Allocated companywide facility level costs

(1,650,000)

Net loss

$ (2,408,000)

Required

a. Divide the class into groups and then organize the groups into three sections. Assign Task 1 to the first section, Task 2 to the second section, and Task 3 to the third section. Each task should be considered independently of the others.

Group Tasks

(1) Assume that Maccoa has excess capacity. The sales staff has identified a large franchise company with 200 outlets that is interested in Maccoa’s software system but is willing to pay only $800 for each system. Ignoring qualitative considerations, should Maccoa accept the special order?

(2) Maccoa has the opportunity to purchase a comparable payroll system from a competing vendor for $600 per system. Ignoring qualitative considerations, should Maccoa outsource producing the software? Maccoa would continue to sell and install the software if the manufacturing activities were outsourced.

(3) Given that Maccoa is generating a loss, should Zayer eliminate it? Would your answer change if Maccoa could increase sales by 1,000 units?

b. Have a representative from each section explain its respective conclusions. Discuss the following.

(1) Representatives from Section 1 should respond to the following: The analysis related to the special order (Task 1) suggests that all variable costs are always relevant. Is this conclusion valid? Explain your answer.

(2) Representatives from Section 2 should respond to the following: With respect to the outsourcing decision, identify a relevant fixed cost and a nonrelevant fixed cost. Discuss the criteria for determining whether a cost is or is not relevant.

(3) Representatives from Section 3 should respond to the following: Why did the segment elimination decision change when the volume of production and sales increased?

using the income statements for 2003 2006 calculate c p s gross profit percentage fo 666900

Research Assignment Using real world data from Colgate Palmolive

Obtain Colgate Palmolive’s (C P) Form 8 K dated December 6, 2004. Companies file an 8 K with the SEC when they want to announce a special event has occurred at their business. As is often the case with Form 8 Ks, C P’s includes as an attachment a press release related to its planned restructuring that was issued the same day the 8 K was filed. You also need to obtain C P’s income statements for 2003, 2004, 2005, and 2006. The easiest way to obtain these income statements is to retrieve either the company’s 2006 and 2005 Form 10 Ks, or its 2006 and 2005 annual reports. From the 2006 annual report or 10 K, you should also read carefully Note 4, Restructuring Activities.” To obtain the Form 10 Ks you can use the EDGAR system following the instructions in Appendix A, or they can be found under the For Investors” link on the company’s corporate. The company’s annual reports are also available on its website.

Required

a. In the Form 8 K, the second paragraph of Item 2.05 Costs Associated with Exit or Disposal Activities,” C P discloses that a charge of $102 million will be incurred as a result of the 17 restructuring projects it is undertaking. Some of the costs described in this paragraph can be considered sunk costs. Identify these and specify a dollar amount that appears to be related to sunk costs.

b. In the third paragraph of Item 2.05 . . .” the company estimates that the total costs eventually incurred for the restructuring will total from $550 to $650 million, after taxes, and that approximately $200 million of these will be incurred in 2005. Based on Note 4 in the company’s 2006 10 K or annual report, what were the actual, after tax restructuring costs incurred in 2005? Based on Note 4, did the company’s 2006 estimate of the total, after tax costs of restructuring differ from the estimate it made in 2004?

c. In the press release section of its 8 K, C P stated that one objective of the restructuring was to increase its gross profit margin. Using the income statements for 2003–2006, calculate C P’s gross profit percentage for each year. Does it appear the company has achieved the goal of increasing its gross profit margin? Show your computations.

assume that you are an accountant hired as a consultant for the department of revenu 666901

Writing Assignment Relevant versus full cost

State law permits the State Department of Revenue to collect taxes for municipal governments that operate within the state’s jurisdiction and allows private companies to collect taxes for municipalities. To promote fairness and to ensure the financial well being of the state, the law dictates that the Department of Revenue must charge municipalities a fee for collection services that is above the cost of providing such services but does not define the term cost. Until recently, Department of Revenue officials have included a proportionate share of all departmental costs such as depreciation on buildings and equipment, supervisory salaries, and other facility level overhead costs when determining the cost of providing collection services, a measurement approach known as full costing. The full costing approach has led to a pricing structure that places the Department of Revenue at a competitive disadvantage relative to private collection companies. Indeed, highly efficient private companies have been able to consistently underbid the Revenue Department for municipal customers. As a result, it has lost 30 percent of its municipal collection business over the last two years. The inability to be price competitive led the revenue commissioner to hire a consulting firm to evaluate the current practice of determining the cost to provide collection services. The consulting firm concluded that the cost to provide collection services should be limited to the relevant costs associated with providing those services, defined as the difference between the costs that would be incurred if the services were provided and the costs that would be incurred if the services were not provided. According to this definition, the costs of depreciation, supervisory salaries, and other facility level overhead costs are not included because they are the same regardless of whether the Department of Revenue provides collection services to municipalities. The Revenue Department adopted the relevant cost approach and immediately reduced the price it charges municipalities to collect their taxes and rapidly recovered the collection business it had lost. Indeed, several of the private collection companies were forced into bankruptcy. The private companies joined together and filed suit against the Revenue Department, charging that the new definition of cost violates the intent of the law.

Required

a. Assume that you are an accountant hired as a consultant for the private companies. Write a brief memo explaining why it is inappropriate to limit the definition of the costs of providing collection services to relevant costs.

b. Assume that you are an accountant hired as a consultant for the Department of Revenue. Write a brief memo explaining why it is appropriate to limit the definition of the costs of providing collection services to relevant costs.

c. Speculate on how the matter will be resolved.

explain the difference between pro forma financial statements and the financial stat 666904

The Getaway Gift Company operates a chain of small gift shops that are located in prime vacation towns. Getaway is considering opening a new store on January 1, 2010. Getaway’s president recently attended a business seminar that explained how formal budgets could be useful in judging the new store’s likelihood of succeeding. Assume you are the company’s accountant. The president has asked you to explain the budgeting process and to provide sample reports that show the new store’s operating expectations for the first three months (January, February, and March). Respond to the following specific requirements.

Required

a. List the operating budgets and schedules included in a master budget.

b. Explain the difference between pro forma financial statements and the financial statements presented in a company’s annual reports to shareholders.

c. Prepare a sample sales budget and a schedule of expected cash receipts using the following assumptions. Getaway estimates January sales will be $400,000 of which $100,000 will be cash and $300,000 will be credit. The ratio of cash sales to sales on account is expected to remain constant over the three month period. The company expects sales to increase 10 percent per month. The company expects to collect 100 percent of the accounts receivable generated by credit sales in the month following the sale. Use this information to determine the amount of accounts receivable that Getaway would report on the March 31 pro forma balance sheet and the amount of sales it would report on the first quarter pro forma income statement.

d. Prepare a sample inventory purchases budget using the following assumptions. Cost of goods sold is 60 percent of sales. The company desires to maintain a minimum ending inventory equal to 25 percent of the following month’s cost of goods sold. Getaway makes all inventory purchases on account. The company pays 70 percent of accounts payable in the month of purchase. It pays the remaining 30 percent in the following month. Prepare a schedule of expected cash payments for inventory purchases. Use this information to determine the amount of cost of goods sold Getaway would report on the first quarter pro forma income statement and the amounts of ending inventory and accounts payable it would report on the March 31 pro forma balance sheet.

write a short report suggesting how the budgeting process could be improved 666907

Budget responsibility

Candice Hargrove, the accountant, is a perfectionist. No one can do the job as well as she can. Indeed, she has found budget information provided by the various departments to be worthless. She must change everything they give her. She has to admit that her estimates have not always been accurate, but she shudders to think of what would happen if she used the information supplied by the marketing and operating departments. No one seems to care about accuracy. Indeed, some of the marketing staff have even become insulting. When Ms. Hargrove confronted one of the salesmen with the fact that he was behind in meeting his budgeted sales forecast, he responded by saying, They’re your numbers. Why don’t you go out and make the sales? It’s a heck of a lot easier to sit there in your office and make up numbers than it is to get out and get the real work done.” Ms. Hargrove reported the incident, but, of course, nothing was done about it.

Required

Write a short report suggesting how the budgeting process could be improved.

determine the amount of sales revenue camtech will report on its second quarter pro 666908

Preparing a sales budget

Camtech, which expects to start operations on January 1, 2008, will sell digital cameras in shopping malls. Camtech has budgeted sales as indicated in the following table. The company expects a 10 percent increase in sales per month for February and March. The ratio of cash sales to sales on account will remain stable from January through March.

Sales

January

February

March

Cash sales

$ 50,000

?

?

Sales on account

80,000

?

?

Total budgeted sales

$130,000

?

?

Required

a. Complete the sales budget by filling in the missing amounts.

b. Determine the amount of sales revenue Camtech will report on its second quarter pro forma income statement.

complete the schedule of cash receipts by filling in the missing amounts 666909

Preparing a schedule of cash receipts

The budget director of Camila’s Florist has prepared the following sales budget. The company had $200,000 in accounts receivable on July 1. Camila’s Florist normally collects 100 percent of accounts receivable in the month following the month of sale.

Sales

July

August

September

Sales Budget

Cash sales

60,000

$72,000

$86,400

Sales on account

$90,000

108,000

129,600

Total budgeted sales

$150,000

$180,000

$216,000

Schedule of Cash Receipts

Current cash sales

?

?

?

Plus collections from accounts receivable

?

?

?

Total budgeted collections

$270,000

$162,000

$194,400

Required

a. Complete the schedule of cash receipts by filling in the missing amounts.

b. Determine the amount of accounts receivable the company will report on its third quarter pro forma balance sheet.

prepare an income statement balance sheet and statement of cash flows for mazzel com 666853

Service versus manufacturing companies

Mazzel Company began operations on January 1, 2008, by issuing common stock for $33,000 cash. During 2008, Mazzel received $39,000 cash from revenue and incurred costs that required $66,000 of cash payments.

Required

Prepare an income statement, balance sheet, and statement of cash flows for Mazzel Company for 2008, under each of the following independent scenarios.

a. Mazzel is a promoter of rock concerts. The $66,000 was paid to provide a rock concert that produced the revenue.

b. Mazzel is in the car rental business. The $66,000 was paid to purchase automobiles. The automobiles were purchased on January 1, 2008, had four year useful lives and no expected salvage value. Mazzel uses straight line depreciation. The revenue was generated by leasing the automobiles.

c. Mazzel is a manufacturing company. The $66,000 was paid to purchase the following items.

(1) Paid $5,900 cash to purchase materials that were used to make products during the year.

(2) Paid $25,000 cash for wages of factory workers who made products during the year.

(3) Paid $2,100 cash for salaries of sales and administrative employees.

(4) Paid $33,000 cash to purchase manufacturing equipment. The equipment was used solely to make products. It had a three year life and a $7,200 salvage value. The company uses straight line depreciation.

(5) During 2008, Mazzel started and completed 2,000 units of product. The revenue was earned when Mazzel sold 1,300 units of product to its customers.

d. Refer to Requirement c. Could Mazzel determine the actual cost of making the 500th unit of product? How likely is it that the actual cost of the 500th unit of product was exactly the same as the cost of producing the 501st unit of product? Explain why management may be more interested in average cost than in actual cost.

assume that ymc provides an incentive bonus to the cfo who is a cma the bonus is equ 666854

Importance of cost classification and ethics

Young Manufacturing Company (YMC) was started when it acquired $40,000 by issuing common stock. During the first year of operations, the company incurred specifically identifiable product costs (materials, labor, and overhead) amounting to $24,000. YMC also incurred $16,000 of engineering design and planning costs. There was a debate regarding how the design and planning costs should be classified. Advocates of Option 1 believe that the costs should be classified as upstream general, selling, and administrative costs. Advocates of Option 2 believe it is more appropriate to classify the design and planning costs as product costs. During the year, YMC made 4,000 units of product and sold 3,000 units at a price of $24 each. All transactions were cash transactions.

Required

a. Prepare an income statement, balance sheet, and statement of cash flows under each of the two options.

b. Assume that YMC provides an incentive bonus to the CFO who is a CMA. The bonus is equal to 13 percent of net income. Compute the amount of the bonus under each of the two options. Identify the option that provides the CFO with the higher bonus.

c. Assume the CFO knows that the design and planning costs are upstream costs that must be recognized as general, selling, and administrative expenses (Option 1). Even so, the CFO convinces management to classify the upstream costs as product cost in order to increase his bonus. Identify two principles in the Statement of Ethical Professional Practice that are violated by the CFO’s behavior.

prepare an income statement assuming that 95 students enroll in a course determine t 666856

Using JIT to minimize waste and lost opportunity

CMA Review Inc. provides review courses for students studying to take the CMA exam. The cost of textbooks is included in the registration fee. Text material requires constant updating and is useful for only one course. To minimize printing costs and ensure availability of books on the first day of class, CMA Review has books printed and delivered to its offices two weeks in advance of the first class. To ensure that enough books are available, CMA Review normally orders 10 percent more than expected enrollment. Usually there is an oversupply of books that is thrown away. However, demand occasionally exceeds expectations by more than 10 percent and there are too few books available for student use. CMA Review had been forced to turn away students because of a lack of textbooks. CMA Review expects to enroll approximately 100 students per course. The tuition fee is $800 per student. The cost of teachers is $25,000 per course, textbooks cost $60 each, and other operating expenses are estimated to be $35,000 per course.

Required

a. Prepare an income statement, assuming that 95 students enroll in a course. Determine the cost of waste associated with unused books.

b. Prepare an income statement, assuming that 115 students attempt to enroll in the course. Note that five students are turned away because of too few textbooks. Determine the amount of lost profit resulting from the inability to serve the five additional students.

c. Suppose that textbooks can be produced through a high speed copying process that permits delivery just in time for class to start. The cost of books made using this process, however, is $65 each. Assume that all books must be made using the same production process. In other words, CMA Review cannot order some of the books using the regular copy process and the rest using the high speed process. Prepare an income statement under the JIT system assuming that 95 students enroll in a course. Compare the income statement under JIT with the income statement prepared in Requirement a. Comment on how the JIT system would affect profitability.

d. Assume the same facts as in Requirement c with respect to a JIT system that enables immediate delivery of books at a cost of $65 each. Prepare an income statement under the JIT system, assuming that 115 students enroll in a course. Compare the income statement under JIT with the income statement prepared in Requirement b. Comment on how the JIT system would affect profitability.

e. Discuss the possible effect of the JIT system on the level of customer satisfaction.

identify the place in the chain where palmer company is exercising its opportunity t 666857

Value chain analysis

Palmer Company invented a new process for manufacturing ice cream. The ingredients are mixed in high tech machinery that forms the product into small round beads. Like a bag of balls, the ice cream beads are surrounded by air pockets in packages. This design has numerous advantages. First, each bite of ice cream melts quickly in a person’s mouth, creating a more flavorful sensation when compared to ordinary ice cream. Also, the air pockets mean that a typical serving includes a smaller amount of ice cream. This not only reduces materials cost but also provides the consumer with a low calorie snack. A cup appears full of ice cream, but it is really half full of air. The consumer eats only half the ingredients that are contained in a typical cup of blended ice cream. Finally, the texture of the ice cream makes scooping it out of a large container easy. The frustration of trying to get a spoon into a rock solid package of blended ice cream has been eliminated. Palmer Company named the new product Sonic Cream. Like many other ice cream producers, Palmer Company purchases its raw materials from a food wholesaler. The ingredients are mixed in Palmer’s manufacturing plant. The packages of finished product are distributed to privately owned franchise ice cream shops that sell Sonic Cream directly to the public. Palmer provides national advertising and is responsible for all research and development costs associated with making new flavors of Sonic Cream.

Required

a. Based on the information provided, draw a comprehensive value chain for Palmer Company that includes its suppliers and customers.

b. Identify the place in the chain where Palmer Company is exercising its opportunity to create added value beyond that currently being provided by its competitors.

explain why a manager of an individual costco store needs different kinds of informa 666858

Business Applications Case Financial versus managerial accounting

An article in the April 12, 2004, edition of BusinessWeek, The Costco Way—Higher Wages Mean Higher Profits,” compared Costco Wholesale Corporation data with Wal Mart’s Sam’s Club data. The tables below present some of the data used to support the article’s claim.

How Costco Spends More on Employees

Costco

Sam’s Club

Average hourly wage rate

$15.97

$11.53

Employees covered by a health care plan

82%

47%

Average annual health care costs per employee

$5,735

$3,500

Employees covered by a retirement plan

91%

64%

Average annual retirement costs per employee

$1,330

$747

Benefits to Costco from Spending More on Employees

Costco

Sam’s Club

Annual employee turnover

6%

21%

Labor and overhead cost as a percent of sales

9.8%

17%

Annual sales per square foot

$795

$516

Annual profit per employee

$13,647

$11,039

Required

a. Is the information in the tables above best described as primarily financial accounting data or managerial accounting data in nature? Explain.

b. Provide additional examples of managerial and financial accounting information that could apply to Costco.

c. Explain why a manager of an individual Costco store needs different kinds of information than someone who is considering lending the company money or investing in its common stock.

divide the class into groups of four or five students per group and then organize th 666859

Group Assignment Product versus upstream and downstream costs

Victor Holt, the accounting manager of Sexton Inc., gathered the following information for 2006. Some of it can be used to construct an income statement for 2006. Ignore items that do not appear on an income statement. Some computations may be required. For example, the cost of manufacturing equipment would not appear on the income statement. However, the cost of manufacturing equipment is needed to compute the amount of depreciation. All units of product were started and completed in 2006.

1. Issued $864,000 of common stock.

2. Paid engineers in the product design department $10,000 for salaries that were accrued at the end of the previous year.

3. Incurred advertising expenses of $70,000.

4. Paid $720,000 for materials used to manufacture the company’s product.

5. Incurred utility costs of $160,000. These costs were allocated to different departments on the basis of square footage of floor space. Mr. Holt identified three departments and determined the square footage of floor space for each department to be as shown in the table below.

Department

Square Footage

Research and development

10,000

Manufacturing

60,000

Selling and administrative

30,000

Total

100,000

6. Paid $880,000 for wages of production workers.

7. Paid cash of $658,000 for salaries of administrative personnel. There was $16,000 of accrued salaries owed to administrative personnel at the end of 2006. There was no beginning balance in the Salaries Payable account for administrative personnel.

8. Purchased manufacturing equipment two years ago at a cost of $10,000,000. The equipment had an eight year useful life and a $2,000,000 salvage value.

9. Paid $390,000 cash to engineers in the product design department.

10. Paid a $258,000 cash dividend to owners.

11. Paid $80,000 to set up manufacturing equipment for production.

12. Paid a one time $186,000 restructuring cost to redesign the production process to implement a just in time inventory system.

13. Prepaid the premium on a new insurance policy covering nonmanufacturing employees. The policy cost $72,000 and had a one year term with an effective starting date of May 1. Four employees work in the research and development department and eight employees in the selling and administrative department. Assume a December 31 closing date.

14. Made 69,400 units of product and sold 60,000 units at a price of $70 each.

Required

a. Divide the class into groups of four or five students per group, and then organize the groups into three sections. Assign Task 1 to the first section of groups, Task 2 to the second section of groups, and Task 3 to the third section of groups.

Group Tasks

(1) Identify the items that are classified as product costs and determine the amount of cost of goods sold reported on the 2006 income statement.

(2) Identify the items that are classified as upstream costs and determine the amount of upstream cost expensed on the 2006 income statement.

(3) Identify the items that are classified as downstream costs and determine the amount of downstream cost expensed on the 2006 income statement.

b. Have the class construct an income statement in the following manner. Select a member of one of the groups assigned the first group task identifying the product costs. Have that person go to the board and list the costs included in the determination of cost of goods sold. Anyone in the other groups who disagrees with one of the classifications provided by the person at the board should voice an objection and explain why the item should be classified differently. The instructor should lead the class to a consensus on the disputed items. After the amount of cost of goods sold is determined, the student at the board constructs the part of the income statement showing the determination of gross margin. The exercise continues in a similar fashion with representatives from the other sections explaining the composition of the upstream and downstream costs. These items are added to the income statement started by the first group representative. The final result is a completed income statement.

based on this information indicate whether ms emerson believes the number of units o 666862

Ethical Dilemma Product cost versus selling and administrative expense

Eddie Emerson is a proud woman with a problem. Her daughter has been accepted into a prestigious law school. While Ms. Emerson beams with pride, she is worried sick about how to pay for the school; she is a single parent who has to support herself and her three children. She had to go heavily into debt to finance her own education. Even though she now has a good job, family needs have continued to outpace her income and her debt burden is staggering. She knows she will be unable to borrow the money needed for her daughter’s law school. Ms. Emerson is the controller of a small manufacturing company. She has just accepted a new job offer. She has not yet told her employer that she will be leaving in a month. She is concerned that her year end incentive bonus may be affected if her boss learns of her plans to leave. She plans to inform the company immediately after receiving the bonus. She knows her behavior is less than honorable, but she believes that she has been underpaid for a long time. Her boss, a relative of the company’s owner, makes twice what she makes and does half the work. Why should she care about leaving with a little extra cash? Indeed, she is considering an opportunity to boost the bonus. Ms. Emerson’s bonus is based on a percentage of net income. Her company recently introduced a new product line that required substantial production start up costs. Ms. Emerson is fully aware that GAAP requires these costs to be expensed in the current accounting period, but no one else in the company has the technical expertise to know exactly how the costs should be treated. She is considering misclassifying the start up costs as product costs. If the costs are misclassified, net income will be significantly higher, resulting in a nice boost in her incentive bonus. By the time the auditors discover the misclassification, Ms. Emerson will have moved on to her new job. If the matter is brought to the attention of her new employer, she will simply plead ignorance. Considering her daughter’s needs, Ms. Emerson decides to classify the start up costs as product costs.

Required

a. Based on this information, indicate whether Ms. Emerson believes the number of units of product sold will be equal to, less than, or greater than the number of units made. Write a brief paragraph explaining the logic that supports your answer.

b. Explain how the misclassification could mislead an investor or creditor regarding the company’s financial condition.

c. Identify the factors that contributed to the breach of ethical conduct. When constructing your answer, you may want to refer to the section Common Features of Criminal and Ethical Misconduct” in Chapter 2 of this text.

d. Review the Statement of Ethical Professional Practice shown in Exhibit 10.14 and identify at least two principles that Ms. Emerson’s misclassification of the start up costs violated.

the supervisor manages the production of sprinkler heads that are used in water irri 666863

1. Aqua, Inc., makes statues for use in fountains. On January 1, 2008, the company paid $13,500 for a mold to make a particular type of statue. The mold had an expected useful life of four years and a salvage value of $1,500. On January 1, 2010, the mold had a market value of $3,000 and a salvage value of $1,200. The expected useful life did not change. What is the relevant cost of using the mold during 2010?

2. Addison Manufacturing Company pays a production supervisor a salary of $48,000 per year. The supervisor manages the production of sprinkler heads that are used in water irrigation systems. Should the production supervisor’s salary be considered a relevant cost to a special order decision? Should the production supervisor’s salary be considered a relevant cost to an outsourcing decision?

first determine what relevant costs premier will incur if it keeps the old machine 666865

First determine what relevant costs Premier will incur if it keeps the old machine.

1. The original cost ($90,000), current book value ($57,000), accumulated depreciation ($33,000), and annual depreciation expense ($11,000) are different measures of a cost that was incurred in a prior period. They represent irrelevant sunk costs.

2. The $14,000 market value represents the current sacrifice Premier must make if it keeps using the existing machine. In other words, if Premier does not keep the machine, it can sell it for $14,000. In economic terms, forgoing the opportunity to sell the machine costs as much as buying it. The opportunity cost is therefore relevant to the replacement decision.

3. The salvage value of the old machine reduces the opportunity cost. Premier can sell the old machine now for $14,000 or use it for five more years and then sell it for $2,000. The opportunity cost of using the old machine for five more years is therefore $12,000 ($14,000 $2,000).

4. Because the $45,000 ($9,000 x 5) of operating expenses will be incurred if the old machine is used but can be avoided if it is replaced, the operating expenses are relevant costs. Next, determine what relevant costs will be incurred if Premier purchases and uses the new machine.

1. The cost of the new machine represents a future economic sacrifice Premier must incur if it buys the new machine. It is a relevant cost.

2. The salvage value reduces the cost of purchasing the new machine. Part ($4,000) of the $29,000 cost of the new machine will be recovered at the end of five years. The relevant cost of purchasing the new machine is $25,000 ($29,000 $4,000).

3. The $22,500 ($4,500 x 5) of operating expenses will be incurred if the new machine is purchased; it can be avoided if the new machine is not purchased. The operating expenses are relevant costs.

The relevant costs for the two machines are summarized here.

Old Machine

New Machine

Opportunity cost

$14,000

Cost of the new machine

$29,000

Salvage value

(2,000)

Salvage value

(4,000)

Operating expenses

45,000

Operating expenses

22,500

Total

$57,000

Total

$47,500

The analysis suggests that Premier should acquire the new machine because buying it produces the lower relevant cost. The $57,000 cost of using the old machine can be avoided by incurring the $47,500 cost of acquiring and using the new machine. Over the five year period, Premier would save $9,500 ($57,000 $47,500) by purchasing the new machine. One caution: this analysis ignores income tax effects and the time value of money, which are explained later. The discussion in this chapter focuses on identifying and using relevant costs in decision making.

flying high inc fhi is a division of the master toy company fhi makes remote control 666866

Flying High Inc. (FHI) is a division of The Master Toy Company. FHI makes remote controlled airplanes. During 2009, FHI incurred the following costs in the process of making 5,000 planes.

Unit level materials costs (5,000 units @ $80)

$ 400,000

Unit level labor costs (5,000 units @ $90)

450,000

Unit level overhead costs (5,000 @ $70)

350,000

Depreciation cost on manufacturing equipment*

50,000

Other manufacturing overhead†

140,000

Inventory holding costs

240,000

Allocated portion of The Master Toy Company’s facility level costs

600,000

Total costs

$2,230,000

Required

a. FHI uses a cost plus pricing strategy. FHI sets its price at product cost plus $100. Determine the price that FHI should charge for its remote controlled airplanes.

b. Assume that a potential customer that operates a chain of high end toy stores has approached FHI. A buyer for this chain has offered to purchase 1,000 planes from FHI at a price of $275 each. Ignoring qualitative considerations, should FHI accept or reject the order?

c. FHI has the opportunity to purchase the planes from Arland Manufacturing Company for $325 each. Arland maintains adequate inventories so that it can supply its customers with planes on demand. Should FHI accept the opportunity to outsource the making of its planes?

d. Use the contribution margin format to prepare an income statement based on historical cost data. Prepare a second income statement that reflects the relevant cost data that Master Toy should consider in a segment elimination decision. Based on a comparison of these two statements, indicate whether Master Toy should eliminate the FHI division.

e. FHI is considering replacing the equipment it currently uses to manufacture its planes. It could purchase replacement equipment for $480,000 that has an expected useful life of four years and a salvage value of $40,000. The new equipment would increase productivity substantially, reducing unit level labor costs by 20 percent. Assume that FHI would maintain its production and sales at 5,000 planes per year. Prepare a schedule that shows the relevant costs of operating the old equipment versus the costs of operating the new equipment. Should FHI replace the equipment?

identify each cost as being relevant or irrelevant to ms hardman s decision and indi 666872

Distinction between relevance and cost behavior

Joyce Hardman is trying to decide which of two different kinds of candy to sell in her retail candy store. One type is a name brand candy that will practically sell itself. The other candy is cheaper to purchase but does not carry an identifiable brand name. Ms. Hardman believes that she will have to incur significant advertising costs to sell this candy. Several cost items for the two types of candy are as follows.

Brandless Candy

Name Brand Candy

Cost per box

$ 4.00

Cost per box

$ 6.00

Sales commissions per box

0.50

Sales commissions per box

1.00

Rent of display space

1,500.00

Rent of display space

1,500.00

Advertising

3,000.00

Advertising

2,000.00

Required

Identify each cost as being relevant or irrelevant to Ms. Hardman’s decision and indicate whether it is fixed or variable relative to the number of boxes sold.

identify the fixed costs and determine the amount of fixed cost for each product 666874

Distinction between avoidable costs and cost behavior

Irby Company makes fine jewelry that it sells to department stores throughout the United States. Irby is trying to decide which of two bracelets to manufacture. Irby has a labor contract that prohibits the company from laying off workers freely. Cost data pertaining to the two choices follow.

Bracelet A

Bracelet B

Cost of materials per unit

$ 18

$ 37

Cost of labor per unit

32

32

Advertising cost per year

8,000

6,000

Annual depreciation on existing equip.

5,000

4,000

Required

a. Identify the fixed costs and determine the amount of fixed cost for each product.

b. Identify the variable costs and determine the amount of variable cost per unit for each product.

c. Identify the avoidable costs and determine the amount of avoidable cost for each product.

roddam company manufactures a personal computer designed for use in schools and mark 666876

Special order decision

Roddam Company manufactures a personal computer designed for use in schools and markets it under its own label. Roddam has the capacity to produce 20,000 units a year but is currently producing and selling only 15,000 units a year. The computer’s normal selling price is $1,400 per unit with no volume discounts. The unit level costs of the computer’s production are $550 for direct materials, $150 for direct labor, and $200 for indirect unit level manufacturing costs. The total product and facility level costs incurred by Roddam during the year are expected to be $2,100,000 and $700,000, respectively. Assume that Roddam receives a special order to produce and sell 3,000 computers at $1,000 each.

Required

Should Roddam accept or reject the special order? Support your answer with appropriate computations.

payne has excess capacity and receives a special order for 4 000 clocks for 13 each 666877

1. Identifying qualitative factors for a special order decision

Required

Describe the qualitative factors that Roddam should consider before accepting the special order described in Exercise 13 5.

2. Using the contribution margin approach for a special order decision

Payne Company, which produces and sells a small digital clock, bases its pricing strategy on a 35 percent markup on total cost. Based on annual production costs for 10,000 units of product, computations for the sales price per clock follow.

Unit level costs Fixed costs

$100,000 50,000

Total cost (a)

150,000

Markup (a x 0.35)

52,500

Total sales (b)

$202,500

Sales price per unit (b ÷ 10,000)

$20.25

Required

a. Payne has excess capacity and receives a special order for 4,000 clocks for $13 each. Calculate the contribution margin per unit; based on it, should Payne accept the special order?

b. Support your answer by preparing a contribution margin income statement for the special order.

would the price computed in requirement a change if production increased to 18 750 u 666878

Establishing price for an outsourcing decision

Green Lawn Inc. makes and sells lawn mowers for which it currently makes the engines. It has an opportunity to purchase the engines from a reliable manufacturer. The annual costs of making the engines are shown here.

Cost of materials (15,000 units x $21)

$315,000

Labor (15,000 units x $26)

390,000

Depreciation on manufacturing equipment*

42,000

Salary of supervisor of engine production

85,000

Rental cost of equipment used to make engines

23,000

Allocated portion of corporate level facility sustaining costs

41,500

Total cost to make 15,000 engines

$896,500

Required

a. Determine the maximum price per unit that Green Lawn would be willing to pay for the engines.

b. Would the price computed in Requirement a change if production increased to 18,750 units? Support your answer with appropriate computations.

determine the change in net income reece would experience if it decides to make the 666879

Outsourcing decision with qualitative factors

Reece Corporation, which makes and sells 79,400 radios annually, currently purchases the radio speakers it uses for $10 each. Each radio uses one speaker. The company has idle capacity and is considering the possibility of making the speakers that it needs. Reece estimates that the cost of materials and labor needed to make speakers would be a total of $8 for each speaker. In addition, the costs of supervisory salaries, rent, and other manufacturing costs would be $166,740. Allocated facility level costs would be $99,600.

Required

a. Determine the change in net income Reece would experience if it decides to make the speakers.

b. Discuss the qualitative factors that Reece should consider.

mulga electronics currently produces the shipping containers it uses to deliver the 666880

Outsourcing decision affected by opportunity costs

Mulga Electronics currently produces the shipping containers it uses to deliver the electronics products it sells. The monthly cost of producing 9,000 containers follows.

Unit level materials

$ 5,000

Unit level labor

5,500

Unit level overhead

3,500

Product level costs*

9,000

Allocated facility level costs

22,000

Omar Container Company has offered to sell comparable containers to Mulga for $2.25 each.

Required

a. Should Mulga continue to make the containers? Support your answer with appropriate computations.

b. Mulga could lease the space it currently uses in the manufacturing process. If leasing would produce $9,000 per month, would your answer to Requirement a be different? Explain.

based solely on financial considerations should mr cassaway sell the taxi and accept 666882

Opportunity costs

John Cassaway owns his own taxi, which he bought an $18,000 permit to operate two years ago. Mr. Cassaway earns $36,000 a year operating as an independent but has the opportunity to sell the taxi and permit for $73,000 and take a position as dispatcher for Sartino Taxi Co. The dispatcher position pays $31,000 a year for a 40 hour week. Driving his own taxi, Mr. Cassaway works approximately 55 hours per week. If he sells his business, he will invest the $73,000 and can earn a 10 percent return.

Required

a. Determine the opportunity cost of owning and operating the independent business.

b. Based solely on financial considerations, should Mr. Cassaway sell the taxi and accept the position as dispatcher?

c. Discuss the qualitative as well as quantitative factors that Mr. Cassaway should consider.

prepare comparative income statements for the company as a whole under two alternati 666883

Segment elimination decision

Lockett Company operates three segments. Income statements for the segments imply that profitability could be improved if Segment A were eliminated.

LOCKETT COMPANY
Income Statements for the Year 2009

Segment

A

B

C

Sales

$191,000

$251,000

$347,000

Cost of goods sold

(147,000)

(99,000)

(200,000)

Sales commissions

(19,000)

(36,000)

(24,000)

Contribution margin

25,000

116,000

123,000

General fixed oper. exp. (allocation of president’s salary)

(45,000)

(51,000)

(46,000)

Advertising expense (specific to individual divisions)

(4,000)

(8,000)

0

Net income

($24,000)

$57,000

$77,000

Required

a. Explain the effect on profitability if Segment A is eliminated.

b. Prepare comparative income statements for the company as a whole under two alternatives: (1) the retention of Segment A and (2) the elimination of Segment A.

haygood transport company divides its operations into four divisions a recent income 666884

Segment elimination decision

Haygood Transport Company divides its operations into four divisions. A recent income statement for Stancil Division follows.

HAYGOOD TRANSPORT COMPANY

Stancil Division

Income Statement for the Year 2009

Revenue

$ 650,000

Salaries for drivers

(430,000)

Fuel expenses

(81,000)

Insurance

(113,000)

Division level facility sustaining costs

(60,000)

Companywide facility sustaining costs

(120,000)

Net loss

$(154,000)

Required

a. Should Stancil Division be eliminated? Support your answer by explaining how the division’s elimination would affect the net income of the company as a whole. By how much would companywide income increase or decrease?

b. Assume that Stancil Division is able to increase its revenue to $700,000 by raising its prices. Would this change the decision you made in Requirement a? Determine the amount of the increase or decrease that would occur in companywide net income if the segment were eliminated if revenue were $700,000.

c. What is the minimum amount of revenue required to justify continuing the operation of Stancil Division?

new product opportunities and transfer pricing plevna manufacturing makes and distri 666826

New product opportunities and transfer pricing Plevna Manufacturing makes and distributes small prefabricated homes in kits. The kits contain all pieces needed to assemble the home. All that is required is that the builder erect the home on a foundation.

Plevna Manufacturing is organized into two divisions: the manufacturing division and the sales division. Each division is evaluated on the basis of its reported profits. The transfer price between the manufacturing division, where the kits are made, and the selling division, which sells the kits, is variable cost plus 10%, a total of about $33,000. The selling price per kit is about $40,000, and selling and distribution costs are about $5,000 per home kit.

The total costs that do not vary in proportion with volume at Plevna Manufacturing amount to about $2,000,000 per year: about $1,500,000 in manufacturing and about $500,000 in the selling division. The company is currently operating at capacity, which is dictated by the machinery in the manufacturing division. Each kit requires about 10 hours of machine time, and the total available machine time is 5,000 hours per year. Plevna Manufacturing is making and selling about 500 kits per year. Increasing the plant capacity is not a viable option in the foreseeable future.

Willie Scott is the firm’s salesperson. Willie has been approached a number of times recently by people wanting to buy cottages to erect on recreational properties. The cottages would be made by modifying the existing home product. The modification process would begin with a completed home kit. The manufacturing division would then incur additional materials and labor costs of $3,000 and three hours of machine time to convert a home kit into a cottage kit.

Willie is proposing that the company split the sales division into two divisions: home sales and cottage sales. The new divisional structure would have no effect on existing administrative, personnel, or selling costs.

Required

Suppose the new division is created. Discuss the issues in choosing a transfer price in this situation. What transfer price for each of the two products, home and cottage kits, would you recommend and why? (If you feel that the appropriate transfer price for each product can be within a range, specify the range.)

decision making with return on investment you are the controller of a chain of dry c 666827

Decision making with return on investment You are the controller of a chain of dry cleaning establishments. You are computing the return on investment for each outlet.

Outlet A, located in a city core, reported a net profit of $130,000. The land on which Outlet Ais located was essentially rural when it was purchased for $100,000. Since then, the city has expanded, and the land is now located in the population center. Comparable undeveloped land in the immediate area of the outlet is worth $2,000,000. The net book value of the outlet building and equipment is $400,000. The replacement cost of the building and equipment is $1,200,000. If the outlet building, equipment, and land were sold as a going concern, the sale price would be $1,500,000. It would cost $250,000 to demolish the building and clear the property for commercial development.

Required

(a) What is the return on this investment?

(b) How would you decide whether this outlet should continue to be operated, sold as a going concern, or demolished and the land sold?

segment analysis commitment and consumption of activity resources shellie s lawn and 666828

Segment analysis, commitment, and consumption of activity resources Shellie’s Lawn and Gardening performs various lawn and garden maintenance activities, including lawn mowing, tree and shrub pruning, fertilizing, and treating for pests. Unlike other lawn and garden businesses in the city, Shellie also specializes in landscape design and planting. Shellie is pleased that her design specialty is so much in demand. However, she is concerned because profits have been falling, even though sales have been growing during the past few years. In an effort to better understand why profits are falling, Shellie prepared the following product line income statement:

SHELLIE’S LAWN AND GARDEN PRODUCT LINE INCOME STATEMENT

 

LAWN MOWING

LAYOUT DESIGN

OTHER MAINTENANCE

TOTAL

Revenues

$287,500

$218,750

$312,500

$818,750

Direct costs

$156,250

$70,000

$181,250

$407,500

Allocated costs

$131,679

$100,191

$143,130

$375,000

Profit

_$429

$48,559

_$11,880

$36,250

The lawn mowing business involves mowing lawns and trimming edges for customers who generally sign up for the season and pay a flat fee based on the surface area mowed and trimmed. The layout design business involves both designing a garden and lawn layout and installing the design. Other maintenance includes tree and shrub pruning and application of chemicals. The direct costs for each line of business are the costs of the materials and wages of the people who work in that line of business. The remaining costs consist mainly of equipment costs but also include office costs. After some deliberation, Shellie decided to allocate the remaining costs of $375,000 on the basis of revenue, reasoning that revenue is a measure of equipment use.

Required

(a) Based on this product line income statement, which business is Shellie likely to focus her efforts on? What is the likely result?

A further analysis of the allocated costs produced the information in the following table. General business costs are $50,000, and the remaining $325,000 represents equipment costs. The trucks are shared equally by all segments, but the other equipment is used by only the indicated segment.

SHELLIE’S LAWN AND GARDEN RESOURCE USE INFORMATION

 

COST

PRACTICAL CAPACITY HOURS

COST DRIVER RATE PER HOUR

HOURS USED

Trucks and related costs

$50,000

800

$62.50

600

Lawn mowing equipment

37,500

1,500

25.00

1,200

Layout design equipment

150,000

400

375.00

400

Other maintenance equipment

87,500

700

125.00

500

 

$325,000

 

 

 

           

(b) For each equipment category in the table above, calculate the cost allocated to Shellie’s service orders based on the number of hours used, and calculate the cost associated with unused capacity.

(c) Prepare a new product line income statement with a column for each product line and a column for the total company. For each product line, include the cost of used equipment capacity and the cost of unused capacity that is attributable only to that product line.

(d) Based on your new product line income statement, what advice do you have for Shellie? How does this advice compare to your response in part a?

choosing an organization structure you are a senior manager responsible for overall 666829

Choosing an organization structure You are a senior manager responsible for overall company operations in a large courier company. Your company has 106 regional offices (terminals) scattered around the country and a main office (hub) located in the geographical center of the country. Your operations are strictly domestic. You do not accept international shipments.

The day at each terminal begins with the arrival of packages from the hub. The packages are loaded onto trucks for delivery to customers during the morning hours. In the afternoon, the same trucks pick up packages that are returned to the terminal in late afternoon and then shipped to the hub, where shipments arrive from the terminals into the late evening and are sorted for delivery early the next day for the terminals.

Each terminal in your company is treated as an investment center and prepares individual income statements each month. Each terminal receives 30% of the revenue from packages that it picks up and 30% of the revenue from the packages it delivers. The remaining 40% of the revenue from each transaction goes to the hub. Each terminal accumulates its own costs. All costs related to travel to and from the hub are charged to the hub. The revenue per package is based on size and service type and not the distance that the package travels. (There are two types of service, overnight and ground delivery, which take between one and seven days, depending on the distance traveled.)

All customer service is done through a central service group located in the hub. Customers access this service center through a toll free telephone number. The most common calls to customer service include requests for package pickup, requests to trace an overdue package, and requests for billing information. The company has invested in complex and expensive package tracking equipment that monitors the package’s trip through the system by scanning the bar code placed on every package. The bar code is scanned when the package is picked up, enters the originating terminal, leaves the originating terminal, arrives at the hub, leaves the hub, arrives at the destination terminal, leaves the destination terminal, and is delivered to the customer. All scanning is done by handheld wands that transmit the information to the regional and then central computer.

The major staff functions in each terminal are administrative (accounting, clerical, and executive), marketing (the sales staff), courier (the people who pick up and deliver the shipments and the equipment they use), and operations (the people and equipment who sort packages inside the terminal).

This organization takes customer service very seriously. The revenue for any package that fails to meet the organization’s service commitment to the customer is not assigned to the originating and destination terminals.

All company employees receive a wage and a bonus based on the terminal’s residual income. This system has promoted many debates about the sharing rules for revenues, the inherent inequity of the existing system, and the appropriateness of the revenue share for the hub. Service problems have arisen primarily relating to overdue packages. The terminals believe that most of the service problems relate to mis sorting in the hub, resulting in packages being sent to the wrong terminals.

Required

(a) Explain why you believe an investment center is or is not an appropriate organization design for this company.

(b) Assuming that this organization is committed to the current design, how would you improve it?

(c) Assuming that this organization has decided that the investment center approach is unacceptable, what approach to performance evaluation would you recommend?

assuming the purchased inventory would be available on demand explain how the compan 666833

Tuscan Manufacturing Company makes a unique headset for use with mobile phones. During 2010, its first year of operations, Tuscan experienced the following accounting events. Other than the adjusting entries for depreciation, assume that all transactions are cash transactions.

1. Acquired $850,000 cash from the issue of common stock.

2. Paid $50,000 of research and development costs to develop the headset.

3. Paid $140,000 for the materials used to make headsets, all of which were started and completed during the year.

4. Paid salaries of $82,200 to selling and administrative employees.

5. Paid wages of $224,000 to production workers.

6. Paid $48,000 to purchase furniture used in selling and administrative offices.

7. Recognized depreciation on the office furniture. The furniture, acquired January 1, had an $8,000 estimated salvage value and a four year useful life. The amount of depreciation is computed as ([cost salvage] ÷ useful life). Specifically, ([$48,000 $8,000] ÷ 4 = $10,000).

8. Paid $65,000 to purchase manufacturing equipment.

9. Recognized depreciation on the manufacturing equipment. The equipment, acquired January 1, had a $5,000 estimated salvage value and a three year useful life. The amount of depreciation is computed as ([cost salvage] ÷ useful life). Specifically, ([$65,000 $5,000] ÷ 3 = $20,000).

10. Paid $136,000 for rent and utility costs on the manufacturing facility.

11. Paid $41,000 for inventory holding expenses for completed headsets (rental of warehouse space, salaries of warehouse personnel, and other general storage costs).

12. Tuscan started and completed 20,000 headset units during 2010. The company sold 18,400 headsets at a price of $38 per unit.

13. Compute the average product cost per unit and recognize the appropriate amount of cost of goods sold.

Required

a. Show how these events affect the balance sheet, income statement, and statement of cash flows by recording them in a horizontal financial statements model.

b. Explain why Tuscan’s recognition of cost of goods sold expense had no impact on cash flow.

c. Prepare a formal income statement for the year.

d. Distinguish between the product costs and the upstream and downstream costs that Tuscan incurred.

e. The company president believes that Tuscan could save money by buying the inventory that it currently makes. The warehouse supervisor said that would not be possible because the purchase price of $27 per unit was above the $26 average cost per unit of making the product. Assuming the purchased inventory would be available on demand, explain how the company president could be correct and why the warehouse supervisor could be biased in his assessment of the option to buy the inventory.

indicate whether each of the following is representative of managerial or of financi 666836

Identifying financial versus managerial accounting characteristics

Required

Indicate whether each of the following is representative of managerial or of financial accounting.

a. Information includes economic and nonfinancial data as well as financial data.

b. Information is global and pertains to the company as a whole.

c. Information is provided to insiders including executives, managers, and operators.

d. Information is factual and is characterized by objectivity, reliability, consistency, and accuracy.

e. Information is reported continuously and has a current or future orientation.

f. Information is provided to outsiders including investors, creditors, government agencies, analysts, and reporters.

g. Information is regulated by the SEC, FASB, and other sources of GAAP.

h. Information is based on estimates that are bounded by relevance and timeliness.

i. Information is historically based and usually reported annually.

j. Information is local and pertains to subunits of the organization.

identifying product versus general selling and administrative costs 666837

Identifying product versus general, selling, and administrative costs

Required

Indicate whether each of the following costs should be classified as a product cost or as a general, selling, and administrative cost.

a. Research and development costs incurred to create new drugs for a pharmaceutical company.

b. The cost of secretarial supplies used in a doctor’s office.

c. Depreciation on the office furniture of the company president.

d. Direct materials used in a manufacturing company.

e. Indirect materials used in a manufacturing company.

f. Salaries of employees working in the accounting department.

g. Commissions paid to sales staff.

h. Interest on the mortgage for the company’s corporate headquarters.

i. Indirect labor used to manufacture inventory.

j. Attorney’s fees paid to protect the company from frivolous lawsuits.

use the following format to classify each cost as a product cost or a general sellin 666838

Classifying costs: product or G, S, & A/asset or expense

Required

Use the following format to classify each cost as a product cost or a general, selling, and administrative (G, S, & A) cost. Also indicate whether the cost would be recorded as an asset or an expense. The first item is shown as an example.

Product/

Asset/

Cost Category

G, S, & A

Expense

Production supplies

Product

Asset

Depreciation on administration building

Depreciation on manufacturing equipment

Research and development costs

Cost to set up manufacturing equipment

Utilities used in factory

Cars for sales staff

Distributions to stockholders

General office supplies

Raw materials used in the manufacturing process

Cost to rent office equipment

Wages of production workers

Advertising costs

Promotion costs

identifying effect of product versus general selling and administrative costs on fin 666839

Identifying effect of product versus general, selling, and administrative costs on financial statements

Required

Kohler Industries recognized accrued compensation cost. Use the following model to show how this event would affect the company’s financial statement under the following two assumptions: (1) the compensation is for office personnel and (2) the compensation is for production workers. Use pluses or minuses to show the effect on each element. If an element is not affected, indicate so by placing the letters NA under the appropriate heading.

Assets

=

Liab.

+

Equity

Rev.

Exp.

=

Net Inc.

Cash Flow

1.

2.

using the following horizontal financial statements model indicate how this event af 666840

Identify effect of product versus general, selling, and administrative costs on financial statements

Required

Milby Industries recognized the annual cost of depreciation on December 31, 2010. Using the following horizontal financial statements model, indicate how this event affected the company’s financial statements under the following two assumptions: (1) the depreciation was on office furniture and (2) the depreciation was on manufacturing equipment. Indicate whether the event increases (I), decreases (D), or has no affect (NA) on each element of the financial statements. Also, in the Cash column, indicate whether the cash flow is for operating activities (OA), investing activities (IA), or financing activities (FA). (Note: Show accumulated depreciation as a decrease in the book value of the appropriate asset account.)

Assets

Equity

Event
No.

Cash

+

Inventory

+

Manuf.
Equip.

+

Office
Furn.

=

Com.
Stk.

+

Ret.
Earn.

Rev.

Exp.

=

Net Inc.

Cash Flow

1.

2.

assuming that feldman made 5 000 units of product and sold 4 000 of them during the 666841

Identifying product versus general, selling, and administrative costs

A review of the accounting records of Feldman Manufacturing indicated that the company incurred the following payroll costs during the month of August.

1. Salary of the company president—$100,000.

2. Salary of the vice president of manufacturing—$40,000.

3. Salary of the chief financial officer—$40,000.

4. Salary of the vice president of marketing—$25,000.

5. Salaries of middle managers (department heads, production supervisors) in manufacturing plant—$120,000.

6. Wages of production workers—$3,940,000.

7. Salaries of administrative secretaries—$81,000.

8. Salaries of engineers and other personnel responsible for maintaining production equipment— $125,000.

9. Commissions paid to sales staff—$126,000.

Required

a. What amount of payroll cost would be classified as general, selling, and administrative expense?

b. Assuming that Feldman made 5,000 units of product and sold 4,000 of them during the month of August, determine the amount of payroll cost that would be included in cost of goods sold.

anthony manufacturing company began operations on january 1 during the year it start 666843

Allocating product costs between ending inventory and cost of goods sold

Anthony Manufacturing Company began operations on January 1. During the year, it started and completed 3,400 units of product. The company incurred the following costs.

1. Raw materials purchased and used—$6,300.

2. Wages of production workers—$7,060.

3. Salaries of administrative and sales personnel—$3,990.

4. Depreciation on manufacturing equipment—$8,740.

5. Depreciation on administrative equipment—$3,670.

Anthony sold 2,040 units of product.

Required

a. Determine the total product cost for the year.

b. Determine the total cost of the ending inventory.

c. Determine the total of cost of goods sold.

explain why recognizing depreciation on equipment used in a manufacturing company af 666844

Financial statement effects for manufacturing versus service organizations

The following financial statements model shows the effects of recognizing depreciation in two different circumstances. One circumstance represents recognizing depreciation on a machine used in a factory. The other circumstance recognizes depreciation on computers used in a consulting firm. The effects of each event have been recorded using the letter (I) to represent increase, (D) for decrease, and (NA) for no effect.

Assets

Equity

Event
No.

Cash

+

Inventory

+

Manuf.
Equip.

+

Office
Furn.

=

Com.
Stk.

+

Ret.
Earn.

Rev.

Exp.

=

Net Inc.

Cash Flow

1.

NA

I

D

NA

NA

NA

NA

I

D

NA

2.

NA

I

D

NA

NA

NA

NA

NA

NA

NA

Required

a. Identify the event that represents depreciation on the computers.

b. Explain why recognizing depreciation on equipment used in a manufacturing company affects financial statements differently from recognizing depreciation on equipment used in a service organization.

identifying the effect of product versus general selling and administrative cost on 666845

Identifying the effect of product versus general, selling, and administrative cost on the income statement and statement of cash flows

Each of the following events describes acquiring an asset that requires a year end adjusting entry.

1. Paid $7,000 cash on January 1 to purchase printers to be used for administrative purposes.

The printers had an estimated useful life of three years and a $1,000 salvage value.

2. Paid $7,000 cash on January 1 to purchase manufacturing equipment. The equipment had an estimated useful life of three years and a $1,000 salvage value.

3. Paid $6,000 cash in advance on May 1 for a one year rental contract on administrative offices.

4. Paid $6,000 cash in advance on May 1 for a one year rental contract on manufacturing facilities.

5. Paid $1,000 cash to purchase supplies to be used by the marketing department. At the end of the year, $200 of supplies were still on hand.

6. Paid $1,000 cash to purchase supplies to be used in the manufacturing process. At the end of the year, $200 of supplies were still on hand.

Required

Explain how acquiring the asset and making the adjusting entry affect the amount of net income and the cash flow reported on the year end financial statements. Also, in the Cash Flow column, indicate whether the cash flow is for operating activities (OA), investing activities (IA), or financing activities (FA). Use (NA) for no effect. Assume a December 31 annual closing date. The first event has been recorded as an example. Assume that any products that have been made have not been sold.

Net Income

Cash Flow

Amount of

Amount of

Event No.

Change

Change

1. Purchase of printers

NA

(7,000)

IA

1. Make adjusting entry

(2,000)

NA

why would earwood price the batteries at a level that would generate a loss for the 666846

Upstream and downstream costs

During 2009, Earwood Manufacturing Company incurred $8,000,000 of research and development (R&D) costs to create a long life battery to use in computers. In accordance with FASB standards, the entire R&D cost was recognized as an expense in 2009. Manufacturing costs (direct materials, direct labor, and overhead) were expected to be $22 per unit. Packaging, shipping, and sales commissions were expected to be $8 per unit. Earwood expected to sell 200,000 batteries before new research renders the battery design technologically obsolete. During 2009, Earwood made 21,000 batteries and sold 18,000 of them.

Required

a. Identify the upstream and downstream costs.

b. Determine the 2009 amount of cost of goods sold and the ending inventory balance.

c. Determine the sales price assuming that Earwood desired to earn a profit margin equal to 25 percent of the total cost of developing, making, and distributing the batteries.

d. Prepare an income statement for 2009. Use the sales price developed in Requirement c.

e. Why would Earwood price the batteries at a level that would generate a loss for the 2009 accounting period?

if the school receives actual sales orders for 725 shirts what amount of profit will 666847

Using JIT to minimize waste and lost opportunity

Julie Kent, a teacher at Tingle Middle School, is in charge of ordering the T shirts to be sold for the school’s annual fund raising project. The T shirts are printed with a special Tingle School logo. In some years, the supply of T shirts has been insufficient to satisfy the number of sales orders. In other years, T shirts have been left over. Excess T shirts are normally donated to some charitable organization. T shirts cost the school $5 each and are normally sold for $6 each. Ms. Kent has decided to order 800 shirts.

Required

a. If the school receives actual sales orders for 725 shirts, what amount of profit will the school earn? What is the cost of waste due to excess inventory?

b. If the school receives actual sales orders for 825 shirts, what amount of profit will the school earn? What amount of opportunity cost will the school incur?

c. Explain how a JIT inventory system could maximize profitability by eliminating waste and opportunity cost.

based on the information provided how much of ms gibson s inventory holding cost cou 666848

Using JIT to minimize holding costs

Tubb Pet Supplies purchases its inventory from a variety of suppliers, some of which require a six week lead time before delivery. To ensure that she has a sufficient supply of goods on hand, Ms. Gibson, the owner, must maintain a large supply of inventory. The cost of this inventory averages $42,000. She usually finances the purchase of inventory and pays a 9 percent annual finance charge. Ms. Gibson’s accountant has suggested that she establish a relationship with a single large distributor who can satisfy all of her orders within a two week time period. Given this quick turnaround time, she will be able to reduce her average inventory balance to $8,000. Ms. Gibson also believes that she could save $8,000 per year by reducing phone bills, insurance, and warehouse rental space costs associated with ordering and maintaining the larger level of inventory.

Required

a. Is the new inventory system available to Ms. Gibson a pure or approximate just in time system?

b. Based on the information provided, how much of Ms. Gibson’s inventory holding cost could be eliminated by taking the accountant’s advice?

determine the amount of the ending inventory balance that would appear on the decemb 666850

Product versus general, selling, and administrative costs

Walton Manufacturing Company was started on January 1, 2008, when it acquired $85,000 cash by issuing common stock. Walton immediately purchased office furniture and manufacturing equipment costing $10,000 and $54,000, respectively. The office furniture had a five year useful life and a zero salvage value. The manufacturing equipment had a $4,000 salvage value and an expected useful life of five years. The company paid $11,000 for salaries of administrative personnel and $16,000 for wages to production personnel. Finally, the company paid $16,000 for raw materials that were used to make inventory. All inventory was started and completed during the year. Walton completed production on 7,000 units of product and sold 6,000 units at a price of $15 each in 2008. (Assume all transactions are cash transactions.)

Required

a. Determine the total product cost and the average cost per unit of the inventory produced in 2008.

b. Determine the amount of cost of goods sold that would appear on the 2008 income statement.

c. Determine the amount of the ending inventory balance that would appear on the December 31, 2008, balance sheet.

d. Determine the amount of net income that would appear on the 2008 income statement.

e. Determine the amount of retained earnings that would appear on the December 31, 2008, balance sheet.

f. Determine the amount of total assets that would appear on the December 31, 2008, balance sheet.

g. Determine the amount of net cash flow from operating activities that would appear on the 2008 statement of cash flows.

h. Determine the amount of net cash flow from investing activities that would appear on the 2008 statement of cash flows.

explain how these events would affect the balance sheet income statement and stateme 666851

Effect of product versus period costs on financial statements

Polk Manufacturing Company experienced the following accounting events during its first year of operation. With the exception of the adjusting entries for depreciation, all transactions are cash transactions.

1. Acquired $70,000 cash by issuing common stock.

2. Paid $9,500 for the materials used to make products, all of which were started and completed during the year.

3. Paid salaries of $5,400 to selling and administrative employees.

4. Paid wages of $6,400 to production workers.

5. Paid $11,900 for furniture used in selling and administrative offices. The furniture was acquired on January 1. It had a $1,400 estimated salvage value and a four year useful life.

6. Paid $38,000 for manufacturing equipment. The equipment was acquired on January 1. It had a $2,000 estimated salvage value and a three year useful life.

7. Sold inventory to customers for $40,000 that had cost $20,000 to make.

Required

Explain how these events would affect the balance sheet, income statement, and statement of cash flows by recording them in a horizontal financial statements model as indicated here. The first event is recorded as an example. In the Cash Flow column, indicate whether the amounts represent financing activities (FA), investing activities (IA), or operating activities (OA).

Assets

Equity

Event
No.

Cash

+

Inventory

+

Manuf.
Equip.*

+

Office
Furn.*

=

Com.
Stk.

+

Ret.
Earn.

Rev.

Exp.

=

Net Inc.

Cash Flow

1

70,000

70,000

70,000

FA

the following transactions pertain to 2009 the first year operations of lepper compa 666852

Product versus general, selling, and administrative costs

The following transactions pertain to 2009, the first year operations of Lepper Company. All inventory was started and completed during 2009. Assume that all transactions are cash transactions.

1. Acquired $2,360 cash by issuing common stock.

2. Paid $720 for materials used to produce inventory.

3. Paid $1,800 to production workers.

4. Paid $540 rental fee for production equipment.

5. Paid $180 to administrative employees.

6. Paid $144 rental fee for administrative office equipment.

7. Produced 300 units of inventory of which 200 units were sold at a price of $12 each.

Required

Prepare an income statement, balance sheet, and statement of cash flows.

prepare a cost of quality report grouping costs into prevention appraisal internal f 666753

Preparing a cost of quality report The following data have just been gathered on last year’s quality related costs at the Ideal Company:

COST CATEGORY

AMOUNT

Product recalls

$325,000

Downtime due to defects

600,000

Warranty claims

420,000

Inspection of and testing of incoming materials

300,000

Product liability lawsuits

500,000

Process quality audits

350,000

Rework costs

2,000,000

Quality training

150,000

Process control monitoring

350,000

Repair costs in the field

375,000

Statistical process control

300,000

Waste

900,000

Net cost of scrap

1,500,000

Supplier certification

350,000

Quality engineering

200,000

Returned products

380,000

Total sales last year were $75,000,000.

Required

(a) Prepare a cost of quality report grouping costs into prevention, appraisal, internal failure, and external failure. Also show costs as a percentage of sales.

(b) Interpret the data and make recommendations to Ideal’s management.

benchmarking field exercise with other students assume that you are an average stude 666759

Benchmarking: field exercise in a company Benchmarking a product, process, or management accounting method takes a great deal of time and effort. Companies have many choices when it comes to conducting a benchmarking study. For example, in following the five stages of the benchmarking process, companies have to decide how to proceed, who to select as benchmarking partners, and what information they are willing to share and to gather.

Required

Locate a company in your local community that has engaged in a benchmarking study. Try to arrange a visit to the company (perhaps through your professor, relative, or friend) in order to talk to employees who have been involved in the benchmarking effort. Using the five stage process, critique the approach that this company followed. What are the similarities and differences between what this company did and the process described in this chapter? Be specific about the procedures that were used and the variables that were assessed. Finally, what were the results of the benchmarking exercise at this company? Was it a success or a failure? Why?

facilities layout lean manufacturing brand management value added activities some fi 666760

Facilities layout, lean manufacturing, brand management, value added activities Some firms in the fashion industry have adopted lean or just in time approaches to maintain or increase their competitive advantage. Read the following articles or other resources to address the questions below: “Brand New Bag: Louis Vuitton Tries Modern Methods On Factory Lines” (C. Passariello, The Wall Street Journal, October 9, 2006, p. A1) and “Zara Thrives By Breaking All Rules” (K. Capell, BusinessWeek, October 20, 2008, p. 66).

Required

(a) Compare Louis Vuitton’s previous and current processes for making a bag. For example, how many people and days are required, what are the workers’ degrees of specialization, and what improvements have resulted?

(b) How did Louis Vuitton’s previous process for making bags support the company’s value proposition?

(c) How have practices from competitors such as Zara changed Louis Vuitton’s view of what its target customers want? Has Louis Vuitton’s value proposition changed? If so, how well will the new process support the company’s value proposition?

(d) What performance indicators do you think are critical in evaluating the performance of this manufacturing operation from the standpoint of customers and the company?

relevant costs qualitative factors cost of quality framework environmental issues kw 666762

Relevant costs, qualitative factors, cost of quality framework, environmental issues Kwik Clean handles both commercial laundry and individual customer dry cleaning. Kwik Clean’s current dry cleaning process involves emitting a pollutant into the air. In addition, the commercial laundry and dry cleaning processes produce sediments and other elements that must receive special treatment before disposal. Pat Polley, Kwik Clean’s owner, is concerned about the cost of dealing with increasingly stringent laws and environmental regulations. Recent legislation requires Kwik Clean to reduce its amount of air pollution emissions. To reduce pollution emissions, Polley is considering the following two options:

  1. Option 1: Invest in equipment that would reduce emissions through filtration. The equipment would involve a large capital expenditure but would bring Kwik Clean into compliance with current regulations for emissions.
  2. Option 2: Invest in a new dry cleaning process that would eliminate current air pollution emissions, partly by using a different solvent than the one currently used. This option would require an even larger capital expenditure than option 1, but the new equipment would reduce some operating costs. Moreover, Kwik Clean might be able to market its environmentally safer process to increase business.

In evaluating the two options and current operations, Polley has enumerated the following items:

1. The price and quantity of solvent used in current operations (and option 1).

2. The price and quantity of the new solvent that would be used in option 2.

3. The purchase price of new equipment for option 1 and for option 2.

4. The cost of removing old equipment and installing new equipment under option 2.

5. The purchase price of the filtration equipment in option 1 as well as the useful life of the equipment.

6. The purchase price of the current equipment and its remaining useful life.

7. The salvage value of the current equipment, which would be sold under option 2.

8. Polley’s salary and fringe benefits.

9. Labor costs for current operations (and option 1) and option 2; labor costs would be lower under option 2 than under option 1.

10. Training costs associated with the new equipment in option 2.

11. Legal fees paid to handle paperwork associated with hazardous waste liabilities connected with the sediments produced when cleaning commercial laundry by the current operations (the same sediments would be produced with the equipment in option 2).

12. Storage and disposal costs associated with the sediments produced when cleaning commercial laundry.

13. Insurance for the equipment and workers; under option 2, insurance fees would be reduced from the current level.

Polley was concerned about recent events that were publicized locally. A newspaper article reported that the Occupational Safety and Health Administration fined one of Polley’s competitors several thousand dollars for unsafe employee working conditions related to handling solvents. Another business incurred a very expensive cleanup for accidental hazardous waste leakage that contaminated the soil. The leakage received major attention in the local television and radio news broadcasts and was headlined in the local newspapers.

Required

(a) Which costs are relevant to Polley’s decision to choose either option 1 or option 2?

(b) What qualitative factors is Polley likely to consider in choosing either option 1 or option 2?

(c) Explain how the cost of quality framework of prevention, appraisal, internal failure, and external failure might be applied to operations with environmental pollution, where failures are defined as accidental spillage or leakage of hazardous wastes or as illegal levels of pollutants. On which of the four cost of quality categories would you advise Polley to focus her attention?

customer service processes non value added activities daniel morris purchased a 42 i 666763

Customer service processes, non–value added activities Daniel Morris purchased a 42 inch plasma television, manufactured by TVCO, from a local electronics store that permits customers to return defective products within 30 days of purchase. Approximately 45 days after Daniel’s purchase, the TV began to malfunction periodically. Because Daniel could not return the TV to the local store, he turned to the warranty information and found that the warranty included picking up the approximately 100 pound TV from the owner’s home, repairing the TV, and delivering the repaired TV to the owner’s home. TVCO’s customer service process for handling warranty repairs is as follows:

1. The customer calls Customer Service (CS) to request authorization of the TV repair.

2. CS requests the customer to mail or fax the receipt, TV model number, and serial number.

3. On receipt of the information, CS locates a nearby repair shop to perform the repair.

4. CS forwards the repair request to the Warranty Department (WD) for approval.

5. On approval, WD informs CS so that CS can inform the customer and fax authorization for the repair to the approved repair shop.

6. The customer contacts the designated repair shop to arrange for the TV pickup. The repair shop picks up the TV.

7. The repair shop diagnoses the problem and orders parts.

8. On receipt of the parts, the shop repairs the TV and delivers it to the customer.

9. If the TV cannot be repaired, TVCO replaces the defective TV with a new one.

Accordingly, Daniel called CS to request authorization to repair the TV and faxed the receipt, TV model number, and serial number to CS. CS located a repair shop (RS1) 30 miles from Daniel’s city of Anytown. On obtaining WD’s approval, CS faxed authorization for the repair to the approved repair shop. Daniel contacted RS1 to arrange for the TV pickup, but RS1 refused to pick up the TV, stating that Daniel’s location is too far away. After several more phone calls to CS, with wait times before talking to a CS representative ranging from 25 to 45 minutes, CS authorized another repair shop, RS2. RS2 picked up the TV, and Daniel informed RS2 that he planned to move to Other town in two weeks and therefore hoped the TV could be repaired by then. RS2 did not look at the TV until Daniel called eight days later to check on progress. RS2 then diagnosed the problem and contacted TVCO for parts for the repair and was told that parts would not be available for several weeks. Because of his impending move to another city, Daniel requested RS2 to return the TV to him, thinking that he would get the TV repaired in Other town.

After moving to Other town, Daniel again called CS to request authorization for the TV repair. After several phone calls with sizable wait times before talking to CS, and several miscommunications between CS and WD that led Daniel to talk to a supervisor, CS located RS3 in Other town. However, RS3 was backlogged and would not pick up the TV for at least a week. RS3 picks up only on weekdays during regular working hours. Moreover, the technician would not look at the TV for at least 10 days after the TV arrived in the shop. Given the length of time that had now passed since Daniel’s first contact with CS, Daniel found this situation unacceptable, so he called CS and asked to talk to a supervisor. The supervisor suggested other approved stores for the repair.

Daniel found RS4, which was willing to pick up the TV at a day’s notice and diagnose problems as soon as possible so that parts could be ordered. Daniel called CS to arrange for authorization, and CS promised to call back soon. After a week with no response, Daniel called CS and was told that WD refused to authorize RS4 to perform the repair because WD thought Daniel still lived in Anytown and RS4 was too far from Anytown. Daniel called the supervisor again, and after a week the supervisor arranged for authorization for RS4 to do the repair. RS4 picked up the TV; by now, more than two months had passed since Daniel first contacted CS, and RS4 could not provide a definite date for completion of the repair. However, as promised, RS4 diagnosed the problem shortly after the TV arrived in the shop and ordered the apparently appropriate part. Disappointingly, changing the part did not correct the problem. TVCO suggested that RS4 try changing another part but could not provide an estimated date of arrival for the part. After Daniel’s further phone calls, TVCO agreed to exchange the defective TV for a new one. By this time, more than three months had passed since Daniel first contacted CS.

Required

(a) Assuming that TVCO has a performance measurement system for CS, what measures do you think the company is using to evaluate CS performance?

(b) What measures reflect what the customer is concerned about?

(c) What changes in the warranty service approval process might improve the process from the customer’s perspective?

(d) Compare how RS3 and RS4 have designed their repair process and explain to RS3 how it can reduce the time spent on non–value added activities.

cost savings replacement decision rossman instruments inc is considering leasing new 666764

Cost savings: replacement decision Rossman Instruments, Inc., is considering leasing new state of the art machinery at an annual cost of $900,000. The new machinery has a four year expected life. It will replace existing machinery leased one year earlier at an annual lease cost of $490,000 committed for five years. Early termination of this lease contract will incur a $280,000 penalty. There are no other fixed costs.

The new machinery is expected to decrease variable costs from $42 to $32 per unit sold because of improved materials yield, faster machine speed, and lower direct labor, supervision, materials handling, and quality inspection requirements. The sales price will remain at $56. Improvements in quality, production cycle time, and customer responsiveness are expected to increase annual sales from 36,000 units to 48,000 units.

The variable costs stated earlier exclude the inventory carrying costs. Because the new machinery is expected to affect inventory levels, the following estimates are also provided. The enhanced speed and accuracy of the new machinery are expected to decrease production cycle time by half and, consequently, lead to a decrease in work in process inventory level from 3 months to just 1.5 months of production. Increased flexibility with these new machines is expected to allow a reduction in finished goods inventory from 2 months of production to just 1 month. Improved yield rates and greater machine reliability will enable a reduction in raw materials inventory from 4 months of production to just 1.5 months. Annual inventory carrying cost is 20% of inventory value.

CATEGORY

OLD MACHINE

NEW MACHINE

Average per unit cost of raw materials inventory

$12

$11

Average per unit cost of work in process inventory

25

20

Average per unit cost of finished goods inventory

46

36

Variable cost per unit sold

42

32

Required

(a) Determine the total value of annual benefits from the new machinery. Include changes in inventory carrying costs.

(b) Should Rossman replace its existing machinery with the new machinery? Present your reasoning with detailed steps identifying relevant costs and revenues.

(c) Discuss whether a manager evaluated on the basis of Rossman’s net income will have the incentive to make the right decision as evaluated in part b.

breakeven time for new product development refer to exhibit 8 14 regarding greyson t 666774

Breakeven time for new product development Refer to Exhibit 8 14 regarding Greyson Technology’s launch of a new digital communications device. Suppose that Greyson reduced the quarterly spending on product development in panel A, which delayed launching the new product for two quarters, at which time the selling price and sales volume would be lower. Specifically, assume the following:

 

Y1,Q1

Y1,Q2

Y1,Q3

Y1,Q4

Y2,Q1

Y2,Q2

Y2,Q3

Y2,Q4

Y3,Q1

Market research (000)

($100)

($50)

             

Product

development (000)

 

($80)

($150)

($150)

($150)

($150)

($150)

($150)

($60)

 

 

Y3,Q1

Y3,Q2

Y3,Q3

Y3,Q4

Y4,Q1

Y4,Q2

Y4,Q3

Y4,Q4

Selling price

$19

$18

$18

$17

$17

$16

$15

$15

Sales quantity (000)

25

35

45

50

50

50

40

30

Required

Assuming that the cost per unit remains $10 and the MSDA expenses remain $120,000 per quarter, determine the difference between the breakeven time metrics under the initial assumptions in panel A and the new assumptions.

breakeven time for new product development refer to exhibit 8 14 regarding greyson t 666775

Breakeven time for new product development Refer to Exhibit 8 14 regarding Greyson Technology’s launch of a new digital communications device. Suppose that Greyson reduced the quarterly spending on product development in panel A, which delayed launching the new product for two quarters, at which time the selling price and sales volume would be lower. Specifically, assume the following:

 

Y1,Q1

Y1,Q2

Y1,Q3

Y1,Q4

Y2,Q1

Y2,Q2

Y2,Q3

Y2,Q4

Y3,Q1

Market research (000)

($100)

($50)

             

Product

development (000)

 

($80)

($150)

($150)

($150)

($150)

($150)

($150)

($60)

 

 

Y3,Q1

Y3,Q2

Y3,Q3

Y3,Q4

Y4,Q1

Y4,Q2

Y4,Q3

Y4,Q4

Selling price

$18

$17

$17

$16

$15

$15

$15

$15

Sales quantity (000)

20

30

40

45

45

35

30

20

After Y4, Q4, the competitive price is expected to remain at $15 and the maximum sales will be 20,000 units.

Required

Assuming that the cost per unit remains $10 and the MSDA expenses remain $120,000 per quarter, determine the breakeven time metric under the new assumptions.

target costing calculations amajor car manufacturer developed the following informat 666776

Target costing calculations Amajor car manufacturer developed the following information as part of its target costing efforts:

Customer Importance Rating by Category

CATEGORY

IMPORTANCE

Safety

140

Comfort and convenience

120

Economy

40

Styling

60

Performance Total

140 500

Target Cost by Function Group

FUNCTION GROUP

TARGET COST

Chassis

$1,400

Transmission

280

Air conditioner

100

Electrical system

700

Other function groups

4,520

Total

$7,000

 

Quality Function Deployment (Correlation) Matrix

 

Function Group

 

 

 

AIR

ELECTRICAL

OTHER FUNCTION

CATEGORIES

CHASSIS

TRANSMISSION

CONDITIONER

SYSTEM

GROUPS

Safety

0.3

0.1

 

0.1

0.5

Comfort and convenience

0.3

 

0.1

0.1

0.5

Economy

0.2

0.2

0.1

0.1

0.4

Styling

0.1

 

 

 

0.9

Performance

0.3

0.2

 

0.1

0.4

Required

(a) Prepare an exhibit showing percentage contributions of each function group to categories of customer requirements.

(b) Prepare a value index exhibit.

(c) Which function groups are candidates for cost reduction?

federal express has developed a measure called its service quality indicator sqi the 666790

Federal Express has developed a measure called its service quality indicator (SQI). The SQI is based on what Federal Express believes are nine key customer requirements. Each of these nine requirements is given a weight indicating the perceived importance of a failure of this requirement to the customer. The nine requirements and their respective weights are:

SERVICE FAILURE

PENALTY AMOUNT

Lost package

50

Damaged package

30

Complaint unresolved

10

Wrong day, late

10

Invoice adjustment required

3

Traces

3

Late pickup stop

3

Missing proof of delivery

1

Right day, late delivery

1

Federal Express tabulates these scores weekly and distributes them throughout the organization. Rewards to senior managers are based on these scores.

effects of transfer price choices mccann company has two divisions division c and di 666797

Effects of transfer price choices McCann Company has two divisions, Division C and Division D. Division C manufactures Part C82 and sells it to Division D, and also sells the same part to the outside market for $50 per unit. Division C has capacity to make 400,000 units of C82 per year. The division’s fixed costs are $5,000,000 per year and its variable costs per unit are as follows:

Direct materials

$20

Direct labor

12

Variable overhead

8

Part C82 is an essential component for Division D’s only product; the division sells 200,000 units per year at a price of $120 per unit. Division D’s fixed costs are $4,000,000 per year and its variable costs per unit, excluding the cost of Part C82, are as follows:

Direct materials

$10

Direct labor

25

Variable overhead

10

Required

Suppose Division C’s demand for C82 from the outside market is currently 150,000 units per year. By how much will McCann’s income decrease if Division D purchases its desired 200,000 units of C82 at $50 per unit from the market rather than from Division C? What transfer price(s) would you suggest to induce both divisions to want Division D to purchase from Division C instead of from the market?

return on investment measurement issues green company has prepared the following inf 666801

Return on investment measurement issues Green Company has prepared the following information for three of its divisions:

DIVISION

HISTORICAL COST OF INVESTMENTS

DIVISION OPERATING INCOME

X

$560,000

$66,500

Y

532,000

64,400

Z

350,000

43,120

Required

(a) Compute each division’s return on investment and residual income, assuming a 10% cost of capital.

(b) Suppose the net book value of each division’s investments is half of the historical cost. Using net book value as the measure of investment, compute each division’s return on investment and residual income, assuming a 10% cost of capital.

(c) Comment on the division rankings in parts a and b.

(d) If the division managers are rewarded on the basis of return on investment or residual income, will they find it attractive to invest in new, more costly equipment?

return on investment components eta company would like to examine the sales margin a 666802

Return on investment components Eta Company would like to examine the sales margin and asset components of return on investment for three of its divisions and has accordingly prepared the following information:

DIVISION

INVESTMENT

DIVISION OPERATING INCOME

SALES

E

$575,000

$75,000

$500,000

F

700,000

91,000

542,000

G

1,000,000

176,000

763,000

Required

(a) Compute each division’s return on investment, sales margin, and turnover.

(b) Comment on the divisions’ relative rankings on the ratios computed in part a.

(c) Compute each division’s residual income, assuming a required return on investment of 8%.

return on investment and residual income the following information pertains to vi di 666804

Return on investment and residual income The following information pertains to VI Division, which has $1,400,000 in investments.

Division sales revenue

$900,000

Less division expenses

480,000

Division income

$420,000

The company’s cost of capital is 10%.

Required

(a) What is the division’s return on investment?

(b) What is the division’s residual income?

segment margins following is the information on paragon company s three product line 666811

Segment margins Following is the information on Paragon Company’s three product lines:

 

PRODUCT LINE

 

1

2

3

Revenue

$7,160,000

$1,900,000

$4,200,000

Variable cost percentage of sales

60%

50%

40%

Other costs

$859,200

$237,500

$693,000

Allocated avoidable corporate costs

$349,000

$156,000

$698,000

Allocated unavoidable corporate costs

$570,800

$206,500

$24,000

Required

(a) Construct a segment margin statement for Paragon Company.

(b) Explain why the segment margins reported for an organization unit must be interpreted carefully.

return on investment michelle gutierrez manager of the components division of fx cor 666815

Return on investment Michelle Gutierrez, manager of the Components Division of FX Corporation, is considering a new investment for her division. The division has an investment base of $4,000,000 and operating income of $600,000. The new investment of $500,000 supports corporate strategy and is expected to increase operating income by $50,000 next year, an acceptable level of return from corporate headquarters’ point of view.

Required

(a) What is the current return on investment (ROI) for the Components Division?

(b) What will the ROI be if Michelle undertakes the new investment?

(c) Suppose Michelle’s compensation consists of a salary plus a bonus proportional to her division’s ROI. Is Michelle’s compensation higher with or without the new investment?

(d) Suggest changes to FX Corporation’s management that will better align performance evaluation and compensation with corporate goals.

return on investment and residual income the newburg flyers operate a major sports f 666816

Return on investment and residual income The Newburg Flyers operate a major sports franchise from a building in downtown Newburg. The building was built in 1940 at a cost of $5,000,000 and is fully depreciated so that it is shown on the company’s balance sheet at a nominal value of $1. The land on which the building was built in 1940 was purchased in 1935 for $10,000 and is valued at this amount for balance sheet purposes. The franchise, which is the company’s only other major investment, cost $100,000 in 1940. Following GAAP at the time of the purchase, the franchise cost has now been fully amortized.

The current assessed value of the building is $200,000. The assessed value of the land, which is located in a prime urban area, is $20,000,000 and reflects the net value of the property if the current building is demolished and replaced with an office and shopping complex. The current value of the franchise, assuming that the league owners would approve a franchise sale, is $50,000,000.

Required

(a) Ignoring taxes in this calculation, if the team earns an income of approximately $3,000,000 per year, what is the return on investment using net book value and historical cost as the measures of investment?

(b) Ignoring taxes in this calculation and assuming that the organization’s cost of capital is 15%, if the team earns approximately $3,000,000 per year, what is the residual income using net book value and historical cost as the measures of investment?

conflicting organization and individual objectives strathcona paper rewards its mana 666819

Conflicting organization and individual objectives Strathcona Paper rewards its managers on the basis of the after tax return on investment of the  assets that they manage—the higher the reported return on investment, the higher the reward. The company uses net book value to value the assets employed in the return on investment calculation. The company’s cost of capital is assessed as 12% after taxes. The organization’s tax rate is 35%.

The manager of the logistics division is faced with an opportunity to replace an aging truck fleet. The current net income after taxes of the logistics division is $7 million, and the current investment base is valued at $50 million. The current net income after taxes and the current investment base, absent any investment in new trucks, are expected to remain at their existing levels.

The investment opportunity would replace the existing fleet of trucks, which have a net book value of about $100,000, with new trucks costing about $50 million net of the trade in allowance for the old trucks. If kept, the old trucks would last another 5 years and would have no salvage value. The new trucks would last 5 years, have zero salvage value, and increase cash flow relative to keeping the old trucks (through increased revenues and decreased operating costs) by about $16 million per year. If purchased, the new trucks would be depreciated for both accounting and tax purposes on a straight line basis.

Required

(a) From the point of view of the company, should this investment be made? Support your conclusion with net present value calculations.

(b) From the point of view of the manager, should this investment be made?

(c) If the manager were rewarded on the basis of after tax residual income, would the manager want to make the investment? Show why or why not.

coordinating divisional activities for many years automobile companies were highly d 666823

Coordinating divisional activities For many years, automobile companies were highly decentralized in terms of functions. The most obvious effect of this heavy decentralization of function was apparent when all the groups needed to work together to accomplish a goal. The highest order of integration occurs in the design of a new automobile.

Reflecting the functional decentralization of automobile manufacturers, the traditional approach to automobile design was for the marketing group to identify a concept. The design group then created an automobile that reflected the marketing group’s idea but incorporated engineering requirements and aesthetics identified by the design group. The purchasing group then identified the parts required by the design and made further modifications to it to incorporate parts that could be made or purchased. Finally, the manufacturing group modified the design to reflect the nature and capabilities of the production process. This process took up to four years and usually resulted in a vehicle that was far removed from the initial design.

What was wrong here? How could the process be improved?

choices in financial control bennington home products sells home products it buys pr 666824

Choices in financial control Bennington Home Products sells home products. It buys products for resale from suppliers all over the world. The products are organized into groups. A few examples of these groups are floor care products, kitchen products, tool products, and paper products. The company sells its products all over the world from regional offices and warehouses in every country where it operates. Because of differences in culture and taste, the product lines and products within those lines vary widely among countries.

The regional offices have administrative staff that manage the operations, place orders to the corporate office, and undertake the usual office administrative functions, and they have sales staff that do the selling directly to stores within each region. The regional offices are evaluated as investment centers because they have responsibility for revenues, costs, and investment levels. The regional offices make suggestions for new products.

The corporate office manages the regional offices and places the orders received from the regional offices with suppliers. The corporate office does the ordering for three reasons. First, the company believes that one ordering office eliminates duplication in ordering activities. Second, it believes that one office ordering for all of the regional offices gives the organization more power when dealing with suppliers. Third, it is believed that one office can develop the expertise to find and negotiate with suppliers of unique and innovative products.

Required

(a) Describe an appropriate system of financial control at the regional level.

(b) Describe an appropriate system of financial control at the corporate office level.

(c) Explain why the systems of financial control should or need not mesh.

process costing connor chemical company s plant processes batches of organic chemica 666714

Process costing Connor Chemical Company’s plant processes batches of organic chemical products through three stages after starting with raw materials: (1) mixing and blending, (2) reaction chamber, and (3) pulverizing and packing. Connor Chemical’s estimates for the total conversion costs for each of the three processing stages are shown in the following table. These costs include production labor assigned to each stage, support labor performing tasks (such as handling the output of the previous stage and setting up for the new stage), and laboratory testing. Additional materials for packing are needed in the pulverizing and packing stage:

 

MIXING AND BLENDING

REACTION CHAMBERS

PULVERIZING AND PACKING

Production labor

$253,000

$1,144,000

$396,000

Engineering support

22,000

50,600

24,200

Materials handling

19,800

19,800

29,700

Equipment maintenance

11,000

35,200

8,800

Laboratory expenses

22,000

22,000

4,400

Depreciation

44,000

176,000

52,800

Power

35,200

85,800

26,400

General and administrative

17,600

17,600

17,600

Total conversion costs

$424,600

$1,551,000

$559,900

Total number of process hours

8,760

35,040

8,760

Required

(a) Determine the estimated conversion cost driver rate per process hour for each stage. (Round to three digits after the decimal point.)

(b) Consider two of Connor Chemical’s representative products, C206 and C208. Both products are derivatives of ethyl oleate and at the start of the process require the same basic raw materials. Using the information below, determine the total cost of a batch of C206 and a batch of C208:

 

C206

C208

Materials:

 

 

Raw materials, beginning of process

$1,488.00

$1,488.00

Packing materials

$175.20

$280.80

Conversion hours:

 

 

Mixing and blending

6 hours

6 hours

Reaction chamber

24 hours

24 hours

Pulverizing and packing

4 hours

8 hours

process costing equivalent units and product cost the information below pertains to 666715

Process costing equivalent units and product cost The information below pertains to October production at Zippy Company’s bottling plant, which produces and bottles sports drinks. Each unit consists of a case of 12 bottles:

 

UNITS

MATERIALS

CONVERSION

Work in process, October 1

2,000

70% complete

60% complete

Started in October

10,000

 

 

To account for

12,000

 

 

Completed and transferred out

8,000

100% complete

100% complete

Work in process, October 30

4,000

40% complete

25% complete

Accounted for

12,000

 

 

Costs, beginning of October

 

$1,050

$3,240

Added during October

 

8,200

22,620

To be accounted for

 

$9,250

$25,860

Required

(a) Using the weighted average method, determine the number of equivalent units of production for materials and conversion during October.

(b) Determine the cost per equivalent unit for materials and conversion for October and the total cost per equivalent unit. (Round to two digits after the decimal point.)

(c) Determine whether the cost per equivalent unit for materials and conversion increased or decreased from the previous month.

job bid and direct and sequential allocations sanders manufacturing company produces 666716

Job bid and direct and sequential allocations Sanders Manufacturing Company produces electronic components on a job order basis. Most business is gained through bidding on jobs. Most firms competing with Sanders bid full cost plus a 30% markup. Recently, with the expectation of gaining more sales, Sanders dropped its markup from 40% to 30%. The company operates two service departments and two production departments. Manufacturing overhead costs and quantities of activities for each department are shown here:

 

SERVICE DEPARTMENTS

PRODUCTION DEPARTMENTS

ITEM

PERSONNEL

MAINTENANCE

MACHINING

ASSEMBLY

Overhead costs

$100,000

$200,000

$400,000

$300,000

Number of employees

5

5

5

40

Maintenance hours

1,500

200

7,500

1,000

Machine hours

0

0

10,000

1,000

Direct labor hours

0

0

1,000

10,000

Costs of the personnel department are allocated on the basis of employees and those of the maintenance department on the basis of maintenance hours. Departmental rates are used to assign overhead costs to products. The machining department uses machine hours, and the assembly department uses direct labor hours for this purpose.

The firm is preparing to bid on job 781, which requires three machine hours per unit produced in the machining department and five direct labor hours per unit produced in the assembly department. The expected direct materials and direct labor costs per unit are $450.

Required

(a) Allocate the service department costs to the production departments using the direct method.

(b) Determine the bid price per unit produced for job 781 using the direct method.

(c) Assume that the costs of the service department incurring the greatest cost are allocated first, and allocate the service department costs to the production departments using the sequential method. When allocating personnel costs, assume the maintenance department has 0 employees.

(d) Determine the bid price per unit produced for job 781 using the sequential method in part c.

job bid price and direct sequential and reciprocal allocations sherman company manuf 666718

Job bid price and direct, sequential, and reciprocal allocations Sherman Company manufactures and sells small pumps made to customer specifications. It has two service departments and two production departments. Data on current year operations follow:

 

SERVICE DEPARTMENTS

PRODUCTION DEPARTMENTS

ITEM

MAINTENANCE

POWER

CASTING

ASSEMBLY

Costs

$750,000

$450,000

$150,000

$110,000

Machine hours

0

80,000

80,000

40,000

Kilowatt hours

40,000

0

200,000

160,000

Direct labor hours

0

0

100,000

60,000

Management allocates maintenance department costs using machine hours, and power department costs using kilowatt hours. Separate cost driver rates are determined on the basis of machine hours for the casting department and on the basis of direct labor hours for the assembly department. It takes 1 machine hour to manufacture a pump in the casting department and 0.5 labor hour to assemble a pump in the assembly department. Direct labor and material costs amount to $32 per pump. A prospective customer has requested a bid on a two year contract to purchase 1,000 pumps every month. Sherman Company has a policy of adding a 25% markup to the full manufacturing cost to determine the bid.

Required

(a) What is the bid price when the direct method is used?

(b) What is the bid price when the sequential method that begins by allocating maintenance department costs is used?

(c) What is the bid price when the reciprocal method is used?

practical capacity cost driver rates and the death spiral youngsborough products a s 666719

Practical capacity, cost driver rates, and the death spiral Youngsborough Products, a supplier to the automotive industry, had seen its operating margins shrink below 20% as its customers put continued pressure on pricing. Youngsborough produced four products in its plant and decided to eliminate products that no longer contributed positive gross margins. The total plant overhead cost is $122,000 per year. Details on the four products are provided here:

 

PRODUCTS

 

A

B

C

D

Production volume (units)

10,000

8,000

6,000

4,000

Selling price

$15.00

$18.00

$20.00

$22.00

Materials per unit

$4.00

$5.00

$6.00

$7.00

Direct labor hours per unit

0.24

0.18

0.12

0.08

Total direct labor hours

2,400

1,440

720

320

           

Youngsborough calculates a plantwide overhead rate by dividing total direct labor hours into total overhead costs. Assume that plant overhead is a fixed cost during the year, but that direct labor is a variable cost. The direct labor rate is $30 per hour.

Required

(a) Calculate the plantwide cost driver rate and use this rate to assign overhead costs to products. Calculate the gross margin for each product and calculate the total gross margin.

(b) If any product is unprofitable in part a, drop this product from the mix. Recalculate the cost driver rate based on the new total direct labor hours remaining in the plant and use this rate to assign overhead costs to the remaining three products. Calculate the gross margin for each product and calculate the total gross margin.

(c) Drop any product that is unprofitable with the revised cost assignment. Repeat the process, eliminating any unprofitable products at each stage.

(d) What is happening at Youngsborough and why? How could this situation be avoided?

whale curve wright company a new systems consulting company is concerned about the p 666725

Whale curve Wright Company, a new systems consulting company, is concerned about the profitability of its customers during the past year. The company has prepared the following data:

CUSTOMER

PROFIT

CUSTOMER

PROFIT

1

$221,000

14

83,000

2

–40,000

15

–179,000

3

–143,000

16

14,000

4

217,000

17

50,000

5

22,000

18

–191,000

6

9,000

19

–90,000

7

101,000

20

30,000

8

–200,000

21

–10,000

9

259,000

22

87,000

10

96,000

23

–158,000

11

208,000

24

–100,000

12

233,000

25

75,000

13

264,000

 

 

Required

(a) Prepare a whale curve, as described in this chapter.

(b) What percentage of total profits did the most profitable 20% of the customers generate?

(c) What percentage of total profits did the least profitable 20% of the customers lose for the company?

salespersons incentives customer profitability in response to how the sales incentiv 666727

Salespersons’ incentives, customer profitability In response to how the sales incentives might be contributing to falling profits despite growing sales, Chan Company’s controller has produced the following information on last year’s sales to two customers that purchased a variety of products from the company:

 

CUSTOMERS

 

CARLSON

DONNER

Sales

$450,000

$400,000

Cost of goods sold

180,000

80,000

MSDA expenses, excluding sales commissions

320,000

65,000

       

Required

(a) Which customer is more profitable for the company?

(b) Compare a sales incentive scheme that pays 2% of sales revenue to an incentive scheme that pays 4% of customer profit. How will each scheme affect salespersons’ desire to increase sales to each customer?

customer lifetime value calculation compute the customer lifetime value for customer 666728

Customer lifetime value calculation Compute the customer lifetime value for Customer 421 based on the data below for the first six years of the customer relationship. Costs (ct) were incurred to promote customer retention to a rate of 0.8 in years 1 through 6.

 

CUSTOMER 421

Initial acquisition cost

$600

n= number of years retained

6

r= retention rate for each of the n years retained

0.8

Cost of capital

0.1

Mt = margin from customer in year t

 

M1

$250

M2

300

M3

325

M4

350

M5

375

M6

400

c1

60

c2

50

c3

50

c4

50

c5

40

c6

40

net promoter score calculation stan s a department store chain has conducted a surve 666729

Net promoter score calculation Stan’s, a department store chain, has conducted a survey to collect data on customer satisfaction and perception of its merchandise and service. Data for responses to the question “How likely is it that you would recommend Stan’s to a friend or colleague?” appear below.

SCORE

NUMBER OF RESPONSES

10

641

9

1,265

8

1,254

7

228

6

548

5

493

4

357

3

63

2

42

1

109

Total

5,000

Required

What is Stan’s net promoter score?

customer profitability a credit card company has classified its customers into the f 666730

Customer profitability A credit card company has classified its customers into the following types for customer profitability analysis:

1. Applies for credit card in response to a low introductory interest rate; transfers balance to new account, but when the low introductory rate expires, the customer transfers the balance to an account with a different credit card company that has offered a low introductory rate.

2. Charges a large dollar volume of purchases; pays balance in full and on time each month.

3. Carries a high balance; pays only the minimum required payment but pays regularly with occasional late payment.

4. Carries a high balance; pays at least the minimum required payment but does not pay in full and always pays on time.

5. Carries a low balance; pays at least the minimum required payment but does not pay in full and always pays on time.

6. Does not use the account but does not close the account.

The following facts pertain to the credit card company’s operations:

  1. Merchants pay the credit card company a percentage of the dollar sales on each credit card transaction.
  2. Customers pay no interest on charges for purchases if the balance is paid in full and on time each month.
  3. The credit card company charges a late fee if the customer’s payment is late.
  4. The credit card company incurs costs to send statements to inactive customers.

Required

Given the preceding information, which customer types would you expect to be the most desirable or profitable, the next most profitable, and so on for the credit card company on a long term basis? Explain your ranking.

customer profitability analysis original activity based costing kronecker company a 666731

Customer profitability analysis, original activity based costing Kronecker Company, a growing mail order clothing and accessory company, is concerned about its growing MSDA expenses. It therefore examined its customer ordering patterns for the past year and identified four different types of customers, as illustrated in the following table. Kronecker sends catalogs and flyers to all its customers several times a year. Orders are taken by mail or over the phone. Kronecker maintains a toll free number for customers to use when placing orders over the phone. Kronecker prides itself on the personal attention it provides shoppers who order over the phone. All purchases are paid for by check or credit card. Kronecker has a very generous return policy if customers are not satisfied with the merchandise received. Customers must pay return shipping charges, but their purchase price is then fully refunded.

 

CUSTOMER

CUSTOMER

CUSTOMER

CUSTOMER

 

TYPE 1

TYPE 2

TYPE 3

TYPE 4

Initial sales

$1,000

$1,000

$2,500

$3,000

Number of items returned

0

4

2

24

Dollar value of items returned

0

$200

$500

$1,500

Number of orders per year

1

6

4

12

Number of phone orders per year

1

0

0

12

Time spent on phone placing orders

0.25 hour

0

0

1 hour

Number of overnight deliveries

1

0

0

12

Number of regular deliveries

0

6

4

0

Prices are set so that cost of goods sold is on average about 75% of the sales price. Customers pay actual shipping charges, but extra processing is required for overnight deliveries. Kronecker has developed the following activity cost driver rates for its support costs:

ACTIVITY

ACTIVITY COST DRIVER RATE

Process mail orders

$5 per order

Process phone orders

$80 per hour

Process returns

$5 per item returned

Process overnight delivery requests

$4 per request

Maintain customer relations (send catalogs and respond

$50 per year

to customer comments or complaints)

 

Required

(a) Using activity based costing, determine the yearly profit associated with each of the four customers described.

(b) Comment on which customers are most profitable and why.

(c) What advice do you have for Kronecker regarding managing customer relationships with the different types of customers represented?

pricing waterfall randolph company s product mix has become more diverse over the pa 666733

Pricing waterfall Randolph Company’s product mix has become more diverse over the past few years. Consequently the company undertook an activity based costing initiative to develop accurate costs for production, as well as marketing, selling, distribution, and administration. The company set list prices that would provide a profit regardless of whether the customer orders were complex or routine. Nevertheless, profits have been falling. The company’s management team decided to examine discounts that had been granted to determine whether these are the reason for poor profit performance. Management was surprised to learn that customers were taking advantage of a large number of possible discounts or allowances, including the following:

1.

Volume discount if 20 or more units are ordered

2%

2.

Pay in full in 15 days

3%

3.

Cooperative advertising allowance for featuring the

4%

 

company’s products in its advertisements

 

4.

Take a large shipment before the end of the quarter in

5%

 

advance of an expected seasonal increase in demand

 

5.

Online ordering discount

2%

6.

Rebate on sales during specific promotional periods

2%

The management team believed that some discounting was necessary to acquire and retain large customers. On deeper investigation, they learned that some of their smaller customers, who were often the most cost conscious, took advantage of every discount or allowance offered. To compare discounts or allowances taken, they compared Customer 1 and Customer 2.

Customer 1 is a long time customer with sales of $200,000 at list prices. This customer takes advantage of each discount or allowance listed in the preceding table. Moreover, this customer has been a loyal customer since Randolph Company’s inception. In appreciation, Randolph’s sales representative offers free freight, which amounts to 3% of the customer’s list price purchases from Randolph. Customer 2 is a more recently acquired customer with sales of $1,000,000 at list prices. This customer only takes advantage of items 1, 3, and 5 in the preceding table.

Required

(a) Compute the total sales discount percentage for Customer 1 and for Customer 2.

(b) Why might Randolph Company’s management team have been unaware of the potentially large total discounts offered to its customers?

(c) What advice do you have for Randolph Company regarding managing its discounts and allowances?

customer lifetime value calculations kem company has begun studying customer lifetim 666734

Customer lifetime value calculations KEM Company has begun studying customer lifetime value for its customers and has prepared the information below for selected customers. For simplicity, management has assumed that for a given customer, the retention rate is the same every year until the customer departs. For Customer 4, costs (ct) were incurred to promote customer retention in years 1 and 2.

 

CUSTOMERS

 

1

2

3

4

Initial acquisition cost

$1,000

$1,000

$1,000

$1,000

n = number of years retained

5

3

5

5

r = retention rate for each of the n years retained

1

1

0.9

1

Cost of capital

0.1

0.1

0.1

0.1

Mt = margin from customer in year t

 

 

 

 

M1

$275

$300

$275

$275

M2

275

300

275

275

M3

275

300

275

300

M4

275

275

300

M5

275

275

300

ct = additional costs to serve and retain customer in year t

 

 

 

 

c1

$0

$0

$0

$50

c2

0

0

0

25

c3

0

0

0

0

c4

0

0

0

c5

0

0

0

Required

(a) Compute the customer lifetime value for each customer for the stated number of years.

(b) Discuss the reasons for differences in customer lifetime value between Customers 1 and 2, Customers 1 and 3, Customers 1 and 4, and Customers 3 and 4.

(c) Compute the customer lifetime value for Customers 1, 2, and 3 assuming that n is very large and the numbers in the table remain about the same each year.

(d) How does information on a customer’s estimated lifetime value help a company manage its customer acquisition and loyalty programs?

pricing customer profitability managing customer relationships read the wall street 666735

Pricing, customer profitability, managing customer relationships Read the Wall Street Journal article “Survival Strategies: After Cost Cutting, Companies Turn toward Price Increases” by Timothy Aeppel (September 18, 2002, p. A1). The article reports “an all out search for new ways to charge more money without raising prices.”

Required

(a) How did Jergens, Inc., use an activity based costing approach to justify the price for an order of odd size metal locating fasteners?

(b) What issues arose in Goodyear Tire & Rubber’s pricing to distributors? What was Goodyear’s response?

(c) What was the outcome of Emerson Electric’s decision to depart from cost based pricing? How can a product costing system contribute to undercosting a low volume or customized product?

(d) How did Wildeck influence customers to purchase products and services that are more profitable to Wildeck? How did Wildeck respond to a competitor’s lower priced storage rack protector? What role should a cost system play in such decisions?

(e) Why was Union Pacific not concerned if it lost its less profitable customers? Will dropping unprofitable customers always lead to an immediate increase in profit?

 

quality cost categories of the four quality costing categories which quality cost is 666743

Quality cost categories Of the four quality costing categories, which quality cost is the most damaging to the organization? Explain. 7 30 JIT manufacturing and cost savings Boris Company introduced JIT manufacturing last year and has prepared the following data to assess the benefits from the change:

CATEGORY

BEFORE THE CHANGE

AFTER THE CHANGE

Production cycle time

50 days

25 days

Inventories

$220,000

$40,000

Total sales

$1,000,000

$1,500,000

Costs as percent of sales:

 

 

Direct materials

25%

20%

Direct labor

20%

15%

Support

27%

17%

Inventory financing costs are 12% per year. Support costs are based on a time driven activity based costing analysis. Estimate the total financial benefits that resulted from the switch to JIT manufacturing operations.

inventory carrying costs smy corporation produces 60 000 videophones per year the co 666744

Inventory carrying costs SMY Corporation produces 60,000 videophones per year. The company estimates its direct material costs for the videophone to be $300 per unit and its conversion (direct labor plus support) costs to be$400 per unit. Annual inventory carrying costs, not included in these costs, are estimated to be 10%. SMY’s average inventory levels are estimated as follows:

Direct material

2 months of production

Work in process (100% complete for materials and 50% for conversion)

2 months of production

Finished goods

1 month of production

Compute the annual inventory carrying costs for SMY Corporation.

relevant cost and revenues changes in facilities layout to facilitate a move toward 666745

Relevant cost and revenues: changes in facilities layout To facilitate a move toward JIT production, AB Company is considering a change in its plant layout. The plant controller, Anita Bentley, has been asked to evaluate the costs and benefits of the change in plant layout. After meeting with production and marketing managers, Anita has compiled the following estimates:

  1. Machine moving and reinstallation will cost $100,000.
  2. Total sales will increase by 20% to $1,200,000 because of a decrease in production cycle time required under the new plant layout. Average contribution margin is 31% of sales.
  3. Inventory related costs will decrease by 25%. Currently, the annual average carrying value of inventory is $200,000. The annual inventory financing cost is 15%. Should AB implement the proposed change in plant layout? Support your answer.

cycle time efficiency and jit walker brothers company is considering the installatio 666746

Cycle time efficiency and JIT Walker Brothers Company is considering the installation of a JIT manufacturing system in the hope that it will improve the company’s overall processing cycle efficiency. Data from the traditional system and estimates for the JIT system are presented here for their Nosun product:

TIME CATEGORY

TRADITIONAL SYSTEM

JIT SYSTEM

Storage

4 hours

1 hour

Inspection

40 minutes

5 minutes

Moving

80 minutes

20 minutes

Processing

2 hours

75 minutes

Required

(a) Calculate processing cycle efficiency (PCE) under the traditional and JIT systems for the Nosun product.

(b) Based strictly on your PCE calculations above, should Walker Brothers implement the JIT system? Explain.

quality improvement programs and cost savings gurland valves company manufactures br 666748

Quality improvement programs and cost savings Gurland Valves Company manufactures brass valves that meet precise specification standards. All finished valves are inspected before being packaged and shipped to customers. Rejected valves are returned to the initial production stage to be melted and recast. Such rework requires no new materials in casting but requires new materials in finishing. The following unit cost data for direct materials, direct labor, and time driven activity based costing (ABC) support are available:

COSTS

CASTING

FINISHING

INSPECTION

PACKING

TOTAL

Direct materials

$225

$12

$0

$8

$245

Direct labor

84

121

24

16

245

Support

122

164

30

20

336

As a result of a quality improvement program, the reject rate has decreased from 6.4% to 5.1%, and the number of rejects has decreased by (6.4% 5.1%) × (10,000) units. Improvements in reject rates have also led to a decrease in work in process inventory from $386,000 to $270,000. Inventory carrying costs are estimated to be 15% per year. Estimate the annual cost savings as a result of the quality improvement, assuming that capacity costs as indicated by the time driven ABC support costs can be reduced if not needed.

group technology and processing cycle efficiency ray brown s company whisper voice s 666749

Group technology and processing cycle efficiency Ray Brown’s company, Whisper Voice Systems, is trying to increase its processing cycle efficiency (PCE). Because Ray has a very limited budget, he has been searching for a way to increase his PCE by using group technology. One of Ray’s manufacturing managers, Maria Lopez, has been studying group technology and claims that with minimal cost that includes downtime in the operation, she can rearrange existing machinery and workers and improve PCE. Ray is quite skeptical about this and decides to allow Maria to rearrange a small part of his operation. For Ray to be satisfied, he has stated that PCE must increase by 12%. PCE data before and after the rearrangement are as follows:

TIME CATEGORY

BEFORE REARRANGEMENT

AFTER REARRANGEMENT

Inspection

30 minutes

15 minutes

Moving

45 minutes

10 minutes

Processing

70 minutes

30 minutes

Storage

55 minutes

20 minutes

Does the change in PCE meet Ray’s requirement? Why or why not?

what can be done to ensure that the standards are set correctly 666372

SETTING STANDARDS, CALCULATING AND USING VARIANCES

Leather Works is a family owned maker of leather travel bags and briefcases located in the northeastern part of the United States. Foreign competition has forced its owner, Heather Gray, to explore new ways to meet the competition. One of her cousins, Walace Hayes, who recently graduated from college with a major in accounting, told her about the use of cost variance analysis to learn about efficiencies of production.

In May 2006, Heather asked Matt Jones, chief accountant, and Alfred Prudest, production manager, to implement a standard costing system. Matt and Alfred, in turn, retained Shannon Leikam, an accounting professor at Harding’s College, to set up a standard costing system by using information supplied to her by Matt’s and Alfred’s staff. To verify that the information was accurate, Shannon visited the plant and measured workers’ output using time and motion studies. During those visits, she was not accompanied by either Matt or Alfred, and the workers knew about Shannon’s schedule in advance. The cost system was implemented in June 2006.

Recently, the following dialogue took place among Heather, Matt, and Alfred:

HEATHER: How is the business performing?

ALFRED: You know, we are producing a lot more than we used to, thanks to the contract that you helped obtain from Lean, Inc., for laptop covers. (Lean is a national supplier of computer accessories.)

MATT: Thank goodness for that new product. It has kept us from sinking even more due to the inroads into our business made by those foreign suppliers of leather goods.

HEATHER: What about the standard costing system?

MATT: The variances are mostly favorable, except for the first few months when the supplier of leather started charging more.

HEATHER: How did the union members take to the standards?

ALFRED: Not bad. They grumbled a bit at first, but they have taken it in stride.

We’ve consistently shown favorable direct labor efficiency variances and direct materials usage variances. The direct labor rate variance has been flat.

MATT: It should be since direct labor rates are negotiated by the union representative at the start of the year and remain the same for the entire year.

HEATHER: Matt, would you send me the variance report for laptop covers immediately?

The following chart summarizes the direct materials and direct labor variances from November 2006 through April 2007 (extracted form the report provided by Matt) Standards for each laptop cover are as follows:

a. Three feet of direct materials at $7.50 per foot.

b. Forty five minutes of direct labor at $14 per hour.

Month

Actual Cost

(Direct Materials

+ Direct Labor)

Direct Materials

Price Variance

Direct Materials

Efficiency Variance

Direct Labor

Rate Variance

Direct Labor

Efficiency Variance

 

November

$150,000

$10,000 U

$5,000 F

$100 U

$5,000 F

December

155,000

11,000 U

5,200 F

110 U

6,500 F

January

152,000

10,100 U

4,900 F

105 U

7,750 F

February

151,000

9,900 U

4,500 F

95 U

6,950 F

March

125,000

9,000 U

3,000 F

90 U

8,200 F

April

115,000

8,000 U

2,000 F

90 U

8,500 F

In addition, the data for May 2007, but not the variances for the month, are as follows:

Laptop covers made in May

2,900 units

Total actual direct materials costs incurred

$68,850

Actual quantity of direct materials purchased and used

8,500 feet

Total actual direct labor cost incurred

$25,910

Total actual direct labor hours

1,837.6 hours

Actual direct labor cost per hour exceeded the budgeted rate by $0.10 per hour.

Required:

1. For May 2007, calculate the price and quantity variances for direct labor and direct materials.

2. Discuss the trend of the direct materials and labor variances.

3. What type of actions must the workers have taken during the period they were being observed for the setting of standards?

4. What can be done to ensure that the standards are set correctly?

discuss the president rsquo s concern about using the result of the engineering stud 666373

COLLABORATIVE LEARNING EXERCISE: STRUCTURED PROBLEM SOLVING; THREE STAY, ONE STRAY:ESTABLISHMENT OF STANDARDS, VARIANCE ANALYSIS

Tasty Apple, an apple chip manufacturer, was established in 1972 by Katherine English. In 2002, Katherine English died, and her son, Mark, took control of the business. By 2007, the company was facing stiff competition from national snack food companies. Mark was advised that the company’s plants needed to gain better control over production costs. To achieve this objective, he hired a consultant to install a standard costing system. To help the consultant in establishing the necessary standards,

Mark sent her the following memo:

MEMO

To: Darlene Swasey, CMA

From: Mark English, President, Tasty Apple

Subject: Description and Data Relating to the Production of Our Cinnamon Apple

Chips

Date: November 28, 2007

The manufacturing process for our chips begins when the apples are placed into a large vat in which they are automatically washed. After washing, the apples flow directly to equipment that automatically peels and removes the apple’s core. The peeled and decored apples then pass by inspectors who manually cut out deep bruises or other blemishes. After inspection, the apples are automatically sliced and dropped into the cooking oil. The frying process is closely monitored by an employee. After they are cooked, the chips pass by more inspectors, who sort out the unacceptable finished chips (those that are discolored or too small). The chips then continue on the conveyor belt to a bagging machine that bags them in 1 pound bags. The bags are then placed in a box and shipped. Each box holds 16 bags.

The raw apple pieces (bruised and blemished), peelings, and rejected finished chips are sold to animal feed producers for $0.08 per pound. The cores are sold to a juice producer for $0.16 per pound. The company uses this revenue to reduce the cost of apples; we would like this fact reflected in the price standard relating to apples.

Tasty Apple purchases high quality apples at a cost of $0.256 per pound. Each apple averages 4.25 ounces. Under efficient operating conditions, it takes four apples to produce one 16 ounce bag of chips. Although we label bags as containing 16 ounces, we actually place 16.2 ounces in each bag. We plan to continue this policy to ensure customer satisfaction. In addition to apples, other raw materials are the cooking oil, cinnamon, bags, and boxes. Cooking oil costs $0.04 per ounce, and we use 3.3 ounces of oil per bag of chips. The cost of cinnamon is so small that we add it to overhead. Bags cost $0.12 each and boxes $0.62.

Our plant produces 9.2 million bags of chips per year. A recent engineering study revealed that we would need the following direct labor hours to produce this quantity if our plant operates at peak efficiency:

Raw apple inspection

3,150

Finished chip inspection

12,000

Frying monitor

6,300

Machine operators

6,300

Boxing

16,250

I’m not sure that we can achieve the level of efficiency advocated by the study. In my opinion, the plant is operating efficiently for the level of output indicated if the hours allowed are about 10 percent higher.

The hourly labor rates agreed upon with the union are as follows:

Raw apple inspectors

$17.68

Finished chip inspectors

13.00

Frying monitor

16.00

Boxing

13.68

Machine operators

15.00

Overhead is applied on the basis of direct labor dollars. We have found that variable overhead averages about 112 percent of our direct labor cost. Our fixed overhead is budgeted at $2,419,026 for the coming year.

Required:

Form groups of three or four students. Each group should complete the following requirements. One member from each group will rotate to another group. The rotating member has the responsibility of comparing and contrasting the solution of his or her group with that of the group being visited.

1. Discuss the benefits of a standard costing system for Tasty Apple.

2. Discuss the president’s concern about using the result of the engineering study to set the labor standards. What standard would you recommend?

3. Develop a standard cost sheet for Tasty Apple’s cinnamon apple chips.

4. Suppose that the level of production was 9.2 million bags of apple chips for the year as planned. Assuming that 9.8 million pounds of apples were used, compute the direct materials usage variance for apples.

 

what is per case payment 666374

CYBER RESEARCH CASE

SETTING AND USING STANDARDS IN A SERVICE SETTING

Standard costing concepts can also be applied to services. Standard service costs are similar in concept to standard product costs. In the medical field, costs of caring for a patient have been increasing at a high rate for many years. Hospitals, for example, have often been paid on a retrospective basis. Essentially, they have been able to recover (from Medicare or their insurers) most of what they spent in treating a patient. Hospitals have thus had very little incentive to control costs. Some argue that retrospective payments encourage hospitals to acquire new and expensive technology and to offer more and more complex procedures. Prospective payments have emerged as an alternative to retrospective payments. Recently a new type of prospective payment has emerged known as “per case payment.”

Required:

Conduct an Internet search on per case payments, and answer the following questions:

1. What is per case payment?

2. Explain the following: “Per case payment can become a viable payment scheme only if the hospital’s case mix can be properly measured.”

3. Discuss the merits of using diagnostic related groups (DRGs) to measure case mix.

4. Patient management categories (PMCs) have been suggested as an alternative approach to measuring case mix. Define PMCs, and discuss their merits.

5. Describe how the per case payment approaches are forms of standard costing discussed in this chapter.

job cost ernie s electronics inc delivered 1 000 custom designed computer monitors t 666695

Job cost Ernie’s Electronics, Inc., delivered 1,000 custom designed computer monitors to its customer, Video Shack. The following cost information was compiled in connection with this order:

Direct Materials Used

Part A327: 1 unit costing $60 per monitor

Part B149: 1 unit costing $120 per monitor

Direct Labor Used

Assembly: 6 hours per monitor at the rate of $10 per hour

Inspection: 1 hour per monitor at the rate of $12 per hour

In addition, the company applies manufacturing overhead costs to jobs at the rate of $5 per direct labor hour. The selling price for each monitor is $350.

Required

(a) Determine the total cost for this job.

(b) Determine the gross margin per monitor.

job cost the following costs pertain to job 923 at becker auto shop 666696

Job cost The following costs pertain to job 923 at Becker Auto Shop.

 

QUANTITY

PRICE

Direct materials:

 

 

Engine oil

11 ounces

$2 per ounce

Lubricant

2 ounces

3 per ounce

Direct labor

3 hours

15 per hour

Overhead costs (based

 

 

on direct labor hours)

 

10 per hour

Determine the total cost for job 923.

job order costing and cost driver rates the brinker company uses a job order costing 666698

Job order costing and cost driver rates The Brinker Company uses a job order costing system at its local plant. The plant has a machining department and a finishing department. The company uses machine hours to allocate machining department overhead costs to jobs and uses direct labor cost to allocate finishing department overhead costs to jobs. Cost and practical capacity estimates for the current year are as follows:

 

MACHINING DEPARTMENT

FINISHING DEPARTMENT

Manufacturing overhead costs

$350,000

$280,000

Machine hours

14,000

1,400

Direct labor hours

3,500

15,400

Direct labor cost

$105,000

$350,000

Required

(a) Determine the cost driver rate for each department.

(b) Cost records for job 101 show the following:

 

MACHINING DEPARTMENT

FINISHING DEPARTMENT

Direct materials cost

$8,000

$1,400

Direct labor cost

$250

$800

Direct labor hours

7

35

Machine hours

50

6

Determine the total cost charged to job 101.

single rate versus departmental rates eastern wood products has two production depar 666699

Single rate versus departmental rates Eastern Wood Products has two production departments: cutting and assembly. The company has been using a plantwide cost driver rate computed by dividing plantwide overhead costs by total plantwide direct labor hours. The estimates for overhead costs and practical capacity quantities of cost drivers for the current year follow:

 

CUTTING

ASSEMBLY

TOTAL

Manufacturing overhead

$25,000

$35,000

$60,000

Direct labor hours

1,000

3,000

4,000

Machine hours

4,000

2,000

6,000

Required

(a) Compute the plantwide cost driver rate.

(b) Determine departmental cost driver rates based on direct labor hours for assembly and machine hours for cutting.

(c) Provide reasons why Eastern Wood might use the method in part a or the one in part b.

fluctuating cost driver rates and effect on markup pricing morrison company carefull 666700

Fluctuating cost driver rates and effect on markup pricing Morrison Company carefully records its costs because it bases prices on the cost of the goods it manufactures. Morrison also carefully records its machine usage and other operational information. Manufacturing costs are computed monthly, and prices for the next month are determined by adding a 20% markup to each product’s manufacturing costs. The cost driver rate is based on machine hours as follows:

MONTH

ACTUAL MACHINE HOURS

MONTH

ACTUAL MACHINE HOURS

January

1,350

July

1,400

February

1,400

August

1,400

March

1,500

September

1,500

April

1,450

October

1,600

May

1,450

November

1,600

June

1,400

December

1,600

Profits have been acceptable until the past year, when Morrison began to face increased competition. The marketing manager reported that Morrison’s sales force finds the company’s pricing puzzling. When demand is high, the company’s prices are low, and when demand is low, the company’s prices are high. Practical capacity is 1,500 machine hours per month. Practical capacity is exceeded in some months by operating the machines overtime beyond regular shift hours. Monthly machine related overhead costs, all fixed, are $70,000 per month.

Required

(a) Compute the monthly overhead cost driver rates that Morrison used last year.

(b) Suggest a better approach to developing cost driver rates for Morrison and explain why your method is better.

process costs pitman chemical company manufactures and sells goody a product that se 666702

Process costs Pitman Chemical Company manufactures and sells Goody, a product that sells for $10 per pound. The manufacturing process also yields 1 pound of a waste product, called Baddy, in the production of every 10 pounds of Goody. Disposal of the waste product costs $1 per pound. During March, the company manufactured 200,000 pounds of Goody. Total manufacturing costs were as follows:

Direct materials

$232,000

Direct labor

120,000

Manufacturing overhead costs

60,000

Total costs

$412,000

Determine the cost per pound of Goody.

process costing equivalent units the information below pertains to july production a 666703

Process costing equivalent units The information below pertains to July production at Porter Company’s paint factory, which produces paints for household interiors:

 

GALLONS

MATERIALS

CONVERSION

Work in process, July 1

3,000

30% complete

20% complete

Started in July

7,000

 

 

To account for

10,000

 

 

Completed and transferred out

6,000

100% complete

100% complete

Work in process, July 31

4,000

25% complete

10% complete

Accounted for

10,000

 

 

Using the weighted average method, determine the number of equivalent units of production for materials and conversion during July.

service department cost allocation direct method san rafael company has two producti 666704

Service department cost allocation, direct method San Rafael Company has two production departments, assembly and finishing, and two service departments, machine setup and inspection. Machine setup costs are allocated on the basis of number of setups, whereas inspection costs are allocated on the basis of number of direct labor hours. Selected information on the four departments follows:

ITEM

DIRECT COSTS

NUMBER OF SETUPS

DIRECT LABOR HOURS

Machine setup

$40,000

0

0

Inspection

15,000

0

0

Assembly

25,000

300

200

Finishing

20,000

100

500

Required

(a) Using the direct method, determine the amount of machine setup costs allocated to the two production departments.

(b) Using the direct method, determine the amount of inspection costs allocated to the two production departments.

sequential allocation carleton company has two service departments and two productio 666705

Sequential allocation Carleton Company has two service departments and two production departments. Information on annual manufacturing overhead costs and cost drivers follows:

 

SERVICE DEPARTMENTS

PRODUCTION DEPARTMENTS

ITEM

S1

S2

P1

P2

Overhead costs

$65,000

$55,000

$160,000

$240,000

Direct labor hours

2,000

1,500

2,000

3,000

Number of square feet

800

1,200

2,400

2,600

The company allocates service department costs using the sequential method. First, S1 costs are allocated on the basis of direct labor hours. Next, S2 costs are allocated on the basis of square footage. The square footage for S1 is assumed to be zero for this purpose. Determine the total overhead costs allocated to each of the two production departments.

direct sequential and reciprocal allocation ming company has two service departments 666706

Direct, sequential, and reciprocal allocation Ming Company has two service departments (S1 and S2) and two production departments (P1 and P2). Last year, directly identified overhead costs were $300,000 for S1 and $300,000 for S2. Information on the consumption of their services follows:

 

USER DEPARTMENTS

SUPPLYING DEPARTMENTS

S1

S2

P1

P2

S1

0%

40%

30%

30%

S2

25%

0%

25%

50%

Required

(a) Determine the service department costs allocated to the two production departments using the direct method.

(b) Determine the service department costs allocated to the two production departments using the sequential method beginning with the allocation of S1 department costs.

(c) Determine the service department costs allocated to the two production departments using the reciprocal method.

what insight does management gain from the approach in part d as compared to the app 666707

Reconciling actual and applied capacity costs Hoyt Company uses a plantwide cost driver rate with machine hours as the cost driver. At the beginning of last year, Hoyt Company estimated its capacity related (overhead) costs as $15,000,000 for a practical capacity of 100,000 machine hours per year. During the year, actual overhead costs were $14,200,000 and production required 90,000 machine hours.

Required

(a) Determine Hoyt Company’s plantwide cost driver rate and calculate the overhead cost applied to production last year.

(b) Suppose the company charges the difference between actual and applied overhead costs to cost of goods sold at the end of the year. Calculate the difference and state whether the result will be an increase or decrease in the previously recorded cost of goods sold.

(c) Suppose now that the company prorates the difference between actual and applied overhead costs to work in process, finished goods, and cost of goods sold. If the proportions of applied indirect cost in the ending balances of these accounts this period are 20% in ending work in process, 45% in finished goods inventory, and 35% in cost of goods sold, by how much will the three accounts be increased or decreased from their previously recorded amounts?

(d) Now suppose that the company wishes to decompose the difference between actual and applied overhead costs to gain further insight into the difference. Compute the difference between actual and estimated overhead cost and the difference between estimated overhead cost and applied overhead cost.

(e) What insight does management gain from the approach in part d as compared to the approaches in parts b and c?

job costing for services the hillman company sells and services lawn mowers snow blo 666708

Job costing for services The Hillman Company sells and services lawn mowers, snow blowers, and other equipment. The service department uses a job order cost system to determine the cost of each job, such as oil changes, tune ups, and repairs. The department assigns conversion costs through a cost driver rate on the basis of direct labor hours. The cost driver rate additionally includes a markup of 25% on the job’s conversion costs in order to provide a reasonable profit for Hillman. The customer’s invoice itemizes prices for parts and labor, where the stated labor rate is the department’s cost driver rate, which includes direct labor costs, assigned overhead costs, and the 25% markup on conversion costs. Hillman Company’s service department estimated the following information for the current year:

Salaries of mechanics

$120,000

Fringe benefits

54,000

General and administrative

18,000

Depreciation

42,000

Billable direct labor hours

4,500

Required

(a) Determine Hillman Company’s service department’s cost driver rate for assigning conversion costs on the basis of billable direct labor hours.

(b) Job 254 required $47.40 of materials and 0.7 direct labor hour. Determine the price charged for job 254.

job costing and departmental cost driver rates the leblanc company employs a job ord 666709

Job costing and departmental cost driver rates The Leblanc Company employs a job order costing system to account for its costs. The company has three production departments. Separate departmental cost driver rates are employed because the demand for overhead resources for the three departments is very different. All jobs generally pass through all three production departments. Data regarding the hourly direct labor rates, cost driver rates, and three jobs for which work was done during June appear below. Jobs 101 and 102 were completed during June, while job 103 was not completed as of June 30. The costs charged to jobs not completed at the end of a month are shown as work in process inventory at the end of that month and at the beginning of the next month:

PRODUCTION DEPARTMENTS

DIRECT LABOR RATES

COST DRIVER RATES

Department 1

$12

150% of direct material cost

Department 2

18

$8 per machine hour

Department 3

15

200% of direct labor cost

 

JOB 101

JOB 102

JOB 103

Beginning work in process

$25,500

$32,400

$0

Direct materials:

 

 

 

Department 1

$40,000

$26,000

$58,000

Department 2

3,000

5,000

14,000

Department 3

0

0

0

Direct labor hours:

 

 

 

Department 1

500

400

300

Department 2

200

250

350

Department 3

1,500

1,800

2,500

Machine hours:

 

 

 

Department 1

0

0

0

Department 2

1,200

1,500

2,700

Department 3

150

300

200

Required

(a) Determine the total cost of completed job 101.

(b) Determine the total cost of completed job 102.

(c) Determine the ending balance of work in process inventory for job 103 as of June 30.

allocating scheduling service costs airport coach service company operates scheduled 666710

Allocating scheduling service costs Airport Coach Service Company operates scheduled coach service from Boston’s Logan Airport to downtown Boston and to Cambridge. A common scheduling service center at the airport is responsible for ticketing and customer service for both routes. The service center is regularly staffed to service traffic of 2,400 passengers per week: two thirds for downtown Boston passengers and the balance for Cambridge passengers. The cost to operate this service center is $7,200 per week normally, but it is higher during weeks when additional help is required to service higher traffic levels. The service center costs and number of passengers serviced during the weeks of August follow:

WEEK

COST

BOSTON PASSENGERS

CAMBRIDGE PASSENGERS

1

$7,200

1,600

800

2

7,200

1,500

900

3

7,600

1,650

800

4

7,800

1,700

850

5

7,200

1,700

700

Required

(a) How much of the service center costs will be allocated to the Boston service and to the Cambridge service if the costs are allocated in proportion to the number of actual passengers?

(b) Suggest an improved approach to allocating the costs and explain why your method is an improvement. Using your approach, how much of the service center costs will be charged to the Boston service and to the Cambridge service?

job cost markup and single rate versus departmental rates modern metalworks company 666711

Job cost, markup, and single rate versus departmental rates Modern Metalworks Company has two departments, milling and assembly. The company uses a job costing system with a plantwide cost driver rate that is computed by dividing plantwide overhead costs by total plantwide practical capacity direct labor hours. The following cost and practical capacity estimates are for October:

 

MILLING

ASSEMBLY

Overhead costs

$120,000

$160,000

Direct labor hours

8,000

12,000

Machine hours

12,000

6,000

The following information pertains to job 714, which was started and completed during October:

 

MILLING

ASSEMBLY

Direct labor hours

10

40

Machine hours

18

8

Direct materials costs

$800

$50

Direct labor costs

$100

$600

Required

(a) Determine the cost of job 714.

(b) Suppose that instead of using the plantwide cost driver rate, the company uses machine hours as the cost driver for applying overhead costs in the milling department, and uses direct labor hours as the cost driver in the assembly department. Compute these departmental cost driver rates and determine the cost of job 714 using these rates.

(c) Using the costs you computed in parts a and b, determine the bid price that Modern Metalworks will quote under each cost system if it uses a 25% markup on total manufacturing cost.

(d) Provide reasons why Modern Metalworks might prefer the method in part a or the one in part b.

single rate versus departmental rates bravo steel company supplies structural steel 666712

Single rate versus departmental rates Bravo Steel Company supplies structural steel products to the construction industry. Its plant has three production departments: cutting, grinding, and drilling. The estimated overhead cost and practical capacity direct labor hours and machine hours for each department for the current year follow:

 

CUTTING

GRINDING

DRILLING

Overhead cost

$504,000

$2,304,000

$2,736,000

Estimated direct labor hours

60,000

96,000

144,000

Estimated machine hours

960,000

480,000

360,000

Job ST101 consumed the following direct labor and machine hours:

 

CUTTING

GRINDING

DRILLING

Direct labor hours

2,000

2,500

3,000

Machine hours

20,000

3,000

2,000

Required

(a) Suppose that a plantwide cost driver rate is computed by dividing plantwide overhead costs by plantwide practical capacity direct labor hours. Determine the overhead cost applied to job ST101.

(b) Determine the departmental cost driver rates and overhead costs applied to job ST101 if machine hours are used as the cost driver in the cutting department and direct labor hours are used as the cost driver for the grinding and drilling departments.

(c) Explain why Bravo Steel might prefer a plantwide rate or departmental cost driver rates.

job costing the gonzalez company uses a job order costing system at its plant in gre 666713

Job costing The Gonzalez Company uses a job order costing system at its plant in Green Bay, Wisconsin. The plant has a machining department and a finishing department. The company uses two cost driver rates for allocating manufacturing overhead costs to job orders: one on the basis of machine hours for allocating machining department overhead costs and the other on the basis of direct labor cost for allocating the finishing department overhead costs. Estimates for the current year follow:

 

MACHINING DEPARTMENT

FINISHING DEPARTMENT

Manufacturing overhead cost

$500,000

$400,000

Machine hours

20,000

2,000

Direct labor hours

5,000

22,000

Direct labor cost

$150,000

$500,000

Required

(a) Determine the two departmental cost driver rates.

(b) Last month, cost records for job 511 show the following:

 

MACHINING DEPARTMENT

FINISHING DEPARTMENT

Direct materials cost

$12,000

$2,000

Direct labor cost

$300

$1,200

Direct labor hours

10

50

Machine hours

80

8

Determine the total costs charged to job 511.

(c) Explain why Gonzalez Company uses two different cost driver rates in its job costing system.

the bartlesville plant of harmon company produces an industrial chemical at the begi 666361

DIRECT MATERIALS, DIRECT LABOR, AND OVERHEAD VARIANCES, JOURNAL ENTRIES

The Bartlesville plant of Harmon Company produces an industrial chemical. At the beginning of the year, the Bartlesville plant had the following standard cost sheet:

Direct materials (10 lbs. @ $1.60)

$16.00

Direct labor (0.75 hr. @ $18.00)

13.50

Fixed overhead (0.75 hr. @ $4.00)

3.00

Variable overhead (0.75 hr. @ $3.00)

2.25

Standard cost per unit

$34.75

The Bartlesville plant computes its overhead rates using practical volume, which is 72,000 units. The actual results for the year are as follows:

a. Units produced: 70,000.

b. Direct materials purchased: 744,000 pounds at $1.50 per pound.

c. Direct materials used: 736,000 pounds.

d. Direct labor: 56,000 hours at $17.90 per hour.

e. Fixed overhead: $214,000.

f. Variable overhead: $175,400.

Required:

1. Compute price and usage variances for direct materials.

2. Compute the direct labor rate and labor efficiency variances.

3. Compute the fixed overhead spending and volume variances. Interpret the volume variance.

4. Compute the variable overhead spending and efficiency variances.

5. Prepare journal entries for the following:

a. The purchase of direct materials.

b. The issuance of direct materials to production (Work in Process).

c. The addition of direct labor to Work in Process.

d. The addition of overhead to Work in Process.

e. The incurrence of actual overhead costs.

f. Closing out of variances to Cost of Goods Sold.

 

misterio company uses a standard costing system during the past quarter the followin 666362

SOLVING FOR UNKNOWNS

Misterio Company uses a standard costing system. During the past quarter, the following variances were computed:

Variable overhead efficiency variance

$ 24,000 U

Direct labor efficiency variance

120,000 U

Direct labor rate variance

10,400 U

Misterio applies variable overhead using a standard rate of $2 per direct labor hour allowed. Two direc labor hours are allowed per unit produced. (Only one type of product is manufactured.) During th quarter, Misterio used 30 percent more direct labor hours than should have been used

Required:

1. What were the actual direct labor hours worked? The total hours allowed?

2. What is the standard hourly rate for direct labor? The actual hourly rate?

3. How many actual units were produced?

 

during the year nosemer experienced the following activity relative to the productio 666363

BASIC VARIANCE ANALYSIS, REVISIONOF STANDARDS, JOURNAL ENTRIE

Nosemer Company produces engine parts for large motors. The company uses a standard cost system for production costing and control. The standard cost sheet for one of its higher volume products (a valve), is as follows:

Direct materials (5 lbs. @ $4.00)

$20.00

Direct labor (1.4 hrs. @ $10.50)

14.70

Variable overhead (1.4 hrs. @ $6.00)

8.40

Fixed overhead (1.4 hrs. @ $3.00)

4.20

Standard unit cost

$47.30

During the year, Nosemer experienced the following activity relative to the production of valves:

a. Production of valves totaled 25,000 units.

b. A total of 130,000 pounds of direct materials was purchased at $3.70 per pound.

c. There were 10,000 pounds of direct materials in beginning inventory (carried at$4 per pound). There was no ending inventory.

d. The company used 36,500 direct labor hours at a total cost of $392,375.

e. Actual fixed overhead totaled $95,000.

f. Actual variable overhead totaled $210,000.

Nosemer produces all of its valves in a single plant. Normal activity is 22,500 units per year. Standard overhead rates are computed based on normal activity measured in standard direct labor hours.

Required:

1. Compute the direct materials price and usage variances.

2. Compute the direct labor rate and efficiency variances.

3. Compute overhead variances using a 2 variance analysis.

4. Compute overhead variances using a 4 variance analysis.

5. Assume that the purchasing agent for the valve plant purchased a lower quality direct material from a new supplier. Would you recommend that the company continue to use this cheaper direct material? If so, what standards would likely need revision to reflect this decision? Assume that the end product’s quality is not significantly affected.

6. Prepare all possible journal entries (assuming a 4 variance analysis of overhead variances).

 

for every small stapler produced the company produces two regular staplers actual op 666364

UNIT COSTS, MULTIPLE PRODUCTS,VARIANCE ANALYSIS, JOURNAL ENTRIES

Business Specialty, Inc., manufactures two staplers: small and regular. The standard quantities of direct labor and direct materials per unit for the year are as follows:

 

Small

Regular

Direct materials (oz.)

6.0

10.00

Direct labor (hrs.)

0.1

0.15

The standard price paid per pound of direct materials is $1.60. The standard rate for labor is $8.00. Overhead is applied on the basis of direct labor hours. A plantwide rate is used. Budgeted overhead for the year is as follows:

Budgeted fixed overhead

$360,000

Budgeted variable overhead

480,000

The company expects to work 12,000 direct labor hours during the year; standard overhead rates are computed using this activity level. For every small stapler produced, the company produces two regular staplers.

Actual operating data for the year are as follows:

a. Units produced: small staplers, 35,000; regular staplers, 70,000.

b. Direct materials purchased and used: 56,000 pounds at $1.55—13,000 for the small stapler and 43,000 for the regular stapler. There were no beginning or ending direct materials inventories.

c. Direct labor: 14,800 hours—3,600 hours for the small stapler; 11,200 hours for the regular stapler. Total cost of direct labor: $114,700.

d. Variable overhead: $607,500.

e. Fixed overhead: $350,000.

Required:

1. Prepare a standard cost sheet showing the unit cost for each product.

2. Compute the direct materials price and usage variances for each product. Prepare journal entries to record direct materials activity.

3. Compute the direct labor rate and efficiency variances. Prepare journal entries to record direct labor activity.

4. Compute the variances for fixed and variable overhead. Prepare journal entries to record overhead activity. All variances are closed to Cost of Goods Sold.

5. Assume that you know only the total direct materials used for both products and the total direct labor hours used for both products. Can you compute the total direct materials and direct labor usage variances? Explain.

limpio inc produces a key ingredient for liquid laundry detergents two chemical solu 666365

DIRECT MATERIALS USAGE VARIANCE, DIRECT MATERIALS MIX AND YIELD VARIANCES

Limpio, Inc., produces a key ingredient for liquid laundry detergents. Two chemical solutions, Chem A and Chem B, are mixed and heated to produce a cleansing chemical that is sold to companies that produce liquid detergents. The cleansing ingredient is produced in batches and has the following standards:

Direct Material

Standard Mix

Standard Unit Price

Standard Cost

Chem A

15,000 gallons

$2.00 per gallon

$30,000

Chem B

5,000

3.00

 15,000

 

Total

20,000 gallons

 

$45,000

Yield

15,000 gallons

   

During March, the following actual production information was provided:

Direct Material

Actual Mix

Chem A

140,000 gallons

Chem B

60,000

 

Total

200,000 gallons

Yield

158,400 gallons

Required:

1. Compute the direct materials mix and yield variances.

2. Compute the total direct materials usage variance for Chem A and Chem B. Show that the total direct materials usage variance is equal to the sum of the direct materials mix and yield variances.

compute the total direct labor efficiency variance show that the total direct labor 666366

DIRECT LABOR EFFICIENCY VARIANCE, DIRECT LABOR MIX AND YIELD VARIANCES

Limpio, Inc., also uses two different types of direct labor in producing the cleansing chemical: mixing and drum filling labor (the completed product is placed into 50 gallon drums). For each batch of 20,000 gallons of direct materials input, the following standards have been developed for direct labor:Limpio, Inc., produces a key ingredient for liquid laundry detergents. Two chemical solutions, Chem A and Chem B, are mixed and heated to produce a cleansing chemical that is sold to companies that produce liquid detergents. The cleansing ingredient is produced in batches and has the following standards:

Direct Labor Type

Mix

SP

Standard Cost

Mixing

2,000 hrs.

$11.00

$22,000

Drum filling  Yield

1,000

8.00

8,000

Total

3,000 hrs.

 

$30,000

Yield

15,000 gallons    

The actual direct labor hours used for the output produced in March are also provided:

Labor Type

Mix

Mixing

18,000 hrs.

Drum filling

12,000

Total

30,000 hrs.

Yield

158,400 gallons

Required:

1. Compute the direct labor mix and yield variances.

2. Compute the total direct labor efficiency variance. Show that the total direct labor efficiency variance is equal to the sum of the direct labor mix and yield variances.

compute the total direct materials usage variance and then break down this variance 666367

DIRECT MATERIALS USAGE VARIANCES, DIRECT MATERIALS MIX AND YIELD VARIANCES

Energy Products Company produces a gasoline additive, Gas Gain. This product increases engine efficiency and improves gasoline mileage by creating a more complete burn in the combustion process.

Careful controls are required during the production process to ensure that the proper mix of input chemicals is achieved and that evaporation is controlled. If the controls are not effective, there can be a loss of output and efficiency.

The standard cost of producing a 500 liter batch of Gas Gain is $135. The standard direct materials mix and related standard cost of each chemical used in a 500 liter batch are as follows:

Chemical

Mix

SP

Standard Cost

Echol

200 liters

$0.200

$ 40.00

Protex

100

0.425

42.50

Benz

250

0.150

37.50

CT 40

50

0.300

15.00

Total

600 liters

 

$135.00

The quantities of chemicals purchased and used during the current production period are shown in the following schedule. A total of 140 batches of Gas Gain were manufactured during the current production period. Energy Products determines its cost and chemical usage variations at the end of each production period.

Chemical

Quantity Used

Echol

26,600 liters

Protex

12,880

Benz

37,800

CT 40

7,140

Total

84,420 liters

Required:

Compute the total direct materials usage variance, and then break down this variance into its mix and yield components. 

nuevo company produces a single product nuevo employs a standard cost system and use 666368

SOLVING FOR UNKNOWNS, OVERHEAD ANALYSIS

Nuevo Company produces a single product. Nuevo employs a standard cost system and uses a flexible budget to predict overhead costs at various levels of activity. For the most recent year, Nuevo used a standard overhead rate equal to $6.25 per direct labor hour. The rate was computed using expected activity. Budgeted overhead costs are $80,000 for 10,000 direct labor hours and $120,000 for 20,000 direct labor hours. During the past year, Nuevo generated the following data:

a. Actual production: 4,000 units.

b. Fixed overhead volume variance: $1,750 U.

c. Variable overhead efficiency variance: $3,200 F.

d. Actual fixed overhead costs: $41,335.

e. Actual variable overhead costs: $70,000.

Required:

1. Determine the fixed overhead spending variance.

2. Determine the variable overhead spending variance.

3. Determine the standard hours allowed per unit of product.

4. Assuming the standard labor rate is $9.50 per hour, compute the direct labor efficiency variance.

 

there were no beginning or ending inventories of direct materials the direct materia 666369

FLEXIBLE BUDGET, STANDARD COST VARIANCES, T ACCOUNTS

Correr Company manufactures a line of running shoes. At the beginning of the period, the following plans for production and costs were revealed:

Units to be produced and sold

25,000

Standard cost per unit:

 
Direct materials $10

Direct labor

8

Variable overhead

4

Fixed overhead

3

Total unit cost

$25

During the year, 30,000 units were produced and sold. The following actual costs were incurred:

Direct materials

$320,000

Direct labor

220,000

Variable overhead

125,000

Fixed overhead

89,000

There were no beginning or ending inventories of direct materials. The direct materials price variance was $5,000 unfavorable. In producing the 30,000 units, a total of 39,000 hours were worked, 4 percent more hours than the standard allowed for the actual output. Overhead costs are applied to production using direct labor hours.

Required:

1. Prepare a performance report comparing expected costs to actual costs.

2. Determine the following:

a. Direct materials usage variance.

b. Direct labor rate variance.

c. Direct labor usage variance.

d. Fixed overhead spending and volume variances.

e. Variable overhead spending and efficiency variances.

3. Use T accounts to show the flow of costs through the system. In showing the flow, you do not need to show detailed overhead variances. Show only the overand underapplied variances for fixed and variable overhead.

as part of its cost control program tracer company uses a standard costing system fo 666370

STANDARD COSTING: PLANNED VARIANCES

As part of its cost control program, Tracer Company uses a standard costing system for all manufactured items. The standard cost for each item is established at the be ginning of the fiscal year, and the standards are not revised until the beginning of the next fiscal year. Changes in costs, caused during the year by changes in direct materials or direct labor inputs or by changes in the manufacturing process, are recognized as they occur by the inclusion of planned variances in Tracer’s monthly operating budgets.

The following direct labor standard was established for one of Tracer’s products, effective June 1, 2007, the beginning of the fiscal year:

Assembler A labor (5 hrs. @ $10)

$ 50

Assembler B labor (3 hrs. @ $11)

33

Machinist labor (2 hrs. @ $15)

30

Standard cost per 100 units

$113

The standard was based on the direct labor being performed by a team consisting of five persons with Assembler A skills, three persons with Assembler B skills, and two persons with machinist skills; this team represents the most efficient use of the company’s skilled employees. The standard also assumed that the quality of direct materials that had been used in prior years would be available for the coming year.

For the first seven months of the fiscal year, actual manufacturing costs at Tracer have been within the standards established. However, the company has received a significant increase in orders, and there is an insufficient number of skilled workers to meet the increased production. Therefore, beginning in January, the production teams will consist of eight persons with Assembler A skills, one person with Assembler B skills, and one person with machinist skills. The reorganized teams will work more slowly than the normal teams, and as a result, only 80 units will be produced in the same time period in which 100 units would normally be produced. Faulty work has never been a cause for units to be rejected in the final inspection process, and it is not expected to be a cause for rejection with the reorganized teams.

Furthermore, Tracer has been notified by its direct materials supplier that lower quality direct materials will be supplied beginning January 1. Normally, one unit of direct materials is required for each good unit produced, and no units are lost due to defective direct materials. Tracer estimates that 6 percent of the units manufactured after January 1 will be rejected in the final inspection process due to defective direct materials.

Required:

1. Determine the number of units of lower quality direct materials that Tracer Company must enter into production in order to produce 47,000 good finished units.

2. How many hours of each class of direct labor must be used to manufacture 47,000 good finished units?

3. Determine the amount that should be included in Tracer’s January operating budget for the planned direct labor variance caused by the reorganization of the direct labor teams and the lower quality direct materials.

 

explain why the fifo method is used for process costing when a standard costing syst 666371

VARIANCE ANALYSIS IN A PROCESS COSTING SETTING SERVICE FIRM

Aspen Medical Laboratory performs comprehensive blood tests for physicians and clinics throughout the Southwest. Aspen uses a standard process costing system for its comprehensive blood work. Skilled technicians perform the blood tests. Because Aspen uses a standard costing system, equivalent units are calculated using the FIFO method. The standard cost sheet for the blood test follows (these standards were used throughout the calendar year):

Direct materials (4 oz. @ $4.50)

$18

Direct labor (2 hrs. @ $18.00)

36

Variable overhead (2 hrs. @ $5.00)

10

Fixed overhead (2 hrs. @ $10.00)

20

Standard cost per test

$84

For the month of November, Aspen reported the following actual results:

a. Beginning work in process: 1,250 tests, 60 percent complete.

b. Tests started: 25,000.

c. Ending work in process: 2,500 tests, 40 percent complete.

d. Direct labor: 47,000 hours at $19 per hour.

e. Direct materials purchased and used: 102,000 at $4.25 per ounce.

f. Variable overhead: $144,000.

g. Fixed overhead: $300,000.

h. Direct materials are added at the beginning of the process.

Required:

1. Explain why the FIFO method is used for process costing when a standard costing system has been adopted.

2. Calculate the cost of goods transferred out (tests completed and transferred out) for the month of November. Does standard costing simplify process costing? Explain.

3. Calculate price and quantity variances for direct materials and direct labor.

compute the standard pounds of direct materials allowed for the production of 300 00 666351

COMPUTATION OF INPUTS ALLOWED, DIRECT MATERIALS AND DIRECT LABOR During the year, Vandy Company produced 300,000 drilling components for oil and gas rigs. Vandy’s direct materials and direct labor standards are as follows:

Direct materials (6.25 lbs. @ $4)

$25.00

Direct labor (1.5 hrs. @ $13)

19.50

Required:

1. Compute the standard pounds of direct materials allowed for the production of 300,000 units.

2. Compute the standard direct labor hours allowed for the production of 300,000 units.

prepare the journal entries associated with direct materials and direct labor 666352

DIRECT MATERIALS AND DIRECT LABOR VARIANCES Choco Company produces a popular candy bar called Megusta. The candy is produced in Cost Rica and exported to the United States. Recently, the company adopted the following standards for one 5 ounce bar of the candy:

Direct materials (5.5 oz. @ $0.06)

$0.33

Direct labor (0.05 hr. @ $2.00)

0.10

Standard prime cost

$0.43

During the first week of operation, the company experienced the following actual results:

a. Bars produced: 150,000.

b. Ounces of direct materials purchased: 855,000 ounces at $0.055.

c. There are no beginning or ending inventories of direct materials.

d. Direct labor: 7,800 hours at $2.25.

Required:

1. Compute price and usage variances for direct materials.

2. Compute the rate variance and the efficiency variance for direct labor.

3. Prepare the journal entries associated with direct materials and direct labor.

how is the variable overhead efficiency variance related to the direct labor efficie 666353

OVERHEAD VARIANCES, FOUR VARIANCE ANALYSIS Young, Inc., uses a standard costing system and develops its overhead rates from the current annual budget. The budget is based on an expected annual output of 220,000 units requiring 1,100,000 direct labor hours. (Practical capacity is 1,210,000 hours.) Annual budgeted overhead costs total $962,500, of which $412,500 is fixed overhead.

A total of 228,800 units using 1,188,000 direct labor hours was produced during the year. Actual variable overhead costs for the year were $572,000, and actual fixed overhead costs were $440,000.

Required:

1. Compute the fixed overhead spending and volume variances. How would you interpret the spending variance? Discuss the possible interpretations of the volume variance. Which is most appropriate for this example?

2. Compute the variable overhead spending and efficiency variances. How is the variable overhead spending variance like the price variances of direct labor and direct materials? How is it different? How is the variable overhead efficiency variance related to the direct labor efficiency variance?

direct materials mix and yield variances verde sabor produces a green enchilada sauc 666354

DIRECT MATERIALS MIX AND YIELD VARIANCES Verde Sabor produces a green enchilada sauce using tomatoes and green chili peppers. Verde developed the following standard cost sheet:

Direct Material

Mix

Mix Proportion

SP

Standard Cost

Tomatoes

630 ounces

0.90

$0.020

$12.60

Chili peppers

70

0.10

0.026

1.82

Total

700 ounces

 

 

$14.42

Yield

577.5 ounces

 

 

 

On March 2, Verde produced a batch of 112,000 ounces with the following actual results:

Direct Material

Actual Mix

Tomatoes

89,600 ounces

Chili peppers

22,400

Total

112,000 ounces

Yield

88,900 ounces

Required:

1. Calculate the yield ratio.

2. Calculate the standard cost per unit of yield.

3. Calculate the direct materials yield variance.

4. Calculate the direct materials mix variance.

prepare the journal entries for the direct materials and direct labor variances 666356

DIRECT LABOR AND DIRECT MATERIALS VARIANCES, JOURNAL ENTRIES Molano Company produces ponchos. The company has established the following direct materials and direct labor standards for one poncho:

Wool (3 yds. @ $3)

$ 9.00

Labor (3.5 hrs. @ $5)

17.50

Total prime cost

$26.50

During the first quarter of the year, Molano produced 25,000 ponchos. The company purchased and used 78,200 yards of wool at $2.90 per yard. Actual direct labor used was 90,000 hours at $5.20 per hour.

Required:

1. Calculate the direct materials price and usage variances.

2. Calculate the direct labor rate and efficiency variances.

3. Prepare the journal entries for the direct materials and direct labor variances.

4. Describe how flexible budgeting variances relate to the direct materials and direct labor variances computed in Requirements 1 and 2.

who is responsible what action should be taken how would your answer change if the u 666357

INVESTIGATION OF VARIANCES Franklin Company uses the following rule to determine whether direct labor efficiency variances ought to be investigated. A direct labor efficiency variance will be investigated anytime the amount exceeds the lesser of $16,000 or 10 percent of the standard labor cost. Reports for the past five weeks provided the following information:

Week

LEV

Standard Labor Cost

1

$14,000 F

$160,000

2

15,600 U

150,000

3

12,000 F

160,000

4

18,000 U

170,000

5

14,000 U

138,000

Required:

1. Using the rule provided, identify the cases that will be investigated.

2. Suppose that investigation reveals that the cause of an unfavorable direct labor efficiency variance is the use of lower quality direct materials than are usually used. Who is responsible? What corrective action would likely be taken?

3. Suppose that investigation reveals that the cause of a significant favorable direct labor efficiency variance is attributable to a new approach to manufacturing that takes less labor time but causes more direct materials waste. Upon examining the direct materials usage variance, it is discovered to be unfavorable, and it is larger than the favorable direct labor efficiency variance. Who is responsible? What action should be taken? How would your answer change if the unfavorable variance were smaller than the favorable?

consider the following environmental factors 666358

OVERHEAD VARIANCES, FOUR VARIANCE ANALYSIS, JOURNAL ENTRIES

Jackman, Inc., uses a standard costing system. The predetermined overhead rates are calculated using practical capacity. Practical capacity for a year is defined as 1,000,000 units requiring 250,000 standard direct labor hours. Budgeted overhead for the year is $750,000, of which $300,000 is fixed overhead. During the year, 900,000 units were produced using 230,000 direct labor hours. Actual annual overhead costs totaled $800,000, of which $300,000 is fixed overhead.

Required:

1. Calculate the fixed overhead spending and volume variances. Explain the meaning of the volume variance to the manager of Jackman.

2. Calculate the variable overhead spending and efficiency variances. Is the spending variance the same as the direct materials price variance? If not, explain how it differs.

3. Prepare the journal entries that reflect the following:

a. Assignment of overhead to production.

b. Recognition of the incurrence of actual overhead.

c. Recognition of overhead variances.

d. Closing out overhead variances, assuming they are not material.

compute the costs of leather and direct labor that should have been incurred for the 666359

STANDARD COSTS, DECOMPOSITION OF BUDGET VARIANCES, DIRECT MATERIALS AND DIRECT LABOR

Vaquero Corporation produces cowboy boots. The company uses a standard costing system and has set the following standards for direct materials and direct labor (for one pair of boots):

Leather (6 strips @ $10)

$60

Direct labor (2 hrs. @ $12)

24

Total prime cost

$84

During the year, Vaquero produced 8,000 pairs of boots. The actual leather purchased was 49,600 strips at $9.98 per strip. There were no beginning or ending inventories of leather. Actual direct labor was 16,800 hours at $12.25 per hour.

Required:

1. Compute the costs of leather and direct labor that should have been incurred for the production of 8,000 pairs of boots.

2. Compute the total budget variances for direct materials and direct labor.

3. Break down the total budget variance for direct materials into a price variance and a usage variance.

Prepare the journal entries associated with these variances.

4. Break down the total budget variance for direct labor into a rate variance and an efficiency variance. Prepare the journal entries associated with these variances.

calculate the standard fixed overhead rate and the standard variable overhead rate 666360

OVERHEAD APPLICATION, OVERHEAD VARIANCES, JOURNAL ENTRIES

Iverson Company produces microwave ovens. Iverson’s plant in Buffalo uses a standard costing system. The standard costing system relies on direct labor hours to assign overhead costs to production. The direct labor standard indicates that four direct labor hours should be used for every microwave unit produced. (The Buffalo plant produces only one model.) The normal production volume is 120,000 units. The budgeted overhead for the coming year is as follows:

Fixed overhead

$1,286,400

Variable overhead

888,000*

Iverson applies overhead on the basis of direct labor hours.

During the year, Iverson produced 119,000 units, worked 487,900 direct labor hours, and incurred actual fixed overhead costs of $1.3 million and actual variable overhead costs of $927,010.

Required:

1. Calculate the standard fixed overhead rate and the standard variable overhead rate.

2. Compute the applied fixed overhead and the applied variable overhead. What is the total fixed overhead variance? Total variable overhead variance?

3. Break down the total fixed overhead variance into a spending variance and a volume variance. Discuss the significance of each.

4. Compute the variable overhead spending and efficiency variances. Discuss the significance of each.

5. Now assume that Iverson’s cost accounting system reveals only the total actual overhead. In this case, a 3 variance analysis can be performed. Using the relationships between a 3 and 4 variance analysis, indicate the values for the three overhead variances.

6. Prepare the journal entries that would be related to fixed and variable overhead during the year and at the end of the year. Assume variances are closed to Cost of Goods Sold.

a company produces a single product in three sizes a b and c prepare a statement sho 666308

A company produces a single product in three sizes, A, B and C. Prepare a statement showing the selling and distribution expenses apportioned over three sizes on the basis indicated and express the total appropriated to each size as,

I] Cost per unit sold II] A percentage of sales turnover and III] Cost per cubic meter of product sold. The expenses and basis of apportionment are as follows,

 

Expenses

Amount Rs.

Basis of Apportionment

Sales salaries

10,000

Direct charge

Sales commission

6,000

Sales turnover

Sales office expenses

2,096

Number of orders

Advertising specific

22,000

Direct charge

Advertising general

5,000

Sales turnover

Packing

3,000

Size of product

Delivery expenses

4,000

Size of product

 

Expenses

Amount Rs.

Basis of Apportionment

Warehouse expenses

1,000

Size of product

Credit collection expenses

1,296

Number of orders

Total

54,392

 

Data relating to the three sizes:

Particulars

Total

Size A

Size B

Size C

Number of salesmen, all paid same salary

10

4

5

1

Number of orders

1,600

700

800

100

% of specific advertising

100

30

40

30

Number of units sold

8,240

3, 440

3, 200

1, 600

Sales turnover

Rs. 2,00,000

Rs. 58,000

Rs. 80,000

Rs. 62,000

Capacity in cubic m. per unit

 

5

8

17

xyz ltd maintains three salesmen x y and z in territory 1 the following information 666309

XYZ Ltd. maintains three salesmen X, Y and Z in territory 1. The following information is obtained for the month of March 2007.

Salary of salesmen Rs. 2,500

Commission Rs. 400

Traveling expenses Rs. 600

Postage and stationery Rs. 200

Telephone and telegraphs Rs. 300

Territory 1 expenses Rs. 2,000

Net sales Rs. 20,000

Cost of sales 60% of sales

From the following additional information prepare a sales performance statement.

Salesmen

Sales

Salary

Commission

Traveling Expenses

Postage and Stationery

Telephone and Telegraph

X

8,000

1,150

200

400

100

150

Y

7,000

700

100

150

50

50

Z

5,000

650

100

50

50

100

Total

20,000

2500

400

600

200

300

direct labor cost method of absorption of overheads is suitable only in those depart 666315

State whether each of the statement is True or False, give reasons in brief.

1. A term synonymous with factory overhead is ‘other expenses.’

2. Allocation and apportionment of overheads is one and the same.

3. Service departments usually do not render services to each other.

4. When actual overheads are more than the absorbed overheads, it is called as over absorption of overheads.

5. Under/over absorption of overheads takes place only when a predetermined rate of overheads is used.

6. A blanket overhead rate means a single overhead rate for the entire factory.

7. Wages of delivery van drivers is a selling overhead.

8. The use of actual overhead absorption rates results in delay in determining cost of production of products.

9. Direct labor cost method of absorption of overheads is suitable only in those departments where work is done by manual labor.

10. Machine hour rate is not suitable for absorption of overheads if the work is done mainly by  machines.

11. If the amount of under/absorption of overheads is significant, it is transferred to Costing Profit and Loss A/c.

calculate material cost price and usage and mix variances 666316

1.Calculate Material Variances from the following details. Standard quantity of materials for producing 1 unit of finished product ‘P’ is 5 kg. The standard price is Rs.6 per kg. During a particular period, 500 units of ‘P’ were produced. Actual material consumed was 2700 kg at a cost of Rs.16, 200.

2.The standard material cost to produce a ton of chemical X is given below:

300 kg of material A @ Rs.10 per kg

400 kg of material B @ Rs.5 per kg

500 kg of material C @ Rs.6 per kg

During a particular period, 100 tons of mixture X was produced from the usage of

35 tons of material A @ Rs.9, 000 per ton

42 tons of material B @ Rs.6, 000 per ton

53 tons of material C @ Rs.7, 000 per ton

Calculate material cost, price, and usage and mix variances.

he gang was engaged for 200 hours during the month which included 12 hours when no p 666320

The following was the composition of a gang of workers in a factory during a particular month, in one

of the production departments. The standard composition of workers and wage rates per hour were as follows.

Skilled: Two workers at a standard rate of Rs.20 per hour each

Semi Skilled: Four workers at a standard rate of Rs.12 per hour each

Unskilled: Four workers at a standard rate of Rs.8 per hour each.

The standard output of the gang was four units per hour of the product. During the month in question, however the actual composition of the gang and hourly rates paid were as under

Skilled: 2 workers @ Rs.20 per hour

Semi Skilled: 3 workers @ Rs.14 per hour

Un skilled: 5 workers @ Rs.10 per hour

The gang was engaged for 200 hours during the month, which included 12 hours when no production was possible due to the machine breakdown. 810 units of the product was recorded as output of the gang during the month. Calculate various labour variances.

from the following information extracted from the books of a manufacturing company c 666321

From the following information extracted from the books of a manufacturing company, calculate Fixed and Variable Overhead Variances.

Particulars

Budgeted

Actual

Production – Units

22, 000

24, 000

Fixed Overheads

Rs.44, 000

Rs.49, 000

Variable Overheads

Rs.33, 000

Rs.39, 000

Number of Days

25

26

Number of man hours

25, 000

27, 000

calculate variable overhead variances 666327

The following data are available in respect of a manufacturing company

Particulars

Budget

Actual

Production units

400

360

Man hours to produce above

8,000

7,000

Variable overheads

Rs.10, 000

Rs.9, 150

The standard time to produce one unit of the product is 20 hours.

Calculate variable overhead variances.

calculate the following overhead variances 666328

The following information is available from the records of a manufacturing company using standard costing system.

Particulars

Standard

Actual

Production

4,000 units

3,800 units

Working days

20

21

Fixed overhead

Rs.40, 000

Rs.39, 000

Variable overheads

Rs.12, 000

Rs.12, 000

Calculate the following overhead variances

I] Variable overhead variance

II] Fixed overhead cost variance

III] Fixed overhead expenditure variance

IV] Fixed overhead volume variance

V] Fixed overhead efficiency variance

VI] Fixed overhead calendar variance

calculate material cost variance price variance quantity variance mix variance and y 666330

ABC Ltd. produces an article by blending two basic raw materials. It operates a standard costing system and the following standards have been set for raw materials.

Material

Standard Mix

Standard Price

A

40%

Rs.4.00

B

60%

Rs.3.00

The standard loss in processing is 15%. During September 2007, the company produced 1, 700 kg of finished output.

The position of stock and purchases for the month of September 2007 is as under,

Material

Quantity as on

Quantity as on

Purchases

Cost

 

1/9/2007 Kg

30/9/2007 Kg

Kg

Rs.

A

35

5

800

3, 400

B

40

50

1, 200

3, 000

Calculate Material Cost Variance, Price Variance, Quantity Variance, Mix Variance and Yield Variance. Assume that First In First Out method is followed for material issues

calculate material cost price quantity mix and yield variances 666332

A company is manufacturing a chemical product making use of four different types of raw materials. The following information is available regarding the standards and actual.

Material

Share of total input

Raw material cost

Quantity

Raw material

 

Standard %

Standard

consumed

cost – Actual

 

 

Rs/kg

Actual

Rs./kg

A

40

50

42, 000

48

B

30

80

31, 000

80

C

20

90

18, 000

92

D

10

100

9, 000

110

There is an inevitable normal loss of 10% during the processing. Actual output 92, 000 kg

Calculate Material Cost, Price, Quantity, Mix and Yield variances.

calculate material cost price quantity mix and yield variances prepare reconciliatio 666333

The standard material inputs required for 1, 000 kg of a finished output are given below.

Material in kg

Quantity [Rs.]

Standard Rate Per Kg – Rs.

P

450

20

Q

400

40

R

250

60

Total Input

1100

 

Less: Standard Loss

100

 

Standard Output

1000

 

Actual production in a period was 20, 000 kg of the finished product for which the actual quantities of materials used and the prices paid thereof are as under.

Material

Quantity Used Kg

Purchased Price Per Unit

 

 

[Rs]

P

10, 000

19

Q

8, 500

42

R

4, 500

65

Calculate Material Cost, Price, Quantity, Mix and Yield Variances. Prepare reconciliation among the  variances

calculate material price and usage variances and labour rate and efficiency variance 666335

The following standards have been set to manufacture a product Direct Materials:

  • 2 units of A @ Rs.4 per unit: Rs.8
  • 3 units of B @ Rs.3 per unit: Rs.9
  • • 15 units of C @ Re.1 per unit: Rs.15
  • • Direct Labour 3 hrs @ Rs.8 per hour: Rs.24
  • • Standard Prime Cost: Rs.56

The company manufactured and sold 6000 units of the product during the year. Direct Material cost

  • was as follows:
  • • 12, 500 units of A @ Rs.4.40 per unit
  • • 18, 000 units of B @ Rs.2.80 per unit
  • • 88, 500 units of C @ Rs.1.20 per unit

The company worked for 17, 500 direct labour hours during the year. For 2500 of these hours the company paid Rs.12 per hour while for the remaining the wages were paid at the standard rate. Calculate material price and usage variances and labour rate and efficiency variances.

calculate the following fixed overhead variances 666337

The following data has been collected from the cost records of a unit for computing various fixed overhead variances for a particular period.

Particular

Details

Number of budgeted working days

25

Budgeted man hour per day

6, 000

Output [budgeted] per man hours

1 unit

Fixed overhead cost as budgeted

Rs.1, 50, 000

Actual number of working days

27

Actual man hour per day

6, 300

Actual output per man hour

0.9 units

Actual fixed overheads

Rs.1, 56, 000

Calculate the following Fixed Overhead Variances

• Expenditure variance

• Calendar variance

• Capacity variance

• Efficiency variance

• Volume Variance

• Cost Variance

you may use average cost method to analyse 666338

A company manufacturing two products uses standard costing system. The following data relating to August 2007 have been furnished to you:

Particulars

Product A

Product B

Standard Cost Per Unit –Direct Materials

Rs.2

Rs.4

Direct Wages

Rs.8

Rs.6

Fixed Overheads

Rs.16

Rs.12

Total

Rs.26

Rs.22

Units processed/in process

 

 

 

Particulars

Product A

Product B

Beginning of tand 50% comoverheads

he month, all materials applied plete in respect of labour and

4, 000

12, 000

End of the month, all materials applied and 80% complete in respect of labour and overheads

8, 000

12, 000

Units completed and transferred to warehouse during the month

16, 000

20, 000

You may use average cost method to analyse

The following were the actual costs recorded during the month.

Direct materials purchased at standard price amount to Rs.2, 00, 000 and actual cost of which is

Rs.2, 20, 000. Direct materials used for consumption at standard price amount to Rs.1, 75, 000. Direct wages for actual hours worked at standard wages rates were Rs.4, 20, 000 and at actual wages rate were Rs.4, 12, 000.

Fixed overheads budgeted were Rs.8, 25, 000 and actual fixed overheads incurred were Rs.8, 50, 000.

Required,

I. Direct material price variance at the point of consumption and at the point of purchase

II. Direct material usage variance

III. Direct wage rate and efficiency variance

IV. Fixed overheads volume and expenditure variance

V. Standard cost of WIP at the end of the month.

compute the following variances from the data given below 666339

Compute the following variances from the data given below.

I. Total sales margin variance

II. Sales margin volume variance

III. Sales margin price variance

IV. Sales margin quantity [sub volume] variance.

Product

Budgeted Quantity Units

Actual Quantity Units

Budgeted Sales Price Per Unit

Actual Sale Price Per Unit

Standard Cost Per Unit

X

240

400

Rs.50

Rs.45

Rs.30

Y

160

200

25

20

15

production with actual profits b bonus amount earned by the workers during the month 666341

A company producing a standard product is facing declining sales and dwindling profits. It has therefore decided to introduce a standard cost system to control cost. To motivate workers to improve the productivity, the management has also decided to introduce an incentive scheme under which employees are paid 20% of the standard cost of materials saved and also 40% of the labour time saved valued at standard labour rate.

The following are the details of the standard cost of the product. Standard Cost Per Unit

Particulars

Amount Rs.

Direct material: 10 kg @ Rs.12 each

120

Direct labour: 3 hours @ Rs.10 each

30

Variable overheads: 3 hours @ Rs.5 each

15

Fixed overheads [based on a budgeted

25

output of 10000 units]

 

Total standard cost per unit

190

Selling price per unit Rs.240

 

During one particular month 9600 units of the product were manufactured and sold incurring the following actual cost:

Particulars

Amount Rs.

Direct materials 90000 kg

1210000

Direct labour 25000 hours

254000

Variable overheads 25000 hours

147000

Fixed overheads

250000

Total cost

1861000

Net profit

419000

Sales

2280000

Required: A]Variances that occurred during the month, duly reconciling the standard profits of actual

production with actual profits. B] Bonus amount earned by the workers during the month under incentive scheme.

standard costs are a estimated costs b budgeted costs c expected costs d scientifica 666345

State the correct answer in each of the following.

1. Cost variance is the difference between A] Standard cost and actual cost B] Standard cost and the budgeted cost C] Standard cost and the actual cost.

2. Standard cost is used A] As a basis for price fixation and cost control through variance analysis. B] To ascertain the break even point C] To establish cost volume profit relationship

3. A standard cost system may be used in A] Either job order costing or process costing B] Job order costing but not in process costing C] Process costing but not in the job order processing D] Neither process costing nor job order costing.

4. An un favourable material price variance occurs because of A] Price increase in raw material B] Price decrease in raw material C] Less than anticipated normal wastage in the manufacturing process D] More than anticipated normal wastage in the manufacturing process.

5. Which of the following is a purpose of standard costing? A] Determine a break even production level B] Control costs C] Allocate cost with more accuracy D] Eliminate the need for subjective decisions by management.

6. Material mix variance arises due to A] Increase or decrease in the cost of material consumed B] Increase or decrease in the normal loss C] Change in the standard proportion of mix D] None of these.

7. Material usage variance is normally chargeable to A] Production department B] Purchase department C] Finished goods D] Materials stores.

8. If actual hours worked exceed the standard hours allowed, the variance, which will occur is called as A] Favourable labour efficiency variance B] Favourable labour rate variance C] Unfavourable labour efficiency variance D] Unfavourable labour rate variance.

9. Standard costs are A] Estimated costs B] Budgeted costs C] Expected costs D] Scientifically predetermined costs.

10. Standards are fixed for A] Costs only B] Costs and Profits C] Costs, profits and sales D] Only for profits.

materials labor and overhead variances bertgon manufacturing has the following stand 666346

MATERIALS, LABOR, AND OVERHEAD VARIANCES Bertgon Manufacturing has the following standard cost sheet for one of its products:

Direct materials (6 ft. @ $5)

$30

Direct labor (1.5 hrs. @ $10)

15

Fixed overhead (1.5 hrs. @ $2*)

3

Variable overhead (1.5 hrs. @ $4*)

6

Standard unit cost

$54

During the most recent year, the following actual results were recorded:

Production

12,000 units

Fixed overhead

$33,000

Variable overhead

$69,000

Direct materials (71,750 ft. purchased)

$361,620

Direct labor (17,900 hrs.)

$182,580

Required:

Compute the following variances:

1. Direct materials price and usage variances.

2. Direct labor rate and efficiency variances.

3. Variable overhead spending and efficiency variances.

4. Fixed overhead spending and volume variances.

discuss which coworkers doug probably consulted to set standards what factors should 666350

SETTING STANDARDS, ETHICAL BEHAVIOR Quincy Farms is a producer of items made from farm products that are distributed to supermarkets. For many years, Quincy’s products have had strong regional sales on the basis of brand recognition. However, other companies have been marketing similar products in the area, and price competition has become increasingly important. Doug Gilbert, the company’s controller, is planning to implement a standard costing system for Quincy and has gathered considerable information from his coworkers on production and direct materials requirements for Quincy’s products. Doug believes that the use of standard costing will allow Quincy to improve cost control and make better operating decisions.

Quincy’s most popular product is strawberry jam. The jam is produced in 10 gallon batches, and each batch requires six quarts of good strawberries. The fresh strawberries are sorted by hand before entering the production process. Because of imperfections in the strawberries and spoilage, one quart of strawberries is discarded for every four quarts of acceptable berries. Three minutes is the standard direct labor time required for sorting strawberries in order to obtain one quart of strawberries. The acceptable strawberries are then processed with the other ingredients: processing requires 12 minutes of direct labor time per batch. After processing, the jam is packaged in quart containers.

Doug has gathered the following information from Joe Adams, Quincy’s cost accountant, relative to processing the strawberry jam.

a. Quincy purchases strawberries at a cost of $0.80 per quart. All other ingredients cost a total of $0.45 per gallon.

b. Direct labor is paid at the rate of $9.00 per hour.

c. The total cost of direct material and direct labor required to package the jam is $0.38 per quart.

Joe has a friend who owns a strawberry farm that has been losing money in recent years. Because of good crops, there has been an oversupply of strawberries, and prices have dropped to $0.50 per quart. Joe has arranged for Quincy to purchase strawberries from his friend’s farm in hopes that the $0.80 per quart will put his friend’s farm in the black.

Required:

1. Discuss which coworkers Doug probably consulted to set standards. What factors should Doug consider in establishing the standards for direct materials and direct labor?

2. Develop the standard cost sheet for the prime costs of a 10 gallon batch of strawberry jam.

3. Citing the specific standards of the IMA code of ethics described in Chapter 1, explain why Joe’s behavior regarding the cost information provided to Doug is unethical.

unique boutique inc is considering two investment projects the estimated net cash fl 666152

Unique Boutique Inc. is considering two investment projects. The estimated net cash flows from each project are as follows:

 

Plant

Retail Store

Year

Expansion

Expansion

1

$ 280,000

$ 260,000

2

260,000

260,000

3

230,000

250,000

4

260,000

250,000

5

270,000

280,000

Total

$1,300,000

$1,300,000

Each project requires an investment of $770,000. A rate of 15% has been selected for the net present value analysis.

Instructions

1. Compute the following for each project:

a. Cash payback period.

b. The net present value. Use the present value of $1 table appearing in this chapter.

2. Prepare a brief report advising management on the relative merits of each project.

continental railroad company wishes to evaluate three capital investment proposals b 666153

Continental Railroad Company wishes to evaluate three capital investment proposals by using the net present value method. Relevant data related to the proposals are summarized as follows:

 

Route

Acquire

New Maintenance

 

Expansion

Railcars

Yard

Amount to be invested

$830,000

$480,000

$410,000

Annual net cash flows:

 

 

 

Year 1

450,000

245,000

215,000

Year 2

400,000

220,000

205,000

Year 3

370,000

190,000

200,000

Instructions

1. Assuming that the desired rate of return is 20%, prepare a net present value analysis for each proposal. Use the present value of $1 table appearing in this chapter.

2. Determine a present value index for each proposal. Round to two decimal places.

3. Which proposal offers the largest amount of present value per dollar of investment? Explain.

the management of genco utilities inc is considering two capital investment projects 666154

The management of Genco Utilities Inc. is considering two capital investment projects. The estimated net cash flows from each project are as follows:

 

 

Distribution

Year

Generating Unit

Network Expansion

1

$650,000

$180,000

2

650,000

180,000

3

650,000

180,000

4

650,000

180,000

The generating unit requires an investment of $2,060,500, while the distribution network expansion requires an investment of $546,660. No residual value is expected from either project.

Instructions

1. Compute the following for each project:

a. The net present value. Use a rate of 6% and the present value of an annuity of $1 table appearing in this chapter.

b. A present value index. Round to two decimal places.

2. Determine the internal rate of return for each project by (a) computing a present value factor for an annuity of $1 and (b) using the present value of an annuity of $1 table appearing in this chapter.

3. What advantage does the internal rate of return method have over the net present value method in comparing projects?

the investment committee of safe hands insurance co is evaluating two projects the p 666155

The investment committee of Safe Hands Insurance Co. is evaluating two projects. The projects have different useful lives, but each requires an investment of $225,000. The estimated net cash flows from each project are as follows:

 

Net Cash Flows

Year

Project I

Project II

1

$70,000

$98,000

2

70,000

98,000

3

70,000

98,000

4

70,000

98,000

5

70,000

 

6

70,000

 

The committee has selected a rate of 12% for purposes of net present value analysis. It also estimates that the residual value at the end of each project’s useful life is $0, but at the end of the fourth year, Project I’s residual value would be $150,000.

Instructions

1. For each project, compute the net present value. Use the present value of an annuity of $1 table appearing in this chapter. (Ignore the unequal lives of the projects.)

2. For each project, compute the net present value, assuming that Project I is adjusted to a four year life for purposes of analysis. Use the present value of $1 table appearing in this chapter.

3. Prepare a report to the investment committee, providing your advice on the relative merits of the two projects.

madison capital group is considering allocating a limited amount of capital investme 666156

Madison Capital Group is considering allocating a limited amount of capital investment funds among four proposals. The amount of proposed investment, estimated income from operations, and net cash flow for each proposal are as follows:

 

 

 

Income from

Net Cash

 

Investment

Year

Operations

Flow

Proposal A:

$540,000

1

$42,000

$150,000

 

 

2

42,000

150,000

 

 

3

42,000

150,000

 

 

4

(18,000)

90,000

 

 

5

(18,000)

90,000

 

 

 

$90,000

$630,000

 

 

 

 

 

Proposal B:

$250,000

1

$ 50,000

$100,000

 

 

2

40,000

90,000

 

 

3

30,000

80,000

 

 

4

15,000

65,000

 

 

5

15,000

65,000

 

 

 

$150,000

$400,000

 

 

 

 

 

Proposal C:

$640,000

1

$ 92,000

$220,000

 

 

2

82,000

210,000

 

 

3

82,000

210,000

 

 

4

62,000

190,000

 

 

5

32,000

160,000

 

 

 

$350,000

$990,000

 

 

 

 

 

Proposal D:

$310,000

1

$ 68,000

$130,000

 

 

2

38,000

100,000

 

 

3

(2,000)

60,000

 

 

4

(2,000)

60,000

 

 

5

(2,000)

60,000

 

 

 

$100,000

$440,000

The company’s capital rationing policy requires a maximum cash payback period of three years. In addition, a minimum average rate of return of 12% is required on all projects. If the preceding standards are met, the net present value method and present value indexes are used to rank the remaining proposals.

Instructions

1. Compute the cash payback period for each of the four proposals.

2. Giving effect to straight-line depreciation on the investments and assuming no estimated residual value, compute the average rate of return for each of the four proposals. Round to one decimal place.

3. Using the following format, summarize the results of your computations in parts (1) and (2). By placing a check mark in the appropriate column at the right, indicate which proposals should be accepted for further analysis and which should be rejected.

 

Cash Payback

Average Rate

Accept for

 

Proposal

Period

of Return

Further Analysis

Reject

A

 

 

 

 

B

 

 

 

 

C

 

 

 

 

D

 

 

 

 

4. For the proposals accepted for further analysis in part (3), compute the net present value. Use a rate of 12% and the present value of $1 table appearing in this chapter. Round to the nearest dollar.

5. Compute the present value index for each of the proposals in part (4). Round to two decimal places.

6. Rank the proposals from most attractive to least attractive, based on the present values of net cash flows computed in part (4).

7. Rank the proposals from most attractive to least attractive, based on the present value indexes computed in part (5).

8. Based upon the analyses, comment on the relative attractiveness of the proposals ranked in parts (6) and (7).

family life publications inc is considering two new magazine products the estimated 666158

Family Life Publications Inc. is considering two new magazine products. The estimated net cash flows from each product are as follows:

Year

Home & Garden

Today’s Teen

1

$230,000

$160,000

2

210,000

280,000

3

190,000

200,000

4

50,000

40,000

5

40,000

40,000

Total

$720,000

$720,000

Each product requires an investment of $440,000. A rate of 15% has been selected for the net present value analysis.

Instructions

1. Compute the following for each project:

a. Cash payback period.

b. The net present value. Use the present value of $1 table appearing in this chapter.

2. Prepare a brief report advising management on the relative merits of each of the two products.

the management of horizon media inc is considering two capital investment projects t 666160

The management of Horizon Media Inc. is considering two capital investment projects. The estimated net cash flows from each project are as follows:

Year

Radio Station

TV Station

1

$160,000

$450,000

2

160,000

450,000

3

160,000

450,000

4

160,000

450,000

The radio station requires an investment of $456,800, while the TV station requires an investment of $1,366,650. No residual value is expected from either project.

Instructions

1. Compute the following for each project:

a. The net present value. Use a rate of 10% and the present value of an annuity of $1 table appearing in this chapter.

b. A present value index. Round to two decimal places.

2. Determine the internal rate of return for each project by (a) computing a present value factor for an annuity of $1 and (b) using the present value of an annuity of $1 table appearing in this chapter.

3. What advantage does the internal rate of return method have over the net present value method in comparing projects?

the investment committee of mr bob restaurants inc is evaluating two restaurant site 666161

The investment committee of Mr. Bob Restaurants Inc. is evaluating two restaurant sites. The sites have different useful lives, but each requires an investment of $445,000. The estimated net cash flows from each site are as follows:

 

Net Cash Flows

Year

Site A

Site B

1

$170,000

$230,000

2

170,000

230,000

3

170,000

230,000

4

170,000

230,000

5

170,000

 

6

170,000

 

The committee has selected a rate of 20% for purposes of net present value analysis. It also estimates that the residual value at the end of each restaurant’s useful life is $0, but at the end of the fourth year, Site A’s residual value would be $340,000.

Instructions

1. For each site, compute the net present value. Use the present value of an annuity of $1 table appearing in this chapter. (Ignore the unequal lives of the projects.)

2. For each site, compute the net present value, assuming that Site A is adjusted to a four year life for purposes of analysis. Use the present value of $1 table appearing in this chapter.

3. Prepare a report to the investment committee, providing your advice on the relative merits of the two sites.

horizon communications inc is considering allocating a limited amount of capital inv 666162

Horizon Communications Inc. is considering allocating a limited amount of capital investment funds among four proposals. The amount of proposed investment, estimated income from operations, and net cash flow for each proposal are as follows:

 

 

 

Income from

Net Cash

 

Investment

Year

Operations

Flow

Proposal A:

$680,000

1

$ 74,000

$210,000

 

 

2

74,000

210,000

 

 

3

74,000

210,000

 

 

4

14,000

150,000

 

 

5

14,000

150,000

 

 

 

$250,000

$930,000

Proposal B:

$155,000

1

$29,000

$ 60,000

 

 

2

64,000

95,000

 

 

3

9,000

40,000

 

 

4

(1,000)

30,000

 

 

5

(11,000)

20,000

 

 

 

$90,000

$245,000

Proposal C:

$281,250

1

$18,750

$ 75,000

 

 

2

18,750

75,000

 

 

3

18,750

75,000

 

 

4

18,750

75,000

 

 

5

(6,250)

50,000

 

 

 

$68,750

$350,000

Proposal D:

$260,000

1

$ 38,000

$ 90,000

 

 

2

38,000

90,000

 

 

3

28,000

80,000

 

 

4

28,000

80,000

 

 

5

23,000

75,000

 

 

 

$155,000

$415,000

The company’s capital rationing policy requires a maximum cash payback period of three years. In addition, a minimum average rate of return of 12% is required on all projects. If the preceding standards are met, the net present value method and present value indexes are used to rank the remaining proposals.

Instructions

1. Compute the cash payback period for each of the four proposals.

2. Giving effect to straight-line depreciation on the investments and assuming no estimated residual value, compute the average rate of return for each of the four proposals. Round to one decimal place.

3. Using the following format, summarize the results of your computations in parts (1) and (2). By placing a check mark in the appropriate column at the right, indicate which proposals should be accepted for further analysis and which should be rejected.

 

Cash Payback

Average Rate

Accept for

 

Proposal

Period

of Return

Further Analysis

Reject

A

 

 

 

 

B

 

 

 

 

C

 

 

 

 

D

 

 

 

 

4. For the proposals accepted for further analysis in part (3), compute the net present value. Use a rate of 10% and the present value of $1 table appearing in this chapter. Round to the nearest dollar.

5. Compute the present value index for each of the proposals in part (4). Round to two decimal places.

6. Rank the proposals from most attractive to least attractive, based on the present values of net cash flows computed in part (4).

7. Rank the proposals from most attractive to least attractive, based on the present value indexes computed in part (5). Round to two decimal places.

8. Based upon the analyses, comment on the relative attractiveness of the proposals ranked in parts (6) and (7).

elisa mcrae was recently hired as a cost analyst by medlab medical supplies inc one 666163

Elisa McRae was recently hired as a cost analyst by Medlab Medical Supplies Inc. One of Elisa’s first assignments was to perform a net present value analysis for a new warehouse. Elisa performed the analysis and calculated a present value index of 0.75. The plant manager, I. M. Madd, is very intent on purchasing the warehouse because he believes that more storage space is needed. I. M. Madd asks Elisa into his office and the following conversation takes place.

I. M.: Elisa, you’re new here, aren’t you?

Elisa: Yes, sir.

I. M.: Well, Elisa, let me tell you something. I’m not at all pleased with the capital investment analysis that you performed on this new warehouse. I need that warehouse for my production. If I don’t get it, where am I going to place our output?

Elisa: Hopefully with the customer, sir.

I. M.: Now don’t get smart with me.

Elisa: No, really, I was being serious. My analysis does not support constructing a new warehouse. The numbers don’t lie, the warehouse does not meet our investment return targets. In fact, it seems to me that purchasing a warehouse does not add much value to the business. We need to be producing product to satisfy customer orders, not to fill a warehouse.

I. M.: Listen, you need to understand something. The headquarters people will not allow me to build the warehouse if the numbers don’t add up. You know as well as I that many assumptions go into your net present value analysis. Why don’t you relax some of your assumptions so that the financial savings will offset the cost?

Elisa: I’m willing to discuss my assumptions with you. Maybe I overlooked something.

I. M.: Good. Here’s what I want you to do. I see in your analysis that you don’t project greater sales as a result of the warehouse. It seems to me, if we can store more goods, then we will have more to sell. Thus, logically, a larger warehouse translates into more sales. If you incorporate this into your analysis, I think you’ll see that the numbers will work out. Why don’t you work it through and come back with a new analysis. I’m really counting on you on this one. Let’s get off to a good start together and see if we can get this project accepted.

What is your advice to Elisa?

global products inc invested 1 000 000 to build a plant in a foreign country the lab 666165

Global Products Inc. invested $1,000,000 to build a plant in a foreign country. The labor and materials used in production are purchased locally. The plant expansion was estimated to produce an internal rate of return of 20% in U.S. dollar terms. Due to a currency crisis, the currency exchange rate between the local currency and the U.S. dollar doubled from two local units per U.S. dollar to four local units per U.S. dollar.

a. Assume that the plant produced and sold product in the local economy. Explain what impact this change in the currency exchange rate would have on the project’s internal rate of return.

b. Assume that the plant produced product in the local economy but exported the product back to the United States for sale. Explain what impact the change in the currency exchange rate would have on the project’s internal rate of return under this assumption.

explain the role of capital investment analysis for these companies 666166

The following are some selected quotes from senior executives:

CEO, Worthington Industries (a high technology steel company): “We try to find the best technology, stay ahead of the competition, and serve the customer. . . . We’ll make any investment that will pay back quickly . . . but if it is something that we really see as a must down the road, payback is not going to be that important.”

Chairman of Amgen Inc. (a biotech company): “You cannot really run the numbers, do net present value calculations, because the uncertainties are really gigantic . . . You decide on a project you want to run, and then you run the numbers [as a reality check on your assumptions]. Success in a business like this is much more dependent on tracking rather than on predicting, much more dependent on seeing results over time, tracking and adjusting and readjusting, much more dynamic, much more flexible.”

Chief Financial Officer of Merck & Co., Inc. (a pharmaceutical company): “. . . at the individual product level—the development of a successful new product requires on the order of $230 million in R&D, spread over more than a decade—discounted cash flow style analysis does not become a factor until development is near the point of manufacturing scale up effort. Prior to that point, given the uncertainties associated with new product development, it would be lunacy in our business to decide that we know exactly what’s going to happen to a product once it gets out.”

Explain the role of capital investment analysis for these companies.

you are considering an investment of 300 000 in either project a or project b for we 666167

You are considering an investment of $300,000 in either Project A or Project B for West Coast Studios Inc. In discussing the two projects with an advisor, you decided that, for the risk involved, an average rate of return of 12% on the cash investment would be required. For this purpose, you estimated the following economic factors for the projects:

 

Project A

Project B

Useful life

4 years

4 years

Residual value

0

0

Net income:

 

 

Year 1

$ 80,000

$ 40,000

2

65,000

55,000

3

55,000

73,000

4

40,000

79,200

 

$240,000

$247,200

 

Project A

Project B

Net cash flows:

 

 

Year 1

$155,000

$115,000

2

140,000

130,000

3

130,000

148,000

4

115,000

154,200

 

$540,000

$547,200

       

Although the average rate of return exceeded 12% on both projects, you have tentatively decided to invest in Project B because the rate was higher for Project B. You noted that the total cash flow from Project B is $547,200, which exceeds that of Project A by $7,200.

1. Determine the average rate of return for both projects.

2. Why is the timing of cash flows important in evaluating capital investments? Calculate the net present value of the two projects at a minimum rate of return of 12% to demonstrate the importance of net cash flows and their timing to these two projects. Round to the nearest dollar.

a company has three production departments a b and c and two service departments x a 666298

A company has three production departments A, B and C and two service departments, X and Y. The following data are extracted from the records of the company for a particular period.

Sr. No.

Particulars

Amount ( Rs.)

01

Rent and Taxes

25,000

02

General lighting

3,000

03

Indirect Wages

7,500

04

Power

7,500

05

Depreciation of Machinery

50,000

06

Sundries

50,000

Additional Data

Particulars

Total

Dept. A

Dept. B

Dept. C

Dept. X

Dept. Y

Direct Wages (Rs.)

50,000

15,000

10,000

15,000

7,500

2,500

Horsepower of Machines

150

60

30

50

10

Cost of Machinery (Rs.)

12,50,000

3,00,000

4,00,000

5,00,000

25,000

25,000

Production hrs worked

6226

4028

4066

Floor space (sq.mtrs)

10,000

2,000

2,500

3,000

2,000

500

Lighting points (Nos.)

60

10

15

20

10

05

Service Departments’ Expenses Allocation :

Department

A

B

C

X

Y

X (%)

20

30

40

10

Y (%)

40

30

20

10

You are required to,

A. Prepare primary and secondary distribution summary according to repeated distribution System.

llocate the cost of service departments to the production departments 666299

X, Y and Z has two production departments and three service departments. Expenses incurred for these departments and other available information is given below.

Particulars

Prod. Dept. A

Prod. Dept. B

Service Dept. Maintenance

Service Dept. Power

Service Dept. Personnel

As per Primary Distribution

1,20,000

1,50,000

20,000

48,000

40,000

Allocation Basis

 

 

 

 

 

Maintenance Hours

80

20

40

20

KWH Consumed

4

16

2

2

Number of employees

60

30

30

18

 

llocate the cost of service departments to the production departments.

calculate the overhead rate per direct labor hour of production departments given th 666300

A company has two production departments and two service departments. The data relating to a period are as follows.

Particulars

Prod. Dept I

Prod. Dept II

Service Dept I

Service Dept II

Direct Materials

80,000

40,000

10,000

20,000

Direct Wages

95,000

50,000

20,000

10,000

Overheads

80,000

50,000

30,000

20,000

Power requirement at normal

20,000

35,000

12,500

17,500

capacity operation (Kwh)

 

 

 

 

Actual power consumption (Kwh)

13,000

23,000

10,250

10,000

The power requirement of these departments are met by a power generation plant. The said plant incurred an expenditure which is not included above, of Rs. 1,21,875 out of which a sum of Rs. 84,375 was variable and the rest fixed

After apportionment of power generation plant costs to the four departments, the service department overheads are to be redistributed on the following basis.

Departments

Prod. Dept I

Prod. Dept II

Service Dept I

Service Dept II

Service Dept I (%)

50

40

10

Service Dept II

60

20

20

You are required to,

i. Apportion the power generation plant costs to the four departments

ii. Reapportion service department cost to production departments

iii. Calculate the overhead rate per direct labor hour of production departments, given that the direct wages rates of Production Dept I, II are Rs. 5 and Rs. 4 per hour respectively.

compute the machine hour rate for the machine 666302

A machine was purchased on 1st January, 2007 for Rs. 5 lakhs. The total cost of all machinery inclusive of the new machine was Rs. 75 lakhs. The following further particulars were available.

Expected life of the machine – 10 years

Scrap value at the end of the life – Rs. 5,000

Repairs and maintenance for the machine during the year Rs. 2,000

Expected number of working hours of the machine per year 4,000

Insurance premium annually for all machines Rs. 4,00

Power consumption for the machine per hour @ Rs. 5 @ per unit = 25 units

Area occupied by the machine – 100 sq feet

Area occupied by other machines – 1,500 sq. feet

Rent per month of the department Rs. 800

Lighting charges for 20 points for the whole department out of which three points are for the new machine – Rs.120 per month

Compute the machine hour rate for the machine.

using two methods of disposal of under absorbed overheads show the implication on th 666304

XYZ Ltd., uses a historical cost accounting system and absorbs overheads on the basis of predetermined rates. The following data are available for the year ended 31st March, 2007.

Particulars

Amount in Rupees

Manufacturing overheads

 

Amount actually spent

1,70,000

Amount absorbed

1,50,000

Cost of goods sold

3,36,000

Stock of finished goods

96,000

Work in progress

48,000

Using two methods of disposal of under/absorbed overheads show the implication on the profits of the company under each method.

overheads are charged to the cost of each product 25 on prime cost do you see anythi 666305

In a certain factory, three products are made from different materials by similar processes. For a typical period, production costs are as under.

 

In Rupees

Particulars

Product A

Product B

Product C

Materials used

1,600

2,000

800

Direct labor cost

1,200

1,000

400

Overheads (Actual)

800

650

350

Overheads are charged to the cost of each product @ 25% on Prime Cost. Do you see anything wrong in principle in this method of charging overheads? If so, suggest a preferable method.

how would unabsorbed overheads be treated in cost accounts 666306

In a manufacturing unit, overhead was recovered at a predetermined rate of Rs. 20 per labor hour. The total factory overhead incurred and the labor hours actually worked were Rs. 45,00,000 and 2,00,000 respectively. During this period, 30,000 units were sold. At the end of the period 5,000 units were held in stock while there was no opening stock of finished goods. Similarly though there was no stock of uncompleted units at the beginning of the period, at the end of the period there were 10,000 incomplete units which may be reckoned as 50% complete.

On analyzing the reasons, it was found that 60% of the unabsorbed overheads were due to defective planning and rest were attributed to increase in overhead costs.

How would unabsorbed overheads be treated in cost accounts?

the three divisions of monster foods are snack goods cereal and frozen foods the div 666111

The three divisions of Monster Foods are Snack Goods, Cereal, and Frozen Foods. The divisions are structured as investment centers. The following responsibility reports were prepared for the three divisions for the prior year:

 

Snack Goods

Cereal

Frozen Foods

Revenues

$1,050,000

$2,450,000

$ 875,000

Operating expenses

420,000

1,400,000

175,000

Income from operations before

 

 

 

service department charges

$ 630,000

$1,050,000

$ 700,000

Service department charges:

 

 

 

Promotion

$ 175,000

$ 350,000

$ 308,000

Legal

87,500

70,000

140,000

 

$262,500

$ 420,000

$ 448,000

Income from operations

$ 367,500

$ 630,000

$ 252,000

Invested assets

$2,100,000

$4,200,000

$1,260,000

1. Which division is making the best use of invested assets and thus should be given priority for future capital investments?

2. Assuming that the minimum acceptable rate of return on new projects is 12%, would all investments that produce a return in excess of 12% be accepted by the divisions?

3. Can you identify opportunities for improving the company’s financial performance?

the truck division of yang motors inc has been experiencing revenue and profit growt 666112

The Truck Division of Yang Motors Inc. has been experiencing revenue and profit growth during the years 2006–2008. The divisional income statements are provided below.

Yang Motors Inc.

Divisional Income Statements, Truck Division

For the Years Ended December 31, 2006–2008

 

2006

2007

2008

Sales

$756,000

$972,000

$1,170,000

Cost of goods sold

475,200

558,000

616,500

Gross profit

$280,800

$414,000

$ 553,500

Operating expenses

167,400

209,880

261,000

Income from operations

$113,400

$204,120

$ 292,500

Assume that there are no charges from service departments. The vice president of the division, Terry Clark, is proud of his division’s performance over the last three years. The president of Yang Motors Inc., Billy Clark, is discussing the division’s performance with Terry, as follows:

Terry: As you can see, we’ve had a successful three years in the Truck Division.

Billy: I’m not too sure.

Terry: What do you mean? Look at our results. Our income from operations has nearly tripled, while our profit margins are improving.

Billy: I am looking at your results. However, your income statements fail to include one very important piece of information; namely, the invested assets. You have been investing a great deal of assets into the division. You had $315,000 in invested assets in 2006, $810,000 in 2007, and $1,950,000 in 2008.

Terry: You are right. I’ve needed the assets in order to upgrade our technologies and expand our operations. The additional assets are one reason we have been able to grow and improve our profit margins. I don’t see that this is a problem.

Billy: The problem is that we must maintain a 20% rate of return on invested assets.

1. Determine the profit margins for the Truck Division for 2006–2008.

2. Compute the investment turnover for the Truck Division for 2006–2008.

3. Compute the rate of return on investment for the Truck Division for 2006–2008.

4. Evaluate the division’s performance over the 2006–2008 time period. Why was Billy concerned about the performance?

casual living furniture inc is a privately held diversified company with five separa 666113

Casual Living Furniture Inc. is a privately held diversified company with five separate divisions organized as investment centers. A condensed income statement for the Outdoor Division for the past year, assuming no service department charges, is as follows:

Casual Living Furniture Inc.—Outdoor Division

Income Statement

For the Year Ended December 31, 2007

Sales

$12,800,000

Cost of goods sold

8,080,000

Gross profit

$ 4,720,000

Operating expenses

1,520,000

Income from operations

$ 3,200,000

The manager of the Outdoor Division was recently presented with the opportunity to add an additional product line, which would require invested assets of $11,000,000. A projected income statement for the new product line is as follows:

New Product Line

Projected Income Statement

For the Year Ended December 31, 2008

Cost of goods sold

$6,000,000

Gross profit

3,360,000

Operating expenses

$2,640,000

Income from operations

1,680,000

Cost of goods sold

$ 960,000

The Outdoor Division currently has $20,000,000 in invested assets, and Casual Living Furniture Inc.’s overall rate of return on investment, including all divisions, is 8%. Each division manager is evaluated on the basis of divisional rate of return on investment, and a bonus equal to $12,000 for each percentage point by which the division’s rate of return on investment exceeds the company average is awarded each year.

The president is concerned that the manager of the Outdoor Division rejected the addition of the new product line, when all estimates indicated that the product line would be profitable and would increase overall company income. You have been asked to analyze the possible reasons why the Outdoor Division manager rejected the new product line.

1. Determine the rate of return on investment for the Outdoor Division for the past year.

2. Determine the Outdoor Division manager’s bonus for the past year.

3. Determine the estimated rate of return on investment for the new product line. Round whole percents to one decimal place.

4. Why might the manager of the Outdoor Division decide to reject the new product line? Support your answer by determining the projected rate of return on investment for 2008, assuming that the new product line was launched in the Outdoor Division, and 2008 actual operating results were similar to those of 2007.

5. Can you suggest an alternative performance measure for motivating division managers to accept new investment opportunities that would increase the overall company income and rate of return on investment?

divide responsibilities between two groups with one group going to the home page of 666114

Divide responsibilities between two groups, with one group going to the home page of Balanced Scorecard Collaborative and the second group going to the home page of Stern Stewart & Co. Balanced Scorecard Collaborative is a consulting firm that helped develop the balanced scorecard concept. Stern Stewart & Co. is a consulting firm that developed the concept of economic value added (EVA), another method of measuring corporate and divisional performance, similar to residual income.

After reading about the balanced scorecard at the bscol.com site, prepare a brief report describing the balanced scorecard and its claimed advantages. In the Stern group, use links in the home page of Stern Stewart & Co. to learn about EVA. After reading about EVA, prepare a brief report describing EVA and its claimed advantages. After preparing these reports, both groups should discuss their research and prepare a brief analysis comparing and contrasting these two approaches to corporate and divisional performance measurement.

a project has estimated annual net cash flows of 50 000 for seven years and is estim 666117

A project has estimated annual net cash flows of $50,000 for seven years and is estimated to cost $240,000. Assume a minimum acceptable rate of return of 12%.

Partial Present Value of an Annuity Table

Present Value of an Annuity of $1 at Compound Interest

Year

6%

10%

12%

15%

20%

1

0.943

0.909

0.893

0.870

0.833

2

1.833

1.736

1.690

1.626

1.528

3

2.673

2.487

2.402

2.283

2.106

4

3.465

3.170

3.037

2.855

2.589

5

4.212

3.791

3.605

3.353

2.991

6

4.917

4.355

4.111

3.785

3.326

7

5.582

4.868

4.564

4.160

3.605

8

6.210

5.335

4.968

4.487

3.837

9

6.802

5.759

5.328

4.772

4.031

10

7.360

6.145

5.650

5.019

4.192

Determine (a) the net present value of the project and (b) the present value index, rounded to two decimal places.

a project has estimated annual net cash flows of 65 000 for six years and is estimat 666125

A project has estimated annual net cash flows of $65,000 for six years and is estimated to cost $265,000. Assume a minimum acceptable rate of return of 10%.

Partial Present Value of an Annuity Table

Present Value of an Annuity of $1 at Compound Interest

Year

6%

10%

12%

15%

20%

1

0.943

0.909

0.893

0.870

0.833

2

1.833

1.736

1.690

1.626

1.528

3

2.673

2.487

2.402

2.283

2.106

4

3.465

3.170

3.037

2.855

2.589

5

4.212

3.791

3.605

3.353

2.991

6

4.917

4.355

4.111

3.785

3.326

7

5.582

4.868

4.564

4.160

3.605

8

6.210

5.335

4.968

4.487

3.837

9

6.802

5.759

5.328

4.772

4.031

10

7.360

6.145

5.650

5.019

4.192

Determine (1) the net present value of the project and (2) the present value index, rounded to two decimal places.

a project has estimated annual net cash flows of 22 000 for four years and is estima 666126

A project has estimated annual net cash flows of $22,000 for four years and is estimated to cost $70,000. Assume a minimum acceptable rate of return of 12%.

Partial Present Value of an Annuity Table

Present Value of an Annuity of $1 at Compound Interest

Year

6%

10%

12%

15%

20%

1

0.943

0.909

0.893

0.870

0.833

2

1.833

1.736

1.690

1.626

1.528

3

2.673

2.487

2.402

2.283

2.106

4

3.465

3.170

3.037

2.855

2.589

5

4.212

3.791

3.605

3.353

2.991

6

4.917

4.355

4.111

3.785

3.326

7

5.582

4.868

4.564

4.160

3.605

8

6.210

5.335

4.968

4.487

3.837

9

6.802

5.759

5.328

4.772

4.031

10

7.360

6.145

5.650

5.019

4.192

Determine (1) the net present value of the project and (2) the present value index, rounded to two decimal places.

a project is estimated to cost 175 665 and provide annual net cash flows of 35 000 f 666127

A project is estimated to cost $175,665 and provide annual net cash flows of $35,000 for 10 years.

Partial Present Value of an Annuity Table

Present Value of an Annuity of $1 at Compound Interest

Year

6%

10%

12%

15%

20%

1

0.943

0.909

0.893

0.870

0.833

2

1.833

1.736

1.690

1.626

1.528

3

2.673

2.487

2.402

2.283

2.106

4

3.465

3.170

3.037

2.855

2.589

5

4.212

3.791

3.605

3.353

2.991

6

4.917

4.355

4.111

3.785

3.326

7

5.582

4.868

4.564

4.160

3.605

8

6.210

5.335

4.968

4.487

3.837

9

6.802

5.759

5.328

4.772

4.031

10

7.360

6.145

5.650

5.019

4.192

Determine the internal rate of return for this project.

the following data are accumulated by green mountain testing services inc in evaluat 666131

The following data are accumulated by Green Mountain Testing Services Inc. in evaluating two competing capital investment proposals:

 

Testing Equipment

Centrifuge

Amount of investment

$34,000

$40,000

Useful life

6 years

8 years

Estimated residual value

0

0

Estimated total income over the useful life

$10,200

$14,000

Determine the expected average rate of return for each proposal.

airwave communications inc is considering an investment in new equipment that will b 666133

Airwave Communications Inc. is considering an investment in new equipment that will be used to manufacture a PDA (personal data assistant). The PDA is expected to generate additional annual sales of 4,800 units at $350 per unit. The equipment has a cost of $910,000, residual value of $50,000, and a 10 year life. The equipment can only be used to manufacture the PDA. The cost to manufacture the PDA is shown below.

Cost per unit: Direct labor

$ 52.00

Direct materials

195.00

Factory overhead (including depreciation)

58.00

Total cost per unit

$305.00

Determine the average rate of return on the equipment.

gardeneer inc is planning to invest 184 000 in a new garden tool that is expected to 666134

Gardeneer Inc. is planning to invest $184,000 in a new garden tool that is expected to generate additional sales of 7,500 units at $38 each. The $184,000 investment includes $54,000 for initial launch related expenses and $130,000 for equipment that has a 10 year life and a $17,500 residual value. Selling expenses related to the new product are expected to be 6% of sales revenue. The cost to manufacture the product includes the following per unit costs:

Direct labor

$ 6.00

Direct materials

11.75

Fixed factory overhead—depreciation

1.50

Variable factory overhead

1.80

Total

$21.05

Determine the net cash flows for the first year of the project, years 2–9, and for the last year of the project.

first union bank corporation is evaluating two capital investment proposals for a dr 666135

First Union Bank Corporation is evaluating two capital investment proposals for a drive up ATM kiosk, each requiring an investment of $300,000 and each with an eight year life and expected total net cash flows of $480,000. Location 1 is expected to provide equal annual net cash flows of $60,000, and Location 2 is expected to have the following unequal annual net cash flows:

Year 1

$90,000

Year 5

$45,000

Year 2

80,000

Year 6

45,000

Year 3

65,000

Year 7

45,000

Year 4

65,000

Year 8

45,000

Determine the cash payback period for both proposals.

family care products company is considering an investment in one of two new product 666136

Family Care Products Company is considering an investment in one of two new product lines. The investment required for either product line is $600,000. The net cash flows associated with each product are as follows:

Year

Liquid Soap

Cosmetics

1

$120,000

$165,000

2

120,000

155,000

3

120,000

140,000

4

120,000

140,000

5

120,000

110,000

6

120,000

90,000

7

120,000

80,000

8

120,000

80,000

Total

$960,000

$960,000

a. Recommend a product offering to Family Care Products Company, based on the cash payback period for each product line.

b. Why is one product line preferred over the other, even though they both have the same total net cash flows through eight periods?

the following data are accumulated by zadok company in evaluating the purchase of 37 666137

The following data are accumulated by Zadok Company in evaluating the purchase of $370,000 of equipment, having a four year useful life:

 

Net Income

Net Cash Flow

Year 1

$67,500

$160,000

Year 2

47,500

140,000

Year 3

(12,500)

80,000

Year 4

(12,500)

80,000

a. Assuming that the desired rate of return is 12%, determine the net present value for the proposal.

b. Would management be likely to look with favor on the proposal? Explain.

maddox excavation company is planning an investment of 205 000 for a bulldozer the b 666139

Maddox Excavation Company is planning an investment of $205,000 for a bulldozer. The bulldozer is expected to operate for 1,600 hours per year for five years. Customers will be charged $95 per hour for bulldozer work. The bulldozer operator is paid an hourly wage of $25 per hour. The bulldozer is expected to require annual maintenance costing $14,000. The bulldozer uses fuel that is expected to cost $30 per hour of bulldozer operation.

Partial Present Value of $1 Table

Present Value of $1 at Compound Interest

Year

6%

10%

12%

15%

20%

1

0.943

0.909

0.893

0.870

0.833

2

0.890

0.826

0.797

0.756

0.694

3

0.840

0.751

0.712

0.658

0.579

4

0.792

0.683

0.636

0.572

0.482

5

0.747

0.621

0.567

0.497

0.402

6

0.705

0.564

0.507

0.432

0.335

7

0.665

0.513

0.452

0.376

0.279

8

0.627

0.467

0.404

0.327

0.233

9

0.592

0.424

0.361

0.284

0.194

10

0.558

0.386

0.322

0.247

0.162

a. Determine the equal annual net cash flows from operating the bulldozer.

b. Determine the net present value of the investment, assuming that the desired rate of return is 10%. Round to the nearest dollar.

c. Should Maddox invest in the bulldozer, based on this analysis?

carnival corporation has recently placed into service some of the largest cruise shi 666142

Carnival Corporation has recently placed into service some of the largest cruise ships in the world. One of these ships, the Carnival Glory, can hold up to 3,000 passengers and cost $530 million to build. Assume the following additional information:

  • The average occupancy rate for the new ship is estimated to be 85% of capacity.
  • There will be 300 cruise days per year.
  • The variable expenses per passenger are estimated to be $80 per cruise day.
  • The revenue per passenger is expected to be $310 per cruise day.
  • The fixed expenses for running the ship, other than depreciation, are estimated to be $80,000,000 per year.
  • The ship has a service life of 10 years, with a salvage value of $90,000,000 at the end of 10 years.

a. Determine the annual net cash flow from operating the cruise ship.

b. Determine the net present value of this investment, assuming a 12% minimum rate of return. Use the present value tables provided in the chapter in determining your answer.

c. Assume that Carnival Corp. decided to increase its price so that the revenue increased to $320 per passenger per cruise day. Would this allow Carnival Corp. to earn a 15% rate of return on the cruise ship investment, assuming no change in any of the other assumptions?

drive by doughnuts has computed the net present value for capital expenditure locati 666143

Drive By Doughnuts has computed the net present value for capital expenditure locations A and B, using the net present value method. Relevant data related to the computation are as follows:

 

Location A

Location B

Total present value of net cash flow

$306,280

$177,660

Amount to be invested

322,400

164,500

Net present value

$ (16,120)

$ 13,160

Determine the present value index for each proposal.

mvp sporting goods company is considering an investment in one of two machines the s 666144

MVP Sporting Goods Company is considering an investment in one of two machines. The sewing machine will increase productivity from sewing 120 baseballs per hour to sewing 180 per hour. The contribution margin is $0.80 per baseball. Assume that any increased production of baseballs can be sold. The second machine is an automatic packing machine for the golf ball line. The packing machine will reduce packing labor cost. The labor cost saved is equivalent to $24 per hour. The sewing machine will cost $354,300, have an eight year life, and will operate for 1,750 hours per year. The packing machine will cost $148,300, have an eight year life, and will operate for 1,500 hours per year. MVP seeks a minimum rate of return of 15% on its investments.

a. Determine the net present value for the two machines. Round to the nearest dollar.

b. Determine the present value index for the two machines. Round to two decimal places.

c. If MVP has sufficient funds for only one of the machines and qualitative factors are equal between the two machines, in which machine should it invest?

southwest chip company is considering two possible investments a delivery truck or a 666148

Southwest Chip Company is considering two possible investments: a delivery truck or a bagging machine. The delivery truck would cost $39,918 and could be used to deliver an additional 36,250 bags of taquitos chips per year. Each bag of chips can be sold for a contribution margin of $0.40. The delivery truck operating expenses, excluding depreciation, are $0.35 per mile for 18,000 miles per year. The bagging machine would replace an old bagging machine, and its net investment cost would be $49,920. The new machine would require three fewer hours of direct labor per day. Direct labor is $16 per hour. There are 250 operating days in the year. Both the truck and the bagging machine are estimated to have seven year lives. The minimum rate of return is 11%. However, Southwest has funds to invest in only one of the projects.

a. Compute the internal rate of return for each investment. Use the table of present values of an annuity of $1 in the chapter.

b. Provide a memo to management with a recommendation.

the capital investment committee of estate landscaping company is considering two ca 666151

The capital investment committee of Estate Landscaping Company is considering two capital investments. The estimated income from operations and net cash flows from each investment are as follows:

 

Greenhouse

Skid Loader

 

Income from

Net Cash

Income from

Net Cash

Year

Operations

Flow

Operations

Flow

1

$16,000

$ 30,000

$26,000

$ 40,000

2

16,000

30,000

21,000

35,000

3

16,000

30,000

16,000

30,000

4

16,000

30,000

11,000

25,000

5

16,000

30,000

6,000

20,000

 

$80,000

$150,000

$80,000

$150,000

Each project requires an investment of $70,000. Straight line depreciation will be used, and no residual value is expected. The committee has selected a rate of 12% for purposes of the net present value analysis.

Instructions

1. Compute the following:

a. The average rate of return for each investment. Round to one decimal place.

b. The net present value for each investment. Use the present value of $1 table appearing in this chapter.

2. Prepare a brief report for the capital investment committee, advising it on the relative merits of the two investments.

the condensed income statement for the european division of cougar motors inc is as 666088

The condensed income statement for the European Division of Cougar Motors Inc. is as follows (assuming no service department charges):

Sales

$875,000

Cost of goods sold

400,000

Gross profit

$475,000

Administrative expenses

282,500

Income from operations

$192,500

The manager of the European Division is considering ways to increase the rate of return on investment.

a. Using the DuPont formula for rate of return on investment, determine the profit margin,

investment turnover, and rate of return on investment of the European Division, assuming that $1,750,000 of assets have been invested in the European Division.

b. If expenses could be reduced by $52,500 without decreasing sales, what would be the impact on the profit margin, investment turnover, and rate of return on investment for the European Division?

the walt disney company has four major sectors described as follows 666089

The Walt Disney Company has four major sectors, described as follows:

  • Media Networks: The ABC television and radio network, Disney channel, ESPN, A&E, E!, and Disney.com.
  • Parks and Resorts: Walt Disney World Resort, Disneyland, Disney Cruise Line, and other resort properties.
  • Studio Entertainment: Walt Disney Pictures, Touchstone Pictures, Hollywood Pictures, Miramax Films, and Buena Vista Theatrical Productions.
  • Consumer Products: Character merchandising, Disney stores, books, and magazines

Disney recently reported sector income from operations, revenue, and invested assets (in millions) as follows:

 

Income from

 

Invested

 

Operations

Revenue

Assets

Media Networks

$2,749

$13,027

$26,926

Parks and Resorts

1,178

9,023

15,807

Studio Entertainment

207

7,587

5,965

Consumer Products

520

2,157

877

a. Use the DuPont formula to determine the rate of return on investment for the four Disney sectors. Round whole percents to one decimal place and investment turnover to one decimal place.

b. How do the four sectors differ in their profit margin, investment turnover, and return on investment?

data for grobe products company is presented in the following table of rates of retu 666090

Data for Grobe Products Company is presented in the following table of rates of return on investment and residual incomes:

 

 

 

 

Minimum

 

 

Income

Rate of

Minimum

Acceptable

 

Invested

from

Return on

Rate of

Income from

Residual

Assets

Operations

Investment

Return

Operations

Income

$643,750

$115,875

(a)

12%

(b)

(c)

$418,750

(d)

(e)

(f)

$62,813

$16,750

$275,000

(g)

12%

(h)

$44,000

(i)

$600,000

$84,000

(j)

10%

(k)

(l)

Determine the missing items, identifying each item by the appropriate letter.

data for the north east south and west divisions of tor max semiconductor communicat 666091

Data for the North, East, South, and West divisions of Tor Max Semiconductor Communication Company are as follows:

 

 

Income

 

Rate of

 

 

 

 

from

Invested

Return on

Profit

Investment

 

Sales

Operations

Assets

Investment

Margin

Turnover

North

$425,000

(a)

(b)

20%

10%

(c)

East

(d)

$50,000

(e)

(f)

8%

1.25

South

$400,000

(g)

$125,000

12%

(h)

(i)

West

$750,000

$180,000

$1,250,000

(j)

(k)

(l)

a. Determine the missing items, identifying each by the letters (a) through (l).

b. Determine the residual income for each division, assuming that the minimum acceptable rate of return established by management is 9%.

c. Which division is the most profitable in terms of (1) return on investment and (2) residual income?

hilton hotels corporation provides lodging services around the world the company is 666092

Hilton Hotels Corporation provides lodging services around the world. The company is separated into three major divisions:

  • Hotel Ownership: Hotels owned and operated by Hilton.
  • Managing and Franchising: Hotels franchised to others or managed for others.
  • Timeshare: Resort properties managed for timeshare vacation owners.

Financial information for each division, from a recent annual report, is as follows (in millions):

 

Hotel

Managing and

 

 

Ownership

Franchising

Timeshare

Revenues

$2,215

$1,510

$421

Income from operations

394

343

99

Total assets

4,825

2,112

507

a. Use the DuPont formula to determine the return on investment for each of the Hilton business divisions. Round whole percents to one decimal place and investment turnover to one decimal place.

b. Determine the residual income for each division, assuming a minimum acceptable income of 14% of total assets. Round minimal acceptable return to the nearest million dollars.

c. Interpret your results.

american express company is a major financial services company noted for its america 666093

American Express Company is a major financial services company, noted for its American Express® card. Below are some of the performance measures used by the company in its balanced scorecard.

Average cardmember spending

Number of merchant signings

Cards in force

Number of card choices

Earnings growth

Number of new card launches

Hours of credit consultant training

Return on equity

Investment in information technology

Revenue growth

Number of Internet features

 

For each measure, identify whether the measure best fits the innovation, customer, internal process, or financial dimension of the balanced scorecard.

several years ago united parcel service ups believed that the internet was going to 666094

Several years ago, United Parcel Service (UPS) believed that the Internet was going to change the parcel delivery market and would require UPS to become a more nimble and customerfocused organization. As a result, UPS replaced its old measurement system, which was 90% oriented toward financial performance, with a balanced scorecard. The scorecard emphasized four “point of arrival” measures, which were:

1. Customer satisfaction index—a measure of customer satisfaction.

2. Employee relations index—a measure of employee sentiment and morale.

3. Competitive position—delivery performance relative to competition.

4. Time in transit—the time from order entry to delivery.

a. Why did UPS introduce a balanced scorecard and nonfinancial measures in its new performance measurement system?

b. Why do you think UPS included a factor measuring employee sentiment?

materials used by the industrial division of crow manufacturing are currently purcha 666095

Materials used by the Industrial Division of Crow Manufacturing are currently purchased from outside suppliers at a cost of $120 per unit. However, the same materials are available from the Materials Division. The Materials Division has unused capacity and can produce the materials needed by the Industrial Division at a variable cost of $95 per unit.

a. If a transfer price of $105 per unit is established and 40,000 units of materials are transferred, with no reduction in the Materials Division’s current sales, how much would Crow Manufacturing’s total income from operations increase?

b. How much would the Industrial Division’s income from operations increase?

c. How much would the Materials Division’s income from operations increase?

the southwest district of pop soft drinks inc is organized as a cost center the budg 666097

The Southwest District of Pop Soft Drinks, Inc., is organized as a cost center. The budget for the Southwest District of Pop Soft Drinks, Inc., for the month ended May 31, 2008, is as follows:

Sales salaries

$406,725

System support salaries

222,300

Customer relations salaries

75,975

Accounting salaries

48,975

Repair and service

134,625

Depreciation of plant and equipment

45,750

Insurance and property taxes

20,475

Total

$954,825

During May, the costs incurred in the Southwest District were as follows:

Sales salaries

$406,200

System support salaries

222,075

Customer relations salaries

89,025

Accounting salaries

48,675

Repair and service

135,375

Depreciation of plant and equipment

45,750

Insurance and property taxes

20,550

Total

$967,650

Instructions

1. Prepare a budget performance report for the manager of the Southwest District of Pop Soft Drinks for the month of May.

2. For which costs might the supervisor be expected to request supplemental reports?

cross country transport company organizes its three divisions the southeast east and 666098

Cross Country Transport Company organizes its three divisions, the Southeast, East, and South regions, as profit centers. The chief executive officer (CEO) evaluates divisional performance, using income from operations as a percent of revenues. The following quarterly income and expense accounts were provided from the trial balance as of December 31, 2008:

Revenues—SE Region

$1,740,000

Revenues—E Region

2,820,000

Revenues—S Region

2,340,000

Operating Expenses—SE Region

1,134,400

Operating Expenses—E Region

2,097,300

Operating Expenses—S Region

1,721,700

Corporate Expenses—Dispatching

500,000

Corporate Expenses—Equipment

525,000

Corporate Expenses—Treasurer’s

375,000

General Corporate Officers’ Salaries

710,000

The company operates three service departments: the Dispatching Department, the Equipment Management Department, and the Treasurer’s Department. The Dispatching Department manages the scheduling and releasing of completed trains. The Equipment Management Department manages the railroad cars inventories. It makes sure the right freight cars are at the right place at the right time. The Treasurer’s Department conducts a variety of services for the company as a whole. The following additional information has been gathered:

 

Southeast

East

South

Number of scheduled trains

400

680

520

Number of railroad cars in inventory

4,800

6,400

5,600

Instructions

1. Prepare quarterly income statements showing income from operations for the three regions. Use three column headings: Southeast, East, and South.

2. Identify the most successful region according to the profit margin. Round to two decimal places.

3. Provide a recommendation to the CEO for a better method for evaluating the performance of the regions. In your recommendation, identify the major weakness of the present method.

hi growth investments inc is a diversified investment company with three operating d 666099

Hi Growth Investments Inc. is a diversified investment company with three operating divisions organized as investment centers. Condensed data taken from the records of the three divisions for the year ended June 30, 2008, are as follows:

 

 

Electronic

Investment

 

Retail

Brokerage

Banking

 

Division

Division

Division

Fee revenue

$1,250,000

$750,000

$1,500,000

Operating expenses

750,000

682,500

1,170,000

Invested assets

3,125,000

250,000

2,000,000

The management of Hi Growth Investments Inc. is evaluating each division as a basis for planning a future expansion of operations.

Instructions

1. Prepare condensed divisional income statements for the three divisions, assuming that there were no service department charges.

2. Using the DuPont formula for rate of return on investment, compute the profit margin, investment turnover, and rate of return on investment for each division.

3. If available funds permit the expansion of operations of only one division, which of the divisions would you recommend for expansion, based on parts (1) and (2)? Explain.

a condensed income statement for the paintball division of outdoor games inc for the 666100

A condensed income statement for the Paintball Division of Outdoor Games Inc. for the year ended January 31, 2008, is as follows:

Sales

$900,000

Cost of goods sold

500,000

Gross profit

$400,000

Operating expenses

274,000

Income from operations

$126,000

Assume that the Paintball Division received no charges from service departments. The president of Outdoor Games Inc. has indicated that the division’s rate of return on a $720,000 investment must be increased to at least 20% by the end of the next year if operations are to continue. The division manager is considering the following three proposals:

Proposal 1: Transfer equipment with a book value of $120,000 to other divisions at no gain or loss and lease similar equipment. The annual lease payments would be less than the amount of depreciation expense on the old equipment by $18,000. This decrease in expense would be included as part of the cost of goods sold. Sales would remain unchanged.

Proposal 2: Reduce invested assets by discontinuing a product line. This action would eliminate sales of $75,000, cost of goods sold of $35,000, and operating expenses of $37,750. Assets of $32,500 would be transferred to other divisions at no gain or loss.

Proposal 3: Purchase new and more efficient machinery and thereby reduce the cost of goods sold by $27,000. Sales would remain unchanged, and the old machinery, which has no remaining book value, would be scrapped at no gain or loss. The new machinery would increase invested assets by $405,000 for the year.

Instructions

1. Using the DuPont formula for rate of return on investment, determine the profit margin, investment turnover, and rate of return on investment for the Paintball Division for the past year.

2. Prepare condensed estimated income statements and compute the invested assets for each proposal.

3. Using the DuPont formula for rate of return on investment, determine the profit margin, investment turnover, and rate of return on investment for each proposal.

4. Which of the three proposals would meet the required 20% rate of return on investment?

5. If the Paintball Division were in an industry where the profit margin could not be increased, how much would the investment turnover have to increase to meet the president’s required 20% rate of return on investment? Round to two decimal places.

the vice president of operations of i4 computers inc is evaluating the performance o 666101

The vice president of operations of I4 Computers Inc. is evaluating the performance of two divisions organized as investment centers. Invested assets and condensed income statement data for the past year for each division are as follows:

 

Personal

Business Computing

 

Computing Division

Division

Sales

$800,000

$1,200,000

Cost of goods sold

460,000

780,000

Operating expenses

180,000

156,000

Invested assets

500,000

2,000,000

Instructions

1. Prepare condensed divisional income statements for the year ended December 31, 2008, assuming that there were no service department charges.

2. Using the DuPont formula for rate of return on investment, determine the profit margin, investment turnover, and rate of return on investment for each division.

3. If management’s minimum acceptable rate of return is 15%, determine the residual income for each division.

4. Discuss the evaluation of the two divisions, using the performance measures determined in parts (1), (2), and (3).

goho manufacturing company is a diversified aerospace company including two operatin 666102

Goho Manufacturing Company is a diversified aerospace company, including two operating divisions, Specialized Semiconductors and Navigational Systems Divisions. Condensed divisional income statements, which involve no intracompany transfers and which include a breakdown of expenses into variable and fixed components, are as follows:

Goho Manufacturing Company

Divisional Income Statements

For the Year Ended December 31, 2008

 

Specialized Semi

Navigational

 

 

conductors Division

Systems Division

Total

Sales:

 

 

 

640 units × $1,320 per unit

$844,800

 

$ 844,800

1,000 units × $1,984 per unit

 

$1,984,000

1,984,000

 

 

 

$2,828,800

Expenses:

 

 

 

Variable:

 

 

 

640 units × $776 per unit

$496,640

 

$ 496,640

1,000 units × $1,560* per unit

 

$1,560,000

1,560,000

Fixed

195,200

254,400

449,600

Total expenses

$691,840

$1,814,400

$2,506,240

Income from operations

$152,960

$ 169,600

$ 322,560

*$1,320 of the $1,560 per unit represents materials costs, and the remaining $240 per unit represents other variable conversion expenses incurred within the Navigational Systems Division.

The Specialized Semiconductors Division is presently producing 640 units out of a total capacity of 800 units. Materials used in producing the Navigational Systems Division’s product are currently purchased from outside suppliers at a price of $1,320 per unit. The Specialized Semiconductors Division is able to produce the components used by the Navigational Systems Division. Except for the possible transfer of materials between divisions, no changes are expected in sales and expenses.

Instructions

1. Would the market price of $1,320 per unit be an appropriate transfer price for Goho Manufacturing Company? Explain.

2. If the Navigational Systems Division purchases 160 units from the Specialized Semiconductors Division, rather than externally, at a negotiated transfer price of $1,160 per unit, how much would the income from operations of each division and total company income from operations increase?

3. Prepare condensed divisional income statements for Goho Manufacturing Company, based on the data in part (2).

4. If a transfer price of $880 per unit is negotiated, how much would the income from operations of each division and total company income from operations increase?

5. a. What is the range of possible negotiated transfer prices that would be acceptable for Goho Manufacturing Company?

b. Assuming that the managers of the two divisions cannot agree on a transfer price, what price would you suggest as the transfer price?

the furnishings company sells furnishings and fixtures over the internet the interna 666103

The Furnishings Company sells furnishings and fixtures over the Internet. The International Division is organized as a cost center. The budget for the International Division for the month ended October 31, 2008, is as follows (in millions):

Customer service salaries

$ 119

Insurance and property taxes

32

Distribution salaries

238

Marketing salaries

336

Engineer salaries

217

Warehouse wages

147

Equipment depreciation

45

Total

$1,134

During October, the costs incurred in the International Division were as follows:

Customer service salaries

$ 153

Insurance and property taxes

31

Distribution salaries

235

Marketing salaries

377

Engineer salaries

213

Warehouse wages

141

Equipment depreciation

45

Total

$1,195

Instructions

1. Prepare a budget performance report for the director of the International Division for the month of October.

2. For which costs might the director be expected to request supplemental reports?

diversified railroad has three regional divisions organized as profit centers the ch 666104

Diversified Railroad has three regional divisions organized as profit centers. The chief executive officer (CEO) evaluates divisional performance, using income from operations as a percent of revenues. The following quarterly income and expense accounts were provided from the trial balance as of December 31, 2008:

Revenues—East Division

$500,000

Revenues—West Division

690,000

Revenues—Metro Division

944,000

Operating Expenses—East Division

303,000

Operating Expenses—West Division

399,600

Operating Expenses—Metro Division

536,000

Corporate Expenses—Shareholder Relations

75,000

Corporate Expenses—Customer Support

192,000

Corporate Expenses—Legal

102,000

General Corporate Officers’ Salaries

180,000

The company operates three service departments: Shareholder Relations, Customer Support, and Legal. The Shareholder Relations Department conducts a variety of services for shareholders of the company. The Customer Support Department is the company’s point of contact for new service, complaints, and requests for repair. The department believes that the number of customer contacts is an activity base for this work. The Legal Department provides reports for division management. The department believes that the number of hours billed is an activity base for this work. The following additional information has been gathered:

 

East

West

Metro

Number of customer contacts

3,200

3,680

5,920

Number of hours billed

640

1,120

960

Instructions

1. Prepare quarterly income statements showing income from operations for the three divisions. Use three column headings: East, West, and Metro.

2. Identify the most successful division according to the profit margin. Round to two decimal places.

3. Provide a recommendation to the CEO for a better method for evaluating the performance of the divisions. In your recommendation, identify the major weakness of the present method.

fresh tracks baking company is a diversified food products company with three operat 666105

Fresh Tracks Baking Company is a diversified food products company with three operating divisions organized as investment centers. Condensed data taken from the records of the three divisions for the year ended June 30, 2008, are as follows:

 

 

 

Retail

 

Bread

Snack Cake

Bakeries

 

Division

Division

Division

Sales

$1,450,000

$1,750,000

$1,000,000

Cost of goods sold

950,000

1,125,000

650,000

Operating expenses

268,000

380,000

170,000

Invested assets

1,812,500

2,187,500

800,000

The management of Fresh Tracks Baking Company is evaluating each division as a basis for planning a future expansion of operations.

Instructions

1. Prepare condensed divisional income statements for the three divisions, assuming that there were no service department charges.

2. Using the DuPont formula for rate of return on investment, compute the profit margin, investment turnover, and rate of return on investment for each division.

3. If available funds permit the expansion of operations of only one division, which of the divisions would you recommend for expansion, based on parts (1) and (2)? Explain.

the vice president of operations of cantor simmons cycle company is evaluating the p 666107

The vice president of operations of Cantor Simmons Cycle Company is evaluating the performance of two divisions organized as investment centers. Invested assets and condensed income statement data for the past year for each division are as follows:

 

Road Bike

Mountain Bike

 

Division

Division

Sales

$750,000

$ 950,000

Cost of goods sold

412,500

560,000

Operating expenses

187,500

181,000

Invested assets

600,000

1,187,500

Instructions

1. Prepare condensed divisional income statements for the year ended December 31, 2008, assuming that there were no service department charges.

2. Using the DuPont formula for rate of return on investment, determine the profit margin, investment turnover, and rate of return on investment for each division.

3. If management desires a minimum acceptable rate of return of 18%, determine the residual income for each division.

4. Discuss the evaluation of the two divisions, using the performance measures determined in parts (1), (2), and (3).

hi tech electronics inc manufactures electronic products with two operating division 666108

Hi Tech Electronics, Inc. manufactures electronic products, with two operating divisions, the Specialized Electronic Component and MP3 Player divisions. Condensed divisional income statements, which involve no intracompany transfers and which include a breakdown of expenses into variable and fixed components, are as follows:

Hi Tech Electronics, Inc.

Divisional Income Statements

For the Year Ended December 31, 2008

 

Specialized Electronic

MP3 Player

 

 

Component Division

Division

Total

Sales:

 

 

 

12,000 units × $126 per unit

$1,512,000

 

$1,512,000

18,000 units × $228 per unit

 

$4,104,000

4,104,000

 

 

 

$5,616,000

Expenses:

 

 

 

Variable:

 

 

 

12,000 units × $86 per unit

$1,032,000

 

$1,032,000

18,000 units × $162* per unit

 

$2,916,000

2,916,000

Fixed

186,000

432,000

618,000

Total expenses

$1,218,000

$3,348,000

$4,566,000

Income from operations

$ 294,000

$ 756,000

$1,050,000

*$126 of the $162 per case represents materials costs, and the remaining $36 per case represents other variable conversion expenses incurred within the MP3 Player Division.

The Specialized Electronic Component Division is presently producing 12,000 units out of a total capacity of 14,400 units. Materials used in producing the MP3 Player Division’s product are currently purchased from outside suppliers at a price of $126 per unit. The Specialized Electronic Component Division is able to produce the materials used by the MP3 Player Division. Except for the possible transfer of materials between divisions, no changes are expected in sales and expenses.

Instructions

1. Would the market price of $126 per unit be an appropriate transfer price for Hi Tech Electronics, Inc.? Explain.

2. If the MP3 Player Division purchases 2,400 units from the Specialized Electronic Component Division, rather than externally, at a negotiated transfer price of $96 per unit, how much would the income from operations of each division and the total company income from operations increase?

3. Prepare condensed divisional income statements for Hi Tech Electronics, Inc., based on the data in part (2).

4. If a transfer price of $120 per unit is negotiated, how much would the income from operations of each division and the total company income from operations increase?

5. a. What is the range of possible negotiated transfer prices that would be acceptable for Hi Tech Electronics, Inc.?

b. Assuming that the managers of the two divisions cannot agree on a transfer price, what price would you suggest as the transfer price?

micro tech company has two divisions the semiconductor division and the pc division 666109

Micro Tech Company has two divisions, the Semiconductor Division and the PC Division. The PC Division may purchase semiconductors from the Semiconductor Division or from outside suppliers. The Semiconductor Division sells semiconductor products both internally and externally. The market price for semiconductors is $250 per 100 semiconductors. Michael Blount is the controller of the PC Division, and Lynn Williams is the controller of the Semiconductor Division. The following conversation took place between Michael and Lynn:

Michael: I hear you are having problems selling semiconductors out of your division. Maybe I can help.

Lynn: You’ve got that right. We’re producing and selling at about 80% of our capacity to outsiders. Last year we were selling 100% of capacity. Would it be possible for your division to pick up some of our excess capacity? After all, we are part of the same company.

Michael: What kind of price could you give me?

Lynn: Well, you know as well as I that we are under strict profit responsibility in our divisions, so I would expect to get market price, $250 for 100 semiconductors.

Michael: I’m not so sure we can swing that. I was expecting a price break from a “sister” division.

Lynn: Hey, I can only take this “sister” stuff so far. If I give you a price break, our profits will fall from last year’s levels. I don’t think I could explain that. I’m sorry, but I must remain firm—market price. After all, it’s only fair—that’s what you would have to pay from an external supplier.

Michael: Fair or not, I think we’ll pass. Sorry we couldn’t have helped.

Was Michael behaving ethically by trying to force the Semiconductor Division into a price break? Comment on Lynn’s reactions

suregrip tire co manufactures automobile tires standard costs and actual costs for d 666049

SureGrip Tire Co. manufactures automobile tires. Standard costs and actual costs for direct materials, direct labor, and factory overhead incurred for the manufacture of 39,000 tires were as follows:

 

Standard Costs

Actual Costs

Direct materials

68,000 pounds at $4.80

72,100 pounds at $5.20

Direct labor

15,600 hours at $16.00

15,400 hours at $15.70

 

 

 

Factory overhead

Rates per direct labor hour,

 

 

based on 100% of normal

 

 

capacity of 21,000 direct

 

 

labor hours:

 

 

Variable cost, $2.70

$41,650 variable cost

 

Fixed cost, $3.65

$76,650 fixed cost

Each tire requires 0.40 hour of direct labor.

Instructions

Determine (a) the price variance, quantity variance, and total direct materials cost variance; (b) the rate variance, time variance, and total direct labor cost variance; and (c) variable factory overhead controllable variance, the fixed factory overhead volume variance, and total factory overhead cost variance.

med tech company a manufacturer of disposable medical supplies prepared the followin 666050

Med Tech Company, a manufacturer of disposable medical supplies, prepared the following factory overhead cost budget for the Assembly Department for August 2008. The company expected to operate the department at 100% of normal capacity of 28,000 hours.

Variable costs:

 

 

Indirect factory wages

$246,400

 

Power and light

179,200

 

Indirect materials

44,800

 

Total variable cost

 

$470,400

Fixed costs:

 

 

Supervisory salaries

$130,000

 

Depreciation of plant and equipment

105,000

 

Insurance and property taxes

22,600

 

Total fixed cost

 

257,600

Total factory overhead cost

 

$728,000

During August, the department operated at 25,250 hours, and the factory overhead costs incurred were indirect factory wages, $216,500; power and light, $162,600; indirect materials, $38,200; supervisory salaries, $130,000; depreciation of plant and equipment, $105,000; and insurance and property taxes, $22,600.

Instructions

Prepare a factory overhead cost variance report for August. To be useful for cost control, the budgeted amounts should be based on 25,250 hours.

the radiology department provides imaging services for memorial medical center 666051

The Radiology Department provides imaging services for Memorial Medical Center. One important activity in the Radiology Department is transcribing tape recorded analyses of images into a written report. The manager of the Radiology Department determined that the average transcriptionist could type 800 lines of a report in an hour. The plan for the first week in July called for 60,000 typed lines to be written. The Radiology Department has two transcriptionists. Each transcriptionist is hired from an employment firm that requires temporary employees to be hired for a minimum of a 40 hour week. Transcriptionists are paid $15.00 per hour. The manager offered a bonus if the department could type more than 70,000 lines for the week, without overtime. Due to high service demands, the transcriptionists typed more lines in the first week of July than planned. The actual amount of lines typed in the first week of July was 72,000 lines, without overtime. As a result, the bonus caused the average transcriptionist hourly rate to increase to $18.00 per hour during the first week in July.

Instructions

1. If the department typed 60,000 lines according to the original plan, what would have been the labor time variance?

2. What was the labor time variance as a result of typing 72,000 lines?

3. What was the labor rate variance as a result of the bonus?

4. The manager is trying to determine if a better decision would have been to hire a temporary transcriptionist to meet the higher typing demands in the first week of July, rather than paying out the bonus. If another employee was hired from the employment firm, what would have been the labor time variance in the first week?

5. Which decision is better, paying the bonus or hiring another transcriptionist?

6. Are there any performance related issues that the labor time and rate variances fail to consider? Explain.

royal essentials inc began operations on january 1 2008 the company produces a hand 666052

Royal Essentials, Inc. began operations on January 1, 2008. The company produces a hand and body lotion in an eight ounce bottle called Eternal Beauty. The lotion is sold wholesale in 12 bottle cases for $80 per case. There is a selling commission of $16 per case. The January direct materials, direct labor, and factory overhead costs are as follows:

DIRECT MATERIALS

 

Cost

Units

Cost

Direct Materials

 

Behavior

per Case

per Unit

Cost per Case

Cream base

Variable

72 ozs.

$0.015

$ 1.08

Natural oils

Variable

24 ozs.

0.250

6.00

Bottle (8 oz.)

Variable

12 bottles

0.400

4.80

 

 

 

 

$11.88

 

DIRECT LABOR

 

Cost

Time

Labor Rate

Direct Labor Cost

Department

Behavior

per Case

per Hour

per Case

Mixing

Variable

16.80 min.

$15.00

$4.20

Filling

Variable

4.20 min.

12.00

0.84

 

 

21.00 min.

 

$5.04

 

FACTORY OVERHEAD

 

Cost Behavior

Total Cost

Utilities

Mixed

$ 230

Facility lease

Fixed

9,694

Equipment depreciation

Fixed

3,600

Supplies

Fixed

600

 

 

$14,124

Part A—Break Even Analysis

The management of Royal Essentials, Inc., wishes to determine the number of cases required to break even per month. The utilities cost, which is part of factory overhead, is a mixed cost. The following information was gathered from the first six months of operation regarding this cost:

2008

Case Production

Utility Total Cost

January

300

$230

February

600

265

March

1,000

300

April

900

292

May

750

275

June

825

280

Instructions

1. Determine the fixed and variable portion of the utility cost using the high low method.

2. Determine the contribution margin per case.

3. Determine the fixed costs per month, including the utility fixed cost from part (1).

4. Determine the break even number of cases per month.

Part B—August Budget

During July of the current year, the management of Royal Essentials, Inc., asked the controller to prepare August manufacturing and income statement budgets. Demand was expected to be 1,200 cases at $80 per case for August. Inventory planning information is provided as follows:

Finished Goods Inventory:

 

 

 

 

 

Cases

Cost

Estimated finished goods inventory, August 1, 2008

 

150

$3,160

Desired finished goods inventory, August 31, 2008

 

100

2,100

Materials Inventory:

 

 

 

 

Cream Base

Oils

Bottles

 

(ozs.)

(ozs.)

(bottles)

Estimated materials inventory, August 1, 2008

500

260

500

Desired materials inventory, August 31, 2008

700

300

400

There was negligible work in process inventory assumed for either the beginning or end of the month; thus, none was assumed. In addition, there was no change in the cost per unit or estimated units per case operating data from January.

Instructions

5. Prepare the August production budget.

6. Prepare the August direct materials purchases budget.

7. Prepare the August direct labor budget.

8. Prepare the August factory overhead budget.

9. Prepare the August budgeted income statement, including selling expenses.

Part C—August Variance Analysis

During September of the current year, the controller was asked to perform variance analyses for August. The January operating data provided the standard prices, rates, times, and quantities per case. There were 1,200 actual cases produced during August, which was 50 more cases than planned at the beginning of the month. Actual data for August were as follows:

 

Actual Direct Materials

Actual Direct Materials

 

Price per Case

Quantity per Case

Cream base

$1.00 (for 72 ozs.)

75 ozs.

Natural oils

6.20 (for 24 ozs.)

25 ozs.

Bottle (8 oz.)

4.50 (for 12 bottles)

12.2 bottles

 

 

 

Actual Direct Labor

 

Actual Direct Labor Rate

Time per Case

Mixing

$15.25

16.50 min.

Filling

11.50

4.50 min.

Actual variable overhead

$125.00

 

Normal volume

1,500 cases

 

The prices of the materials were different than standard due to fluctuations in market prices. Specifically, the prices of the cream base and bottles were below the standard price, while the price of natural oils was above the standard price. The standard quantity of materials used per case was an ideal standard. The Mixing Department used a higher grade labor classification during the month, thus causing the actual labor rate to exceed standard. The Filling Department used a lower grade labor classification during the month, thus causing the actual labor rate to be less than standard.

Instructions

10. Determine and interpret the direct materials price and quantity variances for the three materials.

11. Determine and interpret the direct labor rate and time variances for the two departments.

12. Determine and interpret the factory overhead controllable variance.

13. Determine and interpret the factory overhead volume variance.

14. Why are the standard direct labor and direct materials costs in the calculations for parts (10) and (11) based on the actual 1,200 case production volume rather than the planned 1,150 cases of production used in the budgets for parts (6) and (7)?

trey mcintyre is a cost analyst with global insurance company global is applying sta 666053

Trey McIntyre is a cost analyst with Global Insurance Company. Global is applying standards to its claims payment operation. Claims payment is a repetitive operation that could be evaluated with standards. Trey used time and motion studies to identify an ideal standard of 36 claims processed per hour. The Claims Processing Department manager, Carol Mann, has rejected this standard and has argued that the standard should be 30 claims processed per hour. Carol and Trey were unable to agree, so they decided to discuss this matter openly at a joint meeting with the vice president of operations, who would arbitrate a final decision. Prior to the meeting, Trey wrote the following memo to the VP.

To:

T. J. Logan, Vice President of Operations

From:

Trey McIntyre

Re:

Standards in the Claims Processing Department

As you know, Carol and I are scheduled to meet with you to discuss our disagreement with respect to the appropriate standards for the Claims Processing Department. I have conducted time and motion studies and have determined that the ideal standard is 36 claims processed per hour. Carol argues that 30 claims processed per hour would be more appropriate. I believe she is trying to “pad” the budget with some slack. I’m not sure what she is trying to get away with, but I believe a tight standard will drive efficiency up in her area. I hope you will agree when we meet with you next week.

Discuss the ethical and professional issues in this situation.

at the soladyne division of rogers corporation a manufacturer of specialty materials 666054

At the Soladyne Division of Rogers Corporation, a manufacturer of specialty materials for the electronics industry, the controller used a number of measures to provide managers information about the performance of a just in time (JIT) manufacturing operation. Three measures used by the company are:

  • Scrap Index: The sales dollar value of scrap for the period.
  • Orders Past Due: Sales dollar value of orders that were scheduled for shipment, but were not shipped during the period.
  • Buyer’s Misery Index: Number of different customers that have orders that are late (scheduled for shipment, but not shipped).

1. Why do you think the scrap index is measured at sales dollar value, rather than at cost?

2. How is the “orders past due” measure different from the “buyer’s misery index,” or are the two measures just measuring the same thing?

you have been asked to investigate some cost problems in the assembly department of 666055

You have been asked to investigate some cost problems in the Assembly Department of Digital Life Electronics Co., a consumer electronics company. To begin your investigation, you have obtained the following budget performance report for the department for the last quarter:

Digital Life Electronics Co.—Assembly Department

Quarterly Budget Performance Report

 

Standard

Actual Quantity

 

 

Quantity at

at Standard

Quantity

 

Standard Rates

Rates

Variances

Direct labor

$ 78,750

$113,750

$35,000 U

Direct materials

148,750

192,500

43,750 U

Total

$227,500

$306,250

$78,750 U

The following reports were also obtained:

Digital Life Electronics Co.—Purchasing Department

Quarterly Budget Performance Report

 

Actual

Actual

 

 

Quantity at

Quantity at

Price

 

Standard Rates

Actual Rates

Variance

Direct materials

$218,750

$192,500

$26,250 F

Digital Life Electronics Co.—Fabrication Department

Quarterly Budget Performance Report

 

Standard

Actual Quantity

 

 

Quantity at

at Standard

Quantity

 

Standard Rates

Rates

Variances

Direct labor

$122,500

$101,500

$21,000 F

Direct materials

70,000

70,000

0

Total

$192,500

$171,500

$21,000 F

You also interviewed the Assembly Department supervisor. Excerpts from the interview follow.

Q: What explains the poor performance in your department?

A: Listen, you’ve got to understand what it’s been like in this department recently. Lately, it seems no matter how hard we try, we can’t seem to make the standards. I’m not sure what is going on, but we’ve been having a lot of problems lately.

Q: What kind of problems?

A: Well, for instance, all this quarter we’ve been requisitioning purchased parts from the material storeroom, and the parts just didn’t fit together very well. I’m not sure what is going on, but during most of this quarter we’ve had to scrap and sort purchased parts—just to get our assemblies put together. Naturally, all this takes time and material. And that’s not all.

Q: Go on.

A: All this quarter, the work that we’ve been receiving from the Fabrication Department has been shoddy. I mean, maybe around 20% of the stuff that comes in from Fabrication just can’t be assembled. The fabrication is all wrong. As a result, we’ve had to scrap and rework a lot of the stuff. Naturally, this has just shot our quantity variances.

Interpret the variance reports in light of the comments by the Assembly Department supervisor.

quinn company has two divisions domestic and international invested assets and conde 666061

Quinn Company has two divisions, Domestic and International. Invested assets and condensed income statement data for each division for the past year ended December 31 are as follows:

 

Domestic Division

International Division

Revenues

$675,000

$480,000

Operating expenses

450,000

372,400

Service department charges

90,000

50,000

Invested assets

600,000

384,000

Instructions

1. Prepare condensed income statements for the past year for each division.

2. Using the DuPont formula, determine the profit margin, investment turnover, and rate of return on investment for each division.

3. If management’s minimum acceptable rate of return is 10%, determine the residual income for each division.

the centralized employee travel department of wilson company has expenses of 150 000 666070

The centralized employee Travel Department of Wilson Company has expenses of $150,000. The department has serviced a total of 2,500 travel reservations for the period. The Midwest Division has made 1,000 reservations during the period, and the Southeast Division has made 1,500 reservations. How much should each division be charged for travel services?

Using the data for the Wilson Company along with the data provided below, determine the divisional income from operations for the Midwest and Southeast divisions.

 

Midwest Division

Southeast Division

Sales

$600,000

$750,000

Cost of goods sold

315,000

450,000

Selling expenses

138,750

165,000

the centralized help desk of exton company has expenses of 120 000 the department ha 666071

The centralized Help Desk of Exton Company has expenses of $120,000. The department has provided a total of 12,000 hours of service for the period. The Fabrication Division has used 5,000 hours of Help Desk service during the period, and the Assembly Division has used 7,000 hours of Help Desk service. How much should each division be charged for travel services?

Using the data for the Exton Company along with the data provided below, determine the divisional income from operations for the Fabrication and Assembly divisions.

 

Fabrication Division

Assembly Division

Sales

$1,080,000

$1,200,000

Cost of goods sold

567,000

740,000

Selling expenses

249,750

230,000

partially completed budget performance reports for qual tech company a manufacturer 666076

Partially completed budget performance reports for Qual Tech Company, a manufacturer of air conditioners, are provided below.

Qual Tech Company

Budget Performance Report—Vice President, Production

For the Month Ended April 30, 2008

Plant

Budget

Actual

Over Budget

Under Budget

North Region

$362,460

$360,920

 

$1,540

Central Region

259,980

258,580

 

1,400

South Region

(g)

(h)

$ (i)

 

 

$ (j)

$ (k)

$ (l)

$2,940

 

Qual Tech Company

Budget Performance Report—Manager, South Region Plant

For the Month Ended April 30, 2008

Department

Budget

Actual

Over Budget

Under Budget

Chip Fabrication

$ (a)

$ (b)

$ (c)

 

Electronic Assembly

74,480

75,460

980

 

Final Assembly

119,980

119,560

 

$420

 

$ (d)

$ (e)

$ (f)

$420

 

Qual Tech Company

Budget Performance Report—Supervisor, Chip Fabrication

For the Month Ended April 30, 2008

Costs

Budget

Actual

Over Budget

Under Budget

Factory wages

$ 21,560

$ 23,100

$1,540

 

Materials

60,900

60,480

 

$420

Power and light

3,360

3,990

630

 

Maintenance

5,880

6,440

560

 

 

$ 91,700

$ 94,010

$2,730

$420

a. Complete the budget performance reports by determining the correct amounts for the lettered spaces.

b. Compose a memo to Dana Johnson, vice president of production for Qual Tech Company, explaining the performance of the production division for April.

the following data were summarized from the accounting records for huggins construct 666077

The following data were summarized from the accounting records for Huggins Construction Company for the year ended June 30, 2008:

Cost of goods sold:

 

Service department charges:

 

Residential Division

$300,800

Residential Division

$ 54,240

Industrial Division

167,840

Industrial Division

24,960

Administrative expenses:

 

Net sales:

 

Residential Division

$80,320

Residential Division

$516,000

Industrial Division

66,560

Industrial Division

321,920

Prepare divisional income statements for Huggins Construction Company.

in divisional income statements prepared for franklin electrical company the payroll 666080

In divisional income statements prepared for Franklin Electrical Company, the Payroll Department costs are charged back to user divisions on the basis of the number of payroll checks, and the Purchasing Department costs are charged back on the basis of the number of purchase requisitions. The Payroll Department had expenses of $44,010, and the Purchasing Department had expenses of $18,720 for the year. The following annual data for Residential, Commercial, and Government Contract Divisions were obtained from corporate records:

 

 

 

Government

 

Residential

Commercial

Contract

Sales

$420,000

$500,000

$1,800,000

Number of employees:

 

 

 

Weekly payroll (52 weeks per year)

144

72

108

Monthly payroll

25

20

18

Number of purchase requisitions per year

1,800

1,530

1,350

a. Determine the total amount of payroll checks and purchase requisitions processed per year by each division.

b. Using the activity base information in (a), determine the annual amount of payroll and purchasing costs charged back to the Residential, Commercial, and Government Contract Divisions from payroll and purchasing services.

c. Why does the Residential Division have a larger service department charge than the other two divisions, even though its sales are lower?

harris corporation a manufacturer of electronics and communications systems uses a s 666081

Harris Corporation, a manufacturer of electronics and communications systems, uses a service department charge system to charge profit centers with Computing and Communications Services (CCS) service department costs. The following table identifies an abbreviated list of service categories and activity bases used by the CCS department. The table also includes some assumed cost and activity base quantity information for each service for October.

 

 

 

Assumed Activity

CCS Service Category

Activity Base

Assumed Cost

Base Quantity

Help desk

Number of calls

$ 73,600

2,300

Network center

Number of devices monitored

614,250

9,450

Electronic mail

Number of user accounts

53,550

6,300

Local voice support

Number of phone extensions

127,238

8,775

One of the profit centers for Harris Corporation is the Communication Systems (COMM) sector. Assume the following information for the COMM sector:

  • The sector has 4,000 employees, of whom 50% are office employees.
  • All the office employees have a phone, and 90% of them have a computer on the network.
  • Ninety percent of the employees with a computer also have an e mail account.
  • The average number of help desk calls for October was 0.50 call per individual with a computer.
  • There are 300 additional printers, servers, and peripherals on the network beyond the personal computers.

a. Determine the service charge rate for the four CCS service categories for October.

b. Determine the charges to the COMM sector for the four CCS service categories for October.

waverunner watersports company has two divisions commercial and consumer and two cor 666082

Waverunner Watersports Company has two divisions, Commercial and Consumer, and two corporate service departments, Tech Support and Accounts Payable. The corporate expenses for the year ended December 31, 2008, are as follows:

Tech Support Department

$ 588,000

Accounts Payable Department

231,000

Other corporate administrative expenses

343,000

Total corporate expense

$1,162,000

The other corporate administrative expenses include officers’ salaries and other expenses required by the corporation. The Tech Support Department charges the divisions for services rendered, based on the number of computers in the department, and the Accounts Payable Department charges divisions for services, based on the number of checks issued. The usage of service by the two divisions is as follows:

 

Tech Support

Accounts Payable

Commercial Division

252 computers

5,880 checks

Consumer Division

168

10,920

Total

420 computers

16,800 checks

The service department charges of the Tech Support Department and the Accounts Payable Department are considered controllable by the divisions. Corporate administrative expenses are not considered controllable by the divisions. The revenues, cost of goods sold, and operating expenses for the two divisions are as follows:

 

Commerical

Consumer

Revenues

$5,600,000

$4,760,000

Cost of goods sold

2,940,000

2,240,000

Operating expenses

1,050,000

980,000

Prepare the divisional income statements for the two divisions.

worldwide air inc has two divisions organized as profit centers the passenger divisi 666083

Worldwide Air, Inc., has two divisions organized as profit centers, the Passenger Division and the Cargo Division. The following divisional income statements were prepared:

 

Passenger Division

Cargo Division

Revenues

 

$600,000

 

$600,000

Operating expenses

 

300,000

 

250,000

Income from operations before

 

 

 

 

service department charges

 

$300,000

 

$350,000

Less service department charges:

 

 

 

 

Training

$50,000

 

$50,000

 

Trip scheduling

60,000

 

60,000

 

Reservations

80,000

190,000

80,000

190,000

Income from operations

 

$110,000

 

$160,000

The service department charge rate for the service department costs was based on revenues. Since the revenues of the two divisions were the same, the service department charges to each division were also the same.

The following additional information is available:

 

Passenger

Cargo

 

 

Division

Division

Total

Number of personnel trained

40

10

50

Number of trips

30

50

80

Number of reservations requested

4,000

4,000

a. Does the income from operations for the two divisions accurately measure performance?

b. Correct the divisional income statements, using the activity bases provided above in revising the service department charges.

outdoor athletic equipment co operates two divisions the winter sports division and 666084

Outdoor Athletic Equipment Co. operates two divisions—the Winter Sports Division and the Summer Sports Division. The following income and expense accounts were provided from the trial balance as of June 30, 2008, the end of the current fiscal year, after all adjustments, including those for inventories, were recorded and posted:

Sales—Winter Sports (WS) Division

$ 950,000

Sales—Summer Sports (SS) Division

1,437,500

Cost of Goods Sold—Winter Sports (WS) Division

512,500

Cost of Goods Sold—Summer Sports (SS) Division

687,500

Sales Expense—Winter Sports (WS) Division

150,000

Sales Expense—Summer Sports (SS) Division

205,000

Administrative Expense—Winter Sports (WS) Division

97,000

Administrative Expense—Summer Sports (SS) Division

128,000

Advertising Expense

64,500

Transportation Expense

100,700

Accounts Receivable Collection Expense

58,100

Warehouse Expense

120,000

The bases to be used in allocating expenses, together with other essential information, are as follows:

a. Advertising expense—incurred at headquarters, charged back to divisions on the basis of usage: Winter Sports Division, $28,000; Summer Sports Division, $36,500.

b. Transportation expense—charged back to divisions at a transfer price of $7.60 per bill of lading: Winter Sports Division, 6,000 bills of lading; Summer Sports Division, 7,250 bills of lading.

c. Accounts receivable collection expense—incurred at headquarters, charged back to divisions at a transfer price of $5.60 per invoice: Winter Sports Division, 4,500 sales invoices; Summer Sports Division, 5,875 sales invoices.

d. Warehouse expense—charged back to divisions on the basis of floor space used in storing division products: Winter Sports Division, 25,000 square feet; Summer Sports Division, 12,500 square feet.

Prepare a divisional income statement with two column headings: Winter Sports Division and Summer Sports Division. Provide supporting schedules for determining service department charges.

assume that management has established a 12 minimum acceptable rate of return for in 666086

Assume that management has established a 12% minimum acceptable rate of return for invested assets.

 

Income

 

 

from Operations

Invested Assets

Magazine Division

$ 96,000

$ 800,000

Textbook Division

166,400

640,000

Business Publishing Division

260,400

1,240,000

a. Determine the residual income for each division.

b. Which division has the most residual income?

sweet swiss chocolate company produces chocolate bars the primary materials used in 666020

Sweet Swiss Chocolate Company produces chocolate bars. The primary materials used in producing chocolate bars are cocoa, sugar, and milk. The standard costs for a batch of chocolate (1,000 bars) are as follows:

Ingredient

Quantity

Price

Cocoa

465 pounds

$0.30 per pound

Sugar

168 pounds

$0.50 per pound

Milk

110 gallons

$1.15 per gallon

Determine the standard direct materials cost per bar of chocolate.

carolina furniture company manufactures unfinished oak furniture carolina uses a sta 666021

Carolina Furniture Company manufactures unfinished oak furniture. Carolina uses a standard cost system. The direct labor, direct materials, and factory overhead standards for an unfinished dining room table are as follows:

Direct labor:

standard rate

$17.00 per hour

 

standard time per unit

3 hours

Direct materials (oak):

standard price

$8.60 per board foot

 

standard quantity

16 board feet

Variable factory overhead:

standard rate

$2.60 per direct labor hour

Fixed factory overhead:

standard rate

$1.20 per direct labor hour

Determine the standard cost per dining room table.

vernon bottle company vbc manufactures plastic two liter bottles for the beverage in 666022

Vernon Bottle Company (VBC) manufactures plastic two liter bottles for the beverage industry. The cost standards per 100 two liter bottles are as follows:

 

Standard Cost

 

per 100 Two Liter

Cost Category

Bottles

Direct labor

$1.28

Direct materials

5.21

Factory overhead

0.42

Total

$6.91

At the beginning of August, VBC management planned to produce 620,000 bottles. The actual number of bottles produced for August was 650,000 bottles. The actual costs for August of the current year were as follows:

 

Actual Cost for the

Cost Category

Month Ended August 31, 2008

Direct labor

$ 8,200

Direct materials

34,500

Factory overhead

2,800

Total

$45,500

a. Prepare the August manufacturing standard cost budget (direct labor, direct materials, and factory overhead) for VBC, assuming planned production.

b. Prepare a budget performance report for manufacturing costs, showing the total cost variances for direct materials, direct labor, and factory overhead for August.

c. Interpret the budget performance report.

the following data relate to the direct materials cost for the production of 4 000 a 666023

The following data relate to the direct materials cost for the production of 4,000 automobile tires:

Actual:

130,240 pounds at $1.65

$214,896

Standard:

128,760 pounds at $1.85

$238,206

a. Determine the price variance, quantity variance, and total direct materials cost variance.

b. To whom should the variances be reported for analysis and control?

the following data relating to direct materials cost for march of the current year a 666024

The following data relating to direct materials cost for March of the current year are taken from the records of Top Toys Inc., a manufacturer of plastic toys:

Quantity of direct materials used

40,000 pounds

Actual unit price of direct materials

$1.48 per pound

Units of finished product manufactured

7,600 units

Standard direct materials per unit of finished product

5 pounds

Direct materials quantity variance—unfavorable

$2,700

Direct materials price variance—unfavorable

$5,200

Determine the standard direct materials cost per unit of finished product, assuming that there was no inventory of work in process at either the beginning or the end of the month.

h j heinz company uses standards to control its materials costs assume that a batch 666025

H.J. Heinz Company uses standards to control its materials costs. Assume that a batch of ketchup (1,500 pounds) has the following standards:

 

Standard Quantity

Standard Price

Whole tomatoes

2,400 pounds

$0.40 per pound

Vinegar

130 gallons

2.50 per gallon

Corn syrup

10 gallons

8.00 per gallon

Salt

54 pounds

2.50 per pound

The actual materials in a batch may vary from the standard due to tomato characteristics. Assume that the actual quantities of materials for batch W196 were as follows:

2,500

pounds of tomatoes

115

gallons of vinegar

13

gallons of corn syrup

53

pounds of salt

a. Determine the standard unit materials cost per pound for a standard batch.

b. Determine the direct materials quantity variance for batch W196.

blue ridge bicycle company manufactures mountain bikes the following data for may of 666027

Blue Ridge Bicycle Company manufactures mountain bikes. The following data for May of the current year are available:

Quantity of direct labor used

1,400 hours

Actual rate for direct labor

$16.15 per hour

Bicycles completed in May

280

Standard direct labor per bicycle

5.30 hours

Standard rate for direct labor

$15.25 per hour

Planned bicycles for May

210

a. Determine the direct labor rate and time variances.

b. How much direct labor should be debited to Work in Process?

midlands hospital began using standards to evaluate its admissions department 666029

Midlands Hospital began using standards to evaluate its Admissions Department. The standard was broken into two types of admissions as follows:

 

Standard Time to Complete

Type of Admission

Admission Record

Unscheduled admission

60 minutes

Scheduled admission

40 minutes

The unscheduled admission took longer, since name, address, and insurance information needed to be determined at the time of admission. Information was collected on scheduled admissions prior to the admissions, which was less time consuming.

The Admissions Department employs three full time people (40 productive hours per week, with no overtime) at $21 per hour. For the most recent week, the department handled 48 unscheduled and 150 scheduled admissions.

a. How much was actually spent on labor for the week?

b. What are the standard hours for the actual volume for the week? Round to one decimal place.

c. Calculate a time variance, and report how well the department performed for the week.

one of the operations in the u s post office is a mechanical mail sorting operation 666030

One of the operations in the U.S. Post Office is a mechanical mail sorting operation. In this operation, letter mail is sorted at a rate of one letter per second. The letter is mechanically sorted from a three digit code input by an operator sitting at a keyboard. The manager of the mechanical sorting operation wishes to determine the number of temporary employees to hire for December. The manager estimates that there will be an additional 32,400,000 pieces of mail in December, due to the upcoming holiday season. Assume that the sorting operators are temporary employees. The union contract requires that temporary employees be hired for one month at a time. Each temporary employee is hired to work 150 hours in the month.

a. How many temporary employees should the manager hire for December?

b. If each employee earns a standard $16 per hour, what would be the labor time variance if the actual number of letters sorted in December was 32,814,000?

pine knoll wood products company prepared the following factory overhead cost budget 666032

Pine Knoll Wood Products Company prepared the following factory overhead cost budget for the Press Department for February 2008, during which it expected to require 10,000 hours of productive capacity in the department:

Variable overhead cost:

 

 

Indirect factory labor

$28,000

 

Power and light

4,500

 

Indirect materials

22,000

 

Total variable cost

 

$ 54,500

Fixed overhead cost:

 

 

Supervisory salaries

$36,000

 

Depreciation of plant and equipment

30,000

 

Insurance and property taxes

8,000

 

Total fixed cost

 

74,000

Total factory overhead cost

 

$128,500

Assuming that the estimated costs for March are the same as for February, prepare a flexible factory overhead cost budget for the Press Department for March for 8,000, 10,000, and 12,000 hours of production.

the following data relate to factory overhead cost for the production of 25 000 comp 666034

The following data relate to factory overhead cost for the production of 25,000 computers:

Actual:

Variable factory overhead

$650,000

 

Fixed factory overhead

78,000

Standard:

32,000 hours at $21

672,000

If productive capacity of 100% was 50,000 hours and the factory overhead cost budgeted at the level of 32,000 standard hours was $700,080, determine the variable factory overhead controllable variance, fixed factory overhead volume variance, and total factory overhead cost variance. The fixed factory overhead rate was $1.56 per hour.

banner textiles corporation began january with a budget for 28 000 hours of producti 666035

Banner Textiles Corporation began January with a budget for 28,000 hours of production in the Weaving Department. The department has a full capacity of 36,000 hours under normal business conditions. The budgeted overhead at the planned volumes at the beginning of January was as follows:

Variable overhead

$ 78,400

Fixed overhead

54,000

Total

$132,400

The actual factory overhead was $135,250 for January. The actual fixed factory overhead was as budgeted. During January, the Weaving Department had standard hours at actual production volume of 30,750 hours.

a. Determine the variable factory overhead controllable variance.

b. Determine the fixed factory overhead volume variance.

the data related to osage sporting goods company s factory overhead cost for the pro 666036

The data related to Osage Sporting Goods Company’s factory overhead cost for the production of 60,000 units of product are as follows:

Actual:

Variable factory overhead

$274,500

 

Fixed factory overhead

224,000

Standard:

76,000 hours at $6.30 ($3.50 for variable factory overhead)

478,800

Productive capacity at 100% of normal was 80,000 hours, and the factory overhead cost budgeted at the level of 76,000 standard hours was $490,000. Based upon these data, the chief cost accountant prepared the following variance analysis:

Variable factory overhead controllable variance:

 

 

Actual variable factory overhead cost incurred

$274,500

 

Budgeted variable factory overhead for 76,000 hours

266,000

 

Variance—unfavorable

 

$ 8,500

Fixed factory overhead volume variance:

 

 

Normal productive capacity at 100%

80,000 hours

 

Standard for amount produced

76,000

 

Productive capacity not used

4,000 hours

 

Standard variable factory overhead rate

× $6.30

25,200

Variance—unfavorable

 

 

Total factory overhead cost variance—unfavorable

 

$33,700

Identify the errors in the factory overhead cost variance analysis.

the following data were taken from the records of nomad company for march 2008 666040

The following data were taken from the records of Nomad Company for March 2008:

Administrative expenses

$ 58,000

Cost of goods sold (at standard)

885,500

Direct materials price variance—favorable

1,800

Direct materials quantity variance—unfavorable

2,250

Direct labor rate variance—favorable

900

Direct labor time variance—unfavorable

3,950

Variable factory overhead controllable variance—favorable

4,625

Fixed factory overhead volume variance—unfavorable

11,000

Interest expense

1,800

Sales

1,150,000

Selling expenses

119,325

Prepare an income statement for presentation to management.

dresses by melissa inc manufactures dresses in a small manufacturing facility manufa 666042

Dresses by Melissa Inc. manufactures dresses in a small manufacturing facility. Manufacturing has 15 employees. Each employee presently provides 32 hours of productive labor per week. Information about a production week is as follows:

Standard wage per hour

$10.40

Standard labor time per dress

15 minutes

Standard number of yards of fabric per dress

4.2 yards

Standard price per yard of fabric

$2.65

Actual price per yard of fabric

$2.70

Actual yards of fabric used during the week

7,200 yards

Number of dresses produced during the week

1,900

Actual wage per hour

$10.50

Actual hours per week

480 hours

Instructions

Determine (a) the standard cost per dress for direct materials and direct labor; (b) the price variance, quantity variance, and total direct materials cost variance; and (c) the rate variance, time variance, and total direct labor cost variance.

gulf coast resins company processes a base chemical into plastic standard costs and 666044

Gulf Coast Resins Company processes a base chemical into plastic. Standard costs and actual costs for direct materials, direct labor, and factory overhead incurred for the manufacture of 2,600 units of product were as follows:

 

Standard Costs

Actual Costs

Direct materials

6,850 pounds at $6.40

7,000 pounds at $6.35

Direct labor

2,050 hours at $19.80

2,100 hours at $20.20

Factory overhead

Rates per direct labor hour,

 

 

based on 100% of normal

 

 

capacity of 1,950 direct

 

 

labor hours:

 

 

Variable cost, $2.50

$4,970 variable cost

 

Fixed cost, $4.50

$8,775 fixed cost

Each unit requires 0.6 hour of direct labor.

Instructions

Determine (a) the price variance, quantity variance, and total direct materials cost variance; (b) the rate variance, time variance, and total direct labor cost variance; and (c) variable factory overhead controllable variance, the fixed factory overhead volume variance, and total factory overhead cost variance.

power equipment inc a manufacturer of construction equipment prepared the following 666045

Power Equipment Inc., a manufacturer of construction equipment, prepared the following factory overhead cost budget for the Welding Department for July 2008. The company expected to operate the department at 100% of normal capacity of 4,800 hours.

Variable costs:

 

 

Indirect factory wages

$14,160

 

Power and light

7,680

 

Indirect materials

8,880

 

Total variable cost

 

$30,720

Fixed costs:

 

 

Supervisory salaries

$16,000

 

Depreciation of plant and equipment

43,500

 

Insurance and property taxes

6,740

 

Total fixed cost

 

66,240

Total factory overhead cost

 

$96,960

During July, the department operated at 5,000 standard hours, and the factory overhead costs incurred were indirect factory wages, $14,000; power and light, $9,250; indirect materials, $8,450; supervisory salaries, $16,000; depreciation of plant and equipment, $43,500; and insurance and property taxes, $6,740.

Instructions

Prepare a factory overhead cost variance report for July. To be useful for cost control, the budgeted amounts should be based on 5,000 hours.

elite technologies inc does software development one important activity in software 666046

Elite Technologies Inc. does software development. One important activity in software development is writing software code. The manager of the WritePro Development Team determined that the average software programmer could write 45 lines of code in an hour. The plan for the first week in May called for 6,840 lines of code to be written on the WritePro product. The WritePro Team has four programmers. Each programmer is hired from an employment firm that requires temporary employees to be hired for a minimum of a 40 hour week. Programmers are paid $32.00 per hour. The manager offered a bonus if the team could generate more than 7,200 lines for the week, without overtime. Due to a project emergency, the programmers wrote more code in the first week of May than planned. The actual amount of code written in the first week of May was 7,650 lines, without overtime. As a result, the bonus caused the average programmer’s hourly rate to increase to $36.00 per hour during the first week in May.

Instructions

1. If the team generated 6,840 lines of code according to the original plan, what would have been the labor time variance?

2. What was the actual labor time variance as a result of generating 7,650 lines of code?

3. What was the labor rate variance as a result of the bonus?

4. The manager is trying to determine if a better decision would have been to hire a temporary programmer to meet the higher programming demand in the first week of May, rather than paying out the bonus. If another employee was hired from the employment firm, what would have been the labor time variance in the first week?

5. Which decision is better, paying the bonus or hiring another programmer?

6. Are there any performance related issues that the labor time and rate variances fail to consider? Explain.

athome fixtures company manufactures faucets in a small manufacturing facility 666047

AtHome Fixtures Company manufactures faucets in a small manufacturing facility. The faucets are made from zinc. Manufacturing has 80 employees. Each employee presently provides 36 hours of labor per week. Information about a production week is as follows:

Standard wage per hour

$14.00

Standard labor time per faucet

15 minutes

Standard number of pounds of zinc

1.8 lbs.

Standard price per pound of zinc

$9.50

Actual price per pound of zinc

$9.20

Actual pounds of zinc used during the week

21,900 lbs.

Number of faucets produced during the week

12,000

Actual wage per hour

$14.50

Actual hours per week

2,880 hours

Instructions

Determine (a) the standard cost per unit for direct materials and direct labor; (b) the price variance, quantity variance, and total direct materials cost variance; and (c) the rate variance, time variance, and total direct labor cost variance.

arctic coat company makes women s and men s coats both products require filler and l 666048

Arctic Coat Company makes women’s and men’s coats. Both products require filler and lining material. The following planning information has been made available:

 

Standard Quantity

 

 

Women’s Coats

Men’s Coats

Standard Price per Unit

Filler

2.5 lbs.

4.0 lbs.

$29.00

Liner

6.0 yds.

8.5 yds.

5.00

Standard labor time

0.40 hr.

0.60 hr.

 

Planned production

2,500 units

4,000 units

 

Standard labor rate

$12.80 per hour

$15.50 per hour

 

Arctic Coat does not expect there to be any beginning or ending inventories of filler and lining material. At the end of the budget year, Arctic Coat experienced the following actual results:

 

Women’s Coats

Men’s Coats

Actual production

2,300

3,500

 

Actual Price per Unit

Actual Quantity Purchased and Used

Filler

$27.50

20,400

Liner

5.50

43,200

 

Actual Labor Rate

Actual Labor Hours Used

Woman’s Coat

$12.00

1,050

Man’s Coat

15.80

1,980

The expected beginning inventory and desired ending inventory were realized.

Instructions

1. Prepare the following variance analyses, based on the actual results and production levels at the end of the budget year:

a. Direct materials price, quantity, and total variance.

b. Direct labor rate, time, and total variance.

2. Why are the standard amounts in part (1) based on the actual production at the end of the year instead of the planned production at the beginning of the year?

journalize the closing entries on march 31 665801

Greenway Company is a small rug retailer owned and operated by Lorene Greenway. After the accounts have been adjusted on March 31, the following account balances were taken from the ledger:

Advertising Expense

$25,800

Depreciation Expense

5,100

Lorene Greenway, Drawing

50,000

Merchandise Inventory, March 1

34,500

Merchandise Inventory, March 31

42,150

Miscellaneous Expense

6,350

Purchases

480,000

Purchases Discounts

2,000

Purchases Returns and Allowances

9,000

Sales

925,000

Sales Discounts

4,000

Sales Returns and Allowances

8,000

Salaries Expense

76,300

Transportation In

15,400

Journalize the closing entries on March 31.

the following selected transactions were completed by cardroom supply co which sells 665804

The following selected transactions were completed by Cardroom Supply Co., which sells office supplies primarily to wholesalers and occasionally to retail customers.

Jan.

2

Sold merchandise on account to Kibler Co., $10,000, terms FOB destination,

   

1/10, n/30. The cost of the merchandise sold was $6,500.

 

3

Sold merchandise for $12,000 plus 8% sales tax to cash customers. The cost of

   

merchandise sold was $9,000.

 

4

Sold merchandise on account to Glickman Co., $5,600, terms FOB shipping

   

point, n/eom. The cost of merchandise sold was $3,100.

 

5

Sold merchandise for $8,000 plus 8% sales tax to customers who used Master

   

Card. The cost of merchandise sold was $6,000.

 

12

Received check for amount due from Kibler Co. for sale on January 2.

 

14

Sold merchandise to customers who used American Express cards, $15,000. The

   

cost of merchandise sold was $9,200.

 

16

Sold merchandise on account to Bryan Co., $12,000, terms FOB shipping point,

   

1/10, n/30. The cost of merchandise sold was $7,200.

 

18

Issued credit memorandum for $3,000 to Bryan Co. for merchandise returned

   

from sale on January 16. The cost of the merchandise returned was $1,800.

 

19

Sold merchandise on account to Cooney Co., $15,750, terms FOB shipping

   

point, 2/10, n/30. Added $400 to the invoice for transportation costs prepaid.

   

The cost of merchandise sold was $9,500.

 

26

Received check for amount due from Bryan Co. for sale on January 16 less

   

credit memorandum of January 18 and discount.

 

28

Received check for amount due from Cooney Co. for sale of January 19.

 

31

Received check for amount due from Glickman Co. for sale of January 4.

 

31

Paid Speedy Delivery Service $1,875 for merchandise delivered during January

   

to customers under shipping terms of FOB destination.

Feb

3

Paid First State Bank $1,150 for service fees for handling MasterCard and

   

American Express sales during January.

 

15

Paid $1,600 to state sales tax division for taxes owed on sales.

Instructions

Journalize the entries to record the transactions of Cardroom Supply Co.

journalize the entries to record the transactions of scat trak company for july 665805

The following selected transactions were completed by Scat Trak Company during July of the current year:

July

1

Purchased merchandise from Kermit Co., $18,750, terms FOB destination, n/30.

 

3

Purchased merchandise from Basaway Co., $12,150, terms FOB shipping point,

   

2/10, n/eom. Prepaid transportation costs of $180 were added to the invoice.

 

4

Purchased merchandise from Phillips Co., $13,800, terms FOB destination, 2/10, n/30.

 

6

Issued debit memorandum to Phillips Co. for $1,900 of merchandise returned

   

from purchase on July 4.

 

13

Paid Basaway Co. for invoice of July 3, less discount.

 

14

Paid Phillips Co. for invoice of July 4, less debit memorandum of July 6 and discount.

 

19

Purchased merchandise from Cleghorne Co., $18,000, terms FOB shipping point, n/eom.

 

19

Paid transportation charges of $500 on July 19 purchase from Cleghorne Co.

 

20

Purchased merchandise from Graham Co., $9,000, terms FOB destination, 1/10, n/30.

 

30

Paid Graham Co. for invoice of July 20, less discount.

 

31

Paid Kermit Co. for invoice of July 1.

 

31

Paid Cleghorne Co. for invoice of July 19.

Instructions

Journalize the entries to record the transactions of Scat Trak Company for July.

the following were selected from among the transactions completed by southmont compa 665806

The following were selected from among the transactions completed by Southmont Company during April of the current year:

1

Apr

3

Purchased merchandise on account from Mandell Co., list price $30,000, trade

   

discount 40%, terms FOB destination, 2/10, n/30.

 

4

Sold merchandise for cash, $12,800. The cost of the merchandise sold was $7,600.

 

5

Purchased merchandise on account from Quinn Co., $18,750, terms FOB shipping

   

point, 2/10, n/30, with prepaid transportation costs of $715 added to the invoice.

 

6

Returned $3,500 of merchandise purchased on April 3 from Mandell Co.

 

11

Sold merchandise on account to Campo Co., list price $6,000, trade discount

   

20%, terms 1/10, n/30. The cost of the merchandise sold was $3,200.

 

13

Paid Mandell Co. on account for purchase of April 3, less return of April 6 and discount.

 

14

Sold merchandise on VISA, $52,700. The cost of the merchandise sold was $31,500

 

15

Paid Quinn Co. on account for purchase of April 5, less discount.

 

21

Received cash on account from sale of April 11 to Campo Co., less discount.

 

24

Sold merchandise on account to Elkins Co., $8,150, terms 1/10, n/30. The cost

   

of the merchandise sold was $4,500.

 

28

Paid VISA service fee of $1,500.

 

30

Received merchandise returned by Elkins Co. from sale on April 24, $1,200. The

   

cost of the returned merchandise was $900.

Instructions

Journalize the transactions.

the following selected transactions were completed during august between sellars com 665807

The following selected transactions were completed during August between Sellars Company and Beyer Co.:

Aug

1

Sellars Company sold merchandise on account to Beyer Co., $17,850, terms FOB

   

destination, 2/15, n/eom. The cost of the merchandise sold was $10,700.

 

2

Sellars Company paid transportation costs of $140 for delivery of merchandise

   

sold to Beyer Co. on August 1.

 

5

Sellars Company sold merchandise on account to Beyer Co., $27,550, terms FOB

   

shipping point, n/eom. The cost of the merchandise sold was $16,500.

 

6

Beyer Co. returned $1,800 of merchandise purchased on account on August 1

   

from Sellars Company. The cost of the merchandise returned was $1,050.

 

9

Beyer Co. paid transportation charges of $165 on August 5 purchase from Sellars Company.

     
 

15

Sellars Company sold merchandise on account to Beyer Co., $32,000, terms FOB

   

shipping point, 1/10, n/30. Sellars Company paid transportation costs of

   

$1,243, which were added to the invoice. The cost of the merchandise sold was $19,200.00

 

16

Beyer Co. paid Sellars Company for purchase of August 1, less discount and

   

less return of August 6.

 

25

Beyer Co. paid Sellars Company on account for purchase of August 15, less discount.

 

31

Beyer Co. paid Sellars Company on account for purchase of August 5.

Instructions

Journalize the August transactions for (1) Sellars Company and (2) Beyer Co.

the following selected accounts and their current balances appear in the ledger of h 665808

The following selected accounts and their current balances appear in the ledger of Hobbs’ Co. for the fiscal year ended June 30, 2008:

Cash

$ 68,850

Sales Returns and Allowances

$ 18,900

Accounts Receivable

55,800

Sales Discounts

9,900

Merchandise Inventory

90,000

Cost of Merchandise Sold

963,000

Office Supplies

2,340

Sales Salaries Expense

189,000

Prepaid Insurance

6,120

Advertising Expense

25,470

Office Equipment

57,600

Depreciation Expense—

 

Accumulated Depreciation—

 

Store Equipment

4,140

Office Equipment

9,720

Miscellaneous Selling Expense

990

Store Equipment

105,750

Office Salaries Expense

36,900

Accumulated Depreciation—

 

Rent Expense

19,935

Store Equipment

43,740

Insurance Expense

11,475

Accounts Payable

24,300

Depreciation Expense—

 

Salaries Payable

1,800

Office Equipment

8,100

Note Payable

 

Office Supplies Expense

810

(final payment due 2018)

27,000

Miscellaneous Administrative

 

Jeremiah Hobbs, Capital

241,200

Expense

1,080

Jeremiah Hobbs, Drawing

22,500

Interest Expense

900

Sales

1,351,800

 

 

Instructions

1. Prepare a multiple step income statement.

2. Prepare a statement of owner’s equity.

3. Prepare a report form of balance sheet, assuming that the current portion of the note payable is $2,250.

4. Briefly explain (a) how multiple step and single step income statements differ and (b) how report form and account form balance sheets differ.

journalize the entries to record the transactions of water tech supplies co 665810

The following selected transactions were completed by Water Tech Supplies Co., which sells irrigation supplies primarily to wholesalers and occasionally to retail customers.

July

1

Sold merchandise on account to Upshaw Co., $8,000, terms FOB shipping point,

   

n/eom. The cost of merchandise sold was $4,800.

 

2

Sold merchandise for $15,000 plus 7% sales tax to cash customers. The cost of

   

merchandise sold was $8,800.

 

5

Sold merchandise on account to Westone Company, $16,000, terms FOB destination,

   

1/10, n/30. The cost of merchandise sold was $10,500.

 

8

Sold merchandise for $11,500 plus 7% sales tax to customers who used VISA

   

cards. The cost of merchandise sold was $7,000.

 

13

Sold merchandise to customers who used MasterCard cards, $8,000. The cost of

   

merchandise sold was $4,750.

 

14

Sold merchandise on account to Tyler Co., $7,500, terms FOB shipping point,

   

1/10, n/30. The cost of merchandise sold was $4,000.

 

15

Received check for amount due from Westone Company for sale on July 5.

 

16

Issued credit memorandum for $800 to Tyler Co. for merchandise returned

   

from sale on July 14. The cost of the merchandise returned was $360.

 

18

Sold merchandise on account to Horton Company, $6,850, terms FOB shipping

   

point, 2/10, n/30. Paid $210 for transportation costs and added them to the

   

invoice. The cost of merchandise sold was $4,100.

 

24

Received check for amount due from Tyler Co. for sale on July 14 less credit

   

memorandum of July 16 and discount.

 

28

Received check for amount due from Horton Company for sale of July 18.

 

31

Paid Uptown Delivery Service $3,100 for merchandise delivered during July to

   

customers under shipping terms of FOB destination.

 

31

Received check for amount due from Upshaw Co. for sale of July 1.

Aug.

3

Paid First National Bank $780 for service fees for handling MasterCard and

   

VISA sales during July.

 

10

Paid $1,855 to state sales tax division for taxes owed on sales.

Instructions

Journalize the entries to record the transactions of Water Tech Supplies Co.

the following selected transactions were completed by bodyworks co during october of 665811

The following selected transactions were completed by Bodyworks Co. during October of the current year.

Oct.

1

Purchased merchandise from Mantooth Co., $11,800, terms FOB shipping point,

   

2/10, n/eom. Prepaid transportation costs of $325 were added to the invoice.

 

5

Purchased merchandise from Hauck Co., $17,500, terms FOB destination, n/30.

 

10

Paid Mantooth Co. for invoice of October 1, less discount.

 

13

Purchased merchandise from Lieu Co., $7,500, terms FOB destination, 1/10, n/30.

 

14

Issued debit memorandum to Lieu Co. for $2,500 of merchandise returned from

   

purchase on October 13.

 

18

Purchased merchandise from Fowler Company, $9,600, terms FOB shipping point, n/eom.

 

18

Paid transportation charges of $150 on October 18 purchase from Fowler Company.

 

19

Purchased merchandise from Hatcher Co., $9,750, terms FOB destination, 2/10, n/30.

 

23

Paid Lieu Co. for invoice of October 13, less debit memorandum of October

   

14 and discount.

 

29

Paid Hatcher Co. for invoice of October 19, less discount.

 

31

Paid Fowler Company for invoice of October 18.

 

31

Paid Hauck Co. for invoice of October 5.

Instructions

Journalize the entries to record the transactions of Bodyworks Co. for October.

the following were selected from among the transactions completed by theisen company 665812

The following were selected from among the transactions completed by Theisen Company during December of the current year:

Dec.

3

Purchased merchandise on account from Shipley Co., list price $24,000, trade

   

discount 25%, terms FOB shipping point, 2/10, n/30, with prepaid transportation

   

costs of $615 added to the invoice.

 

5

Purchased merchandise on account from Kirch Co., $10,250, terms FOB destination,

   

2/10, n/30.

 

6

Sold merchandise on account to Murdock Co., list price $18,000, trade discount

   

35%, terms 2/10, n/30. The cost of the merchandise sold was $8,250.

 

7

Returned $1,800 of merchandise purchased on December 5 from Kirch Co.

 

13

Paid Shipley Co. on account for purchase of December 3, less discount.

 

15

Paid Kirch Co. on account for purchase of December 5, less return of December

   

7 and discount.

 

16

Received cash on account from sale of December 6 to Murdock Co., less discount.

 

19

Sold merchandise on MasterCard, $39,500. The cost of the merchandise sold

   

was $23,700.

 

22

Sold merchandise on account to Milk River Co., $11,300, terms 2/10, n/30. The

   

cost of the merchandise sold was $6,700.

 

23

Sold merchandise for cash, $17,680. The cost of the merchandise sold was $9,100.00

 

28

Received merchandise returned by Milk River Co. from sale on December 22,

   

$2,000. The cost of the returned merchandise was $1,100.

 

31

Paid MasterCard service fee of $1,050.

Instructions

Journalize the transactions.

journalize the november transactions for 1 sallis company and 2 byce company 665813

The following selected transactions were completed during November between Sallis Company and Byce Company:

Nov.

2

Sallis Company sold merchandise on account to Byce Company, $12,500, terms

   

FOB shipping point, 2/10, n/30. Sallis Company paid transportation costs of

   

$425, which were added to the invoice. The cost of the merchandise sold was

   

$7,500.00

 

8

Sallis Company sold merchandise on account to Byce Company, $21,600, terms

   

FOB destination, 1/15, n/eom. The cost of the merchandise sold was $13,000.

 

8

Sallis Company paid transportation costs of $879 for delivery of merchandise

   

sold to Byce Company on November 8.

 

12

Byce Company returned $5,000 of merchandise purchased on account on

   

November 8 from Sallis Company. The cost of the merchandise returned was

   

$2,900.00

 

12

Byce Company paid Sallis Company for purchase of November 2, less discount.

 

23

Byce Company paid Sallis Company for purchase of November 8, less discount

   

and less return of November 12.

 

24

Sallis Company sold merchandise on account to Byce Company, $15,000,

   

terms FOB shipping point, n/eom. The cost of the merchandise sold was

   

$9,000.00

 

26

Byce Company paid transportation charges of $400 on November 24 purchase

   

from Sallis Company.

 

30

Byce Company paid Sallis Company on account for purchase of November 24.

Instructions

Journalize the November transactions for (1) Sallis Company and (2) Byce Company.

on april 30 2008 the balances of the accounts appearing in the ledger of headwinds c 665814

On April 30, 2008, the balances of the accounts appearing in the ledger of Headwinds Company are as follows:

Cash

$ 18,300

Sales Discounts

$ 9,375

Accounts Receivable

72,125

Purchases

536,500

Merchandise Inventory, May 1, 2007

87,725

Purchases Returns and Allowances

6,000

Office Supplies

3,025

Purchases Discounts

4,500

Prepaid Insurance

4,500

Transportation In

10,900

Land

35,000

Sales Salaries Expense

156,250

Store Equipment

170,775

Advertising Expense

55,000

Accumulated Depreciation—

 

Delivery Expense

9,000

Store Equipment

55,900

Depreciation Expense—

 

Office Equipment

78,500

Store Equipment

5,900

Accumulated Depreciation—

 

Miscellaneous Selling Expense

10,700

Office Equipment

16,250

Office Salaries Expense

100,000

Accounts Payable

27,825

Rent Expense

31,250

Salaries Payable

2,950

Insurance Expense

3,000

Unearned Rent

8,300

Office Supplies Expense

2,300

Notes Payable

12,500

Depreciation Expense—

 

Kasey Kurtz, Capital

190,050

Office Equipment

1,500

Kasey Kurtz, Drawing

18,750

Miscellaneous Administrative Expense

5,850

Sales

1,106,450

Rent Revenue

6,250

Sales Returns and Allowances

10,000

Interest Expense

750

Instructions

1. Does Headwinds Company use a periodic or perpetual inventory system? Explain.

2. Prepare a multiple step income statement for Headwinds Company for the year ended April 30, 2008. The merchandise inventory as of April 30, 2008, was $94,100.

3. Prepare the closing entries for Headwinds Company as of April 30, 2008.

world boards co is a merchandising business the account balances for world boards co 665815

World Boards Co. is a merchandising business. The account balances for World Boards Co. as of March 1, 2008 (unless otherwise indicated), are as follows:

110

Cash

$ 21,200

112

Accounts Receivable

51,300

115

Merchandise Inventory

200,800

116

Prepaid Insurance

5,600

117

Store Supplies

3,800

123

Store Equipment

156,500

124

Accumulated Depreciation—Store Equipment

18,900

210

Accounts Payable

32,200

211

Salaries Payable

310

Evan Raskind, Capital, April 1, 2007

185,100

311

Evan Raskind, Drawing

45,000

312

Income Summary

410

Sales

1,073,700

411

Sales Returns and Allowances

30,900

412

Sales Discounts

19,800

510

Cost of Merchandise Sold

541,000

520

Sales Salaries Expense

111,600

521

Advertising Expense

27,000

522

Depreciation Expense

523

Store Supplies Expense

529

Miscellaneous Selling Expense

4,200

530

Office Salaries Expense

60,700

531

Rent Expense

27,900

532

Insurance Expense

539

Miscellaneous Administrative Expense

2,600

During March, the last month of the fiscal year, the following transactions were completed:

Mar.

1

Paid rent for March, $2,400.

 

3

Purchased merchandise on account from Huisman Co., terms 2/10, n/30, FOB

   

shipping point, $21,600.

 

4

Paid transportation charges on purchase of March 3, $500.

 

6

Sold merchandise on account to Hillcrest Co., terms 2/10, n/30, FOB shipping

   

point, $8,500. The cost of the merchandise sold was $5,000.

 

7

Received $8,900 cash from Foley Co. on account, no discount.

 

10

Sold merchandise for cash, $27,200. The cost of the merchandise sold was $16,000.00

 

13

Paid for merchandise purchased on March 3, less discount.

 

14

Received merchandise returned on sale of March 6, $1,500. The cost of the merchandise

   

returned was $900.

 

15

Paid advertising expense for last half of March, $2,600.

 

16

Received cash from sale of March 6, less return of March 14 and discount.

 

19

Purchased merchandise for cash, $11,800.

 

19

Paid $9,000 to Bakke Co. on account, no discount.

 

20

Sold merchandise on account to Wilts Co., terms 1/10, n/30, FOB shipping

   

point, $22,300. The cost of the merchandise sold was $13,200.

 

21

For the convenience of the customer, paid shipping charges on sale of March 20, $1,100.00

 

21

Received $17,600 cash from Owen Co. on account, no discount.

 

21

Purchased merchandise on account from Nye Co., terms 1/10, n/30, FOB destination,

   

$19,900.00

 

24

Returned $2,000 of damaged merchandise purchased on March 21, receiving

   

credit from the seller.

 

26

Refunded cash on sales made for cash, $1,200. The cost of the merchandise

   

returned was $700.

 

28

Paid sales salaries of $7,600 and office salaries of $4,800.

 

29

Purchased store supplies for cash, $800.

 

30

Sold merchandise on account to Whitetail Co., terms 2/10, n/30, FOB shipping

   

point, $18,750. The cost of the merchandise sold was $11,250.

 

30

Received cash from sale of March 20, less discount, plus transportation paid on March 21.

 

31

Paid for purchase of March 21, less return of March 24 and discount.

Instructions

1. Enter the balances of each of the accounts in the appropriate balance column of a four column account. Write Balance in the item section, and place a check mark (??) in the Posting Reference column. Journalize the transactions for March.

2. Post the journal to the general ledger, extending the month-end balances to the appropriate balance columns after all posting is completed. In this problem, you are not required to update or post to the accounts receivable and accounts payable subsidiary ledgers.

3. Prepare an unadjusted trial balance.

4. At the end of March, the following adjustment data were assembled. Analyze and use these data to complete (5) and (6).

a.

Merchandise inventory on March 31

 

$196,139

b.

Insurance expired during the year

 

1,875

c.

Store supplies on hand on March 31

 

1,500

d.

Depreciation for the current year

 

9,500

e.

Accrued salaries on March 31:

 

 

 

Sales salaries

$1,200

 

 

Office salaries

800

2,000

5. Optional: Enter the unadjusted trial balance on a 10-column end-of-period spreadsheet (work sheet), and complete the spreadsheet. See Appendix C for how to prepare an end-of-period spreadsheet (work sheet) for a merchandising business.

6. Journalize and post the adjusting entries.

7. Prepare an adjusted trial balance.

8. Prepare an income statement, a statement of owner’s equity, and a balance sheet.

9. Prepare and post the closing entries. Indicate closed accounts by inserting a line in both the Balance columns opposite the closing entry. Insert the new balance in the owner’s capital account.

10. Prepare a post-closing trial balance.

the eclipse video store co is owned and operated by jared helms the following is an 665816

The Eclipse Video Store Co. is owned and operated by Jared Helms. The following is an excerpt from a conversation between Jared Helms and Allison Fain, the chief accountant for The Eclipse Video Store.

Jared: Allison, I’ve got a question about this recent balance sheet.

Allison: Sure, what’s your question?

Jared: Well, as you know, I’m applying for a bank loan to finance our new store in Winterville, and I noticed that the accounts payable are listed as $85,000.

Allison: That’s right. Approximately $78,000 of that represents amounts due our suppliers, and the remainder is miscellaneous payables to creditors for utilities, office equipment, supplies, etc.

Jared: That’s what I thought. But as you know, we normally receive a 2% discount from our suppliers for earlier payment, and we always try to take the discount.

Allison: That’s right. I can’t remember the last time we missed a discount.

Jared: Well, in that case, it seems to me the accounts payable should be listed minus the 2% discount. Let’s list the accounts payable due suppliers as $76,440, rather than $78,000. Every little bit helps. You never know. It might make the difference between getting the loan and not. How would you respond to Jared Helms’ request?

what might be some considerations other than price that might influence kate rsquo s 665817

The following is an excerpt from a conversation between Kate Fleming and Bob Dent. Kate is debating whether to buy a stereo system from Design Sound, a locally owned electronics store, or Big Sound Electronics, an online electronics company.

Kate: Bob, I don’t know what to do about buying my new stereo.

Bob: What’s the problem?

Kate: Well, I can buy it locally at Design Sound for $580.00. However, Big Sound Electronics has the same system listed for $599.99.

Bob: So what’s the big deal? Buy it from Design Sound.

Kate: It’s not quite that simple. Big Sound said something about not having to pay sales tax, since I was out of state.

Bob: Yes, that’s a good point. If you buy it at Design Sound, they’ll charge you 8% sales tax.

Kate: But Big Sound Electronics charges $18.99 for shipping and handling. If I have them send it next day air, it’ll cost $24.99 for shipping and handling.

Bob: I guess it is a little confusing.

Kate: That’s not all. Design Sound will give an additional 1% discount if I pay cash.

Otherwise, they will let me use my VISA, or I can pay it off in three monthly installments.

Bob: Anything else???

Kate: Well . . . Big Sound says I have to charge it on my VISA. They don’t accept checks.

Bob: I am not surprised. Many online stores don’t accept checks.

Kate: I give up. What would you do?

1. Assuming that Big Sound Electronics doesn’t charge sales tax on the sale to Kate, which company is offering the best buy?

2. What might be some considerations other than price that might influence Kate’s decision on where to buy the stereo system?

prepare a projected single step income statement for the year ending july 31 2009 ba 665818

Your sister operates Emigrant Parts Company, an online boat parts distributorship that is in its third year of operation. The income statement is shown at the top of the following page and was recently prepared for the year ended July 31, 2008.

Your sister is considering a proposal to increase net income by offering sales discounts of 2/15, n/30, and by shipping all merchandise FOB shipping point. Currently, no sales discounts are allowed and merchandise is shipped FOB destination. It is estimated that these credit terms will increase net sales by 15%. The ratio of the cost of merchandise sold to net sales is expected to be 65%. All selling and administrative expenses are expected to remain unchanged, except for store supplies, miscellaneous selling, office supplies, and miscellaneous administrative expenses, which are expected to increase proportionately with

Emigrant Parts Company Income Statement For the Year Ended July 31, 2008

Revenues:

   

Net sales

 

$800,000

Interest revenue

 

10,000

Total revenues

 

$810,000

Expenses:

   

Cost of merchandise sold

520,000

 

Selling expenses

90,000

 

Administrative expenses

48,550

 

Interest expense

15,000

 

Total expenses

 

673,550

Net income

 

$136,450

increased net sales. The amounts of these preceding items for the year ended July 31, 2008, were as follows:

Store supplies expense

$12,000

Miscellaneous selling expense

3,000

Office supplies expense

2,000

Miscellaneous administrative expense

1,000

The other income and other expense items will remain unchanged. The shipment of all merchandise FOB shipping point will eliminate all delivery expense, which for the year ended July 31, 2008, were $18,750.

1. Prepare a projected single step income statement for the year ending July 31, 2009, based on the proposal. Assume all sales are collected within the discount period.

2. a. Based on the projected income statement in (1), would you recommend the implementation of the proposed changes?

b. Describe any possible concerns you may have related to the proposed changes described in (1).

the following are inputs and outputs to the baggage claim process of an airline 666007

The following are inputs and outputs to the baggage claim process of an airline:

Baggage handler training

Time customers wait for returned baggage

Maintenance of baggage handling equipment

Number of baggage handlers

Number of damaged bags

On time flight performance

Identify whether each is an input or output to the baggage claim process.

hawley inc manufactures woven baskets for national distribution the standard costs f 666008

Hawley Inc. manufactures woven baskets for national distribution. The standard costs for the manufacture of Folk Art style baskets were as follows:

 

Standard Costs

Actual Costs

Direct materials

1,500 pounds at $35

1,600 pounds at $32

Direct labor

4,800 hours at $11

4,500 hours at $11.80

Factory overhead

Rates per labor hour,

 

 

based on 100% of

 

 

normal capacity of

 

 

5,500 labor hours:

 

 

Variable cost, $2.40

$12,300 variable cost

 

Fixed cost, $3.50

$19,250 fixed cost

Instructions

1. Determine the quantity variance, price variance, and total direct materials cost variance for the Folk Art style baskets.

2. Determine the time variance, rate variance, and total direct labor cost variance for the Folk Art style baskets.

3. Determine the controllable variance, volume variance, and total factory overhead cost variance for the Folk Art style baskets.

the actual and standard factory overhead costs for producing a specified quantity of 666011

The actual and standard factory overhead costs for producing a specified quantity of product are as follows:

Actual: Variable factory overhead

$72,500

 

Fixed factory overhead

40,000

$112,500

Standard: 19,000 hours at $6

 

 

($4 variable and $2 fixed)

 

114,000

If 1,000 hours were unused, the fixed factory overhead volume variance would be:

A. $1,500 favorable.

B. $2,000 unfavorable.

C. $4,000 unfavorable.

D. $6,000 unfavorable.

brewster company produces a product that requires four standard pounds per unit the 666018

Brewster Company produces a product that requires four standard pounds per unit. The standard price is $6.80 per pound. If 1,500 units required 6,400 pounds, which were purchased at $6.50 per pound, what is the direct materials (a) price variance, (b) quantity variance, and (c) cost variance?

Brewster Company produces a product that requires 1.5 standard hours per unit at a standard hourly rate of $15 per hour. If 1,500 units required 2,100 hours at an hourly rate of $15.50 per hour, what is the direct labor (a) rate variance, (b) time variance, and (c) cost variance?

Brewster Company produced 1,500 units of product that required 1.5 standard hours per unit. The standard variable overhead cost per unit is $1.70 per hour. The actual variable factory overhead was $3,900. Determine the variable factory overhead controllable variance.

Brewster Company produced 1,500 units of product that required 1.5 standard hours per unit. The standard fixed overhead cost per unit is $0.50 per hour at 2,500 hours, which is 100% of normal capacity. Determine the fixed factory overhead volume variance.

Prepare an income statement through gross profit for Brewster Company. Assume Brewster sold 1,500 units at $80 per unit.

tipton company produces a product that requires eight standard gallons per unit the 666019

Tipton Company produces a product that requires eight standard gallons per unit. The standard price is $12.40 per gallon. If 800 units required 6,200 gallons, which were purchased at $12.75 per gallon, what is the direct materials (a) price variance, (b) quantity variance, and (c) cost variance?

Tipton Company produces a product that requires four standard hours per unit at a standard hourly rate of $10 per hour. If 800 units required 3,380 hours at an hourly rate of $9.60 per hour, what is the direct labor (a) rate variance, (b) time variance, and (c) cost variance?

Tipton Company produced 800 units of product that required four standard hours per unit. The standard variable overhead cost per unit is $4.50 per hour. The actual variable factory overhead was $14,100. Determine the variable factory overhead controllable variance.

Tipton Company produced 800 units of product that required four standard hours per unit. The standard fixed overhead cost per unit is $1.20 per hour at 3,000 hours, which is 100% of normal capacity. Determine the fixed factory overhead volume variance.

Prepare an income statement through gross profit for Tipton Company. Assume Tipton sold 800 units at $200 per unit.

determine the amount to be paid in full settlement of each of invoices a and b 665751

Determine the amount to be paid in full settlement of each of invoices (a) and (b), assuming that credit for returns and allowances was received prior to payment and that all invoices were paid within the discount period.

 

 

Transportation

 

Returns and

a.

Merchandise

Paid by Seller

Transportation Terms

Allowances

$4,500

$200

FOB shipping point, 1/10, n/30

$ 800

b.

5,000

60

FOB destination, 2/10, n/30

2,500

journalize the adjusting entry for merchandise inventory shrinkage 3 750 665754

The following transactions were completed by Montrose Company during May of the current year. Montrose Company uses a perpetual inventory system.

May

3

Purchased merchandise on account from Floyd Co., $4,000, terms FOB

 

 

Shipping

   

point, 2/10, n/30, with prepaid transportation costs of $120 added to the

   

invoice.

 

5

Purchased merchandise on account from Kramer Co., $8,500, terms FOB

 

 

destination,

   

1/10, n/30.

 

6

Sold merchandise on account to C. F. Howell Co., list price $4,000, trade

 

 

Discount

   

30%, terms 2/10, n/30. The cost of the merchandise sold was $1,125.

 

8

Purchased office supplies for cash, $150.

 

10

Returned merchandise purchased on May 5 from Kramer Co., $1,300.

 

13

Paid Floyd Co. on account for purchase of May 3, less discount.

 

14

Purchased merchandise for cash, $10,500.

 

15

Paid Kramer Co. on account for purchase of May 5, less return of May 10

   

and discount.

 

16

Received cash on account from sale of May 6 to C. F. Howell Co., less

 

 

discount.

 

19

Sold merchandise on account to Comer Co., $3,480, terms 2/10, n/30. The

   

cost of the merchandise sold was $1,400.

 

22

Sold merchandise for cash, $4,350. The cost of the merchandise sold was

 

 

$1,750.

 

25

Received merchandise returned by Comer Co. from sale on May 22, $1,480.

   

The cost of the returned merchandise was $600.

 

31

Paid a service processing fee of $140 for MasterCard sales.

Instructions

1. Journalize the preceding transactions.

2. Journalize the adjusting entry for merchandise inventory shrinkage, $3,750.

determine the amount to be paid in full settlement of each of invoices a and b assum 665763

Determine the amount to be paid in full settlement of each of invoices (a) and (b), assuming that credit for returns and allowances was received prior to payment and that all invoices were paid within the discount period.

 

 

Transportation

 

Returns and

a.

Merchandise

Paid by Seller

Transportation Terms

Allowances

$6,000

$400

FOB shipping point, 1/10, n/30

$1,000

b.

2,500

150

FOB destination, 2/10, n/30

900

determine the amount to be paid in full settlement of each of invoices a and b assum 665764

Determine the amount to be paid in full settlement of each of invoices (a) and (b), assuming that credit for returns and allowances was received prior to payment and that all invoices were paid within the discount period.

 

 

Transportation

 

Returns and

a.

Merchandise

Paid by Seller

Transportation Terms

Allowances

$ 8,150

$200

FOB destination, 2/10, n/30

$1,300

b.

12,750

625

FOB shipping point, 2/10, n/30

3,000

determine the gross profit to be reported on the income statement for the year ended 665771

The following data were extracted from the accounting records of Meniscus Company for the year ended June 30, 2008:

Merchandise inventory, July 1, 2007

$183,250

Merchandise inventory, June 30, 2008

200,100

Purchases

1,279,600

Purchases returns and allowances

41,200

Purchases discounts

20,500

Sales

1,800,000

Transportation in

17,250

a. Prepare the cost of merchandise sold section of the income statement for the year ended June 30, 2008, using the periodic inventory system.

b. Determine the gross profit to be reported on the income statement for the year ended June 30, 2008.

identify the errors in the following schedule of cost of merchandise sold for the cu 665772

Identify the errors in the following schedule of cost of merchandise sold for the current year ended March 31, 2008:

Cost of merchandise sold:

     

Merchandise inventory, March 31, 2008

 

$135,750

Purchases

 

$852,100

 

Plus: Purchases returns and allowances

10,500

   

Purchases discounts

8,000

18,500

 

Gross purchases

 

$870,600

 

Less transportation in

 

7,500

 

Cost of merchandise purchased

 

 

863,100

Merchandise available for sale

 

 

$998,850

Less merchandise inventory, April 1, 2007

 

 

115,150

Cost of merchandise sold

   

$883,700

identify the errors in the following income statement 665775

Identify the errors in the following income statement:

The Euclidian Company Income Statement For the Year Ended March 31, 2008

Revenue from sales:

     

Sales

 

$7,127,500

 

Add: Sales returns and allowances

112,300

   

Sales discounts

60,000

172,300

 

Gross sales

   

$7,299,800

Cost of merchandise sold

   

4,175,100

Income from operations

   

$3,124,700

Expenses:

     

Selling expenses

 

710,000

 

Administrative expenses

 

$525,000

 

Delivery expense

 

$18,100

 

Total expenses

   

1,253,100

Other expense:

   

$1,871,600

Interest revenue

     

Gross profit

   

80,000

     

$1,791,600

determine the amounts of the missing items identifying them by letter 665776

Two items are omitted in each of the following four lists of income statement data. Determine the amounts of the missing items, identifying them by letter.

Sales

$400,000

$500,000

$1,000,000

$ (g)

Sales returns and allowances

(a)

15,000

(e)

30,500

Sales discounts

20,000

8,000

40,000

37,000

Net sales

350,000

(c)

910,000

(h)

Cost of merchandise sold

(b)

285,000

(f)

540,000

Gross profit

200,000

(d)

286,500

150,000

compare the major advantages and disadvantages of the multiple step and single step 665777

On August 31, 2008, the balances of the accounts appearing in the ledger of The Bent Needle Company, a furniture wholesaler, are as follows:

Administrative Expenses

$125,000

Notes Payable

$ 25,000

Building

512,500

Office Supplies

10,600

Cash

48,500

Salaries Payable

3,220

Cost of Merchandise Sold

700,000

Sales

1,275,000

Interest Expense

7,500

Sales Discounts

20,000

Jason Ritchie, Capital

568,580

Sales Returns and Allowances

80,000

Jason Ritchie, Drawing

25,000

Selling Expenses

205,000

Merchandise Inventory

130,000

Store Supplies

7,700

a. Prepare a multiple step income statement for the year ended August 31, 2008.

b. Compare the major advantages and disadvantages of the multiple step and single step forms of income statements.

construct a chart of accounts assigning account numbers and arranging the accounts i 665778

Gemini Co. is a newly organized business with a list of accounts arranged in alphabetical order below.

Accounts Payable

Miscellaneous Administrative Expense

Accounts Receivable

Miscellaneous Selling Expense

Accumulated Depreciation—Office Equipment

Notes Payable

Accumulated Depreciation—Store Equipment

Office Equipment

Advertising Expense

Office Salaries Expense

Cash

Office Supplies

Cost of Merchandise Sold

Office Supplies Expense

Delivery Expense

Prepaid Insurance

Depreciation Expense—Office Equipment

Rent Expense

Depreciation Expense—Store Equipment

Salaries Payable

Income Summary

Sales

Insurance Expense

Sales Discounts

Interest Expense

Sales Returns and Allowances

Jung Qiang, Capital

Sales Salaries Expense

Jung Qiang, Drawing

Store Equipment

Land

Store Supplies

Merchandise Inventory

Store Supplies Expense

Construct a chart of accounts, assigning account numbers and arranging the accounts in balance sheet and income statement order, as illustrated in Exhibit 6. Each account number is three digits: the first digit is to indicate the major classification (“1” for assets, and so on); the second digit is to indicate the sub classification (“11” for current assets, and so on); and the third digit is to identify the specific account (“110” for Cash, and so on).

the debits and credits for three related transactions are presented in the following 665782

The debits and credits for three related transactions are presented in the following T accounts. Describe each transaction.

Cash

(5)

9,310

 

 

Accounts Receivable

(1)

11,750

(3)

2,250

 

 

(5)

9,500

Merchandise Inventory

(4)

1,350

(2)

6,900

Sales

   

(1)

11,750

Sales Discounts

(5)

190

 

 

Sales Returns and Allowances

(3)

2,250

 

 

Cost of Merchandise Sold

(2)

6,900

(4)

1,350

prepare the october 31 2008 closing entries for kavanaugh company 665794

On October 31, 2008, the balances of the accounts appearing in the ledger of Kavanaugh Company, a furniture wholesaler, are as follows:

Accumulated Dep.—Building

$152,300

Notes Payable

$ 120,000

Administrative Expenses

326,500

Salaries Payable

3,400

Building

278,400

Sales

1,567,700

Cash

44,200

Sales Discounts

90,000

Cost of Merchandise Sold

940,000

Sales Returns and Allow.

60,000

Interest Expense

9,600

Sales Tax Payable

24,500

Lillian Kavanaugh, Capital

705,775

Selling Expenses

620,000

Lillian Kavanaugh, Drawing

39,750

Store Supplies

22,900

Merchandise Inventory

130,000

Store Supplies Exp.

12,325

Prepare the October 31, 2008, closing entries for Kavanaugh Company.

what conclusions can be drawn from these ratios concerning the trend in the ability 665795

The Home Depot reported the following data (in millions) in its financial statements for 2005 and 2004:

 

2005

2004

Net sales

$73,094

$64,816

Total assets at the end of the year

38,907

34,437

Total assets at the beginning of the year

34,437

30,011

a. Determine the ratio of net sales to average total assets for The Home Depot for 2005 and 2004. Round to two decimal places.

b. What conclusions can be drawn from these ratios concerning the trend in the ability of The Home Depot to effectively use its assets to generate sales?

compute the ratio of net sales to assets for 2005 round to two decimal places 665796

Kroger, a national supermarket chain, reported the following data (in millions) in its financial statements for 2005:

Total revenue

$56,434

Total assets at end of year

20,491

Total assets at beginning of year

20,763

a. Compute the ratio of net sales to assets for 2005. Round to two decimal places.

b. Would you expect the ratio of net sales to assets for Kroger to be similar to or different from that of Tiffany & Co.? Tiffany is the large North American retailer of jewelry, with a ratio of net sales to average total assets of 0.87.

patel rug company had the following credit sales transactions during march 2008 665797

Patel Rug Company had the following credit sales transactions during March 2008:

Date

Customer

Quantity

Rug Style

Sales

Mar.

3

Samantha McGill

1

10 by 8 Chinese

$14,750

 

8

L. Smith

1

8 by 12 Persian

10,000

 

19

Paula Larkin

1

8 by 10 Indian

11,500

 

26

Amy Pugh

1

10 by 12 Persian

21,000

The March 1 inventory was $26,000, consisting of:

Quantity

Style

Cost per Rug

Total Cost

2

10 by 8 Chinese

$7,500

$15,000

2

8 by 12 Persian

$5,500

11,000

During March, Patel Rug Company purchased the following rugs from Lee Rug Importers:

Date

Quantity

Rug Style

Cost per Rug

Amount

Mar. 10

2

8 by 10 Indian

$ 6,000

$12,000

12

1

10 by 8 Chinese

10,500

10,500

21

3

10 by 12 Persian

16,500

49,500

The general ledger includes the following accounts:

Account

 

Number

Account

11

Accounts Receivable

12

Merchandise Inventory

21

Accounts Payable

41

Sales

51

Cost of Merchandise Sold

a. Record the sales in a two column sales journal. Use the sales journal form shown in 1 at the end of this chapter. Begin with Invoice No. 80.

b. Record the purchases in a purchases journal. Use the purchases journal form shown in Appendix 1 at the end of this chapter.

c. Assume that you have posted the journal entries to the appropriate ledgers. Insert the correct posting references in the sales and purchases journals.

d. Determine the March 31 balance of Merchandise Inventory.

indicate which of the following accounts would be included in the chart of accounts 665798

Indicate which of the following accounts would be included in the chart of accounts of a merchandising company using either the (a) periodic inventory system or (b) perpetual inventory system. If the account would be included in the chart of accounts of a company using the periodic and perpetual systems, indicate (c) for both

(1)

Cost of Merchandise Sold

(6)

Purchases Returns and Allowances

(2)

Purchases Discounts

(7)

Delivery Expense

(3)

Sales

(8)

Sales Returns and Allowances

(4)

Merchandise Inventory

(9)

Transportation In

(5)

Sales Discounts

(10)

Purchases

journalize the entries to record the transactions of lorimer company 665800

The following selected transactions were completed by Lorimer Company during August of the current year. Lorimer Company uses the periodic inventory system.

Aug. 3. Purchased $24,500 of merchandise on account, FOB shipping point, terms 2/10, n/30.

Aug.4. Paid transportation costs of $475 on the August 3 purchase.

Aug.7. Returned $4,000 of the merchandise purchased on August 3.

Aug.11. Sold merchandise on account, $12,700, FOB destination, 2/15, n/30. The cost of merchandise sold was $7,600.

Aug.12. Paid transportation costs of $300 for the merchandise sold on August 11.

Aug.13. Paid for the purchase of August 3 less the return and discount.

Aug.26. Received payment on account for the sale of August 11 less the discount.

Journalize the entries to record the transactions of Lorimer Company.

a well known joke often attributed to paul samuelson is that the market has forecast 665698

Forecasting the Aggregate Stock Market

The early 2002 forecast for 2002 earnings per share for the S&P 500 portfolio was about $53.16 The 10 year Treasury bond yield was about 5.2%. Because the earnings yield on the S&P 500 has been about 1 percentage point below the 10 year Treasury yield, a first guess for the earnings yield on the S&P 500 might be 4.2%. This would imply a P/E ratio of 1/.042 23.8. Our forecast for the S&P 500 index would then be 23.8 + 53 1,261.

Of course, there is uncertainty regarding all three inputs into this analysis: the actual earnings on the S&P 500 stocks, the level of Treasury yields at year end, and the spread between the Treasury yield and the earnings yield. One would wish to perform sensitivity or scenario analysis to examine the impact of changes in all of these variables. To illustrate, consider Table 18.4, which shows a simple scenario analysis treating possible effects of variation in the Treasury bond yield. The scenario analysis shows that forecasted level of the stock market varies inversely and with dramatic sensitivity to interest rate changes.

Some analysts use an aggregate version of the dividend discount model rather than an earnings multiplier approach. All of these models, however, rely heavily on forecasts of such macroeconomic variables as GDP, interest rates, and the rate of inflation, which are difficult to predict accurately.

Because stock prices reflect expectations of future dividends, which are tied to the economic fortunes of firms, it is not surprising that the performance of a broad based stock index like the S&P 500 is taken as a leading economic indicator, that is, a predictor of the performance of the aggregate economy. Stock prices are viewed as embodying consensus forecasts of economic activity and are assumed to move up or down in anticipation of movements in the economy. The government’s index of leading economic indicators, which is taken to predict the progress of the business cycle, is made up in part of recent stock market performance. However, the predictive value of the market is far from perfect.

A well known joke, often attributed to Paul Samuelson, is that the market has forecast eight of the last five recessions.

suppose your research convinces you analog will announce momentarily that it will im 665703

The market consensus is that Analog Electronic Corporation has an ROE = 9%, has a beta of 1.25, and plans to maintain indefinitely its traditional plowback ratio of 2/3. This year’s earnings were $3 per share. The annual dividend was just paid. The consensus estimate of the coming year’s market return is 14%, and T bills currently offer a 6% return.

a. Find the price at which Analog stock should sell.

b. Calculate the P/E ratio.

c. Calculate the present value of growth opportunities.

d. Suppose your research convinces you Analog will announce momentarily that it will immediately reduce its plowback ratio to 1/3. Find the intrinsic value of the stock. The market is still unaware of this decision. Explain why V0 no longer equals P0 and why V0 is greater or less than P0.

what are the expected dividend payout ratios for the two stocks 665707

Your preliminary analysis of two stocks has yielded the information set forth below. The market capitalization rate for both Stock A and Stock B is 10% per year.

 

Stock A

Stock B

Expected return on equity, ROE

14%

12%

Estimated earnings per share, E1

$ 2.00

$ 1.65

Estimated dividends per share, D1

$ 1.00

$ 1.00

Current market price per share, P0

$27.00

$25.00

a. What are the expected dividend payout ratios for the two stocks?

b. What are the expected dividend growth rates of each?

c. What is the intrinsic value of each stock?

d. In which, if either, of the two stocks would you choose to invest?

determine whether each of these fundamental factors would cause p e ratios to be gen 665708

Peninsular Research is initiating coverage of a mature manufacturing industry. John Jones, CFA, head of the research department, gathered the following fundamental industry and market data to help in his analysis:

Forecast industry earnings retention rate

40%

Forecast industry return on equity

25%

Industry beta

1.2

Government bond yield

6%

Equity risk premium

5%

a. Compute the price to earnings (P0 /E1) ratio for the industry based on this fundamental data.

b. Jones wants to analyze how fundamental P/E ratios might differ among countries.

He gathered the following economic and market data:

Fundamental Factors

Country A

Country B

Forecast growth in real GDP

5%

2%

Government bond yield

10%

6%

Equity risk premium

5%

4%

Determine whether each of these fundamental factors would cause P/E ratios to be generally higher for Country A or higher for Country B.

explain why the outcome suggested in part a above should not be possible in a market 665709

“Growth” and “value” can be defined in several ways, but “growth” usually conveys the idea of a portfolio emphasizing or including only issues believed to possess above average future rates of per share earnings growth. Low current yield, high price to book ratios, and high price to earnings ratios are typical characteristics of such portfolios.

“Value” usually conveys the idea of portfolios emphasizing or including only issues currently showing low price to book ratios, low price to earnings ratios, above average levels of dividend yield, and market prices believed to be below the issues’ intrinsic values.

a. Identify and explain three reasons why, over an extended period of time, value stock investing might outperform growth stock investing.

b. Explain why the outcome suggested in part (a) above should not be possible in a market widely regarded as being highly efficient.

assuming the current market price of the stock reflects its intrinsic value as compu 665710

The stock of Nogro Corporation is currently selling for $10 per share. Earnings per share in the coming year are expected to be $2. The company has a policy of paying out 50% of its earnings each year in dividends. The rest is retained and invested in projects that earn a 20% rate of return per year. This situation is expected to continue indefinitely.

a. Assuming the current market price of the stock reflects its intrinsic value as computed using the constant growth DDM, what rate of return do Nogro’s investors require?

b. By how much does its value exceed what it would be if all earnings were paid as dividends and nothing were reinvested?

c. If Nogro were to cut its dividend payout ratio to 25%, what would happen to its stock price? What if Nogro eliminated the dividend?

chip tech rsquo s competitive status until the competitor firm publicly announces it 665711

Chip tech, Inc., is an established computer chip firm with several profitable existing products as well as some promising new products in development. The company earned $1 a share last year, and just paid out a dividend of $.50 per share. Investors believe the company plans to maintain its dividend payout ratio at 50%. ROE equals 20%. Everyone in the market expects this situation to persist indefinitely.

a. What is the market price of Chip tech stock? The required return for the computer chip industry is 15%, and the company has just gone ex dividend.

b. Suppose you discover that Chip tech’s competitor has developed a new chip that will eliminate Chip tech’s current technological advantage in this market. This new product, which will be ready to come to the market in 2 years, will force Chip tech to reduce the prices of its chips to remain competitive. This will decrease ROE to 15%, and, because of falling demand for its product, Chip tech will decrease the plowback ratio to .40. The plowback ratio will be decreased at the end of the second year, at t = 2: The annual year end dividend for the second year (paid at t _ 2) will be 60% of that year’s earnings. What is your estimate of Chip tech’s intrinsic value per share?

c. No one else in the market perceives the threat to Chip tech’s market. In fact, you are confident that no one else will become aware of the change in Chip tech’s competitive status until the competitor firm publicly announces its discovery near the end of year 2. What will be the rate of return on Chip tech stock in the coming year? In the second year ? The third year ?

describe one strength of the two stage ddm in comparison with the constant growth dd 665713

Janet Ludlow’s firm requires all its analysts to use a two stage dividend discount model (DDM) and the Capital Asset Pricing Model (CAPM) to value stocks. Using the CAPM and DDM, Ludlow has valued Quick Brush Company at $63 per share.

She now must value Smile White Corporation.

a. Calculate the required rate of return for Smile White by using the information in the following table:

 

Quick Brush

Smile White

Beta

1.35

1.15

Market price

$45.00

$30.00

Intrinsic value

$63.00

?

Notes:

   

Risk free rate

4.50%

 

Expected market return

14.50%

 

b. Ludlow estimates the following EPS and dividend growth rates for Smile White:

First 3 years

12% per year

Years thereafter

9% per year

Estimate the intrinsic value of Smile White by using the table above, and the two stages DDM. Dividends per share in the most recent year were $1.72.

c. Recommend Quick Brush or Smile White stock for purchase by comparing each company’s intrinsic value with its current market price.

d. Describe one strength of the two stage DDM in comparison with the constant growth DDM. Describe one weakness inherent in all DDMs.

assuming its current market price is equal to its intrinsic value what do you expect 665714

The Digital Electronic Quotation System (DEQS) Corporation pays no cash dividends currently and is not expected to for the next 5 years. Its latest EPS was $10, all of which was reinvested in the company. The firm’s expected ROE for the next 5 years is 20% per year, and during this time it is expected to continue to reinvest all of its earnings. Starting 6 years from now the firm’s ROE on new investments is expected to fall to 15%, and the company is expected to start paying out 40% of its earnings in cash dividends, which it will continue to do forever after. DEQS’s market capitalization rate is 15% per year.

a. What is your estimate of DEQS’s intrinsic value per share?

b. Assuming its current market price is equal to its intrinsic value, what do you expect to happen to its price over the next year? The year after?

c. What effect would it have on your estimate of DEQS’s intrinsic value if you expected DEQS to pay out only 20% of earnings starting in year 6?

using the data in the accompanying table calculate philip morris rsquo s price ndash 665715

At year end 1991, the Wall Street consensus was that Philip Morris’s earnings and dividends would grow at 20% for 5 years, after which growth would fall to a market like 7%. Analysts also projected a required rate of return of 10% for the U.S. equity market.

a. Using the data in the accompanying table and the multistage dividend discount model, calculate the intrinsic value of Philip Morris stock at year end 1991. Assume a similar level of risk for Philip Morris stock as for the typical U.S. stock.

b. Using the data in the accompanying table, calculate Philip Morris’s price–earnings ratio and the price–earnings ratio relative to the S&P 500 Stock Index as of December 31, 1991.

c. Using the data in the accompanying table, calculate Philip Morris’s price–book ratio (i.e., ratio of market value to book value) and the price–book ratio relative to the S&P 500 Stock Index as of December 31, 1991.

Philip Morris Corporation Selected Financial Data Years Ending December 31 ($ millions except per share data)

 

1991

1981

Earnings per share

$4.24

$0.66

Dividends per share

$1.91

$0.25

Stockholders’ equity

12,512

3,234

Total liabilities and stockholders’ equity

$47,384

$9,180

Other data

   

Philip Morris

   

Common shares outstanding (millions)

920

1,003

Closing price common stock

$80.250

$6.125

S&P 500 Stock Index:

   

Closing price

417.09

122.55

Earnings per share

16.29

15.36

Book value per share

161.08

109.43

when is the dollar denominated return equal to the pound denominated return 665734

Suppose a U.S. investor wishes to invest in a British firm currently selling for £40 per share. The investor has $10,000 to invest, and the current exchange rate is $2/£.

a. How many shares can the investor purchase?

b. Fill in the table below for rates of return after 1 year in each of the nine scenarios (three possible prices per share in pounds times three possible exchange rates).

Price per

Share

(£)

Pound

Denominated

Return (%)

Dollar Denominated Return

For Year End Exchange Rate

$1.80/£

$2/£

$2.20/£

£35

       

£40

       

£45

       

c. When is the dollar denominated return equal to the pound denominated return?

calculate the contribution to total performance from currency country and stock sele 665737

Calculate the contribution to total performance from currency, country, and stock selection for the manager in the example below. All exchange rates are expressed as units of foreign currency that can be purchased with one U.S. dollar.

 

EAFE

Weight

Return on

 

Manager’s

Weight

Manager’s

Return

Equity Index

E1 /E0

Europe

0.30

20%

0.9

0.35

18%

Australia

0.10

15

1.0

0.15

20

Far East

0.60

25

1.1

0.50

20

             

list three possible arguments against international equity investment and briefly di 665739

John Irish, CFA, is an independent investment adviser who is assisting Alfred Darwin, the head of the Investment Committee of General Technology Corporation, to establish a new pension fund. Darwin asks Irish about international equities and whether the Investment Committee should consider them as an additional asset for the pension fund.

a. Explain the rationale for including international equities in General’s equity portfolio.

Identify and describe three relevant considerations in formulating your answer.

b. List three possible arguments against international equity investment and briefly discuss the significance of each.

c. To illustrate several aspects of the performance of international securities over time, Irish shows Darwin the accompanying graph of investment results experienced by a U.S. pension fund in the recent past. Compare the performance of the U.S. dollar and non U.S. dollar equity and fixed income asset categories, and explain the significance of the result of the account performance index relative to the results of the four individual asset class indexes.

calculate the expected price change required in the canadian government bond which w 665740

You are a U.S. investor considering purchase of one of the following securities. Assume that the currency risk of the Canadian government bond will be hedged, and the 6 month discount on Canadian dollar forward contracts is .75% versus the U.S. dollar.

Bond

Maturity

Coupon

Price

U.S. government

6 months

6.50%

100.00

Canadian government

6 months

7.50%

100.00

Calculate the expected price change required in the Canadian government bond which would result in the two bonds having equal total returns in U.S. dollars over a 6 month horizon. Assume that the yield on the U.S. bond is expected to remain unchanged.

aglobal manager plans to invest 1 million in u s government cash equivalents for the 665741

Aglobal manager plans to invest $1 million in U.S. government cash equivalents for the next 90 days. However, she is also authorized to use non U.S. government cash equivalents, as long as the currency risk is hedged to U.S. dollars using forward currency contracts.

a. What rate of return will the manager earn if she invests in money market instruments in either Canada or Japan and hedges the dollar value of her investment? Use the data in the following tables.

b. What must be the approximate value of the 90 day interest rate available on U.S. government securities?

Interest Rates (APR)
90 Day Cash Equivalents

Japanese government

2.52%

 

Canadian government

6.74%

 

Exchange Rates
Dollars per Unit of Foreign Currency

 

Spot

90 Day Forward

Japanese yen

.0119

.0120

Canadian dollar

.7284

.7269

 

 

briefly discuss aspects of the otunia environment that favor investing actively and 665743

After much research on the developing economy and capital markets of the country of Otunia, your firm, GAC, has decided to include an investment in the Otunia stock market in its Emerging Markets Commingled Fund. However, GAC has not yet decided whether to invest actively or by indexing. Your opinion on the active versus indexing decision has been solicited. The following is a summary of the research findings:

Otunia’s economy is fairly well diversified across agricultural and natural resources, manufacturing (both consumer and durable goods), and a growing finance sector. Transaction costs in securities markets are relatively large in Otunia because of high commissions and government “stamp taxes” on securities trades. Accounting standards and disclosure regulations are quite detailed, resulting in wide public availability of reliable information about companies’ financial performance.

Capital flows into and out of Otunia, and foreign ownership of Otunia securities is strictly regulated by an agency of the national government. The settlement procedures under these ownership rules often cause long delays in settling trades made by nonresidents. Senior finance officials in the government are working to deregulate capital flows and foreign ownership, but GAC’s political consultant believes that isolationist sentiment may prevent much real progress in the short run.

a. Briefly discuss aspects of the Otunia environment that favor investing actively, and aspects that favor indexing.

b. Recommend whether GAC should invest in Otunia actively or by indexing. Justify your recommendation based on the factors identified in part (a).

 

let rsquo s compute the rate of return on each bond the 1 year bond is bought today 665744

Holding Period Return on Zero Coupon Bonds

Let’s compute the rate of return on each bond. The 1 year bond is bought today for $925.93 and matures in 1 year to its par value of $1,000. Because the bond pays no coupon, total income is $1,000 $925.93 = $74.07, and the rate of return is $74.07/$925.93 .08, or 8%. The 2 year bond is bought today for $841.75. Next year the interest rate will be 10%, and the bond will have 1 year left until maturity. It will sell then for $1,000/1.10 = $909.09. Thus the holding period return is ($909.09 $841.75)/$841.75 = .08, again implying an 8% rate of return. Similarly, the 3 year bond will be purchased for $758.33 and will be sold at year end for 1,000/(1.10)(1.11) = $819.00, for a rate of return ($819.00 $758.33)/$758.33 = .08, again, an 8% return. We conclude from Example 15.1 that when interest rate movements are known with certainty, and all bonds are properly priced, all will provide equal 1 year rates of return. The higher yields on the longer term bonds merely reflect the fact that future interest rates are higher than current rates and that the longer bonds are still alive during the higher rate period. Owners of the short term bonds receive lower yields to maturity, but they can reinvest, or “roll over,” their proceeds for higher yields in later years when rates are higher. In the end, both long term bonds and short term rollover strategies provide equal returns over the holding period, at least in a world of interest rate certainty.

 

suppose that we observe an 8 coupon bond making semiannual payments with 1 year unti 665746

Yields and Forward Rates on Coupon Bonds

Suppose that we observe an 8% coupon bond making semiannual payments with 1 year until maturity, selling at $986.10, and a 10% coupon bond, also with a year until maturity, selling at $1,004.78. To infer the short rates for the next two 6 month periods, we first attempt to find the present value of each coupon payment taken individually, that is, treated as a mini–zero coupon bond. Call d1 the present value of $1 to be received in half a year and d2 the present value of a dollar to be received in 1 year. (The d stands for discounted values; therefore, d1 = 1/(1 + r1), where r1 is the short rate for the first 6 month period.) Then our two bonds must satisfy the simultaneous equations

986.10 = d1 X 40 + d2 = 1,040

1,004.78 + d1 X 50 + d2 = 1,050

In each equation the bond’s price is set equal to the discounted value of all of its remaining cash flows. Solving these equations we find that d1 = .95694 and d2 = .91137. Thus if r1 is the short rate for the first 6 month period, then d1 = 1/(1 + r1) = .95694, so that r1 = .045, and d2 = 1/[(1 + r1)(1 + f2)] = 1/[(1.045)(1 + f2)] = .91137, so that f2 = .05. Thus the two short rates are shown to be 4.5% for the first half year period and 5% for the second.

 

the tables below show respectively the characteristics of two annual pay bonds from 665747

The tables below show, respectively, the characteristics of two annual pay bonds from the same issuer with the same priority in the event of default, and spot interest rates. Neither bond’s price is consistent with the spot rates. Using the information in these tables, recommend either bond A or bond B for purchase. Justify your choice.

Bond Characteristics

 

 

Bond A

Bond B

 

Coupons

Annual

Annual

 

Maturity

3 years

3 years

 

Coupon rate

10%

6%

 

Yield to maturity

10.65%

10.75%

 

Price

98.40

88.34

 

Spot Interest Rates

Term (Years)

Spot Rates (Zero Coupon)

1

5%

2

8

3

11

         

 

construct price and value weighted indexes and calculate the percentage changes in e 665636

Caution: Indexes Under Construction

Suppose there are only two stocks in the market and the following information is given:

 

Price per Share

 

Shares Outstanding

Beginning of Year

End of Year

Betty Co.

10 million

$10

$11

Gray Bull, Inc.

20 million

$20

$25

Construct price and value weighted indexes and calculate the percentage changes in each. The average share price rose from $15 to $18, or $3, so the price weighted index would be up by 3/15 = 20 percent. Average total market value, in millions, rose from $250 to $305, so the value weighted index rose by 55/250 = 22 percent.

prepare journal entries to record the purchase and use of the raw material during 665654

The following standards have been established for a raw material used in the production of product G13:

Standard quantity of the material per unit of output

2.3 liters

Standard price of the material

$19.00 per liter

The following data pertain to a recent month’s operations:

Actual material purchased

5,100 liters

Actual cost of material purchased

$100,725

Actual material used in production

4,700 liters

Actual output

2,040 units of product G13

Required:

a. What is the materials price variance for the month?

b. What is the materials quantity variance for the month?

c. Prepare journal entries to record the purchase and use of the raw material during

the month. (All raw materials are purchased on account.)

the following materials standards have been established for a particular product 665656

The following materials standards have been established for a particular product:

Standard quantity per unit of output

0.2 grams

Standard price

$18.90 per gram

The following data pertain to operations concerning the product for the last month:

Actual materials purchased

4,800 grams

Actual cost of materials purchased

$86,880

Actual materials used in production

4,200 grams

Actual output

21,080 units

Required:

a. What is the materials price variance for the month?

b. What is the materials quantity variance for the month?

what is the materials price variance for the month 665657

The following standards have been established for a raw material used to make product I92:

Standard quantity of the material per unit of output

4.5 pounds

Standard price of the material

$13.90 per pound

The following data pertain to a recent month’s operations:

Actual material purchased

2,000 pounds

Actual cost of material purchased

$26,200

Actual material used in production

1,300 pounds

Actual output

220 units of product I92

Required:

a. What is the materials price variance for the month?

b. What is the materials quantity variance for the month?

what was the labor efficiency variance for the month 665659

The following direct labor standards have been established for product N30A:

Standard direct labor hours

3.3 hours per unit of N30A

Standard direct labor wage rate

$10.50 per hour

The following data pertain to the most recent month’s operations during which 400 units of product N30A were made:

Actual direct labor hours worked

1,100

Actual direct labor wages paid

$11,385

Required:

a. What was the labor rate variance for the month?

b. What was the labor efficiency variance for the month?

c. Prepare a journal entry to record direct labor costs during the month, including the direct labor variances.

the following labor standards have been established for a particular product 665661

The following labor standards have been established for a particular product:

Standard labor hours per unit of output

3.2 hours

Standard labor rate

$19.10 per hour

The following data pertain to operations concerning the product for the last month:

Actual hours worked

5,500 hours

Actual total labor cost

$105,050

Actual output

1,900 units

Required:

a. What is the labor rate variance for the month?

b. What is the labor efficiency variance for the month?

what was the labor rate variance for the month 665662

The following direct labor standards have been established for product S57S:

Standard direct labor hours

1.5 hours per unit of S57S

Standard direct labor wage rate

$14.70 per hour

The following data pertain to last month’s operations:

Actual output of product S57S

720 units

Actual direct labor hours worked

1,000

Actual direct labor wages paid

$14,800

Required:

a. What was the labor rate variance for the month?

b. What was the labor efficiency variance for the month?

what was the variable overhead spending variance for the month 665665

Deschamp Corporation”s variable manufacturing overhead is applied on the basis of direct labor hours. The company has established the following variable manufacturing overhead standards for product O28H:

Standard direct labor hours

2.5 hours per unit of O28H

Standard variable manufacturing overhead rate

$7.70 per hour

The following data pertain to the most recent month’s operations during which 2,160 units of product O28H were made:

Actual direct labor hours worked

5,200

Actual variable manufacturing overhead incurred

$44,980

Required:

a. What was the variable overhead spending variance for the month?

b. What was the variable overhead efficiency variance for the month?

compute the throughput time 665667

Pardun Corporation”s management keeps track of the time it takes to process orders. During the most recent month, the following average times were recorded per order

Days

Wait time

15.6

Inspection time

0.8

Process time

1.6

Move time

0.7

Queue time

3.9

Required:

a. Compute the throughput time.

b. Compute the manufacturing cycle efficiency (MCE).

c. What percentage of the production time is spent in non value added activities?

d. Compute the delivery cycle time.

lindon company s flexible budget for variable manufacturing overhead is given below 665670

Lindon Company”s flexible budget for variable manufacturing overhead is given below:

Cost

Formula

6,000

8,000

10,000

Overhead costs

per DLH

DLHs

DLHs

DLHs

Supplies

$0.20

$1,200

$1,600

$2,000

Indirect labor

0.50

3,000

4,000

5,000

Utilities

0.05

300

400

500

Total overhead cost

$0.75

$4,500

$6,000

$7,500

During a recent period, the company produced 2,500 units of product using 7,600 direct labor hours (DLHs). The standard allows 3 direct labor hours per unit. Actual variable overhead costs incurred were:

Supplies

$1,900

Indirect labor

3,040

Utilities

570

Total overhead cost

$5,510

The company had originally budgeted to produce 2,600 units during the period using 7,800 direct labor hours.

Required:

Prepare a performance report for the period showing only the spending variances for each overhead cost category.

120. The following overhead data are for a department in a large company.

Actual Costs

Static

Incurred

Budget

Activity level (in units)

400

380

Variable costs:

Indirect materials

$9,050

$8,132

Power

$2,540

$2,394

Fixed costs:

Administration

$5,080

$5,100

Rent

$8,590

$8,600

Required:

Prepare a report that would be useful in assessing how well costs were controlled in this department.

prepare a report that would be useful in assessing how well costs were controlled in 665671

The following overhead data are for a department in a large company.

Actual Costs

Static

Incurred

Budget

Activity level (in units)

200

220

Variable costs:

Supplies

$4,050

$4,906

Power

$1,690

$1,892

Fixed costs:

Administration

$6,240

$6,200

Depreciation

$6,280

$6,200

Required:

Prepare a report that would be useful in assessing how well costs were controlled in this department.

compute all four manufacturing overhead variances for layt 665672

Layt Clock Company has developed the following flexible budget for its overhead costs. Manufacturing overhead at Layt is applied to production on the basis of standard machine hours:

Machine Hours

21,600

24,000

26,400

Clocks produced

18,000

20,000

22,000

Variable overhead cost

$127,440

$141,600

$155,760

Fixed overhead cost

$171,072

$171,072

$171,072

Layt was expecting to produce 22,000 clocks last year. The actual results for the year were as follows:

Number of clocks produced

21,500

Machine hours incurred

24,940

Variable overhead cost

$145,899

Fixed overhead cost

$170,540

Required:

Compute all four manufacturing overhead variances for Layt.

what was the fixed overhead budget variance 665673

Parker Company uses a standard cost system in which manufacturing overhead is applied to units of product on the basis of standard direct labor hours. The company”s total budgeted variable and fixed manufacturing overhead costs at the denominator level of activity are $14,000 for variable overhead and $6,000 for fixed overhead. The predetermined overhead rate, including both fixed and variable components, is $4 per direct labor hour. The standards call for 2 direct labor hours per unit of output produced. Last year, the company produced 3,000 units of product and worked 6,200 direct labor hours. Actual costs were $15,500 for variable overhead and $6,300 for fixed overhead.

Required:

a. What is the denominator level of activity?

b. What were the standard hours allowed for the output last year?

c. What was the variable overhead spending variance?

d. What was the variable overhead efficiency variance?

e. What was the fixed overhead budget variance?

f. What was the fixed overhead volume variance?

compute the following variances for cajun 665674

Cajun Candy Corporation manufactures giant gourmet suckers. The cost standards developed by Cajun appear below. Manufacturing overhead at Cajun is applied to production on the basis of standard direct labor hours:

Standard

Standard cost

Standard

quantity

per ounce or

cost per

per sucker

hour

sucker

Direct materials

0.75 ounces

$20.00

$15.00

Direct labor

1.2 hours

$12.00

14.40

Variable overhead

1.2 hours

$3.00

3.60

Fixed overhead

1.2 hours

$5.00

6.00

Total standard cost per sucker

$39.00

The standards above were based on an expected annual volume of 8,000 suckers. The actual results for last year were as follows:

Number of suckers produced

8,200

Direct labor hours incurred

10,000

Ounces of direct materials purchased

7,900

Ounces of direct materials used in production

6,070

Total cost of direct materials purchased

$156,815

Total direct labor cost

$122,800

Total variable overhead cost

$28,600

Total fixed overhead cost.

$47,500

Required:

Compute the following variances for Cajun.

a. Materials price variance.

b. Materials quantity variance.

c. Labor rate variance.

d. Variable overhead spending variance.

e. Variable overhead efficiency variance.

f. Fixed overhead budget variance.

compute the variable manufacturing overhead spending and efficiency variances 665675

Eastern Company uses a standard cost system in which manufacturing overhead is applied to units of product on the basis of standard direct labor hours (DLHs). The denominator activity level is 60,000 direct labor hours, or 300,000 units.

• A standard cost card for the company’s product follows:

Standard

quantity or

Standard price or

Standard

hours

rate

cost

Direct materials

0.25 kilogram

$16 per kilogram

$4

Direct labor

0.20 DLH

$10 per DLH

2

Variable overhead

0.20 DLH

$5 per DLH

1

Fixed overhead

0.20 DLH

$10 per DLH

2

Total standard cost

$9

• Actual data for the year follow:

Units produced and sold

330,000

Actual direct labor hours worked

64,800

Actual variable manufacturing overhead cost

$327,240

Actual fixed manufacturing overhead cost

$612,000

Required:

a. Compute the variable manufacturing overhead spending and efficiency variances.

b. Compute the fixed manufacturing overhead budget and volume variances.

compute the total fixed overhead cost that was originally budgeted 665676

Pierce Company uses a standard cost system in which it applies manufacturing overhead to its product on the basis of standard direct labor hours (DLHs). Below is the standard cost card for the product:

Direct materials, 4.5 feet at $3.80 per foot

$17.10

Direct labor, 3.0 DLHs at $9.50 per DLH

28.50

Variable overhead, 3.0 DLHs at $2.00 per DLH

6.00

Fixed overhead, 3.0 DLHs at $8.00 per DLH

24.00

$75.60

Last year, the company produced 6,000 units of product using 17,000 direct labor hours. The actual total fixed overhead cost for the year was $140,000 and the volume variance was $12,000, favorable.

Required:

a. Compute the total fixed overhead cost that was originally budgeted.

b. Compute the denominator activity figure that the company used in computing predetermined overhead rates.

consider two firms operating in the same industry with identical revenues in all pha 665679

Operating Leverage

Consider two firms operating in the same industry with identical revenues in all phases of the business cycle: recession, normal, and expansion. Firm A has short term leases on most of its equipment and can reduce its lease expenditures when production slackens. It has fixed costs of $5 million and variable costs of $1 per unit of output. Firm B has long term leases on most of its equipment and must make lease payments regardless of economic conditions. Its fixed costs are higher, $8 million, but its variable costs are only $.50 per unit. Table 17.4 shows that Firm A will do better in recessions than Firm B, but not as well in expansions. A’s costs move in conjunction with its revenues to help performance in downturns and impede performance in upturns.

here are four industries and four forecasts for the macro economy match the industry 665684

Here are four industries and four forecasts for the macro economy. Match the industry to the scenario in which it is likely to be the best performer.

Industry

Economic Forecast

a. Housing construction

(i) Deep recession: falling inflation, interest rates, and GDP

b. Health care

(ii) Superheated economy: rapidly rising GDP, increasing inflation and interest rates

c. Gold mining

(iii) Healthy expansion: rising GDP, mild inflation, low unemployment

d. Steel production

(iv) Stagflation: falling GDP, high inflation

discuss how product pricing should differ between universal rsquo s two businesses b 665687

Universal Auto is a large multinational corporation headquartered in the United States. For segment reporting purposes, the company is engaged in two businesses: production of motor vehicles and information processing services.

The motor vehicle business is by far the larger of Universal’s two segments.

It consists mainly of domestic U.S. passenger car production, but it also includes small truck manufacturing operations in the United States and passenger car production in other countries. This segment of Universal has had weak operating results for the past several years, including a large loss in 2002. Although the company does not reveal the operating results of its domestic passenger car segments, that part of Universal’s business is generally believed to be primarily responsible for the weak performance of its motor vehicle segment.

I data, the information processing services segment of Universal, was started by Universal about 15 years ago. This business has shown strong, steady growth that has been entirely internal; no acquisitions have been made.

An excerpt from a research report on Universal prepared by Paul Adams, a CFA candidate, states: “Based on our assumption that Universal will be able to increase prices significantly on U.S. passenger cars in 2004, we project a multibillion dollar profit improvement.”

a. Discuss the concept of an industrial life cycle by describing each of its four phases.

b. Identify where each of Universal’s two primary businesses—passenger cars and information processing—is in such a cycle.

c. Discuss how product pricing should differ between Universal’s two businesses, based on the location of each in the industrial life cycle.

based on historical data and assuming less than full employment periods of sharp acc 665690

The following problems appeared on past CFA examinations.

a. The supply side view stresses that:

i. Aggregate demand is the major determinant of real output and aggregate employment.

ii. An increase in government expenditures and tax rates will cause real income to rise.

iii. Tax rates are a major determinant of real output and aggregate employment.

iv. Expansionary monetary policy will cause real output to expand without causing the rate of inflation to accelerate.

b. In macroeconomics, the crowding out effect refers to:

i. The impact of government deficit spending on inflation.

ii. Increasing population pressures and associated movements toward zero population growth.

iii. A situation where the unemployment rate is below its natural rate. iv. The impact of government borrowing on interest rates and private investment.

c. Based on historical data and assuming less than full employment, periods of sharp acceleration in the growth rate of the money supply tend to be associated initially with:

i. Periods of economic recession.

ii. An increase in the velocity of money.

iii. A rapid growth of gross domestic product.

iv. Reductions in real gross domestic product.

d. If the exchange rate value of the British pound goes from U.S.$1.80 to

U.S.$1.60, then the pound has:

i. Appreciated and the British will find U.S. goods cheaper.

ii. Appreciated and the British will find U.S. goods more expensive.

iii. Depreciated and the British will find U.S. goods more expensive.

iv. Depreciated and the British will find U.S. goods cheaper.

e. The consumer price index is:

i. A measure of the increase in the prices of the goods that is included in the calculation of GDP.

ii. The ratio of the average price of a typical market basket of goods compared to the cost of producing those goods during the previous year.

iii. A comparison of the cost of a typical bundle of goods during a given period with the cost of the same bundle during a prior base period.

iv. Computed in the same manner as the GDP deflator.

f. Changes in which of the following are likely to affect interest rates?

I. Inflation expectations.

II. Size of the federal deficit.

III. Money supply.

i. I and II only.

ii. II and III only.

iii. I and III only.

iv. I, II, and III.

g. According to the supply side view of fiscal policy, if the impact on total tax revenues is the same, does it make any difference whether the government cuts taxes by either reducing marginal tax rates or increasing the personal exemption allowance?

i. No, both methods of cutting taxes will exert the same impact on aggregate supply.

ii. No, people in both cases will increase their saving; expecting higher future taxes, and thereby offset the stimulus effect of lower current taxes.

iii. Yes, the lower marginal tax rates alone will increase the incentive to earn marginal income and thereby stimulate aggregate supply.

iv. Yes, interest rates will increase if marginal tax rates are lowered, whereas they will tend to decrease if the personal exemption allowance is raised.

h. If the Federal Reserve wanted to reduce the supply of money as part of an anti inflation policy, it might:

i. Increase the reserve requirements.

ii. Buy U.S. securities on the open market.

iii. Lower the discount rate.

iv. Buy U.S. securities directly from the Treasury.

describe the conditions under which the calculated forward rate would be an unbiased 665598

Suppose the following table shows yields to maturity of zero coupon U.S. Treasury securities as of January 1, 1996:

Term to Maturity (Years)

Yield to Maturity

1

3.50%

2

4.50

3

5.00

4

5.50

5

6.00

10

6.60

a. Based on the data in the table, calculate the implied forward 1 year rate of interest at January 1, 1999.

b. Describe the conditions under which the calculated forward rate would be an unbiased estimate of the 1 year spot rate of interest at January 1, 1999.

c. Assume that 1 year earlier, at January 1, 1995, the prevailing term structure for

U.S. Treasury securities was such that the implied forward 1 year rate of interest at January 1, 1999, was significantly higher than the corresponding rate implied by the term structure at January 1, 1996. On the basis of the pure expectations theory of the term structure, briefly discuss two factors that could account for such a decline in the implied forward rate.

if you purchase a 2 year zero coupon bond now what is the expected total rate of ret 665600

The current yield curve for default free zero coupon bonds is as follows:

Maturity (Years)

YTM

1

10%

2

11

3

12

a. What are the implied 1 year forward rates?

b. Assume that the pure expectations hypothesis of the term structure is correct.

If market expectations are accurate, what will the pure yield curve (that is, the yields to maturity on 1 and 2 year zero coupon bonds) be next year?

c. If you purchase a 2 year zero coupon bond now, what is the expected total rate of return over the next year? What if you purchase a 3 year zero coupon bond?

(Hint: Compute the current and expected future prices.) Ignore taxes.

d. What should be the current price of a 3 year maturity bond with a 12% coupon rate paid annually? If you purchased it at that price, what would your total expected rate of return be over the next year (coupon plus price change)? Ignore taxes.

what do you expect the rate of return to be over the coming year on a 3 year zero co 665601

The term structure for zero coupon bonds is currently:

Maturity (Years)

YTM

1

4%

2

5

3

6

Next year at this time, you expect it to be:

Maturity (Years)

YTM

1

5%

2

6

3

7

a. What do you expect the rate of return to be over the coming year on a 3 year zero coupon bond?

b. Under the expectations theory, what yields to maturity does the market expect to observe on 1 and 2 year zeros next year? Is the market’s expectation of the return on the 3 year bond greater or less than yours?

briefly discuss why zero coupon stripped u s treasuries could yield more than coupon 665603

A portfolio manager at Superior Trust Company is structuring a fixed income portfolio to meet the objectives of a client. This client plans on retiring in 15 years and wants a substantial lump sum at that time. The client has specified the use of AAA rated securities.

The portfolio manager compares coupon U.S. Treasuries with zero coupon stripped U.S. Treasuries and observes a significant yield advantage for the stripped bonds:

Term (Years)

Coupon U.S. Treasuries

Zero Coupon Stripped U.S. Treasuries

3

5.50%

5.80%

5

6.00

6.60

7

6.75

7.25

10

7.25

7.60

15

7.40

7.80

30

7.75

8.20

Briefly discuss why zero coupon stripped U.S. Treasuries could yield more than coupon U.S. Treasuries with the same final maturity.

an 8 5 coupon 1 000 par bond pays an annual coupon and will mature in 3 years what s 665604

Below is a list of prices for zero coupon bonds of various maturities.

Maturity (Years)

Price of $1,000 Par Bond (Zero Coupon)

1

943.40

2

873.52

3

816.37

a. An 8.5% coupon $1,000 par bond pays an annual coupon and will mature in 3 years. What should the yield to maturity on the bond be?

b. If at the end of the first year the yield curve flattens out at 8%, what will be the 1 year holding period return on the coupon bond?

if you forecast that the yield curve in 1 year will be flat at 7 what is your foreca 665605

Prices of zero coupon bonds reveal the following pattern of forward rates:

Year

Forward Rate

1

5%

2

7

3

8

In addition to the zero coupon bond, investors also may purchase a 3 year bond making annual payments of $60 with par value $1,000.

a. What is the price of the coupon bond?

b. What is the yield to maturity of the coupon bond?

c. Under the expectations hypothesis, what is the expected realized compound yield of the coupon bond?

d. If you forecast that the yield curve in 1 year will be flat at 7%, what is your forecast for the expected rate of return on the coupon bond for the 1 year holding period?

if you believe that the term structure next year will be the same as today rsquo s w 665607

You observe the following term structure:

 

Effective Annual YTM

1 year zero coupon bond

6.1%

2 year zero coupon bond

6.2

3 year zero coupon bond

6.3

4 year zero coupon bond

6.4

a. If you believe that the term structure next year will be the same as today’s, will the 1 year or the 4 year zeros provide a greater expected 1 year return?

b. What if you believe in the expectations hypothesis?